Download NEET PG 2015 Question Paper with Answers

Download NEET PG (The National Eligibility cum Entrance Test Postgraduate) 2015 Question Paper with Answers

1.UpperlimbdeformityinErb'spalsy?
a)Adductionandlateralrotationofarm
b)Adductionandmedialrotationofarm
c)Abductionandlateralrotationofarm
d)Abductionandmedialrotationofarm
CorrectAnswer-B
DeformityinErb'spalsy(positionofthelimb):
Arm:Hangesbytheside;itisadducted&mediallyrotated
Forearm:Extendedandpronated
Thedeformityisknownas`policeman'stiphand'or`porter'stip
hand'


2.NerverootsinvolvedinErb'spalsy:
a)C5,C6
b)C6,C7
c)C7,C8,T1
d)C5,C6,C7,C8,T1
CorrectAnswer-A
Erb-DuchennePalsy
Upperlesionsofthebrachialplexusareinjuriesresultingfrom
excessivedisplacementoftheheadtotheoppositesideand
depressionoftheshoulderonthesameside.Thiscausesexcessive
tractionoreventearingofC5andC6rootsoftheplexus.Itoccursin
thenewbornduringadifficultdeliveryorinadultsafterablowtoor
fallontheshoulder.
Thesuprascapularnerve,thenervetothesubclavius,andthe
musculocutaneousandaxillarynervesallpossessnervefibers
derivedfromC5andC6rootsandwillthereforebefunctionless.
Thefollowingmuscleswillconsequentlybeparalyzed:the
supraspinatus(abductoroftheshoulder)andinfraspinatus(lateral
rotatoroftheshoulder);thesubclavius(depressestheclavicle);the
bicepsbrachii(supinatoroftheforearm,flexoroftheelbow,weak
flexoroftheshoulder)andthegreaterpartofthebrachialis(flexorof
theelbow)andthecoracobrachialis(flexestheshoulder);andthe
deltoid(abductoroftheshoulder)andtheteresminor(lateralrotator
oftheshoulder).
Thus,thelimbwillhanglimplybytheside,mediallyrotatedbythe
unopposedsternocostalpartofthepectoralismajor;theforearmwill
bepronatedbecauseoflossoftheactionofthebiceps.
Thepositionoftheupperlimbinthisconditionhasbeenlikenedto
thatofaporterorwaiterhintingforatip.Inaddition,therewillbea

thatofaporterorwaiterhintingforatip.Inaddition,therewillbea
lossofsensationdownthelateralsideofthearm.
Treatment
ThethreemostcommontreatmentsfromErb'sPalsyare:Nerve
transfers(usuallyfromtheoppositeleg),subscapularisreleasesand
latissimusdorsitendontransfers.

3.Rootvalueofthoracodorsalnerve?
a)C5,C6,C7
b)C8,T1
c)C6,C7,C8
d)TiT2
CorrectAnswer-C
Ans.is'c'i.e.,C6C7C8
Branchesofbrachialplexus
Branchesofbrachialplexusarisesfromdifferentanatomical
segments:-
1.Branchesoftheroots
Nervetoserratusanterior(longthoracicnerve)(C5,C6,C7).
Nervetorhomboideus(dorsalscapularnerve)(C5).
2.Branchesofthetrunks
Theseariseonlyfromtheuppertrunkwhichgivestwobranches.I.
Suprascapularnerve(C5,C6)
Nervetosubclavius(C5,C6)
3.Branchesofthecords
1.Branchesoflateralcord
Lateralpectoral(C5-C7)
Musculocutaneous(C5-C7)
Lateralrootofmedian(C5-C7)
2.Branchesofmedialcord
Medialpectoral(C8,T1)
Medialcutaneousnerveofarm(C8,TO
Medialcutaneousnerveofforearm(C8,Ti).

Ulnar(C7,C8,Ti).C7fibresreachbyacommunicatingbranchfrom
lateralrootofmediannerve.
Medialrootofmedian(C8,T1).
3.Branchesofposteriorcord
Uppersubscapular(C5,C6)
Nervetolatissimusdorsi(thoracodorsal)(C6,C7,CO
Lowersubscapular(C5,C6)
Axillary(circumflex)(C5,C6)
Radial(C5-C8,T1)

4.Teresminorissuppliedby
a)Suprascapularnerve
b)Infrascapularnerve
c)Thoracodorsalnerve
d)Axillarynerve
CorrectAnswer-D
Ans.is'd'i.e.,Axillarynerve[Ref:BDC6h/eVoI.lp.671
Axillarynervesuppliesteresminoranddeltoid.

5.Insertionoflevatorscapulaeis?
a)Lateralborderofscapula
b)Suprolateralpartofscapula
c)Superiorpartofmedialscapulaborder
d)Inferiorangleofscapula
CorrectAnswer-C
Ans.is'c'i.e.,Superiorpartofmedialscapulaborder
[RefRockwood,CharlesA.;Matsen,(2009).Theshoulder,VoI.1]
Origin:
*PosteriortuberclesoftransverseprocessesofC1-C4vertebrae.
Insertion:
*Superiorpartofmedialborderofscapula

6.Numberoflobesinbreast
a)5
b)10
c)15
d)30
CorrectAnswer-C
Ans.is'c'i.e.,15lRef;BDCtr/eVol.Ip.361
Breast(mammarygland)isamodifeilsweatglaadpresentin
thesuperficialfasciaofpectoralregion.
Itconsistsof15-20lobes.
Verticallyitextendsfrom2ndto6thribsatmidclavicularline
andhorizontalextentisfromsternalmargintomidaxillarylineat
thelevelof4hrib.

7.Whichofthefollowingarisesfrom
infraglenoidtubercle-
a)Longheadofbiceps
b)Longheadoftriceps
c)Shortheadofbiceps
d)Coracobrachialis
CorrectAnswer-B
Ans.is'b'i.e.,Longheadoftriceps
Supraglenoidtubercleofscapula:originoflongheadofbiceps.
Infraglenoidtubercleofscapula:originoflongheadoftriceps

8.Notattachedonmedialborderofscapula?
a)Serratusanterior
b)Levatorscapulae
c)Rhamboidesmajor
d)Teresmajor
CorrectAnswer-D
Ans.is'd'i.e.,Teresmajor
Musclesattachedtoscapulaare:-
Coracoidprocess:-Tipofthecoracoidprocessgivesoriginto
coracobrachialis(medially)andshortheadofthebicepslaterally.
Theuppersurfacereceivesinsertionofpectoralisminor.
Spineofscapulaandacromionprocess:-ThereisoriginofDeltoid
andinsertionoftrapezius.
Glenoidtubercle:-Supraglenoidtuberclegivesorigintothelong
headofbicepsandinfraglenoidtuberclegivesorigintolongheadof
triceps.
Lateralborder:-Originsofteresminorandteresmajor.
Medialborder:-Insertionsofserattusanterior(anteriorly);and
rhomboideusmajor,rhomboideusminorandlevatorscapulae
(posteriorly).
Costal(anterior)surface(origin)Subscapularis.
Dorsalsurface(origins)Supraspinatus,infraspinatusandatinferior
anglelatissimusdorsi.

9.ApatienthasaherniatedintervertebraldiscimpingingontherightC5nerve
roots.Whichofthefollowingmovementswouldmostlikelybeaffected?
a)Extensionofthefingers
b)Extensionoftheshoulder
c)Flexionoftheelbow
d)Flexionofthewrist
CorrectAnswer-C
C5helpsmediateflexion,abduction,andlateralrotationoftheshoulder,andflexionofthe
elbow.BothC5andC6mediateextensionoftheelbow.
ExtensionofthefingersismediatedbyC7and8.
ExtensionoftheshoulderismediatedbyC7and8.
FlexionofthewristismediatedbyC6and7.

10.MedialboundryofCubitalfossa?
a)Brachioradialis
b)Pronatorteres
c)Supinator
d)None
CorrectAnswer-B
Ans.is'b'i.e.,Pronatorteres

11.Trueaboutlumbricalsis
a)FlexIPjointsandextendsMCPjoint
b)1stand2ndsuppliedbyradialnerve
c)3and4suppliedbysuperficialbranchofulnar
d)Originfromtendonsofflexordigitorumprofundus
CorrectAnswer-D
Answer.D
Originfromtendonsofflexordigitorumprofundus*
Thefourlumbricalmusclesarisefromthetendonsofflexor
digitorumprofundus.Theyhavedifferentorigins:
Eachpassesdistallytotheradialsideofitsnearest
metacarpophalangealjointofthefingerstobeinsertedinto
thedorsalextensorexpansionofdigitstwotofive.
Theiractionsonthesedigitsareto:
*Extendtheinterphalangealjoints
*Flexthemetacarpophalangealjoints
Theinnervationofthelumbricalsisdual:
*Theradialfirstandsecondlumbricalsaresuppliedby
themediannerve(C8,T1)
*Theulnarthirdandfourthlumbricalsaresuppliedbythedeep
branchoftheulnarnerve(C8,T1)
*Occasionally,thethirdlumbricalcanreceiveitsinnervationfrom
themediannerve.

12.Lateralboundaryofcubitalfossais
formedby:
a)Brachioradialis
b)Pronatorteres
c)Brachialis
d)Biceps
CorrectAnswer-A
Brachioradialis
Boundariesofcubitalfossa-
Laterally-Medialborderofbrachioradialis.
Medially-Lateralborderofpronatorteres.
Base-Itisdirectedupwards,andisrepresentedbyanimaginary
linejoiningthefrontoftwoepicondylesofthehumerus.
Apex-Itisdirecteddownwards,andisformedbytheareawhere
brachioradialiscrossesthepronatorteresmuscle.

13.MuscleofArmwithadditionalsupinator
action?
a)Brachialis
b)Biceps
c)Coracobrachialis
d)Triceps
CorrectAnswer-B
Ans.is'b'i.e.,Biceps

14.Thenervesupplyofnailbedofindex
fingeris?
a)Superficialbrofradialnerve
b)Deepbrofradialnerve
c)Mediannerve
d)Ulnarnerve
CorrectAnswer-C
Ans.is'c'i.e.,Mediannerve

15.Contentofanatomicalsnuffbox
a)Radialartery
b)Brachialartery
c)Ulnarartery
d)Interosseusartery
CorrectAnswer-A
Ans.is'a'i.e.,Radialartery[Ref:AKDuttap.861]
Triangulardepressiononthedorsalandradialaspectofthe
handbecomevisiblewhenthumbisfullyextended.
Boundaries
Media/PosteriorTendonoftheextensorpollicislongus.
Lateral/Anteriortendonoftheextensorpollicisbrevisand
abductorpollicislongus,
RoofSkinandfasciawithbeginningofcephalicveinand
crossedbysuperficialbranchoftheradialnerve
FloorStyloidprocessofradius,trapezium,scaphoidandbase
offirstmetacarpal
ContentsTheradialartery

16.3rdextensorcompartmentofwrist
containstendonof?
a)ECRL
b)ECRB
c)EPL
d)EPB
CorrectAnswer-C
Ans.C)EPL
Thethirdcompartmentcontainstheextensorpollicis
longustendon
,whichoriginatesatthemid-ulnaandinsertsatthe
baseofthefirstdistalphalanx.
IncombinationwiththeEPBtendon,itextendsthethumbatthefirst
carpometacarpalandfirstinterphalangealjoints.

17.Whichleavesthepelvis?
a)Piriformis
b)Sciaticnerve
c)Superiorglutealvessel
d)Inferiorglutealvessel
CorrectAnswer-B
Sciaticnerveleavesthepelvisandrunsposteriorlyinthethigh.
Intheupperangleofpoplitealfossa,sciaticnervedividesintotibial
nerveandcommonperonealnerve

18.Abductionandadductionoffootoccursat
whichjoints
a)Ankle
b)Subtalar
c)Tarso-metatarsal
d)None
CorrectAnswer-B
Ans.is'b'i.e.,Subtalar[RetClinicalarthopaedicsp.7861
Adductionandabductionoffootoccursmainlyatsubtalarjoint.
Movementstakeplaceatanklearedorsiflexionand
plantarflexion.
InversionandeversiontakeplaceatSubtalarjointandmidtarsal
joints.

19.Allaretrueaboutfemoraltriangle,EXCEPT?
a)Lateralmarginisformedbysartorius
b)Floorisformedbyadductorlongus
c)Containsthefemoralvessels
d)Noneoftheabove
CorrectAnswer-D
Allaretrueaboutthefemoraltriangle
Thefemoraltriangleisadepressedareaofthethighlyingdistalto
theinguinalfold.
Itsapexisdistal,itslimitsarethemedialmarginofsartoriuslaterally,
themedialmarginofadductorlongusmediallyandtheinguinal
ligamentproximally(thebase).
Itsfloorisprovidedlaterallybyiliacusandpsoasmajor,mediallyby
pectineusandadductorlongus.
Thefemoralvessels,passingfrommidbasetoapex,areinthe
deepestpartofthetriangle.Lateraltothearterythefemoralnerve
divides.
Thetrianglealsocontainsfatandlymphnodes.

20.Contentoffemoralcanal
a)Femoralbranchofgenitofemoralnerve
b)Genitalbranchofgenitofemoralnerve
c)Femoralvein
d)Lymphnode
CorrectAnswer-D
Ans.is'd'i.e.,Lymphnode[RefBDC5th/eVoI.2p.53,54]
Femoralsheathisafunnelshapedfascialprolongationaround
proximalpartoffemoralvessels,situatedinthefemoraltriangle,
belowtheinguinalligament.Itis3-4cmlong.ltisformedby
fasciailiaca.
Femoralringbbounded:Anteriorlybyinguinalligament,medially
bythelacunarligament,posterior/bypectineuswithitscovering
fascia,andlaterallybyseptumseparatingitfromfemoralvein.
FemoralcanalcontainslymphnodeofcloquetorRosenmuller
andlymphatics.

21.Apatientpresentswithdefective
adductionofthehipjointandpainsinthe
hipandkneejoint.Whichnerveis
involved

a)Obturatornerve
b)Femoralnerve
c)Saphenousnerve
d)Sciaticnerve
CorrectAnswer-A
A.i.e.Obturatornerve
AdductorsofthigharemainlysuppliedbyobturatornerveQ.
Adiseaseinhipjointmaycausereferredpaininknee&medialthigh
becauseoftheircommonnervesupplybyobturatornerveQ.


22.Structurewhichliesoutsidethefemoral
sheath
a)Femoralartery
b)Femoralnerve
c)Femoralvein
d)Genitofemoralnerve
CorrectAnswer-B
Femoralnerve
Femoralsheath
Femoralsheathisafunnelshapedfascialprolongationaround
proximalpartoffemoralvessels,situatedinthefemoraltriangle,
belowtheinguinalligament.Itis3-4cmlong.Itisformedbyfascia
iliaca.
Femoralsheathisdividedinto3separatefascialcompartementsby
septa:?
Lateralcompartment:-
Itcontainsfemoralarteryandfemoral
branchofgenitofemoralnerve.
Intermediatecompartment:-Femoralvein.
Medialcompartment(femoralcanal):-
Itisconicalinshape,wider
aboveadnnarrowbelow.Thewiderupperopeningisknown
asfemoralring,whichispotentiallyaweakpointinlowerabdomen
andisthesiteforfemoralhernia.Femoralringisbounded
:Anteriorlybyinguinalligament,mediallybylacunar
ligament,
posteriorlybypectineuswithitscoveringfascia,
andlaterallybyseptumseparatingitfromfemoralvein.Femoral
canalcontainslymphnodeofcloquetorRosenmullerand
lymphatics.


23.Mostcommonligamentdamagedinknee
injuryis
a)ACL
b)PCL
c)MCL
d)LCL
CorrectAnswer-C
Ans.is'c'i.e.,MCL[lRef;Textbookofsportsmedicine&/ep.
l38l]
Mostcommonlyinjuredligamentinknee-+Medialcollateral
ligament(MCL).
MCLtearisthemostcommonkneeligamentinjury"
MCLinjuryisthemostcommonligamentinjurytotheknee"
Practicalorthopaedicssportsmedicine
MCListhemostcommonlyinjuredkneeligament"
ACListhe2ndmostcommonlyinjuredkneeligament,almostas
frequentasMCL.

24.Whichstructure(s)passesbehindthe
inguinalligament:
a)Femoralbranchofgenitofemoralnerve
b)Femoralvein
c)Psoasmajor
d)All
CorrectAnswer-D
Ai.e.Femoralbranchofgenitofemoralnerve;Bi.e.Femoralvein;C
i.e.Psoasmajor

25.Lateralborderofischeorectalfossais
formedby?
a)Gluteusmaximus
b)Perinealmembrane
c)Pelvicdiaphram
d)Obturatorinternus
CorrectAnswer-D
Ans.is'd'i.e.,Obturatorinternus
Boundriesofischeoanal(ischeorectal)fossaare:-
Anteriorly:-Posteriorborderofperinealmembrane.
Posteriorly:-Gluteusmaximusmuscle,sacrotuberousligament.
Laterally:-Ischialtuberosityandobturatorinternus.
Medially:-Sphincteraniexternus(externalanalsphincter)and
pelvicdiaphragm(levatorani).

26.Arterialbrancheswhichsupplythehead
andneckofthefemuris/are:
a)Medialcircumflexartery
b)Lateralcircumflexartery
c)Profundafemorisartery
d)All
CorrectAnswer-D
A.i.e.Medialcircumflexartery;B.i.e.Lateralcircumflexartery;C.
i.e.Profundafemorisartery
Proximalfemur(head&neck)issuppliedbyarteryofligamentum
teres(branchofobturatorartery),medial(main)&lateralcircumflex
femoralartery
(botharisefromprofundafemorisartery,giveriseto
ascendingcervical(+metaphyseal)andretinacular(+
epiphyseal:lateral&inferior)arteriesandformextracapsular&
intracapsularsubsynovialarterialrings

27.Femoralnervesuppliesallexcept?
a)Pectineus
b)Sartorius
c)Vastusmedialis
d)Obturatorexternus
CorrectAnswer-D
Branchesoffemoralnerveare:?
1. Fromthemaintrunk:-Nervesupplytoiliacus,nervesupplyto
pectineusandafewvascularbranches.
2. Fromanteriordivision:-Intermediatefemoralcutaneousnerve
(intermediatecutaneousnerveofthigh),medialfemoralcutaneous
nerve(medialcutaneousnerveofthigh)andmuscularbranchto
sartorius.
3. Fromposteiordivision:-Saphenousnerve,nervesupplyto
quadricepsfemoris(rectusfemoris,vastusmedialis,vastuslateralis,
vastusintermedius).
4. Articularsupply:-Hipjointissuppliedbynervetorectusfemoris;
kneejointissuppliedbynervetothreevasti.

28.Superiorglutealnervedoesnotsupply?
a)Tensorfasciaelatae
b)Gluteusmedius
c)Gluteusminimus
d)Gluteusmaximus
CorrectAnswer-D
Ans.is'd'i.e.,Gluteusmaximus
Nervesupplyingmusclesofglutealregionare:-
Inferiorglutealnerve:Gluteusmaximus.
Superiorglutealnerve:Gluteusmediusandminimus.
Nervetopiriformis:Piriformis
Nervetoobturatorinternus:Obturatorinternus,Gemellussuperior.
Nervetoquadratusfemoris:Quadratusfemoris,Gemellusinferior.
Tensorfascialataeissuppliedbysuperiorglutealnerve.

29.Whatistrueaboutadductorsofthigh?
a)Ischialheadofadductormagnusisanadductor
b)Profundafemorisarteryisthemainbloodsupply
c)Ischialheadofadductormagnusoriginatesfromadductor
tubercle
d)Adductormagnusisthelargestmuscle
CorrectAnswer-D
Ans.is'd'i.e.,Adductormagnusisthelargestmuscle
Ischialheadofadductormagnusisahamstringmuscle(not
adductor).
Ischialheadofadductormagnusoriginatesfrominferolateralaspect
ofIschialtuberosity(notfromadductortubercle).
Mainarteryofadductor(medial)compartmentofthighisobturator
artery(notprofundafemoris).
Adductormagnusisthelargestmuscleoftheadductor
compartment.

30.

Anerveinjuredinradicalneckdissection
leadstolossofsensationinmedialsideof
thearm,nerveinjuredis?

a)Longthoracicnerve
b)Thoracodorsalnerve
c)Dorsalscapularnerve
d)Medialcutaneousnerveofarm
CorrectAnswer-D
Answer-D.Medialcutaneousnerveofarm
Themedialcutaneousnerveofthearmisthesmallestandmost
medialbranchofhebrachialplexus,andarisesfromthemedial
cord.
Itpiercesthedeepfasciaatthemidpointoftheupperarmtosupply
theskinoverthemedialaspectofthedistalthirdoftheupperarm.

31.Secondaryossificationcenterforlower
endoffemur?
a)Presentatbirth
b)Appearsat6monthsofage
c)Appearsat1yearofage
d)Appearsat5yearsofage
CorrectAnswer-A
Secondarycenteroflowerendoffemurappearsat9thmonthof
intrauterinelife(presentatbirth).
Ossificationoffemur
Thefemurossifiesfromoneprimaryandfoursecondarycentres.
Theprimarycentrefortheshaftappearsintheseventhweekof
intrauterinelife.Thesecondarycentresappear,oneforthelower
endattheendoftheninthmonthofintrauterinelife,oneforthe
headduringthefirstsixmonthsoflife,oneforthegreatertrochanter
duringthefourthyear,andoneforthelessertrochanterduringthe
twelfthyear.
Therearethreeepiphysesattheupperendandoneepiphysisatthe
lowerend.Theupperepiphyses;lessertrochanter,greater
trochanterandhead,inthatorder,fusewiththeshaftatabout
eighteenyears.Thelowerepiphysisfusesbythetwentiethyear.

32.Whichofthefollowingprevents
hyperextensionofthigh
a)Ischiofemoralligament
b)Iliofemoralligament
c)Patellofemoralligament
d)Puboischialligament
CorrectAnswer-B
B.i.e.Iliofemoralligament
Hipextensionislimitedbytheiliofemoralligament,whichpasses
overthefrontofthehipjointandconnectstheilium(hipbone)tothe
femur(thighbone).
Thisligamentelongateswhenthepelvisistiltedbackwards,
restrictingthedistancethejointcanbeextended.
Theiliofemoralligamentalsolimitsexternal(outward)rotationofthe
hipjointwhenflexed,anditrestrainsbothinternal(inward)and
externalrotationwhenthejointisextended

33.Lowerlimitofsuperiormediastinumisat
whichlevel-
a)Ti
b)T1
c)T8
d)T10
CorrectAnswer-B
Ans.is'b'i.e.,T1[Ref.BDC&/evol.lp.245]
Mediastinumisintrapleuralspaceboundedoneithersideby
mediastinalpleura,anteriorlybysternumandposteriorlyby
thoracicvertebralcolumn.
Itisdividedintosuperiorandinferiormediastinumbyaline
passingthroughsternalangleandlowerborderofTnvertebra.

34.Azygousveindrainsinto:
a)Rightsubcostalvein
b)Superiorvenacava
c)Braciocephalic
d)Rightascendinglumbarvein
CorrectAnswer-B
Theoriginoftheazygosveinisvariable.Itisoftenformedbythe
unionoftherightascendinglumbarveinandtherightsubcostal
vein.
Itascendsthroughtheaorticopeninginthediaphragmontheright
sideoftheaortatothelevelofthefifththoracicvertebra.
Hereitarchesforwardabovetherootoftherightlungtoemptyinto
theposteriorsurfaceofthesuperiorvenacava.Theazygosveinhas
numeroustributaries,includingthefifthtoeleventhrightposterior
intercostalveins,therightsuperiorintercostalvein,thehemiazygos
andtheaccessoryhemiazygosveins,andnumerousesophageal,
mediastinalandpericardialveins.

35.Archofaortabeginsandendsatwhich
level:
a)T2
b)T3
c)T4
d)T5
CorrectAnswer-C
Thearchoftheaorta(TransverseAorta)beginsatthelevelofthe
upperborderofthesecondsternocostalarticulationoftherightside,
andrunsatfirstupward,backward,andtotheleftinfrontofthe
trachea;itisthendirectedbackwardontheleftsideofthetrachea
andfinallypassesdownwardontheleftsideofthebodyofthefourth
thoracicvertebra,atthelowerborderofwhichitbecomes
continuouswiththedescendingaorta.

36.WhichamongthefollowingisNOTabranchofArchofAorta?
a)Brachiocephalic
b)Rightcommoncarotid
c)Leftcommoncarotid
d)LeftSubclavian
CorrectAnswer-B
Thearchoftheaortabeginsattheleveloftheupperborderofthesecondsternocostal
articulationoftherightside.Thebranchesgivenofffromthearchoftheaortaarethreein
number:thebrachiocephalicartery(innominate),theleftcommoncarotid,andtheleft
subclavian.
BrachiocephalicArteryisthelargestbranchofthearchoftheaorta.Itdividesintotheright
commoncarotidandrightsubclavianarteries.

37.Whichofthefollowingrepresentthecommonestvariationinthearteries
arisingfromthearchofaorta?
a)Absenceofbrachiocephalictrunk
b)Leftvertebralarteryarisingfromthearch
c)Presenceofretroesophagealsubclavianartery
d)Leftcommoncarotidarteryarisingfrombrachiocephalictrunk
CorrectAnswer-D
Mostcommonanomalyoftheaorticarchseenin10-20%individualsischaracterizedbythe
originoftheleftcommoncarotidarteryfromthebrachiocephalic(innominate)trunk.
Othercommonanomaliesinclude:Afourvesselarchwithseparateoriginsfortheright
commoncarotidandrightsubclavianarteries(2.5%),Originoftheleftvertebralartery
directlyfromafourvesselaorticarchtypicallybetweentheostiaoftheleftcommoncarotid
andsubclavianarteries(2.4-5.8%).

38.Intercostalnerveisabranchof?
a)Brachialplexus
b)Dorsalramiofthoracicspinalnerves
c)Ventralramiofthoracicspinalnerves
d)Ventralramiofcervicalspinalnerves
CorrectAnswer-C
Ans.is'c'i.e.,Ventralramiofthoracicspinalnerves
Ventralramiofupper11ththoracicspinalnervesareknownas
intercostalnervesandventralramusofT12isknownassubcostal
nerve.
Uppersixintercostalnervessupplythoracicwallwhereaslowerfive
intercostalnervesandsubcostalnervesupplythoracicandanterior
abdominalwallsandhenceknownasthoracoabdominalnerves.
Uppertwointercostalnervesalsosupplytheupperlimb.
Thusonly3rdto6tharecalledtypicalintercostalnerves.

39.Distanceofcricopharynxfromincisor
teeth
a)15cm
b)22.5cm
c)27.5cm
d)40cm
CorrectAnswer-A
Ans.is'a'i.e.,15cm[Ref-BDCSn/eVol.lp.269]
Cricopharynxisatthelevelofpharyngo-esophagealjunctioni.e.
beginningofesophagus.Itsdistanceis15cm(6inches)from
incisor.
Cardiacendislowerendofesophaguswhichisat40cmfrom
incisors.

40.

Anteriorpartofinterventricularseptumis
suppliedby-

a)Rightcoronaryartery
b)Leftcoronaryartery
c)Posteriordescendingcoronaryartery
d)None
CorrectAnswer-B
Anterior2/3rdofseptumissuppliedbyLCAandposterior1/3rdisby
RCA

41.Allveinsopeninsinusvenarumexcept-
a)SVC
b)Coronarysinus
c)Anteriorcardiacvein
d)Smallcardiacvein
CorrectAnswer-D
Ans.is'd'i.e.,Smallcardiacvein[Ref:Gray's38e/ep.14791
Smallcardiacveindoesnotopenintosinusvenarum
(Posteriorsmoothpartofrightatrium).

42.Trueaboutatrioventriculargrooveareall
except?
a)Containsleftanteriordescendingcoronaryartery
b)Alsocalledcoronarysulcus
c)Containsrightcoronaryartery
d)Containscircumflexbranchofleftcoronaryartery
CorrectAnswer-A
Ans.is'a'i.e.,Containsleftanteriordescendingcoronaryartery
Grooves(sulci)ofheart
A)Atrioventriculargroove
AtriaareseparatedbyventriclesbyatrioventricularSulcus
(atrioventriculargroove,alsocalledcoronarysulcus).
Itisdividedintoanteriorandposteriorparts.Therighthalfofanterior
partislargeandlodgesrightcoronaryartery.Lefthalfofanterior
partissmallandlodgescircumflexbranchofleftcoronaryartery.
B)Interventriculargrooves
Rightandleftventriclesareseparatedbyinterventriculargrooves.
Anteriorinterventriculargrooveisnearertoleftmarginofheartand
containsanteriorinterventricularartery(alsocalledleftanterior
descendingartery).
Posteriorinterventriculargrooveissituatedondiaphragmatic
(inferior)surface.Itcontainsposteriorinterventricularartery
(continuationofRCA).

43.Whichveinisfoundattheapexofthe
heart?
a)Greatcardiacvein
b)CoronarySinus
c)Anteriorcardiacvein
d)Middlecardiacvein
CorrectAnswer-A
Ans.is'a'i.e.,Greatcardiacvein
[RelBDC4n/eVoI.Ip.251-252;KeithMoore4e/ep.136-137;
Snell's9/ep.1211
GreatCardiacVein:
Thegreatcardiacvein(leftcoronaryvein)beginsattheapexof
heartandascendsalongtheanteriorlongitudinalsulcustothebase
ofventricle.

44.Whichofthefollowingdoesnotdirectly
drainintorightatrium?
a)Greatcardiacvein
b)Anteriorcardiacvein
c)Thebasianvein
d)Venaecordisminimi
CorrectAnswer-A
Ans.(A)Greatcardiacvein
Greatcardiacveindoesnotdirectlydrainintorightatrium.Itdrains
intocoronarysinus,whichinturndrainsintorightatrium.
Veinousdrainageofheart
Coronarysinus:Opensintorightatriumandhasfollowingtributaries
:(i)Greatcardiacvein,(ii)Middlecardiacvein,(iii)Posteriorveinof
leftventricle,(iv)Smallcardiacvein,(v)Obliqueveinofleftatrium
and(vi)Sometimesrightmarginalvein.
Anteriorcardiacvein:Opensintorightatrium.
Venaecardisminimi(thebasianveins):Alltheseareextremely
smallveinsinthewallsofallthe4chambersoftheheart.Theyopen
directlyintotherespectivechambers.They'remostnumerousinthe
rightatrium.
Rightmarginalvein:Moreoftenopensintorightatriumbut
sometimesintocoronarysinus.

45.Tributaryofcoronarysinus?
a)Anteriorcardiacvein
b)Thebesionvein
c)Smallestcardiacvein
d)Greatcardiacvein
CorrectAnswer-D
Ans.(D)Greatcardiacvein
Coronarysinus
Itopensintheposteriorwallofrightatrium,intheposteriorpartof
coronarysulcus.
ItopensintherightatriumbetweenIVCandtricuspidorifices.
CoronarysinusisguardedbyThebesianvalve(Thebesianvalve
(incompletesemilunarvalve)guardstheorificeofcoronarysinus.)
Tributariesofcoronarysinusare:
Greatcardiacvein:-Liesintheanteriorinterventriculargroove.Left
marginalvein
drainsintoit.
Middlecardiacvein:-Liesintheposteriorinterventriculargroove.
Posteriorveinofleftventricle.
Smallcardiacvein:-
Itliesintheposteriorpartofcoronarysulcus
withRCA.Rightmarginnalveinmaysometimesopenintosmall
cardiacvein,moreoften,however,rightmarginalveinopens
directlyintorightatrium.
Obliqueveinofleftatrium(veinofmarshall):-
Itiscontinuousabove
withligamentofIVC.Thesetwostructuresareembryological
remnantsofleftcommoncardinalvein(ductofcuvier).

46.Whicharesegmentsofupperlobeofright
lung?
a)Anterior,posterior,media
b)Lateral,medial,superior
c)Apical,anterior,posterior
d)Basal,medial,lateral
CorrectAnswer-C
Ans.is'c'i.e.,Apical,anterior,posterior

47.Whatistheuppermoststructureinleft
lunghilum?
a)Pulmonaryartery
b)Pulmonaryvein
c)Bronchialartery
d)Leftmainstembronchus
CorrectAnswer-A
Ai.e.,Pulmonaryartery
Arrangementofstructuresinthehilumisasfollows-
l.Anteriortoposterior(sameonbothside):-(i)Superior
pulmonary(ii)Pulmonaryartery,(iii)Bronchus
2.Superiortoinferior-vein,
a)Right:-(i)Eparterialbronchus(superiormost)'(ii)Pulmonary
artery,iii)Hlparterialbronchus,iv)Inferiorpulmonaryvein(inferior
most).
b)Left:-(i)Puhnonaryartery(superiormost),(ii)Primary/Principal
bronchus(iii)Inferiorpulmonaryvein(inferiormost)

48.Lymphaticdrainageofovary?
a)Deepinguinal
b)Superficialinguinal
c)Obturator
d)Paraaortic
CorrectAnswer-D
Ans.is'd'i.e.,Para-aortic
Lymphaticsoftheovarydraintopara-aorticnodesalongsidethe
originoftheovarianartery(L2).

49.Whichstructuredoesnotpassthrough
superiorthoracicaperture-
a)Rightvagus
b)Rightbrachiocephalicartery
c)Thoracicduct
d)Rightrecurrentlaryngealnerve
CorrectAnswer-D
Ans-Ans.is'd'i.e.,Rightrecurrentlaryngealnerve[Ref-
BDC6h/eVol.lp.192]
Thoracicinlet(superioraperture)
Thenarrowupperendofthethorax,whichiscontinuouswith
theneckiscalledtheinletofthethorax'
Thestructurepassingthroughtheinletare:-l.Vkcera:-
Trachea,esophagus,apexoflungwithpleura,remainsofthe
thymus,thoracicduct.

50.Ligamentumarteriosumisderivedfrom:
a)Ductusarteriosus
b)Ductusvenosus
c)Ductusutriculosaccularis
d)Ductusreunions
CorrectAnswer-A
Ans.A:Ductusarteriosus
Theductusarteriosusrepresentsthedistalportionofthesixthleft
aorticarchandconnectstheleftpulmonaryarterytothebeginning
ofthedescendingaorta.
Duringfetallife,bloodpassesthroughitfromthepulmonaryarteryto
theaorta,thusbypassingthelungs.
Afterbirth,itnormallyconstricts,latercloses,andbecomesthe
ligamentumarteriosum.
Apersistentpatentductusarteriosusresultsinhigh-pressureaortic
bloodpassingintothepulmonaryartery,whichraisesthepressure
inthepulmonarycirculation.
Apatentductusarteriosusislifethreateningandshouldbeligated
anddividedsurgically.

51.Baseoftheheartisformedby?
a)Rightatrium
b)Rightventricle
c)Leftatrium
d)Leftventricle
CorrectAnswer-C
Ans.is`c>a'i.e.,Leftatrium>Rightatrium
Base(posteriorsurface)isformedmainlybyleftatriumandpartlyby
rightatrium.ItisseparatedfromT5toT8vertebraebypericardium
Theapex(themostinferior,anterior,andlateralpartas
theheartliesinsitu)islocatedonthemidclavicularline,inthefifth
intercostalspace.Itisformedbytheleftventricle.
Anterior(sternocostal)surfaceisformedmostlybyrightventricle
(major)andrightauricleandpartlybyleftventricleandleftauricle.

52.Rightborderofheartisformedby?
a)Rightventricle
b)Rightatrium
c)SVC
d)IVC
CorrectAnswer-B
Ans.(B)Rightatrium
Externalfeaturesofheart
Thehearthasfollowingbordersandsurfaces:-
A)Borders
Rightborder:-Formedbyrightatrium.
Leftborder(obtusemargin):-Formedmainlybyleftventricleand
partlybyleftauricle(initsuppermostpart).
Inferiorborder(acutemargin):-Formedmainlybyrightventricleand
partlybyleftventriclenearapex.
Upperborder:-Mainlybyleftatriumandpartlybyrightatriumwhere
SVCenters.
Apex:-Formedbyleftventricle.
B)Surfaces
Anterior(sternocostal)surface:-Formedmostlybyrightventricle
(major)andrightauricleandpartlybyleftventricleandleftauricle.
Inferior(diaphragmatic)surface:-Itisformedbyleftventricle(left
2/3)andrightventricle(right1/3).Itistraversedbyposterior
interventriculargroove(PIV)containingPIVbranchofRCA.

53.Trueaboutanatomyofrightventricle:
a)TV&PVSharefibrouscontinuity
b)Moreprominenttrabeculation
c)CristasupraventricularisSeparateTricuspidvalve&Pulmonary
valveandApextrabeculatedboth
d)All
CorrectAnswer-D
AllCorrect
RightventricleissituatedanteriorlyQ.Astheanterior(sternocostal)
surfaceofheartconsistsmainlyofrightventricleQwithrightatrium
onitsrightandanarrowstripofleftventricleleftborder.Thetipof
leftauricularappendagepeepsoverthetoofthisborder.

54.Sensorysupplytotongueisbyall,EXCEPT?
a)Lingualnerve
b)Vagusnerve
c)Glossopharyngealnerve
d)Noneoftheabove
CorrectAnswer-D
Thesensoryinnervationofthetonguereflectsitsembryologicaldevelopment.
Thenerveofgeneralsensationtothepresulcalpartisthelingualnerve,whichalsocarries
tastesensationderivedfromthechordatympanibranchofthefacialnerve.
Thenervesupplyingbothgeneralandtastesensationtothepostsulcalpartisthe
glossopharyngealnerve.
Anadditionalareaintheregionofthevalleculaeissuppliedbytheinternallaryngealbranch
ofthevagusnerve.

55.

Internalanalsphincterisaapartof:
a)Puborectalismuscle
b)Deepperinealmuscles
c)Internallongitudinalfibers
d)Internalcircularfibers
CorrectAnswer-D
D.i.e.Internalcircularfibres
Involuantryinternalanalsphincterisformedbythickeningofcircular
musclelayer(i.e.circularlayerofmuscularisexterna,asmooth
muscle)
ofupperend(2/3or3/4)ofanalcanal.Thissphincter
remainsinthestateoftoniccontractionmostofthetimetomaintain
restingtoneorpressures
(-90cmH20)&topreventleakageoffluid
orflatus.Itscontraction(tonus)ismaintainedbysympatheticfibers
fromsuperiorrectal(periarterial)andhypogastricplexuses;and
inhibited(i.e.sphincterrelaxed)byparasympatheticpelvic
splanchnicnerves.


56.

Shortestpartofmaleurethrais:
a)Prostatic
b)Membranous
c)Bulbar
d)Penile
CorrectAnswer-B
Bi.e.Membranous
-Membranousurethraisshortest&Penileurethraislongestpart.
Prostaticurethraiswidest&mostdilatableportion&Urethralorifice
f/19membranousurethraisnarrowest&leastdilatablepart


57.Posteriorrelationofneckofpancreas?
a)IVC
b)Originofpostalvein
c)Aorta
d)Commonbileduct
CorrectAnswer-B
Ans.is'b'i.e.,Originofportalvein

58.Therightsuprarenalveindrainsintothe
a)Inferiorvenacava
b)Rightrenalvein
c)RightGonadalvein
d)LeftRenalvein
CorrectAnswer-A
Ai.e.Inferiorvenacava
Lefttesticular,ovarianorsuprarenalveinusuallydrainsintoleft
renalvein,
beforeenteringtheIVC
Vein Draininto
Organ
Rt.Suprarenalgland
Rt.Suprarenalvein IVC
Lt.Suprarenalgland
Lt.Suprarenalvein LeftRenalVein
Sameistrueforgonads(testis/ovary)i.e.
Lt.Gonad(testisor Lt.Gonadal(testicular Leftrenalvein
ovary) orovarian)vein
Rt.Gonad(testisor Rt.Gonadal(testicular IVC
ovary) orovarian)
vein

59.Suprarenalglandgetsitsbloodsupply
fromallofthefollowingarteriesexcept:
a)Aorta
b)Renalartery
c)Inferiorphrenicartery
d)Superiormesentricartery
CorrectAnswer-D
D.i.e.SuperiorMesentericartery
RightsuprarenalveindrainintoIVC;andleftsuprarenalveindrain
intoleftRenalveinSuprarenalglandissuppliedbysuprarenal,
renal,inferiorphrenicarteryandaorta

60.Inferiorpancreaticoduodenalarteryisabranchofwhichofthefollowing
artery?
a)Splenicartery
b)Leftgastricartery
c)Gastroduodenalartery
d)Superiormesentericartery
CorrectAnswer-D
Inferiorpancreaticoduodenalarteryisabranchofsuperiormesentericartery.Itsuppliesthe
pancreasandadjoiningpartoftheduodenum.Itsanteriorandposteriorbranches
anastomosewiththebranchesofsuperiorpancreaticoduodenalartery.Thisanastomosisis
theonlycommunicationbetweenthearteriesofforegutandmidgut.
Branchesofsuperiormesentericarteryare:
Inferiorpancreaticoduodenalartery
Jejunalandilealbranches
Ileocolicartery
Rightcolicartery
Middlecolicartery

61.Superiorpancreaticoduodenalarteryisa
branchof?
a)Hepaticartery
b)Splenicartery
c)Gastroduodenalartery
d)Inferiormesentericartery
CorrectAnswer-C
Ans.is'c'i.e.,Gastroduodenalartery
Gastroduodenalarterygivesfollowingbranches:-
1. Rightgastroepiploicartery
2. Superiorpancreaticoduodenalartery

62.Allofthefollowingarebranchesofsplenicartery,except?
a)Hilarbranches
b)ShortGastricArtery
c)ArteriaPancreaticaMagna
d)RightGastroepiploicArtery
CorrectAnswer-D
RightGastroepiploicarteryisabranchofsuperiormesentricartery,abranchofhepatic
artery.TheleftGastroepiploicarteryisabranchofsplenicarterywhichanastomoseswith
therightGastroepiploicartery.

63.Commonhepaticarteryisabranchof-
a)Splenicartery
b)Superiormesentericartery
c)Inferiormesentericartery
d)Coeliactrunk
CorrectAnswer-D
Ans.is'd'i.e.,Coeliactrunk[RefBDC6h/eVoI.2p.2761
Commonhepaticarteryisabranchofcoeliactrunk.

64.Esophagealvaricesoccurinwhich
portionofesophagus?
a)Upper
b)Middle
c)Lower
d)Allsites
CorrectAnswer-C
Ans.is'c'i.e.,Lower[Ref:AKDuttaP.275,2761
Esophagealvaricesoccursatlowerendofesophagusdueto
porto-cavalanastomosis.

65.Externalobliqueformsallexcept?
a)Lacunarligament
b)Pectinealligament
c)Conjointtendon
d)Inguinalligament
CorrectAnswer-C
Inguinalligament(Poupart'sligament)isthefoldedlowerborderof
externalobliqueaponeurosis
Lacunarligament(Gimbernatsligament)isthecrescentshaped
expansionfromthemedialendofinguinalligamentattachedto
pectineallineofpubis.
Pectinealligament(Cooper'sligament)isstrongfibrousband
extendinglaterallyfromthelacunarligamentalongpectineallineof
pubis.Similartolacunarligament,itismadeofexternaloblique
aponeurosis.
Reflectedpartofinguinalligamentextendsfromthelateralcrusof
superficialinguinalringformedbyinguinalligamentupwardstolinea
alba.Itformstheposteriorwallofinguinalcanal.
Conjointtendon(falxinguinalis)isformedbytheaponeurosesof
internalobliqueandtransversusabdominismuscleandisattached
topubiccrest.

66.Inferiorepigastricarteryformsthe
boundryof?
a)Femoraltriangle
b)Hesselbach'striangle
c)Adductorcanal
d)Poplitealtriangle
CorrectAnswer-B
Ans.is'b'i.e.,Hesselbach'striangle
Theinguinaltriangle(Hesselbach'striangle)isaregioninthe
anteriorabdominalwall.Itisalternativelyknownasthemedial
inguinalfossa.
ItwasfirstdescribedbyFrankHesselbach,aGermansurgeonand
anatomist,in1806.
Theinguinaltriangleislocatedwithintheinferomedialaspectofthe
abdominalwall.Ithasthefollowingboundaries:
Medial?lateralborderoftherectusabdominismuscle.
Lateral?inferiorepigastricvessels.
Inferior?inguinalligament.

67.Theboundariesoftheinterconnectionbetweengreatersacandlessersacof
peritoneumknownas'ForamenofWinslow'areall,EXCEPT:
a)Caudatelobeofliver
b)Inferiorvenacava
c)Freeborderoflesseromentum
d)4thpartofDuodenum
CorrectAnswer-D
InterconnectionbetweengreatersacandlessersacofperitoneumisknownasForamenof
Winslow.Ithasthefollowingboundaries:
Superiorboundary:Caudatelobeofliver
Anteriorboundary:Freeedgeoflesseromentumcontainingcommonbileduct,hepatic
arteryandportalvein.
Inferiorboundary:Firstpartofduodenum
Posteriorboundary:Inferiorvenacavaandabdominalaorta

68.Anorectalangleisformedduetoactionof
-
a)Internalanalsphincter
b)Circularmusclelayerofsmoothmuscles
c)Longitudinalmusclelayerofsmoothmuscle
d)Puborectalis
CorrectAnswer-D
Ans.is'd'i.e.,Puborectalis
Theano-rectalangle(ARA)istheanglebetweenlongitudinal
axisofrectum(whichisrepresentedbyposteriorrectalline)and
longitudinalaxisofanalcanal.
Thenormalaveragevalueis95-96"(physiologicalrange65-
100').
ARAisanindirectindicatorofthepuborectalismuscleactivity.
Duringmusclecontraction,ARAbecomesmoreacute,while
duringrelaxingphaseitbecomesobtuse.

69.Prostateanalogueinfemaleis-
a)Skenegland
b)Bulbourethralgland
c)Greatvestibulargland
d)Bartholin'sgland
CorrectAnswer-A
Ans.is'a'i.e.,SkeneglandlRef:Clinicalanatomyp.421]
Prostateanologinfemal>Skeneglands(Periurethral
glands).
Uterusandvaginaanaloginmale>Prostaticutricle.

70.Theuterinearteryisabranchofwhichofthefollowing?
a)Leftcommoniliacartery
b)Internaliliacartery
c)Internalpudendalartery
d)Ovarianartery
CorrectAnswer-B
Uterinearteriesarisefrominternaliliacarteryorhypogastricartery.
Itrunsdownwardsandforwardsalongthelateralpelvicwalluntilthebaseofthebroad
ligament.
Note:Duringpregnancytheuterinearterieshypertrophyandtheircourseisstraightened.
Ref:CompTextbookofObstetricsandGynecology,SadhanaGupta,2011,Page18

71.

Allaretrueabout1stpartofduodenum,
except?

a)5cmlong
b)Issuperiorpart
c)Developsfromforegut
d)Suppliedbysuperiormesentericartery
CorrectAnswer-D
Ans.is'd'i.e.,Suppliedbysuperiormesentericartery
Duodenum
Duodenumis'C'shaped,shortest,widestandmostfixedpartof
smallintestine.Itis25cmlong.Itisdevoidofmesentery.Mostof
theduodenumisretroperitonealandfixed,
exceptatitstwoends
whereitissuspendedbyfoldsofperitoneum,andistherefore
mobile.DuodenumliesoppositeLl,L2andL3vertebrae.
DuodenumisC-shapedcurvewhichenclosestheheadofpancreas
andissubdividedintofourparts:
1.Firstpart(Superiorpart):Itis5cm(2inches)long.Itbeginsat
thepylorusandmeetthesecondpartatsuperiorduodenalflexure.
Thispartappearsasduodenalcaponbariumstudies.
2.Secondpart(Descendingpart):Itisabout7.5cm(3inches)long.
Itisverticalpartwhichbeginsatsuperiorduodenalflexureandmeet
thethirdpartatinferiorduodenalflexure.Theinteriorofsecondpart
ofduodenumshowsfollowingfeatures:
i)Majorduodenalpapilla:Itispresent8-10cmdistaltothepylorus.
ThehepatopancreaticampullaorampullaofVater(jointpartofbile
ductandpancreaticduct)openshere.
ii)Minorduodenalpapilla:Itispresent6-8cmdistaltothepylorus.
Accessorypancreaticductopenshere.

3.Thirdpart(Horizonalpart):Itis10cm(4inches)long.Itbeginsat
inferiorduodenalflexureandpassestowardstheleftinfrontofIVC
behindsuperiormesentericvesselsandrootofmesenterytomeet
4thpartofduodenum.
Fourthpart:Itis2-5cm(1inches)longandrunsupward
immediatelytotheleftofaorta.Itendsatduodenojejunalflexureby
joiningthejejunum.
Arterialsupply
Thepartofduodenumbeforetheopeningofbileduct(major
duodenalpapilla)developsfromforegutandthereforeissuppliedby
coeliactrunk
throughsuperiorpancreaticoduodenalartery,abranch
ofgastroduodenalartery,whichintrunisabranchofcommon
hepaticartery.Partofduodenumdistaltoopeningofbileductis
developedfrommidgutandthereforeissuppliedbysuperior
mesentericartery
throughinferiorpancreaticoduodenalartery.First
partofduodenumreceivesadditionalsupplyfromrightgastricartery,
supraduodenalartery(a
branchofcommonhapaticartery),
retroduodenalbranchofgastrodudenalarteryandrightgastropiploic
artery.

72.Ventralpancreaticductgiveriseto?
a)Body
b)Tail
c)Neck
d)Uncinateprocess
CorrectAnswer-D
Ans.is'd'i.e.,Uncinateprocess
Developmentofpancreas
Pancreasisdevelopedfromthetwopancreaticbuds.
1. Dorsalpancreaticbud:-Itislargerandmostofthepancreasis
derivedfromiti.e.mostofthehead,andwholeneck,body&tail.
2. Ventralpancreaticbud:-Itissmallerandformslowerpartofthe
headofpancreasincludinguncinateprocess.
During7thweekofdevelopment,theventralanddorsalpancreatic
budsfusetoformasinglepancreaticmass.
Afterthefusionofventralanddorsalpancreaticbuds,theirducts
developcrosscommunications.Finalductsystemisformedas
below?
1. Mainpancreaticduct(Ductofwirsung)isformedbytheductof
ventralbud,distalpartofductofdorsalbudandanoblique
communicationbetweenthetwo.Themainpancreaticductjointhe
bileducttoformhepatopancreaticampullathatentersth2ndpartof
duodenumatmajorduodenalpapilla.
2. Accessorypancreaticductisformedbytheproximalpartoftheduct
ofdorsalbud.Itopensinto2ndpartofduodenumatminorduodenal
papilla,2cmproximal(cranial)tomajorduodenalpapilla.
Anomaliesofpancreaticdevelopmentmaybe:
1. Annularpancreas:-Twocomponentsoftheventralbudfailtofuse

andgrowinoppsitedirectionarroundtheduodenumandmeetthe
dorsalpancreaticduct.
2. Pancreaticdivisum(dividedpancreas):-Ventralanddorsalbudsfail
tofusewitheachother.Itisthemostcommoncongenitalanomalyof
pancreas.
3. Inversionofpancreaticduct:-Themainpancreaticductisformedby
theductofdorsalbud,i.e.accessoryductislargerthanthemain
ductandthemaindrainageofpancreasisthroughtheminor
duodenalpapilla.
4. Accessorypancreatictissue:-Maybefoundin?
Wallofstomach,duodenum,jejunumorileum.
Meckel'sdiverticulum.

73.Tailofpancreasedevelopsfrom-
a)Hepaticdiverticulum
b)Dorsalpancreaticduct
c)Ventralpancreaticduct
d)Alloftheabove
CorrectAnswer-B
Ans.is'b'i.e.,Dorsalpancreaticduct[RefI.B.SinghVh/ep.
28]


74.Haustrationsarepresentin-
a)Duodenum
b)Ileum
c)Jejunum
d)Colon
CorrectAnswer-D
Ans.is'd'i.e.,Colon[RelBDC#/eVol.2p.2661
1. Characteristicsfeaturesoflargeintestine(colon)are:-i)
3longitudinalbands,formedbylongitudinalmusclecoat,called
Taeniaecoli.
2. Sacculationorhaustration
3. Fatfilledperitonealpouchescalledappendicesepiploicae.
Thesearenotfoundinappendix,caecumandrectum.
4. Greaterpartisfixedexceptforappendix,transversecolonand
sigmoidcolon.
5. Pyere'spatches(presentinsmallintestine)arenotpresent.

75.Themusclesattachedtoperinealbodyare
A/E
a)Ischiocavernosum
b)Bulbospongiosm
c)Superficialtransverseperinea
d)Deeptransverseperinea
CorrectAnswer-A
A.i.e.Ischiocavernosus
Tenmusclesoftheperineumconvergeandinterlaceintheperineal
body-
a)Twounpaired:(i)Externalanalsphincter,(ii)Fibresof
longitudinalmusclecoatofanalcanal.
b)Fourpaired:-(i)Bulbospongiosus,(ii)Superficialtransverse
perenei,(iii)Deeptransversusperenei,(iv)levatorani,
Infemales,sphincterurethrovaginalisisalsoattachedhere.

76.Trigoneofurinarybladderdevelopsfrom:
a)Mesoderm
b)Ectoderm
c)Endodermofurachus
d)Noneoftheabove
CorrectAnswer-A
Withdifferentialgrowthofthedorsalbladderwall,theureterscome
toopenthroughthelateralanglesofthebladder,andthe
mesonephricductsopenclosetogetherinwhatwillbetheurethra.
Thatpartofthedorsalbladderwallmarkedoffbytheopeningsof
thesefourductsformsthetrigoneofthebladder.
Thus,liningofthebladderoverthetrigoneismesodermalinorigin;
Thesmoothmuscleofthebladderwallisderivedfromthe
splanchnopleuricmesoderm.
Theapexofthebladderiscontinuouswiththeallantois,whichnow
becomesobliteratedandformsafibrouscore,theurachus.
Theurachuspersiststhroughoutlifeasaligamentthatrunsfromthe
apexofthebladdertotheumbilicusandiscalledthemedian
umbilicalligament
oLininingepitheliumofbladdermucosaistransitionalepithelium.
Whenemptymucosaisthrownintorugaeexceptintrigone,where
mucosaissmoothandfirmlyadherent.
ojustbeneaththemucosaoftrigonethereislayerofsmooth
muscle,TrtgonalmuscleofBellwhichreplacesthesubmucouscoat
intrigonearea

77.

Anteriorrelationtoupperpartofrectumin
maleis-

a)Rectovesicalpouch
b)Sacrum
c)Seminalvesicle
d)Ductusdeference
CorrectAnswer-A
Answer-is'a'i.e.,RectovesicalpouchlRef;BDCPleVol.2p.
408}
AnteriorlyInmales
Theuppertwo-thirdsoftherectumisrelatedtothe
rectovesicalpouchwithcoilsofintestineandsigmoidcolon.
Thelowerone-thirdoftherectumisrelatedtothebaseofthe
urinarybladder,theterminalpartsoftheureters,theseminal
vesicles,thedifferentductsandtheprostate.

78.FALSEforseminalvesicles:
September2012
a)Containslargeamountoffructose
b)Storessperms
c)Situatedoneithersidenearprostate
d)Secretionofseminalvesiclegivesmucoidconsistencyto
semen
CorrectAnswer-B
Ans.Bi.e.Storessperms
Spermsarestoredinepididymis

79.Sensorynervesupplyofgallbladderis
through-
a)Vagusnerve
b)Trigeminalnerve
c)Parasympatheticnerve
d)Facialnerve
CorrectAnswer-A
Ans.is'a'i.e.,Vagusnerve[RelBDC&/eVol.3p.2901
PainsensoryfiberstogallbladderarethroughVagus,
synpatheticandphrmicnerves.Thusgallbladderpainmaybe
referredto-a)Throughvagustothestomach(epigastrium).
Throughthesympatheticnervestotheinferiorangleoftheright
scapula.Lateralhornofthoracic7segmentofspinalcordgives
sympatheticfibrestocoeliacganglionthroughgreater
splanchnicnerve.T7segmentreceivespainfibresfromskin
overinferiorangleofscapula.Sovisceralpainisreferredto
somaticarea.
Throughthephrenicnervetotherightshoulder(C4gives
fibrestophrenicnerveandsupraclavicularnerves).

80.Uvulavesicaeseeninbladderisformedfromthefollowingstructure?
a)Medianlobeofprostate
b)Laterallobeofprostate
c)Anteriorlobeofprostate
d)Posteriorlobeofprostate
CorrectAnswer-A
Uvulavesicaeisasmallelevationsituatedimmediatelybehindtheurethralorifice,whichis
producedbytheunderlyingmedianlobeoftheprostate
.Itenlargeswithagedueto
enlargementoftheunderlyingmedianlobeoftheprostateandmayinhibitcompletebladder
emptying.

81.Ligamentwhichpreventsspleentofallin
leftiliacfossa-
a)Leinorenalligament
b)Phrenicolicligament
c)Upperpoleofrightkidney
d)Sigmoidcolon
CorrectAnswer-B
Answer-'b'i.e.,Phrenicolicligament[Ref:Gruy's4Uh/ep.
1107,1108,12141
Spleenliesinleftsideofabdomen(lefthypochondrium).
Butenlargmentofspleen(splenomegaly)doesnotcause
extensionintoleftiliacfossa.
Apathologicallyenlargedspleenextendsdownwardand
mediallytowardsrightiliacfossabecauseprenicocolicligament
andleftcolicflexurepreventadirectdownwardenlargement.

82.FloorofPetittriangleisformedby?
a)Sacrospinalis
b)Internaloblique
c)Rectusabdominis
d)FasciaTransversalis
CorrectAnswer-B
Ans.is'b'i.e.,Internaloblique[RefTextbookofsurgical
anatomyp.148]
BoundariesofPetittriangle(inferiorlumbartriangle)are?
Base-Iliaccrest
Anteriorboundary(abdominalboundary)Posteriorborderof
externalobliquemuscle.
Posteriorboundary(lumbarboundary)-Anteriorborderoflatissimus
dorsi.
Floorisformedbyinternalobliquemuscle.

83.Inwhichofthefollowingvessels
transversemesocolonseen?
a)Rightcolicartery
b)Leftcolicartery
c)Middlecolicartery
d)Iliocolicartery
CorrectAnswer-C
Ans.is'c'i.e.,Middlecolicartery
Mesenteryofgut
Vesselscontainedbymesentery
Mesenteryproper(Mesentery JejunalandIlealbranchesof
ofsmallintestine)
superiormesentericvessels
Transversemesocolon
Middlecolicvessel
Mesoappendix
Appendicularvessels
Sigmoidmesocolon
Sigmoidvessels

84.Ovarianarteryisabranchof:
a)Renalartery
b)Internaliliacartery
c)Abdominalpartoftheaorta
d)Externaliliacartery
CorrectAnswer-C
Theovarianarteryarisesfromtheabdominalpartoftheaortaatthe
levelofthefirstlumbarvertebra.Thearteryislongandslenderand
passesdownwardandlaterallybehindtheperitoneum.Itcrossesthe
externaliliacarteryatthepelvicinletandentersthesuspensory
ligamentoftheovary.
Itthenpassesintothebroadligamentandenterstheovarybyway
ofthemesovarium.

85.Labourpaininuterusiscarriedby
a)Parasympatheticnerves
b)Sympatheticnerves
c)Pudendalnerve
d)Splanchnicnerve
CorrectAnswer-B
Ans.is'b'i.e.,Sympatheticnerves[Ref:Clinicalobstetics
3d/ep.9121
Nervesupplyofuterus:
Theuterusissuppliedbybothsystems,sympatheticand
parasyrnpathetic.
SympatheticsystemfibersarisefomTn,Ltsegmentsandcarry
painfulsensationsfromthebodyoftheuterus.
ParasyrnpatheticfibersarisefromS2,S3,S4(inpelvic
splanchnicnerve)andcarrypainfulsensationsfromcenix.

86.Nervesuplyofcremastricmuscle?
a)Pudendalnerve
b)Femoralbranchofgenitofemoral
c)Genitalbranchofgenitofemoralnerve
d)Ilioinguinalnerve
CorrectAnswer-C
Genitalbranchofgenitofemoralnerve
Cremastricmuscleisamuscleofscrotum.Itissuppliedbygenital
branchofgenitofemoralnerve.

87.Trueabouttheanatomyofgreat
saphenousvein:
a)Startsasacontinuationofmedialmarginalvein
b)Endsoffemoralvein2.5cmbelowtheinguinalligament
c)Thereare2-5valvesbelowtheknee.
d)Ascends2.5-3cmbehindtibialmalleolus
CorrectAnswer-A
Ai.e.Startsascontinuation-ofmedialmarginalvein

88.Posteriorrelationofrightkidneyareall
except-
a)Diaphragm
b)Subcostalnerve
c)11thrib
d)Ilioinguinalnerve
CorrectAnswer-C
Ans.is'c'i.e.,11thrib
Posteriorsurfaceofbothkidneyisrelatedtodiaphragm,medial
andlateralarcuateligament,psoasmajor,quadratuslumborum,
transversusabdominis,subcostalvessels,subcostalnerve,
iliohypogastricnerve,andilioinguinalnerve.
Inaddition,therightkidneyisrelatedto12thribandtheleft
kidneyisrelatedto11thand12thribs.

89.NotTrueaboutbloodsupplyofkidney-
a)RenalveindrainsintoIVC
b)Renalarteryisabranchofcommoniliacartery
c)RightrenalarterypassesbehindIVC
d)Branchesofrenalarteryareendarteries
CorrectAnswer-B
Ans.is'b'i.e.,Renalarteryisabranchofcommoniliac
artery[Re/BDC&/eVol.2p.313-3171
Eachkidneyissuppliedbyrenalartery(branchofabdominal
aorta)andisdrainedbyrenalveintoIVC.
Renalartery:-Rightrenalarteryislongerandpassesbehind
IVC.
Branchesoftherenalarteryareendarteries.

90.Allpassthroughdeepinguinalring,EXCEPT?
a)Spermaticcord
b)Internalspermaticfascia
c)Roundligament
d)Ilioinguinalnerve
CorrectAnswer-D
Ittransmitsthespermaticcordinthemaleandtheroundligamentoftheuterusinthe
femaleintotheinguinalcanal.Internalspermaticfasciaispresentinthespermaticcord.Ilio
inguinalnervepassesonlythroughthesuperficialinguinalcanalandnotthroughdeep.
Inguinalcanal:containsthespermaticcordandtheilioinguinalnerveinthemale,andthe
roundligamentoftheuterusandtheilioinguinalnerveinthefemale.Itisanobliquecanalof
about4cm.long,slantingdownwardandmedialward,andplacedparallelwithandalittle
abovetheinguinalligament;itextendsfromthedeepinguinalringlaterallytothesuperficial
inguinalringmedially.
Deepinguinalring:aholeintransversalisfascialying3cmsuperiortothemidpointofthe
inguinalligament.
Contents:
Spermaticcordinmales
Roundligamentinfemales
Superficialinguinalring:V-shapeddefectinthelowermedialfibresoftheexternal
obliquejustsuperiorandlateraltothepubictubercle.Ilioinguinalnervepassesthrough
superficialinguinalring.
Contentsofinguinalcanal
Spermaticcordinmales
Roundligamentinfemales
Ilioinguinalnerveinbothmalesandfemales

91.Superficialinguinalringisadefectinthe:
a)Internalobliqueaponeurosis
b)Externalobliqueaponeurosis
c)Transverseabdominisaponeurosis
d)Internalobliquemuscle
CorrectAnswer-B
Theinguinalcanalisanobliquepassagethroughthelowerpartof
theanteriorabdominalwall.
Thecanalisabout1.5in.(4cm)longintheadultandextendsfrom
thedeepinguinalring,aholeinthefasciatransversalis,downward
andmediallytothesuperficialinguinalring,aholeinthe
aponeurosisoftheexternalobliquemuscle.Inthemales,itallows
structurestopasstoandfromthetestistotheabdomen.
Infemalesitallowstheroundligamentoftheuterustopassfromthe
uterustothelabiummajus.

92.AllarecomponentsofSpermaticcord
except:
a)Poupart'sligament
b)Genito-femoralnerve
c)Vasdeferens
d)Pampiniformplexus
CorrectAnswer-A
Ai.e.Poupart'sligament
Poupart'sligamentisalsocalledasinguinalligament.Itformsthe
baseofinguinalcanal.
Constituentsofspermaticcordare:
Ductusdeferens
Testicularandcremastericarteries,andthearteryoftheductus
deferens
Pampiniformplexusofveins
Lymphvesselsfromthetestis
Genitalbranchofthegenitofemoralnerveandthesympatheticnerve
plexusaroundthearterytotheductusdeferensandvisceralafferent
nervefibres.
Remainsoftheprocessusvaginalis.

93.Superficialperinealspacecontains?
a)Sphincterurethraemuscle
b)Ischiocavernosusmuscle
c)Deeptransverseperineimuscle
d)Bulbourethralgland
CorrectAnswer-B
Ans.is'b'i.e.,Ischiocavernosusmuscle

94.

3'dpartofduodenumisnotrelated-
a)Superiormesentericvessels
b)Rightureter
c)Headofpancreas
d)Quadratelobeofliver
CorrectAnswer-D
Ans.is'd'i.e.,Quadratelobeofliver[RefBDC6h/eVoL2
p.2sg_262]
Anterior:-
Superiormesentericvessel
Rootofmesentery
Posterior
Rightureter
Rightpsoasmajor
Righttesticularorovarianvessels
IVC
Abdominalaortawithoriginofinferiormesentericartery
Superior
Headofpancreaswithuncinateprocess.
Inferior
Coilsofjejunum

95.Gastrosplenicligamentcontains?
a)Splenicvessels
b)Tailofpancreas
c)Shortgastricartery
d)Portalvein
CorrectAnswer-C
Shortgastricartery

96.Whichofthefollowingisaderivativeof
ventralmesogastrium?
a)Greateromentum
b)Gastrosplenicligament
c)Linorenalligament
d)Lesseromentum
CorrectAnswer-D
Ans.is'd'i.e.,Lesseromentum

97.Correctsequenceofuterinebloodflow-
a)UterineA-ArcuateARadialASpiralA
b)UterineARadialA-ArcuateASpiralA
c)UterineASpiralARadialAArcuateA
d)UterineAArcuateASpiralA-RadialA
CorrectAnswer-A
Answer-is'a'i.e.,UterineA-ArcuateARadialASpiralA[Ref:
TextbookofOBGp.3061
justPriortocontactwithuterus,theuterinearterygives
descendinguterinearteryOaginalartery)whichsupply
theisthmus,cervixanduppervagina.
Afterjoiningtheuterus,theuterinearteryascendsalongthe
lateralmarginofuterusasascendinguterineartery.
Asendinguterinearterygivesseveralbranchesbetweenmiddle
andouterthirdsofmyometrium.Thesearecalledarcuate
arteriesbecauseoftheirsemicircularcourse.
Arcuatearteriesgiveradialarterieswhichinturnterminateas
spiralarteriole.

98.Nervesupplyofstapediusis:
a)2ndnerve
b)3rdnerve
c)5thnerve
d)7thnerve
CorrectAnswer-D
D.i.e.7thnerve

99.Stapediuspullsstapesinwhichdirection-
a)Anterior
b)Superior
c)Inferior
d)Posterior
CorrectAnswer-D
Ans.is'd'i.e.,Posterior[Ref:Textbookofauditory
processingp.361
Contractionofthestapediusmusclepullsthestapesheadin
posteriordirection.
Contractionoftensortympanipullsthemalleusandtympanic
membraneinwardwithalateraltomedialmotion.
Contractionofthesemusclesstiffenthetympanicmembrane
andossicularchaintherebyreducingthemobilityofthemiddle
earanddecreasingthesensitIVItyofeartoair-conductedsound.

100.Sternocleidomastoidmuscleisexamined
by
a)Turningtheheadtowardsthesameside
b)Turningtheheadtowardsoppositeside
c)Shruggingofshoulder
d)Overheadabduction
CorrectAnswer-B
Ans.is'b'i.e.,Turningtheheadtowardsoppositeside[Rel
BDC6h/eVoI.3p.89]
Contractionofonesideofsternocleidomastoidcauses:

1. Tilting(bending)ofheadonsameside
2. Turning(rotation)offaceandheadonoppositeside,whichcan
beappreciatedasturningofchintooppositeside

101.Occulomotornervesuppliesall
extraocularmusclesexcept?
a)Superiorrectus
b)Inferiorrectus
c)Lateralrectus
d)Medialrectus
CorrectAnswer-C
Ans.is'c'i.e.,Lateralrectus

102.Majorcentralnucleusofsympathetic
systemis
a)Nucleusambiguus
b)Nucleustractussolitarius
c)Edinger-Westphalnucleus
d)Hypothalamus
CorrectAnswer-D
Ans.is'd'i.e.,Hypothalamus
Thehighestseatofregulationofautonomicnervoussystem(both
sympathetic&parasympathetic)ishypothalamus.

103.Whatisattachedtothesuperiornuchal
line?
a)Trapezius
b)Scalenusanticus
c)Coracobrachialis
d)BicepsBrachii
CorrectAnswer-A
Ans.is'a'i.e.,Trapezius
[Ref:Snell'sth/ep.3a2l
Musclesattachedtosuperiornuchalline
Occipitalismuscle,
Thespleniuscapitismuscle
Trapeziusmuscle,
Sternocleidomastoidmuscle

104.Dangerousareaofscalpis-
a)Superficialfacia
b)Aponeurosis
c)Subaponeurotictissue
d)Pericranium
CorrectAnswer-C
Ans.is'c'i.e.,Subaponeurotictissue
Loosesubaponeuroticareolartissue(4slayer)iscalled
dangerousareaofscalpbecauseitcontainsemissaryveins
throughwhichinfectioninsubaponeuroticspacemayspread
readilytointracranialvenoussinuses

105.Whichnerveispreservedindissecting
thesuperficialanddeeplobesofparotid
gland-

a)Glossopharyngeal
b)Hypoglossal
c)Lingual
d)Facial
CorrectAnswer-D
Ans.is'd'i.e.,Facial[Ref-AnatomyrecallbyjaredAntevil]
Thefacialnerveseparatesparotidglandintosuperficialand
deeplobes.
Mandibularbranchoffacialnerveismostvulnerabletoinjury
duringparotidsurgery

106.Whichintrinsicocularmuscleissupplied
byparasympatheticinnervation-
a)Superiorrectus
b)Superioroblique
c)Constrictorpupillae
d)Dilatorpupillae
CorrectAnswer-C
Ans.is'c'i.e.,Constrictorpupillae[Ref:Gray's39h/ep.7101
Intraocularmuscles(intraocularmuscles)
Intraocularmusclesare-
A)Musclesofiris
Therearetwotypesofmusclesiniristhatcontrolthesize
ofpupil:
1. Theirissphinctororconstrictorpupillae(circularmuscles):-
Thesemusclesareinnervatedbythepostganglionic
parasympatheticfibresfromEdingerwestphalnucleusvia3'd
nerveandciliaryganglion.Thesemusclescauseconstrictionof
pupil(Miosis).
2. Theirisdilatorordilatorpupillae(railialmuscles)These
musclesareinnervatedbysympatheticsystem
viapostganglionicsympatheticfibresforthedilatorpupillaefrom
neuronsinthesuperiorcervicalganglion.Thesemusclescause
pupillarydilatation(mydriasis).
B)Ciliarymuscles;-->theseareinnervatedbythepostganglionic
parasympatheticfibersfromEWNvia3rdnerveandciliary
ganglion.Thesemuscleshelpinaccommodation.

107.Lipsdoesnotdrainintowhichgroupof
lymphnodes?
a)Submandibularnodes
b)Sublingualnodes
c)Preauricularparotid
d)Noneoftheabove
CorrectAnswer-C
Ans.is'c'i.e.,Preauricularparotid[Ref:BDCfi/eVol.3p.
72-751
Thefacepossessesthreeareasfromwhichlymphatic
drainageisasfollows:-
1. Upperarea,comparisongreaterpartofforehead,lateralYzof
eyelids,conjunctiva,lateralpartofcheekandparotidarea,
drainsintopreauricular(superficial)parotidnodes.
2. Middlearea,comprisingcentralpartofforehead,externalnose,
upperlip,lateralpartoflowerlip,medialhalvesofeyelids,
medialpartofcheek,andgreaterpartoflowerjaw,drainsinto
submandibularnodes.
3. Lowerarea,includingcentralpartoflowerlipandthechin,
drainsintosubmentalnodes.

108.Postganglionicfibrestoparotidglandissuppliedby?
a)Glossopharyngealnerve
b)Auriculotemporalnerve
c)Bothoftheabove
d)Noneoftheabove
CorrectAnswer-B
Preganglionicnervestravelinthelesserpetrosalbranchoftheglossopharyngealnerveand
synapseintheoticganglion.Postganglionicfibersreachtheglandviatheauriculotemporal
nerve.
Nervesupplyofparotidgland
Innervationofthesalivaryglandisasfollows:-
Parasympathetic(secretomotor):
Theyreachtheglandthroughauriculotemporalnerveasthe
followingroute:-
Preganglionicfibers-Originateintheinferiorsalivarynucleus;pass
throughglossopharyngealnerve;itstympanicbranch;tympanic
plexus,andlesserpetrosalnerve.
Relayganglion:-Oticganglion.
Postganglionicfibers:-Passthroughtheauriculotemporalnerveto
reachthegland.
Sympathetic(Vasomotor):
Derivedfromtheplexusaroundthemiddlemeningealartery.
Sensory:derivedfromtheauriculotemporalnerve,exceptforparotid
fasciaandoverlyingskinwhichareinnervatedbythegreatauricular
nerve(c2,c3)

109.Secretomotorfiberstoparotidglandsare
through-
a)Tympanicplexus
b)Geniculateganglion
c)Greaterpetrosalnerve
d)None
CorrectAnswer-A
Ans.is'Ai.e.,Tympanicplexus(Ref:BDC@/eVol.3p.111).

110.Featuresoffacialnervepalsyareall
except-
a)Lossofsalivation
b)Lossoflacrimation
c)Facialmuscleparalysis
d)Lossoftastesensationfromposteriortongue
CorrectAnswer-D
Ans.is'd'i.e.,Lossoftastesensationfromposterior
tongue[Ref:Dhingra5'h/ep.105,106;Gray's39h/ep.12131
Facialnerveparalysisproducesfollowingmanifestations:
Weaknessofthemusclesoffacialexpressionandeve
closure.whichresultsin:
1. Absenceofnasolabialfold.
2. Widepalpebralfissure
3. Epiphora
4. Droopingofangleofmouth
5. Lossofwrinklesofforehead
6. Thefacesagsandisdrawnacrosstotheoppositesideon
smiling.
7. Voluntaryeyeclosuremaynotbepossibleandcanproduce
damagetotheconjunctivaandcornea.
Inpartialparalysis,thelowerfaceisgenerallymoreaffected.
Lossoftastesensationoveranterior2/3ofthetongue.
Intolerancetohigh-pitchedorloudnoises,i.e.Hyperacusis(due
tostapediusmuscleparalysis)
Lossoflacrimationandsalivationoccur

111.Thenervewhichhasthelongest
intracranialcourseis:
a)Fourthcranialnerve
b)Thirdcranialnerve
c)Sixthcranialnerve
d)Fifthcranialnerve
CorrectAnswer-A
Ans.Fourthcranialnerve

112.Largestcranialnerveis:
a)Trochlear
b)Trigeminal
c)Oculomotor
d)Vagus
CorrectAnswer-B
Trigeminal
Longestintracranialcourse
Trochlearnerve
Longestcourseoverallandmost
Vagus
widelydistributed
Smallest(thinnest)cranialnerve
Trochlearnerve
Largest(thickest)cranialnerve
Trigeminalnerve
Theonlycranialnervearisingfrom Trochlearnerve
dorsalaspect
Onlycranialnervedecussating
Trochlearnerve
completelybeforeemerging
Cranialnervemostcommonly
Facialnerve
involvedinbasalskullfracture
Cranialnervemostcommonlv
Abducentnerve
involvedinraisedintracranial
tension
Commonestcranialnerveaffected
Abducentnerve
inspinalanaesthesia
Cranialnervemostcommonly
Oculomotornerve
involvedinintracranialaneurysm
Cranialnervescarrying
3,7,9,10

paraslrrnpatheticfibers

113.Nasopharynxislinedbywhich
epithelium?
a)Stratifiedsquamousnonkerationized
b)Stratifiedsquamouskeratinized
c)Ciliatedcolumnar
d)Cuboidal
CorrectAnswer-C
Ans.is'c'i.e.,Ciliatedcolumnar
Thetwomaintypesofepithelialiningthenasopharynxarestratified
squamous(comprisingapproximately60%
ofnasopharyngealepithelium)andpseudostratifiedcolumnar
respiratoryepitheliumcontainingciliatedcells,gobletcellsand
basalcells.

114.Greaterpetrosalnerveisformedfrom?
a)Geniculateganglion
b)PlexusaroundICA
c)Plexusaroundmiddlemeningealartery
d)Noneoftheabove
CorrectAnswer-A
Ans.is'a'i.e.,Geniculateganglion
Petrosalnerves
Greaterpetrosalnerve
Firstbranchoffacialnerve
Arisesfromgeniculateganglion
Inforamenlacerumitjoinsdeeppetorsalnerveandformsthenerve
topterygoidcanal
Supplieslacrimalglands,nose,mucosalglandsofpharynx,palate.
Lesserpetrosalnerve
Branchof9thcranialnervethroughtympanicplexus(passesviaotic
ganglion)
Itsupliesparotidgland.
Deeppetrosalnerve
Itisabranchofsympatheticplexusaroundinternalcarotidartery.
Itcontainscervicalsympatheticfibers.
Externalpetrosalnerve
Itisaninconsistaentbranchofsympatheticplexusaroundmiddle
meningealartery.

115.Nerve3tdventricleisthecavityof-
a)Mesencephalon
b)Rhombencephalon
c)Diencephalon
d)Telencephalon
CorrectAnswer-C
Ans.is'c'i.e.,Diencephalon(Ref:BDCVol.-36h/ep.324,
441)

PartofBrain
Cavity
A)-forebrain(prosencephalon)
Lateralventricle
i.Telencephalon(cerebrum)
Thirdventricle
ii.Diencephalon(Thalamencephalon)
B)-midbrain(mesencephalon)
Cerebralaqueduct
C)-Hindbrain(rhombencephalon)
FourthVentricle

116.Occulomotornucleusislocatedin-
a)Forebrain
b)Midbrain
c)Pons
d)Medulla
CorrectAnswer-B
Ans.is'b'i.e.,MidbrainlRef:BDC6/eVol.3p'3501
CranialnervesI,IIForebrain
CranialnervesIII,IVMidbrain
CranialnervesV,VI,VII,VUIPons
CranialnervesIX,X,Xl,XllMedulla

117.Whichofthefollowingispuresensory
nerve?
a)Trigeminal
b)Abducent
c)Trochlear
d)Olfactory
CorrectAnswer-D
Ans.is'd'i.e.,Olfactory
Cranialnerves
Pure
Puremotor
Mixed
sensory
Oculomotor(III)
Trochlear(IV)
Olfactory Abducent(VI)
Trigeminal(V)
(I)
Vestibulocochlear Facial(VII)Glossopharyngeal(IX)
Optic(II)
(VIII)
Vagus(X)
Accessory(XI)
Hypoglossal(XII)

118.OlfactoryGreatcerebralveinofGalen
drainsinto-
a)Cavernoussinus
b)Basalvein
c)Internalcerebralvein
d)Straightsinus
CorrectAnswer-D
Ans.is'd'i.e.,Straightsinus
GreatcerebralveinofGalenisformedbytheunionoftwointernal
cerebralveins.
Itis2cmlong.
Itisdrainsintothestraightsinus.

119.Superficialmiddlecerebralveindrains
into-
a)Internalcerebralvein
b)Cavernoussinus
c)GreatcerebralveinofGalen
d)Straightsinus
CorrectAnswer-B
Ans.is'b'i.e.,Cavernoussinus[RefTextbookofneuroanatomy
p.189]
Superficialmiddlecerebralvein
Itrunsalongtheposteriorramusandstemoflateralsulcus.
Itdrainsbloodfromsuperolateralsurfaceofthecerebralhemisphere
intothecavernoussinus.
Itcommunicateswiththesuperiorsagittalandtransversesinus
throughveinofTrolardandveinofLabberespectively.

120.Majorsupplyofmedialsurfaceof
cerebralhemisphere
a)Anteriorcerebralartery
b)Posteriorcerebralartery
c)Middlecerebralartery
d)Posteriorinferiorcerebellarartery
CorrectAnswer-A
Ans.is'a'i.e.,Anteriorcerebralartery[Ref:BDC6th/eVol.3p.
461,462]
Cerebralcortexissuppliedbybranchesofallthreecerebralarteries.
Allthethreesurfacereceivebranchesfromallthreearteries.
Buteachsurfaceissuppliedpredominantlybyoneartery:?
1. Middlecerebralarteryisthemainarteryonsuperolateralsurface.
2. Anteriorcerebralarteryischiefarteryonmedialsurface.
3. Posteriorcerebralarteryisprincipalarteryoninferiorsurface

121.Whichnucleusisnotseeninfloorofthe
4thventricle-
a)Abducensnucleus
b)Facialnucleus
c)Dorsalvagalnucleus
d)Hypoglossalnucleus
CorrectAnswer-B
Ans.is'b'i.e.,Facialnucleus{RefBDCVoI.3&/ep.410;Last's
l2n/ep.482,483}
Medialeminenceispresentineachsideofmediansulcus.It
presentsfacialcolliculusformedbygenu(recurvingfibers)of
facialnerveloopingaroundabducensnucleus.Facialcolliculus
liesinpons(i.e.inpontinepartoffloor).
Hypoglossaltriangleoverlyingthehypoglossalnucleusandvagal
triangleoverlyingdorsalnucleusofvagus.Bothofthesetriangle
liesinthemedulla(medullarypartoffloor)

122.Ventraltegmentaldecussationin
cerebralpeduncleisdueto-
a)Tectospinaltract
b)Tectobulbartract
c)Vestibulospinaltract
d)Rubrospinaltract
CorrectAnswer-D
Ans.isd'ie.,Rubrospinaltract[Ref:BDC6th/eVol.3p.398]
VentraltegmentaldecussationFormedbydecussationof
rubrospinaltract.
DorsaltegmentaldecussationFormedbydecussationof
tectospinalandtectobulbartracts.

123.Whichofthefollowingstructuresseenin
thecavernoussinus?
a)MaxillarydivisionofVnerve
b)MandibulardivisionofVnerve
c)Internalcarotidartery
d)Facialnerve
CorrectAnswer-C
Ans.c.Internalcarotidartery
Contentsofthecavernoussinus
Structuresinthelateralwallofthesinus
Oculomotor(lll)nerve
Trochlear(lV)nerve
Ophthalmic(1stdivisionofV)nerve
Trigeminalganglion
Internalcarotidartery
Abducent(Vl)nerve

124.Trueaboutfallopiantubesareallexcept
:
a)Linedbycuboidalepithelium
b)Isthmusisthenarrowerpartofthetubethatlinkstotheuterus
c)Tubalostiumisthepointwherethetubalcanalmeetsthe
peritonealcavity
d)MUllerianductsdevelopsinfemalesintotheFallopiantubes
CorrectAnswer-A
Thetwouterinetubesareeachabout4in.(10cm)longandliein
theupperborderofthebroadligament.
Eachconnectstheperitonealcavityintheregionoftheovarywith
thecavityoftheuterus.
Theuterinetubeisdividedintofourparts:
Theinfundibulumisthefunnel-shapedlateralendthatprojects
beyondthebroadligamentandoverliestheovary.
Thetubalostiumisthepointwherethetubalcanalmeetsthe
peritonealcavity.
Theampullaisthewidestpartofthetube.
Theisthmusisthenarrowestpartofthetubeandliesjustlateralto
theuterus.
Theintramuralpartisthesegmentthatpiercestheuterinewall.
Function
Theuterinetubereceivestheovumfromtheovaryandprovidesa
sitewherefertilizationoftheovumcantakeplace(usuallyinthe
ampulla).
Theinnermucousmembraneoftheuterinetubeislinedbythe
ciliatedcolumnarepitheliummixedwiththenonciliatedsecretory
cellsorpegcells
TheMullerianductsdevelopsinfemalesintothefallopiantubes,

TheMullerianductsdevelopsinfemalesintothefallopiantubes,
uterusandvagina,whiletheWolffianductsdevelopsinmalesinto
theepididymisandvasdeferens

125.Liningepitheliumofvaginais
a)Squamousepithelium
b)Columnarepithelium
c)Transitionalepithelium
d)Secretoryepithelium
CorrectAnswer-A
Ans:ASquamousepithelium
(Ref:RameshBabup.24]
Vaginalmucousmembraneislinedbynonkeratinizedstratified
squamousepithelium.

126.Oxynticcellsarepresentin-
a)Pylorus
b)Cardiacnotch
c)Body
d)None
CorrectAnswer-C
Ans.is'c'i.e.,Body
Oxyntic.cellsarepresentinprincipalglandsofbodyandfundus.
Theseglandsarefoundinbodyandfundus.Theseglands
containmainlychief(pepticorzymogen)cellsandparietal
(oxyntic)cells.Theseglandsalsocontainmucousneckcells,
stemcellsandenteroendocrinecells(argentaffincells).


127.Firstbonetostartossifying-
a)Femur
b)Tibia
c)Scapula
d)Mandible
CorrectAnswer-D
Ans.is'd'i.e.,Mandible[Ref:T*tbookofanatomywithcolor
Atlasp.I32l
'Themandibleisoneofthefirstbonesinthebodytostart
ossifyingbeingnextinthisrespectonlytotheclavicle..
Thus,FirstbonetostartossifyingClavicle.Secondboneto
startossifyingMandible.

128.Incudomalleolarjointisa?
a)Ellipsoidjoint
b)Pivotjoint
c)Hingejoint
d)Saddlejoint
CorrectAnswer-D
Synovialjointsareclassifiedasfollows:
1)Planesynovialjoints(flatjoints)
2)Hingejoints(Ginglymusjointsorginglymi)
3)Pivotortrochoidjoints
4)Condylar(bicondylar)joints
5)Ellipsoidjoints
Note:Metacarpophalangealjointsareellipsoidjointsfunctionally,
butstructurallytheybelongtocondylarjoints.
6)Saddle(sellar)joints
Thearticularsurfacesarereciprocallysaddleshaped,i.e.concavo-
convex.Examplesarefirst(thumb)carpometacarpaljoint,
sternoclavicularjoint,calcaneocuboidjoint,incudomalleolarjoint
(smallestsaddlejoint)andpatellofemoraljoint.
7)
Ballandsocket(spheroidal)joints

129.Hilton'slawisrelatedto-
a)Venousdrainage
b)Bloodsupply
c)Nerveinnervation
d)Alloftheabove
CorrectAnswer-C
Ans.is'c'i.e.,Nerveinnervation[RelTextbookoperative
orthopaedicsp.7861
Hilton,slawstatesthatnervescrossingajointsuppliesthat
jointbygivingbranchingtothatjoint

130.Spermatogenesisbeginsat-
a)Birth
b)5years
c)Puberty
d)18years
CorrectAnswer-C
Ans.is'c'i.e.,Puberty[Ref:Clinicalembryology3'd/ep'311]
Spermatogenesisreferstotheprocessofformationof
spermatozoa(sperm)fromprimititegermcells(spetmatogonia)'
spermatogenesisbeginsatpubertyandcontinuesthroughout
adultlifetodeclineinoldage.

131.Whichisnotastageofprophase-
a)Diakinesis
b)Leptotene
c)Zygotene
d)Arachytene
CorrectAnswer-D
Ans.is'd'i.e.,ArachytenelRef:Essentialsofmedical
geneticsbyA'K'Dutta4'h/ep'111


132.Fossaovalisisaremnantof-
a)Septumprimum
b)Septumsecundum
c)Septumspurium
d)AVcushion
CorrectAnswer-A
Ans.is'a'i.e.,SeptumprimumlRef:Readbeloul
Actually'floorofthefossaovalisrepresentsseptumprimum.
Fullfossaovalisisaremnantofforamenovale

133.Nephronisderivedfrom?
a)Uretericbud
b)Mesonephricduct
c)Metanephros
d)Mesonephros
CorrectAnswer-C
Developmentofkidney
Uretericbud(mesonephros)arisefrommesonephricductandgives
risetocollectingsystemofkidney(renalpelvis,majorandminor
calyces,collectingtubule)andureter.
Metanephricmesoderm(blastemaormetanephors)arisefrom
nephrogeniccordwhichinturnisderivedfromintermediate
mesoderm.Itgivesrisetoexcretoryunit(nephron),i.e.glomeruli,
PCT,LoopofhenleandDCT.

134.Fibrousstromaofliverisderivedfrom-
a)Foregutendoderm
b)Midgutendoderm
c)Hindgutendoderm
d)Septumtransversum
CorrectAnswer-D
Ans.is'di.e.,SeptumtransversumDevelopmentOfliver
Liverisdevelopedfrom-i)Endodermofforegut(endodermal
diverticulum):Mostoftheliver(includinghepatocytes)is
derivedfromforegutendoderm.ii)Septnmtransversum
mesenchyme(mesoderm):Itgivesrisetoligamentsoftheliver
(exceptligamentumteres),kupffercells,hematopoieticcells,
sinusoidsandfibrous-areolarstromaofliver'
Ligamentumteresisderivedfromleftumbilicalvein.

135.TrueaboutBranchialcystis:
March2013(h)
a)Cystsaremorecommonthansinuses
b)Mostlyarisesfrom2ndbranchialsystem
c)Causesdysphagiaandhoarseness
d)Sinusshouldalwaysbeoperated
CorrectAnswer-B
Ans.Bi.e.Mostlyarisesfrom2ndbranchialsystem

136.Trueaboutnotochordareallexcept?
a)Definesaxisofembryo
b)Servesasprimaryinductor
c)Derivedfromhypoblast
d)Remainsasnucleuspulposus
CorrectAnswer-C
Notochordisabudlikestructureformedbyepiblastcellsextending
fromcranialendofprimitivestreaktocaudalendofprochordalplate,
inbetweentheectodermandendoderm.Siginificancesofnotochord
includesfollowing:-
i. Itdefinestheaxisofembryo.
ii. Itfunctionsastheprimaryinductor,inducingtheoverlyingectoderm
todevelopintoneuralplate(theprimordiumofCNS).
iii. Itservesasthebasisfordevelopmentofaxialskeleton.The
notochordisanintricatestructurearoundwhichvertebralcolumnis
formedandindicatesfuturesiteofvertebralbodies.However,the
notochorddoesnotgiverisetovertebralcolumn,afterdevelopment
ofvertebralbodies,
thenotochorddegeneratesanddisappears,but
partsofitpersistasthenucleuspulposusofintervertebraldisc.

137.Structuresderivedfromtheneuralcrest
are?
a)Pia
b)Dentalpapillae
c)Adrenalmedulla
d)Alloftheabove
CorrectAnswer-D
Di.e.Alloftheabove

138.Hearttubeisformedat-
a)3weeks
b)6weeks
c)10weeks
d)12weeks
CorrectAnswer-A
Ans.is'a'i.e.,3weeks[Ref:EmbryologybyInduKhuranap.
209]
'Tubularheartisformedattheendof3dweek"

139.Hearttubeisformedin
a)Hyaluronicacidsecretedbyendocardium
b)Chondroitinsulfatesecretedbyendocardium
c)Hyaluronicacidsecretedbymyocardium
d)Chondroitinsulfatesecretedbymyocardium
CorrectAnswer-C
Ans-Ans.is'c'i.e.,Hyaluronicacidsecretedbymyocardium
[RefTextbookofHumanEmbryologp.7861
Cardiacjellyisathickextracellularmatrixrichinhyaluronic
acid.Itseparatesendothelialhearttubefrommyocardium.
Cardiacjellyissecretedbymyocardium.

140.Remnantofumbilicalartery
a)Ligamentumarteriosum
b)Ligamentteres
c)Ligamentumvenosum
d)Medialumbilicalligament
CorrectAnswer-D
Ans.is'd'i.e.,Medialumbilicalligament{Ref:Garg2d/ep.
2151}

Embryonicpart
Remnants
Ductusarteriosus
Ligamentumarteriosum
Umbilicalartery
Proximalpart:-Superiorvesical
artery.Distalpart:-Medial
umbilicalligament
Umbilicalvein(left)
Ligamentumteres
Ductusvenosus
Ligamentumvenosus
Septumprimurn
Flooroffossaovalis
Septumseconclum
Annulusovalis/Limbusfossa
ovalis

141.Whichofthefollowingisaderivativeof
paramesonephricductinmales?
a)Trigoneofbladder
b)Paraphoron
c)Prostaticutricle
d)Gartner'sduct
CorrectAnswer-C
Twostructuresarederivedfromparamesonephricductin
males:-

1. Appendixoftestis(hydatidofmorgagni)
2. Prostaticutricle.

142.Myelinationinperipheralnervoussystem
isdoneby
a)Astrocytes
b)Oligodendrocytes
c)Ependymalcells
d)Schwanncells
CorrectAnswer-D
Ans.isdi.e.,Schwanncells
MyelinationincentralnervoussystemOligodendrocytes.
MyelinationinperipheralnervoussystemSchwanncell.


143.Malleusandincusarederivedfrom?
a)1stArch
b)2ndArch
c)3rdArch
d)4thArch
CorrectAnswer-A
Ans.is'a'i.e.,1stArch
lst(mandibulararch):-
MuscularContribution:-Musclesofmastication,Anteriorbellyof
thedigastric,Mylohyoid,Tensortympani,Tensorvelipalatini.
SkeletalContributions:-Maxilla,mandible(onlyasamodelfor
mandible),Incusandmalleus,Meckel'scartilage,Ant.ligamentof
malleus,Sphenomandibularligament.
Nerve:-Trigeminalnerve(V2andV3).
Artery:-Maxillaryartery,externalcarotidartery.

144.Falseaboutlimbusfossaovalis-
a)Situatedabovefossaovalis
b)Inrightatrium
c)Derivedfromseptumprimum
d)AlsocalledAnnulusovalis
CorrectAnswer-C
Ans.is'c'i.e.,Derivedfromseptumprimum
Limbusfossaovalis(alsocalledannulusovalis)isathickenedrimin
rightatrium,presentabovethefossaovalis.
Itrepresentsthelowerfreemarginofseptumsecundum.(not
septumprimum.

145.Sertolicellsarederivedfrom-
a)Genitaltubercle
b)Genitalswelling
c)Primordialgermcells
d)Germinalepithelium
CorrectAnswer-D
Ans.is'd'i.e.,Germinalepithelium[RefI.B.Singhembryology
9thie
p.278]
Testisdevelopsatgenitalridge(urogenitalridge).
'Primordialgermcells'aredevelopedinthe4thweekbyproliferation
ofendodermalcellsofthedorsalwallofhindgut(partofyolksac).
Theprimordialgermcellsmigrateintogenitalridge,where
proliferationofbothgerminalandnongerminalcellsleadsto
formationofgonads.
Genitalridgeiscoveredbygerminalepithelium(previouscoelomic
epithelium),whichproliferatesandformssexcords(primitive
seminiferouscords).Largenumberofsertolicellsarederivedfrom
thesesexcords.
Lyedigcellsarealsoderivedfromsexcord,whichinturnisderived
fromgerminalepithelium.

146.Prolactinsecretingglanddevelopsfrom-
a)Infundibulum
b)Rathke'spouch
c)Tubercinereum
d)3rdventricle
CorrectAnswer-B
Ans.is'b'i.e.,Rathke'spouch
Prolactinissecretedbyanteriorlobeofpituitarywhichdevelops
fromRathke'spouch.

147.Posteriorcardinalveindevelopsinto-
a)Commoniliacvein
b)Superiorvenacava
c)Internaljugularvein
d)Externaljugularvein
CorrectAnswer-A
Answer-a'i.e.,CommoniliacveinfRef:Textbookof
embryologyp.786
Derivativesofposteriorcardinalveins,subcardinalveinsand
supracardinalveinsare:
1.Inferiorvenacavaisderivedfrom:?

1. HepaticsegmentofIVCisderivedfrom:(i)righthepaticcardiac
channel,and(ii)anastomoticchannelbetweensubcardinalveinand
righthepaticcardiacchannel.
2. RenalsegmentofIVCisderivedfromrightsubcardinalvein.This
partreceivesbothrenalandsuprarenalveins.
3. PostrenalsegmentofIVC(majorpartofIVC)isformedby(i)
anastomosisbetweenrightsupracardinalandsubcardinalveins,(ii)
rightsupracardinalvein(lowerpart),and(iii)rightposteriorcardinal
vein(lowestpart).
2.Gonadalveinsdevelopfromsubcardinalveins(distalpart
belowinter-subcardinalorrenalanastomosis).
3.Suprarenalveinsdevelopfromsubcardinalveins(proximal
partaboveinter-subcardinalorrenalanastomosis).
4.Rightcommoniliacveinisderivedfromtherightposterior
cardinalvein(mostcaudalpart).
5.Leftcommoniliacveindevelopsfromtransverse
anastomosisbetweenlowerendofposteriorcardinalveins.


6.Rightrenalveinisamesonephricveinthatdrainsintorenal
segmentofIVC(whichisderivedfromtheupperpartofright
subcardinalvein).
7.Leftrenalveindevelopsfromthreesources:(i)mesonephric
vein(drainintoleftsubcardinalvein),(ii)leftsubcardinalvein
(smallpart),and(ii)pre-aorticintersubcardinalanastomosis


148.Superiorvenacavadevelopsfrom-
a)Rightanteriorcardinalvein
b)Leftanteriorcardinalvein
c)Leftcommoncardinalvein
d)Rightsubcardinalvein
CorrectAnswer-A
Ans.is'a'i.e.,RightanteriorcardinalveinlRef:Textbookof
embryologltp.7861
Superiorvenacava(SVC)isderivedfrom
1. rightanteriorcardinalvein(proximaltobrachiocephalic
anastomosis),and
2. rightcommoncardinalvein.

149.StructuredevelopingfromMullerianduct
inmales?
a)Seminalvesicle
b)Epididymis
c)Prostaticutricle
d)Ureter
CorrectAnswer-C
Ans.(C)Prostaticutricle
RemnantsofMullerianduct(paramesonephricduct)inmales
are:-
Appendixoftestis(HydatidofMorgagni)
Prostaticutricle.

150.Opticcupisderivedfrom?
a)Neuralectoderm
b)Surfaceectoderm
c)Mesoderm
d)Neuralcrest
CorrectAnswer-A
Ans.is'a'i.e.,Neuralectoderm

151.Whichofthefollowingisaderivativeof
Rathke'spouch-
a)Parstuberalis
b)Neurohypophysis
c)Posteriorpituitary
d)Pinealgland
CorrectAnswer-A
Ans.is'a'i.e.,ParstuberalislRef:Textbookofembryology
p.7861
TheRathke'spouchisanectodermalupgrowthfromthe
stomodaeuminfrontofbuccopharyngealmembrane
ThederivativesofRathke'spouchgiverisetothevarious
componentsoftheanteriorpituitary:

1. Anteriorlobe
2. Parstuberalis
3. Parsdistalis
4. Parsintermedia

152.Whichofglialcellismesodermalin
origin-
a)Macroglialcells
b)Microglialcells
c)Oligodendrocytes
d)Ependymalcells
CorrectAnswer-B
Ans.is'b'i.e.,Microglialcells
Microglialcellsarederivedfrommesenchymal(mesodermal)
cells

153.Trigoneofbladderisderivedfrom?
a)Mesonephricduct
b)Paramesonephricduct
c)Absorbedanalmembrane
d)Mullerianduct
CorrectAnswer-A
Mesonephricduct

154.Pisiformarticulateswith-
a)Scaphoid
b)Trapezium
c)Triquetral
d)Lunate
CorrectAnswer-C
Triquetral

155.Spleniculiareseenmostcommonlyin:
a)Colon
b)Hilum
c)Liver
d)Lungs
CorrectAnswer-B
Spleniculioraccessoryspleensaremostcommonly(about80%)seenatthehilumof
thespleen.
Alsoknow:
Locationofaccessoryspleenindecreasingorderare:

Splenichilum
Gastrocolicligament
Tailofpancreas
Greateromentum
Greatercurvatureofstomach
Splenocolicligament
Smallandlargebowelmesentery
Leftbroadligamentofwomen
Leftspermaticcordinmen
Ref:Schwartz9/e,Page1246;Bailey&Love25/e,Page25/e,Page1103

156.StructurespassingthroughCalot's
triangleareallEXCEPT:
a)Portalvein
b)Cysticartery
c)Righthepaticartery
d)LymphnodeofLund
CorrectAnswer-A
Portalvein
THEHEPATOBILIARYTRIANGLEORCYSTOHEPATIC
TRIANGLEORCALOT'STRIANGLE:
Boundaries:
Commonhepaticductmedially
Cysticductinferiorly
Inferiorsurfaceofliversuperiorly
Contents:
Cysticartery
Righthepaticartery
LymphnodeofLund

157.Whatstructurepassesthroughthe
quandrangularspace?
a)Axillarynerve
b)Radialnerve
c)Mediannerve
d)BrachialArtery
CorrectAnswer-A
Ans.is'a'i.e.,AxillarynervelRefCampbell's12h/ep.22131
Axillarynerveandposteriorcircumflexhumeralvesselsare
transmittedthroughquadrangularspace

158.Seminalcolliculusispresentin?
a)Testis
b)Prostate
c)Urethra
d)Scrotum
CorrectAnswer-C
Ans.is'c'i.e.,Urethra
Seminalcolliculusorcolliculusseminalisorverumontanumis
presentintheprostaticurethra.
Featuresoftheprostaticurethra
Urethralcrest-medianlongitudinalmucousfold.
Colliculusseminalis(verumontanum):Anelevationinthemiddleof
theurethralcrestwiththeopeningofaprostaticutricleatitssummit
andanejaculatoryductoneachside.
Openingsofejaculatoryducts:Thesearepresentoneachsideof
theorificeoftheutricle.
Prostaticsinuses:Theseareverticalgroovespresentoneachside
oftheurethralcrest.Theyarepresentwithopeningsofprostatic
glands.

159.Whichofthefollowingisnotaderivative
offoregut?
a)Cecum
b)duodenum
c)Liver
d)Pancreas
CorrectAnswer-A
Ans.is'a'i.e.,Cecum[RefLangman11th/ep.223]:-
Derivativesare:-

Pharynx
Lowerrespiratorytract
Esophagus
Stomach
DuodenumuptotheopeningofthemainpancreaticductLiver
Pancreas

160.Mostimportantfactorintransportacross
amembrane?
a)Chargeofparticle
b)Membranethickness
c)Sizeofparticle
d)Concentrationgradient
CorrectAnswer-D
Ans.D.Concentrationgradient
[RefGuyton12h/ep.48-501]
Transportacrossacellmembraneisdividedintopassiveoractive,
basedonwhetheritisalongtheconcentrationgradientoragainst
theconcentrationgradient.
1)Passivetransport
Itisinthedirectionofconcentrationgradienti.e.,"Downhill
movement".
2)Activetransport
Itisagainstthedirectionofconcentrationgradient,i.e.,"Uphill
movement".

161.Tronsportprocesswhichisagainst
concentrationgradientandcarrier
mediatedis?

a)Facilitateddiffusion
b)Osmosis
c)Activetransport
d)Endocytosis
CorrectAnswer-C
Ans.C.Activetransport
Activetransport:
Itisagainstthedirectionofconcentrationgradient,i.e.,*Uphill
6ovenento.
Energy(e.g.,ATP)isrequired
Carrierproteinisrequired
Examplesareprimaryactivetransportandsecondaryactive
transport.

162.Similaritybetweenactivetransportand
facilitateddiffusion?
a)Energyrequirment
b)Againstconcentrationgradient
c)Carrierprotein
d)Alloftheabove
CorrectAnswer-C
Ans.C.Carrierprotein
1)Passivetransport
Directionofconcentrationgradienti.e.,'Downhillmovement"
Doesnotrequireenergyexpenditure
Exampleare:-
1. Withoutcarriers:-Simplediffirsion,osmosis
2. Throughcarrier:-Facilitateddiftrsion
2)Activetransport
Itisagainstthedirectionofconcentrationgradient,i.e.,*Uphill
6ovenento.
Energy(e.g.,ATP)isrequired
Carrierproteinrequired
Examplesareprimaryactivetransportandsecondaryactive
transport.

163.Transportthroughporesincellmembers
is?
a)Activetranspert
b)Transcytosis
c)Diffusion
d)Endocytosis
CorrectAnswer-C
Ans.C.Diffusion
[Ref:Guytonp.49]
Substancescanmovebysimplediffusiondirectlyalongthepores
andchannelsfromonesideofthemembranetotheother.
Thus,simplediffusioncanoccurbytwopathways.
1. Throughmembranelipidbilayerforlipidsolublesubstances.
2. Throughpores(waterchannel)-+forions/electrolytes.

164. Percentageoftotalbodywatertobody
weightatbirth?
a)90%
b)80%
c)60%
d)50%
CorrectAnswer-B
Ans.B.80%
[RefPrinciplesofmedicalphysiologyp.712]
Themajorcomponentofbodymassiswater.
Thecontributionofwatertobodyweightvarieswithage.
Totalbodywater(TBW)asapercentageofbodyweightdeclines
formashighas90%inearlyfetallifeto75-80%atbirth.
Thereafteritdelinesprogressivelyto60%bytheendofoneyear.
Afterthatitremainsconstant.

165.Epithelialsodiumchannelshas?
a)2a,213
b)la,i(3
c)2a,213,2y
d)2a,113,2y
CorrectAnswer-D
Ans.D.2a,113,2y
[Ref:Principlesofmedicalphysiologyp.241]
TherearetwodifferenttypesofSodiumchannels:-
Voltage-gatedsodiumchannels:
ThesearepresentinExcitablecells.
Theseexistasheterodimerorheterotrimersofalphaandbeta
subunits,containingonealphaandoneortwobetasubunits.
Epithelial-sodiumchannels:
Thesearepresentinepitheliumofabsorptiveorsecretory
epithelium,e.g.colon,sweatglandduct,pancreaticduct,respiratory
passage,anddistaltubuleofkidney.
Theseexistasheterotetramersofalpha,betaandgammasubunits,
containingmostlytwoalpha,onebetaandonegammasubunits.

166.TrueaboutENaCareallexcept?
a)Epithelialchannel
b)Composedof2homoloussubunits
c)PresentinkidneyandGIT
d)Inhibitedbyamiloride
CorrectAnswer-B
Ans.B.Composedof2homoloussubunits
EpithelialNa+channels(ENaC)isratelimitingstepinNa+
reabsorptionacrossseveralepitheliallinings.
Itispresentinkidney(distalpart),GIT(colon),salivary&sweat
glands,andrespiratorytract.
ENaCismadeofthreehomologoussubunits>alpha,beta,and
gamma.
Inlatedistaltubulesandcollectingducts,ENaCisinhibitedby
potassiumsparingdiureticslikeamilorideandtriameterene.

167.pHofintracellularfluidis?
a)SlightlylessthanECF
b)SlightlymorethanECF
c)SameasECF
d)Highlyalkaline
CorrectAnswer-A
Ans.A.SlightlylessthanECF
[Ref:Principlesofmedicalphysiologyp.132]
Extracellularfluid(ECF)haspH7.4(range7.35-7.45)andis
slightlyalkaline
Intracellularfluid(ICF)hasH*ionconcentrationslightlyhigherthan
ECF.Thus,pHofICFisslightlylowthanECF,butstillitisslightly
alkaline.

168.Increasedinplasmaviscosityis
maximallycausedbywhichplasma
protein?

a)Fibrinogen
b)Albumin
c)Globulin
d)Allhaveequaleffect
CorrectAnswer-C
Ans.C.Globulin
[Ref:Clinicalaspectsofbloodp.80]
Theeffectofaproteinonplasmaviscositydependsonitsmolecular
weightandstructure.
Theviscosityishigherifaproteinhas
1. Lessspheroidshape
2. Highermolecularweight
3. Higheraggregationcapacity
4. HighertemperatureorpHsensitivity
Theglobulincausesmaximumincreaseinplasmaviscosityfollowed
byfibrinogen(2dafterglobnlin)andalbumin(3rdinnumber).

169.Oncoticpressureiscontributedby?
a)Sodium
b)Chloride
c)Chloride
d)Albumin
CorrectAnswer-D
Ans.D.Albumin
[Ref:Clinicalphysiology3'd/ep.336]
Oncoticpressure,alsocalledcolloidosmoticpressure,isaformof
osmoticpressureexertedbyproteinsinbloodplasmathatusually
tendstopullthewaterintothecirculatorysystem.
Themajorcontributingproteinforplasmaoncoticpressureis
albumin.

170.Mostrecenttastesensationis?
a)Sweet
b)Sour
c)Bitter
d)Umami
CorrectAnswer-D
Ans.D.Umami
Theumamitasteisthefifthtastewhichisunique.
Theproposedmechanismofumamitasteisthroughglutamatetaste
sensors(glutamatereceptors)withreleaseofneuronalglutamic
acid.
Innature,therearethreeumamisubstances:
Monosodiumglutamate(MSG)
Disodium5-guanosinemosphosphate(GMP)
Disodium5-ionsinemonophosphate(lMP)

171.Motorevokedpotentialassess?
a)Peripheralmotorpathways
b)Centralmotorpathways
c)Bothoftheabove
d)Regenerationinmuscles
CorrectAnswer-B
Ans.B.Centralmotorpathways
Motorevokedpotentials(MEPs)areelectricalsignalsrecordedfrom
neuraltissueormusclefollowingactivationofcentralmotor
pathways.
Itisusedforassessmentofnervoussystem,especiallyduring
intraoperativeneurophysiologicalmonitoring(IONM).
Mostoften,thisisaccomplishedbyusingtranscranialelectrical
stimulation(TES)ofbrainandrecordingofevokedneuralor
myogenicactivitydistal(caudal)toareathatisatriskduringsurgery.

172.Inelectromyograply[EMG]transient
responseatthetimeofinsertionof
eloctrodeindicates?

a)Spontoneousmuscleactivity
b)Voluntarymuscleactivity
c)Inducedmuscleactivity
d)Cellmembranedamage
CorrectAnswer-D
Ans.D.Cellmembranedamage
[Ref:Clinicalphysiology24th/ep.222]
Electromyographyisaprocessofrecordingelectricalactivityof
muscles.
Itcanbedonebyeitherofthetwomethods:
1. Byputtingasurfaceelectrodeandrecordingactivityofunderlying
muscle.
2. Byinsertinganeedleelectrodeintothemuscle.
Therearetheretypesofsignals(byneedleelectrode):-
i)Insertionalactivity:
Itistheelectricalactivityatthetimeofinsertionofelectrodedueto
disruptionofcellmembrane.
Thisshouldbeignored.
ii)Restingactivity(spontaneousactivity).
iii)Voluntarymusclerecruitment.

173.Trueaboutvisualcyclecascade?
a)Associatedwithconformationalchangeinopsin
b)Lightcausesisomerizationofall-trans-retinolto11Cis-retinol
c)Retinol[alcohol]isinvolved
d)Allaretrue
CorrectAnswer-A
Ans.A.Associatedwithconformationalchangeinopsin
Thereisconformationalchangeinopsin.
Lightcausesconversionof11cis-retinaltoalltransretinal.
Retinal(aldehyde)isinvolved(notretinal).
Whenlightfansonphotoreceptors,11-Cisretinalofrhodopsinis
isomerizedtoall-transretinal(Photoisomerization).
Thechangeinphysicalconfigurationissuchthatitcannothold
proteinopsinandthereforetwoareseparated,i.e.rhodopsinis
hydrolyzed.

174.Trueaboutsmoothmusclecontraction?
a)Troponinplaysanimportantrole
b)Calmodulinhasnorole
c)Phosphorylationofmyosin
d)Alloftheabove
CorrectAnswer-C
Ans.C.Phosphorylationofmyosin
[RefGuyton12'h/ep.64-66]
Insmoothmusclethereisnotroponin.
Thereforecalciuminitiatescontractionthroughamechanism
differentfromthatemployedbyskeletalmuscle.
Smoothmusclecontainsacalciumbindingproteincalled
calmodulin.
Increaseincytoplasmic(sarcoplasmic)calciumleadstoitsbinding
tocalmodulin.
Thecalcium-calmodulincomplexactivatesmyosinkinase,also
calledmyosinlightchainkinase(MLCK).
MLCKisaphosphorylasewhichphosphorylatesalightchain
belongingtomyosinchain,oftencalledcross-bridge
phosphorylation.
Thephosphorylatedmyosinheadinteractwithactin,i.e.,cross-
bridgingofmyosinwithactin.
Thecross-bridgingleadstocontraction.

175.IncreaseinDurationofexpirationisdue
to?
a)J-reflex
b)Head'sparadoxicalreflex
c)Hering-Breurereflex
d)Proprioceptors
CorrectAnswer-C
Ans.C.Hering-Breurereflex
[RefGanong24h/ep.664623d/ep.632]
TheHerring-Breuerinflationreflexisanincreaseinthedurationof
expirationproducedbysteadylunginflation,andthe
Herring-Breuerdeflationreflexisadecreaseinthedurationof
expirationproducedbymarkeddeflationofthelung.

176.Walleriandegenerationisfor?
a)Nervedegeneration
b)Muscledegeneration
c)Nerveregeneration
d)Muscleregeneration
CorrectAnswer-A
Ans.A.Nervedegeneration
[Ref:SamsonWright13h/ep.288]
Thepartofnervedistaltothepointinjuryundergoes'secondaryor
Walleriandegeneration',theproximalpartundergoes"primaryor
retrograde"degenerationuptoasinglenodeRanvier.

177.Withdrawalreflexisalsoknownas?
a)Extensionreflex
b)Stretchreflex
c)Golgitendonreflex
d)Flexorreflex
CorrectAnswer-D
Ans.D.Flexorreflex
[RefUnderstandingofmedicalphysiologyp.680]
Withdrawalreflexisatypicalprotectivereflex.
Apainfulstimulusappliedtothehandorfootresultsinwithdrawalof
thelimb.
Withdrawalinvolvesflexionofthelimbthereforeitisalsocalledas
flexionreflex.
Flexionresultsfromcontractionofflexormusclesandrelaxationof
extensormuscles.
Simultaneouscontractionofflexorsandrelaxationofextensorsis
broughtaboutbyreciprocalinnervation.

178.Tetanyinmuscleoccursinspiteof
normalserumCa"level.Whichionis
responsible?

a)Mg2+
b)Ca2+
c)K+
d)Na+
CorrectAnswer-B
Ans.B.Ca2+
[Ref:Principlesofmedialphysiologyp.105,1061]
Duringasingletwitch,theamountofCa+2releasedintosarcoplasm
isnotenoughtoproducetetanictension.
Whenthemuscleisstimulatedinrapidsuccession,Ca+2comesout
intothesarcoplasmwitheachstimulusandthereisaprogressive
accumulationofCa2+inthesarcoplasm.
TetanictensionisreachedwhensarcoplasmicCa2+levelsreach
theirmaximum.

179.Unspecifiedpainpathwayisfor?
a)Neuropathicpain
b)Trauma
c)Visceralpain
d)Psychogenicpain
CorrectAnswer-D
Ans.D.Psychogenicpain
Textbookofpsychotherapyp.6
Idiopathicorunspecifiedpain:Itispurelypsychologicalinnature
andisthereforecalledpsychogenicpain.

180.Painiscarriedbywhichnervefibers?
a)Act,A13
b)Act,Ay
c)AS,C
d)Ay,C
CorrectAnswer-A:C
Ans.A.Act,A13&C.AS,C
[RdGanong24n/ep.92&23'd/ep.89;Principlesofphysiologyp.
5121]
Fastfiberscarryingpain-+Ad(fastpain)
Slowfiberscarryingpain-)C(slowpain)

181.Actionpotentialgeneratesatfastestrate
in?
a)SAnode
b)AVnode
c)BondleofHis
d)Purkinjefibers
CorrectAnswer-A
Ans.A.SAnode
[Ref:Essentialsofmedicalphysiologp.856]
SAnodedischargesimpulsesatfastestrate,hencetherateatwhich
SAnodefiresdeterminestheheartrate,therefore,
SAnodeisnormalpacemakeroftheheart,i.e.,itdeterminesthe
paceoftheheart.

182.Vibrationsenseisdetectedbywhich
typeofreceptor?
a)Merkel'sdisc
b)Ruffini'sendorgan
c)Pacciniancorpuscle
d)Meissner'scorpuscle
CorrectAnswer-C
Ans.C.Pacciniancorpuscle
[RefPrinciplesofmedicalphysiologyp.647]
Tactile(touch)receptors
Fortouch(superficialtouch):-Meissner'scorpuscle(detecttextureof
surface,i.e.roughorsmooth),Merkel'sdisc(detecttwopoint
discrimination).
Forpressure(deeptouch):-Rufini'sendorgan(slowlyadapting).
Vibrations:DetectedbyPaciniancorpuscle(rapidlyadapting).

183.Vibrationsenseisdetectedby?
a)Nociceptors
b)Deepreceptors
c)Superficialreceptors
d)Noneoftheabove
CorrectAnswer-D
Ans.D.Noneoftheabove
[Refprinciplesofmedicalphysiologyp.647]
VibrationsaredetectedbyPaciniancorpuscleswhicharedeep
tactilereceptors.
Tactile(touch)receptors
Superficial(inepidermisandpapillarylayerofdermis)>Merkel's
disc,meissner'scorpuscle.
Deep(indermisandsubcutaneoustissue)-+Ruffini'sendorgan,
Paciniancorpuscle.

184.Vibrationsaredetectedbywhichtypes
ofreceptors?
a)Slowlyadapting
b)Rapidlyadapting
c)Non-adapting
d)Noneoftheabove
CorrectAnswer-B
Ans.B.Rapidlyadapting
Rapidlyadaptingtouchreceptors-Paciniancorpwcle,Meissner's
corpuscle.
Slowlyadaptingtouchreceptors-Merkel'sdisc,Ruffini'sendorgan.
Noadaptation-Nociceptors(Painreceptors),vestibularreceptors,
musclespindle.

185.Whichofthefollowingreceptoris
stimulatedbysustainedpressure?
a)Ruffini'sendorgan
b)Merkel'sdisc
c)Haircells
d)MeissnerCorpuscles
CorrectAnswer-A
Ans.A.Ruffini'sendorgan
[Ref;Ganong23r/ep.I50]
Pressure(sustainedpressure)stimulatesRuffini'sendorgan,

186."Prosopagnosia"ischaracterizedby:
a)Inabilitytoread
b)Inabilitytoidentifyfaces
c)Inabilitytowrite
d)Inabilitytospeak
CorrectAnswer-B
Ans.B.Inabilitytoidentifyfaces
Prosopagnosiaisarecognitiondeficitinwhichthepatientisunable
torecognizefamiliarfaces.
Faceandobjectrecognitiondeficitsareknownasprosopagnosia
andvisualobjectagnosiarespectively.
Thecharacteristiclesionsinprosopagnosiaandvisualobject
agnosiaconsistsofabilateralinfarctionintheterritoryofthe
posteriorcerebralarteriesandinvolvelingualandfusiformgyri.

187.Broca'sareaisconcernedwith:
a)Wordformation
b)Comprehension
c)Repetition
d)Reading
CorrectAnswer-A
Ai.e.Wordformation
Wernicke'sarea-)Siteofintegration.
Broca'sarea-+Motorpartofspeech.

188.Broadman'sareaformotorspeech?
a)Area1,2,3
b)Area4,6
c)Area28,29
d)Area44
CorrectAnswer-D
Ans.D.Area44
[RefFuller's4lep.10;Ganong24nlep.293]
MotorspeechisformedinBroca'sarea(area44).
Striateareaofcortexisprimaryvisualarea(Brodmann'sarea17or
VI)andislocatedonthesidesofcalcarinefissure.

189.Maximumdensityofmusclespindleis
foundin?
a)Calfmuscle
b)Lumbricals
c)Quadriepsmuscle
d)Triceps
CorrectAnswer-B
Ans.B.Lumbricals
[Ref-Essentialsofmedicalphysiologyp.786]
Themorerefinedthefunction,thegreatertheconcentrationof
musclespindles.
Thegreatestconcentrationofspindlesisfoundinthelumbrical
musclesofthehandandtheninthesuboccipitalmusclesandinthe
extraocularmuscles.

190.Functionofepinephirineand
norepinephineinFightor-Flightresponse
is?

a)Increasedbloodflowtoskin
b)Increasedbloodflowtomuscles
c)Bronchoconstriction
d)Bradycardia
CorrectAnswer-B
Ans.B.Increasedbloodflowtomuscles
[Ref,Understandingmedicalphysiologyp.470,471]
Bothepinephrineandnorepinephrine,alongwithcortisol,are
responsibleforcoordinatingthe"Fight-or-flight"responsein
situationsofperceiveddangerorstress.

191.EPSPisdueto?
a)K'influx
b)Na*efflux
c)Na*influx
d)Ca**influx
CorrectAnswer-C
Ans.C.Na*influx
EPSP-OpeningofligandgatedNa+channelresultinginNa+influx.
IPSP-
OpeningofligandgatedCl-channelresultinginCl-influx.
OpeningofligandgatedK+channelresultinginK+influx

192.Theonlyexcitatoryneuronsincerebellar
cortexare?
a)Purkinje
b)Basket
c)Golgi
d)Granulecells
CorrectAnswer-D
Ans.is'd'i.e.,Granulecells[RefBDCVol.3p92)
Purkinjecellsaretheonlyoutputcellsfromcerebellarcortex.
Purkinjecellssendinhibitoryefferentstodeepcerebellarnuclei.
Basketcellsinhibitbodyofpurkinjecellswhilestellatecellsinhibit
dendritesofpurkinjecells.
Granulecellssendfacilitatoryefferentstobasket,stellateand
purkinjecellsthroughparallelfibers.
Climbingfibersandparallelfibersstimulatepurkinjecells.

193.Sleepwalkingisseeninwhichstageof
sleep?
a)REM
b)Stage1-2NREM
c)Stage2-3NREM
d)Stage3-4NREM
CorrectAnswer-D
Ans.is'd'i.e.,Stage3-4NREM
Importanteventsoccuringduringsleep
Deepsleep/slowwavesleepdisorder:-Theseeventsoccurduring
stage3&4ofNREM.Importantdisordersare:?
1. Somnambulism(Nightwalking):-Patientwalksduringsleep.
2. Sleepterrorornightterrors(pavornocturnes):-Thepatient
suddenlygetsupscreaming,withautonomicarousal(tachycardia,
sweating).Sleepterrorsarerarelyrememberedinthemorning(in
contrasttonightmares).Notreatmentisrequiredonlyreassurance
isrequired.However,inseverecasesbenzodiazepinecanbeused.
3. Sleep-relatedenuresis(Nocturnalenuresis/bedwetting):-
Repetitivevoidingoccursduringsleep.Firstlineoftreatmentis
behaviourtherapy.Ifbehavioraltherapyfails,desmopressin(DOC)
andImipraminecanbeused.
4. Bruxism(Teethgrinding)
5. Sleep-talking(Somniloquy).
REMsleepevents
1. Nocturnalpeniletumescence:-Itisspontaneousoccurenceofan
erectionofthepenisduringsleep.Itisanormalphenomenonand
occursfor80-120minutespernightNocturnalpeniletumescence
canbeusedtodifferentiatebetweenpsychogenicimpotenceand

organicimpotenceasnocturnalpeniletumescenceispreservedin
psychogenicimpotencebutnotinorganiccauseofimpotence.
2. Nightmares(dreamanxietydisorder):-Theyarecharacterizedby
fearfuldreamsoccuringinthelastonethirdofnightsleep.The
personwakesupfrightnedandremembersthedreamvividly(in
contrasttonightterror).
3. Narcolepsy:-Thisischaracterizedbyexcessivedaytimesleep,
oftendisturbednighttimesleepanddisturbancesintheREMsleep.
Ageofonsetisbetween10-20years.Thereisirresistabledesireto
sleepandboutsofsuddensleepeachlastingfor10-30minutes
occuringduringdaytime.Inmajorityofcasesnarcolepsyis
associatedwithoneormoreaccessorysymptoms:?
Cataplexy:-Itisthemostcommonaccessorysymptomandis
characterizedbysuddendecreaseinmuscletoneeither,localor
generalized.
Hypnagogichallucination:-Hallucinationoccuringjustbeforefalling
asleep.Whenhallucinationoccursjustbeforeawakeningitiscalled
hyponopompichallucinations.
Sleepparalysis(leastcommon)
Treatmentofnarcolepsyincludestimulantmedications
(methyphenidate,amphetamines)ormodafinil.

194.Calciumabsorptionishamperedby
a)Protein
b)Lactose
c)Acid
d)Phytates
CorrectAnswer-D
Ans.D.Phytates
Calciumabsorptionisincreasedbyi)Lactose,ii)Proteins,andiii)
Acidicenvironment
Calciumabsorptionisdecreasedbyi)Phytates,ii)Phosphates,iii)
Oxalates;iv)Tetracycline;v)alkalineenvironment

195.SuppressorStriponanterioredgeofpre-
centralgyrushasfollowingfunction?
a)Increaseextensortone
b)Painperception
c)Inhibitionofstretchreflex
d)Voluntarymovement
CorrectAnswer-C
Ans.C.Inhibitionofstretchreflex
[RefGanong25th/ep.242]
"Stimulationoftheanterioredgeofprecentralgyruscauses
inhibitionofstretchreflex.

196.Doll'seyereflexisusedin?
a)Hemiplegic
b)Paraplegic
c)Unconsciouspatient
d)Cerebralpalsy
CorrectAnswer-C
Ans.C.Unconsciouspatient
Doll'seyereflex,alsocalledOculocephalicreflex,isusedtoassess
brainfunctioninunconscious/Comatosepatients.
Itchecksthevestibule-ocularreflexinunconsciouspatients.
Headisrotatedfromsidetosidewitheyeskeptopen.
i)Positive(normal)-
Eyesmoveinthedirectionoppositetothatoftheheadmovement.
Itsignifiesintactbrainstem.
ii)Negative(abnormal)-
Eyesmoveinthedirectionofheadmovement.
Itsignifiesbrainstemdamage.

197.

Partofsympatheticsystemwhichsecrete
chemicaltransmitter?

a)Cardiacganglion
b)Cervicalsympatheticchain
c)Adrenalmedulla
d)Thoracicsympatheticchain
CorrectAnswer-C
Ans.C.Adrenalmedulla
[Ref:TextbookofHumanphysiologybysherwoodp.681.]
TheAdrenalmedullaisactuallyamodifiedpartofthesympathetic
nervoussystem.
Theadrenalmedullaconsistsofmodifiedpostganglionic
sympatheticneuronscalledchromaffincells.
Unlikeordinarypostganglionicsympatheticneurons,chromaffine
cells,donothaveaxonalfibersthatterminateoneffectororgans.
Instead,onstimulationbypreganglionicfibers,thechromaffinein
cellsreleasetheirchemicaltransmitterdirectlyintotheblood.

198.Whichofthefollowinghasdirect
innervationfromsympatheticsystembut
noparasymputheticsupply?

a)Heart
b)Intestine
c)Skin
d)None
CorrectAnswer-C
Ans.C.Skin
Skinhasnoparasympatheticsupplybuthas:-
Sympatheticcholinergicsupply-sweatgland
Sympatheticadrenergicsupply-cutaneousbloodvessels.

199.Neuropraxiais?
a)Damagetoaxon
b)Damagetoendoneurium
c)Damagetoepineurium
d)NoStructuraldamage
CorrectAnswer-D
Ans.D.NoStructuraldamage
[Ref:Apley's8'h/ep'231.]
Neuropraxia?Noanatomicaldisruption;axon&myelinsheath
remainintact.
Axontemesis-Disruptionofaxonwithmyelinsheath;Endoneurium
isintact.
Nuerotoemesis-Completesectionofnerve;Axon,myelinsheath,
endometrium,perimeurium,epineuriumallaredisrupted.

200.RMPinsmoothmuscles?
a)-90mV
b)-70mV
c)-150mV
d)-40mV
CorrectAnswer-D
Ans.D.-40mV
RMPFORVARIOUSEXCITABLETISSUES:
Neuron:-70mV
Skeletalmuscle&Ventricle(cardiacmuscle):-90mV
SAnode:-30to-40mV
Smoothmuscle:-30to-50mV
Innerearhaircell:-150mV
RBC:-10mV
Thyroidgland:-50mV
Haircellsbaselinemembranepotential:-60mV

201.WhichofthefollowingstatementisTRUEaboutBohr'seffect?
a)DecreasedaffinityofHbto02isassociatedwithdecreasedpH
&increasedC02
b)DecreasedaffinityofHbto02isassociatedwithincreasedpH
&decreasedC02
c)DecreasedaffinityofHbto02isassociatedwithdecreasedpH
&C02
d)DecreasedaffinityofHbto02isassociatedwithincreasedpH
&C02
CorrectAnswer-A
ThedecreaseinO2affinityofhemoglobinwhenthepHofbloodfallsiscalledtheBohr
effect.
Itiscloselyrelatedtothefactthatdeoxygenatedhemoglobin(deoxyhemoglobin)bindsH+
moreactivelythandoesoxygenatedhemoglobin(oxyhemoglobin).
ThepHofbloodfallsasitsCO2contentincreases,sothatwhenthePCO2rises,thecurve
shiftstotherightandtheP50rises.
Ref:BarrettK.E.,BarmanS.M.,BoitanoS.,BrooksH.L.(2012).Chapter35.GasTransport
&pH.InK.E.Barrett,S.M.Barman,S.Boitano,H.L.Brooks(Eds),Ganong'sReviewof
MedicalPhysiology,24e.

202.HaldenEffectis?
a)CO2deliverybyincreased02
b)02deliverybyincreasedCO2
c)CO2deliverybyincreasedCO2
d)0,deliverybyincreasedCO
CorrectAnswer-A
Ans.A.CO2deliverybyincreased02
[Ref:Ganong24n/ep.644&23'd/ep.612]
BecausedeoxyhemoglobinbindsmoreH+thanoxyhemoglobin
doesandformscarbaminocompoundsmorereadily,bindingofO2,
tohemoglobinreducesitsaffinityforCO2,(Haldaneeffect).

203.Totalsurfaceareaofrespiratory
membrane?
a)25cm'
b)50cm'
c)25m2
d)100m2
CorrectAnswer-D
Ans.D.100m2
[RelPrinciplesofmedicalphysiologyp.340]
Thicknessofrespiratorymembraneisabout0.5pmanditstotal
surfaceareaintwothelungsequalsabout100m2.

204.Damagetopnemotaxiccenteralongwith
vagusnervecauseswhichtypeof
respiration?

a)Chynestokebreathing
b)Deepandslow
c)Shallowandrapid
d)InspiratorySpasm
CorrectAnswer-D
Ans.D.InspiratorySpasm
[RefPrinciplesofmedicalphysiologyp.981).
Damagetopneumotaxiccenterormidpontinetransection(between
upperandlowerpons)produces-
1. Withvagiintact:-Deepandslowbreathing
2. Withbilateralvagotomy:Apneusticbreathing(apneusis),i.e.
sustainedinspiratoryspasmwhichisinterruptedbybriefand
inefficientexpiration.

205.Transpulmonarypressureisthedifferencebetween:
a)Thebronchusandatmosphericpressure
b)Pressureinalveoliandintrapleuralpressure
c)Atmosphereandintrapleuralpressure
d)Atmosphereandintraalveolarpressure
CorrectAnswer-B
Transpulmonarypressureisthepressuredifferencebetween
alveolarpressureandintrapleuralpressure.Beforethestartof
inspirationorattheendofexpirationitisabout+5cmH2O.Positive
transpulmonarypressurekeepsthealveoliopen.
Intrapleuralpressureisthepressurebetweentwolayersofpleura.
Itisabout-5cmH2Obeforethestartofinspirationorattheendof
expiration.
Alveolarpressureisthepressurewithintheterminalairspaces.It
isthesumofpleuralpressureandelasticrecoilpressureofthelung.
Itisatmosphericbeforethestartofinspirationorattheendof
expiration.
Transthoracicpressureisthepressuredifferencebetween
alveolarpressureandpressureatthebodysurface.
Ref:FundamentalsofRespiratoryPhysiologyByASChakrabarty,
Page32

206.Normaltranspulmonarypressureduring
quietbreathing?
a)+8to+5cmH20
b)-8to-5cmH20
c)0to+1cmH20
d)0to-1cmH20
CorrectAnswer-A
Ans.A.+8to+5cmH20
Duringquietbreathing,transpulmonarypressureisbetween+8cm
(atendofinspiration)and+5cm(atendofexpiration).

207.Whatisthepartialpressureofoxygenat760mmHgatmosphericpressure?
a)76
b)160
c)120
d)130
CorrectAnswer-B
Thepressureofagasisproportionaltoitstemperatureandthenumberofmolesper
volume.
P=nRT/V,where,
n=numberofmoles
P=Pressure
R=Gasconstant
T=Absolutetemperature
V=Volume
Thepressureexertedbyonegasinamixtureofgasesisequaltothetotalpressuretimes
thefractionofthetotalamountofgasitrepresents.
Thepartialpressureofoxygenindryairistherefore0.21x760=160mmofHgatsea
level.

Ref:Ganong,23rdEd,Page588

208.Whatisthepartialpressureforoxygenintheinspired
air?
a)116mmHg
b)158mmHg
c)100mmHg
d)0.3mmHg
CorrectAnswer-B
PartialpressureofO2ininspiredair(PiO2)-158mmHg
Gaseousconcentration&itspartialpressureinalveoli:
1.Oxygen:
Concentration&partialpressurecontrolledby,
*RateofabsorptionofO2intoblood
*RateofentryofnewO2intolungsbyventilatoryprocess.
Values:
*PartialpressureofO2ininspiredair(PiO2)
-158mmHg
*PartialpressureofO2inalveolarair(PAO2)
-100mmHg
-Ca
lculatedby"Alveolargasequation".
*PartialpressureofO2inexpiredair(PEO2)
-116mmHg
2.Carbon-di-oxide:
*PartialpressureofCO2ininspiredair(PiCO2)
-0.3mmHg
*PartialpressureofCO2inalveolarblood(PACO2)
-40mmHg
*PartialpressureofCO2inexpiredair(PECO2)
-32mmHg


209.Isocapenicbufferingis?
a)IncrecasedpCO2withincreasedCO2
b)IncrecasedpCO2withdecreasedCO,
c)NormalpCO,withincreasedCO2
d)Noneoftheabove
CorrectAnswer-C
Ans.C.NormalpCO,withincreasedCO2
*Duringexercise,initiallyCO2productionandincreaseinventilation
areproprotionate.
-So,pCO2remainsthesame.
-Thisiscalledisocapnicbuffering.
*But,inlaterstages,ventilationincreasesmorethantherateof
CO2production.
*Asaresult,PCO2decreases.

210.Vitalcapacityismeasuredby?
a)Plethysmography
b)Gas-dilutionmethed
c)Nitrogenwashouttechnique
d)Spirometer
CorrectAnswer-D
Ans.D.Spirometer
Spirometrycanmeasure:-Tidalvolume(TV),inspiratoryreserve
volume(IRV),expiratoryreservevolume(ERV),vitalcapacity(VC),
forcedvitalcapacity(FVC),FEV.
Spirometrycannotmeasure:-Residualvolume(RV),functional
residualcapacity(FRC),totallungcapacity(TLC).

211.Inpatientswithemphysematousbullae,
totallungvolumeisbestdeterminedby?
a)Spirometry
b)Heliumdilutionmethod
c)Plathysmography
d)Anyoftheabove
CorrectAnswer-B
Ans.B.Heliumdilutionmethod
Totallungcapacity(TLC)isdeterminedbyheliumdilutionmethod
whichdoesnotmeasurevolumeinbullae.
TLCmeasuredbyplethysmographyincludesthevolumeofbullae
also.
Thus,gasvolumeinbullaecanbedeterminedbysubtractingthe
TLCdeterminedbyheliumdilutionfromtheTLCdeterminedby
plethysmography.

212.Allaretrueaboutcomplianceoflung
except?
a)ChangeinVolumeperunitchangeinpressure
b)TotalComplianceis0.2L/cm
c)Ameasureofdistensibility
d)Decreasedinemphysema
CorrectAnswer-D
Ans.D.Decreasedinemphysema
[Ref.Principlesofmedicalphysiologyp.325,326;Ganong24h/ep.
632]
Thechangeinvolumeperunitchangeinpressureiscalled
'compliance'.Thetotalcomplianceofbothlungstogetherinthe
normaladulthumanbeingisabout0.2L/cmwater.
Thatis,everytimethetranspulmonarypressureincreasesby1
centimeterofwater,thelungvolumewillexpand0.2L(200mI).
Complianceisameasureofdispensability.
Complianceisincreasedinemphysema.

213.Normalrespiratorycomplianceis?
a)200ml/cmwater
b)50ml/cmwater
c)100ml/cmwater
d)150ml/cmwater
CorrectAnswer-A
Ans.is'a'i.e.,200ml/cmwater
Thelungsandthoraciccagearebothelasticstructures.Hencethey
displayaconstantrelationshipbetweendistendingpressureand
changeinvolume.
Thechangeinvolumeperunitchangeinpressureiscalled
"compliance".
Thetotalcomplianceofbothlungstogetherinthenormaladult
humanbeingisabout0.2L/cmwater.
Thatis,everytimethetranspulmonarypressureincreasesby1
centimeterofwater,thelungvolumewillexpand0.2L(200ml).
Complianceisameasureofdistensibility.

214.Inrelaxationpressurecurve,inchronic
smokersatZerorelaxationpressure?
a)Lungvolumeincreases
b)Lungvolumedecreases
c)Nochangeinlungvolume
d)Anyoftheabove
CorrectAnswer-A
Ans.A.Lungvolumeincreases
ChronicsmokingcausesCOPDwithlossofelasticityoflungand
thereishyperinflationoflung.
Combinedrecoilpressureoflungsandthoraciccageisinwardly
directedinhyperinflatedlungs.
Aslunglosesitselasticity,thoraciccagedistends(expands)toa
highervolumeatzerorelaxationpressure-+Functionalresidual
capacityisincreased.

215.Whichofthefollowingparameter
IndicateseliminationofCO2fromlung?
a)Pa02
b)pH
c)PaCO2
d)HCO,level
CorrectAnswer-C
Ans.C.PaCO2
[Ref:Clinicalphysiologyp.712]
PaCO2(PartialpressureofarterialCO2)Reflectstheadequacy
ofthelungsventilationandCO,elimination(knownasrespiratory
parameter).

216.InzerogravityV/Qratiois?
a)0.8
b)1
c)2
d)3
CorrectAnswer-B
Ans.B.1
Removalofgravityresultsinmoreuniformventilationandperfusion
thisV/Qshouldbe1.
But,inallrecentstudiesithasbeenshown,thatthereareother
(non-gravitational)factorswhichcontributestoV/Qmismatch.
Thereforeevenatzerogravity,Heterogeneityofperfusionand
ventilationoccurs.
Thus,V/Qratiocannotbe1atzerogravity.
But,ingivenscenarioweacceptsimplegravitationalmodel,inwhich
V/QratiomustbeI'thoughtheoretically.

217.Normal02extractionratiooftissues?
a)5%
b)15%
c)25%
d)40%
CorrectAnswer-C
Ans.C.25%
NormalO2extractionratiois25-30%

218.Poiseuille'sequationstatesthat?
a)Bloodflowisdirectlyproportionto2ndpowerofradius
b)Bloodflowisdirectlyproportiomto4thpowerofradius
c)Bloodflowisinverselyproportionto2ndpowerofradius
d)Bloodflowisinverselyproportionto4thpowerofradius
CorrectAnswer-D
Ans.D.Bloodflowisinverselyproportionto4thpowerofradius
Minutevolume:
Amountofairbreathedin(inspired)orout(expired)bythelungsin
oneminute.So,minutevolume=TidalvolumexRespiratoryrate.
Itisatabout6L

219.

Aorticvalveclosureoccursinwhichpartof
cardiaccycle?

a)Beginningofisovolumetriccontraction
b)Beginningofventricularejection
c)Beginningofisovolumetricrelaxation
d)Duringrapidventricularfilling
CorrectAnswer-B
Ans.B.Beginningofventricularejection
Lungcapillaryvolume?150ml.

220.Roleofionchannelsonvascular
endotheliumis?
a)Ca+influx
b)K+efflux
c)Na+influx
d)Cl-effux
CorrectAnswer-D
Ans.D.Cl-effux
O2therapyisusefulinhypoxichypoxialikehypoventilation(COPD,
restrictivedisease)andhighaltitude.
Itisnotusefulinanemichypoxia(anemia,COpoisoning),stagnant
hypoxia(CHF)andhistotoxichypoxia(cyanidepoisoning)

221.ActionPotentialincardiacmuscleshas
howmanyphases?
a)2
b)3
c)4
d)5
CorrectAnswer-D
Ans.D.5
Bloodflow(Q)isthevolumeflowperunittime(cm3/s),whereas
velocityofbloodflow(V)isdisplacementofbloodperunittime
(cm/s).
Increasingtheradiustwotimeswilldecreasethevelocityby4times.

222.Poiseuille'sequationstatesthat
a)Bloodflowisdirectlyproportionto2ndpowerofradius
b)Bloodflowisdirectlyproportionto4thpowerofradius
c)Bloodflowisinverselyproportionto2ndpowerofradius
d)Bloodflowisinverselyproportionto4thpowerofradius
CorrectAnswer-B
Bloodflowisdirectlyproportionedto4thpowerofradius.
POISEUILLE'SLAW:
*Alsoreferredas"Hagen-Poiseille'sLaw".
*Poiseuille'sequationstates,
-Q=P1-P2*{(r4)/(8L)}
-Q-Flowrate
-(P1-P2)-Pressuredifferenceacrossvessel(providedP1>P2).
--Bloodviscosity.
-r-Radius.
-L-Tubelength.
*Ifparametervaluesremainsconstant,
-Bloodflowisdirectlyproportionalto4thpowerofradius.
*Resistanceofvesseltobloodflowcanbecalculatedbycombining
Ohm'slawwithPoiseuille'sequation.
-BysubstitutingvaluesofQfromPoiseuille'slawinOhm'slaw.
*Implying,resistanceismainlyaffectedby,
-Bloodvesselradius,
-Vasodilatation/vasoconstriction.
*Thusultimately,ifparametervaluesremainconstant,
_Resistancetobloodflowisinverselyproportionalto
4thpowerofradius.


223.Aorticvalveclosurecorrespondstothe
beginningof:
September2011

a)Systole
b)Parasystole
c)Isovolumetricrelaxation
d)Isovolumetriccontraction
CorrectAnswer-C
Ans.C:Isovolumetricrelaxation
CardiacDiastole
Itistheperiodoftimewhentheheartrelaxesaftercontractionin
preparationforrefillingwithcirculatingblood.
Duringventriculardiastole,thepressureinthe(leftandright)
ventriclesdropsfromthepeakthatitreachesinsystole.
Whenthepressureintheleftventricledropstobelowthepressure
intheleftatrium,themitralvalveopens,andtheleftventriclefills
withbloodthatwasaccumulatingintheleftatrium.
Theisovolumicrelaxationtime(IVRT)istheintervalfromtheaortic
componentofthesecondheartsound,thatis,closureoftheaortic
valve,toonsetoffillingbyopeningofthemitralvalve.
Likewise,whenthepressureintherightventricledropsbelowthatin
therightatrium,thetricuspidvalveopens,andtherightventriclefills
withbloodthatwasaccumulatingintherightatrium.
Duringdiastolethepressurewithintherightventricleislowerthan
thatinaorta,allowingbloodtocirculateintheheartitselfviathe
coronaryarteries.

224.Cerebralbloodflowisincreasedby?
a)IncreaseinP02
b)IncreaseinPCO2
c)Decreasemetabolicrate
d)Alloftheabove
CorrectAnswer-A
Ans.A.IncreaseinP02
[Ref:www.ncbi.nlm.nih.govl]
Onemajorfunctionofionchannelsinendothelialcellsisthecontrol
ofCa2+influxeitherbyadirectmodulationofCa2+influxpathway
orbyindirectmodulationNa+andClchannels.

225.STSegmentofECGCorrespondsto?
a)Ventriculardepolarization
b)Ventricularrepolarization
c)Atrialdepolarization
d)AVConduction
CorrectAnswer-D
Ans.D.AVConduction
Actionpotentialinmyocardialfibershas5phases:0,1,2,3and4.

226.Cerebralbloodflowisregulatedbyall,EXCEPT:
a)Intracranialpressure
b)ArterialPC02
c)Potassiumions
d)Cerebralmetabolicrate
CorrectAnswer-C
Factorsaffectingoverallcerebralbloodflowinclude:
Intracranialpressure
Localconstriction&dilationofcerebralarterioles
Meanarterialpressureatbrainlevel
Viscosityofblood
Meanvenouspressureatbrainlevel
ThemostimportantextrinsicinfluencesonCBFarerespiratorygastensionsparticularly
arterialPC02.CerebralbloodflowisdirectlyproportionaltoarterialPC02betweentensions
of20and80mmofHg.
Ref:BarrettK.E.,BarmanS.M.,BoitanoS.,BrooksH.L.(2012).Chapter33.Circulation
throughSpecialRegions.InK.E.Barrett,S.M.Barman,S.Boitano,H.L.Brooks(Eds),
Ganong'sReviewofMedicalPhysiology,24e.

227.InanECGthecardiaceventcorrespondingtotheSTsegmentis:
a)Atrialdepolarisation
b)Ventriculardepolarisation
c)Atrialrepolarisation
d)Ventricularrepolarisation
CorrectAnswer-D
PR?IntervalcorrespondswithatrialdepolarisationandconductionthroughAVnode
QRS?correspondswithVentriculardepolarisation+atrialrepolarisation
QT?correspondswithventriculardepolarisation+ventricularrepolarisation
Ref:ReviewofMedicalPhysiologybyGanong,20thEdition,Page532.

228.STSegmentofECGcorrespondsto
whichphaseofactionpotential?
a)Phase0
b)PhaseI
c)PhaseII
d)PhaseIII
CorrectAnswer-C
Ans.C.PhaseII
RelationofECGwithphasesofactionpotential-
Phase0&1=QRScomPlex
Phase2=STsegment
Phase3=Twave.

229.STSegmentofECGCorrespordsto
whichphaseofactionpotential?
a)Rapiddepolarixation
b)Rapidrepolarizedtion
c)Findrepolarizetion
d)PlateuPhase
CorrectAnswer-D
Ans.D.PlateuPhase
Phase-2isplateauPhase.

230.Baroreeaptorsarelocatedin?
a)Tunicamedia
b)Tunicaintima
c)Tunicaadventitia
d)None
CorrectAnswer-C
Ans.C.Tunicaadventitia
Baroreceptorsaremechano-receptorsthatarelocatedinthe
adventitiaofcarotidarteryandaorta,atspecializedLocationscalled
sinuses.

231.Baroreceptorare?
a)Carotidbody
b)Carotidsinus
c)Aorticbody
d)None
CorrectAnswer-B
Ans.is'b'i.e.,Carotidsinus
Baroreceptorsaremechanoreceptorsthatarelocatedinthe
adventiaofcarotidarteryandaorta,atspecializedlocationscalled
sinuses.
1)Carotidsinusisalittlebulgeattherootofinternalcarotidartery,
locatedjustabovethebifurcationofthecommoncarotidartery.Itis
innervatedbythesinusnerve,abranchofglossopharyngeal(IX
cranial)nerve.
2)Aorticarch(aorticsinus)alsocontainsmechenoreceptors(stretch
receptors)whicharesimilartocarotidsinusreceptors.However,
theirafferentnervefiberstravelintheaorticnerve,abranchof
Vagus(Xcranial)nerve.
Thesinusnerve(fromcarotidsinus)andaorticnerve/vagalfibers
(fromaorticsinus)aretogethercalled`Sinoaorticnerves'.They,
together,arealsoreferedtoas'Buffernerves'becausetheyarethe
afferentsofcardiovascularreflexesthatbufferabruptchangesin
bloodpressure.

232.Afterloadisdecreasedby?
a)Exercise
b)Anemia
c)Thyrotoxicosis
d)Alloftheabove
CorrectAnswer-D
Ans.D.Alloftheabove
Afterload(resistanceofferedtoventricularpumpingaction):
Leftventriclehastopumpoutbloodagainstaorticresistance.
Increasedaorticresistance(e.g.,highBP)tendstodecreasestroke
volume.
Decreasedperipheralresistanceincreasescardiacoutpute.g.,in
exercises,AVfistulaorshunt,severeanemia(duetovasodilationby
anemichypoxia),thyrotoxicosis(duetovasodilationcausedby
increasedO2consumption),andwetberi-beri.

233.Oxygensaturationofvenousbloodis?
a)30%
b)50%
c)70%
d)90%
CorrectAnswer-C
Ans.C.70%
Mixedvenousoxygensaturationisthepercentageofoxygenbound
tohemoglobininvenousblood,i.e,bloodreturningtorightsideof
heart.
Normalvalueis60-80%(average70%)
Alsoknow,Arterialoxygensaturationis95-99%(average97%).

234.Bloodpressureisdependanton?
a)Cardiacoutput
b)Heartrate
c)Strokevolume
d)Alloftheabove
CorrectAnswer-D
Ans.D.Alloftheabove
Arterialbloodpressureistheproductofthecardiacoutputandthe
totalperipheralvascularresistance(TPR).
Meanbloodpressureisthemajordeterminantofadequateblood
flowthroughthetissues.

235.Organwithdualbloodsupply?
a)Heart
b)Liver
c)Kidney
d)Lung
CorrectAnswer-B:D
Ans.(B)Liver(D)Lung
Liverhasdualsupply-Hepaticarteryandportalvein
Lunghasdualsupply-Pulmonaryarteryandbronchialartery.

236.Positionofstretchreceptorsinleft
atrium?
a)AVseptum
b)Interatrialseptum
c)Entranceofpulmonaryvein
d)Noneoftheabove
CorrectAnswer-C
Ans.C.Entranceofpulmonaryvein
[Refprinciplesofmedicalphysiologp.753]
Atrialstretchreceptorsarelocatedatthevariousvenoatrialjunction.
1. Atriocavalreceptorsarelocatedintherightatriumjustatthe
entranceofSVCandIVC,
2. Pulmonaryvenoatrialreceptorsarelocatedintheleftatriumjustat
theentranceofpulmonaryvein.

237.TrueaboutBezold-Jarishreflex?
a)Hypertension
b)Tachycardia
c)Hyperpnea
d)Hypotension
CorrectAnswer-D
Ans.D.Hypotension
Theventricularbaroreceptorsarescatteredthroughouttheleft
ventricleandinterventricularseptum.
StimulationoftheserecePtorsbyinjectionofcertaindrugs(e.g.,
serotonine,veratibine,orniiotine)intotheleftcoronaryartery
producesapnea,bradycardiaandhypotension.
ThisiscalledBezold-Jarischreflexorcoronarychemoreflex.

238.TrueaboutCoronarycirculation?
a)250ml/min
b)Majorflowduringsystole
c)Unifromflowduringfullcardiaccycle
d)Alloftheabout
CorrectAnswer-A
Ans,A.250ml/min
Theheartreceivesitsbloodsupplyfromtwocoronaryarteriesright
andleftwhicharisefromtherootoftheaorta.
Coronarybloodflow,atrest,isabout250mlperminute(5%oftotal
cardiacoutput).

239.Glomerularfilterationofasubstance
dependsupon?
a)Lipidsolubilityofsubstance
b)Bindingcapacitytoalbumin
c)Bothoftheabove
d)Noneoftheabove
CorrectAnswer-B
Ans.B.Bindingcapacitytoalbumin
Plasmaproteinbindingofasubstancedecreasesisglomerular
filtration.
Onlyunboundsubstancesarefiltered.
Glomerularfiltrationdoesnotdependonlipidsolubilityasfiltration
occursthroughtheporesinglomerularmembrane
(Lipidsolubilityaffectsthetransportofsubstanceacrosscellular
membrane)

240.GFRmeasurementhelpindetermining?
a)Heartrate
b)Recoveryfromshock
c)Stageofkidneydisease
d)Bloodvolume
CorrectAnswer-C
Ans.C.Stageofkidneydisease
GFRisameasureofproperfunctioningofkidney.
Itsvaluedeterminesthestageofkidneydisease.

241.Tubuloglomerularfeedbackisforregulationof?
a)BP
b)Bloodvolume
c)Na+reabsorption
d)GFR
CorrectAnswer-D
Ans.D.GFR
Toperformthefunctionofautoregulation,thekidneyshavea
feedbackmechanismthatlinkschangesinsodiumchloride
concentrationatthemaculadensawiththecontrolofrenalarteriolar
resistance.
Thisfeedbackhelpstoensurerelativelyconstantdeliveryofsodium
chloridetothedistaltubuleandhelpspreventspuriousfluctuations
inrenalexcretionthatwouldotherwiseoccur.
Thetubuloglomerularfeedbackmechanismhastwocomponents
thatacttogethertocontrolGFR.

242.Glomerulotubularfeedbackisfor
regulationof?
a)BP
b)Bloodvolume
c)Na+reabsorption
d)Renalbloodflow
CorrectAnswer-C
Ans.C.Na+reabsorption
Tubulo-glomerularfeedback-RegulationofGFRinrelationtorenal
bloodflow.
Glomerulo-tubularfeedback-Regulationoftubularreabsorption
(especiallyNa+2)inrelationtoGFR.

243.Mesangialcellcontractionisdoneby?
a)cAMP
b)Dopamine
c)PAF
d)ANP
CorrectAnswer-C
Ans.C.PAF
Contractionofmesangialcellsisproducedby:-Endothelin,
angiotensinII,vasopressin,norepinephrine,PAF,PDGF,
thromboxaneA2,PGF2,LeukotrienesC4&D4,Histamine.
RelaxationofmesangialcellsisproducedbyANP,Dopamine,
PGE2,cAMP.

244.Mechamismofsecretionofammoniain
distaltubuleis?
a)Primaryactivetransport
b)Symport
c)Antiport
d)Passivediffusion
CorrectAnswer-D
Ans.D.Passivediffusion
Ammonia(NH3)transportinvolvespassivediffusionfromtubular
cellsintothetubularlumen.

245.ASubstancehasclearancesameas
inulinclearance,theSubstarceismainly
excretedinurineby?

a)TubularSecretion
b)Glomerularfiltration
c)Botha&b
d)Vascularleakage
CorrectAnswer-B
Ans.B.Glomerularfiltration
Iftherebeasubstancethatpassesfreelyacrosstheglomerular
membranebutisneitherreabsorbednorsecretedbytubularactivity,
urinaryexcretionwouldrepresenttheamountfilteredandnothing
buttheamountfiltered.
Hence,itsclearancewouldindicatehowmuchplasmagetsfiltered
everyminute.
Allthesefeaturesmakeitaverysuitablesubstanceforestimationof
glomerularfiltrationrate.
InulinclearanceisequaltotheGFR,i.e.,126ml/min.

246.NormalUninarypHis?
a)5.0-6.0
b)6.5-7.0
c)8.5-9.0
d)None
CorrectAnswer-B
Ans.B.6.5-7.0
BloodpH?7.4
InterstitialfluidpH?7.34
UrinepH?6.5?7.0(4.5to8.0)

247.IncreasedaldosteroneandADH
secretiomfollowingmajortraumaresults
inallthefollowingexcept?

a)DecreasedNa+excretionisurine
b)IncreasedK'excretioninurine
c)Increasedosmolarityofurine
d)Increasedwaterexcretion
CorrectAnswer-D
Ans.D.Increasedwaterexcretion
Aftermajortrauma,secretionofADHandaldosteroneincreases.
ADH-Causesincreasedwaterreabsorptionfromcollectingduct.
Aldosterone-CausesincreaseNa+&Cl-reabsorptionandincrease
K+secretion/excretion.

248.Whichcarrierpumpistransportingsolutesinthick
ascendinglimbofHenleloop?
a)CarrierpumpNa-K-2Cltransporter.
b)NaCl-cotransporter
c)Na2+-H+exchanger
d)Na2+-K+exchanger
CorrectAnswer-A
CarrierpumpNa-K-2Cltransporter.
REABSORPTIONINTHICKASCENDINGLIMB:
Sodium,Potassium&Chloridereabsorption:
By"Secondaryactivetransport"-
ThroughNa2+-K+-2Cl--carriertransporter.
TransportsoneNa2+,oneK+,&twoCl-.
Activesodiumabsorptionoccurs.
30%filteredNa2+reabsorbed.

249.Receptiveareaofstomach?
a)Antrum
b)Pylorus
c)Body
d)Fundus
CorrectAnswer-D
Ans.D.Fundus
Anatomicallythestomachisdividedintofundus,body(corpus),
antrumandpylorus.
However,functionally,thestomachisdividedintoaproximaland
distalend.
'Proximalstomach'includesfundusandproximall/3dofthebody
and'distalstomach"includesdistal2/3dofthebodyandtheantrum.
Avasovagalreflextriggeredbyswallowingabolusoffoodcauses
theloweresophagealsphinctertoopenandtheproximalstomachto
dilate(receptiverelaxation)toaccommodatetheswallowedfood.
Asaresult,theinternalpressurehardlyrisesinspiteofthe
increasedfilling.
Thus,theproximalstomachservesprimarilythefunctionofstorage.

250.InnerplexusinGITis?
a)Mucosalplexus
b)Submucosalplexus
c)Auerbach'splexus
d)Myentericplexus
CorrectAnswer-B
Ans.B.Submucosalplexus
Myentericplexus(Auerbach'splexus)-Externalplexus.
Meissner'splexus(submucosalplexus)-Innerplexus.

251.Functionofmyentericplexusisto
regulate?
a)GIsecretion
b)Localbloodflow
c)Motility
d)Alloftheabove
CorrectAnswer-C
Ans.C.Motility
Myentericplexus(Aurbach'splexus)controlsGImotility.
Meissner's(submucosal)plexuscontrolsGIsecretionsandlocal
bloodflow.

252.Chymotrypsinogenisactivatedinto
chymotrypsinby:
a)Trypsin
b)Pepsin
c)Renin
d)HCl
CorrectAnswer-A
Ans.A:Trypsin
Trypsinissecretedintotheduodenum,whereithydrolyzespeptides
intoitssmallerbuildingblocks,namelyaminoacids.Trypsin
catalyzesthehydrolysisofpeptidebonds.
TrypsinshaveanoptimaloperatingpHofabout8.
Trypsinsareconsideredendopeptidases,i.e.,thecleavageoccurs
withinthepolypeptidechainratherthanattheterminalaminoacids
locatedattheendsofpolypeptides.
Trypsinisproducedinthepancreasintheformofinactive
trypsinogen.
Itisthensecretedintothesmallintestine,wheretheenzyme
enteropeptidaseactivatesitintotrypsinbyproteolyticcleavage.The
resultingtrypsinsthemselvesactivatemoretrypsinogens
(autocatalysis),chymotrypsinogen,Elastase/proelastase,
CarboxypeptidaseAandB,ColipaseandPhospholipaseA2.

253.Exampleofexopeptidaseis?
a)Trypsin
b)Chymotrypsin
c)Elastase
d)Carboxypeptidases
CorrectAnswer-D
Ans.D.Carboxypeptidases

254.Secretionofbileoutofhepactocytes
occursvia?
a)Passivediffusion
b)Facilitateddiffusion
c)Osmosis
d)Activetransport
CorrectAnswer-D
Ans.D.Activetransport
Bilecontainssubstancesthatareactivelysecretedintoitcrossthe
canalicularmembrane,suchasbileacids'phosphatidylcholine,
conjugatedbilirubin,cholesterol,andxenobiotics.
Eachoftheseentersthebilebymeansofaspecific
canaliculartransPorter.
Itistheactivesecretionofbileacids,however,thatisbelievedtobe
theprimarydrivingforcefortheinitialformationofcanalicularbile.

255.Absorptionofcalciumionisaffected
mostlyby?
a)Calcitriol
b)Parathormone
c)Glucocorticoids
d)ACTH
CorrectAnswer-A
Ans.A.Calcitriol
Withanaverageintakeof1000mgofcalciumitsnetintestinal
absorptionisonly150-250mg/day.Calciumisabsorbedmainlyin
theduodenumandjejunum(proximalintestine)byanactive
transportmechanismregulatedbycalcitriol.
Parathormoneindirectlypromotesabsorptionofcalciumby
increasingtherenalsynthesisofcalcitriol.

256.VitaminDabsorptionisdecreasedby?
a)Proteins
b)Acid
c)Lactose
d)Fatmalabsorption
CorrectAnswer-D
Ans.D.Fatmalabsorption
Absorptionofallfatsolublevitaminsisdecreasedinfat
malabsorption.
ThesefatsolublevitaminsareA,D,EandK.

257.EffectofcholecystokininonGIT?
a)Increasesgastricacidsecretion
b)Increasessmallintestinalperistlasis
c)Increasesgastricmotility
d)Relaxesgallbladder
CorrectAnswer-B
Ans.is'b'i.e.,Increasessmallintestinalperistalsis

258.Smallintestinalperistalsisiscontrolled
by:
a)Myentricplexus
b)Meissnersplexus
c)Vagusnerve
d)Parasympatheticsystem
CorrectAnswer-A
Ai.e.Myentricplexus
Myenteric(Auerbach's)plexusissituatedb/wandinnervatesouter
longitudinal&middlecircularmuscular
layersandisprimarily
concernedwithmotorcontrolQ.
Submucosal(meissner's)plexus
situatedbetweenmiddlecircularlayerandmucosaisprimarly
concernedwithcontrolofintestinalsecretionQasitinnervates
glandularepithelium,intestinalendocrinecells&submucosalblood
vessels.

259.Howisfolicacidabsorbedinproximal
jejunum?
a)Facilitateddiffusion
b)Activetransport
c)Passivetransport
d)Bothactive&passivetransport
CorrectAnswer-D
AnsD.Bothactive&passivetransport
FolatesarePresentinnaturalfoodsandtissuesaspolyglutamates
becausetheseformsservetokeepthefolateswithincell.
Inplasmaandurine,theyarefoundasmonoglutamatesbecause
thisistheonlyformthatcanbetransportedacrossmembranes.
Enzymesinthelumenofthesmallintestineconvertthe
polyglutamateformtothemonoglutamateformofthefolate,whichis
absorbedintheproximaljejunumviabothactiveandpassive
transport.

260.Colipaseis:
a)IssecretedbyOxynticcells
b)Issecretedintheactiveform
c)Helpsgastriclipase
d)EncodedbythegeneCLPS
CorrectAnswer-D
Colipaseisasmallproteincofactorneededbypancreaticlipaseforefficientdietarylipid
hydrolyisis.Itissecretedasaninactiveformprocolipasebythepancreasandinthe
intestinalitisconvertedtotheactiveformbytrypsin.Efficientabsorptionofdietaryfatsis
dependentontheactionofpancreatictriglyceridelipase.ColipasebindstotheC-terminal,
non-catalyticdomainoflipase,therebystabilisingasactiveconformationandconsiderably
increasingtheoverallhydrophobicbindingsite.ItisencodedbythegeneCLPS.
Ref:Ganong'sReviewofHumanPhysiology,21stEdition,Page476

261.Cholecystokininisproducedfrom:
a)Hepatocyte
b)Gastricmucosa
c)Duodenalmucosa
d)Epithelialcellsofdistalcommonbileduct
CorrectAnswer-C
Themajorfactorcontrollingthecontractionofthegallbladderisthehormone
cholecystokinin(CCK),whichisreleasedfromtheduodenalmucosa(Icells)inresponseto
theingestionoffatsandaminoacids.
Reference:
HarrisonsPrinciplesofInternalMedicine,18thEdition,Page2616

262.Somatomedin-Cdeficiencycauses?
a)Achondroplasia
b)Cretinism
c)Growthretardation
d)JuvenileDM
CorrectAnswer-C
An.s.C.Growthretardation
Effectsofgrowthhormoneonskeletalgrowthismediatedby
somatomedins.
Deficiencyofsomatomedinscausesgrowthretardationand
dwarfism.

263.Allareaboutadiponectinexcept?
a)Secretedbyadiposetissue
b)Lowersglucose
c)IncreasesFFAoxidation
d)PositiveCorrelationwithBMI
CorrectAnswer-D
Ans.D.PositiveCorrelationwithBMI
Adiponectionisahormonesecretedbyadiposetissue.
ItlowersglucoseandFFAlevels.
AdiponectinhasnegativecorrelationwithBMI,andmorenegatively
withvisceralfatthansubcutaneousfat.
Adiponectinhasinszlirsensitizingactionanditslevelscorrelatewith
insulinsensitivity.Byinsulinsensitizingaction,adiponectinreduces
tissueTGcontent,increaseFFAoxidation,andincreaseglucose
uptakebytissue.
Adiponectinalsoinhibitsadhesionmolecules(E-selectin,VEAM)
andmacrophages(decreaseuptakeofoxidizedLDL).
Adiponectinreceptorsarefoundinliverandskeletalmuscles.
Recently,receptorsarealsofoundinpancreasp-cells.
Adiponectinsecretionishormoneregulated.Decreasedlevelswith
insulinandglucorticoidsareseen.
Adiponectionlevelsarelowinobesity,type2DM,ischemicheart
disease(IHD)anitmetabolicsyndrome.
Adiponectingeneislocatedonchromosome3.

264.Hormoneresponsibleforgalactopoiesis
?
a)Growthhormone
b)Insulin
c)Oxytocin
d)Prolactin
CorrectAnswer-D
Ans.D.Prolactin
Galactokinesis
Alsocalledmilkejectionormilkletdovn.
Itisbroughtaboutbyoxytocinwhichstimulatescontractionofthe
myoepithelialcellsinthemammaryalveoliandductsie.contraction
oflactiferousductsandsinuses.
Oxytocinsreleasedwhenthenippleisstimulatedduringsuckling.

265.Ionotropiceffectofthyroidhormoneis
by?
a)Membranereceptors
b)cAMP
c)PotentiationofCatecholamines
d)cGMP
CorrectAnswer-C
Ans.C.PotentiationofCatecholamines
[RelGanong24th/ep.j49l
Thethyroidhormonesincreasetheheartrate,cardiaccontractility,
strokevolumeandcardiacoutput,andconsequentlyalsothe
systolicBP.
ButthediastolicBPmayfallduetovasodilatationinseveralvascular
bedswithdecreaseinperipheralvascularresistance.
Thecardiovasculareffectsofthyroidhormonesarepartlymediated
bypotentiationofeffectsofcatecholamines.

266.3-subumitofinsulinreceptorbindsto?
a)Guanylylcyclase
b)Adenylylcyclase
c)IP3-DAG
d)Tyrosinekinase
CorrectAnswer-D
Ans.D.Tyrosinekinase
Insulinreceptors
Theinsulinreceptorisacombinationof4subunitsheldtogetherby
disulfidelinkages-TwoAlphasubunitsandtwoBetasubunits,
Alphasubunitslieentirelyoutsidethecellmembraneandfunctionas
insulinbindingsite.
BetasubunitsPenetratethroughcellmembraneandprotrudeinto
thecytoplasmwhichhasattachedlocaltyrosinekinase.
Thusinsulinreceptorisanexampleofanenzyme-linkedreceptor
(insulinkagroupIIDhormone).

267.Relationshipbetweeninsulinand
glucoseconcentration?
a)Linear
b)Hyperbola
c)Sigmoidal
d)BellShaped
CorrectAnswer-C
Ans.C.Sigmoidal
'TheRelationshipbetweenglucoseconcentrationandinsulin
secretionissigmoidal.
Theglucose-insulinresponsecurvecomefromthediscoverythat
insulinsecretiondidnotrespondasalinearfunctionofglucose
concentration.

268.Insulinmediatedtransportofglucoseis
?
a)Seeninadiposetissue
b)ViaGLUT-2
c)MainMechanisminRBCs
d)Allaretrue
CorrectAnswer-A
Ans.A.Seeninadiposetissue
Insulinstimulatestheuptakeofglucosebymyocytes(skeletal
muscle,cardiacmuscles),adipocytes(adiposetissue)and
hepatocytes.
Tissuesthatdonotdependoninsulinforglucoseuptakeinclude
brain,erythrocytes(RBC),theepithelialcellsofkidney&intestine,
Liver,andCornea&lensofeye.
Themechanismthroughwhichinsulinincreasesglucoseuptakeis
differentindifferenttissues.Inthemuscleandadiposetissues,
insulinincreasefacilitateddiffusionbyincreasingglucosetransporter
(GLUT-4)onthecellmembrane.

269.Notacontentofsperm?
a)Golgiapparatus
b)Mitochondria
c)Lysosome
d)Endoplasmicreticulum
CorrectAnswer-D
Ans.D.Endoplasmicreticulum
Spermatozoan(sperm)hasfollowingparts:

1. Head
2. Middlepiece
3. Principalpiece
4. Endpiece(tail)

270.Acrosomereactionisseenin?
a)Spermatogenesis
b)Oogenesis
c)Fertilization
d)Mensturation
CorrectAnswer-C
Ans.C.Fertilization
Tofertilizetheovumthespermsundergo-

1. Capacitation
2. Acrosomereaction
Acrosomereaction:
Itfollowsthecapacitationofthesperm.
Itoccursinampulla.
Spermbindstozonapellucida?Zonaprotein-3(ZP-3)of
zonapellucidahelpsinattachmentaswellasininducingacrosomal
reaction.
Acrosomereleaseslysomalenzymes,especiallyproacrosinwhich
hashighaffinityforzonapellucida.
Proacrosinhashydrolyticactionthatdegradeszonapellucidainthe
verylocalregionwherethespermcellisattached.
Thisallowsspermatozoatoswimtheirwaytothevitellinemembrane
oftheovum.
Afterthisfertilizationtakesplace.

271.Ironisconservedby?
a)Hemopexin
b)Hepicidin
c)Hemomedins
d)None
CorrectAnswer-A
Ans.A.Hemopexin
[Ref:Vasudevan3n/ep.196]
Humanbodyhasmechanismstoconserveironandpreventitsloss
frombody-
i)WhenRBCislysed,hemoglobinentersintocirculation.
Beingsmallmolecularweightsubstancehemoglobinwillbelost
throughurine.
Topreventthisloss,Hbisimmediatelytakenupbyhaptoglobin.
ii)WhentheglobinpartisremovedfromHb,thehemeisproduced,
andisreleaseditocirculation.
Inordertopreventitsexcretionthroughurine,hemeisboundwith
hemopexin.

272.Mostimportantintracellularbuffer?
a)Bicarbonate
b)Albumin
c)Phosphate
d)Ammonia
CorrectAnswer-C
Ans.C.Phosphate
Bloodbuffers:Hemoglobin,Plasmaproteins,Bicarbonate.
Intracellularbuffers:Phosphates(H2PO4),intracellularproteins.
Urinarybuffers-Bicarbonate,Phosphate,ammonia.

273.Primitiveredcellsfirstoriginatesintheearlyembryoniclifeisin:
a)Liver
b)Yolksac
c)Bonemarrow
d)Spleen
CorrectAnswer-B
Duringembryogenesis,hematopoiesisoccursinextraembryonicyolksac,thefetalliver,the
thymus,andthepretermmarrow.Theoriginofhematopoieticcellsiscloselytiedto
gastrulation,theformationofmesodermcells,andtotheemergenceoftheendothelial
lineage.Hematopoiesisisfirstestablishedsoonafterimplantationoftheblastocyst,withthe
appearanceofprimitiveerythroidcellsinbloodislandsoftheyolksacbeginningatday18
ofgestation.

"Primitive"redcellsderivedfromtheyolksacconstituteadistinct
transienterythroidlineagethatdiffersfrom"definitive"redcellsthat
subsequentlymatureinthefetalliverandmarrow.
Ref:PalisJ.,SegelG.B.(2010).Chapter6.HematologyoftheFetusandNewborn.InJ.T.
Prchal,K.Kaushansky,M.A.Lichtman,T.J.Kipps,U.Seligsohn(Eds),Williams
Hematology,
8e.

274.Convergingpointofbothpathwayin
coagulationisat:
a)FactorVIII
b)StuartfactorX
c)FactorIX
d)FactorVII
CorrectAnswer-B
AnswerisB(StuartfactorX)
TheextrinsicandIntrinsicpathwaysincoagulationconvergeatthe
StuartfactorX.


275.Penileerectionismediatedby?
a)Parasympatheticsystemviamuscarinicreceptors
b)Parasympatheticsystemvianicotinicreceptors
c)Sympatheticsystemviaa-receptors
d)Sympatheticsystemvia(3-receptors
CorrectAnswer-A
Ans.A.Parasympatheticsystemviamuscarinicreceptors
[RefGanong24'h/ep.339-342,Ganong23d/ep.341]
ErectionofpenisMuscariniceffectofparasympatheticsystem.
Ejaculation-alpha-receptorofsympatheticsystem

276.Inhibitionofheartbyvagusismediated
bywhichreceptors-
a)M1
b)M2
c)NN
d)NM2
CorrectAnswer-B
Ans.B.M2
EffectofparasympatheticsystemonheartarethroughM2
receptors.

277.InhibitionofheartbyvagusbyM2
receptorsismedicatedbywhich
mechanism?

a)cAMP
b)Cat'
c)DAG
d)None
CorrectAnswer-A
Ans.A.cAMP
Muscarinicreceptors(cholinergicreceptors)inheartareM2type.
Theseareresponsibleforvagusmediatedbradycardia.
M2mediatedactionsarethroughcAMP

278.Glucagonactivateswhichenzyme?
a)Pepsinogen
b)Trypsinogen
c)Adenylylcyclase
d)None
CorrectAnswer-C
Ans.C.Adenylylcyclase
GlucagonactsthroughcAMPbyactivatingadenylylcyclase.

279.Sizeofplateletsis?
a)1A?
b)2A?
c)1?m
d)2?m
CorrectAnswer-D
Ans.D.2?m
Platelet=2-3micrometer.

280.RespirateryquotientistheRatioof-
a)CO2consumedto02released
b)02releasedtoCO2consumed
c)Co,releasedto02consumed
d)02consumedtoCo,released
CorrectAnswer-C
Ans.C.Co,releasedto02consumed
RQistheratiooftheamountofCO2releasedtotheamountofO2
consumed.
RQofCarbohydrateis1.00,forfatis0.70andforproteinsitisabout
0.82.

281.Intercalateddiscispresentin:
a)Cardiacmuscle
b)Smoothmuscle
c)Skeletalmuscle
d)All
CorrectAnswer-A
A.i.e.Cardiacmuscle

282.IfHbAlCis8%,Whatwillbethevalueof
bloodglucose[mg/d1]?
a)100
b)150
c)200
d)300
CorrectAnswer-C
Ans.C.200
Meanbloodglucose(mg/dl)=(35.6xHbA1C)-77.3=(35.6x8)-
77.3=207.5
Meanbloodglucose(mmoUL)=(1.98xHbA1C)-4.29

283.Preaccelerinis?
a)EaterII
b)FacterV
c)FacterVII
d)FacterX
CorrectAnswer-C
Ans.is'c'i.e.,FacterVII

284.Conversionofprekallikreintokallikrein
requireswhichclottingfactor-
a)XIII
b)XII
c)XI
d)X
CorrectAnswer-B
Ans.is'b'i.e.,XII
[RelEssentialofmedicalphysiologyp.612]
InitiationofintrinsiccoagulationpathwayoccurswhenfactorXIIis
exposedtonegativelychargedsurface.
ThisleadstoactivationoffactorXIItoXIIa.
FactorXIIacanthenhydrolyzeprekallikreintokallikrein,whichin
turnactivatesmoreXIItoXIIa.

285.Caisson'sdiseaseisassociatedwith?
a)Rapiddescendinaircraft
b)Rapiddescendofdeepseadivers
c)Underwaterconstructionworkers
d)Alloftheabove
CorrectAnswer-C
Ans.C.Underwaterconstructionworkers
Caisson'sdisease,(alsocalleddecompressionsickness,Bends,
Diver'sParalysis,Dysbarism)isaparticularformofgasembolism,
whichoccurswhenindividualsareexposedtosuddenloweringof
atmosphericpressurelike-rapidascendofscubaanddeepsea
divers,individualinunpressurizedaircraftinrapidascent(e.g.,in
pilots),underwaterconstructionworkersandextra-vehicularactivity
fromspacecraft.

286.RegardingCaisson'sdiseasewhich
statementamongthefollowingis
CORRECT?

a)Lungdamageiscausedbyairembolism
b)Paininthejointsisduetonitrogenbubbles
c)Tremorsareseenduetonitrogennarcosis
d)HighpressureNervoussyndromecanbepreventedbyusing
mixturesofOxygen&Helium
CorrectAnswer-B
Ans.is.B.aininthejointsisduetonitrogenbubbles
[REF:Ganong22edchapter
37,http://en.wikipedia.org/wiki/Decompression_sickness]
DecompressionSickness:
Asadiverbreathing80%N2ascendsfromadive,theelevated
alveolarPN2falls.N2diffusesfromthetissuesintothelungsalong
thepartialpressuregradient.Ifthereturntoatmosphericpressure
(decompression)isgradual,noharmfuleffectsareobserved;butif
theascentisrapid,N2escapesfromsolution.Bubblesforminthe
tissuesandblood,causingthesymptomsofdecompression
sickness(thebends,caissondisease).Bubblesinthetissuescause
severepains,particularlyaroundjoints,andneurologicsymptoms
thatincludeparesthesiasanditching
Lungdamagecausesairembolismandnotviceversa.Theproblem
ofnitrogennarcosiscanbeavoidedbybreathingmixturesof02and
helium,anddeeperdivescanbemade.However,thehigh-pressure
nervoussyndrome(HPNS)developsduringdeepdiveswithsuch
mixtures.TremorsaresymptomsofhighpressureNervous
syndrome

syndrome
Signsandsymptomsofdecompressionsickness
Bubble
Signs&symptoms(clinical
DCStype
location
manifestations)
Mostlylarge
Localizeddeeppain,rangingfrom
Musculoskeletal joints
mildtoexcruciating.Sometimes
(elbows,
adullache,butrarelyasharppain,
shoulders,hip, Activeandpassivemotionof
wrists,knees, thejointaggravatesthepain,The
ankles)
painmaybereducedbybending
thejointtofindamorecomfortable
position,Ifcausedbyaltitude,
paincanoccurimmediatelyorupto
manyhourslater.
Cutaneous
Skin
Etching,usuallyaroundtheears,
face,neck,arms,anduppertorso,
Sensationoftinyinsectscrawling
overtheskin(formication),
Mottledormarbledskinusually
aroundtheshoulders,upperchest
andabdomen,withitching,Swelling
oftheskin,accompaniedby
tinyscar-likeskindepressions
(pittingedema)
Neurologic
Brain
Alteredsensation,tinglingor
numbnessparesthesias,increased
sensitivityhyperesthesia,Confusion
ormemoryloss(amnesia),
Visualabnormalities,Unexplained
moodorbehaviourchanges,
Seizures,unconsciousness
Neurologic
Spinalcord
Ascendingweaknessorparalysisin
thelegs,Girdlingabdominalor
chestpain,Urinaryincontinenceand
faecalincontinence
Constitutional
Wholebody
Headache,Unexplainedfatigue,
Generalisedmalaise,poorly
localisedaches

localisedaches
Audiovestibular
Lossofbalance,Dizziness,vertigo,
Innerear
nausea,vomiting,Hearingloss
Pulmonary
Lungs
Drypersistentcough,Burningchest
painunderthesternum,
aggravatedbybreathing,Shortness
ofbreath

287.Followingistrueaboutoculocardiac
reflexexcept?
a)Itisalsocalledaschnerphenomenon
b)Itismediatedbyocculomotorandvagusnerve
c)Itischaracterizedbybradycardiafollowingtractiononextra-
ocularmuscles
d)Reflexismoresensitiveinneonates
CorrectAnswer-B
Ans.is'b'i.e.,Itismediatedbyocculomotorandvagusnerve
Oculocardiacreflex
Oculocardiacreflex,isalsoknownasAschnerphenomenon,
Aschnerreflex,orAschner-Dagninireflex,oItischaracterizedby
decreaseinpulserate(bradycardia)associatedwithtractionapplied
toextraocularmusclesand/orcompressionoftheeyeball.
Thereflexismediatedbynerveconnectionsbetweentheophthalmic
branchofthetrigeminalcranialnerveviatheciliaryganglion,and
thevagusnerveoftheparasympatheticnervoussystem.
Thisreflexisespeciallysensitiveinneonatesandchildren,
particularlyduringstrabismuscorrectionsurgery.However,this
reflexmayalsooccurwithadults.
Bradycardia,junctionalrhythmandasystole,allofwhichmaybelife-
threatening,canbeinducedthroughthisreflex.

288.InCOpoisoning,immediateemergency
treatment:
Jharkhand10

a)5%CO2inhalation
b)10%CO2inhalation
c)HighflowO2
d)Nitroglycerine
CorrectAnswer-C
Ans.HighflowO2

289.Bloodtestisbarrierintestisisformed
by?
a)Sertolicells
b)Leydigcells
c)Granulosacells
d)None
CorrectAnswer-A
Ans.A.Sertolicells
[RefGanong23'd/ep.402]
Junctionbetweenadjacentsertolicellsformblood-testisbarrier.

290.Whichofthefollowingisfalseabout
peripheralnerveinjury?
a)Neuropraxiaisirreversible
b)Epineuriumisintactinaxonotmesis
c)Neurotmesisisthemostsevereformofinjury
d)Welleriangenerationstartsinaxonotmesis
CorrectAnswer-A
Ans.A.Neuropraxiaisirreversible
Neuropraxia(Classl)
Thisistheleastsevereformofnerveinjury,withcompleterecovery.
Inthiscase,theaxonremainsintact,butthereismyelindamage
causinganinterruptioninconductionoftheimpulsedownthenerve
fiber.
Mostcommonly,thisinvolvescompressionofthenerveordisruption
tothebloodsupply(ischemia).
NoWalleriandegenerationisseen.

291.Firsttooccurafterasharpnervecut?
a)Chromatolysis
b)Polymorphicarrangement
c)Increasedproteinsynthesis
d)Macrophageactivation
CorrectAnswer-A
Ans.A.Chromatolysis
Within6hoursofinjury,thenucleusmigratestotheperipheryofthe
cellwhereNissl'sgranulesandroughendoplasmicreticulumbreak
upanddisperse.
Thisphenomenoniscalledchromatolysis.
Thepartofnervedistaltothepointinjuryundergoes'secondaryor
Walleriandegeneration',theproximalpartundergoes"primaryor
retrograde"degenerationuptoasinglenodeRanvier.

292.LDHhashowmanyisoenzymes
a)3,basedonBandMpolypeptidesubunits
b)5,basedonBandMpolypeptidesubunits
c)7,basedonHandMpolypeptidesubunits
d)5,basedonHandMpolypeptidesubunits
CorrectAnswer-D
Ans.is'd'i.e.,5,basedonHandMpolypeptidesubunits
lRef:Dineshpuri3'd/ep.1221
LDHisatetramerwithtwotypesofpolypeptideunits:(H)(for
heart)andM(formuscle).
lthasfiveisoenzymes:
LDH1(HHHH),LDH2(HHHM),LDH3(HHMM),LDH4
(HMMM),andLDHs(MMMM).
LDH-IandLDH-2arethepredominantisozymesinmyocardium,
thereforetheseareraisedinMI(LDHf>LDH2),
LDH-fismorespecificformyocardium(asithas4H)than
LDH-2.
ThepredominentisoenzymeinliverisLDH5;HenceLDH5is
raisedinliverdiseaseslikeviralhepatitis.
NormalLDHpatternonelectrophoresisisLDH2>LDHI>LDH
3>LDH4>LDH5.
InMILDHIisraisedmoretharLDHz,So,patternbecomes
LDHI>LDH2>LDH3>LDH4>LDH5.
IncreaseintotalLDHlevelisalsoseeninhemolyticanemia,
hepatocellulardamage,musculardystrophies,leukemia,
carcinomas,cerebrovascularaccident,pancreatitis,kidney
disease,intestinalandpulmonaryinfarction,megaloblastic
anemiaandinfectiousmononucleosis.Thereforestudyofspecific

isozymeismoresignificant.

293.Whichofthefollowingisknownas
suicidalenzyme?
a)Lipoxygenase
b)Cyclooxygenase
c)Thromboxanesynthatase
d)5'nucleotidase
CorrectAnswer-B
Suicidalenzymeisone,whichundergoesself-destructioninorderto
terminateitsownacitivity,e.g.Cyclooxygenase.
SuicidalInhibitionisconversionofasubstratebytheenzymeintoa
metabolite,whichisapotentinhibitoroftheenzyme;example:
Xanthineoxidaseconvertsallopurinoltoalloxanthine(oxypurinol),
whichisamorepotentinhibitorofallopurinol.
Cycloorygenaseisknownassuicideenzymebecauseit
catalyzesitsowndestruction.

294.Ratelimitingenzymeinbileacid
synthesis?
a)Desmolase
b)21-hydroxylase
c)7-hydroxylase
d)12-hydroxylase
CorrectAnswer-C
Ans.is'c'i.e.,7-hydroxylase
Abouthalfofthecholesterolinthebodyisultimatelymetabolizedto
bileacids.
Theprimarybileacidsaresynthesizedfromcholesterolinliver.
Thesearecholicacidandchenodeoxycholicacid.
Ratelimitingenzymeinprimarybileacidssynthesisis7-
hydroxylase(cholesterol7-hydroxylase).
Thisenzymeisinhibitedbybileacidsandinducedbycholesterol.
Thyroidhormonesinducetranscriptionof7a-hydroxylase,thusin
patientswithhypothyroidismplasmacholesteroltendstorise
(becauseofinhibitionof7-hydroxylasewhichinturninhibits
conversionofcholesteroltobileacids).

295.Enzymeactivityisexpressedas:
a)Millimoles/lit?
b)Milligm/lit?
c)Mg/dl
d)Micromoles/min
CorrectAnswer-D
Ans.is.D.Micromoles/min

296.Exampleofallostericinhibition
a)Inactivationofglycogensynthasebyphosphorylation
b)Decreasedsynthesisofglucokinasebyglucagon
c)InhibitionofPFK-1bycitrate
d)Alloftheabove
CorrectAnswer-C
Answer-Ans.is'c'i.e.,InhibitionofPFK-lbycitrate[lRef,
Dineshpuri3d/ep.1161]
Enzyme:-PhosphofructokinaseI
Pathway:-Glycolysis
Stimulator:-AMP,ADP,fructose-6-phosphaie,fuctose-2,6-
bisphophate.
Inhibitor:-ATP,citrate,Ca2+,Mg+

297.Lysyloxidaserequireswhichcofactor-
a)Zn
b)Cu
c)Se
d)Fe
CorrectAnswer-B
Ans.is'b'i.e.,Cu[RelHarper27t'/ep.546]
metal
metalloenzymes
Calcium
Lipase,Lecithinase
Cytochromeoxidase,Tyrosinase,Lysyloxidase,
Copper
Superoxidedismutase,Ascorbicacidoxidase,
Ferroxidase(ceruloplasmin)
Cytochromeoxidase,Xanthineoxidase,Catalase,
iron
Peroxidase
Carbonicanhydrase,alkalinephosphatase,RNA
polymerase,alcoholdehydrogenase,
Zinc
Carboxypeptidase,Prophobilinogensynthase,
glutamatedehydrogenase,lactatedehydrogenase,
Superoxidedismutase
Hexokinase,phosphofructokinase,glucose-6-
Magnesium phosphatase,enolase,creatininekinase,
Phosphatases,kinase
Arginase,Pyruvatecarboxylase,phosphoglucomutase,
Manganese Glycosyltransferase,Hexokinase,Enolase
Potassium
Pyruvatekinase
Selenium
Glutathioneperoxidase
Nickel
Urease
molybednum Xanthineoxidase

molybednum Xanthineoxidase

298.Mechanismofconversionoftrypsinogen
totrypsin-
a)Hydrolysis
b)Phosphorylation
c)Removalofpartofprotein
d)RemovalofCarboxylgroup
CorrectAnswer-C
Ans.is'c'i.e.,Removalofpartofprotein[Ref:Dineshpuri
3'd/ep-1181
Allzymogens(includingtrypsinogen)areactivatedbyremovalof
asmalllengthofprotein(fewaminoacids)fromoneendofthe
molecule.
TrypsinogenTrypsin+peptidefragent
Pepsinogenpepsin+peptidefragment

299.Immediatesourceofenergyis?
a)Cori'scycle
b)HMP
c)ATP
d)TCAcycle
CorrectAnswer-C
Ans.is'c'i.e.,ATP
Therearethreeenergysystemstoprovideenergyformuscular
activities.
Immediateenergysystem:EnergyisprovidedbystoredATPand
creatinephosphate.
Anaerobicglycolyticsystem(lacticacidsystem):Energyis
generatedbyutilizationofglucoseorglycogenbyanaerobic
glycolysis.Thisenergyisalsogeneratedearly.
Aerobicoroxidativesystem:Energyisgeneratedbyutilizationof
glucose/glycogen,andfattyacidsthroughoxidativepathways,e.g.
TCAcycle.
Thesethreeenergysystemsoperateasacontinuum;eachsystem
isalwaysfunctioning,evenatrest.oWhatvariesistherelative
contributioneachsystemmakestototalATPproductionatanygiven
time.
Oxidative(aerobic)
Immediate
Anerobic
system
energysystem
glycolyticsystem Glucoseor
ATP,creatine
Glucoseor
glycogen,fatty
phosphate
glycogen
Substrates
acids
EnergyproductionVery
Fast
Slow
fast
Peakat
0-30sec.
20-180sec.
>3min

Peakat
0-30sec.
20-180sec.
>3min
Limiting
DepletionofCrP, Lacticacidas_. Glycogendepletion
factor
ATP
vitiation
Activity
Powerlifting&
Longersprints
Enduranceevents
example
weight
lifting,short
Middledistance
Teamsports
sprints
teamsports
Jumping,
Ballgames
Ballgames(Soccer,
throwing
(Soccer,rugby)
fieldhockey)

300.Themechanismofactionofuncouplersofoxidativephosphorylation
involves:
a)InhibitionofATPsynthase
b)StimulationofATPsynthase
c)Disruptionofprotongradientacrosstheinnermembrane
d)Disruptionofprotongradientacrossoutermembrane
CorrectAnswer-C
Uncouplersareproteincarriersthatcanfreelypassthroughtheinnermitochondrial
membrane.
Itallowstranslocationoftheprotonsintotheintermembranousspaceduringtheelectron
transportintherespiratorychainbutblockstheformationofprotongradientacrossthe
innermitochondrialmembrane.
Thermogeninaproteinpresentintheinnermitochondrialmembraneofadipocytesis
anexampleofphysiologicuncouplersofoxidativephosphorylation.

Examplesofuncouplersare:2,4dinitrophenol,pentachlorophenol,nigericin,
thyroxinandthermogenin.

Ref:Jaypee'sReviewofMed.BiochemistryByS.M.Rajupage102.

301.TrueaboutNADP-
a)ActsascoenzymeformofNiacin
b)InvolvedinHMPshunt
c)Notinvolvedinglycolysis
d)Allaretrue
CorrectAnswer-D
Ans.is'd'i.e.,Allaretrue
Niacin,intheformofnicotinamide,isincorporatedintothe
structureoftwocoenzymes:nicotinamideadeninedinucleotide
(NAD+)andnicotinamideadeninedinucleotidephosphate
(NADP+)
NADPisinvolvedinHMPshuntandNAPDHisproduced.
InglycolysisNADisinvolved(notNADP).

302.Inmalateshuttle,NADHProduceshow
manyATPs
a)1
b)1-5
c)2
d)2.5
CorrectAnswer-D
Ans-is'd'i.e.,2.5(Ref:Harp*2*/ep.129-130)
Inglycerophosphateshuttle,themitochondrialennlmeislinked
torespiratorychain(ETC)viaaflavoprotein,Soonlyl.5molof
ATPareproduced(Accordingtooldercalculations,2ATPmol
ofATPareproduced).
Inmalateshuttle,themitochondrialenzymeislinkedtoETCvia
NAD,so2.5molofATPareproduced(accordingtoolder
calculations3molofATPareproduced).

303.Notasubstrateforgluconeogenesis-
a)Glycerol
b)Leucine
c)Lactate
d)Propionate
CorrectAnswer-B
Ans.is'b'i.e.,Leucine
Substratesforgluconeognesis?

1. Lactate
2. Allaminoacidsexceptleucineandlysine
3. Pyruvate
4. Propionate
5. Glycerol
6. Intermediatesofcitricacidcycle
Alanineisthemostimportantgluconeogenicaminoacid.

304.Pyruvatedehydrogenaserequiresall
cofactorsexcept
a)Thiamin
b)Riboflavin
c)Niacin
d)Pyridoxin
CorrectAnswer-D
Ans.is'd'i.e.,Pyridoxin[RefHarper29thlep.176]
Pyruvatedehydrogenasecatalysesoxidativedecarboxylationof
pyruvatetoacetylCoA.
ThecoenzymerequiredbyPDHare:-
1. Thiaminepyrophosphate
2. Riboflavin(FAD)
3. CoA
4. Niacin(NAD)
5. Lipoicacid

305.Lactateproducedanaerobicallyisused
by
a)Gluconeogenesis&Glycolysis
b)Coricycle&gluconeogenesis
c)TCAcycle&Glycogenolysis
d)Coricycleonly
CorrectAnswer-B
Ans.is'b'i.e.,coricycle&gluconeogenesis[RefLehninger
4h/ep.523,53g-391
Coricycleorlacticacidcycle
Anaerobicglycolysisinmusclesresultsintheproductionof
lactate,whichcannotbeconvertedintoglucose,
asgluconeogenesisdoesnotoccurinmuscles.
Throughblood,Lactateistransportedtotheliverwhereitis
oxidizedtopyruvate.pynrvatesoproduced,isconverted
toglucosebygluconeogenesis,whichisthentransportedto
themuscle.
Theglucosethusreformedfromlactateagainbecomes
availableforenergypurposeinskeletalmuscle.

306.Citratesynthaseisinhibitedby-
a)ATP
b)ADP
c)Insulin
d)Glucagon
CorrectAnswer-A
Ans.is'a'i.e.,ATP[RefChatterjee&ShindeVh/ep.171,166-
180;Harper28th/ep.145-147


307.Fumarateisformedfromwhichamino
acid
a)Methionine
b)Valine
c)Histidine
d)Tyrosine
CorrectAnswer-D
Ans.is'd'i.e.,Tyrosine[RefHarper29th/ep.166-67]
Allmajormembersofthecitricacidcyclefromcitrateto
oxaloacetateareglucogenicandtherefore,areinvolvedin
gluconeogenesis.Someglucogenicaminoacidsentersthe
TCAcycleaftertransaminatione.g.:

1. Histidine,proline,glutamineandarginineareconvertedtoglutamate
whichisthentransaminatedtoa-ketoglutarate.
2. Isoleucine,methionineandvalineenterbyconversionintosuccinyl
CoA.Propionate(ashortchainfattyacid)alsoenteratthislevel.
3. Tyrosine,andphenylalanineenterbyconversionintofumarate.
4. Tryptophanisconvertedtoalaninewhichisthentransaminatedto
pyruvate.
5. Hydroxyproline,serine,cysteine,threonineandglycineenterby
conversionintopyruvate.

308.Themajorroleof2,3
bisphosphoglycerateinRBCsis-
a)Acid-basebalance
b)Reversalofglycolysis
c)Releaseofoxygen
d)Bindingofoxygen
CorrectAnswer-C
Ans.is'c'i.e.,ReleaseofOxygen[RefLehningerilthiep.171]
RapoportLueberingcycle(Bisphosphoglycerateshunt)
Thiscycleoccursinerythrocytes(ABCs).
InthisproductionofATPbysubstratephosphorylationfrom1,3-BPG
isbypassedbytakingdiversionpathways,i.e.,sidereactionof
glycolyticpathway.
Inthiscycle,1,3-BPGisconvertedto2,3BPGbyanenzyme
bisphosphoglyceratemutase.Then2,3BPGisconvertedto3-
phospholycerateby2,3-bisphosphoglyceratephosphatase.

309.EnzymedeficientinHersdisease-
a)Musclephosphorylase
b)Liverphosphorylase
c)Acidmaltase
d)Debranchingenzyme
CorrectAnswer-B
Ans.isB'i.e.,Liverphosphorylase[RefHarper250/ep.181]
Organ(s)
Type
Enzymedeficiency
affected
vonGierke's
I
Glucose6-phosphatase
Liver,kidney
disease
alpha(14)Glucosidase
II
Pompe'sdisease
Allorgans
(acidmaltase)
Cori's
III
disease/Forbe's
Debranchingenzyme
Muscle,liver
disease
Liver,
IV
Andersen'sdisease Branchingenzyme
myocardium
V
McArdle'sdisease
Phosphorylase
Muscle
VI
Hers'disease
Phosphorylase
Liver
Muscle,
VII
Tarui'sdisease
Phosphofructokinase
RBCs
VIII
Phosphorylasekinase
Liver

310.Immediatemetabolicproductsduring
conversionofFructus1-6bisphosphate
to2moleculesofpyruvate-

a)Glyceraldehyde-3-phosphateand1,3-bisphosphoglycerate
b)Dihydroxyacetonephosphateand1,3bisphosphoglycerate
c)Glyceraldehyde-3-phosphateanddihydroxy-acetonephosphate
d)3-phosphoglycerateand1,3bisphosglycerate
CorrectAnswer-C
Ans.is'c'i.e.,Glyceraldehyde-3-phosphateand
dihydroxyacetonephosphate(Ref:Harper29h/ep.170-177).


311.Inconversionofglucosetoglucose-6-
phosphateinglycolysistrueis
a)Glucokinasehaslowkm
b)Hexokinaseisfoundonlyinliver
c)Glucokinaseisinducedbyinsulin
d)Hexokinaseisnotspecificforglucose
CorrectAnswer-C:D
Ans.is'c'i.e.,Glucokinaseisinducedbyinsulin&'d'i.e.,
Hexokinaseisnotspecificforglucose(Ref:Harper29h/ep.
170-177;Vasudevan#/ep.98)
Hexokinaseisnotspecificforglucosemetabolism.Itisfound
inmosttissuesexceptglucose.
GlucokinasehashighKM.Itisinducedbyinsulin.

312.Majorcarbohydratestoreinthebody-
a)Bloodglucose
b)Glycogeninadiposetissue
c)Hepaticglycogen
d)Noneoftheabove
CorrectAnswer-C
Ans.is'c'i.e.,Hepaticglycogen(Ref:Harper29/ep.161;
Dineshpuri3d/ep.320)
MajorCarbohydratesourceofbodyishepaticglycogen.
Humanscarrysuppliesoffuelwithintheirbody.Caloriesare
storedinthebodyasfat(triglycerides),glycogenandsome
protein.

313.Mostabundantsourceoffuelin
starvation-
a)Liverglycogen
b)Muscleglycogen
c)Adiposetissue
d)Bloodglucose
CorrectAnswer-C
Ans.is'c'i.e.,Adiposetissue[RefHarper29thlep.161&28thle
p.134,140;Vasudevan6th/ep.84,85]
Fat(triglycerides)intheadiposetissueisthelargeststoreofenergy
ofthebody.

314.Majormetabolismofsaturatedfatty
acidsinthemitochondriaiscalledas-
a)-oxidation
b)-oxidation
c)-oxidation
d)Noneoftheabove
CorrectAnswer-A
Ans.is'a'i.e.,-oxidation[RefHarper29"'/ep.208]
-oxidationistheprincipalpathwayforcatabolismofsaturatedfatty
acids.P-oxidationmainlyoccursinthemitochondrialmatrix
(whereasfattyacidsynthesisoccursincytosol).

315.Bileacidsaresynthesizedfrom?
a)Heme
b)Cholesterol
c)Ribulose
d)Arachidonicacid
CorrectAnswer-B
Ans.is'b'i.e.,Cholesterol
Primarybileacidsarecholicacidandchenodeoxycholicacid,which
aresynthesizedfromcholesterolinliver.
Intheintestinesomeoftheprimarybileacidsareconvertedinto
secondarybileacids,i.e.,deoxycholicacid(formedfromcholicacid)
andlithocholicacid(derivedfromchenodexoxycholicacid).
Glycineandtaurineconjugatesofthesebileacidsarecalledasbile
salts.
Forexample,cholicacidisabileacid,anditsglycineconjugate
(glycocholicacid)isabilesalt.
Bilesaltshelpindigestionandabsorptionoffatbyemulsification
andmicellesformation.
Bilesaltsactasdetergents,i.e.,theyhavesurfacetensionlowering
action.
Detergentactionisduetoamphipathicnatureofbilesalts(Note:
Amphipathicmoleculesaremoleculesthatcontainbothhydrophobic
non-polaraswellashydrophilic-polarends).

316.Whichmethodisusedtoseparatea
mixtureoflipids-
a)Electrophoresis
b)Chromatography
c)Isoelectricfocusing
d)PAGE
CorrectAnswer-B
Ans.is'b'i.e.,Chromatography[RefClinicalbiochemistryp.
719]
Extractedlipidsareseparatedintoindividualclassby
chromatography.
Chromatographycanseparateacomplexmixtureoflipidsinto
simplergroup.
Varioustypesofchromatography,usedtoseparate,lipidsare-
Adsorption(solid-liquid)chromatography
Partition(liquid-liquid)chromatography
Thinlayerchromatography(TLC)
Gaschromatography(GC)
Highperformanceliquidchromatography(HPLC)->methodof
choiceifavailable.

317.Transportoflipidsfromtheintestineto
othertissuesisby-
a)Chylomicrons
b)LDL
c)HDL
d)VIDL
CorrectAnswer-A
Ans.is'a'i.e.,Chylomicrons
Transportoflipids
oTherearetwopathwaysoflipidtransportinthebody?
1.Transportofdietrylipid(Exogenouspathway)
*Itisthetransportof1ipid.fromintestinetoliver.
-Chylomicronstransportthedietrylipidfromintestinetoliver.
*InDietthemajorlipidsaretriglycerideandAolesterol.
*Cholesterolisabsorbedassuchinproximalsmallintestineandis
esterifiedtocholesterylester(ChE).oTriglyceridesarehydrolysed
bylipasestoglycerolandfattyacidswhichareabsorbedinintestine.
oInsideintestinalcellstriglycerideissynthesizedbyfattyacids.
-Chylomicronsaresynthesizedinthesmallintestinethatcontain
triglyceride,cholesterylester,cholesterol,phospholipidsand
apoproteinB-48(apoB-48).
*Thesechylomicronsaresecretedintheintestinallymphandreach
thesystemiccirculationviathoracicduct.
-InthecirculationApoEandApoCaretransferedto
chylomicronesbyHDL,sonowchylomicronscontainApoB-48,Apo
E,&ApoC.
*Inthecirculation,Triglyceridesofchylomicronesarehydrolysed

bylipoproteinlipase(LPL)presentonendothelialcellsofvessels
ofskeletalmuscles,adiposetissueandheart.
*Thereleasedfattyacidsareutilizedlocallybythesetissues.
*Thechilomicronparticleprogressivelyshrinksinsizebyactionof
LPLand,cholesterol,phospholipidsandapoCaretransferredto
HDL,creatingchylomicronremnantsthatcontainsmorecholesterol,
lesstriglycerides,apoE&apoB-48.
*ChylomicronremnantsareremovedbyliverbytheLDL
receptors
thatrequireapoEasligand.
Note-ApoC-II,thatistransferedfromHDLtochylomicronsactasa
cofactorforlipoproteinlipase.ApoA-VpromotesLPLmediated
triglyceridelipolysisinVLDLandchilomicrons.
2.TransportofHepaticlipids(Endogenouspathway)
*VLDLissynthesizedinliverthatcontainshightriglyceride,ChE,
cholesterol,phospholipidandApoB-100.
*(VLDLparticlesresembelchylomicronesincompositionexceptthat
VLDLcontainsApoB-100insteadofApoB-48).
*VLDLparticlesaresecretedintheplasmaandaswith
chylomicron,ApoEandApoCaretransferredfromHDLtoVLDL.
NowVLDLcontainsApoB-100,ApoEandApoC.
*Inplasma,triglyceridesofVLDLarehydrolysedbysame
lipoproteinlipase(seeabove)andapoCistransferredtoHDLand
theremmantsarecalledIDL.
*40-60%ofIDLisremovedbyliverviaLDLreceptormediated
endocytosis,thisprocessrequireApoEwhichactsasligandforLDL
receptors.
*Remaining1DLisremodeledbyhepaticlipasetoformLDLthat
containsmaximumcholesterol.
*70%ofLDLisremovedbyliverviaLDLreceptorand30%is
utilizedbyperipheraltissuesasasourceofcholesterol.

318.Chylomicronscoreisformedby?
a)Triglyceride
b)TriglycerideandCholesterol
c)Triglyceride,CholesterolandPhospholipids
d)Freefattyacids
CorrectAnswer-B
Ans.is'B'i.e.,TriglycerideandCholesterol[RefHarper27th/ep.
218;Lehninger4th/ep.633]
Thesurfaceisalayerofphospholipids,withheadgroupsfacingthe
aqueousphase.Triacylglycerolssequesteredintheinteriormakeup
morethan80%ofthemass.
Severalapolipoproteinsthatprotrudefromthesurface(B-48,C-III,
C-II)actassignalsintheuptakeandmetabolismofchylomicron
contents.
Thediameterofchylomicronsrangesfromabout100to500nm.
Core:It'smadeupofneutrallipidsliketriacylglycerolsand
cholesterol/cholesterolesters.Shell:composedofapolipoproteins,
phospholipids.

319.(-oxidationofpalmiticacidyields
a)3acetylCoA
b)129ATPnet
c)131ATPnet
d)16AcetylCoA
CorrectAnswer-B
Bi.e.129ATPnet

320.AllaretrueaboutNiemann-Pickdisease
except-
a)Duetodeficiencyofsphingomyelinase
b)CNSsymptomsintypeA
c)HistiocytesshowingPASpositiveinclusionsandTypeBisless
severe
d)None
CorrectAnswer-D
Ans.isD.None[RefClinicalbiochemistry4th/ep.786]
Niemann-Pickdiseaseisanautosomalrecessive`lysosomalstorage
diseaseduetodeficiencyofsphingomyelinase.
CharacteristichistopathologicalfeatureishistiocytesshowingPAS
positivediastaseresistantinclusionswhichonmicroscopyshows
concentricorparrallellamellararrangement.
Clinicalcasesaredividedinto-
1. TypeA:Thesearemorecommonwithmoreseveredeficiencyof
sphingomyelinase.ThereisvisceralandCNSinvolvement.
Symptomsmaypresentsincebirthanddeathusuallyoccursbefore
theageof4years.
2. TypeB:Thereislessseveredeficiencyofsphingomyelinase.
Patient'shaveonlyvisceralinvolvementbutnoCNSinvolvement.
Patientspresentbytheageof3-4yearswithorganomegalyand
mayremainreasonablyhealthy.

321.Plasminogendomainresembles
a)Fibrinogen
b)LDLreceptor
c)Apolipoprotein(a)
d)Prothrombin
CorrectAnswer-C
Apolipoprotein(a)REF:
http://onlinelibrary.wiley.com/doi/10.1002/pro.5560031222/pdf,http://www.ncbi.nlm.nih.gov/pubmed/7756992
Phylogenyoftheserineproteinasedomainsandanalysisofintron-
exonboundariesandKringlesequencesindicatethathepatocyte
growthfactor/scatterfactor(HGF/SF),hepatocytegrowthfactor-
like/macrophagestimulatingprotein(HGFVMSP),plasminogen,and
apolipoprotein(a)haveevolvedfromacommonancestralgene.The
nameKringlecomesfromtheScandinavianpastrythatthese
structuresresemble.

322.AcetylCoACarboxylaseisstimulatedby
-
a)Starvation
b)Glucagon
c)Citrate
d)Alloftheabove
CorrectAnswer-C
Ans.is'c'i.e.,Citrate[RefHarper29th/ep.219]
Activator Inhibitor
Allosteric(palmitoylCoA) Citrate
Longchainacyl-CoA
Covalent
Insulin
Glucagon,epinephrine

323.Whichofthefollowingenzymeisnota
componentoffattyacidsynthase
complex?

a)AcetylCo-Acarboxylase
b)Ketoacylsynthase
c)Enoylreductase
d)Acetoacetyl
CorrectAnswer-A
TheFattyAcidSynthaseComplexIsaHomodimerofTwo
PolypeptideChainsContainingSixEnzymeActivitiesandtheAcyl
CarrierProtein.
The6enzymesare-
Ketoacylsynthase
Ketoacylreductase
Malonyltransacylase
Dehydratase
Enoylreductase
Thioesterase
AcetylCo-Acarboxylaseistherate-limitingenzymeoffattyacid
synthesisandisanenzyme,whichisnotacomponentoffattyacid
synthasecomplex.


324.Cholesterolisnotaprecursorfor
synthesisof-
a)VitaminD
b)Progesterone
c)Bileacids
d)Lipocortin
CorrectAnswer-D
Ans.is'd'i.e.,Lipocortin
Cholesterolisanimportantprecursorforthesynthesisof:-

1. Steroidhormones:-Progesterone,estrogen,,androgens,
glucocorticoids,mineralocorticoids
2. VitaminD
3. Bileacids

325.HDLiscalledgoodcholesterolbecause-
a)Removescholesterolfromextrahepatictissues
b)Causestransportofcholesteroltoextrahepatictissues
c)StimulatehepaticTGssynthesis
d)Activateslipoproteinlipase
CorrectAnswer-A
Ans.is'a'i.e.,Removescholesterolfromextrahepatictissues
TheHDLparticlesarereferredtoasscavengersbecausetheir
primaryroleistoremovefree(unesterified)cholesterolfrom
extrahepatictissues,whichisthenexcretedthroughbile.Thisisa
crucialmechanismthatpreventstheinappropriateaccumulationof
cholesterolinperipheraltissues.Becauseaccumulationof
cholesterolintissuesisstronglyassociatedwiththedevelopmentof
atherosclerosis,thelevelofHDLinserumisinverselyrelatedtothe
incidenceofMI(myocardialinfarction).Thus,HDLis
cardioprotectiveandanti-atherogenicinnature,andisreferredtoas
"goodcholesterol".
Therefore,HDL-Cholesterolappearstobethebestindependent
predictorofcoronaryarterydisease(inverserelationship)thanany
otherknownriskfactor.ThatmeanslowHDLisamuchstronger
predictorofcoronaryarterydiseasethanincreasedLDLcholesterol
orincreasedtotalcholesterol.

326.

Amidegroupispresentinwhichpartof
protein-

a)Amino-terminal
b)Carboxy-terminal
c)Peptidebond
d)Disulfidebond
CorrectAnswer-C
Ans.is'c'i.e.,Peptidebond[RefTextbookofbiochemistryby
Talwar-p.30]
Apeptidebondisachemicalbondthatconnectsaminoacidsto
eachother.
Apeptidebondessentiallyresultsfromadehydrationsynthesis
reaction.
Itisformedbetweentwoaminoacidswhenthecarboxylgroupof
oneaminoacidreactswiththeaminogroupoftheother,releasinga
moleculeofwater(H20).
PeptidebondistheresultingCO-NHbondandtheresulting
moleculeisanamide.
Thefour-atomfunctionalgroup-C(=0)NH-iscalledanamide
grouporpeptidegroup.
Peptidebondisapartialdoublebond.
Partialdoublebondnaturerenderstheamidegroupplanarandrigid,
makingalltheatomsthatareinvolvedinthepeptidebondlieinaflat
plane.

327.Non-essentialamino-acidsareallexcept
-
a)Basicaminoacids
b)Acidicaminoacids
c)Neutralaminoacids
d)Noneoftheabove
CorrectAnswer-A
Ans.is'a'i.e.,Basicaminoacids[RefHarper29th/ep.266]
There,arethreebasicaminoacids(Arginine,Histidineandlysine),
allofthemareessentialaminoacids
Essentialaminoacids(PVT.TIM.HALL):Phenylalanine,valine,
threonine,tryptophan,isoleucine,methionine,histidine,arginine,
lysine,leucine.
Amongthese,arginineandhistidinearesemiessentialaminoacids.
Nonessentialaminoacids:alanine,asparticacid,aspargine,
cysteine,glutamine,glutamicacid,glycine,proline,tyrosine,serine.
Neutralaminoacids:Alanine,asparagine,cysteine,glycine,
glutamine,isoleucine,leucine,methionine,proline,phenylalanine,
serine,threonine,tyrosine,tryptophan,valine.
Acidicaminoacids(negativelychargedoranion):Asparticacid
(aspartate),glutamicacid(glutamate).
Basicaminoacids(positivelychargedorcation):Arginine,histidine,
lysine.

328.Aminoacidwhichcanbeusedinboth
gluconeogenesisandketogenesis-
a)Threonine
b)Valine
c)Tyrosine
d)Arginine
CorrectAnswer-C
Ans.is'c'i.e.,Tyrosine[RefChatterjee5''/ep.448]
Aminoacidswhichcanbeusedbothingluconeogenesisand
ketogenesis(Bothglucogenic&ketogenicaminoacids)are:-

1. Tyrosine
2. Phenylalanine
3. Tryptophan
4. Isoleucine.

329.Inphenylketonuria,dietrestrictionis
advisedfor-
a)Tyrosine
b)Phenylalanine
c)Maize
d)All
CorrectAnswer-B
Ans.is'b'i.e.,Phenylalanine,Themaintreatmentisa
phenylalaninerestricteddietforlife.


330.Guanidiniumgroupisassociatedwith-
a)Tyrosine
b)Arginine
c)Histidine
d)Lysine
CorrectAnswer-B
Someaminoacidscontainaspecialfunctionalgroupintheir
sidechainwhichprovidessomespecificfunctionstothat
aminoacids.Theseare

1. Hydroxylgroupinserineandthreonine
2. Indoleringintryptophan
3. Amidegroupinaspargineandglutamine
4. 13-Carboxylinglutamicacid
5. Thioetherinmethionine
6. Imidazoleinhistidine
7. Sulfhydrylincysteine
8. Phenolintyrosine
9. y-carboxylinglutamicacid
10. Pyrrolidineinproline
11. Guanidiniuminarginine
12. c-aminoinlysine
13. Benzeneinphenylalanine

331.Creatinineisformedfrom-
a)Glycine
b)Lysine
c)Leucine
d)Histamine
CorrectAnswer-A
Ans.A.Glycine
Creatinineandcreatinearesynthesizedfromglycine,arginine,and
methionine.
Synthesisofcreatineandcreatinine
Creatineandcreatininearenotaminoacids,butspecialized
productsofaminoacids.Creatineissynthesizedfromglycine,
arginine,andmethionine.Thesynthesisstartswiththeformationof
guanidinoacetatefromglycineandarginineinthekidney.Further
reactionstakeplaceintheliverandmuscle.

332.Coenzymeforphenylalaninehydroxylase
is-
a)Tetrahydrofolate
b)Pyridoxalphosphate
c)S-adenosylmethionine
d)Tetrahydrobiopterin
CorrectAnswer-D
Ans.is'd'i.e.,Tetrahydrobiopterin
Phenylalaninemetabolismisinitiatedbyitsoxidationtotyrosine
whichthenundergoesoxidativedegradation

333.Glycineisrequiredinformationofall
except?
a)Heme
b)Purines
c)Glutathione
d)Thyroxine
CorrectAnswer-D
Ans.is'd'i.e.,Thyroxine
Glycine
lycineisanonessentialaminoacidwhichissynthesizedfromserine.
Glycineismetabolizedbyfollowingthreepathways-
Itcanbeconvertedtoserine,areactionthatrequiretetrahydrofolate
(derivativeoffolicacidandpyridoxalphosphateascoenzyme.
Furtherserineismetabolizedbyserinedehydrataseintopyruvate
andNH4+.
Themajorpathwayofglycinedegradationisoxidativecleavageinto
CO,andNH:byglycinecleavagecomplexofliver.H4folateis
requiredwhichisconvertedtoN5,N10-methyleneH4folate.Thus
folicacidisrequiredforglycinemetabolism.
Glycinemaybeoxidativelydeaminatedbyglycineoxidaseto
glyoxylicacid.
Glycineisnecessaryfortheformationoffollowingproducts:-Heme,
purinering,bileacidsconjugation(formationofglycocholicacid,and
glyco-chenodeoxycholicacid),creatine,glutathione,glucose(by
gluconeogenesis).

334.Whichofthefollowingisrequiredinthe
synthesisofacetylcholine-
a)Inositol
b)Carnitine
c)Glycine
d)Choline
CorrectAnswer-D
Ans.is'd'i.e.,Choline[RefPrinciplesofmedicalphysiologyp.
96]
Acetylcholineissynthesizedinthecytosolofnerveterminalfrom
acetyl-CoAandcholine,inthepresenceofcholine-0-
acetyltransferase

335.Mostimportantfactorwhichcauses
lacticacidosisinalcoholics-
a)PruductionofNADH
b)Formationofacetaldehyde
c)Productionofacetate
d)Noneoftheabove
CorrectAnswer-A
Ans.is'a'i.e.,ProductionofNADH[RefPankajNaik3rd/ep.
351]
ExcessintakeofalcoholleadstoexcessiveproductionofNADHwith
aconcomitantdecreaseinNAD..
TheincreasedavailabilityofNADHfavoursthereductionofpyruvate
tolactateandoxaloacetatetomalateanddecreasingitsavailability
forgluconeogenesisanddecreasesynthesisofglucose.Thisresults
inHypoglycemia.
Excessoflactateproductionleadstolacticacidosis

336.Respiratoryquotientofcarbohydrateis:
a)0.5
b)0.8
c)0.75
d)1
CorrectAnswer-D
Ans:D.1
Therespiratoryquotient(orRQorrespiratorycoefficient),is
adimensionlessnumberusedincalculationsofbasalmetabolic
rate(BMR)whenestimatedfromcarbondioxideproduction.Itis
calculatedfromtheratioofcarbondioxideproducedbythebodyto
oxygenconsumedbythebody.Suchmeasurements,like
measurementsofoxygenuptake,areformsofindirectcalorimetry.It
ismeasuredusingarespirometer.
Therespiratoryquotient(RQ)istheratio:
RQ=CO2eliminated/O2consumed

337.Insulindependantcellsare-
a)Pituitocytes
b)Myocytes
c)Adipocytes
d)RBCs
CorrectAnswer-B:C
Ans.is'b'i.e.,Myocytes&'c'i.e.,Adipocytes[RefGanong
24th/ep.435]
Insulinstimulatestheuptakeofglucosebymyocytes(skeletal
muscle,cardiacmuscle),adipocytes(adiposetissue)and
hepatocytes.Tissuesthatdonotdependoninsulinforglucose
uptakeincludebrain,erythrocytes(RBC),theepithelialcellsof
kidney&intestine,Liver,andCornea&lensoftheeye.
Intheliver,insulinstimulatesglucoseentryintohepatocytes
indirectlybyinductionofglucokinasesothattheglucoseentering
thelivercellsispromptlyconvertedtoglucose-6-phosphate
(glucosetrapping).Thiskeepstheintracellularglucose
concentrationlowandfavoursentryofglucoseintotheliver.Thus,
thoughtheliverdonotdependoninsulinforglucoseuptake,insulin
stimulatesglucoseentryintohepatocytes.Thatmeansglucoseentry
canoccurinliverwithouttheactionofinsulin,butthisisfacilitatedby
insulin.Ontheotherhand,myocytes(skeletalandcardiacmuscles)
andadipocytes(adiposetissue)aredependentoninsulinfor
glucoseuptake.

338.Epinephrineincreasesfreefattyacid
levelbycausing-
a)Increasedfattyacidsynthesis
b)Increasinglipolysis
c)Increasingcholesterolcatabolism
d)Alloftheabove
CorrectAnswer-B
Ans.is'b'i.e.,Increasinglipolysis[RefPankajNaik%talep.248]
Epinephrineandglucagonacceleratelipolysisinadiposetissueby
activatinghormonesensitivelipase.Instarvationanddiabetes,
glucagonishigh(andinsulinislow),leadingtoenhancedlipolysis.

339.Energysourceusedbybraininlater
daysofStarvationis
a)Glucose
b)Ketonebodies
c)Glycogen
d)Fattyacids
CorrectAnswer-B
Bi.e.Ketonebodies

340.Whichofthefollowingenzymeactivity
decreasesinfasting?
a)Hormonesensitivelipase
b)Glycogenphosphorylase
c)AcetylCoACarboxylase
d)PhosphofructokinaseI
CorrectAnswer-D
Ans:D.PhosphofructokinaseI
Ans:D.PhosphofructokinaseI
Phosphofructokinase1(PFK1)providesthefirstenzymaticstepat
whichaglucosemoleculebecomescommittedtoglycolysisand
thereforeissubjecttoregulation(Nelson&Cox,2008).PFK1
activitydependsontheconcentrationsofAMP,ADP,andATP
withallostericactivationbyAMPandADPandallosteric
inhibitionbyATP.
Inthefastingstate,glucagoncausesthelivertomobilizeglucose
fromglycogen(glycogenolysis)andtosynthesizeglucose
fromoxaloacetateandglycerol(gluconeogenesis).
GlucagonstimulatesanincreaseincAMP,leadingtoanincreasein
phosphorylationbyproteinkinaseA.
Thewaveofphosphorylationthatspreadsthroughthelivercell
activatesenzymessuchasglycogenphosphorylasethatare
involvedinglycogendegradationwhilesimultaneouslyinhibiting
glycogensynthesis.
Inhibitionofglycogensynthasepreventsfutileresynthesisof
glycogenfromglucose1-phosphate(G1P)viaUDP-Glc.Glucose-6-
phosphatase(G6Pase),agluconeogenicenzymethatispresentin
theliverbutnotinmuscle,thenconvertsG6Ptoglucoseforrelease

intotheblood.
Ref:Harper'sillustratedbiochemistry,3a,ed.,pg.I88andLippincott's
illustratet!reviewsr6't'ed.,pg.107


341.Vitaminformedinthebody-
a)B1
b)B3
c)B6
d)B12
CorrectAnswer-A:B:D
Ans.is'b>a&d'i.e.,B3>B1&B12
Thisquestionisconfusingone(Readtextbelow).
Niacin(VitaminB3)issynthesizedfromtryptophaninsidethe
body(endogenous).
Somevitaminsarealsoformedbybacterialactivityincolon:-
1. VitaminK
2. Riboflavin(VitaminB2)
3. VitaminB12
4. Biotin(VitaminB7)
5. Thiamin(VitaminB1)
Thus,optionsa,b&dallarecorrecthere.Butthebestanswer
amongtheseisniacinasitistheonlyvitaminwhichissynthesized
byproperanabolicmetabolism.

342.Whichofthefollowingisnotseenin12
daysoffasting-
a)Gluconeogenesis
b)Ketogenesis
c)Lipolysis
d)Glycolysis
CorrectAnswer-D
Ans.isi.d.e.,Glycolysis[RefHarper28'Vep.140;Dineshpurl
3'/ep.414]

Intermediate
Latestate{>24
Duration
Earlystage
stage(3-24d) d}
Ketonebody
Glycogenolysis
oxidationBybrain
Gluconeogenesis
Gluconeogenesis
Lipolysis
Pathways
Lipolysis
Byothertissues
Ketogenesis
enhanced
Ketogenesis
Fattyacid
Ketonebody
Protein
utilization
oxidation
degradation
Gluconeogenesis
Glycolysis
TCA
Protein
Pathways
Proteinsynthesis
degradation
slowed
anddegradation
Proteinsynthesis Glycogensynthesis
anddegradation

343.Pyridoxineisrequiredin-
a)Glycolysis
b)TCAcycle
c)Glycogenesis
d)Glycogenolysis
CorrectAnswer-D
Ans.is'd'i.e.,Glycogenolysis[RefDineshpurip.187;Harper
29th/ep.180&27th/ep.159-160]
PLPisacofactorforglycogenphosphorylase,henceitfavors
glycogenolysis.

344.Pruritis[Itching]iscausedbydeficiency
of-
a)HMBsynthase
b)5-ALAdehydratase
c)Uroporphyrinogen-Isynthase
d)Uroporphyrinogen-IIIsynthase
CorrectAnswer-D
Ans.is'd'i.e.,Uroporphyrinogen-IIIsynthase[RefRooks7th/e
p.12.7,12.8]
Cutaneous(erythropoietic)porphyriascauseskinmanifestations
likephotosensitivity,rashandpruritus.
Amongthegivenoptions,UroporphyrinegenIIIsynthasedeficiency
causescutaneousporphyria(Congenitalerythropoieticporphyria).

345.Vitaminactingonintranuclearreceptors
-
a)VitaminK
b)VitaminD
c)VitaminE
d)VitaminE
CorrectAnswer-B
Ans.is'b'i.e.,VitaminD[RefUnderstandingsmedical
physiologyp.408]
Twovitaminsareconsideredashormoneswhichacton
intranuclearreceptors(steroidreceptorfamily).Thereare:-

1. VitaminA(retinoicacid)
2. VitaminD

346.Whichofthefollowinghasantioxidant
property?
a)Selenium
b)Copper
c)Zinc
d)All
CorrectAnswer-D
Ans.is'd'i.e.,All
Theactivityoftheantioxidantenzymesdependsonsupplyof
minerals:?
1. Manganese
2. Copper
3. Zinc
4. Selenium
Manganese,copperandzincarerequiredfortheactivityof
superoxidedismutase.
Seleniumisrequiredfortheactivityofglutathioneperoxidase.

347.Niacinactsascoenzyme-
a)TPP
b)FAQ
c)NAD
d)NAD
CorrectAnswer-C
Ans.is'c'i.e.,NAD
Niacin,intheformofnicotinamide,isincorporatedintothestructure
oftwocoenzymes:nicotinamideadeninedinucleotide(NAD*)and
nicotinamideadeninedinucleotidephosphate(NADI)*).

348.Coenzymenotrequiredinformationof
glutamate-
a)Thiaminepyrophosphate
b)Pyridoxialphosphate
c)Niacin
d)Noneoftheabove
CorrectAnswer-A
Ans.'A'Thiaminepyrophosphate
NAD+isderivedfromnicotinicacid,amemberofthevitaminB
complex,usedinthesynthesisofGABA.
Duringtransaminationreactionglutamateisformed.Pyridoxal
Phosphateactsasacoenzyme.

349.Notametabolicproductofureacycle-
a)Citrulline
b)Ornithine
c)Alanine
d)Arginine
CorrectAnswer-C
Ans.is'c'i.e.,Alanine
Metabolicproductsinureacyclearecarbamoylphosphate,
ornithine,citrulline,argininosuccinate,arginineandfumarate.
Biosynthesisofureaoccursinfivesteps
1-Carbamoylphosphatesynthetase-I(CPS-I),amitochondrial
enzyme,catalyzestheformationofcarbamoylphosphateby
condensationofCO,andammonia.
2-Ornithinetranscarbamoylasecatalyzestheformationofcitrulline
fromcarbamoylphosphateandornithine.
3-Argininosuccinatesynthasecatalyzestheformationof
argininosuccinatefromcitrullineandaspartate.Thisreaction
requires!ATP,
4-Argininosuccinatelyase(arginosuccinase)catalysesthecleavage
ofargininosuccinateintoarginineandfumarate.Fumarateenters
TCAcycle.
5-)Arginasecatalysestheformationofureafromarginineby
hydrolyticcleavageofargininetoyieldureaandornithine.Ornithine
isthusregeneratedandcanentermitochondriatoinitiateanother
roundoftheureacycle.

350.Whichaminoacidisnotinvolvedin
transamination-
a)Alanine
b)Aspartate
c)Lysine
d)Histidine
CorrectAnswer-C
Ans.is'c'i.e.,Lysine
Mostaminoacidsundergotransaminationreactionexceptlysine,
threonine,prolineandhydroxyproline.

351.Boiledcabbageorrancidbutter
smellingurineisseenin
a)Phenylketonuria
b)Tyrosinemia
c)IsovalericAcidaemia
d)Multiplecarboxylasedeficiency
CorrectAnswer-B
Tyrosinemia
REF:Sapira'sart&scienceofbedsidediagnosis-Page138,
TextbookofPediatricEmergencyMedicinebyGaryR.Fleisher,
StephenLudwigPage1566
Tyrosinaemiaisanerrorofmetabolism,inherited,inwhichthebody
cannoteffectivelybreakdowntheaminoacidtyrosine,foundinmost
animalandplantproteins.Itisanautosomalrecessive,which
meanstwocopiesofanabnormalgenemustbepresentinorderfor
thistodevelop.Therearethreetypesoftyrosinemia,eachwith
distinctivesymptomsandcausedbythedeficiencyofadifferent
enzyme.OneofthesymptomsofTyrodinaemiatype1isanodor
likecabbageorrancidbutter.

352.ApoB48issynthesizedin-
a)Liver
b)Kidney
c)Intestine
d)RBCs
CorrectAnswer-C
Ans.is'c'i.e.,Intestine
ItmediateuptakeofLDLbyLDLreceptorsofliver,

353.Allantoinistheendproductof
metabolismof?
a)Glycogen
b)Purine
c)Pyrimidine
d)Histidine
CorrectAnswer-B
Innon-primatemammals,endproductofpurinemetabolismis
allantoinduetopresenceofenzymeuricase.Uricaseconvertesuric
acidtoallantoin.
Humanslacktheenzymeuricase.Therefore,endproductofpurine
catabolisminhumansisuricacid.

354.Trueaboutsigmafactor?
a)Subunitof50sribosome
b)SubunitofDNApolymerase
c)SubunitofRNApolymerase
d)InitiatesDNAreplication
CorrectAnswer-C
Ans-'C'SubunitofRNApolymerase
ThebasicDNA-dependentRNApolymeraseofthebacterium
Escherichiacoliexistsasanapproximately400-kDacorecomplex
consistingoftwoidenticalsubunits,twolarge,andsubunits,
andasubunit.ThecoreRNApolymerase,2,oftentermed
E,associateswithaspecificproteinfactor(thesigma[]factor)to
formholoenzyme,2,orE.
Sigmafactorshaveadualroleintheprocessofpromoter
recognition;associationwithcoreRNApolymerasedecreasesits
affinityfornon-promoterDNA,whilesimultaneouslyincreasing
holoenzymeaffinityforpromoterDNA.

355.Replicationandtranscriptionaresimilar
processesinmechanistictermsbecause
both:

a)UseRNAprimersforinitiation.
b)Usedeoxyribonucleotidesasprecursors.
c)Aresemiconservedevents
d)Involvephosphodiesterbondformationwithelongation
occurringinthe5'-3'direction
CorrectAnswer-D
Di.e.Involvephosphodiesterbondformationwithelongation
occurringinthe5'-3'direction.
InbothDNAandRNAsynthesis,thegeneralstepsofinitiation,
elongationandterminationoccurin
5'-3'directionwiththeformation
ofphosphodiesterbondsQ.


356.Whatareokazzakifragments-
a)LongpiecesofDNAonlaggingstrand
b)LongpiecesofDNAonleadingstrand
c)ShortpiecesofDNAonlaggingstrand
d)ShortpiecesofDNAonleadingstrand
CorrectAnswer-C
Ans.is'c'i.e.,ShortpiecesofDNAonlaggingstrand[Ref:
Lippincott's5thiep.399,401,406]
DNApolymerasesresponsibleforcopyingtheDNAtemplatesare
onlyableto"read"theparentalnucleotidesequencein3'-5'
direction,andtheysynthesizethenewDNAstrandsonlyin5'-
3'direction.
Therefore,2newlysynthesizedchainsmustgrowinopposite
directions:?
1. TheDNAchainwhichrunsinthe3'45'directiontowardsreplication
forkascontinuedstrandiscalledtheleadingstrand.Thisrequires
onlyoneRNAprimer
2. TheDNAchainwhichrunsinthe5'43'directionawayfromthe
replicationforkiscalledlaggingstrand.Itissynthesized
discontinouslyandrequiresnumerousRNAprimers.Asthe
replicationforkmoves,RNAprimersaresynthesizedatspecific
intervals.TheseRNAprimersareextendedbyDNApolymeraseIII
intoshortpiecesofDNAcalledOkazakifragments.

357.Firstpurinenucleotide,whichis
synthesizedinpurinebiosynthesis?
a)AMP
b)GMP
c)IMP
d)UMP
CorrectAnswer-C
Thebiosynthesisofpurinebeginswithribose-5-phosphate,derived
frompentosephosphatepathway(PPP).
Firstintermediateformedinthispathway,5-phosphoribosyl-
pyrophosphate(PRPP),isalsoanintermediateinpurinesalvage
pathway.

358.C4,C5,N7inpurineringarederived
from-
a)Aspartate
b)Glutamine
c)Glutamine
d)CO
CorrectAnswer-C
Ans.is'c'i.e.,Glycine[RefHarper29th/ep.332]
Indenovosynthesis,purineringisformedfromavarietyof
precursorsisassembledonribose-5-phosphate.Precursorsforde
novosynthesisare?
1. GlycineprovidesC4,C5andN7
2. AspartateprovidesNI
3. GlutamineprovidesN3andN9
4. TetrahydrofolatederivativesfurnishC2andG
5. CarbondioxideprovidesC6

359.Mostimportanttoolusedingenetic
engineering
a)Halicase
b)Topoisomerase
c)DNALigase
d)Restrictionendonuclease
CorrectAnswer-D
Ans.is'd'i.e.,Restrictionendonuclease[RefSatyanarayanp.
579]
Geneticengineeringsimplymeansmanipulationofgeneticmaterial
toachievethedesiredgoalinapredeterminedway.
Mostimportantmoleculartoolsingeneticengineeringareenzymes
usedinrecombinantDNAtechnology.(alsocalledgenetic
engineering).
Mostimportantoftheseenzymesisrestrictionendonuclease.
AttheheartofgeneticengineeringistheabilitytocutDNA
moleculesatpreciselydefinedsites.
Thisisusuallydonewithpurifiedbacterialenzyme,therestrication
endonuclease.

360.IfcontentofAis15%,whatisthe
amountofGinDNAaccordingto
Chargaffsrule?

a)15%
b)85%
c)35%
d)70%
CorrectAnswer-C
Ans.is'c'i.e.,35%[RefHarper'sIllustratedbiochemistry27th/e
p.311-313]
Chargaff'srule:
Amountofpurine=Amountofpyrimidine(A+G=T+C)Also
A+G+T+C=100
SinceA=T,Therefore15+G+15+C=100
SinceG=C,Therefore30,+2G=100
2G=704G=35

361.ThegapsbetweensegmentsofDNAon
thelaggingstrandproducedby
restrictionenzymesarerejoinedsealed
by:

a)DNALigases
b)DNAHelicase
c)DNAtopoisomerase
d)DNAphosphorylase
CorrectAnswer-A
Ai.e.DNALigases

362.cDNAfromRNAissynthesizedby-
a)Helicase
b)DNAdependentDNApolymerase
c)Topoisomerase
d)Reversetranscriptase
CorrectAnswer-D
Ans.is'd'i.e.,Reversetranscriptase[RefSatyanarayan4th/ep.
550]
TheenzymeRNAdependentDNApolymerase(reverse
transcriptase)isresponsiblefortheformationofDNAfromRNA.
ThisDNAiscomplementary(cDNA)toviralRNAandcanbe
transmittedintohostDNA.

363.Whichisnotachaperonprotein-
a)Calnexin
b)Proteindisulfideisomerase
c)Calreticulin
d)Calbindin
CorrectAnswer-D
Ans.is'd'i.e.,Calbindin[RefHarper29thlep.598-599&28th/ep.
497]
SomeChaperonesandEnzymesInvolvedinFoldingthatare
LocatedintheRoughEndoplasmicReticulum:-
BiP(immunoglobulinheavychainbindingprotein)
GRP94(glucose-regulatedprotein)
Calnexin
Calreticulin
PDI(proteindisulfideisomerase)
PPI(peptidylprolylcis-transisomerase)

364.Allarefunctionsofglycosaminglycans
except-
a)Lubrication
b)Woundhealing
c)Anticoagulant
d)Transportoflipids
CorrectAnswer-D
Ans.is'd'i.e.,Transportoflipids[RefPankajNaik4th/ep.29]

365.Bilesaltshelpinabsorptionoffatby-
a)Micellesformation
b)Activationoftransporterprotein
c)Creationofconcentrationgradient
d)Alloftheabove
CorrectAnswer-A
Ans.is'a'i.e.,Micellesformation[RefGangong24thlep.465]
Emulsificationistheprocesswhichbreaksdowningestedfats
(mainlytriglycerides)intosmallerdropletssothattheycanbe
digestedmoreefficiently.Thusemulsificationmainlyhelpsin
digestionofingestedfats.
Micellesformationistheprocessinwhichdigestedfats(FFAsand
monoglycerides)areincorporatedintomuchsmallerdroplets
(micelles)sothattheycanbeabsorbedmoreefficiently.Thus,
micellesformationhelpsinabsorptionofdigestedfats.
Detergentactionofbilesaltsisnecessaryforbothemulsificationand
micellesformation.

366.a-aminolevulinicacidisametabolic
productinsynthesisof-
a)Tryptophan
b)Collagen
c)Glycosaminoglycans
d)Heme
CorrectAnswer-D
Ans.is'd'i.e.,Heme
Hemesynthesistakesplaceinallcells,butoccurstogreatestextent
inbonemarrowandliver.Thefirststepinthesynthesisofhemeis
thecondensationofglycineandsuccinylCo-Atoform6-
aminolevulinicacid(6-ALA),whichoccursinmitochondria.This
reactioniscatalyzedby6-ALAsynthasewhichrequirespyridoxal
phosphate(PLP)ascofactor.Thisistheratelimitingstepinheme
synthesis.

367.Whichisaninhibitorofferrochelatase?
a)Lead
b)Mercury
c)Iron
d)Arsenic
CorrectAnswer-A
Ans.is'a'i.e.,Lead[RefEssentialsofbiochemistry4thiep.919]
Ferrochelatase,alsocalledhemesynthase,catalysesthelast
reactioninhemesynthesis.
LeadinhibitsALAdehydratase.Therefore,leadpoisoningcauses
inhibitionofhemesynthesisandexcessiveamountof6-ALAis
excretedinurine.Leadcanalsoinhibitferrochelatase(heme
synthase).


368.Bileacidsconsistofallexcept-
a)Lithocholicacid
b)Taurocholicacid
c)Deoxycholicacid
d)Chendeoxycholicacid
CorrectAnswer-B
Ans.is'b'i.e.,Taurocholicacid[RefGanong22nalech.26]
Taurocholicacidisbilesalt(notbileacid).
Bileacidsaremainlypresentsassodiumorpotassiumsaltswhich
areconjugatedwithglycineortaurinetoformbilesalts.
Primarybileacidsarecholicacidandchenodeoxycholicacid,which
aresynthesizedfromcholesterolinliver.Intheintestinesomeofthe
primarybileacidsareconvertedintosecondarybileacids,i.e.,
deoxycholicacid(formedfromcholicacid)andlithocholicacid
(derivedfromchenodexoxycholicacid).
Glycineandtaurineconjugatesofthesebileacidsarecalledasbile
salts.Forexample,cholicacidisabileacid,anditsglycine
conjugate(glycocholicacid)isabilesalt.

369.TrueaboutcAMPandcGMP-
a)Secondmessengers
b)Actonmembranereceptors
c)Actbypost-translationalmodification
d)Alloftheabove
CorrectAnswer-D
Ans.is'd'i.e.,Alloftheabove
Secondmessengers
Secondmessengersaremoleculesthatrelaysignalsfromthe
membranereceptorstotargetmoleculesinsidethecells.Inabove
describedreceptorssecondmessengersarecAMP,cGMP,
Phosphatidylinositol,diacylgleceral,IP3,Ca.
Secondarymessengersofmembranereceptorsactby
posttranslationmodification,i.e.,modificationofproteins(e.g.,
enzymes)aftertheyhavealreadyformed(i.e.,aftertranslation).

370.Spectroscopyisusedforinteractionof-
a)Electromagneticradiation
b)Protons
c)Alphaparticles
d)Positrons
CorrectAnswer-A
Ans.is'a'i.e.,Electromagneticradiation
Spectroscopyisthestudyofthestructureofatoms/moleculesfrom
studyingtheirinteractionwithelectromagneticradiation.

371.Abnormalproteinswhichareboundto
ubiquitinaredegradedin-
a)Proteosomes
b)Golgiapparatus
c)SmoothER
d)Lysosomes
CorrectAnswer-A
Ans.is'a'i.e.,Proteosomes[RefHarper250/ep.560-561&28th/e
p.498-99]
Ubiquitinplaysmajorroleindegradationofproteinsandis
particularlyassociatedwithdisposalofmisfoldedproteins.
Itissmall,highlyconservedproteinthatplaysakeyroleinmarking
variousproteinsforsubsequentdegradationinproteosomes

372.Enzymedegradationiscausedby-
a)Ubiquitin
b)RNAse
c)Zymase
d)Chaperone
CorrectAnswer-A
Ans.is'a'i.e.,Ubiquitin[RefHarper29thiep.560-561]
Degradationofdefective(misfolded)proteins(includingenzymes)is
causedbyubiquitin

373.G1cNAc-P-P-oligosacharrideis-
a)Proteoglycan
b)Glycoprotein
c)Collagen
d)Phospholipid
CorrectAnswer-B
Ans.is'b'i.e.,Glycoprotein[RefMedicalBiochemistryby
Bhagvanp.312]
Dolicholplaysaroleinpost-translationalmodificationofproteinby
glycosylationtoformglycoproteins.
ItactsintheformofDolicholpyrophosphate(DolicholPP)and
transferOligosaccharidefromdolicholtoglycoproteins
N-acetylgalactosamine-P-P-Dolichol(GLcNAc-P-P-dol)compound
formedinthebionsynthesisoflipid-linkedoligosaccharides.
Dolichol-P-Pactsasacarrierofoligosaccharideandtransferitto
glycoprotein.

374.Prolylhydroxylaserequirewhich
cofactor-
a)Sc
b)VitaminC
c)Mo
d)VitaminK
CorrectAnswer-B
Ans.is'b'i.e.,VitaminC[RefHarper29'Vep.590-593]
Hydroxylationofprolineandlysineresiduetakesplaceduringpost-
translationalmodificationinroughER.Theenzymecatalyzingthe
reactionsareprolylhydroxylase(forproline)andlysylhydroxylase
(forlysine).Boththeseenzymesaredioxygenasesusingmolecular
oxygen(02)andcofactorforboththeseenzymesisvitaminC
(ascorbicacid).a-Ketoglutatrateisacoreductant,whichisoxidized
tosuccinate.

375.Sidechainlinkageinproteoglycons-
a)Covalent
b)Hydrogenbond
c)Electrostaticbond
d)Van-derWaal'sforce
CorrectAnswer-A
Ans.is'a'i.e.,Covalent[RefEssentialsofbiochemistryp.712]
Amongthegivenoptionstwobondsareinvolvedin
proteoglycanstructure:

1. Covalent-Inproteoglyconmonomer,i.e.betweencentralcore
proteinandsidechainofrepeateddisaccharides
2. ElectrostaticInproteoglycanaggregatesbetweenproteoglycan
monomerandhyaluronicacid.

376.Whichoffollowingisnotafreeradical-
a)H202
b)(M?
c)Superoxideanion
d)HOC1-
CorrectAnswer-D
Ans.is'd'i.e.,HOC1?[RefPrinciplesinmedicalpathologyp.
391]
Freeradicalsarechemicalspeciesthathaveasingleunpaired
electronintheouterorbit.Mostofthesearepartiallyreduced
reactiveoxygenformsthatareproducedasanunavoidable
byproductofmitochondria!respiration-alsoknownasreactive
oxygenspecies.
Themostimportantarehydrogenperoxide(1120),Superoxideanion
(02)andhydroxylradical(OH).

377.Notanirreversibleinjury-
a)Pyknosis
b)Pyknosis
c)Karyolysis
d)Blebformation
CorrectAnswer-D
Answer-D.Blebformation
Damagetonucleuscanbeofthreeforms-
Pkynosis-nuclearshrinkage&chromatincondensationand
clumping.
Karyorrhexis-Nuclearfragmentation.
Karyolysis-decreasedbasophiliaduetodissolutionofnucleus.

378.Hypertrophyis-
a)Increaseincellnumber
b)Increaseincellsize
c)Decreaseincellnumber
d)Decreaseincellsize
CorrectAnswer-B
Ans.is'B'i.e.,Increaseincellsize
Hypertrophy
*Hypertrophyreferstoanincreaseinthesizeofcellswithout
increaseinthenumber,resultinginanincreaseinthesizeoftissue.
*Hypertrophyinvolvescellenlargementwithoutcelldivision.
*So,hypertrophiedorganhasjustlargercells,butnonewcells(by
contrast,inhyperplasiathereisincreaseinnumberofnewcells
withoutincreaseinsize).
*NucleiinhypertrophiedcellshaveahigherDNAcontentthanin
normalcellsbecausethecellsarrestinthecellcyclewithout
undergoingmitosis.
*Myocardium(heartmuscle)andskeletalmuscleundergo
hypertrophy.
*Themostcommonstimulusforhypertrophyisincreasedworkload.
Mechanismofhypertrophy
*Theincreaseincellsizeisduetosynthesisofmorestructural
proteins.
*Thegenesthatareinducedduringhypertrophyincludethose
encodingtranscriptionfactors(C-fos,C-jun),growthfactors(TGF-(3,
TGF-1,FGF);andvasoactiveagents(a-agonists,endothelin-1,
angiotensinII).
*Theremayalsobeaswitchofcontractileproteinsfromadultto

fetalorneonatalforms,e.g.,duringmyocardialhypertrophy,thea-
myosinheavychainisreplacedby13-formofthemyosinheavy
chain,whichleadstodecreasemyosineATPaseactivityanda
slower,moreenergeticallyeconomicalcontraction.
*Inaddition,somegenesthatareexpressedonlyinembryoniclife
arere-expressedinhypertrophiedmyocardium,e.g.,inthe
embryonicheart,thegeneforAtrialnatriuraticpeptide(ANP)is
expressedinbothatriumandventricle.Afterbirth,ventricular
regulationofthegeneisdownregulated.Myocardialhypertrophyis
associatedwithreinductionofANFgeneexpression.ANPinduces
saltexcretionbykidneyLbloodvolume&pressure,decreasework
load.
Whydothesechangesoccur?
*Asalreadyexplained,themostcommonstimulusformyocardial
hypertrophyisincreasedworkload.
*Alltheabovemorphologicalchangesthatoccurinhypertrophy
eitherincreasemuscleactivity(tohandletheincreasedworkload)or
decreasetheworkloadtoheart.

379.Hyalinedegenerationisfoundin-
a)MI
b)Parkinson's'disease
c)Yellowfeverhepatitis
d)Basophiliccellofpituitary
CorrectAnswer-C
Answer-C.Yellowfeverhepatitis
1)Intracellularhyaline:councilmanbodiesinyellowfever
2)Extracellularhyaline:nCorporaamylaceaareroundmasseof
hyalineseeninprostateinelderly,brainandspinalcordofold
peopleandoldinfarctsoflung.

380.Programmedcelldeathisknownas-
a)Cytolysis
b)Apoptosis
c)Necrosis
d)Proptosis
CorrectAnswer-B
Ans.is'b'i.e.,Apoptosis
*Programmedcelldeathisapoptosis.
*Butrememberthisimportantfactthatnotallapoptosisare
programmedcelldeath(readtextbelow)Apoptosis
*Apoptosisispathwayofcelldeaththatisinducedbyatightly
regulatedintracellularprograminwhichcelldestinedtodieactivate
enzymesthatdegradethecell'sownnuclearDNA,andnuclear&
cytoplasmicproteins.
*Apoptosisgenerallyinvolvessinglecellsincontrasttonecrosisthat
usuallyinvolveagroupofcells.oApoptosismaybeoftwotypes?
A.Physiological(mostofthetime)Programmedcell
death.
B.PathologicalUnprogrammedcell
death.
A.Physiologicalapoptosis
*Deathbyapoptosisisanormalphenomenonthatservesto
eliminatecellsthatarenolongerneeded.
oItisimportantinthe
followingphysiologicsituation:?
1. Theprogrammeddestructionofcellsduringembryogenesis,
includingimplantation,organogenesis,andmetamorphosis.
2. Hormonedependentinvolutioninadult,e.g.,endometrialcell
breakdownduringmensturalcycle,ovarianfollicularatresiainthe

menopause,theregressionoflactatingbreastafterweaning,and
prostaticatrophyaftercastration.
3. Celldeletioninproliferatingcellpopulationinordertomaintaina
constantnumber,e.g.,intestinalcryptepithelium.
4. Deathofhostcellsthathaveservedtheirusefulpurpose,suchas
neutrophilsafteranacuteinflammatoryresponse,andlymphocytes
attheendofanimmuneresponse.
5. Eliminationofpotentiallyharmfulself-reactivelymphocytesin
thymus.
6. CelldeathinducedbycytotoxicTcellstoeliminatevirusinfected
andneoplasticcells.Samemechanismoccursingraftversushost
disease.
B.Pathologicalapoptosis
*Apoptosismayalsobepathological.
*Whencellsaredamagedbeyondrepair,especiallywhenthe
damageaffectsthecell'sDNA,theirreparablydamagedcellsare
eliminated:
?
1. Celldeathproducedbyinjuriesstimuli-->Radiationandcytotoxic
anticancerdrugsdamageDNA,andifrepairmechanismscannot
copewiththeinjury,thecellkillsitselfbyapoptosis.Inthese
situations,eliminationofthecellmaybeabetteralternativethan
riskingmutationsandtranslocationsinthedamagedDNAwhich
mayresultinmalignanttransformation.
2. Cellinjuryincertainviraldiseases,e.g.,inviralhepatitis.
3. Pathologicatrophyinparenchymalafterductobstruction,suchas
occursinpancreas,parotidgland.
4. Celldeathintumors.

381.Whichisactivatedfornuclear
fragmentationinapoptosis-
a)Caspases
b)Apaf-1
c)Oxygenfreeradicals
d)All
CorrectAnswer-A
Ansis'a'i.e.,Caspases
Caspasesandendonucleasecausechromatinfragmentationin
apoptosis.
Theyareinactive,sofirsttheyshouldbeactivated.
Cytochrome'c'bindswithApaf-1andthiscomplexactivates
caspases.
Caspasescausefragmentationofchromatinandalsoactivate
endonuclease.
Aboutoption'b'
ThoughApaf-1helpsinactivationofcaspases,ithasnorolein
chromatinframentationandItdoesnothavetwoseprateforms
activeorinactive.Itjustformscomplexwithcytochrome'C'to
activatecaspases.

382.Incelldeath,myelinfigures,arederived
from-
a)Nucleus
b)Cellmembrane
c)Cytoplasm
d)Mitochondria
CorrectAnswer-B
Ans.is'b'i.e.,Cellmembrane
Myelinefiguresarecytoplasmicbodiescomposedofconcentric
whorlsofmembranes,derivedfromdamagecellmembrane.
Myelinfiguresappearduringreversibleinjurybuttheyaremore
pronouncedinirreversibleinjury.


383.Lipofuscinis
a)Wearandtearpigment
b)Fatdeposits
c)Bloodpigment
d)Formofcalcification
CorrectAnswer-A
Answer-A.Wearandtearpigment
Itisaninsolublepigment,alsoknownasIandwearortearoraging
pigment.Lipofuscinisnotinjurioustothecelloritsfunctions.
Itsimportanceliesinbeingthetell-talesignoffreeradicalinjuryand
lipidperoxidation.

384.Antiapoptoticgene
a)FLIP
b)P53
c)BAX
d)BIM
CorrectAnswer-A
Answer-A.FLIP
Proapoptoticgenes:Apaf-1,CytochromeC,Bak,Bax,Bim,AIF,
P53,Caspases,TNFRI,FAS(CD95),FADD,BH3onlyproteins
(Bim,Bid,Bad),Smac/DIABLO.
Antiapoptoticgenes:BCL-2,BCL-X,Mcl-1,IAPs,FLIP

385.Inapoptosiswhichis/areinvolved-
a)Apaf-1
b)Bc12
c)Caspases
d)All
CorrectAnswer-D
Ans.is'a'i.e.,Apaf-1;'b'i.e.,BCl2;'c'i.e.,Caspases
Proapoptoticgenes:Apaf-1,CytochromeC,Bak,Bax,Bim,AIF,
P53,Caspases,TNFRI,FAS(CD95),FADD,BH3onlyproteins
(Bim,Bid,Bad),Smac/DIABLO.

386.Notaapoptoticgene
a)P53
b)Bax
c)Mcl-1
d)n-myc
CorrectAnswer-D
Answer-D.n-myc
Proapoptoticgenes:Apaf-1,CytochromeC,Bak,Bax,Bim,AIF,
P53,Caspases,TNFRI,FAS(CD95),FADD,BH3onlyproteins
(Bim,Bid,Bad),Smac/DIABLO.

387.Fatnecrosisiscommonin-
a)Omentum
b)Breast
c)Retroperitonealfat
d)Alloftheabove
CorrectAnswer-D
Thereare2TypesofFatNecrosis
1.Enzymaticfatnecrosis
*Thisisduetoactionoflipaseonadiposetissue.
*Itoccursmostfrequentlyinacutepancreatitisduetoleakageof
lipase.
*Dependingontheseverityofacutepancreatitis,fatnecrosismay
occurin:
-Adiposetissuecontiguoustopancrease,i.e.,retroperitonealfat.
-Adiposetissueinanteriormediastinum.
-Bonemarrow
-Omentalandabdominalfat
2.NonenzymaticorTraumaticfatnecrosis
*Occursduetotrauma
*Isseeninsubcutaneoustissueofbreast,thigh,andabdomen.

388.Caseatingnecrosisoccursin
a)Brain
b)liver
c)kidney
d)lung
CorrectAnswer-D
Answer-D.lung
Gaseousnecrosis,avariantofcoagulativenecrosiscanbe
encounteredinanyorganwherecelldeathisattributabletocertain
organismse.g.,mycobacteriumtuberculosis(TB),syphilisandfungi
(Histoplasma,Coccidioidomycosis).
Pulmonarytuberculosisisthemostcommoncauseofcaseous
necrosis.Thereforeanswershouldbe"lung".

389.Liquefactivenecrosisisseenin:
a)Heart
b)Brain
c)Lungs
d)Spleen
CorrectAnswer-B
Brain

390.Spreadofinfectioncauses
a)Fibrinoidnecrosis
b)Fatnecrosis
c)Liquifactivenecrosis
d)Coagulativenecrosis
CorrectAnswer-C
Answer-C.Liquifactivenecrosis
Liquefactiveorcolliquativenecrosisoccursduetolysosomal
permeabilityandenzymesofleukocytesdigestthetissue
transformingthetissueintoliquidviscousmass.
Tissuearchitectureislost.
Examplesare-Infarctbrainandabscesscavity.

391.MIisatypeof-
a)Coagulativenecrosis
b)Liquefactivenecrosis
c)Caseousnecrosis
d)Fatnecrosis
CorrectAnswer-A
Ans.is'a'i.e.,Coagulativenecrosis
Coagulativenecrosis
*Thisismostcommontypeofnecrosis.
*
Thistypeofnecrosisismostfrequentlycausedbysudden
cessationofbloodflow(ischemia)inorganssuchasheart(MI),
Kidney(ATN),adrenalgland,andspleen.
Note:Brainistheonlyexception,i.e.,.Itistheonlysolidorganin
whichischemialeadstoliquifactivenecrosisnotcoagulative
necrosis.
*Itisalsoseenwithothertypesofinjurye.g.,livernecrosisinviral
hepatitis,Coagulativenecrosisofskinafterburns(Thermalinjury).
*
Whythereispredominantproteindenaturationandnoenzymatic
digestion?
Hypoxiacausesintracellularacidosis(hasbeenexplainedearlier)--
>.tpHresultsindenaturationofproteinswhichincludesnotonly
structuralproteinshutalsoenzymes
So,thereisnoenzymaticdigestion.oThenecroticcellsretaintheir
cellularoutline
forseveraldays.
Liquefactivenecrosis
*Itisthenecroticdegradationoftissuethatrapidly
undergosofteningandliquefactionbecauseoftheactionof
hydrolyticenzymes.
*Itoccursafter

*Itoccursafter
1.Infectioni.e.,suppurativeinflammation(mostcommon).
2.Ischemicnecrosisinbrain.
Note:Brainlacksanysubstantialsupportivestroma,soischemic
necrosisinbrainisliquifactiveunlikeotherorganswhereitis
coagulative.
*Mechanismofliquefactivenecrosis-->Microbesstimulates
accumulationofinflammatorycellsandthesecellsrelease
lysosomalenzymes.
Caseousnecrosis
*Itisavariantofcoagulativenecrosis.
*Itismostcommonlyencounteredwhencelldeathisattributableto
certainorganismse.g.,mycobacteriumtuberculosis(TB)and
fungi(Histoplasma,Coccidioidomycosis).
Whyisitcalledcaseousnecrosis,notcoagulativenecrosis?
*Incontrasttocoagulativenecrosiswheretissuearchitectureis
maintained,incaseousnecrosis,thetissuearchitectureis
completelyobliterated.So,ithasbeencalledcaseousbecauseofits
cheesywhiteappearanceoftheareaofnecrosis.

392.Diabeticfootisassociatedwithfollowing
typeofgangrene-
a)Drygangrene
b)Wetgangrene
c)Gasgangrene
d)Fournier'sgangrene
CorrectAnswer-B
Answer-B.Wetgangrene
Whenoverlyingskinofdrygangrenoustissueisdevitalized,
bacterialinfectionissuperimposedandthecoagulativenecrosis
ismodifiedbyliquifactivenecrosis.Morecommonlyduetovenous
occlusionthenarterialocclusion.
Occursindiabeticfootandbedsores

393.Unfoldedproteinmetabolismis
associatedwith
a)Endoplasmicreticulum
b)Golgiapparatus
c)Mitochondria
d)Noneoftheabove
CorrectAnswer-A
Answer-A.Endoplasmicreticulum
Theunfoldedproteinresponse(UPR)isacellularstressresponse
relatedtotheendoplasmicreticulum(ER).
DiseasesamenabletoUPRinhibitionincludeCreutzfeldt-Jakob
disease,Alzheimer'sdisease,Parkinson'sdisease,andHuntington's
disease.

394.Mostreactivefreeradicalis:
a)Peroxide
b)Carboxyl
c)Hydroxyl
d)Superoxide
CorrectAnswer-C
Ans:C.Hydroxyl
(RefRobbins9/ep480)
Hydroxylradical(*OH)isthemostpotentreactiveoxygenspecies.
Mostreactiveoxygen-derivedfreeradical.
PrincipalROSresponsiblefordamaginglipidsproteins&DNA.

395.Notanexampleofexcesstissuegrowth
a)Granulationtissue
b)Neoplasia
c)Hyperplasia
d)Fibrosis
CorrectAnswer-A
Answer-A.Granulationtissue
Granulationtissueisahallmarkofhealing.Itischaracterizedby
formationofnewsmallbloodvessels(angiogenesisor
neovascularization)andproliferationoffibroblasts

396.1stmediatorofinflammationtobe
releasedis
a)Nitricoxide
b)PAF
c)Histamine
d)IL-1
CorrectAnswer-C
Answer-C.Histamine
Vasoactiveaminesarepresentinpreformedstoresincellsandare
thereforeamongthefirstmediatorstobereleasedduring
inflammation
Thetwoaminesareespeciallyimportant.Theseare:-(i)Histamine
and(ii)Serotonin

397.Rollingofleucocytesonendothelialcells
ismediatedby
a)ICAM-1
b)(3,integrin
c)IL-8
d)P-selectin
CorrectAnswer-D
Answer-D.P-selectin
Rolling&adhesion-
Itisbroughtby-PselectinorCD62
PECAM-1involvedinleucocytemigrationfromendothelialsurface.

398.Perioperativeshockisanexampleof
a)Hypovolemicshock
b)Septicshock
c)Cardiogenicshock
d)Neurogenicshock
CorrectAnswer-A
Answer-A.Hypovolemicshock
"Mostcommoncauseofhypovolemicshockishemorrhagewhich
maybeintraoperativeorpostoperative."

399.Substanceplayingaroleintumor
metastasiscascadeis
a)CollagenaseIV
b)TNF-alpha
c)CD99
d)NM23
CorrectAnswer-A
Ansis'a'i.e.CollagenaseIV
Variousstepsofmetastasisandmoleculesinvolved

1. Detachmentoftumorcells-Downregulationofexpressionofeither
E-cadherinsorcatenins
2. AttachementtoECM(includingbasementmembrane)-Tumorcells
expressintegrinsthathelpsintheattachement.
3. DegradationofECM-proteolyticenzymes(mostimportant
proteasesaremetalloproteinases(MMPs)includingcollegenaseIV).
4. Vasculardisseminationandhomingoftumorcells-Amongadhesion
moleculeCD44isofparticularinterest.

400.

WhichofthefollowingisnotapartofECM?
a)Lectin
b)Fibronectin
c)Laminin
d)Proteoglycans
CorrectAnswer-A
Answer-A.Lectin
Basementmembrane(BM):-PASpositiveamorphousstructure
whichlieunderneathepitheliaofdifferentorgansandendothelial
cells.ComponentsofBMarelaminin,fibronectin,tenascin,collagen
typeIV,enatactin(nidogen),proteoglycan&perlecan(heparan
sulphate).

401.Whichofthefollowingisderivedfrom
fibroblastcells?
a)TGF-13
b)MMP2
c)Collagen
d)Angiopoietin
CorrectAnswer-C
Answer-C.Collagen
Fibroblastsproducetheglycosaminoglycans,collagens,elastic
fibers,reticularfibresandglycoproteinsthatcanbeseeninthe
extracellularmatrix.TheyalsoproducecytokineTSLP.

402.Chemotaxisismediatedby-
a)Histamine
b)LeukotrieneB4andC5a
c)LeukotrieneC4andC3a
d)Bradykinin
CorrectAnswer-B
Ans.is'b'i.e.,LeukotrieneB4andC5a
Chemotaxisisunidirectionallocomotionofleukocytestowardsthe
siteofinjury,orientedalongachemicalgradient.Thischemical
gradientiscreatedbysubstancescalledchemoattractants
(chemotacticagents),whichmaybeexogenous(bacterialproducts)
orendogenous(C5a,LTB-4,IL-I,TNF,IL-8).
C5aisthemostpowerfulchemo-attractant(chemokine).

403.Whichofthefollowingenhancesacute
phaseresponse?
a)a-2microglobulin
b)Transferrin
c)Albumin
d)Retinalbindingprotein
CorrectAnswer-A
Answer-A.a-2microglobulin
a)Positiveacutephaseproteins
Theseproteinsareincreasedduringinflammation.Important
examplesareC-reactiveprotein:CRP(f31-globulin),
a-1antitrypsin,fibrinogen,ferritin,serumamyloidA,hepatoglobulin,
ceruloplasmin,anda-2microglobulin.
b)Negativeacutephaseproteins
Theseproteinsaredecreasedduringinflammation.Important
examplesarealbumin,prealbumin,transferrin,transcortin,
transthyretinandretinalbindingprotein.
Generally,positiveacutephaseproteinsareconsideredasacute
phaseproteins.

404.InterleukinresponsibleforPyrexiais:
a)IL1
b)IL3
c)IL4
d)IL8
CorrectAnswer-A
IL1

405.Interleukin2isproducedby
a)Thelpercells1
b)Thelpercells2
c)Naturalkillercells
d)Basophils
CorrectAnswer-A
Ans.is'a'i.e.,Thelpercells1
A)Thelper-1(TF,')secretes4IL-2andinterferon-y
B)Thelper-2(T 2

0 )secretes->IL-4,IL-5,IL-6,IL-13

406.Processofmigrationofleukocytesthroughtheendotheliumisthrough:
a)Phagocytosis
b)Pinocytosis
c)Diapedesis
d)Apoptosis
CorrectAnswer-C
Thenextstepintheprocessofleukocyterecruitmentismigrationoftheleukocytesthrough
theendothelium,calledtransmigrationordiapedesis.
Transmigrationofleukocytesoccursmainlyinpost-capillaryvenules.
Chemokinesactontheadherentleukocytesandstimulatethecellstomigratethrough
interendothelialspacestowardthechemicalconcentrationgradient,thatis,towardthesite
ofinjuryorinfectionwherethechemokinesarebeingproduced.
Ref:Robbins8thedition,Chapter2.

407.IFN-gammaisproducedby
a)Neutrophills
b)Macrophages
c)T-cells
d)B-cells
CorrectAnswer-C
Answer-C.T-cells
ActivatedT-cells(helper)produceIFN-y,themajorcytokineof
granulomatousinflammation.
IFN-y
HelperT-cellsareactivatedbyIL-2andproduceIFN-y,themajor
cytokineofgranulomatousinflammation
FinallythereisformationofgranulomainducedbyIFN-y.

408.Extremitiesarewarminwhichtypeof
shock
a)Hypovolemicshock
b)Neurogenicshock
c)Anaphylacticshock
d)Cardiogenicshock
CorrectAnswer-B
Answer-B.Neurogenicshock
Inhyperdynamicstageofsepticshockandinneurogenicshock,
extremitiesarewarmduetovasodilatation.

409.Serumamyloidassociatedprotein,found
in?
a)Alzheimer'sdisease
b)Chronicinflammatorystates
c)Chronicrenalfailure
d)Malignanthypertension
CorrectAnswer-B
Ans.is'b'i.e.,Chronicinflammatorystates
a)ChronicinflammatoryconditionslikeRA(mostcommon),TB&
leprosy,osteomyelitis,ankylosingspondylitis,IBD
(Crohn'sdisease,UC),bronchiectasis.
SometumorslikeRenalcellcarcinoma(hypernephroma),Hodgkins
lymphoma


410.HLA-Cw6isassociatedwith
a)Myatsheniagravis
b)Behcet'sdisease
c)Pemphigusvulgaris
d)Psoriasisvulgaris
CorrectAnswer-D
Answer-D.Psoriasisvulgaris
HLAassociationinpsoriasis
Psoriasisvulgaris-Cw6,B13,B17,DRB1*0701/2,B37
Psoriaticarthritis-B27
Generalizedpustularpsoriasisandacrodermatitiscontinuaof
Hallopeau-B27,B8
Pustolosisofpalmsandsoles-Aw19,Bw35

411.Followinginjectionoflymphokines,the
sameclassofimmunoglobulinare
produced.Thisistobereferredas-

a)Groupswitching
b)Clonalselection
c)Hybridisation
d)Classswitching
CorrectAnswer-B
Ans.is'b'i.e.,Clonalselection
*B-cellsareabletomakeaspecificantibodyagainstaspecific
antigen.
*ItisduetopresentofspecificreceptoronB-cells-B-cellreceptor
(BCR),thatisusuallyanimmunoglobulinIgMorIgD.
*AnantigeninteractswithB-cellthatshowsbestfitbyvirtueofits
BCR.
*Theantigenbindstothisreceptor,andtheB-cellisstimulatedto
divideandformaclone(clonalselection).
*Thiscloneofcellswillbecomeplasmacellsthatwillsecretes
antibodyofaparticularspecificityandsameclass.
*Although,B-cellsarethemajorsourceofantibodies(aftertheir
conversionintoplasmacells),Helper-Tcellsarealsoimportant.
*HelperT-cellsactivateB-cellsbysecretingcytokines
(lymphokines).
oInantibodyformationT-cellsareinvolvedearlier
thanB-cells.

Antigenexposure
AntigenpresentingcellspresentthisantigentohelperTcells
Tcellsareactivated

SecretionoflymphokinebyT-cells
B-cellsthatcarryBCRspecifictothatantigenare,stimulated(clonal
selection)
B-cellsproliferateanddifferentiateintoplasmacells
Plasmacellssynthesizeanimmunoglobulinofsamespecificityas
thatcarriedbytheB-cellprecursors
Alsoknow
Classswitching
*Isotypeorclassswitchingisabiologicalprocessthatchangesan
antibodyfromoneclasstoanother.ForexamplefromIgM(in
primaryresponse)toIgG(insecondaryresponse).

412.ShrinkingLungSyndromeisseenin:
a)SLE
b)RheumatoidArthritis
c)Scleroderma
d)Sarcoidosis
CorrectAnswer-A
AnswerisA(SLE):
ShrinkinglungsyndromereferstoaconditiontypicalofSLEthat
consistsofapurelyrestrictiverespiratorydiseasewithnormallung
parenchymaandmarkedlydecreasedlungvolumes.

Shrinkinglungsyndrome
ShrinkinglungsyndromereferstoaconditiontypicalofSLEthatconsistsofapurelyrestrictive
respiratorydiseasewithnormallungparenchymaandmarkedlydecreasedlungvolumes
PathogenticMechanism
Diaphragmaticdysfunctionhasbeenadvocatedasthemainpathogeneticmechanismof
shrinkinglungsyndrome
ClinicalPresentation
Shrinkinglungsyndromeusuallymanifestsasexertionaldyspneaofvariableseverity,whichcan
progressoveraperiodofweeksormonths.(Orthopneaattributedtodiaphragmaticweakness
mayalsooccur).
Pleuriticchestpainisreportedfrequently,andaprevioushistoryofpleurisyandpericarditisis
common.
Physicalexaminationisremarkablynormal.
Investigations
Chestradiographytypicallyshowselevatedhemi-diaphragms,althoughthisisnotauniversal
findinganditsabsencedoesnotexcludethediagnosis.Pleuraleffusions,Pleuralthickening,and
atelectasismaybealsoevidentonplainfilmsorCTscans.
Pulmonaryfunctiontestsshowamarkedrestrictivepattern,withdecreasedforcedvitalcapacity

Pulmonaryfunctiontestsshowamarkedrestrictivepattern,withdecreasedforcedvitalcapacity
(FVC).
Carbonmonoxidediffusioncorrectedbylungvolumesistypicallynormal.
Anti-Roantibodiesmaybepresent,althoughtheydonotofferanadditionaldiagnosticaid.
Prognosis
Theprognosisofthissyndromeisusuallygood(Mostpatientsshowlong-termstabilization)

413.MHC-2ispresentinallexcept
a)Corticalmacrophages
b)Medullarymacrophages
c)Corticalepithelialcells
d)Medullaryepithelialcells
CorrectAnswer-B
Answer-B.Medullarymacrophages
Corticalmacrophages,epithelialcellsanddendriticcellsexpress
highlevelofclassIIMHCmolecule.Medullarymacrophages
expressonlyclassIMHC,whilemedullaryepithelialcellsand
dendriticcellsexpressbothclassI&IIMHCmolecules.

414.Bcellsarelocatedinwhichregionof
lymphnodes
a)Paracorticalregion
b)Corticalfollicles
c)Medullarsinuses
d)Subcapsularregion
CorrectAnswer-B
Answer-B.Corticalfollicles
Locationofimmunecellsinlymphnode:

1. T-cells:Paracorticalarea.
2. B-cells:Corticalfollicles,germinalcenters,medullarycords.

415.PSGNisanexampleofwhichtypeof
hypersensitivity
a)Type-1hypersensitivity
b)Type-2hypersensitivity
c)Type-3hypersensitivity
d)Type-4hypersensitivity
CorrectAnswer-C
Answer-C.Type-3hypersensitivity
Glomerulonephritis(PSGN:Post-streptococalGN)isatypeIII
hypersensitivity.

416.NumberofcriteriaforHLAmatchingare
a)10
b)4
c)16
d)22
CorrectAnswer-A
Answer-A.10
Atleast8HLAmarkersfortheseminimumrequirements:twoA
markers,twoBmarkers,twoCmarkers,andtwoDRB1markers.
Sometimesanadditionalmarker,calledDQ,isalsomatchedmaking
it10markers.

417.Organwithleastchanceofrejection
a)Blood
b)Kidney
c)Heart
d)Liver
CorrectAnswer-D
Answer-D.Liver
HLAmatchingplayaveryminimalroleinlivertransplanttherefore
immunerejectionislesscommoninlivertransplant.

418.Mctypeofgraftrejectionis
a)Hyperacute
b)Acute
c)Chronic
d)Acuteonchronic
CorrectAnswer-C
Answer-C.Chronic
"Acutegraftrejectionisthemostcommonform"

419.Trueaboutserumsicknessis
a)Type2hypersensitivity
b)Canleadtoleukocytoclasticvasculitis
c)Hypercomplementemia
d)Canoccurduetohomologousantigen
CorrectAnswer-B
Answe-B.Canleadtoleukocytoclasticvasculitis
SerumsicknessisatypeIIIhypersensitivity(immunecomplex
mediated)reactionthatresultsfromtheinjectionofheterologousor
foreignproteinorserum.
Whenanantiserumisgiven,thehumanimmunesystemcan
mistaketheproteinspresentforharmfulantigens.Thebody
producesantibodies,whichcombinewiththeseproteinstoform
immunecomplexes.Thesecomplexesprecipitate,enterthewallsof
bloodvessels,andactivatethecomplementcascade,initiatingan
inflammatoryresponseandconsumingmuchoftheavailable
complementcomponent3(C3).Theresultisaleukocytoclastic
vasculitis.

420.Mostlethalcombinationis
a)Autosomalmonosomy
b)Chromosomalmonosomy
c)Autosomaltrisomy
d)Chromosomaltrisomy
CorrectAnswer-A
Answer-A.Autosomalmonosomy
Autosomalmonosomies(lossofoneautosome)areincompatible
withfetaldevelopmentandarenotfoundinlivebirths.Only
monosomycompatiblewithlivebirthisduetoinvolvementofsex
chromosome,i.e.Turnersyndrome(45X).

421.AllofthefollowingaretrueaboutDown
syndromeexcept
a)IncidenceofRobertsoniantranslocationis1:1000
b)Extrachromosomeisofmaternalorigin
c)Mostcommoncauseistrisomy21
d)Mosaicism21hasnoassociationwithmaternalage
CorrectAnswer-A
Answer-A.IncidenceofRobertsoniantranslocationis1:1000
CytogeneticsinDown'ssyndrome

1. Trisomy21:Itismostcommon(95%)cause.Extrachromosomeis
maternalinorigin.
2. Robertsontronslocations(t22:21;t14:21;t15:21):Thisaccounts
for3%(3:100)ofcases.
3. Mosaicismof21:Thisaccountsfor2%ofcases
4. Partialtrisomy:Very-veryrare

422.Hypophosphatemicricketsis
a)AR
b)AD
c)XR
d)XD
CorrectAnswer-D
Answer-D.XD
X-linkeddominantdisorders
VitamineDresistant(X-linkedhypophosphatemic)Rickets.
Familialhypophosphatemia.
FragileX-syndrome.
Incontinentiapigmenti.
Rettsyndrome.

423.Chromosomalinstabilitysyndromeis
a)Fanconisyndrome
b)AtaxiaTelangectasia
c)Bloomsyndrome
d)Alloftheabove
CorrectAnswer-D
Ans.is'd'i.e.,Alloftheabove[RefTalwarGPp.855]
Chromosomalinstabilitysyndromesareagroupofdisorders
characterizedbychromosomalinstabilityandbreakage.
Thereare:
1.Xerodermapigmentosa3.Fanconisyndrome
2.Bloomsyndrome4.Ataxiatelangectasia

424.Structureofchromosomesisstudied
by?
a)C-banding
b)G-banding
c)Q-banding
d)BrdV-staining
CorrectAnswer-B
Answer-B.G-banding
Metaphasecellsarethenfixedwithmethanol/glacialaceticacid
mixtureandstainedbyoneoftheseveralbandingtechniques:-

1. G-Banding(Giemsabanding):Itisusedmostcommonly.
2. Otherbandingtechniques:Q-Banding(Quinacrinebanding),C-
Banding(Constitutivebanding),andR-Banding(Reversestaining
Geimsabanding).

425.Locationofgeneonchromosomeis
identifiedby
a)Karyotyping
b)Geneticmapping
c)Microarray
d)Genomicimprinting
CorrectAnswer-B
Answer-B.Geneticmapping
Amapofthehumangenomeallowstounderstandwheregenesare
located.

426.Tyrosinekinasereceptorisassociated
withproto?oncogene-
a)RAS
b)MYC
c)RET
d)RB
CorrectAnswer-C
Answer-C.RET
Itisagrowthfactorreceptor(receptortyrosinekinase).RETprotein
isareceptorforglialcelllinedderivedneurotrophicfactor.RETis
normallyexpressedinparafollicular'C'cellsofthyroid,adrenal
medullaandparathyroidcellprecursons.
PointmutationinRETprotooncogenesisassociatedwithMEN-2A,
MEN-2B,medullarythyroidcarcinomaandHirschsprungdisease.

427.Reversiblechangefromoncelltypeto
otherisknownas-
a)Hyperplesia
b)Hypertrophy
c)Metaplesia
d)Dysplasia
CorrectAnswer-C
Ans.is'c'i.e.,Metaplesia[Ref:Robbin's9th/ep.376,styep10]
Metaplasiaisanadaptivechangeinwhichoneadult(mature)cell
typeisreplacedbyanotheradult(mature)cell.Itiscompletely
reversible

428.Whichisnotatumorsuppressorgene?
a)WT-1
b)Rb
c)p53
d)RAS
CorrectAnswer-D
RAS[Ref.Robbins7th/ep295]
RASisnotatumoursuppressorgene,itisanoncogene.
Normalgrowthandcelldivisionisunderthecontroloftwotypesof
genes:
(i)Protooncogenes
(ii)Tumoursuppressorgenes
Protooncogenesarethegenesthatnormallycontrolhowoftenacell
dividesandthedegreetowhichitdifferentiates.Theyallowcell
divisionatacontrolledrate.
Whenaprotooncogenemutatesinto,anoncogeneitbecomes
permanently"turnedon"oractivatedevenwhenitisnotsupposed
tobe.Whenthisoccurs,celldividestooquicklywhichcauses
cancer.
Tumoursuppressorgenesarenormalgenesthatslowcelldivision.
Atumoursuppressorgeneislikethebrakepedalonacar,it
normallykeepsthecellfromdividingtooquicklyjustasabrake
keepsacarfromgoingtoofast.
Whenitmutates,celldivisioncangetoutofcontrol.
Functionsoftumoursuppressorgene:-
Repressionofgenesthatisessentialforthecontinuingofthecell
cycle.
AnimportantfunctionoftumoursuppressiongeneisrepairingDNA

damage.
Everytimeacellpreparestodivideinto2newcellsitmustduplicate
itsDNA.Thisprocessisnotperfectandcopyingerrorsometimes
occurs.
Fortunately,cellshaveDNArepairgeneswhichmakeproteinsthat
proofreadDNA.
ButifthegenesresponsiblefortherepairarefaultythentheDNA
candevelopabnormalitiesthatmayleadtocancer.
Ifthereistoomuchdamagetoacell'sDNAtobefixedbytheDNA
repairgenes,thetumoursuppressorgeneisresponsiblefor
destroyingthecellbyaprocesscalledApoptosis.
Ifthetumoursuppressorgeneisnotworking.cellswithDNA
damagewillcontinuetogrowandcaneventuallybecome
cancerous.
ExamplesofTumoursuppressorgene
Gene Cancerassociated
?TGF
Carcinomaofstomach
Preceptor
?E-cadherin
Carcinomaofcolon
?NF-1
Neuroblastomas
Schwannomaand
?NF-2
meningiomas
?APC/13
Carcinomaofstomach
catenin
colon,
pancreas,melanoma
Endometrialandprostate
?PTEN
carcinoma
?SMAD2and Colon,pancreastumour
?SMAD4
?RB
Retinoblastoma,
osteosarcoma
carcinomaofbreast,colon
andLung.
?P53
Mosthumancancers
?WT-1
WilmsTumour
?P16
Pancreatic,breastand
esophagealcancers

esophagealcancers
?BRCA1and Unknown
?BRCA2
?KLF6
Prostate

429.Whichofthefollowinghastumor
promotingeffect?
a)BRAC
b)RB
c)MYC
d)p16
CorrectAnswer-C
Answer-C.MYC
RAS,Mitogen-activatedproteinkinasekinase,VEGF,orAkt

430.CEAis
a)Hormone
b)Glycoprotein
c)Enzyme
d)Tumorassociatedprotein
CorrectAnswer-B
Answer-B.Glycoprotein
Carcinoembryonicprotein(CEA)isaglycoprotein.

431.CEAisincreasedinwhichnon-
neoplasticcondition
a)Hemolyticanemia
b)Pancreatitis
c)BPH
d)Pregnancy
CorrectAnswer-A:B
CEA-Carcinoembryonicantigen(CEA):CEAisalsoaglycoprotein
normallysynthesizedinembryonictissueofthegut,pancreas,and
liver.
Theirserumlevelsarehighincancersofthegastrointestinaltract,
pancreas,ovariancancerandbreast.
CEAlevelsarealsoelevatedincertainnon-neoplasticconditions
e.g.inulcerativecolitis,Crohn'sdisease,hepatitis,andchronic
bronchitis,Alcoholiccirrhosis,smoking,pancreatitis,hemolytic
anemia

432.Carcinomaoriginatingfromglandsis
called
a)Basalcellcarcinoma
b)Squamouscellcarcinoma
c)Adenocarcinoma
d)Fibrosarcoma
CorrectAnswer-C
Answer-C.Adenocarcinoma
BenigntumorofglandsAdenoma.
MalignanttumorofglandAdenocarcinoma.

433.Hematologicalcarcinomaiscommonly
linkedto
a)Nicotine
b)Benzene
c)Lithium
d)Alcohol
CorrectAnswer-B
Answer-B.Benzene
Benzeneexposureisassociatedwithleukemia.

434.Majorcontributiontocachexiawith
advancedcancer?
a)Clathrin
b)Histamine
c)Interferon
d)Tumor-necrosis-factor(TNF)
CorrectAnswer-D
Ans.is'd'i.e.,Turnor-necrosisfactor(TNF)
Patientswithcancercommonlysufferprogressivelossofbodyfat
whichisaccompniedbyprofoundweaknessanorexia,andanemia.
Thiswastingsyndromeisreferredtoascachexia.
oTNFproducedbymacrophagesorsometumorcellsisthemost
importantmediatorofcachexia.

435.Elasticfibersoftunicamediaare
secretedby
a)Fibroblast
b)Endothelium
c)Smoothmuscle
d)Externallamina
CorrectAnswer-C
Amswer-C.Smoothmuscle
Thebloodvesselsaremadeofthreelayers,calledfromtheluminal
sideoutward,thetunicaintima,thetunicamediaandthetunica
adventitia.
1. Thetunicaintimaconsistsofanendotheliumandanysub-
endothelialconnectivetissue.Itisseparatedfromtunicamediaby
internalelasticlamina.
2. Thetunicamediaisthelayerofconcentrically-arrangedsmooth
muscle.Smoothmusclecellshavesecretorycapabilitiesandthe
tunicamediacontainsvaryingamountsofcollagenfibers,elastic
fibers,elasticlamellae,andproteoglycanssecretedbythesmooth
musclecells.

436.Levelofwhichofthefollowingisnot
elevatedinheartdisease
a)LDH
b)5-nucleotidase
c)SGOT
d)ALP
CorrectAnswer-B
Answer-B.5-nucleotidase
5-nucleotidaseiselevatedinliverdisease,bonediseaseand
pregnancy.
ALP(alkalinephosphatase)israisedincongestiveheartfailure

437.Whichproteinisdefectiveindialated
cardiomyopathy?
a)Myosin
b)Troponin
c)Tropomysoin
d)Dystrophin
CorrectAnswer-D
Answer-D.Dystrophin
Dialatedcardiomyopathyoccusduetodefectivecytoskeleton
proteins.Mostimportantproteininovolvedisdystrophin.Other
proteinsaffectedare:(1)Desmin;(2)MLPProteinand(3)aP
crystallineproteins

438.LinesofZahnareseenin-
a)Heart
b)Lung
c)Liver
d)Kidney
CorrectAnswer-A
Answer-A.Heart
LinesofZohnCharacteristicofthrombithatisformedinheartor
aorta.Theyhavevisibleandmicroscopiclaminationsproducedby
alternatingpalelayerofplateletsmixedwithfibrinanddarkerlayer
containingredbloodcells.

439.Smallvesselvasculitisare-
a)ClassicalPAN
b)Wegner'sgranulomatosis
c)Giantcellarteritis
d)All
CorrectAnswer-B
Ans.is`b'i.e.,Wegner'sgranulomatosis
Inflammationofvesselwalliscalledvasculitis.
oOfthesesystemicnecrotizingvasculitis,severaltypesaffectthe
aortaandmediumsizedvessels,butmostaffectsmallvessels,such
asarterioles,venules,andcapillaries-->smallvesselvasculitis.
LargevesselvasculitisMediumvessel
vasculitisSmallvesselvasculitis
oGiantcellarteritisoPolyarteritisnodosaANACA
positiveANCAnegative
(Temporalarteritis)oKawasakidiseaseo
Wegner'sgranulomatosisoBechet'ssyndrome
oTakayasuarteritisoBuerger'sdiseaseo
MicroscopicpolyangitisoHypersensitivity
oCogansyndromeo
ChurgstrausssyndromeoUrticarialvasculitisoDruginduced
FollowinginformationhavebeenaddedinPieofRobbins
oFollowingaresmallvesselsvasculitisandareimmune-complex
mediated:
1. SLE3)Cryoglobulinemia
2. Henochschonleinpurpoura4)Good-pasturedisease
oFollowingsmallvesselvasculitisarepauci-immune,i.e.thereis
paucityofimmunecomplexes:

1.Microscopicpolyangitis3.Chrug-strauss
syndrome
2.Wegenergranulomatosis

440.Heartfailurecellsare
a)Neutrophils
b)Macrophages
c)Lymphocytes
d)Basophils
CorrectAnswer-B
Answer-B.Macrophages
HeartfailurecellsareHemosiderinladenalveolarmacrophages.
Heartfailurecellsareamanifestationofpulmonarycongestionand
edema(asseeninheartfailure).
Ruptureofdilatedandcongestedcapillariesmayresultinminute
intra-alveolarhemorrhages.Thebreakdownoferythrocytesliberates
haemosiderinpigmentwhichistakenupbyalveolarmacrophages,
so-calledheartfailurecells,seeninthealveolarlumina.

441.Heartfailurecellsareseenin-
a)Pulmonaryedema
b)Pulmonaryinfarction
c)Pulmonaryabscess
d)PulmonaryTB
CorrectAnswer-A
Answer-A.Pulmonaryedema
Pulmonaryedema&pulmonaryinfarctioncanbedifferentiatedby
thepresenceofheartfailurecellsinpulmonaryedema.
HeartfailurecellsareHemosiderinladenalveolarmacrophages.
Heartfailurecellsareamanifestationofpulmonarycongestionand
edema(asseeninheartfailure).

442.Obliterativeendarteritisinvasavasorum
isseenin-
a)Hypertension
b)Tuberculosis
c)Syphilis
d)SLE
CorrectAnswer-C
Answer-C.Syphilis
Obliterativeendarteritisthatinvolvesvasovasorumofaortaisseen
inthetertiarystageofsyphilis.Itmayleadtosyphiliticaneursym
(leueticaneurysm).
Itusuallyaffectstheproximalascendingaorta,particularlytheaortic
ring.Syphiliticaortitismayoccasionalyinvolvetheaorticarchor
descendingaorta.

443.Concentrichypertrophyofleftventricle
isseenin-
a)Cong.bicuspidaorticvalve
b)MS
c)AR
d)HOCM
CorrectAnswer-A
Ans.is'a'i.e.,Cong-Bicuspidaorticvalve
Aorticstenosiscausespressureoverloadthatresultsinconcentric
hypertrophy.
oCongenitalbicuspicaorticvalvecausesvalvularaorticstenosis.

444.Whichtypeofarteryismostcommonly
involvedinPAN?
a)Muscular
b)Pulmonary
c)Skin
d)GIT
CorrectAnswer-A
Answer-A.Muscular
Polyarteritisnodosaorclassicalpolyarteritisnodosaissystemic
necrotizingvasculitisofmediumsizedmusculararteries,butdoes
notinvolvesmallvesselsi.e.,arterioles,venulesorcapillaries.
Ittypicallysparesthepulmonarycirculation.
Renalarteryinvolvementisthemajorcauseofdeath.

445.AmountofbloodlossinStageIof
hemorrhagicshockis-
a)<10%
b)<20%
c)<30%
d)<40%
CorrectAnswer-B
Answer-B.<20%
Mildhypovolemia(stage1)(<20%volumeloss):Onlymild
tachycardiaistherewithnormalBP
Rapidlossofupto20%ofthebloodvolume,orslowlossesofeven
largeramounts,mayhavelittleimpactinhealthyadults;greater
losses,however,cancausehemorrhagic(hypovolemic)shock

446.hL/hbloodgroup-
a)lacksH-antigen
b)lacksA-antigen
c)lacksB-antigen
d)Alloftheabove
CorrectAnswer-D
Answer-D.Alloftheabove
InBombaybloodgroup(h/horOhbloodgroup)theprecursor
antigen(Hantigen)isabsent,consequentlyAandBantigenswhich
arederivedfrommodificationofHantigenarenotformed.Thus,
BombaybloodgrouplacksH,AandBantigen.

447.NormalMyeloid-erythroidratiois-
a)l:l
b)2:l
c)3:l
d)4:1
CorrectAnswer-C
Answer-C.3:l
Bonemarrowisthemajorsourceofallhematopoieticcells
(proginatorhematopoieticcells)inpost-natallife.Innormaladults,
ratiooffatcellstohematopoieticcellsis1:1.Ratioofmyeloidto
erythroidis3:1.Ratiooffatcellstoerythroidcellsis4:I.

448.Distinguishingfeatureofreticulocyteis-
a)Constitute10%oftheredcells
b)Nonucleus
c)SmallerinsizethenRBCs
d)MatureinlymPhnodes
CorrectAnswer-B
Answer-B.Nonucleus
Reticulocytesareimmatureredbloodcells
Reticulocytesdonothaveanucleus.

449.LifespanofneonatalRBC?
a)60-90days
b)90-120days
c)120-150days
d)150-200days
CorrectAnswer-A
Ans.is'a'i.e.,60-90days

450.Whatistrueaboutsicklecell-
a)Sicklingoccursbothinheterozygousandhomozygousstate
b)FetalhemoglobinfacilitatesSickling
c)Sicklingisreversiblewithoxygenation
d)SicklingLeadstodecreasedMCHC
CorrectAnswer-C
Answer-C.Sicklingisreversiblewithoxygenation
Sicklingofredcellsisreversibleinitially,i.e.,withoxygenation,HbS
depolymerizesandcellshapenormalizes.
Repeatedepisodesofdeoxygenationandsickling,membrane
damageoccursandcellsbecomeirreversiblysickled,andretain
theirabnormalshapeevenwhenfulloxygenated.

451.WarmantibodyinAIHAis-
a)IgE
b)IgM
c)IgG
d)IgD
CorrectAnswer-C
Answer-C.IgG
Warmantibodyautoimmunehemolyticanemiaisthemostcommon
formofautoimmunehemolyticanemia.
MostcausativeantibodiesareoftheIgGclass,sometimesIgA
antibodiesareculprit.

452.Osmoticfragilitytestiscommonlyused
for-
a)Irondeficienryanemia
b)Megaloblasticanemia
c)HereditaryspherocYtosis
d)Aplasticanemia
CorrectAnswer-C
Answer-C.HereditaryspherocYtosis
IncreasedOsmoticfragility-
Hereditaryspherocytosis
Hemolyticanemia(acquiredimmune)
Malaria
Severepyruvatekinasedificiency

453.Whichofthefollowingfindingsisdiagnosticofirondeficiencyanemia?
a)IncreasedTIBC,decreasedserumferritin
b)DecreasedTIBC,decreasedserumferritin
c)IncreasedTIBC,increasedserumferritin
d)DecreasedTIBC,increasedserumferritin
CorrectAnswer-A
IrondeficiencyanemiaisassociatedwithincreasedTotalironbindingcapacity(TIBC)and
decreasedserumferritin(storageformofiron)
Ref:Harrison'sPrinciplesofInternalMedicine,17thEdition,Page631,663;Davidson's
principlesandpracticeofMedicine,20thEdition,Chapter24,Page1025-1027&1030

454.RistocetintestingvonWillebrand
diseaseshows?
a)Increasedagglutination
b)Decreasedagglutination
c)Normalagglutination
d)Noagglutination
CorrectAnswer-B
Answer-B.Decreasedagglutination
Addingristocetinatafinalconcentrationof1.25g/1toplateletrich
plasma(PRP)ofapatientwithvonWillebranddisease(VWD)
almostinvariablyresultsinareducedagglutinationoftheplatelets
comparedtoanormalPRP.

455.Shelflifeofplateletstobloodbankis
a)5days
b)7days
c)10days
d)21days
CorrectAnswer-A
Answer-A.5days
PlateletsareapprovedbyFDAforstoredupto5daysat20-24
(RoomTemperature)becauseofriskofbacterialcontamination.

456.RoutineRhtypingincludestesting?
a)Aantigen
b)Bantigen
c)Cantigen
d)Dantigen
CorrectAnswer-D
Answer-D.Dantigen
AfterABO,themostimportantantigenintransfusionpracticeisD.
TheDantigenisamemberoftheRhsystem.

457.Followingareseeninpolycythemiavera
except:
a)Mostcommoncauseofpolycythemia
b)Increasederythropoietin
c)Erythropoietinindependentgrowthofredcellprogenitors
d)Intrinsicabnormalityofhematopoieticprecursors
CorrectAnswer-B
Polycythaemiavera(PV)isaclonaldisordercharacterizedby
increasedproductionofallmyeloidelementsresultinginpancytosis
(i.eincreasedredcells,granulocytes,platelets)intheabsenceof
anyrecognizablecause.
Theterm`polycythemiavera'or`polycythemiarubravera'isused
forprimaryoridiopathicpolycythemiaonlyandisthemostcommon
ofallthemyeloproliferativedisorders.
Secondarypolycythemiaorerythrocytosis,ontheotherhand,may
occursecondarytoseveralcausese.g.highaltitude,cardiovascular
disease,apulmonarydiseasewithalveolarhypoventilation,heavy
smoking,inappropriateincreaseinerythropoietin(renalcell
carcinoma,hydronephrosis,hepatocellularcarcinoma,cerebellar
hemangioblastoma,massiveuterineleiomyoma);
clinicalfeatures:
headache,vertigo,tinnitus,visualalterationssyncopeorevencoma.
Increasedriskofthrombosisduetoacceleratedatherosclerosis.
Increasedriskofhemorrhagesduetoincreasedbloodvolumeand
intrinsicplateletdysfunctione.g.epistaxis,pepticulcerdisease
Splenomegalyproducingabdominalfullness.
Pruritus,especiallyafterabath


458.Whichofthefollowingdoesnot
predisposetoleukemia?
a)Geneticdisorder
b)Alcohol
c)Smoking
d)Chemicalexposure
CorrectAnswer-B
Answer-B.Alcohol
Etiologicalagentsareradiation,chemicals(benzene,ethylene
oxide),smokinganddrugs(alkylatingagents,topoisomeraseII
inhibitors).

459.'Hairycellleukemia'isaNeoplastic
proliferationof:
a)T.cells
b)B.cells
c)Myeloidcells
d)Macrophages
CorrectAnswer-B
AnswerisB(BCells)
Wain'CellisararehutdistinctiveB-cellneoplasm.
Hairycellleukemiareview:
Presentationiswithatriadof:
1. SplenomegalyQ:oftenmassive.Howeverhepatomegalyisless
commonwhilelymphadenopathyisrare.
2. PancytopeniaQandthereby,resultinginfections.
3. VasculitislikesyndromeQ:Erythemanodosumandcutaneous
nodulesduetoperivasculitisandPAN.
Course:Hairycellleukemiafollowsanindolentcourse.Q

460.AMLbestprognosisisseenwith?
a)Acutemyelomonocyticleukemia.
b)Acutemonocyticleukemia.
c)Acutepromyeloblasticlukemia(M.3).
d)Erythroleukemia
CorrectAnswer-C
Ans.is'c'i.e.,Acutepromyeloblasticleukemia(M.3)
Acutepromycloblasticleukemia
AlsoknownasM-3
Associatedwitht(15:17)
DIC,chloromascommon
Veryresponsinetoretinoicacidcombinedwithanthracylines.
M.7(acutemegakaryocyticleukemia)mostlyseenindown
syndrome.
French-American-British(FAB)ClassificationofAcuteMyelogenous
Leukemia

461.AMLcausingGumhypertrophy?
a)M1
b)M2
c)M3
d)M4
CorrectAnswer-D
Ans.is'd'i.e.,M4
Inacuteleukemiastheclinicalfeaturesareprimarilyseen
becauseof:
Replacementofnormalcellsofbonemarrowbyleukemiccells
resultinginanemia,thrombocytopenia,neutropenia.Infiltrationof
leukemiccellinvariousextramedullaryorganscausing,
hepalomegaly,splenomegaly.Gumhypertrophyduetoinfiltrationof
gumsbyleukemiccellsisonesuchfeature.
Itischaracteristically
associatedwithAML-M5andAML-M4i.e.(acutemonocytic
leukemia).

462.Radiotherapyinducedradiation
pneumonitismediatedbyallofthe
followingexcept-

a)TNF-c
b)PAF
c)TGF-p
d)NF-kB
CorrectAnswer-B
Answer-B.PAF
ImportantmediatorsofRadiationinducedpneumonitisareTNF-a,
TGF-I3andTh2cellscytokines(IL-4,IL-5,IL-6&IL-13).
mlresponse(II,-2,IFN-y)issuppressedduringradiationpneumonitis

463.Mostcommonlungcancerinnon-
smokersis:
a)Adenocarcinoma
b)Squamouscellcarcinoma
c)Oatcellcarcinoma
d)Noneoftheabove
CorrectAnswer-A
Ans.A:Adenocarcinoma
Adenocarcinomaaccountsfor40%ofnon-smallcelllungcancers.It
usuallyoriginatesinperipherallungtissue.Mostcasesof
adenocarcinomaareassociatedwithsmoking;however,among
peoplewhohaveneversmoked("never-smokers"),adenocarcinoma
isthemostcommonformoflungcancer.
Asubtypeofadenocarcinoma,thebronchioloalveolarcarcinoma,is
morecommoninfemalenever-smokers,andmayhavedifferent
responsestotreatment.

464.Mostcommonhistologicalformoflung
Cathatmetastasizesis-
a)SquamouscellCA
b)Adenocarcinoma
c)Alveolar-carcinoma
d)Smallcellcarcinoma
CorrectAnswer-A
Answer-A.SquamouscellCA
Smallcellcarcinomasarethemostaggressiveoflungtumors,
metastasizewidelyandarevirtuallyincurablebysurgicalmeans.

465.Mostcommontnreofrenalcarcinomais
-
a)Clearcelltype
b)Chromophobetype
c)Papillarytype
d)Tubulartype
CorrectAnswer-A
Answer-A.Clearcelltype
Clearcellcarcinomaisthemostcommontypeofrenalcancer
accountingforabout70%to80%oftherenalcellcancer.

466.CharacteristicfeatureofIgA
nephropathy-
a)Serumcomplimentlevelisnormal
b)Morecommoninoldage
c)ItisatFpeofmembranoproliferativeGN
d)Grosshematuriapresentsafterl0days
CorrectAnswer-A
Answer-A.Serumcomplimentlevelisnormal
ComplimentlevelisnormalinIgAnephropathy.Itiscommonin
children.Grosshematuriaisseenin1-2days.Itisatypeof
mesangioproliferative(notmembranoproliferative)GN.
Itisthemostcommontypeofglomerulonephritisworldwide.

467.IgAnephropathyisnotassociatedwith?
a)FocalMesengialproliferation
b)Grosshematuriawithin1-2daYs
c)OnimmunofluorescencedepositscontainbothIgAandIgG
d)Increasedcomplimentlevel
CorrectAnswer-D
Answer-D.Increasedcomplimentlevel
ComplimentlevelisnormalinIgAnephropathy.Itiscommonin
children.Grosshematuriaisseenin1-2days.Itisatypeof
mesangioproliferative(notmembranoproliferative)GN.
Itisthemostcommontypeofglomerulonephritisworldwide.

468.Mostcommonnephropathyassociated
withmalignaneNis:
a)Focalsegmentalglomerulosclerosis(FSGS)
b)Minimalchangedisease
c)IgAnephropathy
d)Membranousglomerulonephritis
CorrectAnswer-D
Ans:D.Membranousglomerulonephritis
Mostcommonnephropathyassociatedwithmalignancy-
Membranousglomerulonephritis.
In25-30%-Associatedwithmalignancy(solidtumorsofbreast,
lung,colon),infection(hepatitisB,malaria,Schistosomiasis),or
rheumatologicdisorderslikelupusorrarelyrheumatoidarthritis.
Histopathology:
Electron-densedepositsalongtheepithelialsideofthebasement
membranewitheffacementoffootprocessesoverlyingdeposits.
(RefRobbins9/ep917-918;Harrison19/ep1843).

469.Irregularscarredkidneywithpelvic
dilatationisseenwith-
a)Chronicpyelonephritis
b)Polycystickidney
c)Renalarterystenosis
d)Tuberculosisofkidney
CorrectAnswer-A
Answer-A.Chronicpyelonephritis
Pathologicalchangesofchronicpyelonephritisare:

1. Irregularscarringofkidney
2. Thehallmarksofchronicpyelonephritisarecoarsecorticomedullary
scarsoverlyingbluntedcalyces;dilatedpelvisandflatteningof
papillae.

470.NottrueaboutAlport'ssyndrome-
a)X-linked
b)Autosomaldominant
c)Nervedeafness
d)Glomerulonephritis
CorrectAnswer-B
Ans.is'b'i.e.,Autosomaldominant
oAutosomaldominantformalsoexist,butitisveryrare.Thus,
amongthegivenoptions,itisthebestanswer.oOtherthreeoptions
areclassicalfeaturesofAlport'ssyndrome(seeprevious
explanations).

471.Subepithelialdepositsinkidneyareseen
in-
a)MPGN-1
b)GPS
c)PSGN
d)All
CorrectAnswer-C
Ans.is'c'i.e.,PSGN
Glomerulardeposits:(i)Subepithelial:
o
AcuteGN(likePSGN)
oMembranousGN
oHeymanGN
oRPGN
(ii)Subendothelial:
(iii)Basementmembrane:
(iv)Mesangium:
MPGN(Type-I)
SLE
oAcuteON(H'1583)
oMPGN(TypeII)
oGoodpasturesyndrome
oIgAnephropathy,HSP
Remember
oAnionicantigensformsubendothelialdepositsoCationicantigens
formsubepithelialdeposits
oNeutralantigensformmesangial
deposits


472.NotafeatureofPSGN-
a)HTN
b)Increasedurea
c)Increasedcreatinine
d)NormalC3level
CorrectAnswer-D
Answer-D.NormalC3level
ComplementlevelisdecreasedinPSGN.Otheroptionsarecorrect.

473.Infollicularcarcinomachromosomal
translocationis?
a)PAXS-PPARTI
b)RET-PTC
c)ALK-NMPI
d)IAK-TEL
CorrectAnswer-A
Answer-A.PAXS-PPARTI
Follicular-PAX8-PPARrltranslocation

474.Tauproteinseenin?
a)Alzhiemer'sdisease
b)Lewybodydementia
c)Picksdisease
d)Amylodosis
CorrectAnswer-A
Ans.is`a'i.e.,Alzheimer'sdisease
Alzhiemer'sdisease:?
Thereisatrophyoffrontalandtemporallobestovariableextentand
severity.
Thepatternofatrophycanoftenbepredictedinpartbytheclinical
symptomatology.
Theatrophicregionsofcortexaremarkedbyneuronalloss,gliosis,
andthepresenceoftau-containingneurofibrillarytangles

475.Allofthefollowingarefeaturesof
granulomatousthyroiditisexcept?
a)Hyperthyroidism
b)Hypothyroidism
c)Painless
d)Giantcellsonhistology
CorrectAnswer-C
Answer-C.Painless
ClinicalFeatures-
Painfulenlargedthyroid,fever
Hypothyroidism
Malaise
Sorethroat,painreferredtothejaworear.
Subacutethyroiditisisaself-limitedthyroidconditionassociatedwith
atriphasicclinicalcourseofhyperthyroidism,hypothyroidism,and
returntonormalthyroidfunction

476.Sezarycellsshowwhichtlpeofnucleus-
a)Pleomorphic
b)Round
c)Eosinophillic
d)Cerebriform
CorrectAnswer-D
Answer-D.Cerebriform
SezarycellsareneoplasticT-cellsfoundinsezarysyndrome
(cutaneousT-celllymphoma).

477.Prusianbluedetects?
a)Ferriciron
b)Ferrousiron
c)Glycogen
d)Lipids
CorrectAnswer-A
Answer-A.Ferriciron
Itistheclassicmethodfordemonstratingironintissues.Thesection
istreatedwithdilutehydrochloricacidtoreleaseferricionsfrom
bindingproteins

478.Lysosomewithundigestedparticle
insideisknownas-
a)Residualbody
b)Phagosome
c)Phagolysosome
d)Autophagosome
CorrectAnswer-A
Answer-A.Residualbody
ThelysosomesthatpinchedofffromGolgicomplexarecalled
primarylysosomes.Afteraprimarylysosomehasfusedwiththe
vacuoleorvesiclecontainingthematerialtobedigested,itformsthe
secondarylysosome.Aftertheprocessofdigestionhasbeen
completed,asecondarylysosomeformstheresidualbody.

479.ThepredominantisozymeofLDHinLungis:
a)LD-1
b)LD-2
c)LD-3
d)LD-5
CorrectAnswer-C
IsoenzymesofLactateDehydrogenase:Lactatedehydrogenaseisatetramericenzyme
andconsistsoffoursubunits.Thesesubunitscanoccursintwoisoformsi.e.Hisoform(for
heart),Misoform(formuscle).
LDH-1(4H)-intheheartandRBCs
LDH-2(3H1M)-inthereticuloendothelialsystem
LDH-3(2H2M)-inthelungs
LDH-4(1H3M)-inthekidneys,placenta,andpancreas
LDH-5(4M)-intheliverandstriatedmuscle
NormalvalueofLDHinserumis100-200U/L.LDHlevelis100timesmoreinsidetheRBC
thaninplasma,andthereforeminoramountofhemolysiswillresultinafalsepositivetest.
Ref:Harper'sIllustratedBiochemistry,26thEdition,Page57;TextbookofBiochemistryBy
Vasudevan,5thEdition,Page53

480.Homerrosetteisseenin-
a)Neurobastoma
b)Nephroblastoma
c)Hepatoma
d)Ependymoma
CorrectAnswer-A
Answer-A.Neurobastoma
HomerWrightrosettes
Homer-Wrightrosettesarecharacteristicofneuroblastomasand
medulloblastomas.
Mayalsobeseenin-4Primitiveneuroectodermaltumors(PNET),
Pineoblastomas,Retinoblastomas

481.Neuroblastomas-goodprognositcfactor
is?
a)N-mycamplification
b)RASoncogene
c)Hyperdiploidy
d)Translocations
CorrectAnswer-C
Answeris'c'i.e.Hyperdiploidy

482.'Flare'inTripleresponseismediatedby:
a)Axonreflex
b)Arteriolardilation
c)Histaminerelease
d)Localhormones
CorrectAnswer-A
Ai.e.Axonreflex

483.HDLreceptoris-
a)SR-BI
b)LDLR
c)HDLR
d)SR-82
CorrectAnswer-A
Answer-A.SR-BI
HDLisremovedbyHDLreceptorsscavengerreceptorBI(SR-BI),
whichmediatetheselectiveuptakeofcholesterolfromHDL.
Thisreceptorismostabundantinliver,ovariesandadrenalglands.

484."Citronbodies"boatorleafshaped
pleomorphicorganisminanexudateis
a)Cl.welchii
b)Cl.edematiens
c)Cl.septicum
d)Cl.tetani
CorrectAnswer-C
Ans.is'c'i.e.,Clostridiumsepticum
Citronbodiesandboatorleafshapedpleomorphicbacilliwith
irregularstainingsuggestCLsepticum".
Clostridium
Clostridialspeciesare:
Grampositive
Anaerobic(obligateanaerobe)
Sporeforming
Bacilli
MotilebyperitrichateflagellaexceptCperfringensandC.tetanitype
IVwhicharenonmotile.
NoncapsulatedexceptC.perfringensandC.butyricumwhichare
capsulated
Pathogenesisisduetoexotoxinnotendotoxin.
Clostridiumbotulinumcausesbotulismnotgasgangrene.
Gasgangreneiscausedby:
C.perfringens(80%)Cnovyi
C.septicumC.histolyticum

485.Ghon'sfocusreflects:
September2005
a)Miliarytuberculosis
b)Primarycomplex
c)Tuberculouslymphadenitis
d)Postprimarytuberculosis
CorrectAnswer-B
Ans.B:Primarycomplex
OnlyaverysmallpercentofMycobacteriumtuberculosis(MTB)
infectionsresultindisease,andevenasmallerpercentageofMTB
infectionsprogresstoanadvancedstage.
Thebacilliisengulfedbyalveolarmacrophagesmultiplyandgive
risetoasubpleuralfocusoftuberculouspneumonia,commonly
locatedinthelowerlobeorthelowerpartoftheupperlobe.Thisis
knownasGhonfocus.
TheGhonfocustogetherwiththeenlargedhilarlymphnode
constitutestheprimarycomplex.
SmallmetastaticfocicontaininglownumbersofMTBmayalso
calcify.
However,inmanycasesthesefociwillcontainviableorganisms.
ThesefociarereferredtoSimonfoci.
TheSimonfociarealsovisibleuponchestX-rayandareoftenthe
siteofdiseasereactivation.

486.Calrexinandcalreticulinare-
a)Glycoproteins
b)Chaperons
c)Tumormarkers
d)Enzymes
CorrectAnswer-B
Answer-B.Chaperons
Calreticulin&calrexinaremajorCa2+binding(storage)chaperones
intheendoplasmicreticulum.

487.Whichofthefollowingispotassium
Channelopathy-
a)Hypokalemicperiodicparalysis
b)Hyperkalemicperiodicparalysis
c)EpisodicataxiaI
d)LongQT-syndrome
CorrectAnswer-A
Answer-A.Hypokalemicperiodicparalysis
Dent'sdisease(Xlinkedproteinuria&Kidneystones)
Osteopetrosis(recessiveordominant)
BartersyndrometypeIII
BartersyndrometypeIV(associatedwithsensorineuraldeafness)
Hyperkplexia(startledisease)
Juvenilemyoclonusepilepsy
Epilepsy

488.COXtype3isaproductof-
a)COXIgene
b)COX2gene
c)COX3gene
d)Noneoftheabove
CorrectAnswer-A
Answer-A.COXIgene
TheCOX-3isozymeisencodedbythesamegeneasCOX-1
(PTGS1gene),withthedifferencethatCOX-3retainsanintronthat
isnotretainedinCOX-1.Itisnotfunctionalinhumans.

489.Whichofthefollowingisfalseregarding
carcinoidtumor?
a)Neuroendocrinetumor
b)Mostcommonsiteislung
c)Associatedwithserotoninproduction
d)Potentiallymalignanttumor
CorrectAnswer-B
Answer-B.Mostcommonsiteislung
Carcinoidtumorsarisefromtheneuroendocrinecells(Argentaffin
cellsorKulchitskycells).
ThemajorityarefoundnGItract,andmorethan40%insmall
intestine(jejunum&ileum).Thetracheobronchialtreeandlungsare
thenextcommonsitesinvolved.
Carcinoidtumorsmayrarelyarisefromtheovaryorthymus
Carcinoidtumorsarethemostcommonmalignancyoftheappendix.
Allcarcinoidsareconsideredtohavemalignantpotential.
About10%ofcarcinoidssecreteexcessivelevelsofarangeof
hormones,mostnotablyserotonin(5-HT),leadingtocarcinoid
syndrome.ItischaracterizedbyFlushing,Diarrhea,Wheezing,
Abdominalcramping,Peripheraledema

490.Fibrosisassociatedwithlivercirrhosisis
mediatedby-
a)PDGF
b)IFN-y
c)ICAM-1
d)PcAM-l
CorrectAnswer-A
Answer-A.PDGF
Mostimportantmediatorsinvolvedinliverfibrosis-PDGF,PAF,
MMPs,TNF-a,TGF-13,IL-1.

491.Solutioncurrentlyusedforliver
preservationfortransplantis-
a)UWsolution
b)IGLsolution
c)KyotoETsolution
d)RossMarshalCitratesolution
CorrectAnswer-A
Answer-A.UWsolution
UWsolutionhasbecomethegoldstandardlivertransplantationfor
manyyears."

492.Skininvolvementalongwithcollarstud
ulcerationinintestineonradiography.
Diagnosisis-

a)TBintestine
b)Ulcerativecolitis
c)IntestinalAmebiasis
d)Crohn'sdisease
CorrectAnswer-B
Answer-B.Ulcerativecolitis
SkininvolvementcanoccurinInflammatoryboweldiseasei.e.both
Crohn'sdiseaseandUlcerativecolitis.
CollarstudulcerationisradiographicalsignofUlcerativecolitis.
ULCERATIVECOLITIS:
PATHOLOGY:
Colonicmucosalinflammation;rectumalmostalwaysinvolved,with
inflammationextendingcontinuously(noskipareas)proximallyfora
variableextent;histologicfeaturesincludeepithelialdamage,
inflammation,cryptabscesses,lossofgobletcells.
CLINICALMANIFESTATIONS
Bloodydiarrhea,mucus,fever,abdominalpain,tenesmus,weight
loss;thespectrumofseverity(amajorityofcasesaremild,limitedto
rectosigmoid).Inseverecases,dehydration,anemia,hypokalemia,
hypoalbuminemia.
COMPLICATIONS
Toxicmegacolon,colonicperforation;cancerriskrelatedtoextent
anddurationofcolitis;oftenprecededbyorcoincidentwith
dysplasia,whichmaybedetectedonsurveillancecolonoscopic
biopsies.

biopsies.
DIAGNOSIS
Sigmoidoscopy/colonoscopy:mucosalerythema,granularity,
friability,exudate,hemorrhage,ulcers,inflammatorypolyps
(pseudopolyps).Bariumenema:lossofhaustrations,mucosal
irregularity,ulcerations.

493.Whichofthefollowingmarkersis
specificforgastro-intestinalstomal
tumor(GIST)-

a)CD117
b)CD34
c)CD23
d)S-100
CorrectAnswer-A
Ans.is'a'i.e.,CD117
lmmunohistochemistry
oFollowingmarkersarepresentinGISTs:
i)CD117(ckit)-->95%
ii)CD34-->70%
iii)Smoothmuscleactin5%
CD117(ckit)isconsideredthemostspecificmarker
oCD117(ckit)immunoreactivityisthebestdefiningfeatureof
GISTsdistinguishingthemfromtruesmoothmuscletumors
(leiomyoma)andtumorsarisingfromneuralcrest.
oAlthoughCD117(ckit)isconsideredthemostspecificmarkerfor
GIST,itisnotpathognomonicofGISTasothertumorsmayalso
expressCD117.Thesetumorsincludemastcelltumor,germcell
tumors(seminomas),leukemias,malignantmelanoma,
angiolipomas,andsomesarcomas.


494.Primaryhyperparathyroidism,phaeochromocytomamaybeassociatedwith
whichtypeofthyroidcancer?
a)Medullarycarcinomaofthethyroid
b)Papillarycarcinomaofthethyroid
c)Anaplasticcarcinomaofthethyroid
d)Follicularcarcinomaofthethyroid
CorrectAnswer-A
Itischaracterizedclinicallybymedullarythyroidcarcinoma(MTC),
pheochromocytoma,andhyperparathyroidism.Approximately70-
95%ofindividualswithMEN2AdevelopMTC,50%develop
pheochromocytoma,and15-30%develophyperparathyroidism.
Ref:BaileyandLove'sShortPracticeofSurgery,24thEdition,Page802;Harissons
InternalMedicine,18thEdition,Chapter351.

495.Featureofmicroscopicpolyangitisis:
a)IgGdepositsinkidney
b)Bronchospasm
c)Renalinvolvementin80%ofcases
d)Alloftheabove
CorrectAnswer-C
AnswerisC(Renalinvolvementin80%ofcases):
Renalinvolvementisseeninatleast80%ofpatientswithMPA.
Renalinvolvementisseeninatleast80%ofpatientswithMPA
Renalinvolvementisseeninatleast80%ofpatientswithMPA-
CRDTGlomerulonephritisoccursinatleast79%ofpatients?
Harrison
MicroscopicpolyangitisisnotassociatedwithIgGDepositsin
kidney
MicroscopicPolyangitisisapauci-immuneglomerulonephritis.
Immunofluorescenceandelectronmicroscopyshownoimmune
deposits
MicroscopicPolyangitisisnotassociatedwithbronchospasm
Asthma(bronchospasm)andEosinophiliaarefeaturesofChurg-
StraussSyndromeandaretypicallyabsentinmicroscopic
polyangitis.

Features
H.S.
Churg-
purpura Microscopic Wegner's
Strauss
Polyangitis Granulomatosis syndrome
Depositsin
--nt(IgA

kidney
deposits)
-
Bronchospasm -
-
-
(Asthma)
+
Eosinophilia
-
-
-
+

Eosinophilia
-
-
-
+
Predominant
-
p-ANCA
c-ANCA
p-ANCA
ANCA

496.Firstorderkineticsis:
September2005
a)Absorptionofthedrugisindependentoftheserum
concentration
b)Eliminationofthedrugisindependentoftheserum
concentration.
c)Eliminationofthedrugisproportionaltotheserum
concentration
d)Absorptionofthedrugisproportionaltotheserum
concentration
CorrectAnswer-C
Ans.C:Eliminationofthedrugisproportionaltotheserum
concentration.
A0-orderkineticshasaratewhichisindependentofthe
concentrationofthereactant(s).Increasingtheconcentrationofthe
reactingspecieswillnotspeeduptherateofthereaction.Zero-
orderreactionsaretypicallyfoundwhenamaterialthatisrequired
forthereactiontoproceed,suchasasurfaceoracatalyst,is
saturatedbythereactants.
Withfirst-orderelimination,therateofeliminationisdirectly
proportionaltotheserumdrugconcentration(SDC).Thereisa
linearrelationshipbetweenrateofeliminationandSDC.Although
theamountofdrugeliminatedinafirst-orderprocesschangeswith
concentration,thefractionofadrugeliminatedremainsconstant.
Theeliminationrateconstant(Kel)representsthefractionofdrug
eliminatedperunitoftime.

497.Alkalinediuresisinacidicdrug
poisoningisnotdonein?
a)Aspirin
b)Methotrexate
c)Morphine
d)Phenobarbitone
CorrectAnswer-C
Ans.C.Morphine
Acidicdrugs(barbiturate,methotrexate,salicylate)aremoreionized
atalkalineurineandarenotabsorbedfromrenaltubulesiftheurine
isalkaline,andtheirexcretioninurineisincreased.
Therefore,alkalinizationofurineisdonebyNaHCO2inpoisoningof
acidicilrugs.

498.Postmarketingsurveillanceincludedin
whichphaseofdrugclinicaltrial?
a)I
b)II
c)III
d)IV
CorrectAnswer-D
Ans.is'd'i.e.,IV
Surveillanceaftermarketing,i.e.afterthedrugisoutinthemarketis
apartofPhaseIVofclinicaltrials.Itincludesfollow-upofpatients
takingthedrugandadversedrugreaction(ADR)reportingaswell
aslookingfornewertreatmentindications


499.PermissionfromDCGI[Drugcontroller
general,India]isneededbeforewhich
phaseofdrugtrial?

a)Phase1
b)Phase2
c)Phase3
d)Phase4
CorrectAnswer-A
Ans,.A.Phase1
Followingaretheprerequisitesforstartingaclinicaltrialin
India:-

1. PermissionfromDCGl
2. Approvalfromethicscommittee
3. MandatoryregistrationontheICMRmaintainedwebsitewwwctri.in

500.CYP3Ainhibitorsis/are-
a)Ritonavir
b)Amiodarone
c)Verapamil
d)aandc
CorrectAnswer-D
Ans.is'a'i.e.,Ritonavir;'c'i.e.,Verapamil
CYP3A4/3A5inhibitorsare
oRitonaviroErythromycinoItraconazoleo
TroieandomycinoVerapamil
oClarithromycinoAzamulinoDiltiazemo
Ketoconazole
oGrapefruitjuice(Furanocoumarins)

501.Regardingefficacyandpotencyofadrug,allaretrue,EXCEPT:
a)Inaclinicalsetup,efficacyismoreimportantthanpotency
b)Inthelogdoseresponsecurve,theheightofthecurve
correspondswithefficacy
c)ED50ofthedrugcorrespondstoefficacy
d)Drugsthatproduceasimilarpharmacologicaleffectcanhave
differentlevelsofefficacy
CorrectAnswer-C
ED50referstoEffectiveDoseofadrugneededtoproduceaparticularresponsein50%
ofpopulation.Itisaquantitativemeasureofthepotencyofadrug.SmallertheED50value,
morepotentisthedrug.
Ref:EncyclopediaofPsychopharmacologyByIanP.Stolerman,Volume2,Page456

502.Volumeofdistributionofadrugis500ml
andtargetconcentrationofdruginblood
is5g/L.20%ofadministereddrugis
reachedtosystemiccirculation.What
willbetheloadingdoseofthatdrug-

a)1gm
b)5gm
c)12.5gm
d)25gm
CorrectAnswer-C
Ans.C.12.5gm
Loadingdose=(TargetCp*Vd)/F
Targetconcentration(Cp)=5gm/L
Volumeofdistribution=500ml=0.5L
F(Fractionofadministereddrugreachessystemiccirculation)=
20Vo=0.25x0.5
So,loadingdose=r2.5gm

503.Volumeofdistributiondependsuponall
except?
a)Drugdose
b)Plasmaconcentration
c)Extentofabsorption
d)Halflifeofdrug
CorrectAnswer-D
AnS.D.Halflifeofdrug
Afteradrugreachestheblood,itmaybedistributedtovarious
tissues.
Thisisdeterminedbyahypotheticalparameter,volumeof
distribution.
Volumethatwouldaccommodateallthedruginthebody,ifthe
combinationthroughoutwasthesameasinplasmaiscalledvolume
ofdistribution.
Orinsimplewords,itkthefluidvolumethatwouldberequiredto
containalltheadministereddruginthebodywithaconcentration
equaltoplasma.

504.Highvolumeofdistributiondependson
?
a)Highplasmaproteinbinding
b)Lipidsolubility
c)Elimination
d)Halflife
CorrectAnswer-B
Ans.B.Lipidsolubility
[Ref:Clinicalpharmacology3d/ep.31]
Itadrughashighvolumeofdistribution(>42L),thedrugisthought
tobedistributedtoalltissuesofthebody,especiallyfattytissue.
Agivendrugwillhavehighvolumeofdistribution,ifithas:
Highlipidsolubility(non-polardrug)
Lowrateofionization
Lowplasmaproteinbinding

505.Drugswhichisnotmetabolizedby
acetylation?
a)Dapsone
b)Metoclopramide
c)Procainamide
d)INH
CorrectAnswer-B
Ans.B.Metoclopramide
Drugsmetabolizedbyacetylation:
Sulfonamides(includingdapson)rProcainamide
INH
Hydralazine
PAS
Clonazepam

506.Trueaboutdrugmetabolism?
a)GlucuronidationisphaseIreaction
b)MostcommonenzymeinvolvedisCyp3A4/5
c)Reductionismostcommonreaction
d)Cytochromep450isinvolvedphase-IIreaction
CorrectAnswer-B
Ans.is'b'i.e.,MostcommonenzymeinvolvedisCyp3A4/5
(Ref:KatzungIlth/ep.55)
Cytochromep450enzymesaremicrosomalenzymesthatare
involvedinphaseImetabolismofmanydrugs.
MostofthedrugsaremetabolizedbyCyp3A4isoform.
Cyp3A4/5carryoutbiotransformationoflargestnumber(nearly
5O%)ofdrugs.

507.Whichistopicalwayofdrug
administration?
a)Inhaledsteroid
b)Transdermalpatch
c)SublingualNTG
d)Rectaldiazepam
CorrectAnswer-A
Ans,A.Inhaledsteroid
InhaledCorticosteroids(usedinasthma)havehightopicalactivity.
Theyactlocallytoreduceinflammationandhyper-reactivityof
bronchialtree.
Transdermalpatch,sublingualNTGandrectaldiazepamare
systemicroutesofdrugadministration.

508.Trueabouttransdermaldrugdelivery
systemareallexcept?
a)Appliedtochest,abdomenandback
b)Drugisdeliveredataconstantrate
c)Goodoptioninemergencysituations
d)Fentanylisused
CorrectAnswer-C
Ans,C.Goodoptioninemergencysituations
Themicroporemembraneintransdermalpatchissuchthatrateof
drugdeliverytoskinsurfaceislessthantheslowestrateof
absorptionfromtheskin.
Thisoffsetsanyvariationintherateofabsorptionaccordingtothe
propertiesofdifferentsites.Assuch,thedrugisdeliveredata
constantandpredictablerateirrespectiveofsiteofapplication.
Usuallychest,abdomen,upperarm,lowerback,buttockormastoid
regionarcutilized.
TransdermalpatchesofGTN,fentanyl,nicotineandestradiolare
availableinIndia,whilethoseofisosorbidedinitrate,hyoscine,and
clonidinearemarketedelsewhere.

509.Physiologicalantagonistsare?
a)AdrenalineandIsoprenaline
b)Histamineandadrenaline
c)Isoprenalineandpropranolal
d)Alloftheabove
CorrectAnswer-B
Ans.B.Histamineandadrenaline
Physiologicalantagonistsarethosethatproduceoppositeactionby
actingondifferentreceptors.
Example
HistaminecausesbronchoconstrictionviaH1receptorsandthis
actionisantagonizedbyadrenalinewhichcausesbronchodilatation
throughbeta2receptors(optiond).

510.Receptorlevelantagonismisshownby?
a)AdrenalineandIsoprenaline
b)Histamineandadrenaline
c)Isoprenalineandpropranolol
d)Alloftheabove
CorrectAnswer-C
AnS.C.Isoprenalineandpropranolol
Receptorantaoonists(Pharmalogicalantaqonists)
Receptorantagonistsarethosedrugsthatblockstheactionof
agonistbyactingonsamerecePtors.
Example
Isoprenalineisbeta1&beta2receptoragonistwhilepropranololhas
antagonisticactiononbeta1&beta2recePtors(option'c')

511.Fastestreceptormediatedactionis
through?
a)Cellmembranereceptors
b)Intrinsicionchannels
c)Enzymelinkedreceptors
d)Intracellularreceptors
CorrectAnswer-B
Ans.B.Intrinsicionchannels
Fastestactingreceptors-Receptorswithintrinsicionchannels
Slowestactingreceptors?Intracellularreceptors(receptors
regulatinggeneexpressions/transcriptionfactors)?Cytoplasmicor
nuclearreceptors.

512.Sideeffectsofclonidineareallexcept?
a)Xerostomia
b)Sedation
c)Impotency
d)Diarrhea
CorrectAnswer-D
Ans,.D.Diarrhea
Adverseeffectsofclonidineare
(i)drynessofmouth(xerostomia),nose'eye,(ii)sedation,(iii)mental
depression'(iv)constipationand(v)impotency.
Ithasnoeffectonlipidprofile.

513.Lipidinsoluble(3-blokceris-
a)Timolol
b)Carvedilol
c)Pindolol
d)Celiprolol
CorrectAnswer-D
Ans.D.Celiprolol
Lipidinsolublebetablocker:
Acebutolol
Atenolol
Bisoprolol
Betoxalol
Carteolol
Celiprolol
Esmolol
Nodalol
Sotalol
Labetalol

514.Nonselectivea+1blockeris?
a)Carvedilol
b)Timolol
c)Pindolol
d)Acebutolol
CorrectAnswer-A
Ans.A.Carvedilol
Combinedalpha&betablockers:
Carvedilol
Labetalol
Bucindolol
Bevantolol
Nipradilol
Dilevalol
Medroxalol

515.Non-selectiveBeta-blockerwith
sympathomimeticactivity?
a)Pindolol
b)Acebutalol
c)Nodalol
d)Metoprolol
CorrectAnswer-A
Ans.A.Pindolol
Beta-blockerswithintrinsicsympatheomimetic(partialagonist)
activity:
Acebutolol
Carteolol
Pindolol(non-selectivebetablocke).
Bipindolol
Oxprenolol
Penbutolol
Alprenolol
Labetalol
Celiprolol

516.Whichofthefollowingisnon-selective
3rdgenerationBetablocker?
a)Betaxolol
b)Celiprolol
c)Carteolol
d)Nadolol
CorrectAnswer-C
Ans.C.Carteolol
Nonselectivethirdgenerationbeta-blockersarecorteolol,Carvedilol
andlabetalol.

517.Dopaminereceptorwithinhibitoryaction
?
a)D5
b)Di
c)D2
d)None
CorrectAnswer-C
Ans.is'c'i.e.,D2
Twotypesofdopaminereceptors(DI,D2)wereoriginally
described.Threemore(D3,D4,D5)havenowbeenidentifiedand
cloned.AllareGproteincoupledreceptorsandaregroupedintotwo
families:
Dllike:(Dl,D5)areexcitatory
D2like:(D2,3,4)areinhibitory

518.ActionofM,cholinergicreceptors?
a)Skeletalmusclecontraction
b)Acidsecretioninstomach
c)Decreasedheartrate
d)Salivationandlacrimation
CorrectAnswer-C
Ans.C.Decreasedheartrate
Effectofcholinergicsystemonheart(e.g.decreasedheartrate)is
throughM2receptors.

519..Beta-blockershouldbeusedwith
cautioninpatientof-
a)Hypertension
b)Glaucoma
c)Conductiondefect
d)CHF
CorrectAnswer-C
Ans.C.Conductiondefect
Partialandcompleteheartblock
Beta1receptorsincreaseconductioninAVnode.
Betablockersdecreaseconductionbyreducingsympatheticdrive
onbeta1receptors-Worseningofblock.

520.Antimuscarinicdrugusedinoveractive
bladder-
a)Pirenzepine
b)Trospium
c)Tropicamide
d)Atropine
CorrectAnswer-B
Ans.B.Trospium
Drugsusedforoveractivebladder:
Darifenacin,Solefenacin,Tolterodine,Trospiumchloride,
Oxybutynin,Solifenacin,Flavoxate

521.Anti-cholinesterasewithcentralaction?
a)Neostigmine
b)Physostigmine
c)Pyridostigmine
d)Edrophonium
CorrectAnswer-B
Ans.B.Physostigmine
Lipidsolubleagents(organophosphatesandphysostigmine)have
moremarkedmuscarinicandCNSeffect;andstimulategangliabut
actiononskeletalmuscleislessprominent.

522.Contraindicationofantimuscarinicdrug
?
a)Glaucoma
b)Asthma
c)Pepticulcer
d)Urinaryincontinence
CorrectAnswer-A
Ans.A.Glaucoma
Antimuscarinic(e.g.-atropine)drugsarecontraindicatedin:

1. Glaucoma
2. Benignprostatichyperplasia

523.Maximumcycloplegicactionofatropine
isseenat?
a)30-40minutes
b)1-3hours
c)8-10hours
d)1-2weeks
CorrectAnswer-B
Ans.B.1-3hours
Atropineisapowerfulcycloplegicandmydriaticagent.
Mostpotentcycloplegicavailableforoptometrists.
Maximummydriasisisreachedtypicallyin30-40minuteswhile
recoverymaytakeaweekormore.
Cycloplegiacommencesafter30minutesofapplication,withmarked
cycloplegiabeingreachedin1-3hours.
Theeffectmaylastupto6-12daysbeforenormalaccommodationis
restored.

524.Oneofthefollowingisnotasideeffect
ofatropine?
a)Blurringofvision
b)Diarrhoea
c)Urinaryretention
d)Confusionofelderly
CorrectAnswer-B
Ans.B.Diarrhoea
Sideeffectsandtoxicityofatropine
Drymouth
Fever
Difficultyinswallowing
Constipation
Dry,flushedhotskin
Difficultyinmicturition
Palpitation
Delirium
Dilatedpupil
Photophobia
Blurredvision
Respiratorydepression
Cardiovascularcollapse
Psychoticbehavior
Convulsionandcoma

525.Longestactinganti-cholinesterase-
a)Pyridostigmine
b)Ambenonium
c)Edrophonium
d)Echothiophate
CorrectAnswer-D
Ans.D.Echothiophate
Therearetwobasiccategoriesofcholinesteraseinhibitors:

1. Reversibleinhibitors
2. Irreversibleinhibitors
Thereversibleinhibitorsproduceeffectsofmoderateduration,and
theirreversibleinhibitorsproduceeffectsoflongduration.
Echothiophateisirreversibleinhibitor?longacting.

526.Propranololisusedin?
a)Thyrotoxicosis
b)AVblock
c)Cardiacarrest
d)Alloftheabove
CorrectAnswer-A
Ans.A.Thyrotoxicosis
Usesofbeta-blockers:
Hypertension.
Cardiactachyarrhythmias
Myocardialinfarction
Classicalanginapectoris
CHF
Dissectinganeurism
HOCM
Glaucoma
Thyrotoxicosis
Pheochromocytoma
CNSuses-Anxiety,essentialtremor,migraineprophylaxis,alcohol
withdrawal.
EmergencymanagementofsymptomsofTOF.
Prophylaxisofbleedinginportalhypertension.

527.Maximumpotassiumlossiscausedby
whichdiuretics?
a)Furosemide
b)Thiazide
c)Acetazolamide
d)Spironolactone
CorrectAnswer-C
Ans.C.Acetazolamide
ForthesamedegreeofnatriuresisCAseinhibitorscausesmost
markedkaliuresiscomparedtootherdiuretics.

528..Hyoscineisantagonistatwhich
cholinergicreceptor?
a)Muscarinic
b)Nicotinic
c)Both
d)None
CorrectAnswer-A
Ans.A.Muscarinic
Hyoscine(Scopolamine)actsbycompetitiveantagonismof
acetylcholineatmuscarinicreceptors(Non-selectivereceptors).
Ithaslittleeffectatnicotinicreceptors.

529.Fenoldopamisusedinthemanagementof?
a)Hypertensiveemergencies
b)Congestiveheartfailure
c)Migraineprophylaxis
d)Tachyarrhythmias
CorrectAnswer-A
Ans.A.Hypertensiveemergencies
Fenoldopam
Itisperipheral,arteriolardilatorusedinhypertensiveemergencies
andpost-operativehypertension.
ItactsasanagonistofdopamineD1receptors,resultingindilatation
ofperipheralarteriesandnatriuresis.
Fenoldopamincreasesintraocularpressure>shouldbeavoidedin
patientswithglaucoma.

530.Trueaboutcardiacmusclefibers?
a)Digitalisdecreasesforceofcontraction
b)Na+-Ca+exchangerrequiresATPdirectly
c)Na+-Ca+exchangeractstopumpCa2+intoheartmusclecells
d)Allaretrue
CorrectAnswer-C
Ans.C.Na+-Ca+exchangeractstopumpCa2+intoheart
musclecells
3Na+/1Ca+2exchanger
ThispumptransportsCa+2inexchangeofNa+.
WhenNa+concentrationinsidethemyocyteishigh,Na+/Ca+2
exchangercauseeffluxofNa+outofthemyocytesandinexchange
itcausesinfluxofCa+2insidethemyocytes.
Na+/Ca+2exchangerdoesnotrequireATPtofunction,ionsmove
alongtheirconcentrationgradient.

531.Sideeffectsofamiodaroneareallexcept
?
a)Hyperthyroidism
b)Peripheralneuropathy
c)Skindiscoloration
d)Hyperglycemia
CorrectAnswer-D
Ans.D.Hyperglycemia
AdverseeffectsofAmiodarone
Thyroiddysfunction(Hypothyroidismorhyperthyroidism)
Bluishdiscolorationofexposedskin.
Peripheralneuropathy
Myocardialdepression
Pulmonaryfibrosis
Cornealmicrodeposits
Photosensitivity
Hepatitis
Thrombocytopenia(Majorspecificsideeffect).

532.Whichamongthefollowingisrenin
antagonist?
a)Losartan
b)Benazepril
c)Remikiren
d)Imidapril
CorrectAnswer-C
Ans.C.Remikiren
Renininhibitors:Aliskiren,remikiren,enalkiren.
Aliskiren,remikiren,andenalkirenarethedrugsthatinhibitthe
enzymerenin.
SothesedrugsdecreasetheactivityofRAAScausingafallinblood
pressure.
Thesedrugscanbeusedorallyforthetreatmentofchronic
hypertension.

533.Antihypertensivedrugcausingerectile
dysfunction?
a)Calciumchannelblocker
b)ACEinhibitors
c)AT1receptorantagonists
d)13-blockers
CorrectAnswer-D
Ans.D.13-blockers
Importantdrugscausingerectiledysfunction
Beta-blockers
Diuretics(especiallythiazide)
Lithium
Clonidine
OCPs
TCAs&SSRIs
Sedatives/hypnotics

534.Thiazidescausehypercalcemiaby?
a)IncreasedCa"absorption
b)IncreasedPTHsecretion
c)Decreasedcalcitoninsecretion
d)Decreasedcalciumexcretion
CorrectAnswer-D
Ans.D.Decreasedcalciumexcretion
Thiazidescausehypercalcemiaby:-
Reducedurinaryexcretionofcalciumduetoadirecttubulareffector
ECFdepletionwithsecondaryincreaseintubularreabsorptionof
sodiumandcalcium,orboth.
IncreasedboneresponsivenesstotheresorptiveactionofvitaminD
andPTH.

535.Drugaffectingpositivefreewater
clearancewithoutaffectingnegativefree
waterclearance-

a)Loopdiuretics
b)Thiazides
c)Acetazolamide
d)Amiloride
CorrectAnswer-B
Ans.b.Thiazides
Loopdiureticsabolishesthecortico-medullaryosmoticgradientand
blockspositiveaswellasnegativefreewaterclearance.
Thiazidesdecreasepositivefreewaterclearancewithoutaffecting
negativefreewaterclearance.

536.Whichofthefollowingadverseeffectof
ACEinhibitorsisnotduetobradykinin?
a)Cough
b)Angiodema
c)Hypotension
d)Noneoftheabove
CorrectAnswer-C
Ans.C.Hypotension
BradykininandsustanceParesubstrateforACE.
ACEinhibitorsincreaselevelofthesekininsbyinhibitingACE,which
isresponsibleforcoughandangiodema.
Coughandangioedemaareduetoelevatedbradykinin,causedby
inhibitionofbradykinin/srbstancePmetabolisminlungs.

537.WhichACEinhibitorinsafeinrenal
failure?
a)Captopril
b)Enalapril
c)Benazapril
d)None
CorrectAnswer-C
Ans.C.Benazapril
Benazaprilconferrredsubstantialrenalbenefitsinpatientswithout
diabeteswhohadadvancedrenalinsuficiency".
Benazaprilisconsideredsafeinrenalfailure.

538.Sympathomimeticdrugwhichcauses
decreaseinheartrate?
a)Adrenaline
b)Isoprenaline
c)Noradrenaline
d)None
CorrectAnswer-C
Ans.C.Noradrenaline

539.Followingarethesideeffectsof
thiazidesexcept?
a)Hypokalemia
b)Hypocalcemia
c)Hepaticcoma
d)Impotence
CorrectAnswer-B
Ans.is'b'i.e.,Hypocalcemia
Followingarethesideeffectsofthiazides:
Hypokalemia
Acutesalinedepletion,hemoconcentrationandincreasedriskof
peripheralvenousthrombosis
Dilutionsalhyponatremia
Nauseaomittingdiarrhea
Rarelyheadache,giddiness,weakness,parethesias,impotence
Hearingloss
Rashes,photosensitivity
Hyperuricemia
HyperglycemiahyperlipidemiaoHypercalcemia
Magnesiumdepletion
Aggravatedrenalinsufficiency
Briskdiuresisleadingtomentaldisturbanceandhepaticcoma

540.SpironolactoneshouldNOTbegivenwiththefollowingpharmacological
agent:
a)Chlorothiazide
b)b-blocker
c)ACEinhibitors
d)Amlodipine
CorrectAnswer-C
Unlikemostotherdiuretics,K+-sparingdiureticsreduceurinaryexcretionofK+andcan
causemild,moderate,orevenlife-threateninghyperkalemia.
Theriskofthiscomplicationisgreatlyincreasedbyrenaldisease(inwhichmaximalK+
excretionmaybereduced)orbytheuseofotherdrugsthatreduceorinhibitrenin(beta
blockers,NSAIDs,aliskiren)orangiotensinIIactivity(angiotensin-convertingenzyme
inhibitors,angiotensinreceptorinhibitors)
.
SincemostotherdiureticagentsleadtoK+losses,hyperkalemiaismorecommonwhen
K+-sparingdiureticsareusedasthesolediureticagent,especiallyinpatientswithrenal
insufficiency.
Ref:IvesH.E.(2012).Chapter15.DiureticAgents.InB.G.Katzung,S.B.Masters,A.J.
Trevor(Eds),Basic&ClinicalPharmacology,12e.

541.Na*-K*-2Cl-isinhibitedby-
a)Thiazides
b)Acetazolomide
c)Furosemide
d)Amiloride
CorrectAnswer-C
Ans.C.Furosemide
Highefficacy(highceilingorloopdiuretics-inhibitorsofNa+-
K+-2Cl-)
Furosemide
Bumetanide
Torasemide
Ethacrynicacid

542.Foractivityofantipsychotic,actionis
requiredatwhichreceptor-
a)M,muscarinic
b)D1dopaminergic
c)DZdopaminergic
d)5HT4serotonergic
CorrectAnswer-C
Ans.C.DZdopaminergic
TypicalantipsychoticactsbyblockingD2receptors(D2
antagonists).

543.Mechanismofactionofopioids?
a)Inhibitionofcyclooxylgenase
b)Inhibitionofopioidreceptorsatspinallevel
c)Inhibitionofopioidreceptorsatsupraspinallevel
d)Inhibitionofopioidreceptorsatspinalandsupraspinallevel
CorrectAnswer-D
Ans.D.Inhibitionofopioidreceptorsatspinalandsupraspinal
level
Opioidshavebothspinal&supra-spinalcomponentsfortheir
analgesiceffects.

544.Opioid[morphine]causes?
a)Increasedheartrate
b)Increasedmuscletone
c)Mydriaris
d)Respiratorystimulation
CorrectAnswer-B
Ans.B.Increasedmuscletone
Morphinecausesbothstimulantanddepressiveeffects.
OnCNSstimulation(Corticalareaandhippocampus)morphine
causesmuscularrigidity&convulsions.

545.Besidedepression,otheruseofSSRIs?
a)Erectiledysfunction
b)Retrogradeejaculation
c)Prematureejaculation
d)Sterility
CorrectAnswer-A:C
Ans.A.Erectiledysfunction&C.Prematureejaculation
SSRIsareeffectivelyusedinprematureejaculation.
SSRIscancauseprolongationofpreorgasmicplateauandthus
delayejaculation.
SSRIscanalsobeusedinerectiledysfunctionsecondaryto
depression.
OtherusesofSSRIs:
Depression(mostcommonuse)
OCD
Panicdisorder
Socialphobia
PTSD
Generalizedanxietydisorders
Pre-mensturaldysphoricdisorders

546.Lithiumcausesallexcept-
a)Polyuria
b)Nephropathy
c)Ebstein'sanomaly
d)Hyperthyroidism
CorrectAnswer-D
Ans.isdi.e.,Hyperthyroidism
Lithiumisknowntoexacerbatepsoriasisandcauseacne.
ItisknowntocauseEbstein'sanomalyinchildren.
Italsocausesthyroiddysfunction,hypothyroidismandnot
hyperthyroidism.
Lithiumnephrotoxicityiswellknown.

547.Mostcommonrenalsequeloflithium
toxicityis?
a)NephrogenicDM
b)Renaltubularacidosis
c)Glycosuria
d)MPGN
CorrectAnswer-A
Ans.is'a'i.e.,NephrogenicDM
Lithiumassociatedrenaltoxicity
Theuseoflithiumsaltsforthetreatmentofmanic-depressiveillness
mayhaveseveralrenalsequelae,themostcommonofwhichis
nephrogenicdiabetesinsipidusmanifestingaspolyuriaand
polydipsia.
Lithiumaccumulatesinprincipalcellsofthecollectingductby
enteringthroughtheepithelialsodiumchannel(ENaC),whereit
inhibitsglycogensynthasekinase3anddown-regulates
vasopressin-regulatedaquaporinwaterchannels.
Lessfrequently,chronictubulointerstitialnephritisdevelopsafter
prolonged(greaterthan10-20years)lithiumuseandismostlikelyto
occurinpatientsthathaveexperiencedrepeatedepisodesoftoxic
lithiumlevels.

548.Drug(s)notgivenastransdermalpatch:
a)Fentanyl
b)Diclofenac
c)Morphine
d)Clonidine
e)Buprenorphine
CorrectAnswer-B:C
Ans.B,Diclofenac&C,Morphine
[Ref:KDT7th/476
Transdermalfentanyl(Durogesic)hasbecomeavailableforusein
cancer/terminalillness.
Butransskinpatchescontainbuprenorphineanopioidpain
medication.
Clonidinetransdermaldelivery(patch)systemshavebeenavailable
sincethe1980

549.Antidepressantdrugusedinnocturnal
eneuresisis:
a)Imipramine
b)Fluoxetine
c)Trazdone
d)Sertaline
CorrectAnswer-A
Ai.e.Imipramine
AdverseeffectsofTCAs
l.Anticholinergic-Drymouth,badtaste,urinaryretention,blurred
vision,palpitation,constiPation.
2.Sedation,mentalconfusion,weakness.
3.Increasedappetiteandveigfitgain.
4.Sweatingandfinctremer.
5.Decreasedseizurethreshold(clomipramine,maprotiline&
bupropion).
6.Posturalhypotension>MarimumbyamitripSline-Goodman&
Gillman11/ep.4j3.
7.Cardiacarrythmia;Maximumbyamitriptylineanddosulpin.

550.Zonisamideactson?
a)GABAreceptors
b)TtypeCa2+channels
c)Na+Channels
d)Cl-channels
CorrectAnswer-C
Ans.C.Na+Channels
Anti-epilepticscausingprolongationofNa2+channel
inactivation:
Phenytoin
Carbamazepine
Valproate
Lamotrigine
Topiramate
Zonisamide

551.Buprenorphinepartialagonistatwhich
opioidreceptor?
a)Mu
b)Kappa
c)Delta
d)Lambda
CorrectAnswer-A
Ans.is'a'i.e.,Mu
Buprenorphineispartialagonistonmureceptorandantagonistat
Kappareceptor.

552.Dexmedetomidineactsonwhichreceptorforits
analgesicaction?
a)5HT2A
b)D2
c)2A
d)D5
CorrectAnswer-C
Ans.'c'i.e.,2A
Dexmedetomidineisacentrallyactiveselectivealpha(2)agonist
thathasbeenintroducedforsedatingcriticallyillventilatedpatients
inintensivecareunits.

553.Adverseeffectsofphenytoinincludeallofthefollowing
except?
a)Lymphadenopathy
b)Ataxia
c)Hypercalcemia
d)Hirsutism
CorrectAnswer-C
Ans.is'c'i.e.,Hypercalcemia
Phenytoininterfereswithcalciummetabolismbydesensitizingtarget
tissuetovit.D,thiscauseshypocalcemia(nothypercalcemia).
Attherapeuticlevel(10-20microg/ml)-Gumhypertrophy,hirsutism,
hypersensitivity(rashes,lymphadenopathy,DLE,neutropenia),
hyperglycemiaduetoinhibitionofinsulinrelease,megaloblastic
anemia,pseudolymphoma,hypocalcemia,VitaminDdeficiencyand
osteomalacia,andteratogenicity(fetalhydantoinsyndrome).
Attoxiclevel(dose-related)cerebellarsyndrome(ataxia,vertigo),
fallingBP,arrhythmias,drowsiness,mentalconfusion,Glsymptoms
(epigastricpain,nausea,vomiting)andlocalvascularinjurybyiv
injection.

554.Whichofthefollowingdrugisnotusedinthetreatmentofakathisia?
a)Benzodiazepam
b)Propranolol
c)Trihexyphenidyl
d)Haloperidol
CorrectAnswer-D
Akathisiareferstosomaticrestlessnesswhichoccurinpatientstreatedwithfirst
generationantipsychotics.
Amongtheoptionsgivenhaloperidolisanoldgenerationantipsychoticwhichcausesaside
effectofakathisia.
Drugscommonlyusedtotreatakathisiaarepropranolol,benzodiazepinesand
anticholinergics.
Ref:AmericanPsychiatricAssociationPracticeGuidelinesforthetreatmentofPsychiatric
Disorders2006,page649

555.BenzodiazepinebindingsteonGABA
receptorsison?
a)y-subunit
b)a-subunit
c)-subunit
d)-subunit
CorrectAnswer-B
Ans.is'b'i.e.,a-subunit
[Ref:Goodman(tGilman1I'h/ep.405,406;RecEttorsubunit&
coffiplexesp.168]
TheexactsubunitstructuresofnativeGABAreceptorsarestillare
unknown,butitisthoughtthatmostGABAreceptorarecomposed
ofalpha,beta&gammasubunitsthatCoassemblewithsome
uncertainstoichiometry.
BindingsiteofGABAisonbeta-subunit.
Benzodiazepinesiteislocatedontheasubunitbutthestabilization
orcompletionofthatsiteintheassembled.Structurealsorequires
theysubunit.

556.Whichofthefollowingistrue?
a)GlucocorticoidsupregulateMHCexpression
b)GlucocorticoidsactivateT-helpercells
c)GlucocorticoidsactivatecytotoxicTcells
d)Noneoftheabove
CorrectAnswer-A
Ans.A.GlucocorticoidsupregulateMHCexpression
GlucocorticoidsbothinhibitaswellasupregulateMHCexpression
dependinguponcellsandspeciesinvolved.
DexamethasoneincreasesMHCclass2expressiononhuman
endothelialcellsandmonocyteswhereasitdownregulatesitonB-
cells.
GlucocorticoidshaveinhibitoryeffectonbothT-Helpercellsand
cytotoxicTcells.

557.Betablockersusedinthyroidstorm
cause?
a)Quickreliefofsymptoms
b)Increasedmetabolismofthyroxine
c)Blockadeofthyroxinereceptors
d)Decreasedsynthesisofthyroxine
CorrectAnswer-A
Ans.A.Quickreliefofsymptoms
Non-selectivebeta-blocker:
Mostvaluablemeasureinthyroidstorm.
Inthyroidstormmostofthesymptomsarebecauseofadrenergic
overactivityduetoincreasedtissuesensitivitytocatecholaminesin
hyperthyroidism.
Thisincreasedsensitivityisduetoincreasednumberofp-receptors.
So,quickreliefcanbeobtainedbyblockingreceptors.

558.A47-year-oldwomanpresentswithcomplaintsofnervousnessand
increasedsensitivitytohotweather.Sheisdiagnosedwithhyperthyroidism
andprescribedpropylthiouracil.Whatistheprincipalmechanismbywhich
thisdrugacts?

a)DecreasingtheefficacyofTSHbindingtothethyroidTSH
receptor
b)Decreasingtherateofproteolysisofthyroglobulin
c)Increasingtheamountof3,3',5'-triiodothyronine(reverseT3;
rT3)
d)Inhibitingdeiodinationofthyroxine(T4)
CorrectAnswer-D
PropylthiouracilworksprimarilybyinhibitingtheperipheralconversionofT4toT3.
Thethyroidextractsiodidefromtheplasmaand,inanoxidativeprocess,iodinatedtyrosine
residuesinthyroglobulinmolecules.Monoiodotyrosineanddiiodotyrosineareformedand
thencoupledtoproduceeitherthyroxine(tetraiodothyronine,T4)ortriiodothyronine(T3).
ProteolyticcleavageofthyroglobulinmoleculesleadstofreeT3orT4,whichisthen
releasedintothecirculation;T3isseveraltimesmorepotentthanT4.Peripheral
deiodinationofT4atthe5'positionleadstoT3formation(mainlyintheliver);thisstepis
inhibitedbypropylthiouracil.
DecreasingtheefficacyofTSHbinding,decreasingtherateofthyroglobulinproteolysis,
increasingtheamountofrT3formation,andinhibitingtheuptakeofiodideintothethyroid,
wouldalltendtodecreasetheformationofthyroidhormonesinthethyroiditself.

559.Whichofthefollowingisanaromatase
inhibitor?
a)Tamoxifen
b)Letrozole
c)Danazol
d)Taxane
CorrectAnswer-B
Ans.is'b'i.e.Letrozole
Aromataseinhibitorsareoftwotypes
oTypeI(steroidal)aromataseinhibitor-Theycauseirreversible
inhibitionofaromatase,e.g.Exmestane,formestane.oTypeII(non-
steroidal)aromataseinhibitor-Theycausereversibleinhibitionof
aromatasxee.g.Anastrazole,Letrozole,vorozale.

560.Drugwhichiscontraindicatedbefore2nd
stageoflaboris:
March2009

a)Mifepristone
b)Oxytocin
c)Misoprostol
d)Ergometrine
CorrectAnswer-D
Ans.D:Ergometrine
Drugsusedformedicalmethodofinductionoflabourare:
Mifepristone
Oxytocin
Misoprostol(ProstaglandinsEl)
Ergometrineiscontraindicatedinpregnancy,1ststageoflabour,2nd
stageoflabourbeforecrowningoftheheadandinbreechdelivery
priortocrowning.

561.Whichantithyroiddrugcrossesplacenta
?
a)Carbimazole
b)Propylthiouracil
c)Both
d)None
CorrectAnswer-C
Ans.C.Both
Bothpropylthiouracilandmethimazole/carbimazolecrossplacenta.
Butbecauseofhighproteinbindingcapacityofpropylthouracilis
transferredlessacrossPlacenta).
Therefore,itispreferredinpregnancy.

562.Whichofthefollowingoralantidiabetic
drugisinsulinsecretagogues?
a)Metformin
b)Pioglitazone
c)Nateglinide
d)Acarbose
CorrectAnswer-C
Ans.C.Nateglinide
Nateglinideisanoralantidiabeticdrugthatcausesthereleaseof
insulin(insulinsecretagogue).
ItisaD-phenylalaninederivativewhichprincipallystimulatesthe1st
phaseinsulinsecretion.
Itisusedintype2diabetesmellitusalongwithotherantidiabetics,to
controltheprandialriseinbloodglucose

563.Whichofthefollowingantidiabeticdrug
isinsulinsecretogogue?
a)Pramlintide
b)Glucomannan
c)Exenatide
d)None
CorrectAnswer-C
Ans.C.Exenatide
Amongparentralantidiabetic(hypoglycemic)drugs-Enenatideis
insulinsecretagogue,i.e.stimulateinsulinerelease.

564.Oralsoreduetoinhaledsteroidsare
treatedby?
a)Griseofulfin
b)Amphotericin-B
c)Fusidicacid
d)Muprocinointment
CorrectAnswer-B
Ans.B.Amphotericin-B
Oneofthecommonsideeffectofinhaledcorticosteroidsisoro-
pharyngealcandidiasis,whichcanbetreatedby,
Topicalantifungal(nystatin,clotrimazole,amphoterianBoral
suspension).
Systemicoralazoles(fluconazole,itraconazole,posaconazole).

565.PGE1analogueis?
a)Carboprost
b)Alprostadil
c)Epoprostenol
d)Dinoprostone
CorrectAnswer-B
Ans.B.Alprostadil
PGI2analogue-Epoprostenol,treprostinil.
PGE1analogue-Alprostadil,Misoprostol
PGE2analogue-Dinoprostone.
PGF2(alpha)analogue-Carboprost,latanoprost,bimatoprost,
travoprost.

566.Longacting13-2agonistis?
a)Formoterol
b)Isoprenaline
c)Salbutamol
d)Ephedrine
CorrectAnswer-A
Ans.A.Formoterol
Beta-2agonists:
Usedinasthma.
Longacting-Salmeterol,formoterol.
Shortacting-Salbutamol,terbutaline.

567.Triptantakenbynasalrouteis?
a)Sumatriptan
b)Rizatriptan
c)Naratriptan
d)Frovatriptan
CorrectAnswer-A
Ans.A.Sumatriptan
Sumatriptancanalsobetakenas:

1. Nasalspray
2. Suppository
3. Subcutaneousinjection
Besidesumatriptan,Zolmitriptancanalsobeusedbynasalroute.

568.Acetaminophen[Paracetamol]induced
livertoxicityisdueto?
a)N-acetylcystine
b)NAPQ
c)Co-Q
d)Cytochrome'C'
CorrectAnswer-B
Ans.B.NAPQ
ParacetamolismetabolizedtoN-acetyl-
paraaminobenzoquinoneimine(NAPQ)bymicrosomalenzymes.
Thismetabolitehashighaffinityforsulf-hydrylgroupsandcan
combinewiththeenzymesandotherbiomoleculesresultingin
hepatotoxicity.
N-acetylCystineisusedasanantidote.
Itreplenishstheglutathionestoresofliverandpreventsbindingof
thetoxicmetabolitetoothercellularconstituents.

569.Drugswhichareusedinacuteasthma
include?
a)Budesonide
b)Terbutaline
c)Salbutamole
d)Theophylline
e)Sodiumcromoglycate
CorrectAnswer-B:C:D
Ans.is'b'i.e.,Terbutaline,'c'i.e.Salbutamole&'d'i.e.
Theophylline
[Ref:KDTVh/ep.223]
Treatmentofacuteasthma:
Theonlydrugseffectiveforthetreatmentofacuteattackofasthma
arebronchodilators(beta2-receptoragonists,anticholinergics,and
methylxanthines).
Mildattacks:
Forpatientswithmildattackinhalationofashortactingbeta-2
receptoragonist,e.g.salbutamol(albuterol),terbutalineisused.
Aninhaledanticholinergic,e.g.ipratropiummaybeaddedifthereis
nosatisfactoryresponsetobeta2-agonistsalone.
Inpatientswhoarerefractorytoinhaledtherapies,i.v.aminophylline
(theophylline)maybeeffective.
Severeattacks:
Oxygenphtscontinuousadministrationofaerosolizedsalbutamol
(albuterol)plussystemicsteroids,e.g.methylprednisolone,
hydrocortisone.

Recently,MgSO4hasbeentriedinacutesevereasthmabyIVor
inhalationroute.

570.Maximumeffectofbronchodilatationin
asthmaiscausedby?
a)Corticosteroids
b)Theophylline
c)Anticholinergic
d)2-Agonist)
CorrectAnswer-D
Ans.is'd'i.e.,2-Agonist
-agonistsinAsthma
Bronchihave,-adrenergicreceptorswhichcause
bronchodilatationSo,theadrenergicdrugsusedinasthmaare
selective2agonists.
-agonistsarethemosteffectivebronchodilators
2-agonistshavesomeothereffectsalsoonairways(other
thanbonrchodilatation),thatareresponsibleforbeneficialeffectsin
asthma:
Inhibitionofreleaseofmastcellsmediatorsmastcellsstabilizing
action.
Inhibitionofexudationandairwayedema.
Increasedmucociliaryclearance
Decreasedcough
2-agonistshavenoeffectoninflammation
noantiinflammatoryaction.

571.Atypicalsideeffectmontelukast?
a)Goodpasturesyndrome
b)Churg-Strausssyndrome
c)Membranousglomerulonephritis
d)Bronchialasthma
CorrectAnswer-B
Ans.B.Churg-Strausssyndrome
Churg-Strausssyndromecanbeceusedbyleukotrieneantagonists
(e.g,Monteleukast).

572.Trueaboutoralironpreparations?
a)Mostcommonlyusedpreparationisferrousgluconate
b)Ferrousfumarateismostefficient
c)Differentpreparationshavedifferentbiovailability
d)Ferricpreparationsaremoreeffective
CorrectAnswer-C
Ans.C.Differentpreparationshavedifferentbiovailability
Preferredrouteforironsupplementationisoral.
Ferroussaltsareinexpensive,havegoodironcontentandarebetter
absorbedthanferricsalts.
Mostcommonlyusedpreparationisferroussulfute,whichisthe
cheapestandaseffectiveasourceofelementalironasmore
expensivepreparations.
Itcontains32%ironindriedsaltand20%ironinhydratedsalt.
Othereffectiveandinexpensivepreparationsareferrousgluconate
(12%lron)andferrousfumarate(33%iron),whichareequivalentto
ferroussulfate.

573.Trueaboutheparininduced
thrombocytopenia?
a)Lowmolecularweightheparinisbetteralternative
b)Antibodiesareformedagainstplatelets
c)VitaminKisspecificantidote
d)Within12hoursofstartingheparin
CorrectAnswer-B
Ans.B.Antibodiesareformedagainstplatelets
Heparininducedthrombocytopenia(HlT)
HeParininducedthrombocytopeniaisanimportantadverseeffect
ofheparinadministration,usuallycausedbyunfractionatedheparin,
butmayalsobeseenwiththeuseoflowmolecularweightheparin
(LMWH).
HITmaybeoftwotypes:
1. TypeI(Non-immunemediated):-Itismildandheparinmaybe
continued.
2. Type2(Immunemediated):-Itisduetoformationofantibodies
againstplatelets.Paradoricalthrombosiscanoccur.
Heparinmustbediscontinuedimmediately.
WarfarinandLMWarecontraindicated.
Lepirudin(adirectthrombininhibitor)isanticoagulantofchoice.
Alternativesaredanaparoid,hiruitinandArgaboban.

574.Mechanismofactionofticagrelor?
a)Coxinhibition
b)GPIIB/IIIAinhibition
c)Inhibitionofthromboxanesynthase
d)P2Y12receptorantagonist
CorrectAnswer-D
Ans,D.P2Y12receptorantagonist
Ticlopidine,clopidogrel,Prasugrel?theyblockADPmediatedplatelet
activationbyirreversibleantagonismofP2Y12receptoronADP.

575.Mechanismofactionofticgrelor?
a)ReversibleinhibitionofADPaction
b)IrreversibleinhibitionofADPaction
c)ReversibleinhibitionofGPIIb/IIIa
d)IrreversibleinhibitionofGPIIb/IIIa
CorrectAnswer-A
Ans.A.ReversibleinhibitionofADPaction
CangrelorandticagrelotarereversibleantagonistofADP(P2Y12),in
contrasttoticlopidine,whichisirreversibleantagonist.

576.WhichofthefollowingisaPAR
antagonist?
a)Prasugrel
b)Ticlopidine
c)Tirofiban
d)Vorapaxar
CorrectAnswer-D
Ans.D.Vorapaxar
PARsareactivatedafterthrombin-mediatedproteolyticcleavageof
theirN-terminalexodomain.
PlateletactivationbythrombinismediatedviatwoPARs:PAR-Iand
PAR-4.PAR-Iisthemajorhumanplateletreceptor,exhibiting10-
100timeshigheraffinityforthrombinwhencomparedwithPAR-4.
TwoselectivePAR-Iantagonistsareunderclinicalevaluation:
Vorapaxar(SCH530348)andAtopaxar(E5555).

577.Reboundincreaseingastricacid
secretionafterstoppingprotonpump
inhibitortherapyisdueto?

a)Parietalcellhyperplasia
b)Increasedhistaminerelease
c)Hypergastrinemia
d)HypersensitivityofAchreceptors
CorrectAnswer-C
Ans.C.Hypergastrinemia
Reboundacidhypersecretion(RAH)resultsingastricacidsecretion
abovepretreatmentlevelsafteracidsuppression.
PPItherapyleadstodiminishedacidsecretionandantralD-cell
releasesofsomatostatin,whileincreasingG-cellreleaseof
circulatinggastrin.
Theincreasedgastrinconcentrationexertsatrophiceffecton
oxynticmucosa,causinghyperplasiaandincreasedfunctional
capacityoftheenterochromafin-like(ECL)cellandparietalcell.
Increasedacidsecretionduetosustainedhypergastrinemiaisnot
apparentduringPPItherapybutappearswithdrugcessation
theoretically,leadingtoacid-relatedheartburn,acidregurgitationor
dyspepsia,

578.Whichoffollowingisastoolsoftener?
a)Bran
b)Senna
c)Phenalphthalein
d)Docusates
CorrectAnswer-D
Ans.D.Docusates
Stoolsoftener(Docusates,liquidparaffin)
Theysoftenthestoolsbynetwateraccumulationinthelumenbyan
actionontheintestinalmucosa.
Theyemulsifythecoloniccontentsandincreasepenetrationofwater
intofeces.

579.Drugofchoicefordruginducedpeptic
ulcer?
a)Prostaglandinanalogues
b)H2-receptorantagonists
c)Protonpumpinhibitors
d)Antacids
CorrectAnswer-C
Ans.C,.Protonpumpinhibitors
DrugofchoiceforNSAIDsinducedpepticulcer>PPIs
MostspecificdrugforNSAIDsinducedpepticulcer>Prostaglandin
analogue.

580.Atropineplusdiphenoxylatecombination
isusedfor?
a)Glaucoma
b)Iridocyclitis
c)Diarrhea
d)Motionsickness
CorrectAnswer-C
Ans.C.Diarrhea
Diphenoxylate(2.5mg)plusatropin(0.025mg)combinationisused
asantimotilitydrugfortreatmentofdiarrhea.

581.Latestoraldirectthrombininhibitoris?
a)Ximelagatran
b)Indraparinux
c)Dabigatran
d)Fondaparinux
CorrectAnswer-C
Ans.is'c'i.e.,Dabigatran
Ximelagatranwasthefirstoraldirectthrombininhibitorapproved;
however,itwassubsequentlywithdrawnfromthemarketbecauseof
reportsofliverfailure.
Recentlyaneworaldirectinhibitor,dabigatran,wasapprovedfor
useinEuropeforpreventionofvariousthromboembolisminpatients
whohaveundergonehiporkneereplacementsurgery.

582.Ximelagatranisusedas?
a)Antiplatelet
b)Anticoagulant
c)Fibrinolytic
d)Antifibrinolytic
CorrectAnswer-B
Ans.B.Anticoagulant
DirectThrombinInhibitors:
Thisgroupincludeshirudin,lepirudin,bivalirudin,argatroban,
dabigatran,melagatran,andximelagatran.
DabigatranandXimelagatran(aprodrugofmelagatran)canbe
givenorally.
Allotherdrugsareusedparenterally.
ThesedrugsdirectlyinactivatefactorIIa(thrombin).
Thesearetheanticoagulantofchoiceforheparin-induced
thrombocytopenia

583.Specificfeatureofsimvastatin?
a)Mostpotentstatin
b)Longestactingstatin
c)Lipophilic
d)Notmetabolized
CorrectAnswer-C
Ans.C.Lipophilic
StatinsarethemostpowerfulLDLloweringdrugs.
Statinsarethemosteffectiveandbesttoleratedhypolipidemic
drugs.
Simvastatinandlovastatinarelipophilicandhence,theirCNS
penetrationismorethanhydrophilicagentslikepravastatinand
fluvastatin.

584.Statinhavinglongesthalflife:
September2012
a)Rosuvastatin
b)Pravastatin
c)Simvastatin
d)Lovastatin
CorrectAnswer-A
Ans.Ai.e.Rosuvastatin
Rosuvastatin
ItisacompetitiveinhibitoroftheenzymeHMG-CoAreductase,
havingamechanismofactionsimilartothatofotherstatins.
Itsapproximateeliminationhalflifeis19handitstimetopeak
plasmaconcentrationisreachedin3-5hfollowingoral
administration.

585.Antituberculardrugwhichmakesthe
patientnon-infectiveearliest?
a)INH
b)Rifampin
c)Ethambutol
d)Pyrazinamide
CorrectAnswer-A
Ans.A.INH
InthosewithopenorinfectiouspulmonaryTB,thegreatmajorityof
bacilliisfreelyreplicatinginthecavitywallsandisrapidlykilledby
INH,therebyspeedilyrenderingthepatientnon-infections.

586.Thymidineisresponsibleforresistance
towhichantibiotic?
a)Erythromycin
b)Sulfonamide
c)Tetracycline
d)Nitroforantoin
CorrectAnswer-B
Ans.B.Sulfonamide
ThesulfonamidesareantimetabolitesthatcompetewithPABA,
therebypreventingsynthesisoffolicacid.
Thisinhibitionblockstheformationofthymidine,somepurines,
methionineandglycine.
Theenterococciareabletouseexogenousthymidineandare
thereforeintrinsicallyresistanttothesulfonamides.

587.Whichofthefollowingisnotexcretedin
kidney?
a)Ciprofloxacin
b)Ofloxacin
c)Levofloxacin
d)Moxifloxacin
CorrectAnswer-D
Ans.D.Moxifloxacin
Fluroquinolonethatareprimarilyexcretedbyrenal
mechanismsandforwhichdoseadjustmentisneededinclude:
Ciprofloxacin
Gatifloxacin
Cinafloxacin
Levofloxacilin
Lomefloxacilin
Norfloxacin
Ofloxacin

588.Levamisolisa/an?
a)Immunomodulator
b)Immunostimulant
c)Antiheliminthic
d)Alloftheabove
CorrectAnswer-D
Ans.D.Alloftheabove
Levamisolisananti-helminthicdrugwithimmune-modulatoryaction.
Atlowdosesithasimmune-stimulatoryaction.

589.Gametocidalantimalarialdrugforall
speciesofplasmodium?
a)Chloroquine
b)Quinine
c)Primaquine
d)Mefloquine
CorrectAnswer-C
Ans.C.Primaquine
Gametocidal-Actsongametocytes
Forallspecies:Primaquine,artimisinins
ForP.vivax:Chloroquine,quinine

590.Quinineactsonwhichstageof
plasmodiumlifecycle?
a)Exoerythrocytic
b)Pre-erythrocytic
c)Erythrocytic
d)Alloftheabove
CorrectAnswer-C
Ans.C.Erythrocytic
Anti-malarialdrugsactingonErythrocyticschizogony.
Fastacting===Chloroquine,amadioquine,quininamefloquine,
halofantrine,lumefanterine,atovaquone,artimisinin.
Slowacting===Pyrimethamine,proguanil,
sulfonamides,tetracyclines.

591.Whichamongthefollowingispresent
onlyiniv[intravenous]form-
a)Vancomycin
b)Meropenem
c)Streptomycin
d)Alloftheabove
CorrectAnswer-B
Ans.B.Meropenem
Meropenemisgivenintravenously.
Vancornycinisusedintravenouslyforallinfections,exceptfor
pseudomembranouscolitis,whereitisusedbyoralroute.
Streptomycinisadministeredbyintramuscularroute.

592.Emtricitabineisclassifiedas?
a)Alkylatingagent
b)Antimetaboliteanticancer
c)NRTI
d)Noneoftheabove
CorrectAnswer-C
Ans.C.NRTI
Emtricitabineisananti-retroviraldrug.
Itisanucleoside-reversetranscriptaseinhibitor(NRTI).
Itisasyntheticcytidineanalogue.
Majorsideeffectsarelacticacidosisandliverdysfunction.

593.Emtricitabineisa/an?
a)Alkylatingagent
b)Antimetabolite
c)Mitoticinhibitor
d)Noneoftheabove
CorrectAnswer-B
Ans.B.Antimetabolite
Emtricitabineisanantimetabolite,butitisnotananticancerdrug(it
isananti-retroviraldrug).
AIINRTIsareantimetabolites:-
'AlthoughallNRTIshavesamebasicmechanismofaction,different
drugsintheclassserveasantimetabolitesofdifferentpurineand
pyrimidinebasesofDNA.

594.Synergisticactionisshownbyallexcept
?
a)Penicillinplussulfonamide
b)Streptomycinplustetracycline
c)Rifampicinplusdapsone
d)Penicillinplustetracycline
CorrectAnswer-D
Ans.D.Penicillinplustetracycline
Combinationofabactericidalwithabacteriostaticdrugmaybe
synergisticorantagonisticdependingontheorganism.
Iftheorganismishighlysensitivetothecidaldrug-responsetothe
combinationisequaltothestaticdruggivenalone(apparent
antagonism),becausecidaldrugsactprimarilyonrapidlymultiplying
bacteria,whilethestaticdrugretardsmultiPlication.
Thishasbeenseenwithpenicillin+tetracydine/chloramphenicolon
pneumococciwhicharehighlysensitivetopenicillin.
Pneumococcalmeningitistreatedwithpenicillin+tetracyclinehad
highermortalitythanthosetreatedwithpenicillinalone.
Penicillin+erythromycinforgroupAStreptococciandnalidixicacid
+nitrofurationforE.colihavealsoshownantagonism.

595.NeuropathywithINHtherapyisleastin
patients?
a)Havingmalnutrition
b)Alcoholics
c)Fastacetylators
d)VitaminBcomplexdeficiency
CorrectAnswer-C
Ans.C.Fastacetylators
Peripheralneuropathyismorelikelytooccurinslowacetylatorsand
patientswithpredisposingconditionssuchasmalnutrition,
alcoholism,diabetes,AIDS,anduremia.

596.DrugsusedforH.Pyloriareallexcept?
a)Bismuth
b)Amoxicillin
c)Domperidone
d)Clarithromycin
CorrectAnswer-C
Ans.C.Domperidone
DrugsusefulforH.Pyloriinfection
Amoxicillin
Tinidazole/metronidazole
Omeprazole
Ranitidine
Tetracycline
Bismuth
Clarithromycin.

597.Durationoferythromycinusedin
treatmentofdiphtheriais?
a)3days
b)7days
c)14days
d)30days
CorrectAnswer-C
Ans.C.14days
Drugofchoicefordiphtheriaiserythromycin.
Totalcourseofantibioticsisgivenfor14days
AlternativeantibioticsarepenicillinG,clindamycinandrifampir.

598.Longestactingcarbapenems?
a)Imipenem
b)Meropenem
c)Doripenem
d)Ertapenem
CorrectAnswer-D
Ans.D.Ertapenem
Ertapenemisalong-actingcarbepenemwithabroadspectrum
antimicrobialactivitysimilartooldercarbapenems,imipenemand
meropenem.
However,unlikeimipenem,ithasPooractivityagainst
pseudomonas.

599.XDRTBisdefinedas?
a)MDRplusresistancetofluoroquinolone
b)MDRplusresistancetofluoroquinoloneandstreptomycin
c)MDRplusresistancetofluoroquinoloneandAmikacin
d)MDRplusresistancetoAmikacin
CorrectAnswer-D
Ans.D.MDRplusresistancetoAmikacin
MDRisdefinedasresistanceINHandrifampicinwithorwithout
resistancetootherdrugs.XDRisdefinedasresistancetoINHand
rifampicinaswellastoallfluoroquinolonesandoneofnjectable
drugs(capreomycin,kanamycin,amikacin).

600.NeuropathycausedbyINHincreasesin
allexcept?
a)Uremia
b)Hyperthyroidism
c)Diabetesmellitus
d)Poornutrition
CorrectAnswer-B
Ans.is'b'i.e.,Hyperthyroidism
[RefKatzung11'h/ep.1069]
isoniazidinducedperipheralneuropathy:
Peripheralneuropathyisobservedin10to20%ofpatientsgiven
dosesgreaterthan5mg/kg/d,butitisinfrequentlyseenwiththe
standard300-mgadultdose.
Peripheralneuropathyismorelikelytooccurinslowacetylatorsand
patientswithpredisposingconditionssuchasmalnutrition,
alcoholism,iliabetes,AIDS,anduremia.

601.Idoxuridineisusedfortreatmentof?
a)Influenza
b)RSV
c)HSV
d)HIV
CorrectAnswer-C
Ans.C.HSV
IdoxuridineisusedonlytopicallyforkeratoconjunctivitisbyHSV'

602.Whichantibioticshouldnotbegiven
afterdrinkingmilk?
a)Chloramphenical
b)Tetracycline
c)Erythromycin
d)Sulfonamide
CorrectAnswer-B
Ans.B.Tetracycline

603.Post-transplantationhypertensioncan
becausedby:I.Rejection.II.
Cyclosporinenephrotoxicity.III.Renal
transplantarterystenosis(RTAS).IV.
Recurrentdiseaseintheallograft.

a)I,II,III,IVarecorrect
b)I,II,IVarecorrect.
c)I&IIIarecorrect
d)Noneoftheaboveiscorrect.
CorrectAnswer-A
Bothacuteandchronicrejectionmayresultinhypertension.The
formercausesacutefluidretentionandpluggingofperitubular
capillarieswithinflammatorycells.Thismayprogresstointimal
swellingandmedialnecrosisandeventuateinischemiasecondary
toendothelialproliferationandobliterationofsmallvessels.Chronic
rejection,thoughttoberelatedtoprotractedhumoralinjury,results
inobliterationofcapillariesviathedevelopmentofintimal
hyperplasia.Cyclosporinehasavasoconstrictiveeffectwhich,
throughactivationoftherenin-angiotensinsystem,mayleadto
hypertension.RTASisresponsibleforhypertensionin4%to12%of
renalallograftrecipients.Itrespondswelltopercutaneous
angioplasty.Acarefultrialofangiotensin-convertingenzyme
inhibitorsmaybediagnosticofRTAS.Recurrentdiseasesuchas
membranoproliferativeglomerulonephritisandfocalglomerular
sclerosismayresultinsignificanthypertensioninrenalallograft
recipients.

604.Capecitabinebelongstowhichclassof
anticancerdrug?
a)Antimetabolite
b)Alkylatingagent
c)Nitrogenmustards
d)Vincaalkaloids
CorrectAnswer-C
Ans.C.Nitrogenmustards
Antimetabolites
Purineantagonists-Mercaptopurine,Thioguanine,Azathioprine,
Fludarabine,Cladaribine
Pyrimidineantagonists-S-Fluorouracil,Cytosinearabinoside
(cytarabine),CaPecitabine,Gemcitabine.
Folateantagonist-Methotrexate,Pemetrexed.

605.Whichofthefollowingdrugsis
associatedwithuntowardsideeffectof
renaltubulardamage-

a)Cisplatin
b)Steptozotocin
c)Methysergide
d)Cyclophosphamide
CorrectAnswer-A
Ans.is'a'i.e.Cisplatin
Somenephrotoxicagentswhichcausetubularnecrosis.
oAminoglycosidesoColistino
MethoxyfluranesoSulfonamides
oAmphotericinBoCyclosporineo
PolymyxinoTetracyclines
oCephaloridineoIntravenousimmuneglobulino
RadioiodinatedcontrastoAcetaminophenmedium
oCisplatin

606.Drugthatcancausehypertrophicpyloric
stenosisis?
a)Tertacyclin
b)Erythromycin
c)Ampicillin
d)Rifampicin
CorrectAnswer-B
Ans.is'b'i.e.,Erythromycin
oMaternalandinfantuseoferythromycinandothermacrolide
antibiotics
havebeenreportedasriskfactorsforinfantile
hypertrophicpyloricstenosis(IHPS).


607.Bestmethodformethanolpoisoning
treatment:
March2007

a)Ethanol
b)Calciumgluconate
c)Desferroxamine
d)BAL
CorrectAnswer-A
Ans.A:Ethanol
Whenmetabolizedbyhepaticalcoholandaldehydedehydrogenase,
methanolformsformaldehydeandformicacid,bothofwhichare
toxic.
Formicacidistheprimarytoxinthataccountsforthemajorityofthe
aniongap,metabolicacidosis,andoculartoxicity.Formicacid
inhibitscytochromeoxidaseinthefundusoftheeye.Swellingof
axonsintheopticdiscandedemaresultinvisualimpairment.
Degradationofformicacidisfolatedependent.Thus,ifafolate-
deficientpersoningestsethanol,toxicitymaybemoreseveredueto
theincreasedaccumulationofformicacid.
Approximately90-95%ofmethanolmetabolismoccursintheliver,
while5-10%isexcretedunchangedthroughthelungsandkidneys.
Methanolisprimarilymetabolizedbyalcoholandaldehyde
dehydrogenase.
Formaldehydehasashorthalf-life,lastingonlyminutes.
Formicacidismetabolizedmuchmoreslowly,anditbioaccumulates
withsignificantmethanolingestion.PhysicalSign
Ocularphysicalfindingsincludesluggishlyreactiveorfixedand

dilatedpupils.
Retinaledemaorhyperemia
Edemaoftheopticdiscmaybeseen.
Opticatrophymayappearinlatestages(permanentblindness).
CNSsignsincludelethargyandconfusion.
Respiratorysignsincludedyspnea(rarecases)orevenKussmaul
respiration,despiteacidosis.
Cardiacsigns(e.g.,hypotension,bradycardia)arelatesigns
associatedwithapoorprognosis.
Labstudies
Methanolconcentration:Thisstudyconfirmsingestionandhelps
guidetreatment.Rememberthatlowserumconcentrationdonot
ruleoutsignificanttoxicity;latepresentersmayhavelowmethanol
concentrationsbutelevatedformicacidlevelsandsevereclinical
toxicity(e.g.,severemetabolicacidosis,blindness,coma).
Treatment
Supportivemeasures
Attemptedcorrectionofacidosisusingsodiumbicarbonateis
indicatedifpHislessthan7.20.AnalkalemicpHmakesitmore
likelythatformicacidwillexistasitsanion(formate),whichcannot
accesstheCNSandopticnerveasreadily.
Administerfolicacidforseveraldaystopotentiatethefolate-
dependentmetabolismofformicacidtocarbondioxideandwater.
Ethanolinfusionisrecommended:Ethanolisacompetitiveinhibitor
ofalcoholdehydrogenaseand,thereby,impairsthemetabolismof
methanolandethyleneglycol.Ethanolhas10-20timesgreater
affinityforalcoholdehydrogenasethanmethanoldoes.
Plasmaformateconcentrationisofprognosticvalue

608.Loweresophagealsphincterpressureisincreasedbyallofthefollowing
substances,EXCEPT:
a)Gastrin
b)Vasopressin
c)Glucagon
d)Secretin
CorrectAnswer-D
Theloweresophagealsphincter,aphysiologicentitybutnotananatomicstructure,plays
animportantpartinpreventinggastroesophagealreflux.Neural,hormonal,myogenic,and
mechanicalfactorsinfluencethetoneofthesphincter.Gastrin,vasopressin,glucagon,
adrenergicagonists,andcholinergicagentsincreasethepressure.Secretinhasthe
oppositeeffect.

609.Interstitialnephritisisseenwithall
except
a)Betalactaminhibitors
b)INH
c)Diuretics
d)Allopurinol
CorrectAnswer-B
INH[Ref.Harrison17th/ep1806,1807&16th/ep1702,1703]
DRUGSCAUSINGINTERSTITIALNEPHRITIS
Antibiotics

Diuretics Anticonvulsants Miscellaneous
.filactamso
?ThiazideQ ?Phenytoinu
.Captopril0
.
?
?
?H2receptor
SulfonamidesQ FurosemideQ PhenobarbitoneQ blockerse
.
?
?QuinolonesQ
.Omeprazole
Triamterene Carbamazepine
.Vancomycin .NSAIDS?
?Valproicacid
.Mesalazine
?
.Indinavir
Erythromycin
.Minocycline

.Allopurinol
.Rifampicin
?Ethambutol
.Acyclovir

610.Lorcaserinisusedas?
a)Anti-anxiety
b)Anti-smoking
c)Anti-helminthic
d)Anti-obesity
CorrectAnswer-D
Ans.D.Anti-obesity
Lorcaserineisselective5-HT2cagonistwhichdecreasesappetitein
treatmentofobesity.

611.HalflifeofNicotineinblood?
a)15minutes
b)2hours
c)5hours
d)24hours
CorrectAnswer-B
Ans.B.2hours
'Nicotinehasahalf-Iifeofapproximately2hours(range1-2hours).

612.ICcontentofRinger'slactate[mmol/L]?
a)130
b)109
c)4
d)50
CorrectAnswer-C
Ans.C.4
OneliterofRinger'slactatesolutioncontains:-

1. 130mEqofsodiumion=130mmoVl
2. 109mEqofchlorideion=109mmol/L
3. 28mEqoflactate=28mmol/L
4. 4mEqofpotassiumion=4mmol/L
5. 3mEqofcalciumion=1.5mmol/L

613.Flulikesyptomsissideeffectofwhich
antiTBdrug?
a)NH
b)Rifampicin
c)Pyrzinamide
d)Streptomycin
CorrectAnswer-B
Ans.is'b'i.e.,Rifampicin

614.Allofthefollowingareknownadverse
effectsofthalidomide,except:
a)Diarrhoea
b)Teratogenicity
c)DVT
d)Neuropathy
CorrectAnswer-A
Ansis'a'i.e.Diarrhoea
oThalidomidecausesconstipationandnotdiarrhoea.

615.Oxidationofdrugsismainlytakesplace
in?
a)Nucleu
b)SmoothER
c)RoughER
d)Cytoplasm
CorrectAnswer-B
Ans.B.SmoothER
Mostofthemicrosomaldrugmetabolizingenzymes(Cyt.P450)are
locatedonsmoothendoplasmicreticulum.

616.IPC304Bisrelatedto-
a)Punishmentforcrueltybyhusbandorhisrelatives
b)Dowrydeath
c)Deathcausedbynegligence
d)Punishmentofculpablehomicide,notamountingtomurder
CorrectAnswer-B
Ans.is'b'i.e.Dowrydeath
IPC304-Punishmentofculpablehomicide,notamountingto
murder
IPC304A-Deathcausedbynegligence
IPC304B-Dowrydeath:10yearsofimprisonmentwhichcan
extendtolife.
IPC498-Punishmentforcrueltybyhusbandorhisrelatives.

617.IPC201isfor-
a)Punishmentforembalmingbeforeautopsy
b)Perjury
c)Voluntarycausinggrievoushurt
d)Kidnapping
CorrectAnswer-A
Ans.is'a'i.e.Punishmentforembalmingbeforeautopsy[Ref:
Reddy30th/ep.11]
IPC201isforpunishmentforembalmingbeforeautopsy(andalso
forthedisappearanceofevidence).

618.Sec191IPCisfor-
a)Medicalnegligence
b)Hostilewitness
c)Criminalnegligence
d)Assaultpunishment
CorrectAnswer-B
Ans.is'b'i.e.Hostilewitness[RefReddy30th/ep.11]
Perjury,Hostilewitnessandfalsecertificate
191IPC:(i)Givingfalseevidence(perjury)underoathand(ii)
hostilewitness.
192IPC:Fabricatingfalseevidence(perjury).
193IPC:Punishmentforfalseevidence(punishmentforperjury):
imprisonmentupto7years+fine.
194IPC:Givingorfabricatingfalseevidencetoprocureconviction
ofcapitaloffence.
195IPC:Givingorfabricatingfalseevidencetoprocureconviction
ofoffencepunishablewithimprisonmentforlife.
197IPC:Issuingorsigning(attesting)falsecertificatebyadoctoris
acriminaloffence.
201IPC:Causingdisappearanceofevidence.

619.Dyingdeclarationisa:
MAHE10
a)Circumstantialevidence
b)Oralevidence
c)Documentaryevidence
d)Hearsayevidence
CorrectAnswer-C
Ans.Documentaryevidence

620.Novusactusinterveniensis-
a)Factsspeakingforitself
b)Breakingofchain
c)Contributorynegligence
d)Therapeuticmisadventure
CorrectAnswer-B
Ans.is'b'i.e.Breakingofchain
Apersonisresponsibleforhisactionsandtheirconsequences.This
principleappliestocasesofassaultoraccidentalinjuries.However,
sometimessuchcontinuityofeventsisbrokenbyanentirelynew
andunexpectedhappening,duetonegligenceofsomeother
person,i.e."Novusactusinterveniens"(anunrelatedaction
intervening).
Forexample,Ifapersonhasbeenassaultedduetowhichhehas
sustainedalargeliverlaceration,forwhichheisoperatedbya
surgeon.Ifthepatientdiesintraoperativelyorpostoperativelydueto
complicationsrelatedtosurgeryorinjury,thepersonwhohas
assaultedthepatientwillbeheldresponsible.
But,ifthedoctorhasdonesomenegligentactduringsurgery,e.g.
lefttheswaborinstrumentinabdomenduringsurgery;andpatient
diesbecauseofthatact(sepsisduetoswab),thentheresponsibility
maypassfromoriginalincidenttolaternegligentactofdoctorby
principleof`Novusactusintervention'(anunrelatedaction
intervening).Thusthedoctorisresponsiblefornegligentacts,i.e.
criminalnegligence,andtheassailantwillnotbefullyresponsiblefor
theultimateharm.

621.Inquest,notfollowedinIndiais-
a)Policeinquest
b)Magistrateinquest
c)Coroner'sinquest
d)Alloftheabove
CorrectAnswer-C
Ans.is'c'i.e.,Coroner'sinquest
Coroner'scourtusedtobeheldinIndiainBombay(Mumbai).Now,
itisheldnowhereinthecountry.ItwasabolishedevenfromMumbai
in1999.

622.Lengthofthefetusis40cms.Whatwould
betheageofgestation?
a)4months
b)6months
c)7months
d)8months
CorrectAnswer-D
Ans.is'd'i.e.,8months[RefModi23'diep.1016,1037;Dutta
Obstetrics6th/ep.41]
AccordingtoRuleofHasse:Length=5xmonthsofpregnancy.
Thus,40=5xmonthsofpregnancyormonthsofpregnancy=8

623.Extrapermanenttoothtoeruptis-
a)Upperincisor
b)Canine
c)Molar
d)Lowerincisor
CorrectAnswer-A
Ans.is'a'i.e.,Upperincisor[RefTextbookofPediatric
Decntistryp.354]
Mostcommonsupernumeraryteetharepermanent,anteriorincisor
inthemaxilla(maxillaryincisors),calledmesiodens.
Aftermaxillaryincisors,maxillaryandmandibularfourthmolars
calleddistodensordistomolars

624.Trueaboutfingerprintingis-
a)Mostcommontypeiswhorls
b)Mostspecificmethodofidentification
c)Notpresentatbirth
d)Alloftheabove
CorrectAnswer-B
Ans.is'b'i.e.Mostspecificmethodofidentification[RefReddy
30thiep.76]
Fingerprintsarepresentfrombirthbothonepidermisanddermis,
remainconstantthroughoutlifeandcan'tbealteredwithout
destroyingtrueskin.Fingersprintsareduetopapillaryorepidermal
'ridges'onthetipsoffingersandthumb.
Fingerprintpatternisabsolutelyindividuali.e.notwohandsare
entirelyalike,notevenidenticaltwins.That'swhy,itis
best(mostsensitiveandmostspecific)andmostreliablemethodof
identification(Quetelet'sruleofbiologicalvariation).
Loops(67%mostcommon)>Whorls(25%)>archer(7%)>
composite(2%leastcommon)arefourmaintypesofpattern.
Itisacceptedthatchancesof2fingerprintsmatching16ridge
characteristicareinfinitelysmall(Parikh's).Inpractice8-6pointsof
finecomparisionareacceptedasproofofidentity.

625.UVraysexaminationisdonefor-
a)Fadedtattoo
b)Bloodsatins
c)Hairexamination
d)Noneoftheabove
CorrectAnswer-A
Ans.is'a'i.e.Fadedtattoo[RefPrinciplesofForensicMedicine
andToxicologybyRajeshBardale]
Tattoomarkispermanentwhendyepenetratesthedermis.
Ultravioletlampmakesoldtattoosreadilyvisible.Afaded
tattoomarkbecomesvisiblebyuseofInfraredphotographyor
rubbingthepartandexaminingwithmagnifyinglens.

626.Justbeforethebirthwhichepiphysis
appears?
a)Lowerendoffemur
b)Upperendofhumerus
c)Lowerendoffibula
d)Upperendoftibia
CorrectAnswer-A
Ans.is'a'i.e.,Lowerendoffemur[RefReddy30/ep.64,65]
1. Atbirthcenterappearsupperendoftibiaandheadofhumerus.
2. At2monthsintrauterinelife(IUL)Centerappearsformandible,
clavicle,ribs,vertebrae,frontalandparietalbones.
3. At3monthsIULCenterappearsforsacrum.
4. At4monthsIULCenterappearsfortemporalandoccipital
bones.
5. At5monthsIULCenterappearsforcalcaneum.
6. At6monthsIULCenterappearsforsternum.
7. At7monthsIUL4Centerappearsfortalus.
8. At9monthsIUL/birthCenterappearsforlowerendoffemur.
9. AtbirthCenterappearsupperendoftibiaandheadofhumerus.

627.Baseofsphenoidfuseswithoccipitat
theageof-
a)20years
b)30years
c)40years
d)50years
CorrectAnswer-A
Ans.is'a'i.e.,20years[RefDr.AnilAggrarwalp.68]
Spheno-occipitistheearliesttofuse(20years).
Saggitalsutureobliteratesat30-35years.
Coronoid(coronalsuture)obliteratesat35-40years.
Lambdoidsutureobliteratesat45-50years.
Squamoussutureobliteratesat60years.
Spheno-parietalsutureobliteratesat70years.

628.Falseaboutdeclarationofbrainstem
deathinhospital-
a)Presenceofneurologistisnotrequired
b)Drugoverdoseshouldberuledout
c)Patientmustbeincoma
d)Alloftheabove
CorrectAnswer-A
Ans.is'a'i.e.Presenceofneurologistisnotrequired[Ref
PrinciplesofForensicMedicine&ToxicologyByRajesh
Bardalep.134]
Guidelinesfordeclarationofbrainstemdeath
Braindeathneedstobecertifiedbyaboardofdoctor'sconsistingof:
RegisteredMedicalPractioner(RMP)inchargeofhospitalwhere
braindeathhasoccurred.
AnindependentRMP-aspecialist.
ANeurologist/Neurosurgeonnominatedbypanel.
RMPtreatingthepatient.
Thepatientmustbeexaminedbyteamofdoctorsatleasttricewith
areasonablegapoftimeinbetween(atleast6hours).
Noneofthedoctor'swhoparticipateindiagnosisofbraindeath
shouldhaveanyinterestintransplantationororganremovalfrom
cadaver.
Braindeathcertificatehastobesignedbyallthemembersofboard.
Diagnosisofbrainstemdeathdependsuponfollowingfindings
Thepatientmustbeindeepcomaandcauseofthecomamustbe
irreversiblestructuralbrainstemdamage(i.e.prolongedhypoxia,
trauma,illnessortoxicinsult)mustbeestablished.
Exclusionofothercausesofcoma:

Hypothermia,
Drugoverdoseeg:CNSdepressants(benzodiazepines,barbiturates
etc)
Metabolicorendocrinedisturbances
Intoxication(alcohol)
Demonstratingofabsenceofbrainstemreflexes(Pupillaryreflex,
oculovestibularreflex(,cornealreflex,pharyngealandtracheal
reflexes.)
Nospontaneousrespiration.

629.Timeintervalbetweensomaticand
moleculardeathis-
a)5-10min
b)10-30min
c)30-1hr
d)1-2hrs
CorrectAnswer-D
Ans.is'd'i.e.1-2hrs[Refe-bookofPostmortemchangesby
DRDRao]
Deathofthetissuesandcellsindividually(moleculardeath)takes
placeusually1-2hoursafterstoppageofvitalfunctions(somatic
death).
Moleculardeathofvariousorgansoccuratdifferentintervalsafter
somaticdeath:
1. NervoustissueAfter5minutes.
2. Liver-After15minutes.
3. Heart-After45minutes.
4. Kidney4After1hours.
5. MusclesAfter3hours

630.Bishop'stripodoflifeincludesallexcept
-
a)Respiration
b)Circulation
c)Spinalcordreflexes
d)Brainfunctions
CorrectAnswer-C
Ans.is'c'i.e.,Spinalcordreflexes
Somaticdeath
Itisthecompleteandirreversiblestoppageofthecirculation,
respirationandbrainfunctions(bishop'stripodoflife).
Somaticdeathisassociatedwithimmediatesignsofdeath:?
1. Permanentandcompletecessationoffunctionofbrainandflat
electricEEGwithnoresponsetoexternalstimuli;i.e.braindeath.
2. Permanentandcompletecessationoffunctionofheartandflat
ECG.
3. Permanentandcompletecessationoffunctionoflungs

631.Forautopsy,stomachisopenthrough-
a)Lessersac
b)Greatersac
c)Greatercurvature
d)Lessercurvature
CorrectAnswer-C
Ansis'c'i.e.Greatercurvature[RefAtlasofAdultAutopsyp.66]
Stomachisopenedthroughgreatercurvaturewhichallowstosee
lessersac.
Dependingontypeofcase,anyofthebodycavitycanbeopened
first.Spinalcordisroutinelynotopened.
Itisconvenienttostarttheexaminationwiththecavitychiefly
affected.
Incasesofdeathduetoasphyxia(especiallyhangingand
strangulation),neckshouldbeopenedlast.
Spinalcordcanbeapproachedeitherfromposterior(most
preferred)oranteriorapproach.Highcervicalspineinjuriesarebest
seenbyposteriorandthoracicspinebyanteriorapproach.
Bloodforthesampleistakenfromfemoralvein.Thejugularor
subclavianveinscanalsobeused.10-20mlofbloodistakenandit
istakenbeforeautopsy.30mlofbloodshouldbepreserved
(minimumis10ml).

632.Tachenoiredesalenorticaisa
postmortemfindingisrelatedto
a)Eye
b)Muscle
c)Hair
d)Semen
CorrectAnswer-A
Brownishdiscolourationofthescleraduetocellulardebrisanddust-
Tachenoire

633.Bacteriamostcommonlyinvolvedin
boweldecompositionafterdeathis-
a)Streptococcuspyogenes
b)Clostridiumwelchii
c)Pseudomonasaeruginosa
d)None
CorrectAnswer-B
Ans.is'b'i.e.,Clostridiumwelchii[RefReddy30th/ep.149-152]
Boweldecompositionisbroughtaboutbyaerobicandanaerobic
bacteriapresentinsmallintestine(e.g.C.welchii,staphylococcus,
E.colietc.)whichreleaseenzymes(especiallylipaseand
lecithinase)whichactonbodytocausebreakdown.

634.Negativeautopsyisdefinedas-
a)Nocauseofdeathisfoundongrossaswellas
histopathologicalexamination
b)Causeisapparentongrossexaminationbutnoton
histopathologicalexamination
c)Grossfindingsareminimal
d)Causeisapparentongrossexaminationbutnotfoundbecause
ofconstraintsonthepartofdoctor
CorrectAnswer-A
Ans.is'a'i.e.,Nocauseofdeathisfoundongrossaswellas
histopathologicalexamination[RefDr.AnilAggrawalp.124]
Typesofautopsy
Normalautopsy-causeisapparentfromgrossexamination
Defectiveautopsy-causewasascertainable,butwasnot
ascertainedduetoconstrainsonthepartofdoctor,hospital,facilities
etc
Obscureautopsy-grossfindingsareminimal,indecisiveorobscure,
asinadrenalinsufficiency,anestheticoverdose,myxedema,rare
plantpoisons,thyrotoxicosisetc.subsequentexaminationlike
histology,microbiology,toxicologyorserologyrevealthecause.
Negativeautopsy/inconclusiveautopsy-Causeisnotclearfrom
grossaswellassubsequentexaminations.

635.Lastorgantoputrefyinfemalesis:
BHU12
a)Kidney
b)Uterus
c)Brain
d)Spleen
CorrectAnswer-B
Ans.Uterus

636.Adipocereformationisseenin:
a)Deadbodyexposedtoair
b)Deadbodyburiedindamp,claysoil
c)Burialindryhotair
d)All
CorrectAnswer-B
Bi.e.Deadbodyburiedindamp,claysoil
Idealconditionforadipocereformationarewarm(hot)temperature,
moisture(humid)&diminutionofairQ
e.g.deadbodyimmersedin
waterordampsoilQ


637.InAdipocere,colorchangeseenis-
a)Grayishwhite
b)Black
c)Brown
d)Red
CorrectAnswer-A
Ans.is'a'i.e.Grayishwhite[RefReddy30th/ep.155-156]
Adipocereisacrumbly,waxy,water-insolublematerialconsisting
mostlyofsaturatedfattyacids.Dependingonwhetheritwasformed
fromwhiteorbrownbodyfat,adipocereiseithergrayishwhiteortan
incolor.

638.Notafeatureofpostmortemstaining-
a)Occurimmediateafterdeath
b)Commonindependentpart
c)Disappearwithputrefaction
d)Marginsaresharp
CorrectAnswer-A
Ans.is'a'i.e.,Occurimmediateafterdeath[RefReddy30th/e
p.141,142]
Postmortemstainingisanearlysign(notimmediatesign)ofdeath.
Itreferstodiscolorationofskinandinternalorgansafterdeathdue
toaccumulationoffluidbloodintonelesscapillariesandsmallveins
ofdependentpartofthebody.
Itdoesnotappearelevatedabovethesurfacebuthassharply
defined(usuallyhorizontal)margins.
Itisanearlysignofdeath.Itstartsatabout1hour,becomesa
seriesofmottledpatcheswithin1-3hoursandthesepathces
increaseinsizetocoalesceinabout3-6hours.After6-12hours,
lividityisfullydevelopedandfixed(unchangeable),i.e.primary
lividity.Itendswhenputrificationsetsin.Fixationoflividityisdueto
stagnationofbloodindistendedcapillariesandvenules(notdueto
coagulationofblood).

639.Trueaboutcadavericspasm:
a)Developsimmediatelyafterdeath
b)Maydevelophoursafterdeath
c)Developsonlyinfacialmuscles
d)All
CorrectAnswer-A
Ai.e.Developsimmediatelyafterdeath

640.Besttemperatureforputrefactionis-
a)0-10C
b)10-45C
c)45-100C
d)100-150C
CorrectAnswer-B
Ans.is'b'i.e.,10-45C[RefReddy30th/ep.149-152]
Optimumtemperatureforputrefaction10?C-45?C.
Putrefactionisthelaststageinresolutionofbodyfromtheinorganic
toorganicstateandiscertainsignofdeath.
Putrefactionrateinairistwiceascomparedtoinwater,andeight
timeascomparedtoinearth(Casperdictum).
Putrefactionisdelayedindeathduetowastingdiseaseanemia,
debility,poisoningbycarbolicacid,zincchloride,strychineand
chronicheavymetalpoisoning.

641.InIndiaexhumationisorderedby?
a)Magistrate
b)HealthSecretory
c)HealthMinister
d)AnylocalMLA
CorrectAnswer-A
Ans.is'a'i.e.,Magistrate
Exhumation
Exhumationislawfuldiggingoutofaburriedbodyfromthegravefor
thepurposeofidentificationordeterminationofcauseofdeath.
Onlyamagistrate(executivemagistrate)canorderforexhumation.
Inindia,thereisnotimelimitforexhumation,i.e.canbedoneatany
timeafterdeath.
ItisdoneundersupervisionofmedicalofficerandMagistratein
presenceofapoliceofficerwhoprovideswitnessestoidentifygrave,
coffinanddeadbody,
wheneverpossible,Magistrateshouldinform
therelativesandallowthemnottoremainpresentatthetimeof
enquiry.
Thewholeprocedureshouldbeconductedandcompletedinnatural
daylight.
Therefore,itisusuallystartedearlyinmorning.

642.Inapatientofunilaterallossofvision,
thepatienthadinjurytooppositeeye
leadingtocornealopacification.The
patientwasoperatedbycornealgrafting,
hecouldseeclearlyagainwithoneeye.
Theinjuryissaidtobe

a)Simple
b)Grievous
c)Dangerous
d)Hazardous
CorrectAnswer-A
Ans.is'b'i.e.,Grievous
Thispatienthadpermanentvisionlossasitwascorrectedby
surgery(cornealtransplant).
Permanentlossofsightofeithereyeisgrievoushurt.
Injuriescanbeclassifiedas
1
.Asimpleinjuryisonewhichisneitherextensivenorserious,and
whichwouldhealrapidlywithoutleavinganypermanentdeformityor
disfiguration.
2.Agrievousinjuryisone(i)Whichisextensiveorserious(ii)Which
doesnothealrapidly,and(iii)Whichleavesapermanentdeformity
ordisfiguration.Section320IPCdefinesfollowinginjuriesas
grievous.
1. Emasculation(cuttingofthepenis,castration;orcausinglossof
poweroferectionduetospinalinjury).
2. Permanentprivationofthesightofeithereye.

3. Permanentprivationofthehearingofeitherear.
4. Privationofanymember(part,organ,limb)orjoint.
5. Destructionorpermanentimpairingofpowersofanymemberor
joint.
6. Permanentdisfigurationofheadorface.
7. Fractureordislocationofaboneoratooth.
8. Anyhurtwhichendangerouslifeorwhichcausesthesufferertobe,
duringthespaceoftwentydaysinseverebodilypain,orunableto
followhisdailyroutine.Section319IPCdefineshurtasbodilypain,
disease,orinfirmity,causedtoanyperson.
Grievoushurtisacognisableoffenceforwhichapoliceofficercan
arrestapersonwithoutawarrantfrommagistrate.
3.Dangerousinjuryisavarietyofgrievousinjury.Itisaninjury
whichposesanimmediatedangertolifeandisfatalinabsenceof
surgicalaid,e.g.gunshotwounds,compoundskullfracture,trauma
tolargebloodvesselorruptureofinternalorganlikespleen.

643.Chickenfatappearanceisseenin-
a)Antemortemwound
b)Postmortemwound
c)Gunshotinjury
d)None
CorrectAnswer-B
Ans.is'b'i.e.Postmortemwound[RefReddy29th/ep.164,168;
Parikh6th/ep.4.76]
Chickenfatclotisseeninpostmortemwound.
Gapingofthewound,presenceofvitalreactionandincreased
serotoninarefeaturesofantemortemwound.

644.Trueaboutheathematoma:
PGI12
a)Insidethebrain
b)Betweenskullandduramater
c)Betweenskullandpericranium
d)Betweenscalpandpericranium
CorrectAnswer-B
Ans.Betweenskullandduramater

645.Homicidalgunshotwoundcanbe
differentiatedfromsuicidalgunshot
woundby-

a)Multiplegunshotwounds
b)Presenceofgunpowderonhand
c)Nosignofstruggle
d)None
CorrectAnswer-A
Ansis'a'i.e.Multiplegunshotwounds
FollowingareafewwaystoDifferentiatemurderfromsuicide

1. Whereonthebodytheinjuryoccurred:Ashottothesideofthe
head,inthemouth,ortothefrontofthechestisusuallysuicide.
Woundslocatedanywhereelsearemostlikelyhomicide.
2. Distanceofgunfromthebody:Mostsuicideshotsareatcontactor
nearcontactrange,causingaburnmarkaroundthewoundand
leavinggunpowderresidue(whichcanbewipedoff).Atcontact
range,andifthegunisfiredjustaboveabone,suchastheskullor
thesternum,astar-likewoundisproduced.Anythingfurtherawayis
likelyhomicide.
3. Angleoftheshot:Mostsuicideshotsareangledslightlyupward.
4. Numberofshotsfired:Afteroneshot,evenifasuicidevictimisn't
dead,hewouldlikelybeunconsciousorphysicallyunabletofirea
secondtime.Multipleshotsusuallyindicatehomicide.
5. Presenceofgunpowderresidueonvictimshand:Ifamanshot
himself,therewouldbepowderresiduefromunburnedcarbonon
thehandthatfiredthegun.
6. Shotsthroughclothing:Asuicidevictimwillrarelyshootthrough

clothing.Ifheshootshimselfinthechest,whichisunusual,hewill
openhisshirttoexposetheskin.Shotsthroughclothingsuggest
homicide.
7. History,anote,otherfactors:Ifthevictimleftasuicidenote,orwas
knowntohavepersonalproblems,oriftherewasevidenceofdrug
useordrinking,suicideislikely.
8. Evidenceofastruggle:Iftherearescratches,cuts,bruises,
homicideislikely.

646.Chokingischaracteristicofthefollowing
weapon:
a)Revolver
b)Pistol
c)Shotgun
d)Rifle
CorrectAnswer-C
Thechokinglessenstherateofspreadofshotafteritleavesthe
muzzle,increasestheexplosiveforceandincreasesthevelocity.
Shotgunshavevariablechokeadapters.
Differentdegreesareknownasfull-choke,half-chokeand
quarter-chokeorimprovedcylinder.
Ref:
TheEssentialsofForensicMedicineandToxicologybyKS
Reddy,27thedition,Page184.

647.Bevelingofskullisseenin-
a)Broadendoftheentrypointinbulletinjury
b)Narrowendoftheentrypointinbulletinjury
c)Exitpointofbullet
d)Depressedfractureoftheskull
CorrectAnswer-C
Ans.is'c'i.e.,Exitpointofbullet[RefParikh6th/ep.4.43]
Woundofentranceshowsapunchedin(clean)holeintheouter
table.Coneshapedboneisdetachedfromtheinnertableforminga
craterthatislargerthantheholeoftheoutertableandshows
beveling(slopingsurface).
Thus,fromlookingoutside,entrancewoundiscleancutholeand
exitwoundisbevelledopening.

648.Emphysemaaquosumisseenin:
a)Wetdrowning
b)Drydrowning
c)Immersionsyndrome
d)Secondarydrowning
CorrectAnswer-A
Ai.e.Wetdrowning
-Emphysemaaquosumisseeninwet(typical)drowningQwhere
drowningfluidcausesalveolarwalldisruption&enterstissue&
bloodvessels.Ifdeadbodyisthrownintowater,duetohydrostatic
pressurewaterpassesintolungsknownashydrostaticlung.
Oedemaaquosum(nocolumnsoffroth)isseenindrowningof
unconscious.
-ARDS&Fibrosingalveolitisisseeninneardrowningorsecondary
drowningsyndrome.
-DryLungisseenindrydrowninginwhichdeathoccursdueto
laryngealspasm.
-Ifdrowningissurvived,theeventisreferredtoasneardrowning&
complicationsasnear(secondary)drowningsyndrome..
-InImmersionsyndromedeathresultsfromcardiacarrestdueto
vagalinhibition.Q


649.Bestindicatorofantemortemdrowning
is:
Delhi07;TN11

a)Frothinnostrils
b)Cutisanserina
c)Washerwoman'shand
d)Waterinstomach
CorrectAnswer-A
Ans.Frothinnostrils

650.MostspecificsignofAntemortemburns
is-
a)Sootinrespiratorypassage
b)Cyanosisoffingernails
c)Pugilisticattitude
d)Heatruptures
CorrectAnswer-A
Ans.is'a'i.e.,Sootinrespiratorypassage[Ref:Parikh6thlep.
4.152-4.156]
Oneofthemostimportantcharacteristicofantemortemburnsisthe
findingof'Sootandcarbonparticles'inlarynx,tracheaandmaybe
inlungs.
A.Specificsigns(whichindicateantemortemburning):
1. SootandCarbonparticlesinrespiratorypassages
2. Increasedcarboxyhemoglobin(>5%)inblood
3. Signsofvitalreaction
4. Signsofinflammation
5. Redlineofdemarcation
6. Trueblister(vesicle)containingprotein(albumin)andchloride
7. Increasedenzymeandsulfhydryl(-SH)group
8. Increasedserotorineandhistamine
9. Signsofhealingandrepair.
B.Non-specificsigns(mayalsooccurinpostmortemburning):
1. Pugilisticattitude
2. Heathematoma
3. Heatfracture
4. Heatruptures.

651.Sexualgratificationofawomenis
obtainedbyanotherwomaniscalled?
a)Tribadism
b)Bestiality
c)Fellatio
d)Cunnilingus
CorrectAnswer-A
Ans.is'a'i.e.,Tribadism[RefParikh6th/ep.5.50,5.52;Reddy
31"/ep.401;Rao2"'Ilep.373-74;RajesBardale1"/ep.325-26]
Tribadism(Lesbianismorfemalehomosexuality):
Sexual
gratificationofawomenisobtainedbyanotherwomanby
kissing,bodycontact,manipulationofbreastandgenitalia.
Activepartneriscalleddykeorbutchandthepassiveagentis
calledfemme.ThisisnotanoffenceinIndia.

652.Sexwithcadaveriscalled-
a)Necrophilia
b)Exhibitionism
c)Voveyrism
d)Undinism
CorrectAnswer-A
Ans.is'a'i.e.,Necrophilia[RefNarayanReddy30thiep.394]
Necrophilia:Inthiscondition,thereisadesireforsexualintercourse
withdeadbodies.Itissaidtohavesadomasochisticfoundationand
thatdecomposition,foulsmellandcoldnessactasstimulants.

653.Sexualgratificationbyinflictingpainon
partner-
a)Sodomy
b)Sadism
c)Necrophilia
d)Bestiality
CorrectAnswer-B
Ans.is'b'i.e.,Sadism[RefReddy30th/ep.394]
Sexualgratificationisobtainedorincreasedfromactsofphysical
crueltyorinflictionofpainonone'spartner.
Itisseenmorecommonlyinmen.Toobtainsexualgratification,the
sadistmaybite,beat,whip,producecutsonthevictim,etc

654.Masochismmeans:
TN09
a)Sexualintercoursewithdeadbody
b)Sexualpleasurebycontactwitharticlesofoppositesex
c)Sexualpleasurebysufferingofpain
d)Sexualpleasurebyself-stimulation
CorrectAnswer-C
Ans.Sexualpleasurebysufferingofpain

655.Irresistiblesexualdesireinamaleis
knownas:
AIIMS08

a)Nymphomania
b)Tribadism
c)Satyriasis
d)Sadism
CorrectAnswer-C
Ans.Satyriasis
Satyriasisisexcessivesexualdesireinmalewhereheenjoys
havingmultiplesexualpartnersordesiresexcessivesexualactivity.
Thefemale-specifictermforthesameconditionisNymphomania.
Afetishisanabnormalstimulusorobjectofsexualdesire.
Fetishismmeanstheuseofsuchobjectsofsexualgratification
leadingtoorgasm.Forexmapleunderclothing,brassiere,petticoat,
stocking,shoes,etc.
Frotteurism:
Sexualsatisfactionisobtainedbyrubbingagainstpersonsincrowd.
Iftheyattemptintercourse,theyhaveaprematureejaculationor
theyareimpotent.
Itisanuncommonperversionandrarelyoccursalone

656.Sexualgratificationobtainedbyenemais
called-
a)Exhibitionism
b)Fetichism
c)Klismaphillia
d)Frotteurism
CorrectAnswer-C
Ans.is'c'i.e.Klismaphillia[RefInternet]
Klismaphilia(orklysmaphilia),isaparaphiliainvolvingenjoymentof,
andsexualarousalfrom,enemas.

657.Toexaminethevaginalcellsinofthe
rapevictimtestusedis-
a)Lugol'siodinetest
b)Takayamatest
c)Florencetest
d)Precipitintest
CorrectAnswer-A
Ansis'a'i.e.Lugol'siodinetest[RefGRaop.301]
Lugol'siodinetest
Itisdoneonthewashingofglanspenis(orfromamoistblotting
paper)ofaccused,whichthenexposedtoiodinevaporsorlugol's
iodinesolution.Browncolormeansthetestispositive(on4thday).
Browncolorisduetoglycogenpresentinvaginalepithelialcellsof
thevictim.

658.Absorptionelutiontechniqueisusedfor
-
a)Bloodgroupdetection
b)Speciesdetection
c)Seminalstainexamination
d)Noneoftheabove
CorrectAnswer-A
Ans.is'a'i.e.,Bloodgroupdetection[RefParikh6th/ep.7.15-
7.18;SKSinghal4'1'1ep.158-162]
Forbloodgrouping,followingtestsareused:
1)Immunological(serological):
1. absorption-elutiontest,
2. absorption-inhibitiontest,
3. mixedagglutinationtest,
4. latextest.
2)Enzymologicalmethods.
3)Latte'scrustmethod.

659.Testtoknowthespeciesfromblood
stainis-
a)Takayamatest
b)Benzidinetest
c)Precipitintest
d)Teichmann'stest
CorrectAnswer-C
Ans.is'c'i.e.,Preciptin-test[RefParikh6m/ep.7.15-7.18;S.K.
Singhal4'/ep.158-162]
Fordetectionofspecies,serological(immunological)testsare
used.Thesetestsare:

Precipitintest
Haemagglutination
inhibitiontest
Geldiffusiontest.
Doublediffusiontest.
Precipitation
Latexagglutinationtest.
electrophoresis.
Isoenzymemethod
(enzymologicaltest)

660.Allofthefollowingaretrueforingestion
ofacidexcept-
a)Theyarehygroscopic
b)Coagulateproteins
c)Hemoglobintohematin
d)Systemicsymptomsaresevere
CorrectAnswer-D
Ans.is'd'i.e.Systemicsymptomsaresevere
Acidsfix,destroyanderodethesurfacewithwhichtheycomein
contact.Theymainlyactlocallyby
1. Hygroscopicextractionofwaterfromtissues
2. Coagulationofproteins
3. Conversionofhemoglobinintohematin.
Asarulethereisnoremotesystemicactionwithexceptiontoshock.

661.Maxdamageisdonebywhichpoison-
a)Irritantpoison
b)Corrosivepoison
c)Alcohol
d)Opioid
CorrectAnswer-B
Ans.is'b'i.e.,Corrosivepoison[RefRajeshBardalep.437]
Corrosivesfix,destroyanderodethesurfacewithwhichtheycome
incontact.Thus,theycauseextensivetissuedamage.
Irritantsproducelessseverdamagethancorrosivesandproduce
symptomssimulatinggastroenteritis.

662.Leasttoxicformofleadis-
a)Leadacetate
b)Leadoxide
c)Leadcarbonate
d)Leadsulphide
CorrectAnswer-D
Ans.is'd'i.e.,Leadsulphide[RefParikh6thlep.9.17-9.20;
Reddy30thlep.497-498]
Noheavymetalispoisonousinnativeform,exceptlead.Metallic
leadandallitssaltsarepoisonous.Theprincipalsaltswhich
producetoxiceffectsare:(1)Leadacetate(sugaroflead),(2)lead
carbonate(safedaorwhitelead),(3)leadchromate,(4)lead
monoxide(lethrage),(5)leadtetraoxide(redlead,vermilion,sindur)
and(6)leadsulphide(leasttoxic).

663.Charasis:
a)LeavesofCannabisIndica
b)FlowersofCannabisIndica
c)StemofCannabisIndica
d)ResinexudateofCannabisIndica
CorrectAnswer-D
Di.e.ResinexudatesofcannabisIndica

664.Methanolattacks?
a)Cones
b)Rods
c)Ganglioncellsofretina
d)Germinalcelllayer
CorrectAnswer-C
Ans.is'c'i.e.,Ganglioncellsofretina
Methylalcoholismetabolisedveryslowlyandthusstaysforalonger
periodinthebody.
Itisoxidisedintoformicacidandformaldehydeinthetissues.These
toxicagentscauseoedemafollowedbydegenerationoftheganglion
cellsoftheretina,resultingincompleteblindnessduetooptic
atrophy.

665.Arsenicpoisoningpresentswith
symptomsmimicking
a)Cholera
b)Dhaturapoisoning
c)Barbituratespoisoniong
d)Morphinepoisoning
CorrectAnswer-A
Ans:A.)Cholera.
ARSENIC
?Copperarsenite-sheele'sgreen
?Copperacetoarsenite-parisgreen
?Permissiblelimitsofarsenicingroundwater-0.05mg/L
?Thegastroenterictypeofpoisoningresemblescholera
?Greatestquantityisseenin-liver
?Redvelvetyappearanceofmucosaofstomach
?Mostaffectedpartofthestomach?pylorus
?Subendocardialpetechialhemorrhagesoftheventricleistypicalof
arsenicpoisoning(alsofoundinphosphorus,barium,mercuryandin
casesofheatstrokeandacuteinfectiousdiseasee.g.influenza)
?Rigormortislastslongerthanusual,Delaysputrefaction
Acutepoisoning
?Necrosisofintestinalmucosawithhemorrhagicgastroenteritis
?Hypotension,delayedcardiomyopathy
Chronicpoisoning
?AldrichMee'slineonfingernails
?Raindroppigmentationonskin(measteslikerash)
?Chronicconsumptionofwatercontaininghigharsenic
concentrationsleadtovasospasmandperipheralvascular


insufficiency-blackfootdisease
?Diabetes,peripheralneuropathy,gangrene
?Cancerofskin,lung,liver(angiosarcoma),bladder,kidney.
?Arsenophagists-somepeopletakearsenicdailyastonicor
aphrodisiacandacquiretolerance
?Chelatingagent-Dimercaprol
Arsenicwasusedasmurderweaponsinroyalfamilies,andcalled
as"thepoisonofkingsandkingofpoisons"

666.Whichofthefollowingismostspecific
forarsenicpoisoning?
a)Redvelvetystomachmucosa
b)Blueliningongums
c)Tremors
d)Anemia
CorrectAnswer-A
Ans.is'a'i.e.Redvelvetystomachmucosa[RefParikh6th/ep.
9.9-9.11;Reddy30"`/ep.493-494;Essentialsofforensic
medicine-812]
Redvelvetymucosaisatypicalpostmortemfindinginacutearsenic
poisoning.

667.Blackfootdiseaseiscausedby
a)Arsenic
b)Cadmium
c)Lead
d)Mercury
CorrectAnswer-A
Ans.is'a'i.e.,Arsenic
Chronicarsenicexposurecancause'blackfootdisease'asevere
formofperipheralvasculardisease,causinggangreneoflower
limbs.
Arsenicpoisoningmaybeacuteorchronic.
Acutearsenicpoisoning
Itcanpresentinanyofthetwoways:
1. Gastrointestinaltype:Itisthecommonformandresembles
bacterialgastroenteritis(e.g.cholera).Thereisgarlicadourwith
hemorrhagicgastroenteritisandnecrosisofintestinalmucosa.There
istenesmusandanalirritation.Dehydrationmaycauseacute
circulatorycollapse,withhypotension,oliguriaandacutetubular
necrosis(ATN).
2. Fulminantnarcoticform:Largedoseofarsenicisrapidlyabsorbed
withoutproducingGIsymptoms.Thesearegiddiness,headache,
myalgia,formication,vertigoanddeathin2-3hours.
Postmortemappearanceinclude:
1. Redvelvetymucosaofstomachandsubmucouspetechial
haemorrhage.
2. Subendocardialpetechialhaemorrhage.
3. Fattydegenerationofliver,kidneyandheart.
Chronicarsenicpoisoning

Chronicarsenicpoisoninghasfourstages:
1. Firststage(nutritionalandgastrointestinaldisturbances):The
earliestsignisgradualemaciation.Thereislossofappetite,nausea
andintermittentvomitinganddiarrhea.
2. Secondstage(catarrhalchanges):Itresemblescommoncold,i.e.
conjunctivitis,runningnoseandeyes,coughingetc.
3. Thirdstage(skinrash):Thereisclassical'raindroppigmentation',
i.e.patchybrownpigmentationofskin.Atinitialstages,thereisa
vesiculareruptionwhichmayresemblenettlerash.Hyperkeratosis
ofpalmsandsolesoccur.Therearewhitetransversebands
crossingthenails,knownasMee'sline,andindicateperiodsof
arrestedgrowthduetointerferencewithnormalmetabolism.There
isalopeciaandexfoliativedermatitis.
4. Fourthstage(nervousdisturbances):Thereissensoryandmotor
(i.e.mixed)polyneuropathy,withpainfulparesthesiaofhandsand
feetandmuscletenderness.
Theremaybeevidenceofliver(fattyliver)andkidneydamageand
bonemarrowsuppression(causinganemiaandothercytopenias).
Heartmayalsobeinvolved.
Chronicarsenicexposurecanalsocause'blackfootdisease'a
severeformofperipheralvasculardisease,causinggangreneof
lowerlimbs.
Arseniciscarcinogenic,cancausecancersofskin(squamouscell
carcinomaandbasalcellcarcinoma),lung,liver,bladder,kidney,
larynxandlymphoidsystem(leukemia).

668.Marschtestisdoneforpoisoningwith?
a)Lead
b)Mercury
c)Arsenic
d)Nickel
CorrectAnswer-C
Ans.is'c'i.e.Arsenic[RefParikh6thlep.9.9-9.11;Reddy30th/e
p.493-494;Essentialsofforensicmedicine-812]
Marsh'stestandReinsh'stestwereusedarsenicpoisoning,butare
obsoletenow.

669.Papaversemniferummilkexudesfrom?
a)Leaf
b)Root
c)Poppyseed
d)Unripecapsule
CorrectAnswer-D
Ans.is'd'i.e.,Unripecapsule[RefReddy30th/ep.533-534]
Opioids(opiates)arederivedfromthepoppyplant.Opiumisthe
driedjuiceobtainedbyincisionoftheunripecapsuleofthewhite
poppy,papaversomniferum.Naturalderivativesofopiumare
morphine,heroinandcodeine.

670.Arrowpoisonis?
a)Opium
b)Curare
c)Cannabis
d)Cyanide
CorrectAnswer-B
Ans.is'b'i.e.curare[RefDr.AnilAggrawalp.447]
Arrowpoisons?

1. Plantbased-Abrusprecatorius,aconite,calotropis,crotonoil,
curare,strychnine,stropanthus.
2. Animalbased-Batracotoxin(BTX).

671.Acidusedforforgingsignatureis?
a)Sulphuricacid
b)Nitricacid
c)Carbolicacid
d)Oxalicacid
CorrectAnswer-D
Ans.is'd'i.e.,Oxalicacid[RefParikh6th/ep.8.32-8.34;Reddy's
Essentialsofforensicmedicineandtoxicology27th/ep.468-
469]
Oxalicacidisusedtoerasewriting,asableachingagentandin
calicoprinting

672.Smackis-
a)Cocaine
b)Heroine
c)Dhatura
d)Cannabis
CorrectAnswer-B
Ansis'b'i.e.,Heroin
Heroin(smackorbrownsugar)isthemostcommonlyabusedopioid
and2"mostcommonlyabusedsubstance,afteralcohol.

673.MickeyFinnis?
a)Chloroform
b)Methylalcohol
c)Chloralhydrate
d)Ethyleneglycol
CorrectAnswer-C
Ans.is'c'i.e.,Chloralhydrate[RefReddy30th/ep.540]
Mickeyfinnisacombinationofalcoholandchloralhydrate.

674.Thornappleis?
a)Dhatura
b)Naxvomika
c)Opioid
d)Cannabis
CorrectAnswer-A
Ans.is'a'i.e.Dhatura[RefParikh&hiep.10.50-10.51;
Essentialsofforensicmedicine-812]
Dhatura(thornapple)growsonwastelandalloverindiaandisof
twotypes(i)Dhaturaalbaand(ii)dhaturaniger.Allpartsofplantis
poisonous,especiallyseedsandfruits.Activeprinciplesofdhatura
arelevohyoscyamine,hyoscine(scopolamine)andatropine.

675.Acidinjurycanbedifferentiatedfrom
alkaliinjuryby?
a)Liquefactivenecrosis
b)Acidburnsaremoredangerous
c)Perforationismorecommon
d)Affectsmorecommonlyesophagus
CorrectAnswer-C
Ans.is'c'i.e.Perforationismorecommon

676.BestmethodoftreatmentofMethyl
alcoholpoisoningis:
a)Calciumgluconate
b)Ethylalcohol
c)Amphetamines
d)1%Ammonia
CorrectAnswer-B
Bi.e.Ethylalcohol

677.Howmanydaysapersoncanlive
withoutfoodandwater?
a)1to2days
b)2to4days
c)5to7days
d)10to12days
CorrectAnswer-D
Ans.is.d,10to12days[RefParikh6th/ep.3.74;SKSinghalele
p.146]
Feelingofintensehungerlastsfor30-48hours
Emaciation&absorptionofsubcutaneousfatbeginstooccurAfter
4-5days
Newbornmaysurvivewithoutfood&waterFor7-10days
Adultmaysurvivewithoutfood&water10-12days
Iffoodaloneiswithdrawndeathoccursin6-8weeks(50-60days)

678.MTPcanbedonebyasingledoctortill?
a)8weeks
b)12weeks
c)20weeks
d)24weeks
CorrectAnswer-B
Ans.is'b'i.e.,12weeks[RefDutta6th/ep.174;Park22"/ep.
467]
TheActprovidessafeguardstothemotherbyauthorizingonlya
RegisteredMedicalPractitionerhavingexperienceingynecology
andobstetricstoperformabortionwherethelengthofpregnancy
doesnotexceed12weeks.

679.Alcoholistakenwithaeratedsoftdrinks
because-
a)Absorptionisfaster
b)Effectisincreased
c)Toavoidhangover
d)Noneoftheabove
CorrectAnswer-A
Ans.is'a'i.e.,Absorptionisfaster[RefHandbookofAlcoholic
Beverages]
Therateofalcoholabsorptionisdependentonitsconcentration,
beingthehighestfrombeveragescontainingabout20%to30%
alcohol,andonthespeedofitsconsumption.
Alcoholconsumedinacratedbeveraesismorereadilyabsorbed,as
isalcoholtakenonanemptystomach.

680.Blistersareabsentin?
a)Burns
b)Putrefaction
c)ArsenicPoisoning
d)Postmortemcaloricity
CorrectAnswer-D
Ansis'd'i.e.,Postmortemcaloricity[RefReddy03tivep.
139,140;S.K.Singhalp.92]
Thereisnoblisteringinpostmortemcaloricity.
Allotherconditionsareassociatedwithblisterformation

681.Miner'scrampsarealsocalled:
NEET13
a)Heatcollapse
b)Heatcramps
c)Sunstroke
d)Heatexhaustion
CorrectAnswer-B
Ans.Heatcramps

682.Ophitoxemiais?
a)Snakevenompoisoning
b)Scorpionbite
c)Spiderbite
d)Tickbite
CorrectAnswer-A
Ans.is'a'i.e.,Snakevenompoisoning[RefRameshGupta
Zoologybook]
Ophitoxemiaispoisoningbysnakevenom.

683.Afterdeath,bloodistakenfrom?
a)Femoralvein
b)Antecubitalvein
c)Jugularvein
d)Carotidartery
CorrectAnswer-A
Ans.is'a'i.e.Femoralvein[RefForensicNeuropathologyp.174]
"Ideally,thebloodshouldbecollectedfromfemoralveindistalto
clamporligatureandwithoutmilkingthecalforthigh"-Forensic
Neuropathologyp.174
"Femoralveinbloodsampleispreferredoveraheartbloodsample"
-Water-RelatedDeathInvestigationp.269
Bloodforthesampleistakenfromfemoralvein.Thejugularor
subclavianveinscanalsobeused.10-20mlofbloodistakenandit
istakenbeforeautopsy.30mlofbloodshouldbepreserved
(minimumis10ml).

684.Coronamortisis?
a)Isavascularanastomosis
b)Postmortemheart&coronaryarteriesexamination
c)Anothertermusedforrigormortis
d)Noneoftheabove
CorrectAnswer-A
Ans.is'a'i.e.Isavascularanastomosis[RefBRSGross
Anatomyp.112]
Coronamortis:
Thisisalsocalledascircleofdeathandreferstovascularringform
bytheanastomosisofanaberrantarterywiththenormalobturator
arteryarisingfromabranchoftheinternaliliacartery.Atthetimeof
laparoscopicherniathisvesselistornbothendofvesselcanbleed
profusely,becausebotharisefromamajorartery.
Thesurgeonshouldremembertheseanatomiclandmarksandthe
pointofmeshfixationshouldbeselectedsuperiorly,laterallyand
medially.

685.Crystalvioletbloodagarisusedfor
whichbacteria?
a)Corynebacteriumdiphtheriae
b)Staphaureus
c)-hemolyticstreptococcus
d)Meningococcus
CorrectAnswer-C
-hemolyticstreptococcus
Crystalvioletbloodagarisaselectivemediumforisolationofp-
hemolyticgroup-Astreptococcus.
oCrystalvioletpermitsthegrowthofstreptococcusbutinhibits
growthofothergrampositivebacteria(including
staphylococcus)

686.Exampleoftransferofdrugresistance
byconjugation-
a)Staphylococcitorifampicin
b)PneumococcustopenicillinG
c)Mtuberculosistoantituberculardrugs
d)Ecolitostreptomycin
CorrectAnswer-D
Ans.is'd'i.e.,Ecolitostreptomycin[RefBasicsinlaboratory
microbiologyp.781]
Drugresistance
Itreferstounresponsivenessofamicroorganismtoanantimicrobial.
Drugresistancemaybeacquiredby:
1.Mutation
Itmaybesinglestep(entercoccitostreptomycine,staphlococcito
rifampicin)ormultistep(erythromycin,tetracycline,chloramphenicol,
salmonellaforciproflaxacin).
Mutationaldrugresistanceisalsoimportantintuberculosis.
2.Genetictransfer
Drugresistancefromoneorganismtoothermaybetransferredby:
Conjugation(mostcommon):Itisresponsibleformultidrug
resistance.ItisimportantforresistanceofS.typhiagainst
chloramphenicolandE.coliagainststreptomycin.
Transduction:Mostimportantfortransferofresistancein
staphylococciby(3?lactamase.
Transformation(lesssignificant):Importantinpneumococci
resistancetopenicillinG

687.Multipledrugresistanceistransferred
through-
a)Transduction
b)Transformation
c)Conjugation
d)Mutation
CorrectAnswer-C
Ans.is'c'i.e.,Conjugation
ResistancetomultipledrugsistransmittedbyRfactor(plasmid)
-Rfactoristransferredfromonebacteriumtootherbyconjugation.
-Transductionandmutationusuallycauseresistancetoonedrug.
-Transformationisnotinvolvedsignificantlyindrugresistance.

688.Mediumusedforantibioticsensitivity:
a)CLEDagar
b)Hektoenagar
c)Mueller-Hintonagar
d)Saltmilkagar
CorrectAnswer-C
Ans.is.'c'i.e.,Mueller-Hintonagar
Antibioticsensitivitytesting
Antibioticsensitivitytestingiscarriedouttodetermineappropriate
antibiotictobeusedforaparticularstrainisolatedfromclinical
specimens.
Antibiotictestingcanbecarriedoutbytwobroadmethods:
a)Discdiffusiontests
b)Dilutiontests
A)Discdiffusiontests
Therearemostcommonlyusedmethodstodetermineantibiotic
susceptibility.
Discsimpregnatedwithknownconcentrationofantibioticsandare
placedontheculturemediumthathasbeeninoculatedwitha
cultureofbacteriumtobetested.
Antibioticsensitivityisdeterminedbyzoneofinhibitionofbacterial
growtharoundthedisc.
Selectionofmedia
Mediumthatsupportsbothtestandcontrolstrainsisselectedfor
carryingoutantibioticsusceptibilitytesting.
Forexample:
Muller-Hintonagarisusedforgram-negativebacilliand
staphylococcusspp.

Bloodagarisusedforstreptococcussppandenterococcusspp.
ChocolateagarisusedforHaemophilusinfluenzae
Wellcotestmediumusesantibioticssulfonamideandcotrimoxazole
Mueller-Hintonagarismostcommonlyusedmediumforantibiotic
sensitivitytesting
Typeofdiscdiffusiontest.
Discdiffusiontestsarefollowingtypes:?
i)Kirby-Bauerdiscdiffusionmethod-mostcommonlyused.
ii)Stokediscdiffusionmethod
iii)Primarydiscdiffusiontest


689.Shadowcastingisusedin-
a)Lightmicroscopy
b)Electronmicroscopy
c)Opticalmicroscopy
d)Fluoroscencemicrospopy
CorrectAnswer-B
Ans.is'b'i.e.,Electronmicroscopy[RefCellandmolecular
biologyp.726]
Twostandardmethodsofstaininginelectronmicroscopyare:

1. Shadowcasting
2. Negativestaining

690.ModifiedZiehl-neelsenstainingisused
for:
a)Mycobacteriumtuberculosis
b)Mycobacteriumbovis
c)Nocardia
d)Alloftheabove
CorrectAnswer-D
Ans.is.'d'i.e.,Alloftheabove

691.ViralDNAisintegratedintoBacterial
DNAin:
a)Transduction
b)Lysogenicconversion
c)Transformation
d)Conjugation
CorrectAnswer-B
Ans.(b)Lysogenicconversion

692.Frozenphenomenonisusedfor-
a)Sterilizationofheatsensitivematerial
b)Killingthermophilicbacteria
c)Preservationofmicroorganisms
d)Stimulatingmetabolismofmicroorganism
CorrectAnswer-C
Ans.is'c'i.e.,Preservationofmicroorganisms
[RefMicrobiologyApplicationBasedApproachp.189]
Somepsychrophilicbacteriacangrowat0?C,butsubzero
temperatureswillinhibitthemetabolismofmicroorganismingeneral.
Freezingiscommonlyusedtopreservefoods,drugsandlaboratory
specimensbecauseiteffectivelystopsmicrobialgrowth.
However,subzerotemperaturemaynotkillmicroorganism
(especiallypsychrophilic)andmayinfactpreservethemalongwith
thematerialbeingfrozen.
Thisphenomenonhasbeenusedbymicrobiologiststostoreand
preservemicroorganisms.

693.Trueaboutuniversalprecautionsareall
except-
a)Topreventtransmissionofbloodbornepathogens
b)Includesuseofhandwashing
c)Considerthatallbodyfluidsarecontaminatedwithblood
d)Includesuseofglovesandmasks
CorrectAnswer-B
Ans.is'b'i.e.,Includesuseofhandwashing[RefMicrobiology
forsurgicaltechniques
p.796]
Handwashingisapartofstandardprecautionsnotuniversal
precautions
Universalprecautions
Universalprecautionsisanapproachtoinfectioncontroltotreatall
humanbloodandcertainhumanbodyfluidsasiftheywereknownto
beinfectiouswithHIV,HBVandotherbloodbornepathogens.
Bloodbornepathogenstandardrequires.
Employeestoobserve'universalprecautions'topreventcontactwith
bloodorotherpotentiallyinfectiousmaterial(OPIM)
TreatallbloodandOPIMwithproperprecautionslikeuseofgloves,
masksandgown.
Otherpotentiallyinfectiousmaterialincludes:
1. Bodyfluids:CSF,semen,vaginalsecretion,synovialfluid,pleural
fluid,pericardialfluid,peritonealfluid,amnioticfluid,salivaindental
procedures,anybodyfluidcontaminatedwithblood.
2. Anyunfixedtissueororgan(otherthanskin)fromhuman.
3. HIV-containingcells,fluidsorcultures.
Universalprecautionshavebeenupdatedintostandardprecautions
thatstateallbodyfluid(exceptsweat)shouldbeconsidered

infectious.

694.Nucleicacidisnotfoundin-
a)Virus
b)bacteria
c)Fungus
d)Prions
CorrectAnswer-D
Ans.is'd'i.e.,Prions[RefHarrison19'/ep.451&17th/ep.2647]
Prionsareinfectiousparticlewhichcontainsproteinonly.
Theydonothavenucleicacid.

695.Granulomatosisinfantisepticaiscaused
by:
a)Pseudomonas
b)Chlamydiatrachomatis
c)GroupDstreptococci
d)Listeria
CorrectAnswer-D
Granulomatosisinfantisepticaisanintra-uterineinfectionofthe
newborncausedbylisteria.
Thepredominantfeatureofthisconditionisextensivefocalnecrosis
affectingespeciallytheliverand,lessoften,thelungs.
Themortalityrateisveryhigh.
Occasionallymeningitismaybeassociatedwithit.
Theorganismscanbeisolatedfromtheaffectedareasinthechild
andfrequentlyfromthegenitaltractofthemother,whomayormay
notmanifestthedisease.
Ref:Harrisonsprinciplesofinternalmedicine,18thedition,Page:
1196.

696.Salmonellaandshigellacanbe
differentiatedfromother
enterobacteriaceaememberbyisolation
on:

a)MacConkeyagar
b)Mannitolsaltagar
c)BCYEmedium
d)XLDagar
CorrectAnswer-D
Ans.is.'d'i.e.XLDagar
XLD(xyloseJysinedeoxycholate)agarisaselectivedifferential
mediumforisolationofGram-negativeentericpathogens
fromfecalspecimensandotherclinicalmaterial.
oItisespeciallysuitablefortheisolationofshigellaandsalmonella
specias

697.Trueaboutanthraxtoxinareallexcept:
a)Hasthreefractions
b)IncreasecAMP
c)Codedbyplasmid
d)Inhibitsproteinsynthesis
CorrectAnswer-D
Ans.is.'d'i.e.Inhibitsproteinsynthesis

698.Chlamydiapneumoniaecausescauses:
a)LGV
b)Atherosclerosis
c)Inclusionconjunctivitis
d)Trachoma
CorrectAnswer-B
Ans.is.'b'i.e.,Atherosclerosis

699.NottureaboutElTorbiotypeofvibrio
cholerae?
a)Lowermortality
b)LessSAR
c)Lesschancesofsurvivalinenvironment
d)VP(+)
CorrectAnswer-C
Ans.is'c'i.e.,Lesschancesofsurvivalinenvironment

700.Trueaboutvibrioparahemolyticus?
a)Polarflagella
b)Nonhalophilicvibrio
c)Non-capsulated
d)RequiresNaCI
CorrectAnswer-D
Ans.is'd'i.e.,RequiresNaCI
Itishalophilicvibrio.
*Inhabitsthecoastalsea,whereitisfoundinftshesarthopodssuch
asshrimpsandctabsandmolluscssuchasoyster.
*Itresemblesthecholeravibrioexceptthat:
-->Itiscapsulated.
-->Showsbipolarstaining
*Producesperitrichousflagellawhengrownonsolidmediun(V.
choleraehaspolarflagella),inliquidmediumpolarflagellaare
formed.
*Itgrows,onlyinmediacontainingNaCl,optimumconc.is2-4
%,ltsenteropathogeniciscloselylinkedto
itsabilitytocausehemolysisonWagatsumaagarttheKanagawa
phenomenon.

701.Hansen'sbacillusisculturedin:
a)LJmedium
b)Robertson'scookedmeatmedium
c)Footpadofmice
d)Sabraud'sagar
CorrectAnswer-C
Ans.is.'c'i.e.,Footpadofmice

702.Tennisracketcellsisseenin-
a)Sarcomabotyroides
b)Vaginaladenocarcinoma
c)Leiomyomauterus
d)Seminoma
CorrectAnswer-A
Ans.is'a'i.e.,Sarcomabotryoids
oTumorcellsofsarcomabotryoidsaresmallandhaveovalnuclei,
withsmallprotrusionsofcytoplasmfromoneend,sotheyresemble
atennisracket.

703.Mostcommonorganisminvolvedin
nosocomialinfection-
a)Staphaureus
b)E.coli
c)Legionella
d)Streppneumonia
CorrectAnswer-A
Ans.is'a'i.e.,Staphaureus
.UTIisthemostcommontypeofnosocomialinfection.
.MostcommoncauseofUTI--->Ecoli
.MostcommoncauseofUTIinICU---->Candida
.Overallstaphaureusisaleadingcauseofnosocomialinfection.
.Staphaureusisthemostcommoncauseofsurgicalwound
infection-
.Mostcommoncauseofprimarybacteremia-->Coagulase
negativestaphylococcus.

704.Draughtsmancoloniesareseenwith:
a)Anthrax
b)Pnuemococci
c)Pertussis
d)Yersenia
CorrectAnswer-B
Pnuemococci
Duetoalphahemolysk,coloniesofpneumococcircsemblecolonies
ofStr.virtdans.Butonfurtherincubationthecolonies
ofpneumococcibecomeflatwithraisededgesandcentral
umbonation,sothatconcentricringsareseenonthesurfacewhen
viewedfromabove-draughtsmanorCarromcoinappearance.

705.Allaretrueaboutchromobacteritun
violaceumexcept?
a)Gramnegative
b)Producesviolet-coloredpigment
c)Normalflorainhuman
d)Causescellulitis
CorrectAnswer-C
Ans.c.Normalflorainhuman

706.Oilpaintappearanceonnutrientagaris
seenin-
a)Streptococcuspyogenes
b)Staphylococcusaureus
c)Bordtellapertussis
d)H.influenzae
CorrectAnswer-B
Ans.is'b'i.e.,Staphylococcusaureus
Staphylococcusisfacultativeanaerobe.OptimumpHforgrowthis
7.4-7.6andoptimumtemperatureis37?C.
Staphaureusproducesgoldenyellowpigment,whichismaximumat
22?C.
Mostofthestaphylococcusspeciesgrowinthepresenceof10%
NaCl

707.Streptococcalpneumoniaepneumonia
presentat-
a)<5years
b)5-15years
c)20-25years
d)30-40years
CorrectAnswer-A
Ans.is'a'i.e.,<5years[Ref:TextbookofMicrobiologyby
Parijap.197]
Streptococcuspneumoniaeistheleadingcauseofpneumonia,both
lobarandbronchopneumonia.
Pneumoniaiscommonattheexteremeofages:
Children(especially<5yearsofage)
Elderly(>60-65years)
Serotypes6,14,18,19and23areresponsibleformostcasesof
pneumoniainchildren,whileserotypes1,3,4,7,8and12cause
pneumoniainadults.

708.Mostcommonagegroupaffectedby
streptococcuspyogenes-
a)<5years
b)5-15years
c)20-25years
d)30-40years
CorrectAnswer-B
Ans.is'b'i.e.,5-15years[RefParijap.191]
Sorethroat(tonsillitis)isthemostcommonmanifestationof
streptococcalinfectionandaffects5-15yearsagegroup.

709.Naegler'sreactionisdueto:
a)Coagulase
b)Hyaluronidase
c)Lecithinase
d)Noneoftheabove
CorrectAnswer-C
Ans.is.'c'i.e.Lecithinase

710.Oropharyngealcommensalwhich
predisposestocandidiasis-
a)Hemophilusinfluenzae
b)Streptococcus
c)Staphylococcus
d)Lactobacillus
CorrectAnswer-B
Ans.is'b'i.e.,Streptococcus[RefTextbookofpolymicrobial
disease-Chapter18]
Anexampleofthecomplexityofcoaggregationmaybetherangeof
intergenericcoaggregationsoccuringbetweentheoralfungal
pathogencandidaalbicansandotheroralspeciesthatmayplayan
importantroleinthecolonizationoftheoralcavitybycandida
albicans.
Althoughstreptococcalspecies,namely,streptococcusgordonii,
streptococcusoralisandstreptococcussanguins,exhibitthehighest
affinitiesforCalbicans,Calbicans(aswellasCandidadubliniensis)
havebeenshowntocoaggregatewithFusobacteriumspeciesin
suspension.
Actinomyceshasalsobeenshowntocoaggreatewithcandida
albicans.

711.PrimarycomplexofMbovisinvolves:
a)Tonsilandlung
b)Tonsilandintestine
c)Tonsilandskin
d)SkinandIntestine
CorrectAnswer-B
Ans.is.'b'i.e.,Tonsilandintestine
Ans.is.'b'i.e.,Tonsilandintestine
PrimarycomplexofMtuberculosis(infectionbyinhalation)
i)Lunglesion
ii)Enlargedhilarlymphnodes
oPrimarycomplexofMbovis(infectionbydrinkingmilk)
i)Tonsil
ii)Cervicalnodesorintestine(ileocecalregionand
mesentriclymphnodes)

712.Hemophilusparainfluenzaerequres-
a)FactorV
b)FactorX
c)FactorV&X
d)FactorVII
CorrectAnswer-A
Ans.is'a'i.e.,FactorV[Ref:Ananthanaran9"/ep.327&8thie
p.330]
H.influenzae,H.aegyptius,H.haemolyticusFactorX&V.
H.Parainfluenzae,H.Parahemolyticus,H.ParaphrophilusFactor
V.
H.ducreyi,H.aphrophilusFactorX.

713.Brill-Zinsserdiseaseis:
a)RecrudescentofRprowazekiiinfection
b)RecrudescentofRtyphiinfection
c)RecrudescentofRconoriiinfection
d)None
CorrectAnswer-B
Ans.is.'b'i.e.,RecrudescentofRtyphiinfection

714.Allaretrueaboutlisteriaexcept:
a)Grampositive
b)PALCAMagarisusedforisolation
c)Characteristictumblingmotilityat37?C
d)Umbrellashapedgrowth
CorrectAnswer-C
Ans.is.'c'i.e.,Characteristictumblingmotilityat37?C
L.monocytogenesisagram-positive'
coccobacillus'(coccoidrod)withatendencytooccurinchains.
Peritrichousflagellaareproducedbythebacillusoptimallyat20-
30?Cbutonlyscantilyornotatallat37?C
Culturemediausedforisolationarebloodagar,chocolateagar,
andPALCAMagar.
Itgrowsonordinarymediawithinatemperaturerangeof1?to45?C.
Mostcasesofhumandiseasearecausedbyserotypes1/2a,1/2b
and4b.
Theorganismcanbefoundasapartofthegastrointestinalflorain
healthyindividuals.
HumandiseaseduetoL.monocytogenesgenerallyoccursinthe
settingofpregnancyorimmunosuppression.

715.Listeriaresistsphagocytosisin
phagosomes(phagolysosomes)dueto:
a)-hemolysin
b)Caspases
c)Cellmembraneadhesionmolecules
d)Opacityassociatedprotein(OAP)
CorrectAnswer-A
Ans.is.'a'i.e.,-hemolysin
Mostimportantstepinpathogenesisoflisteriosisisthesurvivaland
multiplicationofLmonocytogenesinphagocytes(macrophages),
hostepithelialcellsandhepatocytes.
*Themostimportafltdeterrminantofpathogenesisk
listeriolysinO(LLO),Abeta-hemolysin.LLOcauseslysisof
membraneofphagosomeswithinphagocytesand
helpsinintracellularsurvivalofLmonocytogenes.
*Ironisanimportantvirulencefactor.Listeriaproducesiderophores
andareabletoobtainironfromtransferrin.Immunityto
listeriaisprimarycellmediated.Immunitycanbetransferredby
sensitizedlymphocytesbutnotbyantibodies.

716.Mostcommonformofleptospirosis:
a)Weil'sdisease
b)Ictericform
c)Hepatorenalform
d)Anictericform
CorrectAnswer-D
Ans.(d)Anictericform
Itismostcommon(90%)andmildform.Most
commonfindingkfnerwithconjunctivalsuftrcion


717.Bacteriathatcangroweveninthe
presenceofantiseptic:
a)Staphylococcus
b)Streptococcus
c)E.coli
d)Pseudomonas
CorrectAnswer-D
Ans.is.'d'i.e.,Pseudomonas

718.Achronicalcoholicispresentingwith
clinicalfeaturesofmeningitis.Most
likelyorganismwhichwillgrowonCSF
culture:

a)Streptococcuspneumoniae
b)N.meningitidis
c)Listeriamonocytogenes
d)F.coli
CorrectAnswer-A
Streptococcuspneumoniae

719.Vincent'sanginaiscausedbyBorrelia
vincentiialongwith:
a)Lactobacillus
b)Lactobacillus
c)Fusobacterium
d)Bacteroides
CorrectAnswer-C
Ans.(c)Fusobacterium

720.Bartonellaquintanacauses:
a)Trenchfever
b)Scrubtyphus
c)Endemictyphus
d)Epidemictyphus
CorrectAnswer-A
Ans.is.'a'i.e.,Trenchfever

721.Whatistrenchfever:
a)Q-fever
b)5-daysfever
c)Boutonneusefever
d)Indianticktyphus
CorrectAnswer-B
Ans.is.'b'i.e.,5-daysfever

722.ProteinAofstaphylococcusbindsto
a)IgA
b)IgG
c)IgD
d)IgE
CorrectAnswer-B
Ans.is.b.IgG

723.Whichstreptodornaseismostantigenic
inhumanbeings:
a)A
b)B
c)C
d)D
CorrectAnswer-B
Ans.is.b.B

724.Pneumonicplagueisspreadby:
a)Biteofinfectedflae
b)Directcontactwithinfectedtissue
c)Ingestionofcontaminatedfood
d)Dropletinfection
CorrectAnswer-D
Ans.is.'d'i.e.,Dropletinfection

725.ProteusisolatedfromapatientofUTIwill
showwhichboichemicalreaction:
a)Phenylpyruvicacidreaction
b)Bileesculinreaction
c)Colchicinesensitivity
d)Bacitracinsensitivity
CorrectAnswer-A
Ans.is.'a'i.e.,Phenylpyruvicacidreaction
stofthemexceptprovidenciastrains,producepowerfulurease.
-->Acharactersticfeatureofproteusbacilliis"PPAreaction'
-->Itisduetopresenceofenzymepheny'alaninedeaminasewhich
convertsphenylalaninetophenylpymvicacid

726.Nottrueaboutgasgangrene:
a)MostcommoncauseisClperfringens
b)Extensivenecrosisofmuscles
c)Clperfringensproduceheat-labilespores
d)Metronidazoleisthedrugofchoice
CorrectAnswer-D
Ans.is.'d'i.e.Metronidazoleisthedrugofchoice

727.Mostsensitivetestinsyphilis-
a)VDRL
b)TP-PA
c)RPR
d)FTA-ABS
CorrectAnswer-B
Ans.is'b'i.e.,TP-PA[RefHarrisonMolep.1137,1138
e.718th/ep.1385,Ananthanarayan9th/ep.374,375&p.374-78]
TPPAisthemostsensitiveserologicaltestoverall(consideringall
stagesofsyphilis).
Alltheserologicaltestshave100%sensitivityinsecondarystageof
syphilis
Test Primary Secondary Latent
Tertiary
VDRL/ 78(74-
100
95(88-
71(37-
RPR
87)
100)
94)
FTA- 84(70-
100
100
96
ABS
100)
TP?PA
89
100
100
NA

728.Mostcommonsiteforstaphylococcus
carrier:
a)Skin
b)Nose
c)Oropharynx
d)Perineum
CorrectAnswer-B
Ans.is.b.Nose

729.Brazilianpurpuricfeveriscausedby:
a)Bordetellapertussis
b)Haemophilusaegypticus
c)Haemophilusduceryi
d)Haemophilusparinfluenzae
CorrectAnswer-B
Ans.is.'b'i.e.,Haemophilusaegypticus

730.EscharisseeninalltheRickettsial
diseasesexcent:
a)Scrubtyphus
b)Rickettsialpox
c)Indianticktyphus
d)Endemictyphus
CorrectAnswer-D
Ans.is.'d'i.e.,Endemictyphus

731.Freshwaterswimmingleadstoinfection
by-
a)Bordetellapertussis
b)Corynebacteriumdiphtheriae
c)Mtuberculosis
d)Pseudomonas
CorrectAnswer-D
Ans.is`d'i.e.,Pseudomonas[RefAlcamo'sfundamentalsof
microbiologyp.669]
"Extendedswimminginfreshwaterpoolscanirritateandbreak
downskinintheearcanalallowingbacteriasuchaspseudomonas,
staphylococcusorstreptococcustopenetrateandcauseotitis
externa"
Pseudomonasisthemostcommoncauseofswimmer'sear
infection.
Pseudomonasalsocauses"hottubrash"syndromeorhot-tub
folliculitis,associatedwithuseofhottub,lesscommonlyassociated
withwhirlpoolsorswimmingpools.

732.Commonestcauseforpuerperalsepsis
is:
a)Streptococci
b)Anaerobes
c)Gonococci
d)Staphylococci
CorrectAnswer-A
Streptococci

733.Mostcommoncomplicationofdiphtheria
is-
a)Myocarditis
b)Pneumonia
c)Meningitis
d)Endocarditis
CorrectAnswer-A
Ans.is'a'i.e.,Myocorditis[RefWithtext]
"Myocarditisisthemostcommoncomplicationofdiphtheriaandthe
leadingcauseofdeathindiphthericcases,whoseincidenceis10-
25%"
EssentialsinMicrobiology"Complicationsofdiphtheriainclude
myocarditis(themostcommoncomplication),thrombocytopenia,
vocalcardparalysisandneuritis"Clinicalmicrobiology.
"Mostcommoncomplicationsofdiphtheriaaremyocarditisandtoxic
neuritis"NMS
Twomostcommoncomplicationsofdiphtheriaare:
1.Myocarditis
2.ToxinneuritisParalysisofsoftpalateisthemostcommon
manifestationofneuritis.

734.Allaretrueaboutanaerobicinfection
except-
a)Mostinfectionsareendogenous
b)Exudatesandswabsarebestforculture
c)SpecimenforUTIissuprapubicaspiration
d)TheyarefoundnormallyonskinandGIT
CorrectAnswer-B
Ans.is'b'i.e.,Exudatesandswabsarebestforculture[Ref
Essentialsofmicrobiologyp.224]
Mostanaerobicbacteriathatcauseinfectionaremembersofour
normalindigenousfloraandanaerobicinfectionsareusually
endogenous,causedbytissueinvosionbybacterianormally
resisdentorrespectivebodysurfaces.
Anaerobicbacteriaarenormallypresentonskin,mouth,
nasopharynx,upperrespiratorytract,gutandvagina.
Idealspecimensforanaerobicculturesaresamplesofneedle
aspiratesandpropertissuespecimens.Anaerobicswabsareusually
discouraged.
Importantspecimensare:-
1. Localabscess:Needleaspirates.
2. Pulmonary:Transtrachealaspirates,lungaspirates,pleuralfluid,
protectedbronchialwash.
3. Abdominal:Abdominalabscessaspirate.
4. Urinarytract:Suprapubicbladderaspirate.
5. Genitaltract:Culdocentesisspecimen,endometrialswabs.
6. CNS:CSF,Aspirateofabscess

735.Mostcommonrouteofinfectionin
pasteurellacellulitis-
a)Animalbitesorscratches
b)Aerosolsordust
c)Contaminatedtissue
d)Humantohuman
CorrectAnswer-A
Ans.is'a'i.e.,Animalbitesorscratches[RefAnanthanarayan
9thlep.325;7h/ep.326;Harrison19thlep.830&le/ep.1235]
Transmissiontohumansmayoccurbytworoutes?
1)Directinoculationthroughskin
ThemostcommonmodeoftransmissionofP.microcidaisdirect
inoculationbyabiteorscratch.Mostoftheinfectionsarecausedby
cats.
Infectionmayalsooccurviadepositionoforganismoninjuredskin
ormucosalsurfacesduringlicking.
2)Throughtherespiratorytract
Thisisthesecondmostcommonmodeoftransmission.
Infectionviatherespiratorytractoccursfromcontactwith
contaminateddustorinfectiousdroplets(aerosoldust).

736.Trueaboutlegionella:
a)Mostcommonmodeoftransmissionisaerosolinhalation
b)Thereisnomantomantransmission
c)Prolongedcarrierarecommon
d)Allaretrue
CorrectAnswer-B
Ans.is.'b'i.e.,Thereisnomantomantransmission

737.Hebranoseiscausedby:
a)Frischbacillus
b)Staphaureus
c)Pseudomonas
d)C.diphtheriae
CorrectAnswer-A
Ans.is.'a'i.e.,Frischbacillus

738.Granulomainguinaleiscausedby:
a)H.ducreyi
b)Chlamydiatrachomatis
c)Treponemapallidum
d)Calymmatobacterium
CorrectAnswer-D
Ans.is.'d'i.e.,Calymmatobacterium
Granulomalenerutn(granulomainguinal)-->Calymmatobacterium

739.Izumifeveriscausedby:
a)Pseudomonasaeruginosa
b)Burkholderiamallei
c)Yersiniapseudotuberculosis
d)Pasteurellamultocida
CorrectAnswer-C
Ans.is.'c'i.e.,Yersiniapseudotuberculosis

740.Trueaboutwidaltest:
a)Anti-0antibodypersistslonger
b)0antigenofS.paratyphiisused
c)H-antigenismostimmunogenic
d)Felixtubeisusedfor'H'agglutinationlnar
CorrectAnswer-C
Ans.is.'c'i.e.,H-antigenismostimmunogenic

741.AbsenceofVi-antibodyinatyphoid
patienthas:
a)Goodprognosis
b)Badprognosis
c)Norelationwithprognosis
d)Indicateswidalnegative
CorrectAnswer-B
Ans.is.'b'i.e.,Badprognosis

742.ClinicalsignificanceofViantigenofS.
typhiis:
a)Helpsindiagnosis
b)Highlyimmunogenic
c)Mostimportantantigenforwidaltest
d)AntibodyagainstVi-antigenisusedfordiagnosisofcarrier
CorrectAnswer-D
Ans.is.'d'i.e.,AntibodyagainstVi-antigenisusedfordiagnosisof
carrier

743.Non-motilebacteriumis-
a)Vibrio
b)Clostridiumsepticum
c)Clostridiumperfringens
d)Legionella
CorrectAnswer-C
Ans.is'c'i.e.,Clostridiumperfringens[RefAnanthanarayan
9th/ep.262,263,252-257]
AllclostridiaaremotilebyperitrichousflagellaexceptC/tetaniVI
andClperfringenswhicharenon-motile.

744.Whichofthefollowingisnon-motile:
a)Pseudomonasaeruginosa
b)Burkholderiamallei
c)Burkholderiapseudomallei
d)Noneoftheabove
CorrectAnswer-B
Ans.is.'b'i.e.,Burkholderiamallei

745.Phagocytosisofmycobacterium
tuberculosisbymacrophagesismainly
mediatedby:

a)IL6
b)IL3
c)IL12
d)IFNGamma
CorrectAnswer-D
Ans.is.'d'i.e.,IFNGamma

746.NottrueaboutVipolysaccharidevaccine
oftyphoid:
a)Singledoseisgiven
b)Revaccinationat3years
c)Givenatbirth
d)Givensubcutaneously
CorrectAnswer-C
Ans.is.'c'i.e.,Givenatbirth

747.Ehrlichiaphagocytophilamainlyaffects-
a)RBC
b)Platelets
c)Neutrophils
d)Macrophages
CorrectAnswer-C
Ans.is'c'i.e.,Neutrophils[RefEssentialsofclinical
microbiologyp.712]
Anaplasmaphagocytophilum(formerlyknownasEhrlichia
phagocytophila)isacausativeagentoftick-bornefeverinsheep
andpasturefeverincattle.
Itisanobligateintracellularbacteriumwhosemaintargetis
neutrophilsbutitcanalsoinfectmonocytes.

748.Testtodifferentiatestaphylococcifrom
micrococci:
a)Catalasetest
b)Coagulasetest
c)Novobiosinsensitivity
d)Oxidationfermentation
CorrectAnswer-D
Oxidationfermentation

749.Nottrueaboutcorynebacterium
hormannii
a)Adiphtheroid
b)Non-pathogenicsaprophyte
c)Toxigenic
d)AlsoknownasCpseudodiphthericum
CorrectAnswer-C
Ans.is.'c'i.e.Toxigenic

750.Trueaboutcorynebacteriumdiphtheriae:
a)Alltypesproducetoxin
b)Toxinproductionisdependentuponcriticalconcentrationof
iron
c)Heatstabletoxin
d)InhibitcAMP
CorrectAnswer-B
Ans.is.'b'i.e.Toxinproductionisdependentuponcritical
concentrationofiron

751.Bileesculinagarisusedfor?
a)GroupAstreptococcus
b)GroupBstreptococcus
c)GroupCstreptococcus
d)Enterococcus
CorrectAnswer-D
Ans.is'd'i.e.,Enterococcus
Enterococcus
MajorityoftheinfectionsarecausedbyE.faecalisandE.faecium.
LessfrequentlyisolatedspeciesareE.gallinarum,E.durans,E.
hiraeandE.avium.
Enterococciarenormalinhabitantsofthelargebowelofhuman
adults,
althoughtheyusuallymakeup<1%oftheculturable
intestinalmicroflora.
Theyarecatalasenegative(asallstreptococci).
Theircharacteristicfeatureisthattheycangrowinpresenceof
:?

1. 40%bile
2. 6.5%Sodiumchloride
3. AtpH9.6
4. At45?C(relativeheatresistantsurving60?Cfor30minutes)
5. In0-1%methylenebluemilk
Theyhydrolyzeesculin.Theygrowinpresenceof40%bileand
hydrolyzeesculinBileesculinpositive.
TheyarePYR(PyrrolidonylArylamidase)positive.
Theyareusuallynon-hemolytic(gamma-hemolytic),butsome-times
mayshowalphaorbetahemolysis.

752.Heatingat60?Cfor30minutewould
isolate:
a)Staphylococci
b)Enterococci
c)Micrococci
d)Streptococci
CorrectAnswer-B
Enterococci

753.Whichofthefollowingbelongsto
Herpesviridae:
a)Variola
b)Adenovirus
c)HPV
d)RKvirus
CorrectAnswer-D
Ans.d.RKvirus

754.HIVenvelopisformedby:
a)Hostcell
b)Virus
c)Both
d)None
CorrectAnswer-C
Ans.c.Both

755.WhichHIV-virusismoredangerous:
a)HIV-1
b)HIV-2
c)Botharesame
d)Itdependsonhostfactors
CorrectAnswer-A
Ans.a.HIV-1

756.Mostcommonmodeoftransmissionof
HIVsexualtransmission:
a)Blood&bloodproducts
b)Occupational
c)Perinatal
d)Breastfeeding
CorrectAnswer-A
Ans.a.Blood&bloodproducts

757.Whichviralgeneactsascarcinogenin
causingcarcinomacervix-
a)P24-gene
b)E-gene
c)L-gene
d)H-gene
CorrectAnswer-B
Ans.is'b'i.e.,E-gene[RefHarrison150/ep.1199-1200]
HPVisthemostimportantcauseofcervicalcancer.
ProductsofE-genes(E6,E7)arerelatedtoimmortalizationor
malignanttransformationofkeratinocytesbyinterferingwithp53and
Rbgenes,respectively

758.Whichinfluenzaestrain,notofhuman
originandcancausepandemic:
a)HiNi
b)H2N2
c)H5Ni
d)H9Ni
CorrectAnswer-C
Ans.c.H5N1

759.Trueaboutinfluenzavaccine:
a)Livevaccineisusedmostcommonly
b)Livevaccineisgivenbynasaldrops
c)Killedvaccineisgivenintramuscularindeltoid
d)Allarecorrect
CorrectAnswer-B
Ans-B
Liveattenuatedvaccines-->Theseareusedasnasaldrops
Killedvaccine-->Itisusedmosfcommonly.Itisgiven
subcutaneouslyin2doses,3-4weeksapart.Protectiveefficacyis
70-90%anddurationofprotectionis3-6months.

760.Whichofthefollowingcaninfectovary-
a)Mumpsvirus
b)EBV
c)CMV
d)Measlesvirus
CorrectAnswer-A
Ans.is'a'i.e.,Mumpsvirus[RefClinicalmicrobiologyp.273]
Mumpsbeginsasaprimaryinfectionintherespiratorytractand
spreadsbyviremiatoglandulartissuesincludingthesalivaryglands,
pancreas,testesandovaries.

761.Whichisnotapoxvirus:
a)Vacciniavirus
b)Molluscumcontagiosum
c)Tanapoxvirus
d)Coxsackievirus
CorrectAnswer-D
Ans.d.Coxsackievirus

762.Integrationofviralgenomeintohostcell
chromosomecanleadsto-
a)Malignancy
b)Latency
c)Alteredgrowth
d)Alloftheabove
CorrectAnswer-D
Ans.is'd'i.e.,Alloftheabove[RefClinicalmicrobiologyp.312]
Integrationofviralgenomesintohostcellchromosomecancause
alterationin:
1. Hostcellsurface
2. Metabolicfunction
3. Cellgrowthandreplicationpattern
4. Malignanttransformation
5. Latentinfection

763.Nottrueaboutparamyxoviruses-
a)Belongtofamilymyxovirus
b)AreDNAviruses
c)Havelinearnucleicacid
d)Antigenicallystable
CorrectAnswer-B
Ans.is'b'i.e.,AreDNAviruses[RefAnanthanarayan9`h/ep.
497-498]
MyxovirusesareenvelopedRNAviruses.
Theyarecharacterizedbyabilitytoadsorbontomucoprotein
(affinityformucin)onerythrocytescausingagglutinationof
erythocytes.
Majorrouteofinfectionofallthesevirusisrespiratoryrouteby
dropletinfection.
Myxovirusaredividedintotwoimportantfamilies?
1. OrthomyxovirusInfluenza
2. ParamyxovirusMumps,measles,RSV,Parainfluenzavirus

764.HTLV-1canbetransmittedby-
a)Bloodtransfusion
b)Dropletinhalation
c)Contaminatedwater
d)Animalbite
CorrectAnswer-A
Ans.is'a'i.e.,Bloodtransfusion[RefManualofclinical
microbiologyVol.1p.1131]
HTLV-1andHTLV-2infectionsaretransmittedby?

1. Sexually(mainlybymalestofemale)
2. Vertically(frommothertochildbyprolongedbreastfeeding)
3. Parenterally(throughdruguseandbloodtransfusion)

765.Trueaboutrotavirusvaccine:
a)Killedvaccine
b)Givensubcutaneous
c)Pentavalentvaccine
d)Shouldbegivenbefore5years
CorrectAnswer-C
Ans.c.Pentavalentvaccine

766.VonMagnusphenomenon-
a)Isanormalreplicativecycle
b)Virusyieldhaslowhemagglutination
c)Virushashighinfectivity
d)Virusyieldshashighhemagglutinationtitrebutlowinfectivity
CorrectAnswer-D
Ans.is'd'i.e.virusyieldshashighhemagglutinationtitrebutlow
infectivity
Abnormalreplicativecyclesofviruses
.Vonmagnusphenomenonwhencellsareinfectedwithahighdose
ofinfluenzavirus,thereisdefectiveassemblyduringreplicationand
the
producedvirushashighhemagglutinintitrebutlowinfectivity.
.AbortiveinfectionDuetodefectivematurationorassembly,either
noreleaseofvironsoccurortheprogenyofvirusisnoninfectious.
.DefectivevirusesSomevirusesaregeneticallydefectiveand
requirehelpofsimultaneouslyinfectedhelpervirusforreplication.
eg.
HDV,Roussarcomavirus.

767.Trypanosomacruziistransmittedby:
a)Tsetsefly
b)Reduviidbug
c)Culexmosquito
d)Sandfly
CorrectAnswer-A
Ans.a.Tsetsefly

768.Duodenalaspirateisusedindiagnosis
of:
a)Ehistolytica
b)Giardialamblia
c)Taeniasolium
d)Leishmania
CorrectAnswer-B
Ans.b.Giardialamblia
DuodenalaspirateisusedforGlambia,osinesis,Fheptica,andS
stercoralis.

769.Amoebiasisisnottransmittedby:
a)Feco-oralroute
b)Sexualtransmission
c)Bloodandbloodproducts
d)Vectortransmission
CorrectAnswer-D
Ans.d.Vectortransmission

770.Recrudescencesarecommonlyseenin
whichmalaria:
a)Pvivax
b)Povale
c)Pmalariae
d)Pfalciparum
CorrectAnswer-D
Ans.d.Pfalciparum

771.Riverblindnessiscausedby-
a)Onchocerca
b)Loaloa
c)Ascaris
d)B.malayi
CorrectAnswer-A
Ans.is'a'i.e.,Onchocerca
OnchocerciasisisalsoknownasRiverblindnessandRobles
disease.

772.Whichofthefollowingcelltypesarethe
mostpotentactivatorofT-cell?
a)Bell
b)FollicularDendriticCells
c)Maturedendriticcells
d)Macrophages
CorrectAnswer-C
Ans.(c)MatureDendriticcellsRef.Robins8/e,p192,Harrison18/e,
p2657
"MaturedendriticcellsarethemostpotentactivatorofnaiveT-cell"
DendriticCells:
Bonemarrowderivedcells
Therearetwotypesofcellswithdendriticmorphology:
-Interdigitatingdendriticcells
-Folliculardendriticcells
Interdigitatingdendriticcellsorjustdendriticcellsarethemost
importantantigenpresentingcellsforinitiatingprimaryimmune
responseagainstproteinantigens.Thisisduetofollowingreasons:
-Thesecellsarelocatedattherightplacetocaptureantigens,
i.e.underepithelia,intheinterstitialofalltissue.
-Theyexpressvarietyofreceptors(includingTLR,mannose)for
capturingmicrobes.
-Inresponsetomicrobesdendriticcellsexpressthesame
chemokinereceptorsastonaiveT-cells.
-TheyexpresshighlevelsofMHCclassIImoleculesaswellas
co-stimulatorymoleculesB.7-1andB.7-2.Orin
otherwordstheypossessallthemachineryneededforpresenting
antigenstoandactivatingCD4+Tcells.

Ontheotherhandfolliculardendriticcells(doesnotarisefrombone
marrow)
arepresentinthegerminalcentresoflymphoidfollicles
wheretheytrapantigensboundtoantibodiesorcomplement.
Folliculardendriticcellsplaysaroleinongoingimmuneresponseby
presentingantigenstoB-cellsandselectingtheB-cellsthathavethe
highestaffinityfortheantigen.

773.Thymusdependentareainspleen-
a)Mantellayer
b)Perifollicularregion
c)Malphigiancorpuscle
d)Alloftheabove
CorrectAnswer-C
Ans.is'c'i.e.,Malphigiancorpuscle
Thymusdependent(T- Thymusindependent(B-
cellscollect)

cellscollection)
1.Spleen
1.Spleen
Periarteriallymphoid
Perifollicularregion
collect
Mantellayer
Malphigiancorpusclein 2.Lymphnode
whitepulp
Corticalfollicles
2.Lymphnode
Germinalcentres
Paracorticalarea
Medullarycords

774.Precipitationincomparisonto
agglutinationrequires-
a)LesspH
b)Hightemperature
c)Specificenzyme
d)Solubleantigen
CorrectAnswer-D
Ans.is'd'i.e.,Solubleantigen[RefAnanthanarayan9tVep.
105]merav


775.TrueaboutVDRLtest-
a)Non-specific
b)Slideflocculationtest
c)Bestfollowedfordrugtherapy
d)All
CorrectAnswer-D
Ansis'd'i.e.,All

776.Rosewaalertestis-
a)Complementfixationtest
b)Pricipitationingel
c)Ringprecipitation
d)Passivehemagglutinationtest
CorrectAnswer-D
Ans.is'd'i.e.,Passivehemogglutinationtest[Ref
Ananthanarayan8th/ep.107,108,Harrison18th/ep.1469,1470]


777.Weilfelixreactioninscrubtyphusis/are
positivefor:
a)OX-19
b)OX-2
c)BothOX-19&OX-2
d)OX-K
e)OX-19,OX-2&OX-K
CorrectAnswer-D
Ans:d.OX-K[RefAnanthanarayan9th/410;Medical
MicrobiologybyGreenwood16th/373]
Thisreactionisanagglutinationtestinwhichseraaretestedfor
agglutininstothe0antigensofcertainnon-motileProteusstrains
OX-19,OX-2&OX-X
Thebasisoftestisthesharingofanalkali-stablecarbohydrate
antigenbysomerickettsiae&bycertainstrainsofproteus,P.
vulgarisOX19&OX2&P.mirabilisOXK.
disease
OX-19
OX-2
OX-K
Epidemic +++
+
_
typhus
Brill-
Usually(?
Zinsser
ve)orweakly
-
disease
(+ve)
Endemic +++
+/-
-
typhus
Tickborne
spotted
++
++
-
fever

fever
Scrub
-
-
+++
typhus

778.ComplementFixationtestis:
September2005
a)VIDAL
b)Coombstest
c)Wassermannreaction
d)VDRL
CorrectAnswer-C
Ans.C:Wassermannreaction
Thecomplementfixationtest(CFT)wasextensivelyusedinsyphilis
serologyafterbeingintroducedbyWassermanin1906.However,
thereisnowatrendtoreplacetheCFTwiththesimpleflocculation
tests.
AlthoughCFTisconsideredtobearelativelysimpletest,itisavery
exactingprocedurebecause5variablesareinvolved.Inessencethe
testconsistsoftwoantigen-antibodyreactions,oneofwhichisthe
indicatorsystem.
Thefirstreaction,betweenaknownvirusantigenandaspecific
antibodytakesplaceinthepresenceofapredeterminedamountof
complement.Thecomplementisremovedor"fixed"bytheantigen-
antibodycomplex.
Thesecondantigen-antibodyreactionconsistsofreactingsheep
RBCwithhaemolysin.Whenthisindicatorsystemisaddedtothe
reactants,thesensitizedRBCswillonlylyseinthepresenceoffree
complement.TheantigensusedforCFTtendtobegroupantigens
ratherthantype-specificantigens.InorderfortheCFTtobesetup
correctly,theoptimalconcentrationofhaemolyticserum,
complement,andantigenshouldbedeterminedbytitration.
TheWassermanntestisnolongerinuse.


779.Exampleofprecipitationtestis
a)Rosewaalertest
b)Widaltest
c)Latexagglutination
d)Kahntest
CorrectAnswer-D
Ans.is'd'i.e.,Kahntest
ItisaHeterophiletubeagglutinationtest,usedtodiagnose
Rickettsiae.

780.Whichofthefollowingcomplement
factorsisamarkerofhumoralrejection?
a)C3d
b)C3b
c)C4d
d)C5a
CorrectAnswer-C
Ans:C.C4d
(Ref:Robbins9/ep234)
C4dfactorisamarkerforhumoralrejection.
Acuteantibody-mediatedrejectionismanifestedmainlybydamage
toglomeruliandsmallbloodvessels.
Typically,thelesionsconsistofinflammationofglomeruliand
peritubularcapillaries,associatedwithdepositionofthecomplement
breakdownproductC4d,whichisproducedduringactivationofthe
complementsystembytheantibody-dependentclassicalpathway.
Smallvesselsmayalsoshowfocalthrombosis.

781.PrimoryT-celldeficiencyis-
a)Ecto-5'nucleotidasedeficiency
b)Commonvariableimmunodeficiency
c)DiGeorgesyndrome
d)Wiskott-Aldrichsyndrome
CorrectAnswer-C
Ans.is'c'i.e.,DiGeorgesyndrome[Ref:Atlasofimmunologyp.
537]
I.Cellularimmunodeficiencies(Tcelldefects)

1. Thymichypoplasia(DiGeorgesyndrome)
2. Chronicmucocutaneouscandidiasis
3. Purinenucleosidephosphorylase(PNP)deficiency
II.Combinedimmunodeficiencies(BandTcelldefects)
1. Cellularimmunodeficiencywithabnormalimmunoglobulinsynthesis
(Nezelofsyndrome)
2. Ataxiatelangiectasia
3. Wiskott-Aldrichsyndrome
4. Immunodeficiencywiththymoma
5. Immunodeficiencywithshort-limbeddwarfism

782.Oakley-fulthorpeprocedureis-
a)Agglutinationtest
b)Precipitationtest
c)Singlediffusioninonedimension
d)Doublediffusioninonedimension
CorrectAnswer-B:D
Ans.is'd>b'i.e.,Doublediffusioninonedimension>
Precipitationtest[Ref:Ananthanarayan9th/ep.105]
Immunodiffusiontestsareprecipitationreactioningels.
Importantexamplesofimmunodiffusiontestare:-

1. Oudinprocedure->singlediffusioninonedimension.
2. Oakleyfulthorpeprocedure-,Doublediffusioninonedimension
3. Mancinimethodorradialimmunodiffusion->Singlediffusionintwo
dimensions.
4. Ochterlonyprocedure->Doublediffusionintwodimensions

783.MolecularmassofIgG[inKDa]
a)150
b)400
c)1000
d)1500
CorrectAnswer-A
Ans.is'a'i.e.,150
Antibody Molecularmass(KDa)
IgG
150
IgA
160,400
IgM
950,1150
IgD
175
IgE
190

784.Haptensareimmunogenicwhenthey
covalentlybindto-
a)Lipidcarrier
b)Polysaccharidecarrier
c)Proteincarrier
d)Anyoftheabovecarrier
CorrectAnswer-C
Ans.is'c'i.e.,Proteincarrier[RefEssentialsofmicrobiologyp.
91]
Hapten
Haptenisasubstance,whichitselfisunabletoinduceantibody
synthesis,i.enon-immunogenicbutmaybeabletoreactspecifically
withantibody.Thus,haptencanfunctionasantigenbutnotas
immunogen.
Haptensareincompleteantigens,whichbecomecompleteantigens
whentheycovalentlycombinewithcarriermoleculeorschleper.
Aftercombinationwithcarriermolecule,itbecomescomplete
antigenandcaninduceanimmuneresponse.

785.IgMappearsinfetusatwhatgestational
age-
a)10weeks
b)20weeks
c)30weeks
d)atbirth
CorrectAnswer-B
Ans.is'b'i.e.,20weeks
IgMistheearliestimmunoglobulinclasstobesynthesizedbythe
fetus,beginningat20weeksofage.

786.Theserumconcentrationofwhichofthe
followinghumanIgGsubclassis
maximum?

a)IgG1
b)IgG2
c)IgG3
d)IgG4
CorrectAnswer-A
Ans.is'a'i.e.,IgG1[RefAnanthanarayan9th/ep.97&8`"/ep.98;
Harrison19m/ep.372&18tVep.2674]


787.Exampleofneutrilizationreaction-
a)VDRL
b)Widaltest
c)Kahntest
d)Naglerreaction
CorrectAnswer-D
Ans.is'd'i.e.,Naglerreaction[RefAnanthanarayan9th/ep.112]
ImportantneutralizationtestsareNaglerreaction,dicktestand
schicktest.

788.Whichdoesnotstimulateactive
immunity-
a)Subclinicalinfection
b)Clinicalinfection
c)Vaccination
d)Transplacentalantibodyinnewborn
CorrectAnswer-D
Ans.is'd'i.e.,Transplacentalantibodyinnewborn[Ref
Ananthanarayan9th/ep.81-83]
Adaptive(acquired)immunityisoffollowingtypes:?
1)Activeimmunity
Itissocalledbecausehost'simmunesystemactivelyproduce
immunity.
Antigenicstimulusinducestheimmunesystemtoproduce
antibodiesorcellularimmuneresponse.
Activeimmunitymaybe:-
1. Natural:Duetoinfection(eitherclinicalorsubclinical/inapparent)
2. Artificial:Vaccination(immunization)
2)Passiveimmunity
Itissocalledbecauseantibodiesareproducedinanotherorganism
andthenreceivedpassivelybythehost.
Passiveimmunitymaybe:-
1. Natural:Transportofantibodiesacrosstheplacentafrommotherto
fetus.
2. Acquired:Administrationofimmunoglobulin/Antibody.

789.Whichisspecificforacquiredimmunity
?
a)Immunologicalmemory
b)Affectedbygeneticmakeup
c)Noantigenexposure
d)Alloftheabove
CorrectAnswer-A
Ans.is'a'i.e.,Immunologicalmemory[RefHarrison19th/ep.
372]
Acquired(adaptive)immunityisfoundonlyinvertebrates
ItisbasedonthegenerationofantigenreceptorsonTandB
lymphocytesbygermlinegenerearrangements.
Itisspecific.
Itisdevelopedasaresultofanantigenicstimulus(Immunological
priming)orbypassivetransferofantibodies.
Itprovidesimmunologicalmemory;subsequentantigenexposure
leadstomorerapidandvigorousimmuneresponses.
Theadaptiveimmunesystemconsistofduallimbsofcellularand
humoralimmunity.

790.Latticephenomenonisseenin-
a)Neutrilizationreaction
b)Complementfixationtest
c)Precipitationtest
d)Alloftheabove
CorrectAnswer-C
Ans.is'c'i.e.,Precipitationtest[RefTextbookofclinical
microbiologyp.785]
Thelatticehypothesiswasproposedbymarrack(1934)toexplain
themechanismofprecipitation.
Accordingtothisconcept,multivalentantigenscombinewithbivalent
antibodiesinvaryingproportiondependingontheantigen-antibody
ratiointhereactingmixture.
Precipitationresultswhenlargelatticeisformedconsistingof
alternatingantigenandantibodymolecules.Thisispossibleonlyin
thezoneofequivalence.
Inthezoneofantigenorantibodyexcess,thelatticedoesnot
enlarge,asthevalenciesoftheantibodyandtheantigen,repectively
arefullysatisfied.Ineithercasesextensivelatticecannotbeformed
andprecipitionisinhibited.
Thelatticehypothesisholdsgoodforagglutinationalso.

791.Complementdeficiencyhasnotbeen
implicatedincausing?
a)SLE
b)PNH
c)Hereditaryangiodema
d)Membranousnephritis
CorrectAnswer-D
Ans.is'd'i.e.,Membranousnephritis[RefPariza3'/ep.121]
Complementdeficiencyanddisease
Deficiencyofearlycomponents(C1,C2,C4)causesimmune-
complexdiseases(e.g.SLE),andpyogenicinfections.
DeficiencyofC3andC3bcausesrecurrentpyogenicinfections.
Terminalcomplementcomponent(C5throughCo)deficienciesand
deficienciesofthealternativepathway(Properdin,C3,FactorD)
haveastrongeffectonsusceptibilityto,aswellasseverityof,
neisserialinfections.
InheriteddeficiencyofC,esteraseinhibitorcasesHereditary
angioneuroticedema.
AcquireddeficiencyofDecayaccelaratingfactor(DAF)causes
Paroxysmalnocturnalhemoglobinuria(PNH).DAFisaregulatorin
complementsystemwhichincreasesdissolutionofC3convertase
(C4b2a)

792.Whichisnottrueaboutmacrophages:
a)ActivationbyIFN-y
b)Majorcellsinchronicinflammation
c)M2typeinvolvedininflammation
d)Phagocyticcells
CorrectAnswer-C
Ans.c.M2typeinvolvedininflammation

793.Prozonephenomenonisdueto
a)Antigenexcess
b)Antibodyexcess
c)False+vereaction
d)False-vereaction
CorrectAnswer-B:D
Ans.(b)and(d)AntibodyexcessandFalse-negativeRef.
Ananthanarayan8/e,p104,9/105
Zonephenomenon(seeninagglutinationandprecipitation)consists
of3parts:
1. Prozone=Abexcess=weakorabsentprecipitationreaction=
False-ve
2. Zoneofequivalence=peakamountofprecipitation.
3. Postzone=Agexcess=weakorabsentprecipitationreaction.

794.Monoclonalantibodybindsto?
a)Epitope
b)Paratope
c)Bothepitopeandparatope
d)Noneoftheabove
CorrectAnswer-A
Ans.is'a'i.e.,Epitope[Ref:Ananthanarayan9th/ep.88&71*p.
81;Harrison18thiep.2673&17th/ep.2036]
Epitope
Alsoknownas"antigenicdeterminant".
Itisantibodybindingsiteofantigen.

795.Whentransferfactorisgivenas
treatmentresultsin-
a)Naturalactiveimmunity
b)Artificialactiveimmunity
c)Artificialpassiveimmunity
d)Adoptiveimmunity
CorrectAnswer-D
Ans.is'd'i.e.,Adoptiveimmunity

796.IndirectCoomb'stestdetects:
a)AntibodiesattachedtoRBCSurface
b)Antibodiesintheserum
c)AntigensattachedtoRBCSurface
d)Antigensintheserum
CorrectAnswer-B
AnswerisB(Antibodiesintheserum)
IndirectCoomb'stestdetectsIgGantibodiesintheserum(e.g.Anti-
DAntibodies).DirectCoomb'stestdetectsIgGAntibodies(or
complements)attachedtothesurfaceofRBCs.
DirectAntihumanGlobulinTestDetectsRBCssensitizedwith
IgGorComplements(C3BorC3d)
(DAT;Coomb's;DirectCoomb's)DetectsIgGAntibodies(or
complements)attachedtothesurfaceofRBCs.
IndirectAntihumanGlobulinTest
DirectCoomb'sTest
IntheDirectCoomb'stest,redbloodcells(RBCs)sensitizedwith
IgGantibodies(orC3b,C3d)areagglutinatedwhenCoomb's
reagent(RabbitAnti-IgGantibody)isaddedtothetesttube
DetectsAntibodiesintheSerum
IndirectCoomb'sTestIntheIndirectCoomb'stestIgGantibodies
(e.g.Anti-D)intheserummustfirstbindtobloodgroupType0Test
RBCsaddedtothetesttube.AdditionofCoombsReagent,then
causesthesensitizedType0TestRBCstoagglutinate,indicating
thatIgGantibodiesarepresentintheserum.

797.Whichofthefollowingisnotaninvivo
test?
a)Elek'sgelprecipitationtest
b)Schicktest
c)Lepromintest
d)Tuberculintest
CorrectAnswer-A
Ans.is'a'i.e.,Elek'sgelprecipitationtest[RefReadbelow]
Elek'sgelprecipitationisaninvitrotestfortoxigenicityofC
diphtheriae.
Otherthreeoptionsareskintests(i.e.invivotests).

798.Whichofthefollowingcanbeusedfor
obtainingspecimenforisolationof
microorganisminlaboratorydiagnosis:

a)Meningococcalrash
b)Bloodinstaphylococcalfoodpoisoning
c)ThroatswabinRheumaticfever
d)Bloodinpost-streptococcalGN
CorrectAnswer-A
Ans.is.a'i.e.Meningococcalrash

799.Morulaformisseeninwhichinfection?
a)Chlamydiae
b)Bartonellaquintana
c)Mycoplasmahominis
d)Ehrlichia
CorrectAnswer-D
Ans.is'd'i.e.,Ehrlichia[RefAnanthanarayanhlep.409;
Vasanthakumarip.310]
Ehrlichiaegrawwithinphagosomesofphagocytesasmulberry-like
clusterscalledmorula.

800.Achildispresentingwithvomitingand
abdominalpainafter5hoursofeating
somefood.Themostlikelycausative
orgonism:

a)Bacilluscereus
b)Cl.perfringens
c)Cl.botalinum
d)V.cholerae
CorrectAnswer-A
Ans.is.'a'i.e.Bacilluscereus

801.Preventionofcatheterinducedurinary
tractinfectionisby?
a)Prophylacticantibiotics
b)Useoffacemask
c)Closeddrainagetechnique
d)Alloftheabove
CorrectAnswer-C
Ans.is'c'i.e.,Closeddrainagetechnique[RefMicrobiologyin
clinicalpracticep.450]
Preventionofcatheter-associatedinfectiondependson:

1. Asepticcathetertechniques.
2. Antisepticlubricantcontainingchlorhexidineandlocalinstillationof
1%chlorhexidineintothebladder.
3. Closeddrainagecathatertechnique.
4. Excellenthandwashingtechniques,usingchlorhexidinedetergent
andgooddryingofhands.
5. Useofglovesandisolationofpatientswithinfectionsduetomultiple
antibiotic-resistantstrains.

802.BubusformiswhichstageofLGV:
a)Prmary
b)Secondary
c)Tertiary
d)Latent
CorrectAnswer-B
Ans.is.'b'i.e.,Secondary

803.Nosocomialinfectionismostcommonly
causedby:
March2004

a)Gramnegativebacilli
b)Grampositivebacilli
c)Gramnegativecocci
d)Mycoplasma
CorrectAnswer-A
Ans.Ai.e.Gramnegativebacilli

804.ABOnon-secretorsaremoreproneto?
a)Infection
b)Autoimmunity
c)Heartdisease
d)Carcinoma
CorrectAnswer-A:B:C
Ans.is'a'i.e.,Infection,'b'i.e.,Autoimmunity&'c'i.e.,Heart
disease[Refwww.dadamo.com]
Therearetwotypesofpeople(independentofbloodgroup)
Secretor:Thesepersonssecretetheirbloodgroupantigenintoother
bodyfluidslikesalivaetc.
Nonseretor:Theydonotsecretetheirbloodgroupantigenintobody
fluids.
Non-secretorsappeartohaveanincreaseinprevalenceofavariety
ofautoimmunediseaselikeankylosingspondylitis,reactivearthritis,
GravesdiseaseandSjogren'ssyndrome.
Non-secretorsareatagreaterriskofdevelopingdiabetes,MIand
heartdisease.
Non-secretorsareatagreaterriskforrecurrenturinarytract
infectionsandcandidainfection.
Non-secretorshavemoreoraldiseasesandmoredigestive
problems.

805.InRidealwalkermethod,platesare
incubatedfor?
a)<2days
b)2-3days
c)6-8days
d)>10days
CorrectAnswer-B
Ans.is'b'i.e.,2-3days[RefTextbookofsterilizatione.7
disinfectionp.233]
InRidealwalkermethod,forcheckingefficiencyofdisinfectant,
bactericidalactivityisdeterminedagainstsalmonellatyphi
suspension.
Subculturesareperformedfromboththetestandphenolatintervals
of2.5,5,7.5and10minutes.
Theplatesareincubatedfor48-72hoursat37?C.

806.CD3isamarkerfor?
a)B-cells
b)T-cells
c)NK-cells
d)Monocytes
CorrectAnswer-B
Ans.is'b'i.e.,T-cells[Ref:Robbin's9th/ep.590&8`5/ep.600
table(13.5)]
CD-3isknownasPanT-cellmarker.

807.Commonvariabledeficiencyisdueto-
a)AbsentBcells
b)ReducednumberofBcells
c)DefectiveBcelldifferentiation
d)Alloftheabove
CorrectAnswer-C
Ans.is'c'i.e.,DefectiveBcelldifferentiation[Ref:Robbin's
9m/e
p.241&8th/ep.233]
Mostpatientswithcommonvariableimmunodeficiencyhavenormal
ornear-normalnumbersofBcellsinthebloodandlymphoidtissues.
TheseBcells,however,arenotabletodifferentiateintoplasma
cells.
Theclinicalmanifestationsarecausedbyantibodydeficiency.
Thefeaturecommontoallpatientsishypogammaglobulinemia,
generallyaffectingalltheantibodyclassesbutsometimesonlyIgG.

808.PerformanceofcomponentsofPQLIis
countedbetween
a)-1to+1
b)0to1
c)0to100
d)None
CorrectAnswer-C
Ans.is'c'i.e.,0to100
Thesubjectivecomponentofwellbeing-4Qualityoflife
While"levelofliving"isanobjectivecomponent,"qualityoflife"
comprisestheindividual'sownsubjectiveevaluation.
Theindexforqualityoflifeis"Physicalqualityoflifeindex(PQLI)".
ThePQLIisanattempttomeasurethequalityoflifeorwell-beingof
acountry.
Physicalqualityoflifeindexconsolidatesthreeindicators:?
1. Literacyrate
2. Infantmortalityrate
3. Lifeexpectancyatage1year(LE)
PQLIrangesfrom0to100.
Foreachcomponent,theperformanceofindividualcountriesis
placedonascaleof0to100,where0representsanabsolutely
definedworstperformanceand100representsanabsolutely
definedbestperformance.
Thecompositeindexiscalculatedbyaveragingthethreeindicators,
givingequalweighttoeachofthem.
TheresultingPQLIthusalsoisscaled0to100.

809.Humandevelopmentindexincludesall
except?
a)Longevity
b)Knowledge
c)Income
d)Literacyrate
CorrectAnswer-D
Ans.is'd'i.e.,Literacyrate
Humandevelopmentindex
HDIisacompositeindexcombiningindicatorsrepresentingthree
dimensions.
i. Longevity:Lifeexpectancyatbirth.
ii. Knowledge:Meanyearsofschooling(grossenrolmentratio)and
expectedyearofschooling.(InoldereditionsofPark,i.e.,21st/eand
olderthanthat,itwasadultliteracyrateinsteadofexpectedyear
schooling).
iii. Income:GNIPerCapita(InoldereditionsofPark,itwasGDPper
capitainsteadofGNIpercapita).

810.Human,animal,fomiteorobjectsfrom
whichinfectiveorganismentersthehost
iscalled?

a)Source
b)Reservoir
c)Carrier
d)None
CorrectAnswer-A
Ans.is'a'i.e.,Source[Ref:Park23rdlep.92-97]
Sourceis'theperson,animal,objectorsubstancefromwhich
infectiousagentpassestohost',i.e.manacquiresinfectionfrom
source.
Reservoiris'anyperson,animal,insect,plant,soilorsubstancein
whichaninfectiousagentlivesandmultiplies'.Infectiousagentis
dependentonreservoirforsurvival.Fromreservoiritcanbe
transmittedtosusceptiblehost.Thusareservoirmayactasa
sourceofinfectionwhenapersonacquiresinfectiondirectlyfroma
reservoir.

811.Reservoirisdefinedas?
a)Person,animalorobjectfromwhichinfectiousagentis
transmittedtohost
b)Person,animalorsubstanceinwhichinfectiousagentlivesand
multiplies
c)Personoranimalinwhichinfectiousagentcausesadisease
d)Noneoftheabove
CorrectAnswer-B
Ans.is'b'i.e.,Person,animalorsubstanceinwhichinfectious
agentlivesandmultiplies.


812.Measlesisinfectivefor:
March2009
a)Onedaybeforeand4daysafterrash
b)Fourdaysbeforeandfivedaysafterrash
c)Entireincubationperiod
d)Onlyduringscabsfalling
CorrectAnswer-B
Ans.B:Fourdaysbeforeandfivedaysafterrash
Measles/EnglishMeaslesisspreadthroughrespiration(contactwith
fluidsfromaninfectedperson'snoseandmouth,eitherdirectlyor
throughaerosoltransmission),andishighlycontagious--90%of
peoplewithoutimmunitysharingahousewithaninfectedpersonwill
catchit.
Theinfectionhasanaverageincubationperiodof14days(range6-
19days)andPeriodofcommunicabilityinmeaslesisapproximately
4daysbeforeand5daysaftertheappearanceoftherash.
Measlesisaninfectionoftherespiratorysystemcausedbyavirus,
specificallyaparamyxovirusofthegenusMorbillivirus.
Morbilliviruses,likeotherparamyxoviruses,areenveloped,single-
stranded,negative-senseRNAviruses.Symptomsincludefever,
cough,runnynose,redeyesandageneralized,maculopapular,
erythematousrash.

813.NotincludedinBradfordHill'scriteria?
a)Strengthofassociation
b)Consistencyofassociation
c)Specificityofassociation
d)Lackoftemporalassociation
CorrectAnswer-D
Ans.is'd'i.e.,Lackoftemporalassociation[RefPark23"/ep.
88-89]
Itisthepresenceoftemporalassociation(notlackorabsence)that
formsacriteriaforcausalrelationship.
Criteriathatsuggestcausalrelationship(BradfordHill's
criteria)
Likehoodofacausalrelationshipisincreasedbythepresenceofthe
followingcriteria:?
1. Temporalassociation
2. Strengthofassociation
3. Specificityofassociation
4. Consistencyofassociation
5. Biologicalplausibility
6. Coherenceofassociation
Hill'scriteria(sometimesalsoknownas'Surgeongeneral'sCriteria'
ofcausalassociation):inepidemiologyareAnalogoustoKoch's
Postulates(ofcausalassociationbetweenamicrobeanddisease)in
microbiology.

814.Mostimportantcriterianincausal
relationshiphypothesis?
a)Temporalassociation
b)Coherenceofassociation
c)Specificityofassociation
d)Strengthofassociation
CorrectAnswer-A
Ans.is'a'i.e.,Temporalassociation[RefPark23rd/ep.89]
Mostimportantcriteriatemporalassociation
Weakestandmostdifficultcriteriatoestablishspecificityof
association

815.
Theweakestcriterianincausal
relatianshiphypothesis?
a)Temporalassociation
b)Coherenceofassociation
c)Specificityofassociation
d)Strengthofassociation
CorrectAnswer-C
Ans.is'c'i.e.,Specificityofassociation

816.Benefitofscreeningis?
a)Preventionofdisease/cancer
b)Earlytreatmentofdisease
c)Providerehabilitation
d)Diagnosingallthemissingcases
CorrectAnswer-B
Ans.is'b'i.e.,Earlytreatmentofdisease
Screening
istheactivesearchforthediseaseamongapparently
healthypeople.
Screeningintendstoidentifyhealthypersonthoselikelytohavea
diseaseoratincreasedriskofadisease
understudy,thusenablingearlierinterventionandmanagementin
thehopetoreducemortalityandsufferingfromadisease.
ScreeningisatypeofsecondarypreventionEarlydiagnosis,
treatment

817.Multiphasicscreeningmeans-
a)Applicationofthetwoormorescreeningtestsincombinationat
onetime
b)Applicationoftwoormorescreeningtestsincombinationat
differenttime
c)Applicationoftwoormorescreeningtestsincombinationat
differentgeographicalareas
d)Applicationofseparatescreeningtestsfordifferentdiseases
CorrectAnswer-A
Ans.is'A'i.e.,Applicationofthetwoormorescreeningtestsin
combinationatonetime
Multiphasicscreening
Ithasdefinedtheapplicationoftwoormorescreeningtestsin
combinationwithalargenumberofpeopleatonetimethantocarry
outseparatescreeningtestsforasingledisease.
Itincreasesthecostofhealthserviceswithoutanyobservable
benefit.Example-AnnualHealthCheckups
High-riskorselectivescreeningisdoneonhigh-riskgroups
example-MRIscreeningofbreastdoneannuallyin25-yearold
womenwithBRCA1,BRCA2mutationsinfirst-degreerelative

818.Randomizationisdonetoreduce?
a)Recallbias
b)Selectionbias
c)Berksonianbias
d)Reportingbias
CorrectAnswer-B
Ans.is'b'i.e.,Selectionbias

819.BenefitofRCT?
a)Fasterstudy
b)Cheaperstudy
c)Noselectionbias
d)Suitableforraredisease
CorrectAnswer-C
Ans.is'c'i.e.,Noselectionbias[RefPark23rdlep.81-83]
RandomizationistheheartofRCT.Randomizationisastatistical
procedurebywhichtheparticipantsareallocatedintostudygroup
(inwhichinterventionisgiven)andcontrolgroup/referencegroup(in
whichinterventionisnotgiven).
Itisworthnotingthatrandomisationisdonewhiledividingthe
participantsintostudygroupandcontrol(reference)group,andnot
whileselectingsubjectsforstudy,i.e.randomizationisdoneafter
thesampleofsubjectshasalreadybeenselected.Therefor,each
participanthas'equalandknownchance'offallingintoeitherstudy
grouporcontrolgroup.
Randomizationisanattempttoeliminatebiasandallow
comparability.Itwillgivethegreatestconfidencethatthegroupsare
comparablesothat"like"canbecomparedwith"like".Itensures
thattheinvestigatorhasnocontroloverallocationofparticipantsto
eitherstudyorcontrolgroup,thuseliminatingwhatisknownas
selectionbias.

820.Mostcommonlyusedblindingtechnique
inepidemiologicalstudies?
a)Singleblinding
b)Doubleblinding
c)Tripleblinding
d)Noneoftheabove
CorrectAnswer-B:C
Ans.is'b>c'i.e.,Doubleblinding>Tripleblinding[Ref
Statisticsandepidemiology3'/ep.46]
Mostcommonlyusedblindingisdoubleblinding,i.e.mostofthe
epidemiologicalstudiesarestartedasdoubleblinded.Butmore
aftenthannotitbecomestripleblindedasmorethantwopeoplein
thestudyareblindedforexamplethestatician(analyzer)
performingtheanalysisisoftenblindedinadditiontodoctor
(investigator)andpatient(studysubject).

821.Mosteffectiveblindingtechnique?
a)Singleblinding
b)Doubleblinding
c)Tripleblinding
d)Anyoftheabove
CorrectAnswer-C
Ans.is'c'i.e.,Tripleblinding[RefReadbelow]
MostcommonlyuseblindingDoubleblinding
BestblindingtechniquetoeliminatebiasTripleblinding

822. WhichofoneofthefollowingisNOTa
utilizationrate?
a)Populationbedratio
b)Bedoccupancyrate
c)Bedturnoverratio
d)Averagelengthofstay
CorrectAnswer-A
Ans.is'a'i.e.,Populationbedratio[RefPark23rd/ep.26&22"d/e
p.25]

1. Proportionofinfantswhoare"fullyimmunized"againstthe6EPI
diseases.
2. Proportionofpregnantwomenwhoreceiveantenatalcare,orhave
theirdeliveriessupervisedbyatrainedbirthattendant.
3. Percentageofpopulationusingvariousmethodsoffamilyplanning.
4. Bed-occupancyrate
5. Averagelengthofstay
6. Bedturn-overratio(i.e.discharges/average

823.CalculateIMRifinapopulationof100000
thereare3000livebirthsinayearand
150infantdeathsinthesameyear-

a)75
b)18
c)5
d)50
CorrectAnswer-D
Ans.is`d'i.e.,50

824.Trueaboutcombinedprospective-
retrospectivestudytrueis?
a)Exposure(+)nt,disease(+)nt
b)Exposure(+)nt,disease(-)nt
c)Exposure(-)nt,disease(+)nt
d)Exposure(-)nt,disease(-)nt
CorrectAnswer-A
Ans.is'a'i.e.,Exposure(+)nt,disease(+)nt[RefPark23"'/ep.
79-85]
Therearefollowingtypesofcohortstudy:?
A)Prospectivecohortstudy
Outcomehasnotyetoccurredwhenthestudyhasbegun:Only
exposurehasoccurred;welookfordevelopmentofsamediseasein
bothexposedandnon-exposedgroups
B)Retrospectivecohortstudy
Bothexposureaswellasoutcomehaveoccurredwhenthestudy
hasbegun:Firstwegobackintimeandtakeonlyexposureinto
consideration(cohortsidentifiedfrompasthospital/collegerecords),
thenlookfordevelopmentofsamediseaseinbothexposedand
non-exposedgroups
C)Combinedprospective-retrospectivecohortstudy
Bothexposureaswellasoutcomehaveoccurredwhenthestudy
hasbegun:Firstwegobackintimeandtakeonlyexposureinto
consideration(cohortsidentifiedfrompasthospital/collegerecords),
thenlookfordevelopmentofsamediseaseinbothexposedand
non-exposedgroups;latercohortisfollowedprospectivelyinto
futureforoutcome.

825.Proportionalmortalityrateis?
a)Numberofdeathduetoaparticularcause
b)Numberofdeathduringthatyear
c)Numberofdeathinonemonth
d)None
CorrectAnswer-A
Ans.is'a'i.e.,Numberofdeathduetoaparticularcause
Proportionalmortalityrate(ratio)
Proportionalmortalityratemeasurestheproportionoftotaldeath
duetospecificcauseorproportionofdeathsinaparticularage
group.
Itisdefinedas"numberofdeathsduetoaparticularcause(orin
specificagegroup)per100totaldeaths".
Itisthe'simplestmeasureofestimatingtheburdenofdisears'inthe
community.
Itisauseful'healthStatusindicator';indicatesmagnitudeof
preventablemortality.
Itisusedwhenpopulationdataisnotavailable.
Itdoesnotindicatetheriskofmembersofpopulationcontractingor
dyingfromthedisease.

826.Populationattributableriskisdefinedas
thedifferencebetween:
a)Incidenceinexposedandincidenceinnon-exposedcompared
withincidenceinnon-exposed
b)Incidenceinpopulationandincidenceinexposedcompared
withincidenceinpopulation
c)Incidenceinpopulationandincidenceinnon-exposed
comparedwithincidenceinpopulation
d)Incidenceinpopulationandincidenceinexposedcompared
withincidenceinnon-exposed
CorrectAnswer-C
Ans.c.Incidenceinpopulationandincidenceinnon-exposed
comparedwithincidenceinpopulation
populationattributableriskisdefinedasthedifferencebetween
incidenceinpopulationandincidenceinnon-exposedcompared
withincidenceinpopulation.

827.Slectionbiasoccursduring?
a)Recruitment
b)Treatment
c)Analysis
d)Observation
CorrectAnswer-A
Ans.is'a'i.e.,Recruitment[Ref:Essentialsofepidemiologyin
PublicHealthp.270]
Selectionbiasesaredistortionthatresultfromprocedureusedto
selectsubjectsandfromfactorsthatinfluencestudyparticipation.
Groupstobecomparedaredifferentiallysusceptibletotheoutcome
evenbeforetheexperimentalmaneuverisperformed.
SelectionbiasusuallyoccursattheStageofrecruitmentof
participants.
Selectionbiasesislesslikelytooccurinacohortstudycomparedto
case-controlorcross-sectionalstudybecausestudyparticipantare
selectedbeforetheoutcomeoccurs.
Randomizedcontroltrialsdonothaveselectionbiasas
randomizationeliminatesselection(investigator)bias.

828.Selectionbiasoccursmainlyin
a)Cohortstudy
b)Case-controlstudy
c)RCT
d)Allhaveequalchances
CorrectAnswer-B
Ans.is'b'i.e.,Case-controlstudy
Case-controlstudy>cross-sectionalstudy>Retrospectivecohort
study>Prospectivecohortstudy>RCT
Selectionbias:Casesandcontrolsmaynotberepresentativeofthe
populationortheremaybesystematicdifferencesbetweenboth.

829.Beststudytechniquetostudythe
occurenceofrareadverseeffectsofdrug
?

a)Case-controlstudy
b)Cohortstudy
c)Clinicaltrial/experimentalstudy
d)Cross-sectionalstudy
CorrectAnswer-C
Ans.is'c'i.e.,Clinicaltrial/experimentalstudy[RefClinical
Researchformedicine2009/ep.62]
Experimentalepidemiologyisalsocalledtrial.Broadlyspeaking,a
trialreferstoputtingsomethingtoatest.Thisallowsthetermtobe
usedinreferencetoatestofatreatmentforthesickoratestofa
preventivemeasureintendedtoavertillness,injuryordisease.
Therefore,thedefiningfeatureofanexperimentalstudyisitsability
toallocateorassigninterventionsortreatmenttoexperimentunit.
Insimplewords,thestudyofatreatment(Drugs,surgical
intervention)orpreventivemeasure(e.g.vaccination)onliving
subjectsisknownasexperimentalstudyortrial.

830.Bladdercancercanoccurinthosewho
areworkinginchimneyfor25years.
Whichisthebeststudyforthis
relationship?

a)Meta-analysis
b)Cross-sectionalstudy
c)Randomizedcontroltrialwithdoubleblinding
d)Cohortstudy
CorrectAnswer-A
Ans.is'a'i.e.,Meta-analysis
Asasinglestudyunit,doubleblindRCTisthebest:
"Theefficacyofnewinterventionsaremostreadilyacceptedifthe
resultsarefromrandomizedcontroledtrial"
PeerReview&Biomedicalpublication
However,overallmeta-analysisisabetterstudysinceitcombines
thedatafrommultipleRCTandalsofromothertypesofstudy.
"Randomizedcontrolledtrials(RCT)providethestongest,most
relevantevidencetoinformpractice.Someevidencehierarchies
placesystematicreviewandmeta-analysisaboveRCTssincethese
oftencombinedatafrommultipleRCTs,andpossiblyfromother
studytypeaswell"?Epidemiologyataglance
So,systematicreviewandmeta-analysisofRCTsarethebest
epidemiologicalstudies.

831.Whichisanexampleofcasecontrol
study?
a)Thalidomideandteratogenicity
b)Framinghamheartstudy
c)PVCandangiosarcomaofliver
d)Doll&HillStudy
CorrectAnswer-A
Ans.is'a'i.e.,Thalidomideandteratogenicity[Ref:Park
23rd/e
p.74,75]

832.Advantageofcase-controloveracohort
study?
a)Attributableriskcanbecalculated
b)Odd'sratiocanbecalculated
c)Forraredisease
d)bandc
CorrectAnswer-D
Ans.is'b'i.e.,Odd'sratiocanbecalculated&ci.e.,Forraredisease
oAcohortstudyismorereliablethanacase-controlstudyforan
associationbetweenasuspectedriskfactorandsubsequent
diseasebecauserelativeriskcanbeestimatedbycohortstudy,
whilecase-controlstudymeasuresonlyanestimateofrelativerisk
(oddsratio).
WhytheCase-ControlStudyissuitableforararediseasebutnot
CohortStudy?
oInthecohortstudy,weproceedfromeffecttocauseandifthe
studyisfortherarediseasewemaygetveryfewcasesornocase
attheendofthestudy.Forexample,ifararediseasehasan
incidencerate.01per1000(1per100000)populationandwetakea
sampleof100peopletoexposetheriskfactor,therewillbeveryfew
casesattheendofthestudyasthediseaseisveryrareandhasa
lowincidenceof1per100000population.(youcanexpect,Howlow
willbetheincidenceinasampleof100people).
oOntheotherhand,inthecase-controlstudy,wecanchoose
controlsforthefewavailablecasesandthehistoryof
possible/suspectedexposure(s)canbeexplored.

833.Immunitystartsafterhowmanydaysof
yellowfevervaccination?
a)7-10days
b)2-3weeks
c)4-5weeks
d)2-3months
CorrectAnswer-A
Ans.is'a'i.e.,7-10days[RefPark23rd/ep.283]
Yellowfevervaccine
Itisaliveattenuatedfreezdried(lyophilized)vaccine,preparedfrom
17Dstrain.
Itisgivenbysubcutaneousrouteatinsertionofdeltoid.
Immunitylastsfrom7daysofvaccinationtill35years.Thevalidityof
thevaccinationcertificatebegins10daysafterthedateof
vaccinationandextendsupto10years.
Diluentusedforreconstitutioniscoldphysiologicalsalineand
reconstitutedvaccineshouldbeusedwithin30minutes.Coldchain
temperatureforstorageis-30?to+5?C.
Yellowfevervaccineistheonlylivevaccinethatcanbegivenin
pregnancy,ifrequired.Yellowfeverandcholeravaccinecannotbe
giventogether,aminimumgapof3weeksisrequiredbetweenthe
two.

834.WhichDipheriavaccineisrecommended
ina14yearsoldgirl?
a)DPT
b)DT
c)Tdap
d)None
CorrectAnswer-C
Ans.is'c'i.e.,Tdap[Ref:www.tapcoi.com]
AccordingtoIAP(IndianAcademyofPaediatricians)Tdapis
recommendedforadolescents(10-18years).(NoteTdapcontainsa
lowerconcentrationofdiphtheriaandpertusistoxoidsthanDtaP)
Note:
DPTDiphtheria,Cellularpertussis,Tetanu,usedbefore7years
DtaPDiphtheria,Tetanus,acellularpertussis
TdapTetanus,diphtheria(lowdose),acellularpertussis(low
dose)->usedinadolescents.

835.Dropletnucleiisatypeof?
a)Verticaltransmission
b)Directtransmission
c)Indirecttransmission
d)Biologicaltransmission
CorrectAnswer-B
Ans.is'b'i.e.,Directtransmission[RefPark23"1/ep.97-100,768]
Communicablediseasemaybetransmittedfromthereservoiror
sourceofinfectiontoasusceptiblehostinmanydifferentways.
Modesoftransmissionmaybe:?
1. Directtransmission:Directcontact,dropletinfection,contactwith
soil,inoculationintoskinormucosa,verticaltransmission(through
placenta).
2. Indirecttransmission:Vehicle-borne,vector-borne,air-borne,
fomite-borne,byuncleanedhand&fingers.

836.Whichisnotadirecttransmission?
a)Dropletinfection
b)Verticaltransmission
c)Transmissionbymosquito
d)Soilcontact
CorrectAnswer-C
Ans.is'c'i.e.,Transmissionbymosquito

837.AftertakingMMRlivevaccine,
conceptionshouldnotoccurwithin?
a)2weeks
b)4weeks
c)8weeks
d)10weeks
CorrectAnswer-B
Ans.is'b'i.e.,4weeks[RefCDCguidlinesofvaccinationin
pregnancy]
Measles-mumps-rubella(MMR)vaccineanditscomponentvaccines
shouldnotbeadministeredtowomenknowntobepregnant.
Becausearisktobefetusfromadministrationoftheselivevirus
vaccinescannotbeexcludedfortheoreticalreasons,womenshould
becounseledtoavoidbecomingpregnantfor28daysafter
vaccination
withMMRvaccineoritscomponentvaccinesorvaricella
vaccine.

838.Disinfectionofurineiswhichtypeof
disinfection?
a)Precurrent
b)Concurrent
c)Preconcurrent
d)Terminal
CorrectAnswer-B
Ans.is'b'i.e.,Concurrent[RefPark23"1/ep.127]
Typesofdisinfection
Therearefollowingtypesofdisinfection:?
1)Precurrent(prophylactic)disinfection
Itisdoneasapreventivemethodbeforeillness,i.e.personisnotill,
forexamplechlorinationofwater,pasteurizationofmilk,and
handwashing.
2)Concurrentdisinfection
Itisdonewhenpersonisill.Itistheapplicationofdisinfective
measuresassoonaspossibleafterthedischargeofinfectious
materialfromthebodyofpatientorafterthesoilingofarticleswith
suchmaterial,i.e.,thediseaseagentisdestroyedassoonasitis
releasedfromthebody,andinthiswayfurtherspreadoftheagent
isstopped.e.g.,disinfectionofurine,faeces,vomit,contaminated
linen,clothes,hands,dressing,aprons,glovesetc.
3)Terminaldisinfection
Itistheapplicationofdisinfectivemeasuresafterthepatienthas
takendischargefromhospitalorhe/shehasdied.e.g.,disinfection
ofhospitalrooms&floor,burningorburialofsoiledmaterial.

839.Notafreezedriedvaccine?
a)OPV
b)Measles
c)DPT
d)Rubella
CorrectAnswer-C
Ans.is'c'i.e.,DPT[Ref:Park23"/ep.109&22"d/ep.104]
Vaccinewhichmustbestoredinthecoldpartbutneverallowedto
freez.
Typhoid DPT TT
HepatitisB
DT
BCG Diluents

840.Mostwidelyusedvaccine,besideOPV?
a)BCG
b)TT
c)Influenza
d)Pneumococcal
CorrectAnswer-A
Ans.is'a'i.e.,BCG[Refwww.ncbi.nlm.mih.]
"Makingwideruseoftheworld'smostwidelyusedvaccine:Bacille
calmette-Guerinerevaccinationreconsidered"
"Thebacillecalmette-Guerin(BCG)vaccinehasexistedfor80years
andisoneofthemostwidelyusedofallcurrentvaccines,reading>
80%ofneonatesandinfantsincounterieswhereitispartofthe
nationalchildhoodimmunizationprogramme

841.Trueaboutchickenpox?
a)CausedbyHerpessimplextype-7
b)SARis90%
c)Infectiousperiosis7dayspriorto7daysafteronsetofrash
d)Affectscommonly10-15yearsold
CorrectAnswer-B
Ans.is'b'i.e.,SARis90%[RefPark23"/ep.144]
ThecausativeagentofchickenpoxisVaricella-Zostervirus
(HerpessimplextypeIII).
Itisanacuterespiratoryinfectionwithincubationperiod10-21days.
Infectionisacquiredthroughrespiratorytractviaairdropletsor
rarelyfromconjunctiva.
Infectiousperiod(communicableperiod)forchickenpoxis2
dayspriorto5daysafteronsetofrash,
withaveryhigh
secondaryattackrateof90%.
Chicken-poxusuallyaffectschildrenofagegroup5-9years.

842.Chickenpoxrashdoesnotinvolve?
a)Trunk
b)Axilla
c)Palms&soles
d)Back
CorrectAnswer-C
Ans.is'c'i.e.,Palms&soles[RefPark23'/ep.144]
Rashofchickenpox:Superficial,unilocular,centripetal,
pleomorphic,symmetrical,affectsflexorsurfacesandaxilla,spares
palmsandsales,hasinflammationaround,rapidevolutionanddew-
droponrosepetalappearnace.

843.95%carrierand5%casesareseenin?
a)Measles
b)Diphtheria
c)Rabies
d)HepatitisB
CorrectAnswer-B
Ans.is'b'i.e.,Diphtheria[Ref:Park23rd/ep.160]
DiphtheriaiscausedbyCorynebacteriumdiphtheriae,agram-
positivebacterium.
Sourceofinfectioniseithercaseorcarrier,withcarriersbeingthe
mostcommonsourcesofinfection,theirratioisestimatedto
be95carriersfor5clinicalcases.Nasalcarriersaremore
dangerousthanthroatcarriers.Inmunuzationdoesnotprevent
carrierstate.


844.FollowingisHibconjugatevaccine?
a)Capsularpolysaccharide
b)Cellwallpolysaccheride
c)Capsularpolysaccheridewithcarrier
d)PRPwithcarrier
CorrectAnswer-C:D
Ans.is'd>c'i.e.,PRPwithcarrier>Capsularpolysaccheride
withcarrier[RefPariza4th/ep.3401
ConjugatedHibvaccine
includePRP(polyribosyl
ribitolphasphate)covalentlylinkedtocarrierprotein.PRPisthe
Capsularpolysaccaride
ofHinfluenzaetypeB(Hib).
CurrentlythreetypesofHib(HinfluenzaetypeB)vaccinesare
available.Thesevaryin?

1. Proteincarrierused
2. Themolecularsizeofsaccharide
3. Mehtodofconjugationofproteintosaccharide
Thesevaccinesare-
1. HbOC(mutantdiphtheriatoxinasthecarrierprotein)
2. PRP-T(Tetanustoxoidasthecarrierprotein)
3. PRP-OMP(majorOMPofNmeningitidisserogroupBascarrier)

845.TrueaboutPertussisis?
a)Mostoftheinfectionsaresubclinical
b)MostinfectivestageisParoxysmalstage
c)DrugofchoiceisErythromycin
d)Cerebellarataxiamaybeacomplication
CorrectAnswer-C
Ans.is'c'i.e.,DrugofchoiceisErythromycin[RefPark23'/e
p.161]
Pertussis(Whoopingcough)
Pertussis,alsocalled'100daycough',iscausedbyBordetella
pertussis(only5%casesarecausedbyB.parapertussis).
Sourceofinfectionisacaseofpertussis.Thereisnosubclinical
caseorchroniccarrierstate.
Periodofinfectivity(communicability)extendsfromaweekafter
exposusetoabout3weeksaftertheonsetofparoxysmal
stage.Catarrhalstageismostinfective.Secondaryattackrate
ishigh,i.e.90%.
Fordiagnosis,goldstandardisisolationoforganisminculturefrom
nasopharyngealsecretion.
Erythromycinisthedrugofchoicefortreatmentofcasesas
wellasforcontacts.Isolationperiod
is4weeksoruntil
paroxysmscease.

846.Trueaboutinfluenzainfectivity?
a)Communicableperiodis5daysbeforeto5daysaftertheonset
ofsymptoms
b)Sourceofinfectionisclinicalcase
c)Therearenosubclinicalcases
d)Allarecorrect
CorrectAnswer-B
Ans.is'b'i.e.,Sourceofinfectionisclinicalcase[RefPark
23rdiep.154]
Influenza
InfluenzavirusaRNAvirus,belongstoorthomyxovirus.
Sourceofinfectionofinfluenzaisaclinicalcaseorsubclinicalcase.
Majorreservoirofinfluenzavirusexistsinanimalandbirds.
Incubationperiodis18-72hours.Mostoftheinfectionsare
subclinical.Clinicalcasespresentwithcough,fever,myalgiaand
headache.
Complicationsincludepneumonia,encephalitis,Reye'ssyndrome
(withtype-Bvirus);GBsyndromeandgastricflu/GITsymptoms(with
type-Bvirus).
PeriodofCommunicabilityis1-2daysbeforeto1-2daysafteronset
ofsymptoms.

847.WhatiscommoninH5N1andH7N7
strainsofinfluenza?
a)Frequentendemicinfectioninman
b)Havesamefrequencyofantigenicvariation
c)Strainsofavianinfluenza
d)Allarecorrect
CorrectAnswer-C
Ans.is'c'i.e.,Strainsofavianinfluenza[RefEssentialsof
microbiologyp.701;Harrison18th/ep.1494;Park23rd/ep.153]
Avianinfluenza(Birdflu)
ItiscausedmostlybyH5N,strainofinfluenza-A.Itwasdetectedin
Hongkongin1997duringapandemicinpoultry.Itisapandemic
withhighmortalityrate(60%).
Othertypesofinfluenzaviruseswhichhavebeenobservedtocause
avianinfluenzaareH7N7andH9N2ofinfluenza-A.
Recentlyanoutbreakofavianinfluenzawascausedby112INT,in
China,in2013.
Drugofchoiceforavianinfluenzaisoseltamivir.

848.Notusedfortreatmentand/or
prophylaxisofseasonalinfluenza-
a)Amantidine
b)Rimantidine
c)Oseltamivir
d)Acyclovir
CorrectAnswer-D
Ans.is'd'i.e.,Acyclovir
Twoclassesofantiviraldrugsareavailableforthetreatmentand
preventionofinfluenza.
1. Neuraminidaseinhibitors:Zanamivir,Oseltamivir,peramivir
2. Adamantanes:Amantidine,rimantidine

849.Capsularpolysaccharidederivedvaccine
isavailableforallmeningocciexcept?
a)GroupA
b)GroupB
c)GroupC
d)GroupY
CorrectAnswer-B
Ans.is'b'i.e.,GroupB[RefPark23rdiep.166]
MeningococcalvaccineispreparedfromCapsularpolysaccharide.
VaccinesareavailabeforgroupA,C,YandW-135.Bivalent(A,C),
trivalent(A,C,W135)andtetravalent(A,C,Y,W135)vaccinesare
available.
MeningoccalgroupBvaccinewasnotavailableforlasttwo
decades.Recently,groupBvaccinehasalsobeendevelopedin
manycountries(UK,Ireland)andisapartofroutineimmunization
schedule.
Now,youmustbethinkingthatwhytheanswerofthisquestionis
groupBmeningococci.Thisisbecausemeningococcalvaccineis
notpreparedfromcapsularpolysaccharide.

850.Vaccinederivedpoliovirusoutbreaks
aredueto?
a)Type-2virus
b)Type-3virus
c)Type-1virus
d)Alloftheabove
CorrectAnswer-A
Ans.is'a'i.e.,Type-2virus
"Themaincauseofvaccinederivedpoliovirus(VDPV)outbreaksis
currentlytype-2componentofOPV"...Park23rd/e
"Currently,thetype-2componentcontainedintrivalentOPV
accountsformorethan90%ofallCVDPVcases".

851.Falseregardingpoliovaccination?
a)Bothkilledandlivevaccinesareavailable
b)FirstOPVisgivenat4weeks
c)OPVinducesbothhumoralandintestinalimmunity
d)IPVisgivenintramuscularly
CorrectAnswer-B
Ans.is'b'i.e.,FirstOPVisgivenat4weeks[RefPark23rd/e
p.206-210]
FirstOPV(zerodose)isgivenatbirth.
Therearetwopoliovaccines:killed(IPV)andliveattenuated(OPV)
OPVinducesbothhumoralandintestinalimmunity,whereasIPV
inducesonlyhumoralimmunity.
IPVisgivenintramuscularly(preferrable)orsubcutaneously.

852.Reservoirofinfectioninentericfever?
a)Birds
b)Cow
c)Cattle
d)Man
CorrectAnswer-D
Ans.is'd'i.e.,Man[RefPark23rdlep.235]
TyphoidfeveristheresultofsystemicinfectionmainlybyS.typhi
foundonlyinman.Thetermentericfeverincludebothtyphoidfever
(causedbyS.typhi)andparatyphoidfever(causedbyS.Paratyphi
&'C').
Reservoir-Manistheonlyreservoir.Carriersaremoreimportant
thancases.
Sourceofinfection
PrimaryFeces,Urine
Secondary-Water,food,fingers,flies.

853.Rideal-walkercoefficientfordisinfectant
usedforcholerastoolshouldbe?
a)2
b)4
c)7
d)10
CorrectAnswer-D
Ans.is'd'i.e.,10[RefPark23'1/4p.129-30&22"/ep.120]
Forcholerastool:?
"Themosteffectivedisinfectantforgeneraluseisacoal-tar
disinfectantwithaRideal-Walker(RW)coefficientof10ormore
suchascresol".-Park250

854.WomentravelingfromBihartoDelhiis
suspectingtohaveKala-azar.Suitable
investigationis?

a)P24antigen
b)Rk-39test
c)ComboRDT
d)HRP-2antigen
CorrectAnswer-B
Ans.is'b'i.e.,rk-39test[RefPark23rdlep.305,306;Essentialsof
microbiologyp.785]
LaboratorydiagnosisofKala-azar
ThedemonstrationofparasiteLDbodiesintheaspiratesofthe
spleen,liver,bonemarroworlymphnodeisonlywaytoconfirm
visceralleishmaniasis.
Hematologicalfindings:TESR,anaemia,reversedalbumin-globulin
ratio&leucopenia.
AldehydetestofNapierisasimpletestwidelyusedinIndiaforthe
diagnosisofKala-azar.
Serologicaltest:Directagglutinationtest,rk39dipsticktest,ELISA
&Indirectfluorescentantibodytest.
Leishmanintest:Thistestisbasedonskinreaction.

855.Mostcommonrouteofnosocomial
infection[Hospital-acquiredinfection]?
a)Droplettransmission
b)Directcontact
c)Indirectcontact
d)Vehicletransmission
CorrectAnswer-B
Ans.is`B'i.e.,Directcontact[RefTextbookofEnvironmental
microbiologyp.819]
Therearefollowingtypesofmodesoftransmissionofhospital-
acquiredinfections.
Contacttransmission
Itisthemostcommonandmostpreventablemeansoftransmission.
Itisdividedintotwotypes-
Directcontact:Itinvolvescontactofbodysurfacetobodysurface
withaphysicaltransferofmicroorganisms.Handcontactismost
commonmodeoftransmission.
Indirectcontact:Itinvolvesbodysurfacecontactwitha
contaminatedintermediateobject.

856.Behavioralsurveillancesurveyisdonein
?
a)Malaria
b)Filaria
c)AIDS
d)TB
CorrectAnswer-C
Ans.is'c'i.e.,AIDS[Refwww.cdc.gov/hiv]
Behavioralsurveillancesurveyisdoneinpersonswhoareathigh
riskforHIVinfection.
Surveillanceisconductedinrotating,annualcyclesinthreedifferent
populationsatincreasedriskforHIV:?
1. Gay,bisexualandothermenwhohavesexwithmen(MSMcycle).
2. Personswhoinjectdrugs(IDUcycle).
3. HeterosexualsatincreasedriskforHIVinfection(HETcycle).

857.DrugofchoiceforMasstherapyunder
filariacontrolProgramme?
a)Albendazole
b)Ivermectin
c)DEC
d)Mebendazole
CorrectAnswer-C
Ans.is'c'i.e.,DEC[Refnvbdcp.gov.in]
Everypersonabove2yearsofagelivingintheendemicarea
(exceptforpregnentwomen&seriouslyillperson)shouldbegiven
Diethylcarbamazinecitrate(DEC)tablets.
0-2yearsN.1.
t.,2-5years1tabletof100mg
6-14years2tabletsof100mg
15years3tabletsof100mg

858.Whichofthefollowinglarvicideisused
underurbanMalariaScheme?
a)Malathion
b)Parathion
c)DDT
d)Abate
CorrectAnswer-D
Ans.is'd'i.e.,Abate[Refnvbdcp.gov.in]
FollowingchemicallarvicidesareusedintheUrbanMalariaScheme
Programme
1. Temephos(Abate)
2. Bti(WPey,12AS)Bacillusthuringiensisisraelensis.

859.Regularinsectisidalsprayisdonewhen
APIis?
a)>1
b)>2
c)<1
d)<2
CorrectAnswer-B
Ans.is'b'i.e.,>2[Ref:Park20lep.384]
AreawithAPI<2focalspaying
AreawithAPI>2regularspray
AreashavingAnnualParasiteIndex(API)>2
Regular2roundsofinsecticidalspraywithDDT/Malathion/Synthetic
Pyrethroidsatthedoseof1,2,0.5mg/sqmeterrespectively.
Entomologicalassessmentforvectorbehavioranddevelopmentof
insecticidalresistance
Activeandpassivesurveillanceiscarriedoutonregularbasisevery
fortnight.
PresumptiveTreatmenttoallfevercasesandradicaltreatmenttoall
slidepositivecasesisgiven

860.MainfunctionofsodiumcitrateinORS?
a)Toincreaseabsorptionofglucosebycotransport
b)Tocorrectelectrolyteimbalance
c)TocorrectAcidosis
d)Tocorrectdehydration
CorrectAnswer-C
Ans.is'c'i.e.,TocorrectAcidosis
Themainfunctionofsodiumcitrateistocorrectacidosis?
"ThecitrateinORSisneededforthetreatmentofacidosis,which
frequentlyoccurswithdehydration""Efficacyofsodiumcitrate
equalstosodiumbicarbonateforcorrectionofacidosisindiarrhea"

861.ForpostexposureprophylaxisforHIV
theregimenis?
a)Zidovodin+lamivudinfor4weeks
b)Lamivudin+ritonavirfor4weeks
c)Zidovudin+lamivudin+Indinavirfor4weeks
d)Singledosezidovudin+lamivudin+Indinavir
CorrectAnswer-C
Ans.is'c'i.e.,Zidovudin+lamivudin+Indinavirfor4weeks
Postexposureprophylaxis
Anti-retroviraldrugforpost-exposureprophylaxisshouldbeinitiated
assoonaspossibleaftertheexposurewithinthefirstfewhoursand
nolaterthan72hours.
Usuallycombinationoftwonucleosidereversetranscriptase
inhibitors(zidovudinandlamivudin)plusaproteaseinhibitor
(ritonavirorindinavir)isgivenfor4weeks.
Toproventmothertochildtransmission,thepreferredregimenis
singledoseofnevirapinetomotheratonsetoflabourandtochild
within72hoursofbirth.Zidovudineisalsoused,butrequires
multipledosage.Caesareansectionreducestheriskoftransmission
by50%.

862.Trueaboutpost-exposureprophylaxisin
HIV?
a)Shouldbegivenin5daysofexposure
b)Singledosenevirapinepreventsmothertochildtransmission
c)Givenfor2weeks
d)StandardprotocolistouseAnyTwoNRTIswithnootherdrugs
CorrectAnswer-B
Ans.is'b'i.e.,Singledosenevirapinepreventsmothertochild
transmission
[RefHasbeenexplained]

863.Incubationperiodofplasmodiumvivax
is-
a)5-7days
b)7-10days
c)10-14days
d)15-30days
CorrectAnswer-C
Ans.is'c'i.e.,10-14days

864.Drugofchoiceforplasmodiumvivaxis:
September2006
a)Mefloquine
b)Chloroquine
c)Artesunate
d)Quinine
CorrectAnswer-B
Ans.B:Chloroquine
Chloroquineremainsthetreatmentofchoiceforclinicalcureand
suppressiveprophylaxisofalltypesofmalaria,exceptthatcaused
byresistantP.falciparum.
Inshorttimevisitorstochloroquine-sensitiveendemicareas,
suppressivedoseshouldbestarted1weekbeforeandcontinuedfor
4weeksafterreturning.

865.Maximumrelativeriskattributedby
obesitytowhichcondition?
a)Hypertension
b)CHD
c)DM
d)Cancer
CorrectAnswer-C
Ans.is'c'i.e.,DM[RefObesityclinicalmanagementp.712]
"Recently,ameta-analysisof89studiesexaminingtherelativerisk
ofobesity-relatedco-morbiditiesshowedthatthestrongest
associationwaswithtype2diabetesmellitus"
"ProportionoftypeIIdiabetesthatisattributabletoobesitywas
approximately61%whereastheproportionofCHDthatwast
attributabletoobesitywasapproximately17%"

866.RuleofHalvesisrelatedto?
a)Obesity
b)Burns
c)Blindness
d)Hypertension
CorrectAnswer-D
Ans.is'd'i.e.,Hypertension[RefPark23'/ep.376]
RuleofHalves:Hypertensionisan'Icebergdisease'.Onlyabouthalf
ofhypertensivesubjectsingeneralpopulationofmostofthe
developedcountriesareawareofcondition,onlyhalfofthoseaware
oftheproblemwerebeingtreatedandonlyhalfofthosetreated
wereconsideredadequatelytreated.

867.Totalcommunicationmeans?
a)Useofallmethodsofcommunicationforadvertisment
b)Useofallmethodsofcommunicationforschoolteaching
c)Useofallmethodsofcommunicationforcommunity
participation
d)Usingeverycommunicationoptiontoteachdeafchild
CorrectAnswer-D
Ans.is'd'i.e.,Usingeverycommunicationoptiontoteachdeaf
child[RefDevelopment-behavioralpediatricsp.392]
Totalcommunicationisphilosophyofeducatingchildrenwith
hearingloss(deafchildren)thatincorporatesallmeansof
communication,i.e.formalsigns,naturalgestures,fingerspelling,
bodylanguage,listening,lipreadingandspeech.

868.Newborncarecornerispresentin?
a)NICU
b)OPD
c)Labourroom
d)Wardssideroom
CorrectAnswer-C
Ans.is'c'i.e.,Labourroom[Refdghs.gov.bd]
Newborncarecornerisaspacewithinthedeliveryroomwhere
immediatecareisprovidedtoallnewborns
Healthfacility
Allnewbornsatbirth
Sicknewborns
MCHlevelI:
Newborncarecorner
Promptreferral
PHC,Subcentre (NBCC)inlabourroom
Newborn
MCHlevel11:
NBCCinlabourroomand stabilizationunit
CHC,First
operationtheatre
(NBSU)
referralunit
NBCCinlabourroomand Specialnewborn
(FRU)
operationtheatre
careunit(SNCU)
MCHlevel
Districthospital

869.Smokingispreventivefor?
a)Lungcancer
b)Chronicbronchitis
c)Ulcerativecolitis
d)CHD
CorrectAnswer-C
Ans.is'c'i.e.,Ulcerativecolitis
Smokinghasbeenshowntohavesomeprotectiveeffectin-

1. Ulcerativecolitis
2. Parkinson'sdisease
3. Endometrialcanceranduterinefibroid
4. Pre-eclampsia
5. Thyroidcancer
6. Skincancer(melanoma)
7. Psychiatricsymptoms
8. Aphthousstomatitis

870.Accordingfemalesterlization2014
guidelines,eligibilitycriteriaforfemale
sterilizationareallexcept?

a)Agebetween22-49years
b)Shouldhaveatleast1child
c)Unmarriedwoman
d)Partnerisnotsterilized
CorrectAnswer-C
Ans.is'c'i.e.,Unmarriedwoman[Refwww.tvhealth.org]
Followingaretheeligibilitycriteriaforfemalesterilization(2014)?
1. Clientsshouldbeever-married.
2. Femaleclientsshouldbeabovetheageof22yearsandbelowthe
ageof49years.
3. Thecoupleshouldhaveatleastonechild,whoseageisaboveone
year,unlessthesterilizationismedicallyindicated.
4. Clientsortheirspouses/partnersmustnothaveundergone
sterilizationinthepast(notapplicableincasesoffailureofprevious
sterilization).
5. Clientsmustbeinasoundstateofmind,soastounderstandthefull
implicationsofsterilization.
6. Mentallyillclientsmustbecertifiedbyapsychiatristandastatement
shouldbegivenbythelegalguardian/spouseregardingthe
soundnessoftheclient'sstateofmind.
7. Arelevantmedicalhistory,physicalexaminationandlaboratory
investigationsneedtobecompletedtoascertaineligibilityfor
surgery.

871.WhichstatehaslowestIMR?
a)UttarPradesh
b)Kerala
c)Maharashtra
d)TamilNadu
CorrectAnswer-B
Ans.is'b'i.e.,Kerala[RefPark23rd/ep.563-561
Keralahaslowest-

1. Infantmortalityrate
2. Neonatalmortalityrate
3. Postneonatalmortalityrate
4. Childmortalityrate

872.Grossreproductionrateis?
a)Numberofgirlsborntoamotherinherreproductiveage
b)Numberofboysborntoamotherinherreproductiveage
c)Numberoftotalchildrenborntoamotherinherreproductive
age
d)Numberoflivesbirthsper1000women
CorrectAnswer-A
Ans.is'a'i.e.,Numberofgirlsborntoamotherinher
reproductiveage[RefPark23'/ep.489]
Grossreproductionrate
Averagenumberofgirlsthatwouldbeborntoamarriedwomanif
sheexperiencesthecurrentfertilitypatternthroughouther
reproductivespan(15-44or49year)assumingnomortality.
GRRinIndiais1.1(1.2inruralareasand0.8inurbanareas).

873.Inatownthereare2500livebirthwithin
sixmonth.Duringsameperiod5women
diedduetoperipartuminfection,5died
duetoelectrocution,2dieddueto
obstructedlaborand3diedduetoPPH.
WhatistheMMR?

a)4per1000livebirth
b)6per1000livebirth
c)40per1000livebirth
d)60per1000livebirth
CorrectAnswer-A
Ans.is'a'i.e.,4per1000livebirth[RefPark23rd/ep.559]
Inthisquestion,pregnancyrelateddeathsare10(infection,
obstructedlaborandPPH).Electrocutionisnotapregnancy/labor
relateddeathsoexcludedfromnumerator..

874.Vitalstatisticsinapopulationare?
a)Sexratio
b)Agecomposition
c)Birthrate
d)Dependencyratio
CorrectAnswer-C
Ans.is'c'i.e.,Birthrate
Vitalstatisticsarestatisticsconcerningtheimportanteventsin
humonlife,suchasbirth,death,marriagesandmigration.Theseare
-
Birthrates
Deathrates
Infantmortalityrate
Fertilityrate
Mortalilityrates(perinatalmortalityrate,MMR,CMRetc)

875.Whatisexponentialgrowth?
a)Rapidgrowthinpopulationthatleadstodisbalanceinbirthand
deaths
b)Slowgrowthrate
c)Growthlimitedbylimitingfactors
d)None
CorrectAnswer-A
Ans.is'a'i.e.,Rapidgrowthinpopulationthatleadsto
dysbalanceinbirthanddeaths[RefO.P.Ghai7Vep.93]
ExponentionalVslogisticpopulationgrowth
Thepopulationgrowthofaspeciesisregulatedbylimitingfactors
(resources)thatexistwithinthespeciesenvironment.
Populationgrowthmaintainsequilibriuminallspeciesundernormal
conditionsbacauseoftheselimitingfactors.
Apopulationsoverallgrowthrateisaffectedbythebirth
rate
anddeathrate.
Therateofincreasewithinapopulationsisrepresentedbythebirth
rateminusdeathrate.
Whenthegrowthrateinapopulationisrepresentedbythebirthrate
minusthedeathrate,thepopulationremainsataconstantlevel.

876.Sampleregistrationsystemisdoneonce
in?
a)6months
b)1year
c)2years
d)5years
CorrectAnswer-A
Ans.is'a'i.e.,6Months


877.Inacommunityof1000000population
105childrenwereborninayearoutof
which5wasstillbirths,and4diedwithin
6monthsafterbirth.TheIMRis?

a)40
b)90
c)120
d)150
CorrectAnswer-A
Ans.is'a'i.e.,40[Ref:Readbelow]
Inthegivenquestionoutof105deliveries,5werestillbirths-3Thus,
livebirthsare100.
Infantdeathsare4.

878.Maternalmortalityrateisdefinedas?
a)Maternaldeathper1000totalbirths
b)Maternaldeathper1000livebirths
c)Maternaldeathper1000women
d)Maternaldeathper1000womenofreproductiveage
CorrectAnswer-D
Ans.is'd'i.e.,Maternaldeathper1000womenofreproductive
age[RefPark23rd/ep.558-559]
Maternalmortalityrate=Totalno.offemaledeathdueto
complicationofpregnancychildbirthorwithin42daysofdelivery/
Totalno.ofwomenofreproductiveagex1000


879.NPUforeggis?
a)70
b)80
c)85
d)100
CorrectAnswer-D
Ans.is'd'i.e.,100
Thenetproteinutilization,orNPU,istheratioofamino
acidmassconvertedtoproteinstothemassofaminoacids
supplied.

NPUforegg:100 Wheat:51
Fish:77
NPUformeat:80 Pulses:45-50 Rice:65
NPUformilk:81 Soyabean:55

880.Skeletalfluorosisoccurswithfluoride
levelinwater?
a)<1.5mg/L
b)1.5-3mg/L
c)3-6mg/L
d)>10mg/L
CorrectAnswer-C
Ans.is'C'i.e.,3-6mg/L
Dentalfluorosis>1.5mg/L(PPM)
Skeletalfluorosis3-6mg/L(PPM)
Cripplingfluorosis>10mg/L(PPM)

881.Mid-daymealprovides?
a)1/2ofenergy
b)1/3ofprotein
c)30gmpulse/day
d)Allarecorrect
CorrectAnswer-C
Ans.is'c'i.e.,30gmpulse/day[RefPark23"Vep.662]
Mid-dayMealProgramme
Itisalsoknownas'SchoolLunchProgramme'.Itwaslaunched
in1961underMinistryofEducation.Thefeaturesofprogramme
are:

1. Mealshouldbeasupplementandnotasubstituetohomediet.
2. Themealshouldsupplyatleast1/3oftotalenergyrequirementand
Y2oftotalproteinrequirement.
Amodel-menuofmid-dayschoolmealisasfollows:-
Amid-dayschoolmeal
Foodstuffs
g/day/child
Cerealsandmillets
75
Pulses
30
Oilsandfats
8
Leafyvegetables
30
Non-leafyvegetables 30

882.Maximumproteinisfoundin?
a)Egg
b)Soyabean
c)Rice
d)Wheat
CorrectAnswer-B
Ans.is'b'i.e.,Soyabean

883.Tabletforsupplementationofironand
folicacidforadultcontains?
a)20mgiron,5001,tgfolicaci
b)40mgiron,250folicacid
c)100mgiron,500ligfolicacid
d)100mgiron,100hgfolicacid
CorrectAnswer-C
Ans.is'c'i.e.,100mgiron,500jigfolicacid

884.Limitingaminoacidincereals?
a)Methionine
b)Tryptophan
c)Lysine
d)Cysteine
CorrectAnswer-C
Ans.is'c'i.e.,Lysine
Food
Deficiency
Cereals Lysine&threonine
Wheat
Lysine&threonine
Maize
Tryptophan&lysine
Pulses
Methionine&cysteine
Soybean Methionine

885.VanaspatiGheeisfortifiedwith?
a)Iodine
b)VitaminA
c)Iron
d)Calcium
CorrectAnswer-B
Ans.is'b'i.e.,VitaminA
Examplesoffoodfortification:-

1. IodisationofSalt
2. AdditionofvitaminAandDinvanaspati(2500IUvitaminAand175
IUvitaminDper100gm).
3. Fluoridationofwater.
4. Calciumaddedtofruitjuices.
5. Folicacidaddedtoflour.

886.VillageHealthandNutritionDay(VHND)
isobserved?
a)Everyweek
b)Everymonth
c)Every6month
d)Everyyear
CorrectAnswer-B
Ans.is'B'i.e.,Everymonth
TheVillage-HealthandNutritionday(VHND)
istobeorganized
onceeverymonthonafixedday(suchasthesecondSaturday).
ThedaycanbedecidedbytheVHWSC(villagehealthandwater
sanitationcommittee)ineachvillageatanyoneoftheAnganwadi
centers(AWCs)inthatvillage,preferably,alltheAWCsshouldbe
coveredbyrotation.
Onthatday,AnganwadiworkerandotherVHWSCmemberswill
mobilizevillagerstoassembleinAWC.

887.Phrynodermaisdueto...deficiency-
a)VitaminD
b)Niacin
c)VitaminA
d)Essentialfattyacid
CorrectAnswer-D
Ans.is'd'i.e.,Essentialfattyacid
Invitamin'A'deficiencythereistoadlikeskinalsoknownas
phrynoderma.
oButthisisduetoassociateddeficiencyofessentialfattyacids.

888.Proteinqualityassessmentisbestdone
by?
a)NPU
b)Biologicalvalue
c)Digestibillitycoefficient
d)Aminoacidscore
CorrectAnswer-A
Ans.is'a'i.e.,NPU
oNetproteinutilizationisconsideredtobemostpractilebecauseit
istheproductofbiologicalvalueanddigestibilitycoefficientdivided
by100.

889.Nutrientwhichislostmaximumin
polishedrice?
a)Proteins
b)Thiamine
c)Ascorbicacid
d)Calcitriol
CorrectAnswer-B
Ans.is'b'i.e.,Thiamine
Effectofmillingonrice
Themillingprocessdeprivesthericegrainofitsvaluablenutritive
elements

890.Glycemicindexisdefinedas:
a)Glucosecontrolinlast3months
b)Measureofthechangeinthebloodglucosefollowingingestion
ofproteins
c)Measureofthechangeinthebloodglucosefollowingingestion
ofcarbohydrate
d)Measureofthechangeinthebloodglucosefollowingingestion
offats.
CorrectAnswer-C
TheGlycemicindex(GI)ofacarbohydratecontainingfoodisa
measureofthechangeinthebloodglucosefollowingitsingestion

891.Lowglycemicindexisclassifiedasvaluelessthan:
a)25
b)45
c)55
d)65
CorrectAnswer-C
Conceptofglycemicindexhasutilityinmanagementofdiabetesandobesity.
GIrange
Classification
Example

Mostfruitsandvegetablesexceptpotatoesandwatermelon,
LowGI
55orless pastabeans,lentils
MediumGI
56-69
Sucrose,brownrice,basmatirice
70or
HighGI
Cornflakes,whitebread,candybar
more
Ref:Park22ndedition,page568

892.Caloricrequirementinanadultmalefor
heavywork?
a)1800kcal/d
b)2300kcal/d
c)3000kcal/d
d)3500kcal/d
CorrectAnswer-D
Ans.is`d'i.e.,3500kcal/d[RefPark23rdlep.634]
IndianReferenceIndianReference
Energyrequirements Man
Woman
Lightwork
2320kcal/day 1900kcal/day
Moderatework
2730kcal/day 2230kcal/day
Heavywork
3490kcal/day 2850kcal/day

893.Differencesbetweenhumanmilkand
cowmilkareallofthefollowingexcept:
September2005

a)Cowmilkhascomparativelymorefat
b)Cowmilkhascomparativelymoreprotein
c)Cowmilkhascomparativelymorecalcium
d)Cowmilkisirondeficient
CorrectAnswer-D
Ans.D:Cowmilkisirondeficient
Cowmilkhascomparativelymoreenergy,fat,protein,minerals,iron
andcalciumHumanmilkhascomparativelymoreoflactoseand
vitaminCbutisdeficientofiron.

894.Skinfoldthicknessismeasuredinallofthefollowingplaces,EXCEPT:
a)Midtriceps
b)Biceps
c)Suprapubic
d)Suprailiac
CorrectAnswer-C
Harpendencallipersareusedtomeasureskinfoldthickness.
Skinfoldthicknessismeasuredinmidtriceps,biceps,suprailiacandsubscapular
areas
.
Thesumofthesemeasurementsshouldbelessthan40mminboysand50mmingirls.
Ref:Park21stedition,page369.

895.Whichistrueofpathogenicmosquitoes
?
a)Anopheleshasspottedabdomen
b)Mansonilayseggssingly
c)Culexcauseyellowfever
d)Aedeshasstrippedyellowscales
CorrectAnswer-D
Ans.is'd'i.e.,Aedeshasstrippedyellowscales[RefPark23rdie
p.769&22nd/ep.714]
Aedesmosquitoesareeasilydistinguishedbywhitestripesona
blackbody.BecauseofthestripedorbandedCharacteroftheirlegs
theyaresometimesreferredtoastigermosquito.
Anopheleshasspottedwings(notspottedabdomen).
Mansonilayseggsinclusters.
Yellowfeveriscausedbyaedesmosquito(notculex).

896.Whichisnotanarylorganophosphate?
a)Malathion
b)Parathion
c)Chlorthion
d)Diazinon
CorrectAnswer-A
Ans.is'a'i.e.,Malathion
Organophosphates

Alkyl
Aryl
Malathion
Diazinon
Sulfotepp
Parathion
Demeton
Chlorpyrifos Triclorfon
Chlorthion
HETP
Paraoxon
TEPP

897.Trueaboutculexlarvae?
a)Restparalleltosurfacewater
b)LongPalmatehair
c)Siphontubepresent
d)Allaretrue
CorrectAnswer-C
Ans.is'c'i.e.,Siphontubepresent[RefPark23'/ep.769&
22"d/ep.714]


898.Horrock'sapparatusisusedtomeasure
?
a)Windvelocity
b)Humidity
c)Chlorinedemand
d)Coolingpower
CorrectAnswer-C
Ans.is'c'i.e.,Chlorinedemand
-ChlorinedemandforwatercanbeestimatedbyHorrock's
apparatus.
-Chlorinedemandofwateristheamountofchlorinethatisneeded
todestroybacteriaandtooxidizealltheorganicmatterand
ammoniacalsubstancespresentinwater.
-Itisthedifferencebetweentheamountofchlorineaddedtothe
water,andtheamountofresidualchlorineremainingattheendofa
specificperiodofcontact(usually1hr),atagiventemperatureand
pHofthewater

899.IndicatorusedinHorroch'sapparatus?
a)Bleachingpowder
b)Soda-lime
c)Potassiumpermagnate
d)Starchiodine
CorrectAnswer-D
Ans.is'd'i.e.,Starchiodine[RefPark23'/ep.717]
InHorrock'sapparatus,6cupsarefilledwithwaterandindicator
(starchiodine)isaddedinincreasingquantityineachcup(1unitin
lst,2unitsin2nd,3unitsinthirdandsoon).
Thefirstcupwhichshowsthebluecolorismultipliedby2,for
exampleif2ndcupshowsbluecolourthan4grams(2x2)of
bleachingpowderwillrequiredtodisinfect455litreofwater.

900.Inwatertestingfromawell[containing
75000litresofwater]byHorrock's
apparatus,thereisbluecolourfrom
4thcuponwards.Whatistheamountof
bleachingpowderrequiredtodisinfect
thewater?

a)1000gm
b)1300gm
c)1600gm
d)2000gm
CorrectAnswer-B
Ans.is'b'i.e.,1300gm
InHorrock'sapparatus,6cupsarefilledwithwaterandindicator
(starchiodine)isaddedinincreasingquantityineachcup(1unitin
1st,2unitsin2nd,3unitsinthirdandsoon).
Thefirstcupwhichshowsthebluecolorismultipliedby2,for
example,ifthe2ndcupshowsabluecolourthan4grams(2x2)of
bleachingpowderwillberequiredtodisinfect455litreofwater.
Bleachingpowderrequired=8*75000/455=1318

901.Daylightfactorinthekitchenshouldbe?
a)5%
b)8%
c)10%
d)15%
CorrectAnswer-C
Ans.is'c'i.e.,10%[RefPark22"d/ep.687,688]
Minimumrecommendeddaylightfactor:
Livingroom8%
Kitchens10%

902.Anemometermeasures?
a)Humidity
b)Airvelocity
c)Roomtemperature
d)Radianttemperature
CorrectAnswer-B
Ans.is'b'i.e.,Airvelocity[RefPark23'/ep.749&22"d/ep.695]

903.SIunitofluminalintesityis-
a)Candela
b)Lumen
c)Lux
d)Coulomb
CorrectAnswer-A
Ans.is'a'i.e.,Candela[RefBASAKp.4]
CandelaistheSIbaseunitofluminousintensity,i.e.luminous
powerperunitsolidangleemittedbyapointlightsourceina
particulardirection.
SIunitofluminousintensity-?Candela
SIunitofluminousflux-?Lumen
SIunitofilluminance-?Lux

904.WasteSharpsshouldbedisposedin?
a)Blackbag
b)Yellowbag
c)Bluebag
d)Noneofthese
CorrectAnswer-C
Ans.is'c'i.e.,Bluebag[RefPark23'/ep.793-794]

905.Whichcategorywasteisdisposedinred
bags?
a)Category1
b)Category2
c)Category3
d)Category10
CorrectAnswer-C
Ans.is'c'i.e.,Category3[RefPark23'/ep.793,794]

906.Allareincineratedexcept?
a)Humananatomicalwaste
b)Animalwaste
c)Infectedsolidwaste
d)Brokenthermometers
CorrectAnswer-D
Ans.is'd'i.e.,Brokenthermometers[Ref:Park23"1/ep.791&
22"/ep.738]
"Wastetypesnottobeincineratedare:(a)pressurizedgas
container;(b)largeamountofreactivechemicalwastes;(c)silver
saltsandphotographicorradiographicwastes;(d)Halogenated
plasticssuchasPVC;(e)wastewithhighmercuryorcadmium
content,suchasbrokenthermometers,usedbatteries,andlead-
linedwoodenpanels;and(J)sealedampulesorampulescontaining
heavymetals"

907.Incinerationisusedforwhichcategory
ofwaste?
a)Category1
b)Category7
c)Category4
d)Category10
CorrectAnswer-A
Ans.is'a'i.e.,Category1[RefPark23`alep.793-794]
Incinerationisdoneforcategory1,2,3&6

908.Micropolysoporafaenicauses?
a)Baggasosis
b)Farmer'slung
c)Suberosis
d)Sequousis
CorrectAnswer-B
Ans.is'B'i.e.,Farmer'slung
MicropolysporafaeniinHayorgraindustcausesfarmer'slung.

Anthracosis
Silicosis
Coaldust
Siderosis
Silica
Byssinosis
Iron
Farmer'slung
Cottondust(textileindustry)
Sequousis
Hayorgraindust(micropolysporafaeni)
Suberosis
Moldyredwoodsawdust
Detergentworker's
Moldycorkdust
lung
Enzymeadditives
Baggassosis
Sugarcanedust(thermoactinomyces
sacchari)

909.Indisastermanagementallaretrue
except
a)Mitigationbeforeadisasterstrikes
b)Responseinpre-disasterphase
c)Yellowcolourisformediumpriority
d)Gastroenteritisiscommonestinfectionafterdisaster
CorrectAnswer-B
Ans.is'B'i.e.,Responseinpre-disasterphase
Disastermanagement
DisasterImpact
Mostinjuriesaresustainedduringtheimpact,andthusthe
greatestneedforemergencycareoccursinthefirstfewhours.
Themanagementcanbedividedinto:-
Search,rescueandfirstaid:Mostimmediatehelpcomesfrom
uninjuredsurvivors.
Fieldcare:Asmanyinjuredpatientscomesimultaneously,
emergencyservicesshouldbeproperintermsofpriorityand
numbersoffacilities.
Triage:Triageconsistsofclassifyingtheinjuredbasedonthe
severityofinjuriesandthelikelihoodoftheirsurvivalwithprompt
medicalintervention.Themostcommontriageclassificationsystem
usedinternationallyisfourcolourcodesystem:?
1. Red Highpriority
-
treatmentortransfer
2. Yellow Mediumpriority
-
3. Green
Ambulatorypatients

4. Black
Deadormoribund
-
patients
Tagging:Allpatientsshouldbeidentifiedwithtags.
Identificationofthedead:Properrespecttodeadisofgreat
importance

910. Indisastermanagementfollowingare
practicedexcept
a)Triage
b)Rehabilitation
c)Massvaccination
d)Disasterresponse
CorrectAnswer-C
Ans.is'c'i.e.,Massvaccination
DisasterManagementincludesthreeaspects:
1.Disasterimpactandresponse:
Search,rescue,andfirst-aid
?Fieldcare
?Triage
?Tagging
?Identificationofthedead
2.Rehabilitationorrecovery:
?Watersupply
?Basicsanitationandpersonalhygiene
?Foodsafety
?Vectorcontrol
3.Mitigation:Measuresdesignedeithertopreventhazardsfrom
causingdisasterortoreducetheeffectsofthedisaster.This
alsoincludespreparednessforanyimpendingdisastersorin
disaster-proneareas.


911.TrueaboutAnganwadiworkerisall
except?
a)Mostlyfemale
b)Trainingfor4months
c)UnderICDSscheme
d)Coversapopulationof2000
CorrectAnswer-D
Ans.is'D'i.e.,Coversapopulationof2000
AnganwadicenterismainlymanagedbyAnganwadiworker.
MostlyAnganwadiworkerisafemaleworker.
Sheisahealthworkerchosenfromthecommunityandgiven4
monthstraininginhealth,nutritionandchildcare.
oneAWCisfor400-800population

912.AsperICDSscheme,thereshouldbe
oneAnganwadicentreforapopulation
of?

a)1000-1500
b)2000-25000
c)400-800
d)100-200
CorrectAnswer-C
Ans.is'c'i.e.,400-800
TherecommendationsofAnganwadicenterare:?

1. Rural/urbanprojects->Oneanganwadicentereper400-800
population,andonemini-anganwadicenterper150-400population.
2. Tribal/Hilly/OtherdifficultareasOneanganwadicentreper300-800
population,andonemini-anganwadicentreper150-300population.

913.AllaretrueforASHAworkerEXCEPT:
March2013
a)InformsaboutbirthanddeathsinhervillagetoPHC
b)Educationatleasttill4thclassorhigher
c)Localresident
d)Worksper1000peopleofanarea
CorrectAnswer-B
Ans.Bi.e.Educationatleasttill4thclassorhigher
ASHA/AccreditedSocialHealthActivists(ASHAs)
ASHAsmustprimarilybefemaleresidentsofthevillagethatthey
havebeenselectedtoserve,whoarelikelytoremaininthatvillage
fortheforeseeablefuture.
Married,widowedordivorcedwomenarepreferredoverwomen
whohaveyettomarrysinceIndianculturalnormsdictatethatupon
marriageawomanleaveshervillageandmigratestothatofher
husband.
ASHAsmusthaveclasseighteducationorhigher,preferablybe
betweentheagesof25and45,andareselectedbyand
accountabletothegrampanchayat(localgovernment).

914.TrueaboutASHAareallexcept
a)Oneper1000ruralpopulation
b)Mobiliserofantenatalcare
c)Femalevoluntaryworker
d)Skilledbirthattendant
CorrectAnswer-D
skilledbirthattendant/Ref:Park20/e,p380-381(19k,p365)1
Ref.littp://molifw.nic.in/NRHM/stakeholders.htm
ASHAisthecentralcomponentoftheNationalRuralHealthMission
(NRHM)
NationalRuralHealthMission(NRHM)waslaunchedtoaddressthe
healthneedsofruralpopulation,especiallythevulnerablesections
ofsociety
OneofthekeycomponentsoftheNationalRuralHealthMissionis
toprovideeveryvillageinthecountrywithatrainedfemale
communityhealthactivist--`ASHA'orAccreditedSocialHealth
Activist.
Selectedfromthevillaitselfandaccountabletoit,theASHA
willhetrainedtoworkasaninterfacebetweenthecommunityand
thepublicehealthsystem.
FollowingarethekeycomponentsofASHA:
ASHAmustprimarilybeawomanresidentofthevillage--married/
widowed/divorced,preferablyintheagegroupof25to45years.
Sheshouldbealiteratewomanwithformaleducationuptoclass
eight.Thismayberelaxedonlyifnosuitablepersonwiththis
qualificationisavailable.
ASHAwillbethefirstportofcallforanyhealthrelateddemandsof
deprivedsectionsofthepopulation,especiallywomenandchildren,
whofinditdifficulttoaccesshealthservices

ASHAwillbeahealthactivistinthecommunitywhowillcreate
awarenessonhealth
anditssocialdeterminantsandmobilizethe
communitytowardslocalhealthplanningandincareasedutilization
andaccountabilityoftheexistinghealthservices.Shewouldbea
promoterofgoodhealthpractices.
Shewillcounselwomenonbirthpreparedness,importanceofsafe
delivery,breastfeedingandcomplementaryfeeding,immunization,
contraception
andpreventionofcommoninfectionsincluding
ReproductiveTractInfection/SexuallyTransmittedInfection
(RT1s/STIs)andcareoftheyoungchild
Thegynaecomastiacausingdrugscanhecategorizedinto:
Thefirsttypearedrugsthatactexactlylikeestrogens,suchas
diethylstilbestrol,birthcontrolpills,digitalis,andestrogencontaining
cosmetics.
Thesecondtypeisdrugsthatenhanceendogenousestrogen
formation
suchasgonadotropinsandclomiphene.
Thethirdtypeisdrugsthatinhibittestosteronesynthesisandaction
such
asspironolactone,ketoconazole,metronidazoleand
cimetidine.
Thefinaltypeisdrugsthatactbyunknownmechanismssuchas
isoniazid,methyldopa,captopril,tricyclicantidepressants,diazepam
andheroin.
Alsoknow,
TestosteronealsocausesGynaecomastia

915.Allofthefollowingaredutiesofan
ASHAworkerexcept:
a)Primaryscreeningforprevalenceofnon-communicable
diseases
b)AdministeringzerodoseofDPTandOPV
c)AssessingthesuccessofnationalprogramsunderANM
d)All
CorrectAnswer-B
Ans:C.AdministeringzerodoseofDPTandOPV
Ref:Park24Iep936,23Iep449,22Iep414;MinistryofHealthanti
FamilyWelfare(MoHFW).(2005b).ReadingMaterialforASHA.
GovernmentofIndia)
ASHAdoes'ntreceivefinancialremunerationforadministeringzero
doseofDPTandOPVisnotthefunctionofASHA.
ASHAPaymentsunderJananiSurakshaYojana(JSY):On45th
Day:
6visitsininstitutionaldeliveries(Day3,7,14,21,28,
7visitsinhomedeliveries(Day1,3,7,14,21,28,42).
Birthweightrecord
ImmunizedwithBCG,firstdoseofOPV&DPT
Birthregistration
Mother&childaresafe

916.ASHAgetsremunerationonallexcept?
a)Institutionaldelivery
b)ZerodoseofOPVandBCG
c)Recordingbirthweight
d)Birthregistration
CorrectAnswer-B
Ans.is'b'i.e.,ZerodoseofOPVandBCG

917.Swajaldharaprogrammeisassociatedwith:
a)Provisionofsafedrinkingwater
b)Provisionoffoodsupplementsfordestitutewomen
c)Provisionofreliefforvictimofsexualabuse
d)Provisionofhealthcareforsicktribals
CorrectAnswer-A
Swajaldharaisacommunityleadparticipatoryprogramme,whichaimsatprovisionofsafe
drinkingwaterinruralareaswiththefullownershipofthecommunity.
Swajaldharahas2components:
SwajaldharaI?foraGramaPanjayathoragroupofPanjayath
SwajaldharaII?foradistrictassuch
Ref:Park,Edition21,Page-419

918.Groupof4-8expertstalkinginfrontofa
largegroupofaudienceisknown
as:
September2011

a)Symposium
b)Workshop
c)Seminar
d)Paneldiscussion
CorrectAnswer-D
Ans.D:PanelDiscussion
Inapaneldiscussion,4-8personswhoarequalifiedtotalkaboutthe
topicsitanddiscussagivenproblem,orthetopic,infrontofalarge
groupofaudience
PanelDiscussion
Apanelconsistsofasmallgroupoffouroreightpersons,whocarry
onaguidedandinformaldiscussionbeforeanaudienceasifthe
panelweremeetingalone.
Theproceedingsofthepanelshouldbethesameasthose
describedforinformaldiscussion:volunteeringoffacts,
askingquestions,statingopinions-allexpressedwithgeniality,with
respectforthecontributionsofothermembers,withoutspeech
making,andwithoutmakinginvidiouspersonalreferences.
Thisprimaryfunctionshouldoccupyapproximatelytwo-thirdsofthe
allottedtime-sayfortyminutesofanhour'smeeting.
Thesecondaryfunctionofthepanelistoanswerquestionsfromthe
audience.
Thisdiscussionmethodissuitableforusewhenarelativelylarge
audienceisanticipated.

audienceisanticipated.
Thedisadvantageofthemethodisthatitconfinesmostofthe
discussiontothepanelitself.
Theaudiencelistensandisgivenachancetoaskquestions,butfor
themostpartispassiveandreceptive.
Paneldiscussions,ifwellconducted,areusuallymoreinterestingto
theaudiencethanisthesingle-speakerforum.
Theyprovidesufficientlyvariedclashofopinionandpresentationof
factstogiveeventhequietmembersoftheaudienceafeelingof
vicariousparticipation.
Qualityandtasksofleadershipinpaneldiscussionaresimilarto
thosedescribedforinformaldiscussion.
Theleadermustinadditiontakespecialcaretoselectpanel
memberswhocanthinkandspeakeffectively.
Hemustalsobesurethattheypreparethemselvestodiscussthe
subject.
Duringthediscussionbythepaneltheleaderhassubstantiallythe
samedutiesasininformaldiscussionexceptthatheshouldkeep
himselfmoreinthebackgroundaschairmanofthepanel.
Hecandosobecauseeachmemberofthepanelisinrealityan
assistanttotheleaderandisresponsibleforspecificcontributionsto
theproceedings.
Whenthesubjectisthrownopentothehouse,itistheleader'sjob
torecognizeappropriatequestionsandtorejectthosenotbearing
onthesubjectorinvolvingpersonalities.
Somequestionshemayanswerhimself,butusuallyheshould
repeatthequestionandcallupononeofthepaneltoanswerit.
Bypreliminaryannouncementtheleadermayalsotelltheaudience
thattheymaydirectquestionsatparticularmembersofthepanelif
theychoose.
Inanycase,duringthequestionperiodtheleaderneedstomaintain
strictcontrol.Onmanyoccasionsthismaybethetoughestpartofhis
assignmenttocarryoffefficientlyandwithgoodhumor.
Whileitiscustomarytoconfineaudiencequestionstoaspecific
period,someleaderspermitquestionsfromtheflooratanytime.
Unlessverycarefullylimitedbytheleader,thispracticemay
interferewitheffectivediscussionbythepanel.
Arrangingthepanelproperlywilllendeffectivenesstothisformof

Arrangingthepanelproperlywilllendeffectivenesstothisformof
discussion.Themembersshouldfacetheaudience.Itisimportant
thateachpanelmemberadjusthischairsothathecanseeevery
othermemberwithouteffortthechairmanwillalsofindthatthebest
placesforhisreadiestspeakersareattheextremeendsofthe
table.Heshouldkeepthemorereticentmemberscloseto,himso
thathecanreadilydrawthemoutwithdirectquestions.
Ifthequieteronessitonthefringesofthepanel,themorevoluble
membersarequitelikelytomonopolizethediscussion.

919.Allareprinciplesofprimaryhealthcare
except?
a)Intersectoralcoordination
b)Communityparticipation
c)Appropriatetechnology
d)Decentralisedapproach
CorrectAnswer-D
Ans.is'd'i.e.,Decentralisedapproach
oThereare4mainprinciplesofprimaryhealthcare:
1)Equitabledistribution
2)Communityparticipation
3)Intersectorialcoordination
4)Appropriatetechnology

920.ScreeningunderRNTCPemphasizeson:
a)Sputummicroscopy
b)ChestX-ray
c)PCR
d)Sputumculture
CorrectAnswer-A
Ans.a.Sputummicroscopy
Over-relianceoncheslX-raywasadrawbackofNational
TuberculosisProgramme(NTP)whichwasovercomebyRevised
NationalTuberculosisControlProgramme(RNTCP)whichstarted
diagnosingpatientbysputummicroscopy
'Anation-widenetworkofRNTCPqaalityassureddesignated
sputumsmearmicroscopylaboratorieshasbeenset
up,providingappropriate,available,affordableandaccessible
diagnosticservicesforTBsuspectsandcases.

921.WhichofthefollowingscreeningmethodsisusedunderRNTCP?
a)Active
b)Passive
c)Mass
d)Alloftheabove
CorrectAnswer-B
UnderRNTCP,activecasefindingisnotpursued.Casefindingispassive.Patients
presentingthemselveswithsymptomssuggestingtuberculosisarescreenedwithtwo
sputumsmearexaminations.
Ref:Park21stedition,page381.

922.DrugNOTusedinpulmonary
hypertension
a)Calciumchannelblocker
b)Endothelinreceptorantagonist
c)Alphablocker
d)Prostacyclin
CorrectAnswer-C
alphablockers[Ref-Harrison17th/ep1577,1578]Pulmonary
hypertension
Generalmanagement
Diuretictherapymaybeusefulasitrelievespulmonaryedema.
Anticoagulanttherapyisadvocatedforallpatients.
Specificmanagement
Calciumchannelblockers
Patientswhohavesubstantialreductionsinpulmonaryarterial
pressureinresponsetoshortactingvasodilatorsatthetimeof
cardiaccatheterizationshouldheinitiallytreatedwithcalcium
channelblockers.
Endothelinreceptorantagonist
Bostenanisanon-selectiveendotheliumreceptorantagonist,isan
approvedt/t,forpatientswhoareNYHA.functionalclassesIIIand
IV.
Phosphodiesterase-5inhibitors
SlidenafilisusedforpatientswhoareNYHAfunctionalclassesII
and
Prostacyclins
IloprostisaprostacyclinanalogueusedinPAHpatientswhoare

NYHAfunctionalclassesIIIandIV.
Pulmonarycirculationisuniqueinthatitaccommodatesabloodflow
thatisalmostequaltothatofalltheotherorgansofbodybutstill
maintainslowpressure.
Thefactorsresponsibleforlowpressureinpulmonarycirculation
(evenwithlargevolumeofblood)are:--Largerdiameterof
pulmonaryvesselsduetothinwallofpulmonaryarteryand
arterioles.
-Greatercompliance(distensibility)ofpulmonaryvessels.
numoralfactorsresponsibleformaintainingpulmonarycirculation:?
Normally
NOcausesvosodilationandproliferationofsmoothmusclesbyrin
theconc.ofcGMP--)thisincreasesthediameterofpulmonary
vessels.
causesvosodilationanddecreasedproliferationofsmoothmuscles
byincreasingtheconc.ofcAMPincreaseinthediameterof
pulmonaryvessels.Prostaglandinalsodecreasescoagulation.
Endothelincausesvasoconstrictionandincreasesmoothmuscle
proliferationsilumenofpulmonaryvessels.-Normallythe
vasodilatorsandantiprolifeaveeffectsofNOandPGI7dominate
InPulmonaryhypertension
ThereislproductionofNO(andcGMP)andPGI2(andcAMP),t
vasoconstrictionandproliferationofsmooth

muscles,stlumenofpulmonaryvessel.Decreasedproductionof
PGI2alsocausesincreasedcoagulation.

ThereisTproductionofendothelin,/vasoconstrictionandsmooth
muscleproliferation,.1lumen.
Drugtherapyinpuhnonan,hypertensionistargettedatthesegrowth
factorpathwayswhichareinvolvedinthepathogenesisof
pulmonaryhypertension:
Endotheliumreceptorantagonist
Bostenanisanendothelinreceptorantagonist.Itprevents
endothelinmediatedcontractionofvessels.Phosphodiesterase-5


inhibitors
NitricoxidemediatesitsactionthroughincreasingcGMP
concentration.IncreasedcGMPrelaxesthevessels.cGMPis
degradedbyanenzymephosphodiesterase-5(PDE-5).
-Slidenafanilisaphosphodiesterase5inhibitor.Itreducesthe
degradationofcGMP,thuscausingvascularrelaxationandreducing
pulmonaryhypertension.
Prostacyclins
Inpulmonaryhypertension,thelevelofprostacyclineisreduced.
Thiscausespulmonaryconstrictionasprostacyclincausesdilatation
ofpulmonaryvessels.


923.RegardingPPVvaccinefollowingistrue
?
a)Givenatbirth
b)Obtainedfromcellwallpolysaccharide
c)Indicatedinsicklecelldisease
d)Commonlyused
CorrectAnswer-C
Ans.is'c'i.e.,Indicatedinsicklecelldisease[RefEssentialsof
microbiologyydiep.391]
Pneumococcalpolysaccharidevaccine(PPVorPPSV)isprepared
fromcapsular(notcellwall)polysaccharide.Itisnotgivenbefore2
yearsofage
Itisnotforgeneraluse,butgiveninconditionswhichpredisposeto
pneumococcalinfection,likesicklecellanemia.

924.Allareincludedinsleephygieneexcept
?
a)Healthydiet
b)Sleepingontime
c)Sleepingindarkroom
d)Exercisebeforesleep
CorrectAnswer-D
Ans.is'd'i.e.,Exercisebeforesleep[RefComprehensiveGuide
forsleepp.173]
Sleephygienecanbeconsideredthecornerstoneofenvironmental
modificationandconsistsofdevelopinggoodsleephabitsandan
environmentthatisconducivetosleep.
Sleephygieneincludesthephysicalsleepsetting,sleepschedule
andsleeppractices(e.g.pre-bedtimeroutine),allofwhichinfluence
effectivesleep.
Theprinciplesofsleephygieneinclude?
1. Eatingahealthydiet
2. Limitingamountofcaffeineintake
3. Goingtobedandgettingoutofbedatconsistenttime
4. Sleepingindark,quieteandtemperaturecontrolledrookona
comfortablemattressandpillow.

925.Aproblemvillageisdefinedasifwater
sourceis?
a)>0.5km
b)>1km
c)>1.6km
d)Noneoftheabove
CorrectAnswer-C
Ans.is'c'i.e.,>1.6km[RefPark22"`Ilep.428&21"/ep.418]
Aproblemvillagehasbeendefinedasone
Wherenosourceofsafewaterisavailablewithinadistanceof1.6
km,or
Wherewaterisavailableatadepthofmorethan15meters,or
Wherewatersourcehasexcesssalinity,iron,fluoridesandother
toxicelements,or
Wherewaterisexposedtotheriskofcholera.

926.Nottrueaboutstrategicplanformalaria
control20122017?
a)ObjectiveisAPI<1per10,000
b)50%reductioninmortalityby2017
c)Annualincidence<10per1000by2017
d)Completetreatmenttoatleast80%ofpatients
CorrectAnswer-C
Ans.is'c'i.e.,Annualincidence<10per1000by2017
StrategicactionplanformalariacontrolinIndia:2012-2017
ObjectiveofplanistoachieveAPI<1per10,000populationby
theendof2017.
Goalsofstrategicplanare:

1. Screnningallfevercasessuspectedformalaria(60%through
qualitymicroscopyand40%byrapiddiagnostictest).
2. TreatingallP.falciparumcaseswithfullcourseofeffectiveACTand
primaquine,andallP.vivaxcaseswith3dayschloroquineand14
daysprimaquine.
3. Equippingallhealthinstitutions(PHClevelandabove),especialityin
high-riskareas,withmicroscopyfacilityandRDTforemergencyuse
andinjectableartemisininderivatives.

927.Observationundernursingcarefor24
hoursinanhospitalisdefinedas?
a)Inpatient
b)Outpatient
c)Observationstatuspatient
d)Urgentcarepatient
CorrectAnswer-C
Ans.is'c'i.e.,Observationstatuspatient
Observationstatuspatients:Theseareneitherinpatientsnor
outpatients.Thesepatientsareplacedinahospitalbed(ofteninan
inpatientunit)afterdisplayingsigns/symptomsthatrequireadditional
work-up.Observationalstayisusuallylimitedto24hoursthenthe
physicianmustdeterminewhetherpatient'sconditionwarrants
inpatientaddmissionordischarge.Ifthepatientisdischargeditis
calledobservationstatuspedient,howeverifthepatientisadmitted
tothehospitalthenstatusischangedtoinpatient.
Inpatient:Apatientisadmittedinhospitalroomforanovernightor
morethanthat.
Outpatient:Apatientreceivesadiagnosisand/ortreatmentbut
doesnotstayovernight.

928.UnderRNTCP,DOTSprovidergetshow
muchhonorariumaftercompletionof
treatment?

a)150Rs
b)250Rs
c)500Rs
d)1000Rs
CorrectAnswer-B
Ans.is'bi.e.,250Rs[Refwww.pbnrhm.org]
HonorariumtoDOTproviderforcureorcompletedTBpatient
treatmentis250Rspercare.

929.

Allofthefollowingareareapartofmedial
wallofthemiddleearexcept?

a)Promontory
b)Fenestravestibule
c)Pyramid
d)Subiculum
CorrectAnswer-C
Ans.is'c'i.e.,Pyramid[RefDhingra avep.6]
S
Medialorinnerorlabyrinthicwall(parieslabyrinthica)ofmiddle
ear
Itisformedbylabyrinthandseparatesthemiddleearcavityfrom
internalear.Ithasfollowingfeatures:-Abulgecalledas
promontoryformedbybasalturnofcochlea.
Fenestravestibuli(ovalwindow)
liesposterosuperior(behindand
above)tothepromontoryandopensintoscalavestibuli.
Itisoccupiedbyfootplateofstapesfixedbyannularligament.
Its,sizeonaverageis3.25mmlong&1.75mmwide.
Fenestracochleae(roundwindow)
liesposteroinferiortothe
promontoryandopensintoscalatympaniofcochlea.Itisclosedby
secondarytympanicmembrane.
Theroundwindowisclosestto
ampullaofposteriorsemicircularcanal.Roundwindowisa
triangularopening.Itsdiameterisbetween1.8to2.3mm.

930.Truestatementaboutmalignantotitis
externais:
a)Notpainful
b)Commonindiabeticsandoldage
c)Causedbystreptococcus
d)Alloftheabove
CorrectAnswer-B
Ans.is.B.Commonindiabeticsandoldage

931.Whatistheintensityindecibelofnormal
conversationinhumans?
a)30dB
b)60dB
c)90dB
d)150dB
CorrectAnswer-B
Ans.is'b'i.e.,60dB[RefDhingra5thlep.23]
Intensityisthestrengthofsoundwhichdeterminesitsloudness.Itis
usuallymeasuredindecibels(dB).
Followingareintensitieswhenapersonisatadistanceofonemeter
fromsoundsource.
Whisper
30dB
Normalconversation 60dB
Shout
90dB
Discomfortofear
120dB
Paininear
130dB

932.Ceruminousglandspresentintheear
are:
a)Modifiedeccrineglands
b)Modifiedapocrineglands
c)Mucousgland
d)Modifiedholocrineglands
CorrectAnswer-B
Ans.is.B.Modifiedapocrineglands

933.ApatientwithearcomplaintsshowedpositiveHennebertsign.Whichofthe
followingconditionshowspositiveHennebertsign?
a)Meniere'sdisease
b)Acousticneuroma
c)Neuronitis
d)Glossopharyngealneuralgia
CorrectAnswer-A
Thepresenceofafistulaissuspectedifnystagmusoccursorifthepatientperceives
movementofavisualtargetthatisfixedafterapplyingpositivepressuretotheouterear
canal.
Apositivetestresult(ie,Hennebertsign)suggestseitheraperilymphfistulaor
Meniere'sdisease.

934.LighthousesigninseeninASOMin
whichstage?
a)Stageofsuppuration
b)Stageofhyperaemia
c)Stageofresolution
d)Stageofpre-suppuration
CorrectAnswer-A
InthestageofsupperationofASOM,pusformationoccurs,
henceinthisstagepulsatileotorrheaorlighthousesignin
seen.


935.WhatistheroleofSodiumFluoridein
otosclerosis?
a)Itrestorestheelectrolyteequilibrium
b)IthastensrecoveryoftheOverstimulatedCochlea
c)Itquickensthematurityoftheactivefocusandreduces
osteoclasticresorption
d)Itrepolarizesthecochlearcells
CorrectAnswer-C
Ans.is'c'i.e.,Itquickensthematurityoftheactivefocusand
reducesosteoclasticresorption
[RefCurrentDiagnosisandtreatmentinOtolaryngology2nd/e
p.678;OtosclerosisandStapedectomy:Diagnosis,
Management,andComplicationbyGlasscock(7hieme)1st
(2004)p.61,62]
MechanismofactionofSodiumfluorideinotosclerosis

1. Reducesosteoclasticboneresorptionandincreasesosteoblastic
boneformation:Theseactionsreduceboneremodellinginactively
expandingosteolyticlesionsandpromoterecalcification.
2. Inhibitsproteolyticenzymesthatarecytotoxictothecochlea:
Inhibitionofproteolyicenzymesthatarecytotoxictothecochleais
believedtopreventsensorineuraldeafness.

936.Chemicallabyrinthectomyby
transtympanicrouteisdoneinMeniere's
diseaseusingwhichdrug?

a)Amikacin
b)Gentamicin
c)Amoxycillin
d)Cyclosporine
CorrectAnswer-B
Ans.is'b'i.e.,Gentamicin[RefLevineSC,HaberkampTJ.
Labyrinthectomytocorrectvertigo
Operativetechniquesinotolaryngology-Head&necksurgery.
2001.12:141-143.]
Chemicallabyrinthectomyinmeneiresdiseaseisdoneusing
Gentamicinlocally.

937.Whichofthefollowingtestsisnotused
todifferentiatebetweencochlearand
retrocochlearhearingloss?

a)SISITEST
b)Evokedresponseaudiometry
c)Thresholdtonedecaytest
d)Recruitment
CorrectAnswer-C
Ans.is'c'i.e.,Thresholdtonedecaytest[RefDhingra5th/ep.
31;zr/ep.28]


938.Whendoestherudimentarycochlea
developinthefetus?
a)Firstweek
b)4thto8thweek
c)8thto12thweek
d)16to20thweek
CorrectAnswer-B
Ans.is'b'i.e.,4thto8thweek[RefChap172-Teviewofmedical
embryology-BenPasky]
DevelopmentofInnerear
Afterimplantation,aroundthesecondtothirdweekthedeveloping
embryoconsistsofthreelayers:endoderm,mesoderm
andectoderm.
Thefirstpartoftheeartodevelopistheinnerear,whichbeginsto
formfromtheectodermaroundthe22nddayoftheembryo's
development.Specifically,theinnerearderivesfromtwothickenings
calledoticplacodesoneithersideofthehead.Eachoticplacode
recedesbelowtheectoderm,formsanoticpitandthenanotic
vesicle.Thisentiremasswilleventuallybecomesurroundedby
mesenchymetoformthebonylabyrinth.
Aroundthe33'ddayofdevelopment,thevesiclesbeginto
differentiate.Closertothebackoftheembryo,theyformwhatwill
becometheutricleandsemicircularcanals.Closertothefrontofthe
embryo,thevesiclesdifferentiateintoarudimentarysaccule,which
willeventuallybecomethesacculeandcochlea.
Partofthesacculewilleventuallygiveriseandconnecttothe
cochlearduct.Thisductappearsapproximatelyduringthesixth

weekandconnectstothesacculethroughtheductusreuniens.

939.Otosclerosisaffectswhichbone?
a)Stapes
b)Incus
c)Malleus
d)None
CorrectAnswer-A
Ans.is'a'i.e.,Stapes
Typesofotosclerosis
Followingtypesofotosclerosishavebeendescribed:-
1.Fenesteralorstapedialotosclerosis
Itisthemostcommontype(80-90%)Thelesionstartsjustinfrontof
theovalwindowinanareacalledfissulaantefenestram"and
causesstapesfootplatefixationandconductivehearingloss.
Hearinglossispurelymechanical.
2.Cochlearotosclerosis(Retrofenestralotosclerosis)
Whenpresent,itisalmostalwaysassociatedwithstapedial
(fenestral)otosclerosis.Itinvolvesregionofroundwindoworother
areasintheoticcapsule.Itcausesmixedorsensorineuralhearing
losswhichisbelievedtobetoxicduetodiffusionofcytotoxic
enzymesintothefluidofthemembranouslabyrinth(Incontrastto
stapedialotosclerosis,wherethedeafnessispurelymechanicaland
isconductive).Tinnitusismorecommonincochlearotosclerosis.
3.Histologicotosclerosis
Thistypeofotosclerosisremainsasymptomaticandcausesneither
conductivenorsensorineuralhearingloss.

940.Externalauditorycanalisformedby:
a)1stbranchialgroove
b)1stvisceralpouch
c)2ndbranchialgroove
d)2ndvisceralpouch
CorrectAnswer-A
Ans.is.A.1stbranchialgroove

941.Whichperforationofthetympanic
membraneismostcommonlyseenwith
tubotympanicCSOM?

a)Central
b)Anterosuperior
c)Posterosuperior
d)Posteroinferior
CorrectAnswer-A
Ans.is'a'i.e.,Central[RefDhingra5thlep.77;Pediatric
otolaryngology2"/ep.478]TubotympanicCSOM
Itisalsoknownassafeearasitdoesnotcauseanyserious
complications
Infectionisislimitedtoantero-inferiorpartofmiddleearcleft
(eustachaintube&mesotympanum)andisassociatedwithcentral
perforationinparstensaoftympanicmembrane.

942.Mostcommoncauseofotitisexternais?
a)Fungalinfection
b)Bacterialinfection
c)Seborrheicdisease
d)HerpesZoster
CorrectAnswer-B
Ans.is'b'i.e.,Bacterialinfection[RefClinicalENT5th/ep.223]
Mostcommoncauseofotitisexternaisbacterialinfection.
Twomostcommoncausativebacteriaarestaphylococcusaureus
andpseudomonas

943.Whichofthefollowingisacauseof
objectivetinnitus?
a)ImpactedWax
b)Carotidarteryaneurysm
c)Meniere'sdisease
d)Ototoxicdrugs
CorrectAnswer-B
Ans.is'b'i.e.,Carotidarteryaneurysm[RefiDhingraSth/ep.
145;ScottBrown's7th/eVol-3p.4029-4030;Tulip.125-126]


944.Pulsatiletinnitusisafeatureof?
a)Glomustumour
b)acousticneuroma
c)malignantotitsexterna
d)meneire'sdisease
CorrectAnswer-A
Ans.is'a'i.e.,Glomustumour[RefLoganTurner10th/ep.214]
Theearliestsymptomsofglomustumourispulsatiletinnitus
(earliest)andhearingloss.Hearinglossisconductiveandslowly
progressive.Thesearefollowedbybloodstainedotorrhoeaand
earache.

945.Histelberger'ssignisseenin?
a)Acousticneurom
b)GlomusTumour
c)Nasalangiofibroma
d)Acutesuppurativeotitismedia
CorrectAnswer-A
Ans.is'a'i.e.,Acousticneurom[RefActaOtorhinolaryngol
Belg.1987;41(1):40-8.TheHitselbergersignasaperception
phenomenon.BenzB,BaumgartenD.]
Hitselberger'ssign
InAcousticneuroma-lossofsensationinthepostero-superiorpart
ofexternalauditorymeatussuppliedbyArnold'snerve(branchof
Vagusnervetoear).

946.Ethmoidalinfundibulumliesbetween?
a)Bullaethmoidalisanduncinateprocessofethmoid
b)Middleandinferiorturbinate
c)HiatussemilunarisandInferiormeatus
d)Wingofsphenoidandmaxillaryantrum
CorrectAnswer-A
Ans.is'a'i.e.,Bullaethmoidalisanduncinateprocessof
ethmoid
Thehiatussemilunarisisboundedinferiorlybythesharpconcave
marginoftheuncinateprocessoftheethmoidbone,andleadsintoa
curvedchannel,theinfundibulum,boundedabovebythebulla
ethmoidalisandbelowbythelateralsurfaceoftheuncinateprocess
oftheethmoid.

947.Whatliesbetweenthemiddleand
inferiorturbinate?
a)Middlemeatus
b)Superiormeatus
c)Hiatussemmilunaris
d)Inferiormeatus
CorrectAnswer-A
Ans.is'a'i.e.,Middlemeatus[RefScottBrown7'Vevol2p.
1329]
Superiormeatus*Belowsuperiorturbinate(betweensuperior
andmiddleturbinates)
MiddlemeatusBelowmiddleturbinate(betweenmiddleand
inferiorturbinates)
InferiormeatusBelowinferiorturbinate.

948.

Thenarrowestpartofthenasalcavityis?
a)Internalnasalvalve
b)Antrochoanalregion
c)1stnasalturbinate
d)Regionofinferiorconcha
CorrectAnswer-A
Ans.is'a'i.e.,Internalnasalvalve[RefJafekBW.Anatomyand
physiologyofthenose.JafekBW,StarkAK,eds.ENTSecrets.
Philadelphia,Pa:Hanley&Belfus;1996.77-83.]
Theinternalnasalvalveinvolvestheareaboundedbyupperlateral
cartilage,septum,nasalfloor,andanteriorheadoftheinferior
turbinate.Thismakesupthenarrowestportionofthenasalairway

949.

Narrowestpartofthenasalcavityis?
a)Vestibule
b)Choanae
c)Inferiorturbinate
d)Middleturbinate
CorrectAnswer-C
Ans.is'c'i.e.,Inferiorturbinate[RefHeidariZ,Mahmoudzadeh-
SaghebH,KhammarT,KhammarM(May2009).
"Anthropometricmeasurementsoftheexternalnosein18-25-
year-oldSistaniandBaluchaboriginewomeninthesoutheast
ofIran".FoliaMorphol.(Warsz)68(2):88-92]
Theinternalnasalvalvecomprisestheareaboundedbytheupper
lateral-cartilage,theseptum,thenasalfloor,andtheanteriorheadof
theinferiorturbinate.Inthenarrow(leptorrhine)nose,thisisthe
narrowestportionofthenasalairway.

950.Saddlenosedeformityisseenin?
a)PrimarySyphilis
b)Secondarysyphilis
c)Tertiarysyphilis
d)LupusVulgaris
CorrectAnswer-C
Ans.is'c'i.e.,Tertiarysyphilis[RefDhingra5th/ep.184]
Syphilisofnoseoccursas:-

1. Primary:-Rareandmanifestsasprimarychancreofthevestibule.
2. Secondary:-Rarelyrecognizedandmanifestsassimplerhinitis,
crustingandfissuring.
3. Tertiary:-Thisisthestageinwhichnoseisinvolvedmost
commonly.Thereisformationofgummafollowedbybony
perforationofnasalseptum.Bridgeofthenosecollapsescausinga
saddledeformity.Atrophicrhinitismayoccurasacomplication.

951.Allofthefollowingaretrueabout
vasomotorrhinitisexcept?
a)Itisallergic
b)Itisduetoparasympatheticoveractivity
c)Resistantcasesmayneedcryotherapy
d)Itmayleadtohypertrophicrhinitis
CorrectAnswer-A
Ans.is'a'Itisallergic[RefScott'sBrown7thiep.2122]
Vasomotorrhinitis
Vasomotorrhinitisisanonallergicconditionthatinvolvesaconstant
runnynose,sneezingandnasalcongestion,i.e.,thenoseisstuffyor
runnyforreasonsotherthanallergiesandinfections.Theexact
etiologyisunknown,buttriggersincludeemotions,odors,poorair
quality,spicyfoods,andmedicationsideeffects.

952.Allofthefollowingaretrueabout
vasomotorrhinitisexcept?
a)Itisanonallergiccondition
b)itisduetoparasympatheticoveractivity
c)Itmaycauseparoxysmalepisodesofsneezing
d)Itisaninfectivecondition
CorrectAnswer-D
Ans.is'd'i.e.,Itisaninfectivecondition[RefDhingra5th/ep.
170]


953.Inevaluationofacaseofimmotilenasal
cilia,whichofthefollowing
investigationsshouldproveuseful?

a)Rhinogram
b)Xraynasalandparanasalsinuses
c)Sweatsodiumlevels
d)Nitricoxidetest
CorrectAnswer-D
Ans.is'd'i.e.,Nitricoxidetest[RefLeighMW,PittmanJE,
CarsonJL,FerkolTW,DellSD,DavisSD.Clinicalandgenetic
aspectsofprimaryciliarydyskinesia/Kartagenersyndrome.
GenetMed.2009Jul.11(7):473-87]
Screeningtestsforimmotileciliasyndrome
Nitricoxide:Measuringexhalednasalnitricoxide,whichismostly
reducedinprimaryciliarydyskinesia,isagoodscreeningtestfor
immotile-ciliasyndromewithagoodnegativepredictivevalue.
Studieshavedemonstratedarelationshipbetweennasalnitricoxide
levels,nasaloxidesynthasemRNAexpression,andciliarybeat
frequency.Thereisalsoasignificantinversecorrelationbetweenthe
degreeofaplasiaand/orhypoplasiaoftheparanasalsinusand
nasalnitricoxidevaluesinprimaryciliarydyskinesiapatients.

954.Allofthefollowingarefeaturesof
ethmoidalpolypexcept?
a)Commoninadults
b)CommonlySingular
c)Commonlybilateral
d)Isusuallyallergic
CorrectAnswer-B
Ans.is`b'i.e.,CommonlySingular[RefDhingra5thlep.187;Tuli
1"/ep.173]
Antrochoanalpolypgrowsbackward(posteriorly),therefore,itmay
notbevisibleonanteriorrhinoscopy,whileposteriorrhinoscopy
showssmooth,greyishwhite,sphericalmassinchoana.
Ontheotherhand,ethmoidalpolypsgrowanteriorly.Therefore,they
arebestseenonanteriorrhinoscopyandmaynotbeseenon
posteriorrhinoscopy.

955.Allofthefollowingaretreatmentsof
multiplebilateralethmoidalpolyps
except?

a)Functionalendoscopicsinussurgery
b)Intranasalethmoidectomy
c)Extranasalethmoidectomy
d)CaldwllLucSurgery
CorrectAnswer-D
Ans.is'd'i.e.,CaldwilLucSurgery[RefScottBrown,Vol-IL
p.1701;Dhingra5`"/ep.188,430]
Surgicaltreatmentofethmoidalpolyps:

1. Functionalendoscopicsinussurgery(FESS):Surgeryofchoice
whenavailable.
2. Polypectomy:Whenthereareoneortwopedunculatedpolyps.
3. Intranasalethmoidectomy:Indicatedwhenpolypsaremultipleand
sessile.
4. Extranasalethmoidectomy:Thisisindicatedwhenpolypsrecur
afterintranasalprocedures.
5. Transantralethmoidectomy:Indicatedwheninfectionandpolypoidal
changesarealsoseeninthemaxillaryantrum.Inthiscaseantrumis
openedbycaldwell-Lucapproachandtheethmoidalaircells
approachedthroughthemedialwalloftheantrum.

956.Whatisthetreatmentofchoicefor
ethmoidalpolyps?
a)FunctionalEndoscopicsinussurgerywithpolypectomy
b)Intranasalethmoidectomy
c)Extranaslethmoidectomy
d)Transantralethmoidectomy
CorrectAnswer-A
Ans.is'a'i.e.,FunctionalEndoscopicsinussurgerywith
polypectomy[RefSeeaboveexplanation]


957.
Thearterywhichleadstobleedingin
Woodruff'sareais?
a)Anteriorethmoidalartery
b)Sphenopalatineartery
c)Greaterpalatineartery
d)Superiorlabialartery
CorrectAnswer-B
Ans.is'b'i.e.,Sphenopalatineartery[RefScottBrown7h/eVol-
2p.1597;Dhingra5th/ep.190]
Woodruff'sarea:?
Itissituatedundertheposteriorendofinferiorturbinate.
Sphenopalatinearteryanastomoseswithposteriorpharyngealartery
here.Posteriorepistaxisoccursinthisarea.

958.Intrathecalfluoresceinwithendoscopic
visualizationisusefulindiagnosisof?
a)Deviatednasalseptum
b)Multipleethmoidalpolyps
c)RhinitisMedicamentosa
d)CSFRhinorrhoea
CorrectAnswer-D
Ans.is'd'i.e.,CSFRhinorrhoea[Ref:LoganTurner10(h/ep.
28,Dhingra5th/ep.179]
DetectionofsiteofCSFleakinCSFRhinorrhoea

1. HRCT:-HRCTwithorwithoutgodoliniumisthemosthelpfulstudy
foridentifyingthesiteofleak,i.e.,investigationofchoice.
2. MRI:-MRIwithheavyT2weightedimagemayhighlightCSF
sufficientlytoshowtheleak.
3. CTcisternography:-CTscanafterinjectionofcontrastdyeis
effectiveinpatientswithanactiveleak.
4. Radioisotopecisternography:-Radioisotopeinjectedintrathecally
andmeasured.
5. Fluorescindye:-Intrathecalfluorescinwithendoscopic
visualization.

959.Allofthefollowingarefeaturesofa
nasalforeignbodyexcept?
a)Foulsmellingdischarge
b)Epistaxis
c)Nasalobstruction
d)Septalperforation
CorrectAnswer-D
Ans.is'd'i.e.,Septalperforation[RefDhingraSaVep.176;
LoganTurner10'h/ep.63]
Foreignbodyinnose
Anasalforeignbodyisanythingthatgetsstuckinsidethenose.
Inanimateforeignbody(object)ismorecommonthananimate
foreignbody.Commonobjectsfoundinnosesincludefoodmaterial
(peas,beans,nuts),tissuepaper,beads,toysandrocks.Animate
foreignbodiesareworms,larvaeormaggots.Mostcasesofforeign
bodiesinthenoseandnasalcavityarenotseriousandoccurin
toddlersandchildrenfrom1-8years.Becausechildrendevelopthe
abilitytopickupobjectsatabouttheageof9months,thisproblem
ismuchlesscommonbefore9monthsofage.

960.Allofthefollowingarefeaturesofa
nasalforeignbodyexcept?
a)Vestibulitis
b)Epistaxis
c)Nasalobstruction
d)Deviatedseptum
CorrectAnswer-D
Ans.is'd'i.e.,Deviatedseptum[RefDhingra5thiep.176;Logan
Turner10`"/ep.63]


961.

Allofthefollowingarecausesofperforation
ofcartilaginouspartofnasalseptumexcept?

a)Tuberculosis
b)Leprosy
c)Lupus
d)Syphilis
CorrectAnswer-D
Ans.is'd'i.e.,Syphilis[RefDhingra5th/ep.166;Scott-Brown's
Otolaryngology7th/eVol-2chapter-124p.1583]
Anasalseptalperforationisathrough-and-throughdefectinany
portionofthecartilaginousorbonyseptumwithnooverlying
mucoperichondriumormucoperiosteumoneitherside.
Perforationofbonypart:-Syphilis
Perforationofcartilaginouspart:-TB,leprosy,lupus.

962.Allofthefollowingaretrueaboutnasal
myiasisexcept?
a)Commoninvasomotorrhinitis
b)Intensenasalirritationpresent
c)Meningitismayoccurinseveredisease
d)Chloroformwaterisoneofthemodesoftreatment
CorrectAnswer-A
Ans.is'a'i.e.,Commoninvasomotorrhinitis[RefDhingra5th/e
p.178]


963.ETHMOIDALBULLAEareseenin?
a)Posteriorethmoidalaircells
b)Middleethmoidaircells
c)Superiorethmoidalaircells
d)Inferiorethmoidalaircells
CorrectAnswer-B
Ans.is'b'i.e.,Middleethmoidaircells[Ref:LoganTurner10th
lep.
379;Dhingra5th/ep.153,154]


964.Whatisthetypeofepitheliumofthe
adenoid?
a)Pseudostratifiedciliatedcolumnarepithelium
b)Nonkeratinizedsquamousepithelium
c)Cuboidalepithelium
d)Columnarepitheliumwithgobletcells
CorrectAnswer-A
Ans.is'a'i.e.,Pseudostratifiedciliatedcolumnarepithelium
[RefWiatrakBJ,WoolleyAL.Pharyngitisandadenotonsillar
disease.CummingsCW,FredricksonJM,HarkerLA,Crause
CJ,SchullerDE,RichardsonMA.OtolaryngologyHeadand
NeckSurgery...V/e.London:Mosby;1998.188-215.]
Theadenoidiscoveredbyapseudostratifiedciliatedcolumnar
epitheliumthatisplicatedtoformnumeroussurfacefoldsThe
nasopharyngealepitheliumlinesaseriesofmucosalfolds,around
whichthelymphoidparenchymaisorganizedintofolliclesandis
subdividedinto4lobesbyconnectivetissueseptaSeromucous
glandsliewithintheconnectivetissue,andtheirductsextend
throughtheparenchymaandreachthenasopharyngealsurface.

965.Allofthefollowingarefeaturesof
enlargedadenoidsexcept?
a)Otitismedia
b)Nasalobstruction
c)Failuretothriveofchild
d)Esophagitis
CorrectAnswer-D
Ans.is'd'i.e.,Esophagitis[RefDhingraViep.254]
Clinicalfeaturesofenlargedadenoids
Enlargedandinfectedadenoidsmaycausenasal,aural(ear),
andgeneralsymptoms.
1)Nasalsymptoms
Nasalobstructionisthecommonestsymptom.Thisresultinmouth
breathing.Astherespirationandfeedingcannottakeplace
simultaneously,childfailstothrive.
Othersymptomsarenasaldischarge(wetbubblynose),sinusitis,
epistaxisandtonelessvoicewithlossofnasalquality(Rhinololia
clausa).
2)Auralsymptoms
Tubalobstruction
Otitismedia:-Recurrentacuteotitismedia,CSOM,serousotitis
media
3)Generalsymptoms
Adenoidfacies:Elongatedfacewithdullexpression,open
mouth,prominent&crowdedupperteeth,hitchedupperlip,pinched
inappearanceofnoseandhigharchedpalate.Pulmonary
hypertension,aprosexia(lackofconcentration).

966.Anteriortonsillarpillarisformedby?
a)Palatopharyngealfold
b)Palatoglossalfold
c)Pterygopalatinearch
d)Valleculae
CorrectAnswer-B
Ans.is'b'i.e.,Palatoglossalfold
Anteriortonsillarpillar-Palatoglossalfold
Posteriortonsillarpillar-Palatopharyngealfold

967.Oneofthefollowingistrueregarding
Zenkerdiverticulum?
a)Itisapulsiondiverticulum
b)Itprojectsanteriorly
c)Commonlyseeninyoungmales
d)Itisbetweensuperiorandmiddleconstrictor
CorrectAnswer-A
Ans.is'A'i.e.,Itisapulsiondiverticulum
HypopharyngealdiverticulumorZenker'sdiverticulumor
pharyngealpouch
Itisapulsiondiverticulumwherepharyngealmucosaherniates
throughtheKillian'sdehiscence,aweakareabetweentwopartsof
theinferiorconstrictor.
Itisthemostcommonesophagealdiverticulum.
Thediverticulaariseposteriorlyinthemidlineofneck.Themouthof
thediverticulaisinthemidlinebutprojectslaterally(usuallyleft
laterally)
Zenker'sdiverticulaarerarelyseenbelow30yrsofage,most
patientsareover50.

968.Allofthefollowingareextrinsic
laryngealmembranesexcept?
a)Quadrangularmembrane
b)Hyoepiglotticligament
c)Cricotrachealmembrane
d)Thyrohyoidmembrane
CorrectAnswer-A
Ans.is'a'i.e.,Quadrangularmembrane
[RefMeratiAL,BielamowiczSA.TextbookofLaryngology.San
Diego:PluralPublishingInc2006.]
Quadrangularmembrane

1. Aryepiglotticligament(superiorborderofmembrane)
2. Vestibularligament(Inferiorborderofmembrane

969.Allofthefollowingaretrueabout
SpasmoidicDysphoniaexcept?
a)Itmaybeofadductororabductortype
b)AbductortypeischaracterizedbyWhisperingqualityofvoice
c)AdductortypeischaracterizedbyBreathlessness
d)ItisfocalLaryngealdystonia
CorrectAnswer-C
Ans.is'c'i.e.,Adductortypeischaracterizedby
Breathlessness
[RefiSulicaL(December2004)."Contemporarymanagementof
spasmodicdysphonia".CurrentOpinioninOtolaryngology&
HeadandNeckSurgery.12(6)]


970.Whichofthefollowingcancersdonot
presentwithcervicallymphnode
involvement?

a)GlotticCancer
b)SubglotticCancer
c)Papillarythyroidcancer
d)Oralcancer
CorrectAnswer-A
Ans.is'a'i.e.,GlotticCancer[RefDhingra5thlep.327;41'p.
283]
Truevocalcordsaredevoidoflymphatic,hencelesschanceof
cervicalnodalmetastasis

971.Whichofthefollowingisnota
premalignantconditionoralcancer?
a)Leukoplakia
b)Erythroplakia
c)Oralsubmucousfibrosis
d)SystemicSclerosis
CorrectAnswer-D
Ans.is'd'i.e.,SystemicSclerosis[RefDevita7"Vep.982;
Bailey
&love25`Vep.735]
Lesionswithincreasedriskofmalignancyinoralcancer
Premalignantcondition:-Leukoplakia,Erythroplakia,Speckled
erythroplakia,chronichyperplasticcandidiasis.
Conditionsincreasingrisk:-Oralsubmucousfibrosis,syphilitic
glossitis,sideropenicdysphagia(PetersonKellysyndrome).
Riskisdoubtful:-Orallichenplanus,DLE,Dyskeratosiscongenita.

972.Whichisthenarrowestportionofthe
esophagus?
a)Atthecricopharyngealsphincter
b)Atthecrossingoftheleftmainbronchus
c)Attheleveloftheaorticarch
d)Atthediagphragmaticaperture
CorrectAnswer-A
Ans.is'a'i.e.,Atthecricopharyngealsphincter
[GrayH.Chapter35:Mediastinum.StandringS,ed.Gray's
Anatomy:TheAnatomicalBasisofClinicalPractice.40`VeNew
York,NY:ChurchillLivingstoneElsevier;2008.939-57.]
Cricopharyngealsphincteristhenarrowestportionofesophagus

973.Allofthefollowingareindicationsfor
tracheostomyexcept?
a)Comaafterheadinjury
b)Maxillofacialinjury
c)Bilateralabductorpalsy
d)Superiorlaryngealnervepalsy
CorrectAnswer-D
Ans.is'd'i.e.,Superiorlaryngealnervepalsy[RefAjay
Yadavzithiep.215;Dhingra5thiep.337]
Tracheostomy
Atracheostomyisasurgicalproceduretocreateanopeningthrough
theneckintotheanteriorwalloftrachea.
Atubeisusuallyplacedthroughthisopeningtoprovideanairway
andtoremovesecretionsfromthelungs.Thetubeis
calledtracheostomytube.

974.InCaldwellLucoperation,theapproach
isthroughthe?
a)SublabialApproachleadingtoopeningofmandibularantrum
b)Throughthesphenopalatinerecess
c)Openingofmaxillaryantrumthroughgingivolabialapproach
d)Superiormeatus
CorrectAnswer-C
Ans.is'c'i.e.,Openingofmaxillaryantrumthrough
gingivolabialapproach[RefDhingraStilep.422]
Caldwell-Lucoperationisaprocessofopeningthemaxillaryantrum
throughcaninefossabysublabialapproachanddealingwiththe
pathologyinsidetheantrum.Inthissurgery,antrumisreached
throughaincisioningingivolabialsulcus(fromlateralincisorto
2"dmolar)andthenopeningofantruminthisarea.

975.CaldwellLucSurgeryhasitsapproachto
themaxillaryantrumthrough?
a)Gingivolabialsulcus
b)Inferiororbitalrim
c)Nasalseptum
d)Cribriformplate
CorrectAnswer-A
Ans.is'a'i.e.,Gingivolabialsulcus[RefDhingra5thIep.422]

976.

Submucosalresectionisthetreatmentof
choiceof?
a)DNSinadults
b)DNSinchildren
c)Sluder'sNeuralgia
d)Nasalpolyp
CorrectAnswer-A
Ans.is'a'i.e.,DNSinadults[RefDhingra5th/ep.423]
Submucousresection(SMR)isasurgicalproceduretocorrectthe
deformityofnasalseptum.
Theprincipleofthisprocedureistoremovedeviatedcartilageand
bonefrombeneaththemucosalliningofnasalseptum,leavinga
correctedseptumlargelycomposedofscartissue.
Submucousresectionisnotadvocatedinchildrenupto17yearsof
ageasitmayinterferewithdevelopmentofthefacialbones.

977.Allofthefollowingareremovedin
verticalhemilaryngectomyexcept?
a)HalfGlottis
b)HalfSupraglottis
c)Halftongue
d)HalfSubglottis
CorrectAnswer-C
Ans.is'c'i.e.,Halftongue[RefDhingra4th/ep.284;
http://128.255.52.245/oto/Beta/database/contents]
Verticalhemilaryngectomymeansexcisionofonehalfofthelarynx
ononeside,i.e.,verticalhalfisremovedwhichincludeverticalhalf
ofsupraglottis,glottisandsubglottis.
Horizontalhemilaryngectomyistheexcisionofsupraglottisonly,
alsoknownassupraglotticlaryngectomy.

978.Allofthefollowingarecomplicationsof
maxillarysinuslavageandinsufflation
except?

a)Airembolism
b)Orbitalinjury
c)Epistaxis
d)Facialnerveinjury
CorrectAnswer-D
Ans.is'd'i.e.,Facialnerveinjury[RefSchlemmerKD,Naidoo
SK.Complicatedsinusitisinadevelopingcountry,a
retrospectivereview.IntJPediatrOtorhinolaryngol.2013May
17]
ComplicationofMaxillarysinuslavageandinsufflation
Complicationsofnonendoscopicdrainageprocedurescanbeminor
orsevere.
Themostcommoncomplicationisfailuretoenterthesinusbecause
ofimproperpositioningofthetrocar,incompletepenetrationofthe
sinusmucosa,orthepresenceofahypoplasticantrum.
Epistaxismayoccurbecauseoflacerationofthenasalmucosaor
preexistingcoagulopathiesnecessitatingpacking.
Severecomplicationsincludeorbitalinjury,airembolism,anddeath
secondarytoinjectionofairintothesinus.

979.Miraclefruitisusedtochangethetaste
from?
a)SourtoBitter
b)SourtoSweet
c)Bittertosweet
d)Saltytosweet
CorrectAnswer-B
Ans.is'b'i.e.,SourtoSweet[RefPeterHanelt,ed.
(2001).Mansfeld'sencyclopediaofagriculturalandhorticultural
crops2.Springer.p.1660.ISBN3-540-41017-1.]
MiracleFruitcontainsaglycoproteincalledmiraculin,whichbindsto
thetongue'stastebudswhenthefruitisconsumed.
Miraculinactsasasweetnessinducerwhenitcomesincontactwith
acids,causingsourfoodstotastesweet,temporarily.

980.Themainmuscleaffectedincongenital
musculartorticollisis?
a)Sternocleidomastoid
b)Trapezius
c)ScalenusAnticus
d)Omohyoid
CorrectAnswer-A
Ans.is'a'i.e.,Sternocleidomastoid
[Ref:Cooperman,DanielR.(1997).Karmel-Ross,Karen,ed."The
DifferentialDiagnosisofTorticollisinChildren".Physical&
OccupationalTherapyinPediatrics.17(2):1-11]
Thecauseofcongenitalmusculartorticollisisunclear.Birthtrauma
orintrauterinemalpositionisconsideredtobethecauseofdamage
tothesternocleidomastoidmuscleintheneck.Otheralterationsto
themuscletissuearisefromrepetitivemicrotraumawithinthewomb
orasuddenchangeinthecalciumconcentrationinthebodywhich
causesaprolongedperiodofmusclecontraction.

981.Whereistheauditorycortexlocated
insidethebrain?
a)Superiortemporalgyrus
b)Inferiortemporalgyrus
c)Area3,1,2
d)Cingulategyrus
CorrectAnswer-A
Ans.is'a'i.e.,Superiortemporalgyrus[RefPickles,
James
0.(2012).AnIntroductiontothePhysiologyofHearing(4th
ed.).Bingley,UK:EmeraldGroupPublishingLimited,pp.215-217.]
PrimaryAuditoryCortexislocatedbilaterally,roughlyattheupper
sidesofthetemporallobes-inhumansonthesuperiortemporal
plane,withinthelateralfissureandcomprisingpartsofHeschl's
gyrusandthesuperiortemporalgyrus,includingplanumpolareand
planumtemporale(roughlyBrodmannareas41,42,andpartially
22).

982.Allofthefollowingarefeaturesof
TubotympanicCSOMexcept?
a)Profusedischarge
b)Hearingloss
c)Extremepain
d)Sometimesparadoxicalimprovementinhearingisseen
CorrectAnswer-C
Ans.is'c'i.e.,Extremepain[RefDhingra5th/ep.77;Pediatric
otolaryngology2"/ep.478]
ClinicalfeaturesoftubotympanicCSOM

Profusemucopurulentdischargewhichisnotfoulsmelling,i.e.,non-
foulsmellingdischarge
Hearingloss(conductivetype).Ifsensorineuralcomponentalso
occurs(i.e.,mixedtype),itarousesthesuspicionoftoxicdeafness.
Sometimes,patientreportsaparadoxicaleffect,i.e.,hearsbetterin
thepresenceofdischargethanwhentheearisdry.Thisisdueto
roundwindowshieldingeffectproducedbydischargewhichhelpsto
maintainphasedifferential.
Thereisnopain,ifitoccursitisduetoassociatedotitisexternanot
duetootitismedia.
Sincetheinfectedareaisopenatbothends,dischargedoesnot
accumulateinthemiddleearcavity
Ossicularchainismostlyuninvolved,ifinvolvedonlylongprocessof
incusisinvolved.

983.FeaturesofUsher'sSyndromeincludeall
except?
a)NightBlindness
b)VisualImpairment
c)MultipleNeurofibromas
d)Hearingdeficit
CorrectAnswer-C
Ans.is'c'i.e.,MultipleNeurofibromas[RefMetsMB,YoungNM,
PassA,LaskyJB(2000)."EarlydiagnosisofUshersyndromein
children".TransactionsoftheAmericanOphthalmological
Society.98:237-45.]
Ushersyndrome
Ushersyndromeisarelativelyraregeneticdisordercausedbya
mutationinanyoneofatleast11genesresultinginacombination
ofhearinglossandvisualimpairment,andisaleadingcauseof
deafblindness.Ushersyndromeisincurableatpresent.
OthernamesforUshersyndromeincludeHallgrensyndrome,
Usher-Hallgrensyndrome,retinitispigmentosadysacusissyndrome,
anddystrophiaretinaedysacusissyndrome.
Thissyndromeischaracterizedbyhearinglossandagradualvisual
impairment.Thehearinglossiscausedbyadefectiveinnerear,
whereasthevisionlossresultsfromretinitispigmentosa(RP),a
degenerationoftheretinalcells.Usually,therodcellsoftheretina
areaffectedfirst,leadingtoearlynightblindnessandthegradual
lossofperipheralvision.Inothercases,earlydegenerationofthe
conecellsinthemaculaoccurs,leadingtoalossofcentralacuity.In
somecases,thefovealvisionisspared,leadingto"doughnut
vision";centralandperipheralvisionareintact,butanannulusexists

aroundthecentralregioninwhichvisionisimpaired.

984.Whicheyemusclehasradial,
longitudinalandcircularfibres?
a)SphinctorPupillae
b)DilatorPupillae
c)LevatorpalpebraeSuperioris
d)Ciliarymuscle
CorrectAnswer-D
Ans.D.Ciliarymuscle
Theciliarymuscleisaringofsmoothmuscleintheeye'smiddle
layer(vascularlayer)thatcontrolsaccommodationforviewing
objectsatvaryingdistancesandregulatestheflowofaqueous
humourintoSchlemm'scanal.
Itchangestheshapeofthelenswithintheeye.
Theciliaryfibershavecircular,longitudinal(meridional)andradial
orientations.

985.Inprimaryopen-angleglaucoma
pilocarpineeyedropslowersthe
intraocularpressurebyitsdirectaction
onthe:

a)Trabecularmeshwork
b)Ciliaryepithelium
c)Longitudinalfibresoftheciliarymuscle
d)Alloftheabove
CorrectAnswer-D
Ans.D
Pilocarpineaseyedropsitisusedtomanageangleclosure
glaucoma
untilsurgerycanbeperformed,ocularhypertension,
primaryopenangleglaucoma,andtobringaboutconstrictionof
thepupilfollowingitsdilation.
MechanismofAction:
Pilocarpineisusedasamioticandinthetreatmentof
glaucoma
Pilocarpine
contractstheciliarymuscle,causingincreasedtension
onthescleralspurandopeningofthetrabecularmeshworkspaces
tofacilitateoutflowofaqueoushumor.Outflowresistance
isreduced,loweringintraocularpressure(IOP).

986.Axiallengthofeyeballis:
March2005
a)16mm
b)20mm
c)24mm
d)28mm
CorrectAnswer-C
Ans.C:24mm
Theeyeofthenewbornishypermetropicandtheaverageaxial
lengthisabout18mm.Attheageof3yearsitis23mmandfromage
3-14yearsitincreasesby1mm.
Intheemmetropiceye,theaxiallength(fromtheposteriorcorneal
surfacetotheretina)variesfrom22to26millimeters.Inthe
emmetropiceye(whichhasnorefractiveerror),therangeofcorneal
refractingpowerisbetween39and48diopters,whiletherangeof
lenticularrefractingpowerisbetween15and24diopters.

987.RequiredforIOLpowercalculation?
a)Cornealtopography
b)Gonioscopy
c)IndirectOphthalmoscopy
d)Keratometry
CorrectAnswer-D
Ans,D.Keratometry
IOLpowercalculationrequireskeratometryandbiometry(axial
lengthofeyeball).

988.Sturm'sconoidreferstoconfigurationof
theraysrefractedthrough:
a)Concavesphericalsurface
b)Convexsphericalsurface
c)Toricsurface
d)Irregularsurface
CorrectAnswer-C
Ans.Toricsurface
SturmSConoid
Itisanopticalconditioninwhichtherefractivepoweroflensor
corneaisnotthesameinallmeridians,hencetwofocalpoints
separatedbyafocalintervalareformedwhichiscalledtheSturm's
Conoid.

989.Trueaboutimagingspectrometryis?
a)Allowssimultaneousmeasurementsofreflectancespectra
alongaline
b)Isusefulindiagnosingagainsttheruleastigmatism
c)Spectrometryisanessentialinvestigationbefore
trabeculectomy
d)ItisbasedontheprincipleofSturm'sConoid
CorrectAnswer-A
Ans.A.Allowssimultaneousmeasurementsofreflectance
spectraalongaline
'ImagingsPectrometryisanewtechniquethatpermitssimultaneous
measurementsofreflectancespectraatdifferentlocationsalonga
line.
Resultsarethree-dimensionalimages,whosecoordinatesare
location,wavelength,andreflectance.
Aconventionalfunduscameraisadaptedtoaspectrographandan
intensifiedchargecoupleddevice(CCD)matrixdetectorsystem.
ConsideringtheradiationtransPortinsinglefunduslayers,thelocal
distributionoftheconcentration-thicknessproductofxanthophyll,
melanin,andchoroidalbloodcanbecalculated'

990.Refractoryerrormeasuredbyallexcept
?
a)Keratometry
b)Retinoscopy
c)Refractometry
d)Spectrometry
CorrectAnswer-D
Ans.D.Spectrometry
Theprocedureofdeterminingandcorrectingrefractiveerrors
istermedasreftaction.Therefractioncomprisestwo
complementarymethods:-

1. Objectivemethods:Objectivemethodsofrefractioninclude:-(i)
Retinoscopy,(ii)Refractometry,(iii)Keratometry,
2. Subjectivemethods:Theseare:-(i)Subjectiveverificationof
refraction,(ii)Subjectiverefiningofrefraction,(iii)Subjective
binocularbalancing.

991.Ansiokoniais?
a)Projectionofdifferentcolouredimagesintovisualcortex
b)Projectionofdifferentshapedimagesintovisualcortexoftwo
retinae
c)Changeinthevelocityofperceivedobjects
d)Partialintermittentvisualloss
CorrectAnswer-B
Ans.B.Projectionofdifferentshapedimagesintovisualcortex
oftworetinae
Anisoeikoniaisdefinedasaconditionwhereintheimagesprojected
onthevisualcortexfromthetworetinaeareabnormallyunequalin
sizeorshape-

992.Mostpowerfulrefractorysurfaceofeye
is-
a)Conjunctiva
b)Cornea
c)Vitreous
d)Lens
CorrectAnswer-B
Ans.B.Cornea
Thecornea,withtheanteriorchamberandlens,refractslight,with
thecorneaaccountingforapproximatelytwo-thirdsoftheeye'stotal
opticalpower.
Inhumans,therefractivepowerofthecorneaisapproximately43
dioptres.
Thedioptericpowerofreducedeyeis+5OD,ofwhich+44Dis
contributedbycorneaand+16Dbythecrystallinelens.
Totaldiaoptericpowerofschematiceyeis+58D,ofwhichcornea
contributes+43Dandthelens+15D.

993.Strabismicamblyopiaismorecommon
inpatientswith:
a)Intermittentsquint
b)Alternatesquint
c)Constantsquint
d)Latentsquint
CorrectAnswer-C
Ans.Constantsquint

994.Astigmatismisdefinedas?
a)Refractoryerrorwhereinrefractionvariesalongdifferent
meridians
b)RefractoryerrorduetolongAPlengthofeyeball
c)Varyingrefractoryerrorinbotheyes
d)VaryingShapeperceptionbybotheyes
CorrectAnswer-A
Ans.A.Refractoryerrorwhereinrefractionvariesalong
differentmeridians
Astigmatismisatypeofrefractiveerrorwhereintherefractionvaries
inthedifferentmeridia.
Consequently,theraysoflightenteringintheeyecannotconverge
toapointfocusbutformfocallines.
Therefractiveerroroftheastigmaticeyestemsfromadifferencein
degreeofcurvaturerefractionofthetwodifferentmeridians(i.e.,the
eyehasdifferentfocalpointindifferentplanes).
Forexample,theimagemaybeclearlyfocusedonretinainthe
horizontalplane,butnotintheverticalplane.
Themostcommoncauseofastigmatismisabnormalityofcorneal
curvature.
Otherlesscommoncausearelenticular(curvatureabnormalityof
lens,obliquepositionoflens)andretinal(obliqueplacementof
macula).



995.Whatisregularastigmatism?
a)Astigmatisminwhichtheprincipalmeridiansareparallel
b)Astigmatisminwhichtheprincipalmeridiansareperpendicular
c)Asymptomaticastigmatism
d)Astigmatismasaresultofcataractsurgery
CorrectAnswer-B
RegularstigmatismPrincipalmeridiansarependicular
IrregularastigmatismPrincipalmeridiansarenot
perpendicular.


996. Anteroposteriorchangeinlengthofeye
iscalled?
a)Anisokonia
b)Curvaturalanisotropia
c)AxialAmetropia
d)Emmetropia
CorrectAnswer-C
Ans.C.AxialAmetropia
Ametropia(aconditionofrefractiveerror)isdefinedasastateof
refraction,whereintheparallelraysoflightcomingfrominfinityare
notfocusedonretina,ratherfocusedeitherinfront(inmyopia)or
behind(inhypermetropia)thesensitivelayerofretina.
AxialAmetropia:
Itisthecommonestformofametropia(bothmyopiaand
hypermetropia).
Inhypermetropia,thereisanaxialshorteningofeyeball.
So,imageisformedbehindtheretina.
Inmyopia,thereisanaxiallengtheningofeyeball.
So,imageisformedinfrontoftheretina.
1mmchangeinaxiallengthleadstoametropiaof3D.
Forexample1mmshorteninginaxiallengthcauseshypermetropia
of3D.

997.Inacaseofmyopia,LASIKwillprovide
correctionupto?
a)20D
b)12D
c)6D
d)4D
CorrectAnswer-B
Ans.B.12D
SurgicalProcedure
Myopiacorrection
RadialKeratomy
-2to-6D
Photorefractivekeraotomy 2to6D
LASIK
Upto-12D
Extractionoflens
-16to-18D
PhakicIOL
>-12D
Intercornealring(ICR)
1-6D

998.Whatisreversehypopyon?
a)Collectionofpusinthevireous
b)Collectionofemulsifiedsiliconoilinanteriorchamber
c)Abscessintheorbit
d)Seenincornealulcerclosetobeingruptured
CorrectAnswer-B
Ans,B.Collectionofemulsifiedsiliconoilinanteriorchamber
Reversehypopyon
Collectionofsiliconoilintheanteriorchambermayleadtothe
appearanceofareversehypopyon.
Thisisduetotheemulsifiedoilbeinglessdenserthantheaqueous
layer

999.Pterygiumallaretrueexcept:
a)Arisefromanypartofconjunctiva
b)Cancauseastigmatism
c)Surgeryistreatmentofchoice
d)UVexposureisriskfactor
CorrectAnswer-A
Ans.Arisefromanypartofconjunctiva

1000.Silkretinaisseenin?
a)Hypermetropia
b)Myopia
c)Astigmatism
d)Presbyopia
CorrectAnswer-A
Ans.A.Hypermetropia
Clinicalfindingsofhypermetropia

1. Smalleyeballandcornea
2. Shallowanteriorchamber
3. Fundusshowspseudopapillitisandshotsilkappearance.
4. Degenerativeretinoschisis

1001.Pseudopapilitiswithsilkshot
appearanceisseenin?
a)Hypermetropia
b)Myopia
c)Astigmatism
d)Presbyopia
CorrectAnswer-A
Ans.A.Hypermetropia
Clinicalfindingsofhypermetropia

1. Smalleyeballandcornea
2. Shallowanteriorchamber
3. Fundusshowspseudopapillitisandshotsilkappearance.
4. Degenerativeretinoschisis

1002.1mmchangeaxiallengthoftheeyeball
wouldchangetherefractingpowerof
theeyeby?

a)1D
b)2D
c)3D
d)4D
CorrectAnswer-C
Ans.C.3D
1mmchangeinaxiallengthleadstoametropiaof3D.
ForexampleImmshorteninginaxiallengthcauseshypermetropia
of3D.

1003.Lensometerdetects?
a)Correctpowerofapairofglasses
b)Cornealtopography
c)Biochemicalconstitutionoflens
d)PowerofIOL
CorrectAnswer-A
Ans.A.Correctpowerofapairofglasses
Lensometer
Alensmeterorlensometer,alsoknownasafocimeterorvertometer,
isanophthalmicinstrument.
Itismainlyusedbyoptometristsandopticianstoverifythecorrect
prescriptioninapairofeyeglasses,toproperlyorientandmark
uncutlenses,andtoconfirmthecorrectmountingoflensesin
spectacleframes.
Lensmeterscanalsoverifrthepowerofcontactlenses,ifaspecial
lenssupportisused.

1004.Allofthefollowingaretreatmentsof
myopiaexcept?
a)LASIK
b)Phakicintraocularlens
c)RadialKeratotomy
d)Holmiumlaserthermoplasty
CorrectAnswer-D
Ans.D.Holmiumlaserthermoplasty
Refractivesurgeriesformyopia
Radialkeratotomy
Laserinsitukeratomileusis(LASIK)
Phakicintraocularlens(IOL)
Orthokeratology
Photorefractivekeratotomy(PRK)
Extractionoflens(Fucala'soperation)
Intercornealringimplantation

1005.Indexmyopiaisseenin?
a)Nuclearcataracts
b)Chorioretinitis
c)Choroidalmelanoma
d)Posterioruveitis
CorrectAnswer-A
Ans.A.Nuclearcataracts
Nuclearchangesofaginginduceamodificationofrefractiveindexof
lensandproduceanindexmyopia.
'Nuclearcataractscauseageneraldecreaseinthetransperancyof
thelensnucleus.Theyareassociatedwithindexmyopia"

1006.Suddenpainlesslossofvision-Allare
causesexcept?
a)CRAO
b)CSR
c)Acutecongestiveglaucoma
d)VitereousHemorrhage
CorrectAnswer-C
Ans.C.Acutecongestiveglaucoma

1007.Allaretrueregardingcorneaexcept:
a)Endotheliumhelpinmaintainingdehydratedstate
b)Oxygenismostlyderivedbyepitheliumdirectlyfromtheair
throughtearfilm
c)Glucosesupplyforcornealmetabolismismainlyderivedfrom
theaqueous
d)Cornealthicknessismoreatcenterthanperiphery
e)Richlyvascular
CorrectAnswer-D:E
Answer-(D)Cornealthicknessismoreatcenterthanperiphery
(E)Richlyvascular
Outer&fibrouscoatofEYEBALL.
Transparent,anterior1/6thsegmentofeyeball.
Non-vascular
Mostoftherefractionineyeoccuratanteriorsurfaceofcornea(air-
tearinterface),i.e.,Anteriorsurfaceofcorneaisthemostimportant
refractivestructureofeye.
Themostactivelymetabolisinglayersofthecorneaareepithelium&
endothelium.

1008.AllofthefollowingarecausesCrystal
keratopathyexcept?
a)Cystinosis
b)Schnyder'sDystrophy
c)Bietti'sDystrophy
d)Diabetes
CorrectAnswer-D
Ans.D.Diabetes
CausesofCrystallineKeratopathy:-Infections,Schnydercorneal
dystrophy,Bietticorneoretinaldystrophy,Cystinosis,
Lymphoproliferativedisorders,Medication-induced.

1009.Cornealtattooingmaybedone
with:
a)Goldchloride
b)Calciumchloride
c)Coppersulfate
d)Potassiumpermanganate
CorrectAnswer-A
Ans.Goldchloride

1010.Subconjunctivalhemorrhagesare
evidentinthefollowingcasesexcept?
a)Whoopingcough
b)Scurvy
c)Purpura
d)Pellagra
CorrectAnswer-D
Ans.is'd'i.e.,Pellagra
Subconjunctivalhaemorrhage
Occursduetotheruptureofsmallvessels.
Thecondition,thoughunsightly,istrivial.Thiscanoccur
spontaneouslyinelderlypeoplewithfragilevesselsorthosewith
systolichypertensionorafterlocaloculartraumaoreyesurgery.
Veryminuteecchymoses,orpossiblythromboses,areseenin
severeconjunctivitis;largeextravasationsaccompanysevere
straining,especiallyinoldpeople,asonliftingheavyweightsor
vomiting.
Theyarenotinfrequentlyseeninchildrenwithwhoopingcoughand
mayoccurinscurvy,blooddiseasessuchaspurpura,orinmalaria.
Recurrentsubconjunctivalhaemorrhageswarrantinvestigationsfor
ableedingdiathesesorleukaemia.Thedifferentialdiagnosis
includesKaposisarcoma.
Moreseriousarethelargesub-conjunctivalecchymoseswhichseep
forwardsfromthefonixfollowingheadinjuries.Theyareduetoan
extravasationofbloodalongtheflooroftheorbit,secondarytoa
fractureofthebaseoftheskull.
Infracturesofthesphenoidthebloodappearslateronthetemporal
sidethanelsewhere.

Haemorrhagesalsoresultfromsevereorprolongedpressureonthe
thoraxandabdomen,asinpersonssqueezedinacrowdorby
machinery.
Thebloodgraduallychangescolourandgetsabsorbedin1to3
weekswithouttreatment.
Theuseofaspirinandnon-steroidalanti-inflammatorydrugs
(NSAIDs)shouldbeavoidedandifmildocularirritationispresent,
artificialtearscanbeprescribedfourtosixtimesaday.

1011.KayserFleischerringisfoundinwhichlayerofcornea?
a)Bowman'sCapsule
b)Substantiapropria
c)Descemet'smembrane
d)Endothelium
CorrectAnswer-C
Kayser-Fleischerringstaketheformofacrescenticrusty-browndiscolorationofthe
deepestlayerofthecornea(Descemetmembrane).Inthepurelyhepaticstageofthe
disease,theringsmaynotbeevident(in25percentofcases),buttheyarevirtuallyalways
present(ifproperlysought)oncetheneurologicsignsmanifests.Aslit-lampexamination
maybenecessaryfortheirearlydetection,particularlyinbrown-eyedpatients,butinthe
majorityofpatientswithneurologicsignstheringscanbevisualizedwiththenakedeyeor
withtheaidofanindirectophthalmoscopefocusedonthelimbus.
AlsoKnow:
Kayser-Fleischerrings
areasignofWilson'sdisease,whichinvolvesabnormalcopper
handlingbytheliverresultingincopperaccumulationinthebodyandischaracterisedby
abnormalitiesofthebasalgangliaofthebrain,livercirrhosis,splenomegaly,involuntary
movements,musclerigidity,psychiatricdisturbances,dystoniaanddysphagia.The
combinationofneurologicalsymptoms,alowbloodceruloplasminlevelandKF
ringsisdiagnosticofWilson'sdisease.

Ref:RopperA.H.,SamuelsM.A.(2009).Chapter37.InheritedMetabolicDiseasesofthe
NervousSystem.InA.H.Ropper,M.A.Samuels(Eds),AdamsandVictor'sPrinciplesof
Neurology,9e.

1012.Kayser-Fleischerrings(KFrings)areseenin:
a)Pterygium
b)Hematochromatosis
c)Wilson'sdisease
d)Menke'skinkedhairsyndrome
CorrectAnswer-C
Wilson'sdisease

1013.Posteriorstaphylomaisseenin:
September2005
a)Myopia
b)Hypermetropia
c)Astigmatism
d)Presbyopia
CorrectAnswer-A
Ans.A:Myopia
Staphylomaisthetermgiventoaneyewhosesclero-uvealcoats
arestretched(alsoknownasectasia).Thismostcommonlyoccurs
posteriorly,athoughanteriorstaphylomaalsoisrecognised.
Posteriorstaphyloma
Progressivemyopia(ormegamyope)mostcommoncause.
Glaucoma
Scleritis
Necrotizinginfection
Surgery/trauma
Radiotherapy
Posteriorstaphylomaaffectstheposteriorpoleoftheeyeandis
linedbythechoroid.
Theectaticportionisnotvisibleexternallybutcanbedetectedby
fundoscopyandB-scanUSG.
Anteriorstaphyloma
Themostcommoncauseforanteriorstaphylomaissloughing
cornealulcerwinchperforatesandhealswiththeformationofa
pseudocorneabytheorganizationofexudatesandlayingdownof
fibroustissue.
Differentialdiagnoses

Buphthalmos(congenitalglaucoma)
Axialmyopia
Macrophthalmos:seeninneurofibromatosistypel(NF1)
Coloboma

1014.

A30yearoldmanpresentstotheclinicwithpainintheeye,watering,rednessand
photophobia.Examinationofhiseyesshowscircumcornealcongestionandkeratic
precipitates.

Assertion:KPsareproteinaceousdepositsoccurringinatriangularfashioninthe
inferiorpartofcornea.

Reason:MuttonfatKPsareseeningranulomatousiridocyclitisandiscomposedof
epitheloidcellsandmacrophages.

a)BothAssertionandReasonaretrue,andReasonisthecorrect
explanationforAssertion
b)BothAssertionandReasonaretrue,andReasonisnotthe
correctexplanationforAssertion
c)Assertionistrue,butReasonisfalse
d)Assertionisfalsebutreasonistrue
CorrectAnswer-B
Keraticprecipitatesareproteinaceousdepositsoccurringinatriangularfashioninthe
inferiorpartofcorneaduetoconvectioncurrentsintheaqueoushumor.MuttonfatKPsare
seeningranulomatousiridocyclitisandiscomposedofepithelioidcellsandmacrophages.
Theyareusually10-15innumber.
Ref:ComprehensiveOphthalmologybyAKKhurana,4thEdition,Page142

1015.Scleritisismostcommonlyassociated
with:
March2005

a)Diabetes
b)Osteoarthritis
c)Rheumatoidarthritis
d)Hypertension
CorrectAnswer-C
Ans.C:Rheumatoidarthritis
Scleritisisasevere,destructive,chronic,painful,andpotentially
blindinginflammatorydiseaseoftheConjunctiva,Scleraand
Episcleratissues.
Symptomsincludesrednessandsevereeyepain,whichmayradiate
toadjacentareas,theforehead,cheek,orbehindtheeye.Thisis
usuallyassociatedwithlightsensitivity,teary,andinsomecases,
reducedorpoorvision.Theaffectedeyeoftenhasabluishhueor
becomesanintensepurple.
Thereareseveraldifferentsub-typesofScleritis
NodularScleritisischaracterizedbyafocalareaofinflammation,
immovable,andtender,inflamednodulesontheeye.
DiffuseScleritis
DiffuseAnteriorScleritisisthemostcommontype,andis
characterizedbywidespreadinflammationoftheAnteriorportionof
theSclera,thewhiteoftheeye.TheDiffusetypeofScleritisis,
fortunately,themostbenignformofScleritisandthemost
responsivetotherapy.
NecrotizingScleritisislikelytheworstformofthedisease,

sometimesleadingtolossoftheeyefrommultiplecomplications,
severepain,oroccasionallyperforationoftheglobe.Itisoften
associatedwithseveresystemicdiseaseandinvolvementofmultiple
organs.Anassociatedtypeofvascularinflammation,called
Vasculitis,maythreatenthelivesofthosepatientsafflicted.Pain
withthisconditionisusuallyextreme,anddamagetotheSclerais
oftenmarked.NecrotizingscleritisalsoknownasScleromalacia
perforansischaracterizedbyseverethinningoftheScleraofthe
Eye,allowingforlocaloutpouchingsoftheunderlyingdarkUveal
tissue.Therearelargeabnormalbloodvesselscrossingareasof
Scleralloss.Theconditionoccursinanotherwisewhiteand"quiet"
Eye,withoutpain.ThistypeofScleritisisassociatedwithsevere
RheumatoidArthritis,occasionallyseeninWegener's
GranulomatosisandRelapsingPolychondritis.
PosteriorScleritisisquiterare,butusuallypresentswithpooror
doublevision,severepain,proptosis(forwarddisplacementofthe
eye),Uveitis(inflammationinsidetheUveaTract),andlimitationof
eyemovement.AnexudativeRetinaldetachment(fluidunderthe
Retina)maycauseseverevisualloss,Angle-ClosureGlaucoma
fromChoroidaleffusion.
About50%ofScleritispatientsareassociatedwithsystemic
autoimmunedisordersincludingrheumatoidarthritis,gout,wegener
granulomatosis,RelapsingPolychondritis,SystemicLupus
Erythematosus,PolyarteritisNodosa,AnkylosingSpondylitis,with
infections,orchemicalorphysicalinjuries.
Itoccursmostofteninpeoplebetweentheagesof30and60(itis
rareinchildren).Scleritismaybetheinitialoronlypresentingclinical
manifestationofthesepotentiallylethaldisorders.

1016.Unilateralfrontalblisterswithupperlid
edemawithconjunctivitisisseenin?
a)AcanthamoebaKeratits
b)HerpesSimplex
c)HerpesZosterOphthalmicus
d)NeuroparalyticKeratitis
CorrectAnswer-C
Ans.C.HerpesZosterOphthalmicus
Ocularlesions:Combinationof2ormoreofthefollowingwith
subsidenceofskineruptions:-
i)Conjunctivitis(Mostcommonocularlesion)
ii)Zosterkeratitis:
Punctatekeratitis,Microdendriticcornealulcer(Pseudodendritic
Keratitis.),nummularanteriorstromalkeratitis,Disciformkeratitis,
neuroparalyticulceration,exposurekeratitis,mucousplaque
keratitis,Keratouveitiswithendothelitis,Sclerokeratitis(least
common).
Theendotheliumisafavouredsiteofattackandacuteendothelial
celllossoccursduringherpeszosterkeratouveitis.
iii)Episcleritis/Scleritis,Iridocyclitis(Uveitis).
iv)Acuteretinalnecrosis,anteriorsegmentnecrosis,phthisisbulbi.
v)Secondaryglaucoma.

1017.Whichorganismcanpeneteratecorneal
endothelium?
a)Aspergillusfumigatus
b)StaphylococcusAureus
c)NeisseriaeGonorrhae
d)Hemophilusinfluenza
CorrectAnswer-A
Ans.A.Aspergillusfumigatus
Fungi-Canpenetrateintactcornealendothelium
Neisseriasp,C.Diptheriae,H,Aegyptus,Listeria-Canpenetrate
intactcornealepithelium

1018.AllofthefollowingaretrueaboutKeratoconus,except:
a)IncreasedcurvatureofcorneaAstigmatism
b)Astigmatism
c)K.Fringcornea
d)Thickcornea
CorrectAnswer-D
ThemainpathologicalchangesinKeratoconusarethinningand
ectasiaofthelenswhichoccurasaresultofdefectivesynthesisof
mucopolysaccharideandcollagentissue.

1019.Allofthefollowingaretruefor
sympatheticophthalmitisexcept:
a)Affectstheinjuredeye
b)Mostlyresultsfromapenetratingwound
c)Autoimmuneetiology
d)DalenFuch'snodulesmaybeseen
CorrectAnswer-A
Ans.A:Affectstheinjuredeye
Sympatheticophthalmitisisaconditioninwhichserious
inflammationattacksthesoundeyeafterinjurytotheothereye.

1020.Redkeraticprecipitatesareseenin?
a)Granulomatousuveits
b)Hemorrhagicuveitis
c)Oldhealeduveitis
d)Acuteanterioruveitis
CorrectAnswer-B
Ans.is.bi.e.,Hemorrhagicuveitis
Keraticprecipitates(KPs)
KPsareproteinaceouscellulardepositsoccurringatthebackof
cornea(cornealendothelialdeposits).Keraticprecipitatesare
formedbytheaggregationofpolymorphonuclearcells,lymphocytes,
andepitheloidcells.Inthesettingofuveitis,thebimicrosopic
appearanceofKPmayyeildimportantdiagnosticcluesforthe
identificationoftheunderlyinginflammatorydisorder:?
MuttonfatKP:-Large,yellowishKPs,arecharacteristic
ofgranulomatousuveitis.Thesearecomposedofepitheloidcellsand
macrophages.Theyarelarge,thickfluffy,lardaceousKPs,havinga
greasyorwaxyappearance.
SmallormediumKPs(granularKPs):-Thesearecomposedof
lymphocytesandarecharacteristicofnon-granulomatousuveitis.
Thesearesmall,roundandwhitishprecipitates
RedKPs:-ComposedofRBCsandinflammatorycells.Theseare
seeninhemorrhagicuveitis.
OldKPs:-Inhealeduveitis.TheabovedescribedKPsshrink,fade,
becomepigmentedandirregularinshapewithcrenatedmargins.

1021.Thezonulessuspendingthelensare
attachedtothe?
a)Rootofiris
b)Ciliarybody
c)Anteriorvitreous
d)Limbus
CorrectAnswer-B
Ans.B.Ciliarybody
Theciliaryzonules(Zonulesofzinnorsuspensoryligamentsoflens)
holdthelensinpositionandenabletheciliarymuscletoactonit.
Theseconsistessentiallyofaseriesoffibreswhichrunfromthe
ciliarybodyandfuseintotheouterlayerofthelenscapsulearound
theequatorialzone.

1022.Lensattachedtociliarybodyvia?
a)Limbus
b)Zonules
c)VitreousHumour
d)Rootofiris
CorrectAnswer-B
Ans.B.Zonules

1023.Cataractiscausedby?
a)Hypoparathyroidism
b)Cigratesmoking
c)Non-ionizingradiation
d)Alloftheabove
CorrectAnswer-D
Ans.is'd'i.e.,Alloftheabove

1024.Allofthefollowingarecausesof
posteriorsubcapsularcataractexcept-
a)Myotonicdystrophy
b)Wilson'sDisease
c)Ionizingradiation
d)Congenitalcataract
CorrectAnswer-D
Answer-D.Congenitalcataract
Myotoicdystrophy
Wilson'sdisease
Atopicdermatitis
Corticosteroids
Trauma
Galactosemia
Infrared/heatcataract(glass-blower'sorglassworker)

1025.Theintraocularlensincataractsurgery
isplacedin?
a)Surfaceofiris
b)Capsularbag
c)Overthefaceofviterous
d)Aroundthelimbus
CorrectAnswer-B
Ans.B.Capsularbag
Asaclikestructureremainingwithintheeyefollowingextracapsular
cataractextractionorPhacoemulsification.
Theimplantedintraocularlensisplacedwithinthisstructureto
recreatetheusualphakicstate.

1026.Nonfoldablelensismadeof-
a)Silicon
b)Acrylic
c)PMMA
d)Hydrogel
CorrectAnswer-C
Ans.C.PMMA
TypesofIOL
Dependingonthematerialofmanufacturing,followingtypesofIOLs
arethere
1. RigidIOLs:-Madeentirelyfrompolyrnethylmethacrylate(PMMA).
2. FoldableIOLs:-Areusedafterphacoemulsificationandaremadeof
silicon,acrylic,hydrogelandcollamer.
3. RollableIOIs:Ultra-thinIOLsandaresuedafterphokonittechnique
(micro-incision:1mm).
Thesearemadeofhydrogel.

1027.Rigidgaspermeablelensaremadeof-
a)Plymethymethacrylate
b)Hydroxymethylmethacrylate
c)CopolymerofPMMA,Siliconcontainingmonomer&cellulose
acetylbutyrate
d)CelluloseacetaeButyrate
CorrectAnswer-C
1.Hardlenses:
Madeofpokymethylmethacrylate(PMMA)
2.Rigidgaspermeable(RGP)lenses:
CopolymerofPMMA,siliconcontainingvinylmonomer&cellulose
acetatebutyrate(CAB)areusedtomanufactureRGPlenses.
3.Softlenses:
Thesearemadeupofhydroxymethymethacrylate(HEMA)

1028.Axiallengthofeyeballismeasuredby
?
a)AmodeUltrasonography
b)BmodeUltrasonography
c)MmodeUltrasonography
d)Bothaandb
CorrectAnswer-A
Ans.is'a'i.e.,AmodeUltrasonography
UltrasonographvinOphthalmology
AMode
Transduceriscoupleddirectlytotheeyethroughtheuseofmethyl
cellulose.
Recordingaredoneinagraphicmode.Heightoftherecordedspike
onverticalaxisisameasureoftheamplitudeoftheecho,the
positionofthespikealongthehorizontalaxisindicatesthearrivalof
theechoonthetranducer.
Diagnosisisbasedonthebasisofamplitude,position,extentand
movementoftheabnormalechoesalonwiththesoundattenuating
propertiesoftheabnormality.
Ithasaspecialroleinbiometryi.e.axiallengthwhichisvery
essentialforsurgicalplanning.Bmode
Thetransduceriscoupledtotheeyebyeitherthegelappliedtothe
closedlidorbyasalinebath
Isisadisplayoftwo-dimensionalcrosssectionalimages.
TheechoesinBscanaredisplayedasspotsandthebrightnessof
echoesindicatesitsamplitude.

1029.Snowflakecataractisassociatedwith:
September2008
a)Hypertension
b)Adultdiabetes
c)Trauma
d)Juvenilediabetes
CorrectAnswer-D
Ans.D:Juvenilediabetes
SnowflakeCataracts:Thistypeofcataractrepresentsdotsof
varioussizes(andsometimesvariouscolours)distributed
throughoutthecortexsurroundingthenucleusfor360?.
ThisisthemostcommoncataractseeninDownSyndrome(extra
copyofchromosome21alsocalledtrisomy21)andjuvenile
diabetesmellitus.
Theseusuallyhaveverylittleeffectonvision.
Adultdiabeticcataractshowscorticaland/ornuclearand/or
subcapsular(sameasagerelated)

1030.WhatisthetypeofGalactosemia
cataract?
a)Snowflake
b)Oildrop
c)Bluedot
d)Polychromaticlustre
CorrectAnswer-B
Ans,B.Oildrop

1031.Rosettecataractisseendueto:
a)Trauma
b)Copperforeignbody
c)Diabetes
d)Hyperparathyroidism
CorrectAnswer-A
Ans.Trauma

1032.Vossiusringoccursin
a)Lensdislocation
b)Concussioninjury
c)Penetratinginjury
d)Extracapsularextraction
CorrectAnswer-B
Bi.e.Concussioninjury

1033.Mostcommonetiolopathogeneticcause
ofglaucomais:
March2012

a)Raisedpressureinepiscleriticveins
b)Decreasedoutflow
c)Increasedformationofaqueoushumour
d)Increasedscleraloutflow
CorrectAnswer-B
Ans:Bi.e.Decreasedoutflow
Asustainedincreaseinintraocularpressuremaybedueto
increasedformationoftheaqueoushumour,difficultyinitsexit,ora
raisedpressureintheepiscleralveinsofthese,thefirst&lastrarely
occur,anditfollowsthatraisedintraocularpressureisessentially
duetoanincreasedresistancetothecirculationoftheaqueousat
thepupiland/oritsdrainagethroughtheangleoftheanterior
chamber.

1034.Feature(s)ofInfantileglaucomais/are
except:
a)Aniridiamaybeassociated
b)Treatmentincludestrabeculotomy
c)Buphthalmoscanoccur
d)Corneaisthin&clear
e)MaybeassociatedwithSturge-webersyndrome
CorrectAnswer-D
Answer-D.Corneaisthin&clear
Answer-D.Corneaisthin&clear
Primaryinfantileglaucomaisararedevelopmentaldefectinthe
iridocornealfiltrationangleoftheanteriorchamberthat
preventsaqueousfluidfromproperlydrainingfromtheeye.
Thisobstructionincreasestheintraocularpressure,which,if
untreated,damagestheopticnerve.Infantileglaucomacan
causecompleteblindnessifleftuntreated.
Glaucomacanalsooccurininfantsaftertraumaorintraocular
surgery(eg,cataractextraction).Glaucomaassociatedwith
anotheroculardisorder,suchasaniridia,Lowesyndrome,
orSturge-Webersyndrome,iscalledsecondaryglaucoma.
Buphthalmos(blueandthinsclera,strechedlimbus)isseen.
Treatment:Trabeculotomy,Goniotomy,combined
trabeculotomyandtrabeculotomy+/-MMC(Mitomycin)


1035.Whatisnottrueaboutcongenital
glaucomaofeye?
a)Photophobiaismostcommonsymptom
b)Haab'sStriaemaybeseen
c)Thinandbluescleraseen
d)Anteriorchamberisshallow
CorrectAnswer-A
Ans.A.Photophobiaismostcommonsymptom

1036.Descemetmembranebreachisseenin
?
a)Angleclosureglaucoma
b)Buphthalmos
c)AcuteIridocyclitis
d)Subconjunctivalhemorrhage
CorrectAnswer-B
Ans.B.Buphthalmos
Haabstriae
arediscretecornealopacitiesappearaslineswith
doublecontourduetoruptureinDescemet'smembrane.

1037.Earliestsignofprimarycongenital
glaucoma?
a)Cornealedemawithwatering
b)Haab'sStriae
c)Bluesclera
d)Myopia
CorrectAnswer-A
Ans.,A.Cornealedemawithwatering
Mostcommonsymptom-Watering(lacrimation)
2dmostcommonsymptom-Photophobia
Mosttroublesomesymptom-Photophobia(Childavoidslight)
Firstsign-Cornealedemawithwatering

1038.Selectivealpha2agoinstusedin
glaucoma?
a)Tirriolol
b)Epinephrine
c)Dipivefrine
d)Brimonidine
CorrectAnswer-D
Ans.is'd'i.e.,Brimonidine

1039.Latanoprostusedtopicallyinglaucoma
primarilyactsby?
a)Decreasingaqueoushumorformation
b)Increasinguveoscleraloutflow
c)Releasingpupillaryblock
d)Increasingtrabecularoutflow
CorrectAnswer-B
Ans.is'b'i.e.,Increasinguveoscleraloutflow
Ans.is'b'i.e.,Increasinguveoscleraloutflow
LatanoprostisananalogofprostaglandinF2athatincreases
uveoscleraloutflowandinducesmiosis.
AntiglaucomaDrugs:MechanismofLoweringIntraocular
Pressure(IOP):
Drugsthatincreasetrabecularoutflow
?Miotics(e.g.,pilocarpine)
?Epinephrine,dipivefrine
?Bimatoprost
Drugsthatincreaseuveoscleraloutflow
?Prostaglandins(latanoprost)
?Epinephrine,dipivefrine
?Brimonidine
?Apraclonidine
Drugsthatdecreaseaqueousproduction
?Carbonicanhydraseinhibitors(e.g.,acetazolamide,dorzolamide)
?Alphareceptorstimulatorsinciliaryprocess(e.g.,epinephrine,
dipivefrine,clonidine,brimonidine,apraclonidine)
?Beta-blockers
Hyperosmoticagents-(e.g.,glycerol,mannitol,urea).


1040.Mioitcsaretreatmentofchoicefor?
a)Angleclosureglaucoma
b)Openangleglaucoma
c)Buphthalmos
d)SympatheticOphthalmia
CorrectAnswer-A
Ans.A.Angleclosureglaucoma

1041.Whichofthefollowingdrugisalpha2
agonist?
a)Apraclonidine
b)Timolol
c)PGanalogues
d)Verampamil
CorrectAnswer-A
Ans.A.Apraclonidine
Apraclonidine?Selectivealpha2agonist.
Usefulinglaucoma.
Alphaagonists(Brimonidineandapraclonidine)arecontraindicated
inhypertensivecrisis.
Dipivefrinisaprodrugwhichisconvertedintoepinephrineinsidettre
eyeUall,socansafelybeusedinhypertension.

1042.
Whatisthecauseofglaucomain
retinoblastoma?
a)Blockgeoftrabecularnetwork
b)Neovascularisation
c)Masseffectofthetumour
d)Lysisofthelens
CorrectAnswer-B
Ans.B.Neovascularisation
Retinoblastomaisacauseofneovascularglaucoma,

1043.Glaucomadrainagedevices?
a)Drainaqueoushumourtotheposteriorsegment
b)Drainaqueoushumourtoanexternaldevice
c)Openthetrabeculaemechanically
d)Reducetheaqueoussecretionbycompressingtheciliary
epithelium
CorrectAnswer-B
Ans.B.Drainaqueoushumourtoanexternaldevice
Glaucomadrainagedevicesaredesignedtodivertaqueoushumor
fromtheanteriorchambertoanexternalreservoir,whereafibrous
capsuleformsabout4-6weeksaftersurgeryandregulatesflow.
Thesedeviceshaveshownsuccessincontrollingintraocular
pressure(IOP)ineyeswithpreviouslyfailedtrabeculectomyandin
eyeswithinsufficientconjunctivabecauseofscarringfromprior
surgicalproceduresorinjuries.
Theyalsohaveshownsuccessincomplicatedglaucomas,suchas
uveiticglaucoma,neovascularglaucoma,andpediatricand
developmentalglaucomas,amongothers.

1044.A44yearoldwomanpresentsSudden
painlesslossofvisionwithhistoryof
previoussimilarepisodefundoscopy
showsnoglow.Whatcouldbethe
possiblediagnosis?

a)VitreousHemorrhage
b)RhegmatogenousRetinalDetachment
c)Acutecongestiveglaucoma
d)FungalKeratitis
CorrectAnswer-A
Ans.,A.VitreousHemorrhage
Vitreoushemorrhagereferstobleedingintothevitreouschamberor
aspacecreatedbyvitreousdetachment.
Patientspresentwithsuddenonsetoffloaters(blackspotsinfrontof
theeye)wherethehemorrhageissmall,andtheremaybesudden
painlesslossofvisionifthehemorrhageislarge.

1045.Allofthefollowingaretruefor
retinopathyofprematurityexcept:
a)Occursinprematureinfantsduetolatecrying
b)Duetohypoxiathereoccursneovascularizationfollowedby
fibroproliferation
c)Endresultisbilateralblindness
d)Blindnesscanbepreventedbyearlydiagnosisandablationof
vascularprematureretinawithcryotherapyorphotocoagulation
CorrectAnswer-A
Ans.Occursinprematureinfantsduetolatecrying

1046.Retinopathyofprematurityis
commonlypredisposedby-
a)Lessgestationage
b)Lowbirthweight
c)O2toxicity
d)Carbohydrateexcess
CorrectAnswer-A
Ans.is'a'i.e.,Lessgestationage

1047.Ealesdiseaseis:
a)Recurrentopticneuritis
b)Recurrentpappilloedema
c)Recurrentperiphelbitisretinae
d)None
CorrectAnswer-C
Ans.Recurrentperiphelbitisretinae

1048.WhatisfalseaboutEale'sdisease
amongstthefollowwing?
a)Retinaldetachmentmayoccur
b)AKTisgiven
c)Opticneuritis
d)VitreousHemorrhage
CorrectAnswer-C
Ans.C.Opticneuritis

1049.Snowballappearanceisseenin?
a)Posterioruveitis
b)Sarcoidosis
c)Anterioruveitis
d)VitreousHemorrhage
CorrectAnswer-B
Ans.B.Sarcoidosis
Causesofintermediateuveitis,thusSnowballOpacitiesof
Vitreous?
Candidiasis
TB,syphilis
Sarcoidosis
Multiplesclerosis
Lymedisease.

1050.Prutchnersretinopathyinassociated
with-
a)DiabetesMellitus
b)Wilson'sdisease
c)Headtrauma
d)Rheumatoidarthritis
CorrectAnswer-C
Ans.C.Headtrauma
Purtscher'sretinopathy
Near-confluentcotton-woolspotsclusteredaroundanotherwise
normalopticnerveheadinaneyeofapatientwhohadsustaineda
severebluntinjurytotheheadandchest.
Later,itwasdiscoveredtobeassociatedwithseveralnon-traumatic
systemicdiseases.
Conditionsassociated:
Severehead,chest&longbonediseases.
Fatembolismsyndrome.
Amnioticfluidembolism
Actuepancreatitits.
SLE

1051.Subhyaloidhemorrhageis?
a)Boatshaped
b)Crescentshaped
c)Round
d)Flameshaped
CorrectAnswer-A
Ans,.A.Boatshaped
Retrohyaloid(subhyaloid)hemorrhage('Boat-shapedorscaphoid):-
Locatedanterior(internal)totheretina,withintheretrohyoidspace.
(note-thetermhyaloidreferstohyaloidbody,anothernamefor
vitreoushumor).

1052.Whatistrueaboutretinalhemorrhage
innewborn?
a)Morecommonininstrumenteddeliveries
b)Resolvein6-8months
c)Commonlyunilateral
d)Associatedwithintrauterineinfection
CorrectAnswer-A
Ans.A.Morecommonininstrumenteddeliveries
Birth-rdatedRHininfantsoccursinone-quarterofnormaldeliveries
andarefarmorecommonafterinstrumentaldeliveries.
Commonlybilateral,theywerepredominantlyintraretinal,posterior,
resolvedrapidly,andveryrarelypersistedbeyond6weeks.

1053.Mostcommonagerelatedchangein
vitreous?
a)Anteriorvitreousdetachment
b)Posteriorvitreousdetachment
c)Vitreoushemorrhage
d)Vitritis
CorrectAnswer-B
Ans.B.Posteriorvitreousdetachment
YanoffWrites(Mostcommonagerelatedeventinvitreousis
posteriorvitreousdetachment'

1054.Muscaevolitantesisseenin?
a)Vitreousdetachment
b)VitreousHemorrhage
c)Remainsofprimitivehyaloidvasculatur
d)Eale'sdisease
CorrectAnswer-C
Ans,C.Remainsofprimitivehyaloidvasculatur
Muscaevolitantesarephysiologicalvitreousopacitiesandrepresent
theresiduesofprimitivehyaloidvasculature.

1055.Allaretrueregardingopticneuritis
except:
a)Decreasedvisualacuity
b)Decreasedpupillaryreflex
c)Abnormalelectroretinogram
d)Abnormalvisualevokedresponseretinogram
CorrectAnswer-C
Ci.e.Abnormalelectroretinogram
Electroretinogramindicatestheactivityofretinal(esp.rods&
cones)functionandhasnoroleinassessingthefunctionalintegrity
oftheopticnerveQ.
Soitcan'tbeabnormalinopticneuritis.

1056.AnisocoriainHorner'ssyndromeisdue
to
a)Oculosympatheticpalsy
b)Oculoparasympatheticpalsy
c)Oculomotornervepalsy
d)Abducensnervepalsy
CorrectAnswer-A
Answer-A.Oculosympatheticpalsy
Anisocoriaisaconditioncharacterizedbyanunequalsizeof
theeyes'pupils.
Itcanbeanentirelyharmlessconditionorasymptomofmore
seriousmedicalproblems.
Anisocoriahasvariouscauses:
Physiologicalanisocoria:About20%ofnormalpeoplehaveaslight
differenceinpupilsizewhichisknownasphysiologicalanisocoria.
Inthiscondition,thedifferencebetweenpupilsisusuallylessthan
1mm.[3]
Horner'ssyndrome
Horner'ssyndromeisoculosympatheticpalsy.
Horner'ssyndromeconsistsofclassicaltriadofipsilateral:-
1. Ptosis,
2. Miosis,
3. Anhydrosis(lossofsweating)
Otherfeaturesare:-Lossofcilio-spinalreflex,Enophthatnos,
Heterochromia(ipsilateralirisisoflightcolour),thepupilisslowto
dilate,slightelevationofinferioreyelid,normalpupillaryreflex.

1057. Themostcommontypeofstrabismus
seeninmyopesis?
a)IntermittentExotropia
b)IntermittentEsotropia
c)Esotropiahypotropiacomplex
d)ExoptropiaHypotropiacomplex
CorrectAnswer-A
Ans.A.IntermittentExotropia
MyopiaandIntermittentexoptropia
Traditionally,itisbelievedthatthepresenceofmyopiamaybe
associatedwithadecreaseddemandforaccommodationandhence
lowerconvergence.
Thismaypredisposetoanincreasedriskofdevelopingexotropia
Alternately,itishypothesizedthatintermittentexotropiamayleadto
developmentofrnyopiaduetoincreasedaccommodativedemand
andincreasedconvergencemaybenecessarytocontrolthe
exodeviationthatcancontributetoincreasedacommodationand
myopiainintermittentdistanceexotropia.

1058.Dalrymple'ssignofocularGraves'
diseaserefersto:
a)Retractionoftheupperlid
b)Lidlag
c)Proptosis
d)Alloftheabovecombinedly
CorrectAnswer-A
Ans.Retractionoftheupperlid

1059.Macularsparingisassociatedwithlesionsin:
a)Opticnerve
b)Lateralgeniculatebody
c)Occipitalcortex
d)Opticchiasma
CorrectAnswer-C
Macularsparing,thatis,lossofperipheralvisionwithintactmacularvision,isalso
commonwithoccipitallesions
Ref:Ganong'sReviewofMedicalPhysiology23rdedition,Chapter12.

1060.

Pupillaryreflexpathway-Allofthefollowing
areapartexcept?

a)EdingerWestphalnucleus
b)Pretectalnudes
c)Medialgeniculatebody
d)Retinalganglioncell
CorrectAnswer-C
Ans.C.Medialgeniculatebody
Sensoryefferent)componentoflightrfiex=Optic(1st)nerve
Motor(efferent)componentoflightreflex=Occulomotor(3rd)nerve

1061.Sixthcranialnervepalsycausesofleft
eyecauses?
a)Accomodationparesisinleftgaze
b)Ptosisoflefteye
c)Adductionweaknessoflefteye
d)Diplopiainleftgaze
CorrectAnswer-D
Ans.D.Diplopiainleftgaze
Sixthnervesupplieslateralrectus,thereforeitspalsyresultsin
abductionweakness(notadductionweakness).
Inleftgaze,thereisabductionoflefteyeandadductionofrighteye.
Ifthereisparalysisoflateralrectusoflefteye(66nerveparalysis),
abductionoflefteyewillnotbepossibleinleftgaze,whileadduction
ofrighteyeisnormal.
Therefore,therewillbediplopiainleftgaze.

1062.DShapedpupilisseenin?
a)Iridodialysis
b)Iridodonesis
c)AnteriorUveitis
d)Anteriorsynechiae
CorrectAnswer-A
Ans.,A.Iridodialysis
Iridodtalysisisdetachmentoflrisfromitsrootattheciliarybody.
ItresultsinDshapedpupilandablackbiconvexareaseenatthe
periphery.

1063.Bestmethodofdetectionofretained
glassintraocularforeignbodyis:
a)CTscan
b)Radiography
c)Ultrasonography
d)Tonography
CorrectAnswer-A
Ans.CTscan

1064.Pulsatileproptosisisafeatureof?
a)Orbitalvarix
b)Retinoblastoma
c)Cortico-cavernousfistula
d)Covernoussinusthrombosis
CorrectAnswer-C
Ans.is'c'i.e.,Cortico-cavernousfistula
Proptosis
Proptosisisbulgingoftheeyeball(forwardbulging)beyondthe
orbitalmargins.Thoughthewordexophthalmosissynonymouswith
proptosis;somesourcedefinexophthalmosasaprotrusionofglobe
greaterthan18mmandproptosisasaprotrusionequaltoorless
than18mm.Proptosismaybeclassifiedasfollows:?
UnilateralProptosisProptosisofoneeye.
Inflammatorylesions:-Orbitalcellulitis,abscess,cavernoussinus
thrombosis,etc.
Vasculardisturbances:-Haemorrhage,varicoseorbitalveins,
haemangioma,etc.
Cystsandtumour:-Dermoidcyst,osteoma,lymphoma,
lymphosarcoma,glioma,meningiomaofopticnerve,retinoblastoma
andmetastaticdepositsinorbitNeuroblastoma,breast,prostate,
lung,GIT,Kidney,Ewing'stumor,melanoma,wilmstumor
(Nephroblastoma)].
Systemicdiseases-Leukemiasandendocrinedisturbancessuch
asGraves'diseaseandthyrotropicexophthalmosininitialstages.
Paralysisofextraocularmusclesasincompleteophthalmoplegia.
MucoceleofPNS'-Frontal(mostcommon),ethmoid,maxillary.
BilateralProptosisProptosisofbotheyes.

evelopmentalanomaliesoftheskull-Oxycephaly(towerskull).
Endocrineexophthalmos,boththyrotoxicandthyrotropic.
Inflammatorylesions-Cavernoussinusthrombosis.
Tumours-lymphosarcoma,lymphoma,pseudotumour,
nephroblastoma,Ewing'ssarcoma.
Systemicdisease-Histocytosis(Hand-schullerchristondisease),
amyloidosis,wegner'sgranulomatosis.
Intermittentproptosis
Proptosisdevelopingintermittentlyandrapidlyinoneeyewhen
venousstasisisinducedbyforwardbendingorloweringthehead,
turningtheheadforcibly,hyperextensionoftheneck,coughing,
forcedexpirationwithorwithoutcompressionofthenostrils,or
pressureonjugularveins.Themostimportantcasueisorbitalvarix
(varicocele).
Pulsatileproptosis:-Pulsatileproptosisisseenin
caroticocovernousfistula;saccularaneurysmofophthalmicartery;
andduetotransmittedcerebralpulsationasseeninmeningocele,
neurofibromatosisandtraumaticoroperativehiatus.

1065.Inwhichofthefollowingconditions
Berlin'sedemais
a)Openangleglaucoma
b)Aftercataractsurgery
c)Afterconcussionaltrauma
d)Diabeticretinopathy
CorrectAnswer-C
Ci.e.Afterconcussiontrauma
BlunttraumatoeyemayproduceBerlin'sedemaorcommotio
retinaeQ
whichisacloudyswellingcharacterizedbyagrey
appaerance,mostfrequentlyinthetemporalregion.Itmayalso
manifestascherryredspotinthefovealregion.

1066.Allofthefollowingarecomplicationsof
traumatichyphemaexcept?
a)Rebleeding
b)PupillaryBlock
c)CornealUlcer
d)Posteriorsynechiae
CorrectAnswer-C
Ans.C.CornealUlcer
Complicationsoftraumatichyphema

1. Obstructionoftrabecularmeshworkwithassociatedintraocular
pressureelevation
2. Peripheralanteriorsynechiae(PAS)
3. Posteriorsynechiae
4. Cornealbloodstaining
5. Rebleeding:Canoccurwhentheinitialclotretractsandlyses
allowingforasecondepisodeofbleeding.Rebleedsaregenerally
moreseverethantheinitialbleed,morelikelytoleadtoglaucoma,
cornealbloodstaining,andsynechiaeformation.Ithasbeen
reportedtooccur3.5%to38%ofthetimeandprobably5-10%
overall.
6. Pupillaryblock
7. Amblyopia(pediatricpatients)

1067.Allofthefollowingdrugsincreasethe
riskofpostoperativenauseaand
vomitingaftersquintsurgeryin
childrenexcept?

a)Halothane
b)Opiods
c)Propofol
d)NitrousOxide
CorrectAnswer-C
Ans,C.Propofol
Strabismussurgeryonchildrenisanindependentriskfactorfor
postoperativenauseaandvomiting.
PropofolIsUsedInPredisposedIndividualsAsItHas
VeryLessEmetogenecity

1068.Whatisthemostcommoneyelesionin
HIV?
a)KaposiSarcomaofLid
b)CMVRetinitis
c)Cottonwoolspots
d)Choroiditis
CorrectAnswer-C
Ans.C.Cottonwoolspots
Themostcommonabnorrnalfindingonfundoscopicexaminationis
cofton-woolspots.

1069. FeaturesofUsher'sSyndromeinclude
allexcept?
a)NightBlindness
b)VisualImpairment
c)MultipleNeurofibromas
d)Hearingdeficit
CorrectAnswer-C
Ans.C.MultipleNeurofibromas
Ushersyndrome
Ushersyndromeisarelativelyraregeneticdisordercausedbya
mutationinanyoneofatleastIIgenesresultinginacombinationof
hearinglossandvisualimpairment,andisaleadingcauseof
deafblindness.
Ushersyndromeisincurableatpresent.
OthernamesforUshersyndromeincludeHallgrensyndrome,
Usher-Hallgrensyndrome,retinitispigmentosadysacusissyndrome,
anddystrophiaretinaedysacusissyndrome.
Thissyndromeischaracterizedbyhearinglossandagradualvisual
impairment.Thehearinglossiscausedbyadefectiveinnerear,
whereasthevisionlossresultsfromretinitispigmentosa(RP),a
degenerationoftheretinalcells.
Usually,therodcellsoftheretinaareaffectedfirst,leadingtoearly
nightblindnessandthegraduallossofperipheralvision.
Inothercases,earlydegenerationoftheconecellsinthemacula
occurs,leadingtoalossofcentralacuity.
Insomecases,thefovealvisionisspared,leadingto'doughnut
vision";centralandperipheralvisionareintact,butanannulusexists
aroundthecentralregioninwhichvisionisimpaired.


1070.Pilocarpineisusedinallofthe
followingexcept:
September2006

a)Primary,OpenAngleGlaucoma
b)MalignantGlaucoma
c)AcuteAngleClosureGlaucoma
d)ChronicSynechialAngleClosureGlaucoma
CorrectAnswer-B
Ans.B:MalignantGlaucoma
Managementofangle-closuresecondarytociliochoroidal
effusionisdirectedattwoprocesses:
Thefirstisuvealinflammation,whichistreatedwithoralsteroidsto
reducetheeffusionandallowthechambertodeepenspontaneously
astheinflammationsubsides.
Thesecondisanteriorrotationofthelens-irisdiaphragm,whichis
reversedwithcycloplegics.Thepressureistreatedintheacute
settingwithaqueoussuppressantsandoralcarbonicanhydrase
inhibitors.Pilocarpineiscontraindicatedbecauseitwillcause
anteriorrotationofthelens-irisdiaphragm.

1071.CHARGEsyndromeincludesallexcept
?
a)EyeColoboma
b)Congenitalheartdisease
c)Urinarytractdefects
d)EsophagealAtresia
CorrectAnswer-D
Ans,D.EsophagealAtresia
CHARGESYNDROME
C-Colobomaoftheeye,centralnervoussystemanomalies.
H-Heartdefects.
A-Atresiaofthechoanae.
R-Retardationofgrowthand/ordevelopment.
G-Genitaland/orurinarydefects(Hypogonadism,undescended
testicles,besideshypospadias).
E-Earanomaliesand/ordeafrressandabnormallybowl-shaped
andconcaveears,knownas'lopears".

1072.Parasitosisofextraoculareyemuscles
isseenin?
a)Trichinosis
b)Cysticercosis
c)Amoebiasis
d)Ascariasis
CorrectAnswer-A
Ans,A.Trichinosis
'Althoughmanyparasitescantheoreticallyinvolvetheextraocular
musclethemostfrequentformofparasiticinfestationofextraocular
musclesistrichinosis'

1073.Theprincipleoftotalinternalreflection
isusedby?
a)Gonioscope
b)Pachymeter
c)Ophthalmoscope
d)Lensometer
CorrectAnswer-A
Ans.A
Gonioscopyisanessentialdiagnostictoolandexamination
techniqueusedtovisualizethestructuresoftheanteriorchamber
angle.
Masteringthevarioustechniquesofgonioscopyiscrucialinthe
evaluationofglaucomapatients.
Gonioscopyisrequiredtovisualizethechamberanglebecause
undernormalconditionslightreflectedfromtheanglestructures
undergoestotalinternalreflectionatthetear?airinterface.Atthe
tear?airinterface,thecriticalangle(approximately46?)isreached
andlightistotallyreflectedbackintothecornealstroma.This
preventsdirectvisualizationoftheanglestructures.Allgonioscopy
lenseseliminatethetear?airinterfacebyplacingaplasticorglass
surfaceadjacenttothefrontsurfaceoftheeye.Thesmallspace
betweenthelensandcorneaisfilledbythepatient'stears,saline
solution,oraclearviscoussubstance.Dependingonthetypeof
lens

1074.
Immediatetreatmentofacute
dacryocystitisis?
a)Antibioticsanddrainageofabscesssifpresent
b)Dacryocystorhinostomy
c)Dacryocystectomy
d)Nasaldecongestants
CorrectAnswer-A
Ans.A.Antibioticsanddrainageofabscesssifpresent

1075.Whichofthefollowingisthecommon
causeofrespiratoryfailuretype2?
a)Chronicbronchitisexacerbation
b)Acuteattackasthma
c)ARDS
d)Pneumonia
CorrectAnswer-A
Answer-A.Chronicbronchitisexacerbation
TypeIIrespiratoryfailureoccursduetoalveolarhypoventilation

1076.Aspirin-sensitiveasthmaisassociated
with:
a)Obesity
b)Urticaria
c)Nasalpolyp
d)Extrinsicasthma
CorrectAnswer-C
TheanswerisC(Nasalpolyp):
`AspirinassociatedAsthmausuallybeginswithperennialvasomotor
rhinitisthatisfollowedbyhyperplasticrhinosinusitiswithnasal
polyps'--Harrisons
AspirinassociatedAsthma:
Primarilyaffectsadults,althoughtheconditionmayoccurin
childhood.
Usuallybeginswithperennialvasomotorrhinitisthatisfollowedby
hyperplasticrhinosinusitiswithnasalpolyps.
Progressiveasthmathenappears.
Onexposuretoevenverysmallquantitiesofaspirin,affected
individualstypicallydevelopocularandnasalcongestionandacute,
oftensevereepisodesofairwaysobstruction.
Deathmayfollowingestionofaspirin.

1077.Whichofthefollowingisseenin
sarcoidosis
a)Hypercalcemia
b)Hypocalcemia
c)Hyperphosphatemia
d)Hypophosphatemia
CorrectAnswer-A
Answer-A.Hypercalcemia
Granulomaofsarcoidosiscansecret1-25(OH)2vitaminD.
Therefore,patientsofsarcoidosismaydevelophypercalcemia.

1078.Centralbronchiectasisisseenwith
a)CysticAdenomatoidMalformation
b)Cysticfibrosis
c)Bronchocarcinoma
d)Tuberculosis
CorrectAnswer-B
Ans.is'b'i.e.,Cysticfibrosis
Thedistribution()I.bronchiectasismarbeimportantdiagnostically
AcentralPerihilar
allergicbronchopulmonaryaspergillosis.

PredominantupperlobeoMiddleandlowerlobeCystic
fibrosisoroneofitsvariants.DistributionisconsistentwithPCD
LowerlobeinvolvementisMiddlelobeandlingularsegmentof
theLULinvolvementischaracteristicofnontuberculous
mycobacteria(NTM).Idiopathicbronchiectasis


1079.BronchiectasisSiccaisseenwith
a)Tuberculosis
b)Pertussis
c)Cysticfibrosis
d)Kartagenersyndrome
CorrectAnswer-A
Ans.is'a'i.e.,Tuberculosis
BronchiectasisSiccaorDryBronchiectasisistypically
associatedwithTuberculosis.

Tuberculosisisassocitedwithatypeofdrybronchiectasiscalled
BronchiectasisSicca,whichispredominantlyseeninupperlobes.
DryBronchiectasis(BronchiectasisSicca)istypicallycharacterized
byabsenceofcopiousamountofsputumwhichisusuallyahall
markofbronchiectasis.
Drycoughassociatedwithhemoptysisisthetypicalpresentation
Endobronchialtuberculosiscommonlyleadstobronchiectasis,either
frombronchialstenosisorsecondarytractionfromfibrosis.Traction
bronchiectasischaracteristicallyaffectsperipheralbronchi(which
lackcartilagesupport)inareasofend-stagefibrosis

1080.InapatientwithCOPD,best
managementoptionis
a)Quitsmoking
b)Bronchodilators
c)Lowflowoxygen
d)Mucolytics
CorrectAnswer-C
Answer-C.Lowflowoxygen
Therapyisstartedwithshort-actingbronchodilator(beta-agonistor
anticholinergic).
Long-termoxygentherapyisusedinallpatientswithCOPDwho
havechronichypoxemia

1081.Inapatientthereisdyspneainupright
positionwhichisrelievedinsupine
position,Diagnosis?

a)Tachypnea
b)Orthopnea
c)Paroxysmalnocturnaldyspnea
d)Platypnea
CorrectAnswer-D
Answer-D.Platypnea
Platyapnea(Orthodeoxia)
Dyspnoeawhenapatientmovestosittingorstandingpositionfrom
arecumbentposition.

1082.Mostcommoncauseofidiopathic
interstitialpneumoniais
a)Sarcoidosis
b)Organizingpneumonia
c)Idiopathicpulmonaryfibrosis
d)Lipoidpneumonia
CorrectAnswer-C
Answer-C.Idiopathicpulmonaryfibrosis
Idiopaticpulmonaryfibrosis
Idiopathicnonspecificinterstitialpneumonia

1083.ClickingnoiseinPneumomediastinum
isknownas
a)Hammansign
b)Trailsign
c)Kussmaulsign
d)None
CorrectAnswer-A
Answer-A.Hammansign
Crunchingorclickingnoiseheardsynchronouslywiththeheartbeat
onauscultationandbestheardintheleftlateraldecubitusposition.It
isassociatedwith"Pneumomediastinum".

1084.Emphysemapresentswithallexcept
a)Cyanosis
b)Barrelshapedchest
c)Associatedwithsmoking
d)TypeIrespiratoryfailure
CorrectAnswer-A
Answer-A.Cyanosis
Dyspnoea
Coughorwheezing(somepatient)
Weightloss
Barrel-Chest
FEVCandFEV1arereduced.TLC,RC,andFRCareincreaseddue
tohyperinflation.
Cyanosisisrare(incontrasttochronicbronchitis)
Emphysema(COPD)causestype-1respiratoryfailure

1085.Allareseeninemphysemaexcept
a)Decreasedvitalcapacity
b)Hyperinflation
c)Rhonchi
d)ReducedDlco
CorrectAnswer-C
Answer-C.Rhonchi
Coughorwheezing(somepatient)
Weightloss
Barrel-Chest
FEVCandFEV1arereduced.TLC,RCandFRCareincreaseddue
tohyperinfcation.
Cyanosisisrare(incontrasttochronicbronchitis)
Emphysema(COPD)causestype-1respiratoryfailure

1086.Mostcommoncauseofpleuraleffusion
inAIDSpatients
a)Kaposisarcoma
b)TB
c)PneumocystisJiroveci
d)Mycoplasma
CorrectAnswer-A
Ans.is'a'i.e.,Kaposisarcoma
Ophthalmologicaldiseases
Themostcommonabnormalfindingsonfundoscopicexamination
arecottonwoolspots.
CMVretinitisisthemostsevereocularcomplicationandoccurs
whenCD4T-cellscountislessthan50/ml.Ittypicalpresentsas
perivascularhemorrhageandexudaewithCottage-Cheese
appearance.
Acuteretinalnecrosissyndrome,alsocalledprogressiveouter
retinalnecrosis(PORN)iscausedbyHSVandVZV
OthermanifestationsarechorioretinitisbytoxoplasmaandPcarinii,
kaposisarcomaofeyelid,andlymphoma.

1087.Mostcommoncauseoflungabscessis
comatosepatient
a)Staphaureus
b)Oralanaerobes
c)Klebsiella
d)Tuberculosis
CorrectAnswer-B
Answer-B.Oralanaerobes
Mostlungabscessesinmoribundintubatedpatientsaredueto
anaerobicbacteria,likepeptostreptococcus,Bacteroidesetc.Lung
abscess
Thetermpulmonaryabscessdescribesalocalsuppurativeprocess
withinthelung,characterizedbynecrosisoflungtissues
Etiology
Asaspirationoforopharyngealsecretionsisthemostcommon
cause,organismsmostcommonlycausinglungabscessarethose
normallyfoundinoralcavity,i.e.,Anaerobicbacteria(Bacteroides,
Fusobacterium,peptococcusspecies).OtherorganismsareS.
aureus,Klebsiella,Nocardiaandgramnegativebacteria.

1088.Ifapersonishavingventriular
tachycardia,extrasystolesappearsto
a)Pwave
b)QRScomplex
c)Twave
d)Rwave
CorrectAnswer-B
Answer-B.QRScomplex
ExtrasystoleinventriculartachycardiaappearsinQRScomplex
whenanirritablefocusinanypartoftheventricularmyocardium
activatestheventriclesbeforethearrivalofthenextnormalwaveof
depolarisationfromtheatriaaventricularextrasystoleisproduced.

1089.MostcommonarrhythmiainICU
patients-
a)Atrialflutter
b)Atrialfibrillation
c)PSVT
d)NPAT
CorrectAnswer-B
Answer-B.Atrialfibrillation
MostcommonarrhythmiainI.C.U.patientAtrialfibrillation
MostcommonarrhythmiainapatientwithCardiacarrest
Ventricularfibrillation

1090.Mostcommonmechanismof
arrhythmia
a)Re-entry
b)Earlyafterdepolarization
c)Lateafterdepolarization
d)Automaticity
CorrectAnswer-A
Ans.is'a'i.e.,Re-entry
Themostcommonarrhythmiamechanismisre-entry.
Fundamentally,re-entryisdefinedasthecirculationofanactivation
wavearoundaninexcitableobstacle.
Re-entryappearstobethebasisformostabnormalsustainedSupra
VentricularTachycardias(SVTs)andVentriculartachycardia.
Examplesofre-entryare:-
PolymorphicVentriculartachycardiainpatientswithagenetically
determinedionchannelabnormalitysuchastheBrugadasyndrome,
catecholaminergicpolymorphicVentriculartachycardia.


1091.Patientof1stdegreeheartblock
complainsofdizziness.Besttreatment
forthispatientis

a)Atropine
b)Isoprenaline
c)Adrenaline
d)Pacemaker
CorrectAnswer-D
Answer-D.Pacemaker
Themostdefinitiveorreliabletreatmentforpatientwithsymptomatic
A.V.conductionsystemistemporaryorpermanentpacing.

1092.Allareusedforsecondaryprevention
ofMIexcept
a)Aspirin
b)Statins
c)Betablockers
d)Warfarin
CorrectAnswer-D
Answer-D.Warfarin
MedicinesusedinthesecondarypreventionofM.I.
LongtermdualantiplatelettherapywithaspirinandP2Y12receptor
blocker.
Statins(highintensity).
Angiotensinconvertingenzymeinhibitorsinpatientwithdiabetes
heartfailure,leftventricularejectionfraction.
f3blockers.

1093.Notrecommendedincoronaryartery
diseasepatients
a)Dailyexercise
b)Potassium
c)Vitamin-E
d)Statins
CorrectAnswer-C
Answer-C.Vitamin-E
InterventionstudiesusingvitaminEtopreventcardiovascular
diseaseorcancerhavenotshownefficacy

1094.Digitalisisusedinmitralstenosiswhen
patientdevelops
a)Atrialfibrillation
b)Rightventricularfailure
c)Acutepulmonaryedema
d)Myocarditis
CorrectAnswer-A
Answer-A.Atrialfibrillation
DrugsusefulinslowingtheventricularrateofpatientswithAF
Betablockers,
Nondihydropyridinecalciumchannelblockers(e.g.,verapamilor
diltiazem),and
Digitalisglycosides

1095.Whichoneofthefollowingisnotan
earlycomplicationofacutemyocardial
infarction?

a)Papillarymuscledysfunction
b)Ventricularseptaldefect
c)Paricarditis
d)Dressler'ssyndrome
CorrectAnswer-D
Answer-D.Dressler'ssyndrome
Dressler'ssyndromeisalatecomplicationofmyocardialinfarction.It
usuallyoccurs1-8weeksaftermyocardialinfarctions

1096.Mostcommonmalignanttumorofheart
inadults
a)Sarcoma
b)Rhabdomyoma
c)Lipoma
d)Paraganglioma
CorrectAnswer-A
Answer-A.Sarcoma
Almostallprimarycardiacmalignanciesaresarcomas.

1097.Whichisthebestwaytodifferentiate
betweenstableanginaandNSTEMI?
a)ECG
b)Cardiac-biomarker
c)TransthoracicEchocardiography
d)MultiuptakegatedAcquisitionscan
CorrectAnswer-B
Answer-B.Cardiac-biomarker
ThedifferentiatingfeaturebetweenAnginaandMIistheelevationof
cardiacmarkers?.(noelevationisseeninAngina)

1098.AetiologyofDresslerSyndromeis
a)Viral
b)Autoimmune
c)Idiopathic
d)Toxinmediated
CorrectAnswer-B
Answer-B.Autoimmune
Immunologicalfactorsarethoughtstobeofprimaryimportance.
Theimmunecomplexeshataregeneratedaredepositedintothe
pericardium,pleuraandlungs.

1099.Predisposingfactorsforcoronary
arterydiseaseinclude,allExcept:
a)Homocysteinemia
b)LipoproteinB
c)Fibrinogen
d)plasminogenactivatorinhibitors1
CorrectAnswer-B
AnswerisB(.LipoproteinB)
Predisposingfactorsforcoronaryarterydiseaseincludean
increasedlipoprotein'a'andnotlipoprotein'6'.


1100.Murmurheardinaorticstenosis
a)Right2ndintercistal,lowpitchmurmur
b)Apex,lowpitchmurmur
c)LeftSternalarea,lowpitchmurmur
d)Pen-systolicmurmur,highpitchmurmur
CorrectAnswer-A
Answer-A.Right2ndintercistal,lowpitchmurmur
TypicallyheardatthebaseoftheheartinAorticarea(second
intercostalspace).
Harshquality.
GenerallybeginsafterSiandendSbeforeS2.

1101.Whichofthefollowingstatements
aboutatrialmyxomasistrue
a)MostcommoninLeftAtrium
b)MorecommoninMales
c)Distantmetastasisareseen
d)Mostmyxomasarefamilial
CorrectAnswer-A
AnswerisA(MostcommoninLeftAtrium)
CardiacMyxomasareusuallylocatedintheatria,mostcommonin
theleft.
Cardiacmyxomas
ArethemostcommontypeofprimarycardiactumorsQ
Occuratallagesandshownosexpreference(mixesequallywith
bothsexes)Q
Mostcardiacmyxomasaresporadic,
whilesomemaybefamilial
Sporadicmyxomas:
AresolitaryQ
LocatedinAtria,mostcommonlyintheleftQ
Unlikelytohavepost-oprecurrencee
OccurinyoungerindividualsQ
Familialmyxomas:
Aremultiple?
Morelikelytohavepostoprecurrencee
Myxomasarebenigntumorsandthereforedistantmetastasisare
notseen.?


1102.Duroziez'ssignisseenin
a)AorticRegurgitation
b)TricuspidRegurgitationMitralstenosis
c)Pericardialeffusion
d)None
CorrectAnswer-A
Answer-A.AorticRegurgitation
Insevereaorticregurgitation,gradualpressureoverthefemoral
arteryleadstoasystolicanddiastolicbruit.
Referstosystolicanddistolicmurmursheardoverthefemoralartery
whilepartiallycompresslngthevesselwiththediaphragmofthe
stethoscope.

1103.Beckstriadisseenin
a)Constrictivepericarditis
b)Restrictivecardiomyopathy
c)Cardiactamponade
d)Noneoftheabove
CorrectAnswer-C
Answer-C.Cardiactamponade
Beck'striadischaracteristicofcardiacTamponade,itincludes
:
Increasedvenouspressure
Decreasedarterialpressure
Muffledheartsounds,silentheart(duetopresenceoffluidin
pericardium).

1104.Theseverityofmitralstenosiscanbe
judgedby-
a)Intensityofmurmur
b)Durationofmurmur
c)LeftventricularS3
d)LoudS1
CorrectAnswer-B
Answer-B.Durationofmurmur
DurationdependsonseverityofMS.
InsevereMS,themiddiastolicmurmurislongandmergeswiththe
presystolicmurmurtoproduceholodiastolicmurmur..

1105.Widepulsepressureisseeninall
except:
a)PDA
b)Aorticstenosis
c)AorticRegurgitation
d)A.V.malformation
CorrectAnswer-B
AnswerisB(AorticStenosis)
AorticStenosisisassociatedwithanarrowpulsepressure.
PatentDuctusArteriosus(PDA),AorticRegurgitationandAV
Malformations(Arteriovenousshunting)areallassociatedwitha
widepulsepressure.


1106.Allaretruefortransplantedkidney
except
a)Humoralantibodyresponsibleforrejection
b)CMIisresponsibleforrejection
c)Previousbloodtransfusion
d)HLAidentitysimilarityseenin1:100people
CorrectAnswer-D
Answer-D.HLAidentitysimilarityseenin1:100people
Withinanyparticularfamily,sibling'shavea7:4chanceofbeing
HLAidentical.Incontrastamongunrelated
people,theprobabilitiesofHLAidentityinseveralthousand
dependinguponphenotypeinvolved'Itisduetothefactthat
HLAcomplexisinheritedintactastwohaplotypes.

1107.AllareseeninNephroticsyndrome
except
a)Atherosclerosis
b)Thrombo-embolism
c)IncreasedproteinClevels
d)Lipiduria
CorrectAnswer-C
Answer-C.IncreasedproteinClevels
Nephroticsyndromeisaclinicalcomplexcharacterizedbyanumber
ofrenalandextrarenalfeatures,mostprominentofwhichare
Proteinuria(inpractice>3.0to3.5gm/24hrs),
Hypoalbuminemia,Edema
Hypertension
Hyperlipidemia,Lipiduria
Hypercoagulabilty(resultofLossofAntithrombinIII)

1108.Basketweaveappearanceofglomerular
basementmembraneonelectron
microscopyisseenin

a)Alportsyndrome
b)AcutepoststretptococcalGN
c)Polyarteritisnodosa
d)Giantcellarteritis
CorrectAnswer-A
Answer-A.Alportsyndrome
Basketweaveappearanceofglomerularbasementmembraneon
ElectronMicroscopyisseeninAlport'ssyndrome.
InAlport'sSyndrome,theglomerularbasementmembraneshows
irregularthinningandthickeningwithalamellatedbasket-weave
appearanceinthethickenedareaduetoextensiveremodelingand
injuryofthebasementmembrane.

1109.Allofthefollowingcausesacuterenal
failureexcept
a)Pyelonephritis
b)Snakebite
c)Rhabdomyolysis
d)Analgesicnephropathy
CorrectAnswer-D
Answer-D.Analgesicnephropathy
Analgesicnephropathycauseschronicinterstitialnephritisand
presentswithchronickidneydisease.
AKIisaseriouscomplicationofsnakebitesbytheviperidaefamily

1110.FeaturesofHepatorenalsyndromeare
a)Urinesodium<10meq/1
b)Normalrenalhistology
c)Renalfunctionabnormalevenafterliverbecomenormal
d)aandb
CorrectAnswer-D
AnswerisA&B(urineNa<10meq/landNormalRenal
Histology)
Hepatorenalsyndromeisassociatedwithnormalrenalhistologyand
supportedbyaurinesodiumexcretionl0meq/L
Hepatorenalsyndrome
Hepatorenalsyndromeisdefinedasastateoffunctionalrenal
failure(ReducedGFR)inpatientswithsevereliverdisease
Structurally/Histologicallythekidneysarenormalandrecover
functionaftersuccessfullivertransplantation.
Thepathogenetichallmarkofhepatorenalsyndromeisintenserenal
vasoconstrictionwithcoexistentsystemicvasodilatation
Thediagnosisofhepatorenalsyndromeisconsideredinaccordance
withthefollowingdiagnosticcriteria.
DiagnosticofHepatorenalSyndrome
Majorcriteria
Lowglomerularfiltrationrate.asindicatedbyserumcreatinine>1.5
mg/dLor24-hrcreatinineclearance<40mL/min
Absenceofshock,ongoingbacterialinfection,fluidlosses,and
currenttreatmentwithnephrotoxicdrugs
Nosustainedimprovementinrenalfunction(decreaseinserum
creatinineto1.5nig/dLorincreaseincreatinineclearanceto40
mL/min)afterdiureticwithdrawalandexpansionofplasmavolume


with1.5Lofaplasmaexpander
Proteinuriamg/d1,andnouhrasonographicevidenceofobstructive
uropathyorparenchymalrenaldiseaseAdditionalcriteria
Urinevolume<500mL/d
Urinesodium<10meq/L
Urineosmolalitygreaterthanplasmaosmolality
Urineredbloodcells<50/high-power.field
Serumsodiumconcentration<130niEqL
Note:Allmajorcriteriamustbepresentforthediagnosisof
hepatorenalsyndrome.
Additionalcriteriaarenotnecessaryforthediagnosisbutprovide
supportiveevidence.


1111.AllaretrueaboutGFRexcept
a)30-40%decreaseafter70yearsofage
b)Bestestimatedbycreatinineclearance
c)C.K.DisdefiedasGFR<30ml/min/1.732for4weeks
d)GFRisdependentonheightinchildren
CorrectAnswer-C
Answer-C.C.K.DisdefiedasGFR<30ml/min/1.732for4
weeks


1112.Dialysisindications
a)Hypertension
b)Hypokalemia
c)Pericarditis
d)Metabolicalkalosis
CorrectAnswer-C
Answer-C.Pericarditis
Indicationsofdialysisinchronicrenalfailure
Pericarditisorpleuritis(urgentindication).
Progressiveuremicencephalopathyorneuropathy,withsignssuch
asconfusion,asterixis,myoclonus,wristorfootdrop,or,insevere
cases,seizures(urgentindication).
Aclinicallysignificantbleedingdiathesisattributabletouremia
(urgentindication).
Persistentmetabolicdisturbancesthatarerefractorytomedical
therapy;theseincludehyperkalemia,metabolicacidosis,
hypercalcemia,hypocalcemia,andhyperphosphatemia.
Fluidoverloadrefractorytodiuretics.
Hypertensionpoorlyresponsivetoantihypertensivemedications.
Persistentnauseaandvomiting.
Evidenceofmalnutrition.

1113.Polyuriawithlowfixedspecificgravity
urineisseenin?
a)Diabetesmellitus
b)Diabetesinsipidus
c)Chronicglomerulonephritis
d)Potomania
CorrectAnswer-C
Answer-C.Chronicglomerulonephritis
Polyuriawithfixedlowspecificgravityisafeatureofchronicglomerulonephritis.
Specificgravitymeasuresthekidney'sabilitytoconcentrateordiluteurineaboutplasma.
Becauseurineisasolutionofminerals,salts,andcompoundsdissolvedinwater,the
specificgravityisgreaterthan1.000.Themoreconcentratedtheurine,thehighertheurine
specificgravity.
Anadult'skidneyshavearemarkableabilitytoconcentrateordiluteurine.
Ininfants,therangeforspecificgravityislessbecauseimmaturekidneysarenotableto
concentrateurineaseffectivelyasmaturekidneys.
Alowspecificgravityoccursinthreesituations.
Indiabetesinsipidus,thereisanabsenceordecreaseofanti-diuretichormone.Without
anti-diuretichormone,thekidneysproduceanexcessiveamountofurine,oftenupto15to
20litersperdaywithlowspecificgravity.
Glomerulonephritisandpyelonephritiscausedecreasedurinevolumeandlowspecific
gravity.Inthesediseases,damagetothekidney'stubulesaffectstheabilityofthekidneyto
re-absorbwater.Asaresult,theurineremainstodilute.
Thethirdreasonforlowspecificgravityisrenalfailure,whichresultsinafixedspecific
gravitybetween1.007and1.010.Inrenalfailure,theremainingfunctionalnephrons
undergocompensatorystructuralandhypertrophicchanges.Thesecompensatorychanges
resultinurinethatisalmostisotonicwithplasma.Therefore,apatientexperiencingrenal
failurewillpresentwithspecimensmeasuringthesame,orfixed,specificgravityregardless

ofwaterintake.

1114.Whichofthefollowingmicroorganism
isincriminatedininfectionafter
hemodialysis

a)Chlamydia
b)Grampositiveorganisms
c)Gramnegative
d)Anaerobes
CorrectAnswer-B
Answer-B.Grampositiveorganisms
Hemo-dialysiscatheter-relatedbloodstreaminfections(CRBSIs)are
amajorcomplicationoflong-termcatheterusinHD.Grampositive
organismareseenfollowedbygramnegativeorganisms.

1115.Disease,doesnotrecurinthekidney
afterrenaltransplantis:
a)Alportsyndrome
b)Amyloidosis
c)GoodPasteur'ssyndrome
d)Diabeticnephropathy
CorrectAnswer-A
AnswerisA(Alport'ssyndrome):
Alportissyndromehasnotbeenmentionedtorecurinkidneyaftera
renaltansplant.


1116.LowserumcopperduetoATP7Agene
isdueto?
a)Dubin-johnson'ssyndrome
b)Wilsondisease
c)Menkedisease
d)Gilbert'sdisease
CorrectAnswer-C
Answer-C.Menkedisease
Menke'sdisease,alsoknownaskinkyhairdisease,isanX-linked
neurodegenerativediseaseofimpairedcoppertransport,duetoATP
7AgenelocatedonXp12-13.

1117.Significantweightlossisdefinedas:
a)5%weightlossin1-2months
b)5%weightlossin2-3months
c)10%weightlossin1-2months
d)10%weightlossin2-3months
CorrectAnswer-A
AnswerisA(5%weightlossin1-2months):
Significantweightlossisdefinedas5%weightlossinImonth.
Percentweightchangeoveraperiodoftimeiscalculatedusing
theperson'scurrentbodyweightandperson'susualbodyweight.
Usualweight--CurrentweightPercentweightchange=Usual
weight

1118.HepaticEncephalopathyispredisposed
byall,Except:
a)Hyperkalemia
b)Dehydration
c)Constipation
d)GIBleeding
CorrectAnswer-A
AnswerisA(Hyperkalemia):
HepaticEncephalopathyispredisposedbyHypokalemiaandnotby
Hyperkalemia


1119.Whichisnottrueaboutalcoholic
hepatitis:
a)Gammaglutamyltransferaseisraised
b)SGPTisraised>SGOT
c)SGOTisraised>SGPT
d)Alkalinephosphataseisraised
CorrectAnswer-B
AnswerisB
SGOT/SGPTratiogreaterthan2ishighlysuggestiveofalcoholic
hepatitisandcirrhosis.
ASTissynonymouswithSGOT&ALTissynonymouswithSGPT
IngeneralASTandALTlevelsriseparalleltoeachother.In
alcoholicliverdiseasetheASTrisesoutofproportiontoALTsuch
thattheratioofASTandALTmaybecomegreaterthan2:
StigmataofAlcoholichepatitis/cirrhosisthataidindiagnosis:
1. BilateralenlargedparotidsQ.
2. GynaecomastiaQ
3. TesticularatrophywithlossofbodyhairQ
4. WastingofmusclemassQ
5. DuputyrenscontractureQ
AST(SGOT)outofproportiontoALT(SGPT)seeninQ
1. AlcoholichepatitisQ
2. FattyliverinpregnancyQ
GammaGlutamyltransferaselevelscorrelatewithlevelsof
Alkalinephosphataseandareasensitiveindicaterofbiliarytract
disease--obstructivejaundice.Itisnotanindicatorofalcoholic
liverdisease.


Remember:GGTisthemostsensitiveindicatorofbilian,tract
disease


1120.Rockallscoreisusedforprognosisof
patientsof
a)UpperGIbleeding
b)LowerGIbleeding
c)Hepaticencephalopathy
d)IBD
CorrectAnswer-A
Answer-A.UpperGIbleeding

1121.Whichisahormonedependentliver
tumor?
a)Adenoma
b)Hemangioma
c)Hepatocellularcarcinoma
d)Hemangiopericytoma
CorrectAnswer-A
Answer-A.Adenoma
Adenomasareassociatedwithcontraceptivehormoneuse.

1122.Whichofthefollowingisnottrue
regardingamoebicliverabscess
a)Multipleabscessesismorecommon
b)Mayruptureintothepleuralcavity
c)Forasymptomaticluminalcarriersdiloxanidefuroateisthedrug
ofchoice
d)Mostlyinvolvingtherightlobeofliver
CorrectAnswer-A
Ans.is'a'i.e.,Multipleabscessesaremorecommon
Amoebicliverabscessisthemostcommonextra-
intestinalmanifestationofamoebiasis.
Itistheinvolvementoflivertissuebytrophozoitesoftheorganism
Entamoebahistolyticaandofitsabscessduetonecrosis.
Mostcommonintheposterosuperiorsurfaceoftherightlobe.
Usually,thereisonelargesolitaryabscess.-
Thenecroticcontentsofliverabscessareclassically
describedasanchovy-saucepus.
Pleuropulmonaryinvolvementofthemostfrequentcomplicationof
amoebicliverabscess.
ALAisusuallysolitaryandpyogenicliverabscessisusuallymultiple.
MultipleALA,althoughrare,arefrequentlyconfusedwithpyogenic
liverabscess.
However,Tayaletal.,fromIndiashowedthattheexistenceof
multipleALAisnotuncommonaspreviouslythoughtand
superinfectionorco-infectionwithpyogenicorganismsiscommonin
suchcases.
Treatment
Treatmentdependsonthetypeofinfection:

1)Asymptomaticcarrier-->Luminalagents(Iodoquinol,
Paromomycine)-->DiloxanidefuroateistheDOC.
2)Acutecolitis(dysentery)-->Metronidazoleor
Tinidazole+Luminalagent.
3)Liverabscess-->MetronidazoleorTinidazoleorOrnidazole
+Luminalagent.

1123.Poikilocytosisandanisocytosisisseen
in
a)Megaloblasticanaemia
b)Irondeficiencyanaemia
c)Nutritionaldeficiencyanaemia
d)Thalassemia
CorrectAnswer-B
Answer-B.Irondeficiencyanaemia
AnisocytosismeansthatRBC'sareunequalinsizeindicatingthat
someoftheRBC'sareeithertoobigortoosmall.
PoikilocytosismeansthatsomeoftheRBC'sareabnormally
shaped.

1124.Allofthefollowingarecharacteristic
featuresoftreatmentofirondeficiency
anemiawithoralironsupplements,
except

a)BioavailabilityisenhancedwithvitaminC
b)Theproportionofironabsorbedreducesashemoglobin
improves
c)Thereticulocytecountshouldbegintoincreaseintwoweeks
andpeakin4weeksthissuggestsgoodresponsetotreatment
d)Thetreatmentshouldbediscontinuedimmediatelyonce
hemoglobinnormalizestopreventsideeffectsofiron
CorrectAnswer-D
Answer-D.Thetreatmentshouldbediscontinuedimmediately
oncehemoglobinnormalizestopreventsideeffectsofiron
Thereticulocytecountbegintoincreasewithin4-7daysafter
initiationoftherapyandpeakat1.5weeks.
Typicallyforironreplacementtherapy,upto200mgofelemental
ironperdayisgiven,usuallyasthreeorfourirontablets(each
containing50-65mgelementaliron)givenoverthecourseofthe
day.
Adoseof200mgofelementalironperdayshouldresultin
absorptionofironupto50mg/day.Thissupportsaredcell
productionlevelof2-3timesnormalinanindividualwithanormally
functioningmarrowandappropriateerythropoietinstimulus.
Asthehemoglobinlevelrise,erythropoietinstimulationdecreases,
andtheamountofironabsorbedisreduced.
Thegoaloftherapyinindividualswithiron-deficiencyanemiaisnot
onlytorepairtheanemia,butalsotoprovidestoresofatleast0-5-1

onlytorepairtheanemia,butalsotoprovidestoresofatleast0-5-1
gofiron.
Thissustainedtreatmentforaperiodof6-12monthsaftercorrection
oftheanemiawillbenecessary.

1125.Whichofthefollowingisnotexpected
inacaseofMicrocyticHypochromic
anemia

a)ReducedserumIron
b)ReducedtotalRBCdistributionwidth
c)NormalFerritinlevels
d)IncreasedTIBC
CorrectAnswer-B
Answer-B.ReducedtotalRBCdistributionwidth
Thefirstchangeinirondeficiencyanemiaisdecreasedinironstore,
whichismanifestedasdecreasedserumferritinlevel.
Bonemarrowirondecreasesearlierthanserumiron.
Thereismicrocytichypochromicanemia(microcytosisprecedes
hypochromia).

1126.Zievesyndromeischaracterizedbyall
except
a)Alcoholabuse
b)Hemolysis
c)Hypertriglyceridemia
d)Pancreaticlipasedeficiency
CorrectAnswer-D
Answer-D.Pancreaticlipasedeficiency
Zieve'ssyndromeisanacutemetabolicconditionthatcanoccur
duringwithdrawalfromprolongedalcoholabuse
Zievesyndromeisarareconditioncharacterizedbyhemolytic
anemiainconjunctionwithsecondaryhyperlipidemiainpatients
sufferingfromalcohol-relatedtoxicliverdamage.

1127.Hemoglobinwithzeta2andgamma2
chainsareseeninwhichofthe
following

a)GowerI
b)GowerII
c)Portland
d)FetalGb
CorrectAnswer-C
Answer-C.Portland
HbGowerIZeta2/epsilon2
HbPortlandZeta2/gamma2
HbGowerHAlpha2/epsilon2

1128.HAMtestisbasedupon:
a)GPIAnchorProteins
b)Complement
c)Spectrinprotein
d)Mannosebindingproteins
CorrectAnswer-B
AnswerisB(Complements)
HAMtestisbaseduponsusceptibilityofRBC'stocomplement
mediatedlysisinpatientswithPNH.
HAMtestisusedfbrthediagnosisofPNH(ParoxysmalNocturnal
Haemoglobinuria).HAMtest(Acidifiedserumlysistest)
demonstrateslysisofRBCafteractivationofcomplementbyacid.In
acidifiedserumcomplementisactivatedbythealternatepathway.In
patientswithPNH,RBCareunusuallysusceptibletocomplement,
andundergolysiswhenincubatedwithacidifiedfreshserum.


1129.Allofthefollowingaremajorcomplicationsofmassivetransfusion,
except:
a)Hypokalemia
b)Hypothermia
c)Hypomagnesemia
d)Hypocalcemia
CorrectAnswer-A
Ans:A.)Hypokalemia.
Complicationsusuallyseenwithmassivebloodtransfusionare
1)hyperkalemia,
2)hypocalcemia,
3)hypomagnesemia
4)hyperammonemia,
5)hypothermia,
6)Acidosis
7)dilutionalcoagulopathiesandDIC(mostworrisomeproblemafter
massivetransfusionandistheusualcauseofdeathaftermassive
bloodtransfusion)and
8)ARDS.
Thelethaltriadofacidosis,hypothermia,andcoagulopathy
associatedwithMTisassociatedwithahighmortalityrate.

1130.AllaretrueaboutCNSleukemia
except
a)CNSirradiationisgiven
b)Intrathecalmethotrexateisgiven
c)Seenwithacutemyeloidleukemia
d)SingleblastinCSFissufficientfordiagnosis
CorrectAnswer-C
Answer-C.Seenwithacutemyeloidleukemia
MostchildrenwithleukemiahavesubclinicalCNSinvolvementatthe
timeofdiagnosis.
Fewchildrenshowcentralnervoussysteminvolvementatthetime
ofdiagnosis,mostareasymptomaticbutsomehavefeaturesof
raisedintracranialtension.
CNSinvolvementismostlyduetoALL.CNSinvolvementismore
commoninALLthanAML.

1131.ThetissueoforiginoftheKaposi's
sarcomais?
a)Lymphoid
b)Vascular
c)Neural
d)Muscular
CorrectAnswer-B
Ansis'b'i.e.,Vascular
KaposiSarcoma
Kaposisarcomaisanintermediategrade,multicentricvascular,
tumour
*Thepathogenesisofkaposisarcomaiscomplex,fundamentallyit
isanangioproliferativediseasethatisnotatrueneoplasticsarcoma.
Itisamanifestationofexcessiveproliferationofspindlecellsthatare
believedtobeofvascularoriginandhavefeaturesincommonwith
endothelialandsmoothmusclecells.
*Grossly-Threestagesofthediseasecanbeidentified
Patch(1ststage)
Plaque(intermediatestage)
LINodule(laststage)
*Histologically-Thechangesarenonspecificintheearlypatchstage
andmorecharacteristicinthenodularstagewhichshowsheetsof
plumpproliferatingspindlecellsandendothelialinthedermisor
subcutanoustissue

1132.Thrombocythemiaischaracterizedby
a)Plateletselevation
b)Lowplatelets
c)Neutrophilia
d)Monocytosis
CorrectAnswer-A
Answer-A.Plateletselevation
Thrombocythemiaorthrombocytosisistheelevationofplatelets.

1133.Whichofthefollowingisrequiredfor
propereffectsofInsulin?
a)Selenium
b)Iron
c)Copper
d)Chromium
CorrectAnswer-D
Inassociationwithinsulin,chromiumpromotestheutilizationof
glucose
Chromiumisacomponentofaproteinnamelychromodulinwhich
facilitatesthebindingofinsulintocellreceptorsites
Chromium
Itisanessentialnutrientforthemaintenanceofnormalglucose
tolerance
Itsdeficiencycausesinsulinresistance.
Chromiumadministrationhasalsobeenshowninseveralstudiesto
lowerglucoseandinsulinlevelsinpatientswithtype2diabetes.
Ithasbeenclassifiedasnotessentialformammals.(Cr(III)or
Cr3+).
Chromiumdeficiencyiscontroversialorisatleastextremelyrare.
Ithasbeenattributedtoonlythreepeopleonparenteralnutrition,
whichiswhenapatientisfedaliquiddietthroughintravenousdrips.
Incontrast,hexavalentchromium(Cr(VI)orCr6+)isverytoxicand
mutagenicwheninhaled.
Cr(VI)hasnotbeenestablishedasacarcinogenwheninsolution,
althoughitmaycauseallergiccontactdermatitis(ACD).
Dietarysupplementsforchromiumincludechromium(III)picolinate,
chromium(III)polynicotinate,andrelatedmaterials.

chromium(III)polynicotinate,andrelatedmaterials.
Glutathioneperoxidaserequiresselenium
Copperisanimportantconstituentofcatalse,cytochromeoxidase
andtyrosinase.
Zincisalsonecessaryforthestorageandsecretionofinsulin

1134.Glucosefeverisrelatedwith-
a)Glucagon
b)Parathyroid
c)GH
d)Aldosterone
CorrectAnswer-D
Answer-D.Aldosterone
HypoglycemiainAddisondiseaseismanagedwith
hydrocortisone/dexamethasone.
AdministrationofI.V.glucoseinAddisonleadstodevelopmentof
feverandiscalledas"glucosefever".
Inpatientswithadrenalinsufficiency,whohavenotreceived
glucocorticoidsglucoseinfusionmaycausehighfever(glucose
fever)followedbycollapseanddeath.

1135.Notassociatedwithdiabetesmellitus
a)Cushingsyndrome
b)Acromegaly
c)Hypothyroidism
d)Pheochromocytoma
CorrectAnswer-C
Ans.is'c'i.e.,Hypothyroidism

1136.PatientoninsulininCKDstage4.What
isthedoseadjustmentofinsulin
required?

a)Increasedinsulin
b)Decreasedinsulin
c)Normalinsulin
d)AddDPP-4inhibitors
CorrectAnswer-B
Answer-B.Decreasedinsulin
Insulinrequirementsshowabiphasiccourseindiabeticpatientswith
renaldisease.Itisnotuncommonforglucosecontroltodeteriorate
asrenalfunctiondeteriorates,asincreasinginsulinresistancecan
affectbothtypeIandtype2diabetics.

1137.Whichisthebestindicatorforshort
termcontrol(2-3weeks]ofblood
glucose?

a)Serumfructosamine
b)HbAlc
c)Bloodsugar
d)Urinesugar
CorrectAnswer-A
Answer-A.Serumfructosamine
SerumfructosamineTellssugarfluctuationsin2-3weeks
GlycosylatedhemoglobinTellssugarfluctuationsinprevious6-8
weeks.

1138.PostPrandialcapillaryglucoseshould
bemg/dlforadequatediabetescontrol
a)<100mg/dl
b)<140mg/dl
c)<180mg/dl
d)<200mg/dl
CorrectAnswer-C
Answer-C.<180mg/dl
HbAIC-two
Preprandialcapillaryplasmaglucose-70-130mg/d1<
Peakpostprandialcapillaryplasmaglucose-180mg/d1<
Bloodpressure-130/80

1139.Hyperpigmentationisseenwithwhich
hormone?
a)FSH
b)LH
c)TSH
d)ACTH
CorrectAnswer-D
Answer-D.ACTH
Hyperpigmentationoftheskinandmucousmembranesoften
precedesallothersymptomsbymonthstoyears.
Itiscausedbythestimulanteffectofexcessadrenocorticotrophic
hormone(ACTH)onthemelanocytestoproducemelanin.
ThehyperpigmentationiscausedbyhighlevelsofcirculatingACTH
thatbindtothemelanocortin1receptoronthesurfaceofdermal
melanocytes.
Othermelanocyte-stimulatinghormonesproducedbythepituitary
andothertissuesincludealpha-MSH(containedwithintheACTH
molecule),beta-MSH,andgamma-MSH.Whenstimulated,the
melanocytechangesthecolorofthepigmenttoadarkbrownor
black.
TheincreasedMSHinAddison'scausesmelanocytestodisperse
melaninintheepidermisthusincreasingpigmentation.

1140.ThemostcommoncauseofCushing's
syndromeis:
a)Pituitaryadenoma
b)Adrenaladenoma
c)EctopicACTH
d)Iatrogenicsteroids
CorrectAnswer-D
AnswerisD(Iatrogenicsteroids):
"ThemostcommoncauseofCushing'ssyndromeisIatrogenic
administrationofsteroidsforavarietyofreasons."


1141.Themostcommoncauseofmalignant
adrenalmassis
a)Adrenocorticalcarcinoma
b)Malignantphaeochromocytoma
c)Lymphoma
d)Metastasisfromanothersolidtissuetumor
CorrectAnswer-D
Ans.is'd'i.e.,Metastasisfromanothersolidtissuetumor
Themostcommoncauseofadrenaltumorsismetastasisfrom
anothersolidtumorlikebreastcancerandlungcancer.

Malignant
Percentage
Adrenocorticalcarcinoma
2-5%
Malignant
<I%
pheochromocytoma
Adrenalneuroblastoma
<0-1%
Lymphomas(incl.primary
<1%
adrenalymphoma)
Metastases(mostfrequent:
15%
Breast,lung)

1142.PrimaryHyperaldosteronismcanbe
diagnosedbyallofthefollowing
criteria,except:

a)DiastolicHypertensionwithoutedema
b)Hyperaldosteronismwhichisnotsupressedbyvolume
expansion
c)LowPlasmaReninActivity
d)MetabolicAcidosis
CorrectAnswer-D
AnswerisD(MetabolicAcidosis)
Class,TriadofBiochemicalCriteriafordiagnosisofPrimary
Hyperaldoteronism
Hypokalemiawithinappropriatekaliuresis(Metabolicalkalosis)
Suppressedplasmareninactivity
ElevatedAldosteronelevelsthatdonotfallappropriatelyin
responsetovolumeexpansionorsodiumload
TakenfromManualofEndocrinology&Metabolism41h/150

1143.MostcommoncauseofAddison's
DiseaseinIndiais:
a)Autoimmune
b)Postpartum
c)HIV
d)Tuberculosis
CorrectAnswer-D
AnswerisD(Tuberculosis):
'Thecommonestcauseofadrenalinsufficiency(Addison'sdisease)
inunderdevelopedcountriesisTuberculosis


1144.Femalewithbloodsugarof600mg%
andsodiumof110mEq.Insulinwas
given,whatwillhappentoserum
sodiumlevels?

a)Sodiumincrease
b)Sodiumdecrease
c)Sodiumunaffected
d)Relativesodiumdeficiency
CorrectAnswer-A
Answer-A.Sodiumincrease
Astheglucoseleveldecreases,thereisdecreaseintheosmolarity
ofextracellularfluid.Thiscausesmovementofintracellularfluid
backintothecellularcompartmentproducingincreaseinserum
sodium.

1145.Whichofthefollowingpresentswith
hypokalemiaandmetabolicacidosis?
a)Diarrhea
b)Vomiting
c)Nasogasticsuction
d)Nasogasticsuction
CorrectAnswer-A
Ans-A.Diarrhea
Diarrheacauseshypokalemiawithmetabolicacidosis.
Vomitingnosogastricsuctionandconn'ssyndromecausemetabolic
alkalosis.

1146.HypertensionwithHypokalemiaisseen
in:
a)BartterSyndrome
b)Liddle'sSyndrome
c)Gitelman'sSyndrome
d)Alloftheabove
CorrectAnswer-B
AnswerisB(Liddle'sSyndrome)
Liddle'sSyndromeistypicallyassociatedwithHypokalemiaand
Hypertension.Rartter'sSyndromeandGitelman'sSyndromeare
alsoassociatedwithhypokalemiabutwithouthypertension.

Liddle'sSyndrome:Review
Pathophysiology:
?Autosomaldominantdisorder.
?Geneticdefectinthecollectingtubulesodiumchannel,resultinginincreasedsodiumreabsorptionandlack
ofinhibitionbyhigherlevelsofintracellularsodium
AgeofPresentation
?Oftendiagnosedatyoungage,butcanpresentinadulthoodduetophenotypicvariation.
Clinicalpresentation
?Classictriadofhypertension,metabolicalkalosis,andhypokalemia.
?Consideriffamilyhistoryofhypertensionand/orhypokalemia.atyoungage
Labdata
?Metabolicalkalosis,hrpokalemia(althoughsomearelownormal),lowurinaryaldosteronesecretion
Treatment:
?Lifelong.Potassium-sparingdiureticwhichclosesthesodiumchannel(AmilorideorTriamterene).
Spironolactonedoesnotworkbecausealdosteroneisnotcausingthesodiumchanneltobeopen.


1147.Incobalamindeficiencywhichisnot
seen
a)Microcyticanemia
b)Longtractsigns
c)Lossofproprioception
d)Rhombergsign
CorrectAnswer-A
Answer-A.Microcyticanemia
Cobalminedeficiencycausesmegaloblastic(macrocytic)anemia
(notmicrocytic).
Cobalaminedeficiencyalsocausessubacutecombined
degenerationofspinalcordduetoinvolvementofposteriorcolumn,
affectingvibration,finetouch,andimbolance(Rombergsign).

1148.HypernatremiacausesallEXCEPT
a)Seizure
b)Thrombus
c)Brainhemorrhage
d)Centralpontinemyelinosis
CorrectAnswer-D
Answer-D.Centralpontinemyelinosis
Complicationofhypernatremiaarebrainhemorrhage,seizures,
coma,thromboticcomplicationsandraisedICT.
Centralpontinemyelinosisisclassicallyassociatedwithoverlay
rapidcorrectionofhyponatremia.
Clinicalfeatures-
Patientsareirritable,restlessweakandlethargic
Somehavehighpitchedcryandhyperpnea.
Alertpatientareverythirsty.
Hypernatremiacausesfeveralthoughmanypatientshave
underlyingprocessthatcontributestothefever.

1149.Allofthefollowingareassociatedwith
hyponatremiaexcept
a)anorexia
b)Convulsions
c)Drowsiness
d)Myalgia
CorrectAnswer-D
Answer-D.Myalgia
Anorexia,nausea&vomiting
Coma
Convulsions
Drowsiness
Headache
Circulatoryfailureandhypotension
Hyponatremiacanalsocausemusclecrampsandweakness.

1150.Mostcommontypeofmultiple
sclerosis?
a)Relapsingremittingtype
b)Secondryprogressivemultiplesclerosis
c)Progresiverelapsingmultiplescelrosis
d)Primaryprogresivemultiplesclerosis
CorrectAnswer-A
Answer-A.Relapsingremittingtype
Repapsing-remittingmultiplesclerosis(RRMS)
Thisisthemostcommonformofmultiplesclerosis.
About85%ofpeoplewithM.S.areinitiallydiagnosedwithrelapsing-
remittingmultiplesclerosis.

1151.InEEGwhichtypeofwavesareseenin
metabolicencephalopathy
a)Alpha
b)Beta
c)Gamma
d)Delta
CorrectAnswer-D
Answer-D.Delta
E.E.G.hasbeenwidelyusedtoevaluatemetabolicencephalopathy.
TheE.E.G.findingsareabnormalinacuteencephalopathystages
Metabolic
'vthm
encephalopathy
GradeI(almost
Dominantactivityisrhythmwithminima,theta
normal)
activity
GradeII(mildly
Dominantthetabackgroundwithsomealpha
abnormal)
anddeltaactivities
GradeII
Continuousdeltaactivetypredominates,little
(morderately
activityoffasterfrequencies
abnormal)
GradeIV(severely
Low-amplitudedeltaactivityorsuppression-
abnormal)
burstpattern
GradeV(extremely Nearly"flat"tracingorelectrocerebralinactivity
abnormal)


1152.Glasgowcomascalemotor4
represents?
a)Withdrawalorflexion
b)Decorticateposturing
c)Decorticateposturing
d)Localisepain
CorrectAnswer-A
Answer-A.Withdrawalorflexion

1153.Apatientafteranaccidentwas
unconscious.Onphysicalexamination
therewasunilateralpupillarydilatation
Possiblereasonforthesameis

a)Uncalherniation
b)Tonsillarherniation
c)Cingulateherniation
d)Transcalvarialherniation
CorrectAnswer-A
Answer-A.Uncalherniation
Transtentorialherniationisthedisplacementofmedialtemporallobe
intothetentorialopeningitisusuallyseenafterextradural
hemorrhage.

1154.IncreasedICTisshownby
a)Miosis
b)Systemichypotension
c)Tachycardia
d)ReductioninGCS
CorrectAnswer-D
Answer-D.ReductioninGCS
IncreasedICTleadstobradycardiawithHypertension.Uncal
herniationofbrainleadstoipsilateralpupillarydilatation.Reduction
inGCSduetodamagetoreticularactivatingsystemleadsto
developmentofcoma.

1155.Lateralmedullarysyndromeisduetotheocclusionofwhichofthefollowing
vessels?
a)Posteriorsuperiorcerebellarartery
b)Anteriorinferiorcerebellarartery
c)Basilarartery
d)Vertebralartery
CorrectAnswer-D
LateralmedullarysyndromeisotherwiseknownasWallenberg'ssyndromeorPICAsyndromeor
vertebralarterysyndrome.
Occlusivediseaseoftheintracranialsegmentofthevertebralarteryisamuchmorefrequent
causeofthelateralmedullarysyndrome.
Signsandsymptomsinclude:
Ipsilateralside
Horner'ssyndrome
Decreaseinpainandtemperaturesensationonipsilateralsideofface
Cerebellarsigns(ataxia)
Contralateralside:
Decreasedpainandtemperatureoncontralateralbody
Dysphagia,dysarthria,hoarseness,paralysisofvocalcord
Vertigo,nausea,vomiting,hiccups
Nystagmus,diplopia
Nofacialorextremitymuscleweaknessseeninthissyndrome.
Ref:PhysicalMedicineandRehabilitationBoardReviewBySaraCuccurullo,2004,Page
11;Harrison'sInternalMedicine17thedChapter364.CerebrovascularDiseases,
BrainstemdisordersbyPeterPUrban,LouisRCaplanpage205-207.

1156.Mostcommonsiteofhypertensive
intraparenchymalhemorrhageinbrain:
March2013

a)Pons
b)Thalamus
c)Putamen
d)Cerebellum
CorrectAnswer-C
Ans.Ci.e.Putamen
Sitesofintracerebralhemorrhageincludesputamen(55%),
thalamus(20-3-%),cerebrllum(10%),pons(5-7%)andsubcortical
whitematter(10-15%)


1157.Plaquesjaunesareseenin
a)Syphilis
b)Headinjury
c)Endocarditis
d)Atherosclerosis
CorrectAnswer-B
Answer-B.Headinjury
PlaqueJaunesisatermusedtodescribethecharacteristicgross
appearanceofoldtraumaticcontusionsonthesurfaceofbrainfrom
previousheadinjuries.

1158.RademeckercomplexinEEGisseenin
-
a)SSPE
b)vCJD
c)cCJD
d)Kuru
CorrectAnswer-A
Answer-A.SSPE
Characteristicperiodicactivity(Rademeckercomplex)isseenon
EEGshowingwidespreadcorticaldysfunctioninSSPE.
Itischaracterisedbyhighvoltagespikeoccuningathiglrlrequency
of0.5-1.5seconds.

1159.Commonestcauseofcerebrovascular
accident
a)Infarction
b)Infarction
c)Embolism
d)Aorticdissection
CorrectAnswer-A
Answer-A.Infarction
1. Ischemic(85%)(infarction):Causesareembolism(75%ofischemic
stroke)andthrombosis(25%ofischemicstroke).
2. Hemorrhagic(15%):Intraparenchymal,subdural,epidural,
subarachnoid.

1160.Whichofthefollowingsitesis
responsiblefortheamnesticdefectin
Wernicke'sKorsakoffsyndrome:

a)Mamillarybody
b)Thalamus
c)PeriventricularGreymatter
d)Hippocampus
CorrectAnswer-B
AnswerisB(Thalamus):
TheAmnesticeffectinWernicke'sKorsakoffSyndromeisrelatedto
lesionsinthedorso-medialnucleiofthethalamus.
`Lesionsinthedorsomedialnucleusofthethalamusseemtobethe
bestcorrelateofthememorydisturbanceandconfabulation'-
Robbins


1161.CriteriaforBrainstemdeathincludes:
a)PositiveDoll'seyeReflex
b)Absentpupillarylightreflexanddelatedpupils
c)Pinpointpupils
d)Positivevestibulo-ocularreflex
CorrectAnswer-B
AnswerisB(Absentpupillarylightreflexanddelatedpupils):
Brainstemdeathisdefinedbytheabsenceofallbrainstemmediated
cranialnervereflexes.
PupillaryLightReflexisabrainstemmediatedcranialnervereflex
thatisabsentinbrainstemdeath.Thepupilsareusuallymidsized
butmaybedilated(shouldnothoweverbesmall).
Occulo-cephalic(Doll'seye)reflexandVestibulo-ocular(Caloric)
reflexarebothbrainstemmediatedcranialnervereflexsthatshould
beabsentinBrainstemdeath.
CriteriaforBrainDeath/Brainstemdeath
ThedefinitionofBrainstemdeathrequiressimultaneous
demonstrationthatthepatienthasirreversiblylostthecapacityof
consciousness(Coma)andthecapacitytobreathe(Apnoea)bothof
whicharedependentonintactbrainstem.
ClinicalassessmentoftheintegrityofBrainstemhastwo
componentsincludingassessmentoftheintegrityofbrainstem
mediatedcranialnervereflexesandtheApneatest.
DeepUnresponsiveComa
Patientswithbraindeathshowthedeepestcomapossiblewithtotal
unresponsivenesstoallstimuli.
Nospontaneousmovement
Noresponsetoexternalstimuli(Verbal/Deeppain)

AbsenceofallBrainstemCranialReflexes
Allreflexesmediatedbycranialnervemustbeabsent
AbsentPupillaryLightreflex(CNII,III)
(Pupilsareusuallymidsizedbutmaybeenlargedbuttheyshould
notbesmall)
AbsentCornealReflex(CNV,VII)
AbsentVestibulocochlearReflex(CNIII,IV,VI,VIII)(Noeye
movementinresponsetocaloricirrigationofears)
AbsentOculocephalicReflex
(AbsentDoll'seyereflex)Eyeswillmovewiththehead
(Nooccularmovementsonrapidturningofhead).
AbsentGagReflex(CNIX,X)
AbsentTrachealCoughReflex(CNX)
AbsentCentralMotorResponsetoPain(CNV,VII)(Noresponseto
deepsomaticstimulation).
CompleteAponeainpressureofhypercarbia(Absenceof
BrainstemRespiratoryReflex)
Norespiratoryeffortinresponsetohypercarbianeedstobe
demonstratedtoshowthataponeaisduetobrainstem(medullary)
damage
Absentrespiratorymovementduringdisconnectionfromthe
ventilatorwiththePaCO2>60mmHgisrequiredforthetesttobe
valid
CranialnervereflexesinBrainStemDeathtesting:
Cranial
Reflex
Notes
nerves
Pupillarylight HAI
Usebrightlightsource(not
reflex
ophthalmoscope)inadimmed
environment.Lookforboth
directandconsensualreaction.
Importantreflexthatinterrogatesatlevelof
midbrain
Strokecorneawithgauze,whilstgently
Cornealreflex V,VII
holdingeyesopen;avoidtraumato
cornea.
ThevariousnucleiofVarefound
throughoutthewholelengthofthe
brainstem,whilst


brainstem,whilst
thatofVII(facialnerve)isintheupper
medulla.
Central
Applydeeppressurestimulationcentrally
V,VII
responseto
(e.g.supra-orbitalridge)andperipherally
(e.g.nailbed).Lookforcentralmotor
deepsomatic
responseinthedistributionofthefacial
nerve.
Peripheralstimulationmayillicitperipheral
stimulation
spinalreflexes.
III,IV,VI, Checkpatencyofexternalauditorycanal
Coldcaloric
VIII
withauroscope.Flexheadto30?(orapply
30?headuptiltifcervicalspineinjuryis
vestibulo-
suspected).Slowlyirrigatecanalwith
ocular
5OrriL
reflex
ice-coldwaterover60s.Observefor
nystagmusforafurther30s.Contra-
indicatedin
trauma-relatedotorrhea.
ThenucleiofIIIandIVliewithjinthe
midbrain,whilstthoseofVIandVIIIarein
the
medulla.
Oculocephalic
Rapidlateralmovementofthehead
VIII,Ill,VI
reflex
normallyresultsineyedeviationtothe
(Doll'seye
contralateralside,testingbrainstemgaze
reflex)
mechanism.
Inbrainstemdeatheyesremaininafixed
positionwithintheorbit.
Gagreflex
IX,X
Stimulateuvulaunderdirectvisionwith
throatspatula,observingforcontractionof
softpalate.
ThenucleiofIXandXlieinthemedulla.
Tracheal
X
Exposepatienttoumblicus.Stimulate
coughreflex
tracheatolevelofcarinabyintroductionof
sterilesuctioncatheterdownendotracheal
tube.Observeforcoughresponse

tube.Observeforcoughresponse

1162.Whichofthefollowingisthemost
commoninitialpresentingfeatureof
multiplesclerosis:

a)OpticNeuritis
b)CerebellarAtaxia
c)Internuclearophthalmoplegia
d)Diplopia
CorrectAnswer-A
AnswerisA(OpticNeuritis):
OpticNeuritisisthemostfrequentinitialpresentingfeatureofMS
amongsttheoptionsprovided.
Themostcommonearliestpresentingfeaturesofmultiplesclerosis
aretransientsensorydefectsfollowedbyvisualdisturbancesdueto
opticneuritis.
InitialSymptomsofMS
Percentof
Percentof
Symptom
Symptom
Cases
Cases
Sensoryloss
37
Lhermitte's 3
Opticneuritis
36
Pain
3
Weakness
35
Dementia
2
Paresthesias
24
Visualloss 2
Diplopia
15
Facialpalsy 1
Ataxia
11
Impotence
1
Vertigo
6
Myokymia
1
Paroxysmal
4
Epilepsy
1
attacks
4
1

4
1
Bladder
Falling

1163.AbsenceofCorpuscallosumleadsto
a)Hemiparesis
b)Hemisensoryloss
c)Astereognosis
d)Noneurologicalmanifestations
CorrectAnswer-D
Noneurologicalmanifestations[Ref:Internetreference]
Agenesisofcorpuscallosumisararebirthdefect(congenital
disorder)inwhichthereiscompleteorpartialabsenceofcorpus
callosum.
Corpuscallosumisabandoftissueconnectingthetwohemispheres
ofthebrain.
Fibresofcorpuscallosumarisefromthesuperficial
layersofthecerebralcortexandtheyprojecttothehomotypic
regions
ofthecontralateralcortexbypassingthroughthecorpus
collosumwhilecrossingthemidline.
ClinicalfeaturesofAgenesisofcorpuscallosum
Signsandsymptomsofagenesisofcorpuscallosumvarygreatly
amongindividuals.
Patientsusuallydonothaveanyneurologicalmanifestations.
However,somefeaturescommoninagenesisofcorpuscallosum
are:-Visionimpairment
*Lowmuscletone(hypotonia)
Poormotorcoordination
*Delayinmotormilestonessuchassittingandwalking.
Lowperceptionofpain
*Delayedtoilettraining
*Chewingandswallowingdifficulties
*Earlyspeechandlanguagedelays


Socialdifficulties
Othercharacteristicssometimesassociatedwithcallosaldisorders
are:-Seizures
*Spasticity
*Early,feedingdifficultiesandorgastricreflux-.
*Hearingimpairments
*Abnormalheadandfacialfeatures
*Mentalretardation
Investigation
CTandMRIreveal"Batwing"deformityoftheventricles.
Treatment
Therearecurrentlynospecificmedicaltreatmentsforcallosal
disorders.
Animportantpoint
Theneurologicalabnormalitiesassociatedwithcorpuscallosumare
notcausedbyabsenceofcorpuscallosumperse.
*Theseconditionsarebelievedtobecausedduetoassociated
cerebralanomaliesratherthanincorpuscallosumpersc.
*Themostcommonassociatedbrainanomalieswithabsentcorpus
callosumare
*Dandywalkermalformation
*lnterhenfisphericcystwithhydrocephalus
*Migrationaldisorder
*Absenceoftheinferiorvertnis
*Thechildrenwhohadthebestprognosiswithoutanysignificant
neurologicalsequalewerethosewithisolatedagenesisofcorpus
collosum.
*Thechildrenwiththeworstprognosisandneurologicalsequale
werethosewithagenesisofcorpuscallosumandmigrational
disorderwithorwithoutDandy--walkermalformation.
*Henceprognosisisdeterminedprimarilybytheunderlyingor
associatedmalformation.


1164.Whichoneofthefollowingiscorrect
regardingEaton-Lambertsyndrome-
a)Itcommonlyaffectstheocularmuscle
b)Neostigmineisthedrugofchoiceforthissyndrome
c)Repeatedelectricalstimulationenhancesmusclepowerinit.
d)Itiscommonlyassociatedwithadenocarcinomaoflung
CorrectAnswer-C
AnswerisC(Repeatedelectricalstimulationenhancesmuscle
power):
'PatientswithLambert?Eatenmyaethenicsyndromeshow
incrementalratherthandecrementalresponseonrepeatednerve
stimulation'?Harrison17th/2674
'Muscleresponsetostimulationofitsmotornerveincreases
remarkablyifnerveisstimulatedrepeatedlyeveninmusclesthatare
clinicallyweak'-CMDT
LambertEatenMyastheniaSyndromecommonlyinvolves
proximallimbmusclesandmusclesoftrunk
Extraocularmusclesarethemostcommonlyinvolvedin
MyaestheniaGravisandnotinLambertEatenMyasthenia
Syndrome
Neostigmineisnotconsideredthedrugofchoice
Plasmapharesisandimmunosuppressionformmainstayof
treatment3,4Diaminopyridineisthedrugofchoicefor
enhancementofneuromusculartransmission.
Pyridostigmine(orNeostigmine)maybesympatomaticallyhelpful
buttheirresponseisvariable?Harrison&CMDT
LambertEatenMyastheniaSyndromeisassociatedwithsmall
cellcarcinomaoflung


LambertEatenMyastheniaSyndromeisassociatedwithsmallcell
carcinomaoflungandnotadenocarcinoma


1165.ThefollowingarecomponentsofBrown
Sequardsyndromeexcept:
a)Ipsilateralextensorplantarresponse
b)Ipsilateralpyramidaltractinvolvement
c)Contralateralspinothalamictractinvolvement
d)Contralateralposteriorcolumninvolvement
CorrectAnswer-D
AnswerisD(Contralateralposteriorcolumninvolvement):
Brown?Sequardsyndromeorhemisectionofthespinalcordleads
tolossofjointpositionandvibratorysense(posteriorcoluntn
movement)ontheipsilateralsideandnotonthecontralateralside.?
Harrison16th/2441,144
Brownsequardsyndrome:Hemisectionofspinalcord
Ipsilateralinvolvementofcorticospinaltract:ipsilaterallossofmotor
power.
Ipsilateralinvolvementofposteriorcolumn:ipsilaterallossofjoint
positionandvibratorysense.
Contralateralinvolvementofspinothalamictract:contralateralloss
ofjointpositionandvibratorysense.
Segmentalsignssuchasradicularpainmuscleatrophyorlossof
deeptendonreflexesarcunilateral(Lowermotorneuronsignsat
leveloflesion).


1166.FirstSymptomsofparkinsonsdisease
is-
a)Posturalinstability
b)Rigidity
c)Tremors
d)Bradykinesia
CorrectAnswer-C
Answer-C.Tremors
Parkinsonismisaprogressivedegenerative,extrapyramidaldisorder
ofmusclemovement,duetodysfunctioninbasalganglia,
comprisingfourcardinalfeatures
Bradykinesiaorhypokinesia
Musclerigidity
Restingtremor

1167.Whichofthefollowingmetalionsis
associatedwithsecondary
Parkinsonisms:

a)Mangnese(Mn)
b)Magnesium(Mg)
c)Selenium(Se)
d)Molybednum(Me)
CorrectAnswer-A
AnswerisA(Mangnese(Mn)):
Manganeseionexposureisimplicatedinthefreeradicaldamageof
thebasalgangliacausingParkinsonism.
ToxinsImplicatedinParkinsonism(Harrison)
Manganese(Mn)
MPTP(1Methyl->Phenyl?1,2,3,6tetrahydropyridine)
Carbonmonoxide
Carbondisulphide
Ctanide
Hexane
Methanol
ToxinsreportedtoinduceParkinonism:(HandbookofAtypical
Parkinsonism)
Betelnut(plusantiPsychotics)
Carbonmonoxide
Contrastagentforcardiaccatheterization
Cyanide
Ethanolintoxication,ethanolwithdrawal
Ethyleneglycol

Herbicides(paraquat,diquat,glyphosate)
Heroin
Hydrogensulfide
Kava-kava
Manganese
Maneb(ManganeseEthylene-Bis-Dithiocarbamate)
Mercury
Methanol
Methcathinone(manganeseephedrone)
MPTP(I-methyl-4-phenyl-1,2,3,6-tetrahydropyridine)
Organicsolvents(carbondisulfide,n-hexane,toluene,
trichloroethylene)
Organophosphateinsecticidepoisoning
Petroleumproducts

1168.Isaacsyndromaischaracterisedby-
a)Peripheralnerveexcitability
b)Opsoclonus
c)Encephalomyelitis
d)Limbicencephalitis
CorrectAnswer-A
Answer-A.Peripheralnerveexcitability
Isaccsyndrome(neuromyotonia)
Peripheralnervehyperexcitability
Spontaneousandcontinuousmusclefiberactivityofperipheral
nerveorigin.
Clinicalfeaturesincludecramps,muscletwitching(fasciculationsor
myokymia)
Stiffness
Delayed
Musclerelaxation(pseudomyotonia)
Spontaneousorevokedcarpalorpedalspasms.

1169.InAlzheimer'sdisease(AD)whichof
thefollowingisnotseen:
a)Aphasia
b)Acalculia
c)Agnosia
d)Apraxia
CorrectAnswer-C
Ans.None>C.AgnosiatRej.KaplananalJaadocksSynopsisof
Psychiatry10/cpi-t?
Aphasia,Apraxia,AcalculiaandAgnosiamayallbeseenin
Alzheimer'sDementia.AgnosiainAlzheimer'sdiseaseusually
presentslateinthediseaseandisnotincludedintheICD-10
DiagnosticcriteriafordementiainAlzheimer'sdiseasewithearly
onsetandhencemaybeselectedasthesinglebestanswerby
exclusion


1170.DOCfortreatmentofSSPE-
a)Abacavir
b)Inosinepranobex
c)Glatiramer
d)Interferon
CorrectAnswer-B
Answer-B.Inosinepranobex
Inosinepranobexisusedasanimmune-modulatorforthe
managementofpatientswith-
Immune-depressionsufereingfromviralinfectionsasSSPE
recurrentherpessimplexgenitalwarts.

1171.Mostcommonacutecomplicationof
dialysisis
a)Hypotension
b)Bleeding
c)Dementia
d)Musclecramps
CorrectAnswer-A
Ans.is'a'i.e.,Hypotension
Hypotensionisthemostcommonacutecomplicationof
hemodialysisparticularlyamongpatientswithdiabetesmellitus.
Factorsinvolvedare:-
Excessiveultrafiltration,withinadequatecompensatoryvascular
filling,impairedvasoactiveorautonomicresponse,osmolarshifts,
overzealoususeofantihypertensives


1172.Whichvitamintoxicityisassociated
withexcessivesweating-
a)Choline
b)Biotin
c)Foliccid
d)VitaminB
CorrectAnswer-A
Answer-A.Choline
Toxicityfromcholineresultsin-
Hypotension
Cholinergicsweating
Diarrhea
Salivation
Fishybodyodor

1173.Dermatitismaybeaclinical
manifestationofdeficiencystatesofall
offollowingnutrientsexcept-

a)Biotin
b)Niacin
c)Pyridoxine
d)Thiamine
CorrectAnswer-D
Answer-D.Thiamine
VitaminB3(Niacin)deficiencycausesdermatitis.BiotinandVitamin
B6(pyridoxin)deficiencycausesseborrheicdermatitis.

1174.Aspirindecreasestheriskof
developmentofwhichofthefollowing-
a)Colorectalcancer
b)Stomachcancer
c)Carcinoid
d)MALToma
CorrectAnswer-A
Ans.A.Colorectalcancer
Regularaspirinusereducestheriskofcolonadenomasand
carcinomasaswellasdeathfromlarge-bowelcancer

1175.Lafora'sdiseasepresentswith-
a)G.T.C.S
b)Myoclonicepilepsy
c)Petitmalepilepsy
d)Partialseizures
CorrectAnswer-B
Answer-B.Myoclonicepilepsy
Loforadiseaseisanautosomalrecessivepoliencephalopathyoflate
childhoodorearlyadultlife.
Itischaracterizedbyprogressivedementia,dysarthria,visualloss,
pyramidal&cerebellarsigns,andmyoclonic&otherseizures.
Thediagnosticpathologicalfindingisloforabodies.

1176.Mosaicpatternofcomentlineis
characteristicallyseenin-
a)Hyperthyroidism
b)Paget'sdiseaseofbone
c)Renalosteodystrophy
d)Osteomalacia
CorrectAnswer-B
Answer-B.Paget'sdiseaseofbone
Mosaicpatternofcementlineischaracteristicallyassociatedwith
paget'sdiseaseofthebone

1177.Kaposisarcomaiscommonlyseenin?
a)Upperlimbs
b)Lowerlimbs
c)HeadandNeck
d)Trunk
CorrectAnswer-B
Ans.is'b'i.e.,Lowerlimbs
"Itoccurspredominantlyontheskinandcaninvolvevirtuallyany
organ,perhapsexceptthebrain.Theinitiallydescribedform,now
knownasclassicKS,predominantlyinvolvesthelowerextremitiesof
elderlymen.KSisnowthemostcommontumorseeninHIV-
infectedpatients"-Devita'sOncology


1178.Mostcommoncauseofdeathincancer
is-
a)Bleeding
b)Infection
c)Respiratoryfailure
d)Renalfailure
CorrectAnswer-B
Answer-B.Infection
Themostcommoncausesofdeathinpatientswithcancerare
infection(leadingtocirculatoryfailure),respiratoryfailure,hepatic,
andrenalfailure.Intestinalblockagemayleadtoinanitionand
starvation.

1179.Hyperuricemiacanbecausedbyall
except-
a)Ethanol
b)Thiazide
c)Furosemide
d)None
CorrectAnswer-D
Ans.is.D.None
Causesofdrugordietinducedhyperuricemia
Diuretics(thiazidesandloopdiuretics)
Cyclosporineandtacrolimus
Lowdosesalicylates.
Ethambutol
Pyrazinamide
Ethanol
Levodopa
Methoxyflurane
Laxativeabuse(alkalosis)
Saltrestriction

1180.AllofthefollowingaretrueaboutGout,
except:-
a)Occursduetoaccumulationofuratecrystalsinjoint
b)Canbepptdbypyrazinamide
c)Birefringementcrystalsarepresentinjoint
d)Occursmoreinfemales
CorrectAnswer-D
AnswerisD(Occursmoreinfemales):
Goutismorewidespreadinmenthaninwomen.
Womenrepresentonly5-20%ofpatientswithgout.
Womenareseldomaffecteduntilaftermenopause.


1181.Mostcommonorganismassociated
withreactivearthritisis:
a)Staphylococcus
b)Shigella
c)Chlamydia
d)Yersinia
CorrectAnswer-C
Chlamydia[Ref:Harrison17/ep2113;
http://www.emedicine.com/derm/TOPIC207.HTM;
http://www.entedicine.com/med/TOPIC1998.11TM]
Reactivearthritisisasystemicdisorderofunknownetiologythatis
definedbythedevelopmentofconjunctivitis,urethritis,arthritis,and
mucocutaneouslesionsfollowinganepisodeofinfectionelsewhere
inthebody.
In1916,HansReiterdescribedthetriadofnongonococcalurethritis,
conjunctivitis,andarthritisinayoungGermanofficerwithbloody
dysentery.Theclassictriadofthedisease,namelyurethritis,arthritis,
andconjunctivitis,ispresentinonlyonethirdofthepatients.
Reactivearthritisisfrequentlyassociatedwiththehumanleukocyte
antigenB27
(1-ILA-B27)haplotype.
Theetiologyofreactivearthritisremainsuncertain.Themost
acceptedtheoryaboutthepathophysiologyofreactivearthritis
involvesinitialactivationbyamicrobialantigen,followedbyan
autoimmunereactionthatinvolvestheskin,eyes,andjoints.
Twoformsarerecognized:asexuallytransmittedformanda
dysentericform.GastrointestinalinfectionswithShigella,
Salmonella,andCampylobacter
speciesandothermicroorganisms,
andgenitourinaryinfectionsespeciallywithChlamydiatrachomatis

havebeenfoundtotriggerreactivearthritis.
Youngchildrentendtohavethepostdysentericform,whereas
adolescentsandyoungmenaremostlikelytoacquirereactive
arthritisaftertheyhaveurethritis.
It'snotclearwhichorganismismostcommonlyassociatedwith
reactivearthritis.BothShigellaandChlamydiaappeartohemost
common.Aftergoingthroughmanyarticlesfromjournalsonthenet,
Chlamydiaappearstohethemostcommon.Wewouldprefertogo
withChlamydia.(Howeverifanyonefindsareliablereference
documentingthemostcommonorganism,pleasemailusatour
emailid.
ThearticleonReactiveArthritisinthejournal"BestPractice&
ResearchClinicalRheumatology"Vol.20,No.6,pp.1119e1137,
2006writes-"Theprevalenceisestimatedtobe30-40casesper
100,000adults;theannualincidenceisestimatedtobe4.6/100,000

.forChlamydia-inducedarthritisand5/100,000.forenterobacteria-
inducedreactivearthritis.4,5However,realnumbersmaybe
significantlyhigher?
Thefollowingarticle"Frequencyoftriggeringbacteriainpatients
withreactivearthritisandundifferentiatedoligoarthritisandthe
relativeimportanceofthetestsusedfordiagnosis"inAnnRheum
Dis.2001April;60(4):337343atthefollowingwebsite-
http://www.pubmedcentralmih.gov/articlerenderfcgi?artid=1753604
writesitsconclusionas:
"CONCLUSIONS--Chlamydiatrachomatis,yersinia,andsalmonella
canbeidentifiedasthecausativepathogeninabout50%ofpatients
withprobableorpossibleReAiftheappropriatetestsareused."


1182.Inlongstandingrheumatoidarthritis
whichwillbeseen-
a)Milkalkalisyndrome
b)Nephrolithiasis
c)Paradoxicalaciduria
d)Secondaryamylodosis
CorrectAnswer-D
Answer-D.Secondaryamylodosis
ReactiveamyloidA(AA)amyloidosis,oneofthemostsevere
complicationsofRA,isserious,potentiallylifethreateningdisorder
causedbydepositionofAAamyloidfibrilsinmultipleorgans

1183.Whichisthemostcommontumor
leadingtodeathinadults?
a)Lungcancer
b)Prostatecancer
c)Colorectalcancer
d)Leukemia
CorrectAnswer-A
Answer-A.Lungcancer
Lungcancerconstitutesupto29%ofallcancerrelateddeathsin
malesand26%ofallcancerrelateddeathinwoman.

1184.A23-yearoldwomanhasexperienced
episodesofmyalgias,pleuraleffusion,
pericarditisandarthralgiaswithoutjoint
deformityovercourseofseveralyears.
Thebestlaboratoryscreeningtestto
diagnoseherdiseasewouldbe-

a)CD,lymphocytecount
b)Erythrocytesedimentationrate
c)Antinuclearantibody
d)Assayforthyroidhormones
CorrectAnswer-C
Answer-C.Antinuclearantibody
SensitivetestforSLEAntinuclearantibodies(ANA)
SpecifictestforSLEAntidsDNA,AntiSm

1185.Whichofthefollowingantibodiesis
highlyspecificforsystemiclupus
erythematosus-

a)Anti-Sm
b)Anti-RO-1
c)Anti-UIRNP
d)Anti-Centromere
CorrectAnswer-A
Answer-A.Anti-Sm
SensitivetestforSLEAntinuclearantibodies(ANA)
SpecifictestforSLEAntidsDNA,AntiSm

1186.Gaisbocksyndromeisknownas
a)PrimaryFamilialPolycythemia
b)HighAltitudeErythrocytosis
c)SpuriousPolycythemia
d)PolycythemiaVera
CorrectAnswer-C
AnswerisC(SpuriousPolycythemia)
GaisbocksyndromereferstoSpuriousPolycythemiaorRelative
Erythrocytosisduetodecreasedplasmavolume.


1187.AlzehiemertypeIIastrocyteareseenin
-
a)Hepaticencephalopathy
b)Alzehiemer's
c)Parkinsonism
d)Biswangerdisease
CorrectAnswer-A
Answer-A.Hepaticencephalopathy
SwollenastrocytesinhepaticencephalopathyarecalledAlzheimer
typeIIastrocytes.TheirnucleiarelargeandappearclearinH&E
stains.TheyarealsoseeninWilsondisease.

1188.PolyarticularonsetJRAinvolvesmore
thanhowmanyjoints-
a)3
b)4
c)5
d)6
CorrectAnswer-C
Answer-C.5
PolyarticularJRA
Itischaracterizedbyinvolvementof5ormorejoints.
Therearetwosubtypes:-
i)PolyarticularRApositive
Itischaracterizedbysymmetricaljointinvolvementalongwith
Uveitis,andrheumatoidnodules.
RAfactorandANAarepositive.
ii)PolyarticularRAnegative
RAfactorandrheumatoidnodulesarenotseen.

1189.Streptococcusbovisinfectionis
associatedwith-
a)CLL
b)Hairycellleukemia
c)Colorectalcancer
d)Multiplemyeloma
CorrectAnswer-C
Answer-C.Colorectalcancer
ColonandrectaltumorsStreptococcusbovis(bacteremia)

1190.Whichofthefollowingisgivento
decreaseSerumTriglycerides?
a)Fibrates
b)Statine
c)Ezetimibe
d)Niacin
CorrectAnswer-A
Answer-A.Fibrates
Fibratesaredrugsofchoiceforhypertriglyceridemia(typeIV)and
chylomicronemia(typeI).

1191.Chvosteksignisseenin:
March2013
a)Hypercalcemia
b)Hypoparathyroidism
c)Insulinoma
d)Diabetesmellitus
CorrectAnswer-B
Ans.Bi.e.Hypoparathyroidism
Hypocalcemia
Idiopathichypoparathyroidismisassociatedwith:
Geneticsyndromeofhypoparathyroidism,
-Addison'sdiseaseand
?Mucocutaneouscandidiasis
Pseudohypoparathyroidism:Nodeficiencyofparathormone,but
targetorgan(boneandkidney)areunresponsivetoitsaction
Chvostek'ssign:Contractionoffacialmuscles,elicitedbylight
tappingofthefacialnerve
ECGchanges:ProlongationofQTinterval

1192.Bloodtransfusionassociatedacute
lunginjuryoccursdueto-
a)Nosocomialinfections
b)HLAmediated
c)Auto-immunedisorder
d)Geneticsusceptibility
CorrectAnswer-B
Answer-B.HLAmediated
TRALIusuallyresultsfromthetransfusionofdonorplasmathat
containshightitreantiHLAclassIIantibodiesthatbindrecipient
leucocytes.
Theleucocytesaggregateinthepulmonaryvasculatureandrelease
mediatorsthatincreasecapillarypermeability.
Testingthedonor'splasmaforAntiHLAantibodiescansupportthis
diagnosis.

1193.I.R.I.S.is-
a)Immunereconstitutionidiopathicsyndrome
b)Immunereconstitutionimmunologicalsyndrome
c)Immunereconstitutioninflammatorysyndroma
d)Inflammatoryreconstitutionimmunesyndrome
CorrectAnswer-C
Answer-C.Immunereconstitutioninflammatorysyndroma
Theterm"immunereconstitutioninflammatorysyndrome"(IRIS)
describesacollectionofinflammatorydisordersassociatedwith
paradoxicalworseningofpreexistinginfectiousprocessesfollowing
theinitiationofhighlyactiveantiretroviraltherapy.

1194.Dietarydeficiencyofwhichvitamin
usuallydoesnotexist-
a)Vitamin-B6
b)Thiamine
c)Vitamin-E
d)Vitamin-D
CorrectAnswer-C
Answer-C.Vitamin-E
DietarydeficiencyofvitaminEdoesnotexist.

1195.ManifestationsofvitaminE.deficiency
areallexcept-
a)Hemolyticanemia
b)Posteriorcolumnabnormalities
c)Cerebellarataxia
d)Autonomicdysfunction
CorrectAnswer-D
Answer-D.Autonomicdysfunction
Theclinicalmanifestationsareedema,hemolyticanemia(dueto
fragileredcell'smembraneasaresultoflipidperoxidation)and
thrombocytosis.Nerveandmusclemembranedamagemayoccur.
VitaminEdeficiencycausesaxonaldegenerationofthelarge
myelinatedaxonsandresultsinposteriorcolumnand
spinocerebellarsymptoms.

1196.Martelsignisseenin-
a)Gout
b)Ankylosingspondylitis
c)Osteoarthritis
d)Rheumatoidarthritis
CorrectAnswer-A
Answer-A.Gout
Martel'ssignisnotpresentinallcasesofgoutyarthritis.
Martel'ssign,whichisaradiologicalsign(straightarrow)todescribe
theoverhangingmarginofthenewbonealonetheedgeoferosion.

1197.HIVRNAbyPCRcandetectaslowas
a)30copiesviralRNA/mlofblood
b)40copiesviralRNA/mlofblood
c)50copiesofviralRNA/mlofblood
d)60copiesofviralRNA/mlofblood
CorrectAnswer-B
Ans.is'b'i.e.,40copiesviralRNA/mlofblood
ThisassaygeneratesdataintheformofnumberofcopiesofHIV
RNApermillilitreofserumorplasmaandcanreliablydetectasfew
as40copiesofHIVRNApermililitreofplasma.
Researchbasedassaycandetectdowntoonecopy/ml.

1198.Allareseenincarney'striadexcept-
a)Atrialmyxoma
b)GIST
c)Chondroma
d)Paraganglioma
CorrectAnswer-A
Answer-A.Atrialmyxoma
Extra-adrenalparaganglioma(e.g.extraadrenalphaeo-
chromocytoma)
Gastrontestinalstromaltumorspreviouslyknownasgastric
epithelioidleiomyosarcoma
Pulmonarychondromahamartomaonly2ofthe3tumorsare
presentatthetimeofdiagnosistypicallyaffectsyoungpeople.

1199.Allaretrueaboutcross-matchingof
bloodexcept-
a)Mandatoryinallcasesexceptemergency
b)Recipientserumistestedagainstdonorpackedcells
c)Donorserumistestedagainstrecipientpackedcells
d)Involvesvisibleagglutination
CorrectAnswer-C
Answer-C.Donorserumistestedagainstrecipientpacked
cells
Crossmatchinginvolvestestingthepatientsserumwithdonorcells
todeterminewhetherthepatienthasanantibodywhichmaycause
ahemolytictransfusionreaction

1200. Smokingmaybeassociatedwithallof
thefollowingcancers,except:
a)CaLarynx
b)CaNasopharynx
c)CaBladder
d)None
CorrectAnswer-D
AnsisNone>CaNasopharynx
AllthegivenoptionsarementionedbyHarrison:
Harrison17/ep2737writes-"Tobaccosmokingcausescancerofthe
lung,oralcavity,naso-,oro-,andhypopharynx,nasalcavityand
paranasalsinuses,larynx,esophagus,stomach,pancreas,liver,
kidney(bodyandpelvis),ureter,urinarybladder,anduterinecervix
andalsocausesmyeloidleukemia.Thereisevidencesuggesting
thatcigarettesmokingmayplayaroleinincreasingtheriskof
colorectalandpossiblypremenopausalbreastcancer,butthereis
noassociationwithpostmenopausalbreastcancer.Theredoesnot
appeartobeacausallinkbetweencigarettesmokingandcancerof
theendometrium,andthereisalowerriskofuterinecanceramong
postmenopausalwomenwhosmoke."
Thustheanswershouldbenone,howeverNasopharyngeal
carcinomaisfoundtobeleastassociatedwithsmoking.Head&
NeckCancersbyEnslowJacobs2003ep492writes-"Association
betweensmokingandnasopharyngealcarcinomahasbeenshown
onlybyafewstudies.IftheassociationoftobaccoandNPCisreal,
thepossiblemechanismoftumorinductionwouldbethe
nitrosaminesanditsprecursorsintobacco."


1201.Inwhichofcausesoforalulcer,Auto-
antibodiesarenotseen?
a)Behcetdisease
b)SLE
c)Pemphigus
d)Celiacdisease
CorrectAnswer-A
Answer-A.Behcetdisease
Behcet'sdiseaseisclassifiedamongthevasculitideslaboratory
diagnosticdoesnotincluderegularlyautoantibodiesassociatedwith
vascularmanifestationsofsystemicautoimmunedisease.

1202.IncorrectaboutLAMBsyndrome-
a)Lentigines
b)AtrialMyxoma
c)Myaesthenicsyndrome
d)BlueNevi
CorrectAnswer-C
Answer-C.Myaesthenicsyndrome
LAMBsyndromeischaracterizedbypresenceof:-
Lentigines
AtrialMyxoma
Bluenevi.

1203.Bestmanagementafterhumanbite-
a)Ampicillinplussulbactam
b)ClindamycinplusTMP-SMX
c)Fibroquinolone
d)Doxycycline
CorrectAnswer-A
Answer-A.Ampicillinplussulbactam
1. Ampicillin/sulbactam
2. Imipenem
3. Cefoxitininpenicillinallergics

1204.TheVitaminwhichhasinhibitoryeffect
onwoundhealingis-
a)Vitamin-A
b)Vitamin-E
c)Vitamin-C
d)VitaminB-complex
CorrectAnswer-B
Ans.is'b'i.e.,Vitamin-E.
SystemicvitaminEandglucocorticoidsinhibittheinflammatory
responseandcollagensynthesis,therebypossiblyimpedingthe
healingprocess.


1205.Numberofbarrbodiesinklinfellter's
syndromeis-
a)0
b)1
c)2
d)3
CorrectAnswer-B
Answer-B.1
FoundinfemaleBut-
KleinefeltersyndromeismalewithoneBarrbody.
TurnersyndromeisfemalewithoutBarrbody.

1206.

Whichoftheseisnotapartofextracellular
matrix:

a)Laminin
b)Fibronectin
c)Integrins
d)Collagen
CorrectAnswer-C
Ci.e.Integrins
-Extracellularmatrixproteinsarecollagen,elastin,fibrillin,
fibronectin,lamininandproteoglycans(GAGs)Mn-"CallElaFor
LastProse"Whereascelladhesionmoleculespresentincytoplasm
orcellmembraneincludeintegrins,selectins,cadherinsand
immunoglobulinfamilyCAMsQ.
-Integrinsarecellsurfaceadhesionproteinthatprovidelinkage
betweencelloutside&inside,betweencell-cellandbetweencell&
extracellularmatrix.

1207.Whichofthefollowingsubstancesis
primarilyfoundintendons?
a)Collagen
b)Fibrin
c)Fibrillin
d)Protedglycans
CorrectAnswer-A
Answer-A.Collagen
Tendonisprimarilymadeupofcollagen.

1208.Whichofthefollowingconditiondoes
notcausemultiplepainfululcerson
tongue?

a)TB
b)Sacroidosis
c)Herpes
d)Behcetdisease
CorrectAnswer-B
Answer-B.Sacroidosis
Painfululcersinmouth
Apthousulcers
Behcetdisease
Denturestomatitis
Thermalburns
Tuberculosis
Herpes
Carcinomatongue
Arsenicpoisoning

1209.DOCforacuteattackofHereditary
angioneuroticedema
a)Danazol
b)Cl,inhibitorconcentrate
c)Icatibant
d)Methylprednisolone
CorrectAnswer-B
Ans.is'b'i.e.,Clinhibitorconcentrate
Medication
C1inhibitorconcentrate(Plasma-derived)(Berinert,BerinertP,
Cinryze.
RecombinantC1inhibitorConestatalfa(Ruconest,Rhucin).
BradykininBzreceptorantagonistIcatibant(Firazyr).
KallikreininhibitorEcallantide(Kalbitor)
Plasma


1210.Inapatientwhowasbroughtto
casualtyafterRTAwithpulserate108,
SBP80.Whichfluidistobegiven
ideally?

a)Plasma
b)NormalSaline
c)Blood
d)5%dextrose
CorrectAnswer-B
Answer-B.NormalSaline
Initialresuscitationrequiresrapidreexpansionofthecirculating
intravascularbloodvolumealongwithinterventionstocontrol
ongoinglosses.
Volumeresuscitationisinitiatedwiththerapidinfusionofeither
isotonicsalineorabalancedsaltsolutionsuchasRinger'slactate

1211.Whichcanbegiveinhemorrhagic
stroke?
a)Normalsaline
b)Colloids
c)Bloodtransfusion
d)Hypertonicfluids
CorrectAnswer-A
Answer-A.Normalsaline
Normalsalineinitiallyshouldbeusedformaintenanceand
replacementfluids:
Hypotonicfluidsarecontraindicatedastheymayexacerbate
cerebraledemaandintracranialpressure.
Hypervolemiashouldbeavoidedasitmayworsencerebraledema.

1212.Flushingwithniacininreducedby-
a)Laropiprant
b)Premedicationwithaspirin
c)Tachyphylaxis
d)Alloftheabove
CorrectAnswer-D
Answer-D.Alloftheabove
Laropiprant(selectiveprostaglandinD2receptorantagonid)
Premedicationwithaspirin
Flushingassubjecttotachyphylaxisandoftenimproveswithtime.

1213.TrueabourdruginducedSLEisexcept
?
a)Female:Maleratio=9:1
b)Anti-histoneAntibodies
c)CNSinvolvementnotcommon
d)Renalinvolvementnotcommon
CorrectAnswer-A
Answer-A.Female:Maleratio=9:1
Itispredominantincaucasians
IthaslessfemalepreddictionthanSLE
Itrarelyinvolveskidneysorbrain
ItisrarelyassociatedwithantiDsDNA
Itiscommonlyassociatedwithantibodiestohistones
Itusuallyresolvesoverseveralweeksafterdiscontinuationofthe
offendingmedication.

1214.Obesityisseeninallexcept
a)Cushingsyndrome
b)Pickwiniansyndrome
c)Praderwillisyndrome
d)Sipplesyndrome
CorrectAnswer-D
Ans.is`d'i.e.,Sipplesyndrome
Importantsyndromesassociatedwithobesity
Albrighthereditaryosteodystrophy(pseudohypoparathyroidismtype
la)
Alstromsyndrome
Bardet-Biedlsyndrome
Beckwith-Wiedemannsyndrome
Carpentersyndrome
Cohensyndrome?Prader-willisyndrome

1215.Manworkinginhotenvironment&
drinkinglotsofwaterwithoutintakeof
saltsisliabletodevelop-

a)Heathyperpyrexia
b)Heatcramps
c)Heatstroke
d)Heatencephalopathy
CorrectAnswer-B
Answer-B.Heatcramps
Heatcramps/Miner'scramps/Stoker'scramp/Firemen'scramp
Thesearepainfulspasmofvoluntarymuscleswhichfollowsternous
workinahotatmosphere.
Thesearecausedbylossofwaterandsaltinprofuseprespiration
(sweating).

1216.AfterRoadtrafficaccidentapatient
presentedtocasualtywithvitals
showingBPof90/60mmHgwithheart
of56bpm.Whichkindofshockoccurs
?

a)Cardiogenic
b)Neurogenic
c)Distributive
d)Hypovolemiashock
CorrectAnswer-B
Answer-B.Neurogenic
Neurogenicshockisadistributivetypeofshockresultinginlow
bloodpressure,occasionallywithaslowedheartrate,thatis
attributedtothedisruptionoftheautonomicpathwayswithinthe
spinalcord.
Itcanoccurafterdamagetothecentralnervoussystemsuchas
spinalcordinjury.

1217.Intracranialpressuremaybeincreased
byallofthefollowingdrugsexcept-
a)HypervitaminosisA
b)Corticosteroids
c)Quinolones
d)Aminoglycosides
CorrectAnswer-D
Ans.is'd'i.e.,Aminoglycosides

1218.Drainageofcervicalabscessisan
exampleof-
a)Cleancontaminatedwound
b)Cleanuncontaminatedwound
c)Uncleanuncontaminatedwound
d)Dirtyinfectedwound
CorrectAnswer-D
Answer-D.Dirtyinfectedwound
ClassIV:Thisclassofwoundisconsidereddirty-contaminated.
Theseincludewoundsthathavebeenexposedtofecalmaterial.
CervicaldrainageabscessisanexampleofClassIVwound.

1219.Mostcommonlocationofectopic
salivaryglandis-
a)Cervicallymphnodes
b)Anteriormediastinum
c)Posteriortriangle
d)Parathyroidgland
CorrectAnswer-A
Answer-A.Cervicallymphnodes
Salivaryglandheterotopiaiswheresalivaryglandacinicellsare
presentinanabnormallocationwithoutanyductsystem.
Themostcommonlocationisthecervicallymphnodes.
Otherreportedsitesofheterotopicsalivaryglandtissuearethe
middleear,parathyroidglands,thyroidgland,pituitarygland,
cerebellarpontineangle,softtissuemedialtosternocleidomastoid,
stomach,rectumandvulva

1220.ClassictriadinRenalcellcarcinoma
includesallofthefollowing,Except:
a)Hematuria
b)Hypertension
c)Flankmass
d)AbdominalPain
CorrectAnswer-B
AnswerisB(Hypertension):
HypertensionmaybeseeninpatientswithRenalcellcarcinomabut
itdoesnotfirmpartoftheclassicallydescribedtriad.
ClassicTriadofRenalcellcarcinoma(seenin10-20%of
patients)
Hematuria(Gross)
Pain(Abdominal/Flank)
Mass(Abdominal/Flank)


1221.Allofthefollowingareparaneoplastic
syndromesforrenalcellcarcinoma
except-

a)Fever
b)Anaemia
c)Amyloidosis
d)AcanthosisNigricans
CorrectAnswer-D
Answer-D.AcanthosisNigricans
Hypertension
Abnormalliverfunction(Staufferssyndromeienonmetastatic
hepaticdysfunction)
Hypercalcemia
Neuromyopathy
Amyloidosis
IncreasedESR(MCparaneoplasticsyndrome)
Dysfibrogenemia
Galactorrhoea
Feminizationandmasculanization

1222.ESWLiscontraindicatedinwhichofthe
followingstones-
a)Cysteinestones
b)OxalateStones
c)Uratestones
d)Phosphatestones
CorrectAnswer-A
Answer-A.Cysteinestones
ContraindicationsforESWL-

1. UncorrectedBleedingdisorder
2. Cysteinestones
3. Weightgreaterthan300pounds
4. Pregnancy
5. CardiacPacemaker
6. SevereOrthpaedicdeformity
7. UTI
8. SevererenalFailure

1223.Surgeryforundescendedtestisisrecommendedatwhatage?
a)6months
b)12months
c)24months
d)36months
CorrectAnswer-A
Optimumageforsurgeryincaseofundescendedtestisis6months.
Thesurgicalproceduredoneiscalledorchidopexy.
Earlierintervention(Sixmonthsofage)shouldbeconsideredinordertotheoretically
preventthecomplicationsofcrytorchidismthatmaybemanifestedbefore1yearofage.
Ref:NursingcareofthepediatricsurgicalpatientByNancyTkaczBrowne,Page147-8;
CPDT,20thEdition,Page974;Campbell'sUrology,9thEdition,Page3775

1224.Pseudoclaudicationiscausedby
a)FemoralArterystenosis
b)PoplitealArterystenosis
c)Lumbarcanalstenosis
d)RadialArterystenosis
CorrectAnswer-C
AnsisCi.e.LumbarCanalStenosis
Pseudoclaudicationisthetermusedforneurogenicclaudication
causedduetolumbarcanalstenosis.Itssymptomsmimic
intermittentclaudicationofPVD.
Neurogenicclaudication,consistsofbackandbuttockorlegpain
inducedbywalkingorstandingandrelievedbysittingorchanging
posture.Symptomsinthelegsareusuallybilateral.Unlikevascular
claudication,symptomsareoftenprovokedbystandingwithout
walking.
Spinalstenosiscanbeacquired,congenital,orduetoacombination
ofthesefactors.Congenitalcausesincludeachondroplasia,
idiopathic.Acquiredcausesaredegenerativediseases(spondylosis,
spondylolisthesis,scoliosis),trauma,spinesurgery,metabolicor
endocrinedisorders(epidurallipomatosis,osteoporosis,
acromegaly,renalosteodystrophy,hypoparathyroidism),andPaget's
disease.
DifferencebetweenVascularandNeurogenicclaudication
Patientswithvascularclaudicationalwayshavepainwhentheywalk
arelativelyconstantdistanceonlevelground;theydonothave
variabledayswhentheycanwalkforconsiderablygreaterdistances
withoutpain.Patientsofvascularclaudication,oftenknowexactly
howfarorforhowlongtheycanwalkbeforethesymptomswill

occur.Thisisincontrasttopatientswithneurogenicclaudication
wherethesymptomsoccasionallyoccuratrestoratwithhighly
variablewalkingdistances.Paininneurogenicclaudicationcaneven
occuronstanding.
Painofvascularclaudicationgetsrelievedonstoppingwithinfew
minutes,andthisresolutionofpainoccursevenifthepatientsimply
stopsandstandsinplace.Patientswithneurogenicclaudication
usuallyhavetositdowntorelievetheirpain.Neurogenic
claudicationpaingetsrelievedbychangingpostureandleaning
forwards.
Patientswithneurogenicclaudicationoftencanwalkfurtheronan
incline,whereasvascularclaudicationismarkedworsenedifthe
patientisonanincline.

1225.

Whichofthefollowingisthemost
conservativeneckdissection-

a)Supraomohyoidneckdissection
b)Radicalneckdissection
c)Modifiedradicalneckdissection
d)Allareconservative
CorrectAnswer-A
Answer-A.Supraomohyoidneckdissection
Selectiveneckdissection-PreservationofanyofthelevelsI
throughVduringneckdissectionisknownasSelectiveneck
dissection-Theprinciplebehindpreservationofcertainnodal
groupsisthatspecificprimarysitespreferentiallydraintheir
lymphaticsinapredictablepattern.TypesofSNDincludethe
supraomohyoidneckdissection,thelateralneckdissection,andthe
posterolateralneckdissection.

1226.

Supraomohyoiddissectionisatypeof-
a)Selectiveneckdissection
b)Modifiedradicalneckdissection
c)Radicalneckdissection
d)Posterolateraldissection
CorrectAnswer-A
Answer-A.Selectiveneckdissection
Selectiveneckdissection-PreservationofanyofthelevelsI
throughVduringneckdissectionisknownasSelectiveneck
dissection-Theprinciplebehindpreservationofcertainnodal
groupsisthatspecificprimarysitespreferentiallydraintheir
lymphaticsinapredictablepattern.TypesofSNDincludethe
supraomohyoidneckdissection,thelateralneckdissection,andthe
posterolateralneckdissection.

1227.Alagillesyndrome-allofthefollowing
aretrueexcept-
a)MutationinJAG1AndNotch2geneareseen
b)CanCauseAutoimmunehepatitis
c)AutosomalRecessiveDisease
d)Valvularanomalisofheartseen
CorrectAnswer-C
AnswerC.AutosomalRecessiveDisease
Autosomaldominantdisease
JAGIandNOTCH2mutation
AffectstheliverandHeart
Heart
Pulmonarystenosis
TetralogyofFallot
VSD
Overridingaorta

1228.AbsenceofUltrahepaticbileductleads
towhichsyndrome?
a)VonMeyenburgComplexes
b)PolycysticLiverDisease
c)CaroliDisease
d)AlagilleSyndrome
CorrectAnswer-D
Answer-D.AlagilleSyndrome
Itisanautosomalrecessiveconditioncharacterizedbyabsenceof
bileductsinportaltractAlsocalledarteriohepaticdysplasia
Characteristicfeatures:
Geneticdisorderwithvascular,biliaryandotheranomalies.
Absenceofintrahepaticbileductswithclinicalseverityrangingfrom
severeneonatalcholestasismimickingbiliaryatresiatochildhood
intermittentjaundice.
Progressiontocirrhosisisrare.

1229.Allofthefollowingarepremalignant
conditionsexcept-
a)Bowen'sDisease
b)SenileKeratosis
c)XerodermaPigmentosum
d)PyodermaGangrenosum
CorrectAnswer-D
Answer-D.PyodermaGangrenosum
Bowen'sDisease
Lupusvulgaris
Longstandingchroniculcer(e.g.marjolin's
ActinicKeratosis
ulcers)
Xeroderma
Followingburn,venousulcer,oldscaretc.
pigmentosum
SenileKeratosis

1230.Adson'stestispositivein-
a)Cervicalrib
b)Cervicalspondylosis
c)Cervicalfracture
d)Cervicaldislocation
CorrectAnswer-A
Ans.is'a'i.e.,Cervicalrib
Adson'stestisaprovocativetestforThoracicOutletSyndrome
accompaniedbycompressionoftheSubclavianarterybyacervical
ribortightenedanteriorandmiddlescalenemuscles.

1231.Chimericchemotherapyisbeing
investigatedforthetreatmentofwhich
malignancy?

a)Leukemia
b)RenalCellCarcinoma
c)CAPancreas
d)GlioblastomaMultiforme
CorrectAnswer-A
Answer-A.Leukemia
ArtificialTcellreceptors(alsoknownaschimericTcellreceptors,
chimericimmunoreceptors,chimericantigenreceptors(CARs))are
engineeredreceptors,whichgraftanarbitraryspecificityontoan
immuneeffectorcell.Typically,thesereceptorsareusedtograftthe
specificityofamonoclonalantibodyontoaTcell.
TwoCAR-TtherapieshavebeengrantedthreetotalFDAapproval
forthetreatmentofpatientswithhematologicmalignancies.
ThemostadvanceddatatodateconcernCARTtherapiesforthe
treatmentofpatientswithmultiplemyelomaandchroniclymphocytic
leukemia.
Amongthefirststudiesinvestigatinganti-CD19CARTtherapeutic
efficacywasacasereporttreatingapatientwithCLL.

1232.AnteriorMediastinalnodesareincluded
inwhichleveloflymphnodes-
a)I
b)III
c)V
d)VII
CorrectAnswer-D
Answer-D.VII
Levelsofnecklymphnodes
LevelI,submentalandsubmandibulargroup
LevelII,upperjugulargroup
LevelIII,middlejugulargroup
LevelIV,lowerjugulargroup
LevelV,posteriortrian-glegroup
LevelVI,anteriorcompartment.
LevelVII,anteriorandsuperiormediastinal

1233.InCeliacarterycompressionsyndrome
whichstructureisthemaincauseof
compression-

a)MedianArcuateLigament
b)RectusSheath
c)DeepInferiorEpigastricartery
d)LacunarLigament
CorrectAnswer-A
Answer-A.MedianArcuateLigament
Medianarcuateligamentsyndrome(MALS,alsoknownasceliac
arterycompressionsyndrome,celiacaxissyndrome,celiactrunk
compressionsyndromeorDunbarsyndrome)isacondition
characterizedbyabdominalpainattributedtocompressionofthe
celiacarteryandpossiblytheceliacgangliabythemedianarcuate
ligament.
Theabdominalpainmayberelatedtomeals,maybeaccompanied
byweightloss,andmaybeassociatedwithanabdominalbruit
heardbyaclinician.Itisalsocalledceliacarterycompression
syndrome.

1234.WhichIsFalseaboutstressurinary
incontinence-
a)Morecommoninmen
b)Itoccursduringincreasedabdominalpressure
c)Itisduetoweakeningofpelvicfloormuscles
d)Prostatesurgerymaybeacause
CorrectAnswer-A
Answer-A.Morecommoninmen
Leakingurineuponsuddenincreaseinabdominalpressure.
Itismostcommontypeinwomen.
Causesare
Weakeningofpelvicfloormuscles/urethra/sphinctermuscles.
BPH,CAprostateorprostatesurgery.

1235.Whichofthefollowingurethralanomaly
isthemostcommon-
a)Hypospadias
b)Pinholemeatus
c)Epispadias
d)Strictureurethra
CorrectAnswer-A
Ans.is'a'i.e.,Hypospadias
Hypospadiasisaconditioninwhichtheurethralmeatusopenson
the
undersideofpenis*ortheperineum(i.e.ventralsurfaceofpenis)
proximaltothetipoftheglanspenis.
Hypospadiasisthemostcommon*congenitalmalformationofthe
urethra.(alsoknowthatthemostcommoncongenitalmalformatioof
theurinarytractisDuplicationofrenalpelvis*)
Occursin1:250malebirths.

1236.Whichofthefollowingtypesofshock
wiilusuallyhavewarmperipheral
extremities?

a)HypovolemicShock
b)NeurogenicShock
c)CardiogenicShock
d)AnaphylacticShock
CorrectAnswer-B
Answer-B.NeurogenicShock
Bradycardia
Hypothermia
Lossofsympathetictone-->arterious&venousvasodilation
Warmanddryskin(theheartrateslowsdown,aswellaswarm,dry,
andflushedskinduetovenouspoolingsecondarytolossof
vasculartone)
Othersallshockhavecoldextremities(anaphylatic,cardiogenicand
hypovolemic)

1237.Antibioticprophylaxisisbestgiven-
a)1daybeforesurgery
b)2hoursbeforesurgery
c)Beforethetimeofincision
d)Onlypostoperatively
CorrectAnswer-C
Answer-C.Beforethetimeofincision
Aboutprophylacticantibioticspointstoknoware?Bydefinition
prophylaxisislimitedtothetimepriortoandduringtheoperative
procedure.
bemosteffectivetheprophylacticantibioticagentshouldbe
administeredintravenouslybeforetheincisionismadesothatthe
tissuelevelsarepresentatthetimethewoundiscreatedand
exposedtothebacterialcontamination.

1238.Whichofthefollowingisafeatureof
crushsyndrome-
a)Hypokalemia
b)Hypercalcemia
c)Myoglobinuria
d)Hypophosphatemia
CorrectAnswer-C
Answer-C.Myoglobinuria
Featuresofcrushsyndrome:
Hyperkalemia
Hypocalcemia
Hyperphosphatemia
Hyperuricemia
Lacticacidosis
Myoglobinuria
IncreasedBUN&creatinine.

1239.CrushSyndromeisassociatedwithall
ofthefollowingfeaturesexcept-
a)Hypercalcemia
b)Hyperkalemia
c)Increasedserumcreatinine
d)Myoglobinuria
CorrectAnswer-A
Answer-A.Hypercalcemia
Featuresofcrushsyndrome:
Hyperkalemia
Hypocalcemia
Hyperphosphatemia
Hyperuricemia
Lacticacidosis
Myoglobinuria
IncreasedBUN&creatinine.

1240.Whichofthefollowingstonesare
commonwithinfection?
a)Struvite
b)XanthineStones
c)Cysteinestones
d)CalciumOxalatestones
CorrectAnswer-A
Answer-A.Struvite
Calciumphosphatecanbecombinedwithammoniumand
magnesiumtoformthetriplephosphatecalculusieCalcium
amnomiummagnesiumphosphatealsok/astruvitestones.
Smoothanddirtywhiteandsoltitary.
Seeninalkalineurineespwithproteusinfectionwhichsplitureato
ammonia.

1241.Allthefeaturesofmembranousurethral
injuryexcept
a)bloodofmeatus
b)Retentionofurine
c)Pelvicfracture
d)Perinealbutterflyhematoma
CorrectAnswer-D
Ans.is'd'
1.Urethralruptureisalsooftwotypes:
a.Bulbarurethralinjury-MC
b.Membranousurethralinjury
2.Extravasationofurine(+blood)inbulbarurethralinjury
a.Itisasuperficialextravasation.
b.IftheBuck'sfasciaremainsintact,extravasationofbloodand
urineareconfinedtothepenileshaft.HoweverdisruptionofBuck's
fasciaallowsextravasatedcontentsintoaspacelimitedbycolle's
fasciaformsascrotalandperinealbutterflyhematomawhichcan
extenduptheabdominalwall.
3.ExtravasationofurineinMembranousurethralinjury&
Extraperitonealbladderrupture.
a.Extravasationissameinboth
b.Itisadeepextravasation
c.Urineextrvasatesinthelayersofthepelvicfasciaandthe
retroperitonealtissues
d.Urinecollectsintheperivesicalspace.
e.Thetypicalfindingoncystogramisextravasationofcontrast
materialintothepelvisaroundthebaseofthebladder.

1242.HighFlyingprostateisasignof-
a)ExtraperitonealBladderrupture
b)IntraperitonealBladderRupture
c)MembranousUrethralInjury
d)BulbarUrethralInjury
CorrectAnswer-C
Answer-C.MembranousUrethralInjury
Theprostateisattachedtopubisbypuboprostaticligamentand
disruptionofthepuboprostaticligamentwithacompleteruptureof
theurethracanleadtoafloatingprostate--Vermooten'ssign
(highflyprostate).
Basedonascendingurethrogram,posteriorurethralinjuryis
classifiedas(McCallum-Colapintoclassification).
1. TypeI:Elongationofposteriorurethra,butintact
2. TypeII:Prostate"pluckedoff''membranousurethra
withextravasationofurineabovesphincteronly--Floatingprostate
--Vermooten'ssign

3. TypeIII:Totaldisruptionoftheurethrawithextravasationof
urinebothaboveandbelowthesphincter.

1243.BPHinvolves-
a)Centralzone
b)Peripheralzone
c)Transitionalzone
d)Prostatecapsule
CorrectAnswer-C
Answer-C.Transitionalzone
Carcinomaofprostatearisesmostcommonlyintheperipheralzone
Benignprostatichyperplasiaoriginatesinthetransitionzone
BPHtypicallyaffectsthesubmucousgroupofglandsinthe
transitionalzone,forminganodularenlargement.
Eventually,thisovergrowthcompressesthePZglandsintoafalse
capsuleandcausestheappearanceofthetypical'lateral'lobes.
Ref-BaileyandLoveVolume-1/Edition27th


1244.Prostatecancerthatislimitedtothe
capsuleandnottheurethrawouldbe
satgedas-

a)Tx
b)T1
c)T2
d)T3
CorrectAnswer-C
Answer-C.T2

1245.A65yearoldmalepresentswithCA
prostate.Thetumourislimitedtothe
capsuleanditispalpableonPR
examination.Thepatientisdiagnosed
asstageT1b.Thebesttreatmentwould
be-

a)Radicalprostatectomy
b)Chemotherapy
c)Palliativeradiotherapy
d)Orchidectomy
CorrectAnswer-A
Answer-A.Radicalprostatectomy
T1bT1c,T2
Radicalprostactectomyorradiotherapy
Externalbeamradiotherapy-T1orlowT2disease
Branchytherapy-lowT1disease

1246.

WhichofthefollowingVitamindeficiencyisseeninshortbowelsyndromewithilealresection?
a)VitaminK
b)VitaminB12
c)VitaminB1
d)Folicacid
CorrectAnswer-B
Answer-B(VitaminB12)
Short-bowelsyndromeisadisorderclinicallydefinedby
malabsorption,diarrhea,steatorrhea,fluidandelectrolyte
disturbances,andmalnutrition.
ChangesseeninTerminalIlealResection
MalabsorptionofbilesaltsandvitaminB12(whicharenormally
absorbedinthisregion)
VitaminB12MalabsorptionMegaloblasticanemia
BilesaltsMalabsorptionUnabsorbedbilesaltsescapeintothe
colonandstimulatefluidsecretionfromthecolonwaterydiarrhea
DecreasedbilesaltsinthebileCholesterolgallstones

1247.If90%ofjejunumandiliumisremoved
thenallofthefollowingfeatureswillbe
seenexcePt-

a)Steatorrhoea
b)Diarrhoea
c)Weightgain
d)Megaloblasticanemia
CorrectAnswer-C
Answer-C.Weightgain
ChangesseeninTerminalilealresection
Resectionofterminalileumresultsinmalabsorptionofbilesaltsand
Vit.B12whicharenormallyabsorbedinthisregion
VitBIZMalabsorption:Megaloblasticanemia
Bilesaltsmalabsorption:-Unabsorbedbilesaltsescapeintocolon
wheretheystimulatefluidsecretionfromthecolon,resultingin
waterydiarrhoea.
Reductioninbilesaltpoolleadtosteatorrheaandmalabsorptionof
fatsolublevitamins(duetomalabsorptionoffat).
Unabsorbedfattyacidsbindwithcalcium,andthusleadtoincreased
concentrationoffreeoxalates(oxalatesbindwithcalciumnormally
andthereforeescapewithoutintestinalabsorption)Freeoxalates
areabsorbed.Increasedconc.ofbodyoxalatesleadtooxalate
kidneystones.

1248.PerihepaticfibrosisoccurringinFitz
HughCurtisSyndromeisdueto-
a)PelvicInflammatoryDisease
b)BileDuctInjry
c)ChronicAlcoholism
d)ViralHepatitis
CorrectAnswer-A
Answer-A.PelvicInflammatoryDisease
Itisthoughttoresultfromdirectintraperitonealspreadofinfection
towardstheperihepaticregionfrominitialpelvic
inflammation/infection
Trichomonasvaginalis,UreaplasmaurealyticumandMycoplasma
hominismayalsocauseFHCS.

1249.WhatistheT/tofptwithcarcinoid
tumorofappendixofsizemorethan2
cm-

a)Righthemicolectomy
b)Appendicectomy
c)Appendicectomy+abdominalCTscan
d)Appendicectomy+24hrsurinaryHIAA
CorrectAnswer-A
Ansis'a'ie.Righthemicolectiomy

1250.A25yearoldmaleisreceiving
conservativemanagementforan
appendicularmasssince3daysnow
presentswitharisingpulserate,
tachycardiaandfever.Themodeof
managementmustbe-

a)Ochsnersherrenregimen
b)Continueconservativemanagement
c)Proceedtolaparotomyandappendicectomy
d)Intravenousantibiotics
CorrectAnswer-C
Answer-C.Proceedtolaparotomyandappendicectomy
Ifanappendixmassispresentandtheconditionofthepatientis
satisfactory,thestandardtreatmentistheconservativeOchsner?
Sherrenregimen.
10-20%needemergencyoperationduetospreadinginfection.

1251.Underwhatguidelinesistreatment
startedforapatientpresentingwith
appendicularmassonCTScan?

a)OchsnerSherrenRegimen
b)Conservativemanagementanddischarge
c)ImmediateLaprotomy
d)Kocher'sRegimen
CorrectAnswer-A
Answer-A.OchsnerSherrenRegimen
OchsnerSherrenregimenistheexpectantmanagementgivingtoa
patientwithanappendixmass.Itisexpectantbecauseitisexpected
thatthesymptomsandsignsthepatientpresentedwithwillimprove
duringthecourseofthemanagementandthepatientmaylaterbe
scheduledforelective/intervalappendicectomy.

1252.Treatmentofchoiceofmucinous
carcinomaofGallBladderconfinedto
thelaminapropriais-

a)Simplecholecystectomy
b)Extendedcholecystectomy
c)Cholecystectomywithwedgeresectionofliver
d)Chemotherapyonly
CorrectAnswer-A
Answer-A.Simplecholecystectomy
Treatment-
StageIandII-simplecholecystectomy
StageIII?cholecystectomy+adjacenthepaticresection(atleast
2cmdepth)+regionallymphadenectomy
Poorprognosis

1253.NigroRegimenisusedfor-
a)AnalCarcinoma
b)RectalCarcinoma
c)SigmoidColonCarcinoma
d)DuodenalCarcinoma
CorrectAnswer-A
Answer-A.AnalCarcinoma
Nigroregimenreferstocombinedchemotherapyandradiotherapy
forprimarytreatmentofmalignanttumorsoftheanalcanal.

1254.A35yearoldmalecamewithjaundice,
palpablemassintheright
hypochondriumnotassociatedwith
pain.Theprobablediagnosisis-

a)CarcinomaAmpullaofVater
b)Acutecholecystitis
c)ChronicCholecystitis
d)ChronicPancreatitis
CorrectAnswer-A
Answer-A.CarcinomaAmpullaofVater
Courvoisier'slawstatesthatinthepresenceofanenlargedgall
bladderwhichisnontenderandaccompaniedwithjaundicethe
causeisunlikelytobegallstones.Thisisbecausegallstonesare
formedoveralongerperiodoftime,andthisresultinashrunken,
fibroticgallbladderwhichdoesnotdistendeasily.

1255.A60yearmalepresentedwithjaundice,
palestools,darkurineandmassinthe
epigastricregion.Whichofthe
followingdiagnosisinunlikely?

a)Pancreaticcancer
b)BiliaryCancer
c)PeriampullaryCancer
d)ChronicCholecystitis
CorrectAnswer-D
Answer-D.ChronicCholecystitis
Chroniccholecystitisischaracterizedbyrepeatedattacksofpain
(biliarycolic)thatoccurwhengallstonesperiodicallyblockthecystic
duct.

1256.Allofthefollowingmayleadtoagall
bladdercarcinomaexcept-
a)GallBladderPolyps
b)Typhoidcarriers
c)Exposuretocarcinogenslikenitrosamine
d)EchinococcusGranulosusInfection
CorrectAnswer-D
Answer-D.EchinococcusGranulosusInfection
Riskfactorsforgallbladder

1. Gallstones
2. Adenomatousgallbladderpolyps(particularlypolypslargerthan10
mm)
3. Calcified(porcelain)gallbladder
4. Choledochalcyst
5. Estrogens
6. Anomalouspancreaticobiliaryductjunction
7. Exposuretocarcinogens(azotoulene,nitrosamine)
8. Typhoidcarriers
9. Sclerosingcholangitis
10. Cholecystoentericfistula

1257.AllofthefollowingarefalseforGall
Bladdercarcinomaexcept:
March2005

a)Carriesagoodprognosis
b)Gallstonesmaybeapredisposingfactor
c)Commonlysquamouscellcarcinoma
d)Jaundiceisrare
CorrectAnswer-B
Ans.B:Gallstonesmaybeapredisposingfactor
Primarycarcinomaofthegallbladderisanuncommon,aggressive
malignancythataffectswomenmorefrequentlythanmen.
Olderagegroupsaremostoftenaffected,andcoexistinggallstones
arepresentinthevastmajorityofcases.
Thesymptomsatpresentationarevagueandaremostoftenrelated
toadjacentorganinvasion.Imagingstudiesmayrevealamass
replacingthenormalgallbladder,diffuseorfocalthickeningofthe
gallbladderwall,orapolypoidmasswithinthegallbladderlumen.
Adjacentorganinvasion,mostcommonlyinvolvingtheliver,is
typicallypresentatdiagnosis,asisbiliaryobstruction.Periportaland
peripancreaticlymphadenopathy,hematogenousmetastases,and
peritonealmetastasesmayalsobeseen.Thevastmajorityof
gallbladdercarcinomasareadenocarcinomas.
Becausemostpatientspresentwithadvanceddisease,the
prognosisispoor.
Theradiologicdifferentialdiagnosisincludesthemorefrequently
encounteredinflammatoryconditionsofthegallbladder,
xanthogranulomatouscholecystitis,adenomyomatosis,other
hepatobiliarymalignancies,andmetastaticdisease.

hepatobiliarymalignancies,andmetastaticdisease.
Theprevalenceoflymphaticspreadishighingallbladdercarcinoma.
Lymphaticmetastasesprogressfromthegallbladderfossathrough
thehepatoduodenalligamenttonodalstationsneartheheadofthe
pancreas.
Threepathwaysoflymphaticdrainagehavebeensuggested:the
cholecystoretropancreaticpathway,thecholecystoceliacpathway,
andthecholecystomesentericpathway.
Thecysticandpericholedochallymphnodesarethemostcommonly
involvedatsurgeryandareacriticalpathwaytoinvolvementofthe
celiac,superiormesenteric,andpara-aorticlymphnodes.

1258.Whichofthefollowingstomaisformed
inHartman'sprocedure?
a)EndColostomy
b)EndIliostomy
c)LoopIliostomy
d)Caecostomy
CorrectAnswer-A
Answer-A.EndColostomy
Hartman'sprocedure
Thisisacompromisetypeofresectioninwhichtherectumand/or
sigmoidcolonareresectedandanendcolostomydone.
Anastomosisandperinealdissectionofthedistalrectumand
perineumarenotdone.

1259.Mostcommonsiteofcolorectal
carcinomais-
a)Rectum
b)SigmoidColon
c)AscendingColon
d)DescendingColon
CorrectAnswer-A
Answer-A.Rectum
Rectosigmoidregionisthemostcommonsiteforcolorectal
carcinoma.Rectumisinvolvedin38%ofcasesandsigmoidcolonis
involvedin21%cases.

1260.Apatientundergoingaminorsurgicalprocedureisgivenlignocaine
injection.
Assertion:Localanaestheticsactsbyblockingnerveconduction.
Reason:Smallfibersandnonmyelinatedfibersareblockedmoreeasily
thanlargemyelinatedfibers.

a)BothAssertionandReasonaretrue,andReasonisthecorrect
explanationforAssertion
b)BothAssertionandReasonaretrue,andReasonisnotthe
correctexplanationforAssertion
c)Assertionistrue,butReasonisfalse
d)Assertionisfalse,butReasonistrue
CorrectAnswer-B
Localanaestheticsblocksnerveconductionbydecreasingtheentryofsodiumionsduring
upstrokeofactionpotential.
Finallyitblocksdepolarizationtoreachthresholdpotentialandconductionblockoccurs.
Smallfibersaremoresensitivetolocalanaestheticsthanlargefibersandnonmyelinated
fibersareblockedeasilythanmyelinatedfibers.
Ref:EssentialsofMedicalPharmacologybyKDTripathi,5thEdition,Pages321-3

1261.Whichofthefollowingisnottrueabout
Boerhaavesyndrome?
a)Perforationoftheesophagusduetobarotrauma
b)Mostcommonsiteisleftposteromedialaspect3-5cmsabove
thegastroesophagealjunction
c)Painistheearlymanifestation
d)Mostcasesfollowaboutofheavyeatingordrinking
CorrectAnswer-B
Answer-B.Mostcommonsiteisleftposteromedialaspect3-5
cmsabovethegastroesophagealjunction
Vomitingisthoughttobethemostcommoncause,othercauses
includeweightlifting,defecation,epilepticseizures,abdominal
trauma,compressedairinjury,andchildbirth,allofwhichcan
increasethepressureintheesophagusandcauseabarogenic
esophagealrupture.
Twocommonriskfactorsincludealcoholismandexcessive
indulgenceinfood.
Ruptureoccursmostcommonlyintheleftposterolateralwallofthe
distalthirdoftheesophaguswithextensionintotheleftpleural
cavity.
Symptomsconsistofvomiting,lowerthoracicpain,and
subcutaneousemphysema.
Typically,thepatientwillpresentwithpainatthesiteofperforation,
usuallyintheneck,chest,epigastricregion,orupperabdomen.

1262.Whichofthefollowingistrueabout
MalloryWeisstear-
a)Itisamucosaltearnotextendingthroughthemusclelayer
b)Itismorecommoninwomenthanmen
c)Itiscommoninyoungindividuals
d)Itisassociatedwithachalasiacardia
CorrectAnswer-A
Answer-A.Itisamucosaltearnotextendingthroughthe
musclelayer
AMallory-Weisstear(MWT)isforcefulorretchingvomitingmay
producealongitudinalmucosaltearimmediatelybelowsquamo
columnarjunctionatthecardiaorgastroesophagealjunctionmainly
andalsoinoesophagus.
Seeninmales,alcholicsandpregnantfemales.
Themeanageismorethan60and80%aremen.
Hyperemesisgravidarum,whichisseveremorningsickness
associatedwithvomitingandretchinginpregnancy,isalsoaknown
causeofMallory-Weisstear.

1263.Gastriccarcinomaisassociatedwithall
EXCEPT?
a)Inactivationofp53
b)OverexpressionofC-erb
c)OverexpressionofC-met
d)ActivationofRAS
CorrectAnswer-D
Ans.is'd'ActivationofRAS
Inthecourseofmulti-stepstomachcarcinogenesis,variousgenetic
andepigeneticalterationsofoncogenes,tumor-suppressorgenes,
DNArepairgenes,cellcycleregulatorsandcelladhesionmolecules
areinvolved.Geneticalterationingastriccancerinclude:
Intestinaltypegastriccancer:K-rasmutation,APCmutation,pS2
methylation,HMLH1methylation,pIemamethylation,p73deletion
andC-erbB-2amplification.
1. Diffusetypegastriccaner:CDHIgene(E-Cadherin)mutation,K-
samamplification.
2. Forbothtype:Telomerasereduction(telomeraseshortening),
hTERTexpression,geneticinstability,overexpressionofthecyclinE
&CDC25B&E2FIgenes,p53mutations,reducedexpression,
CD44aberrabonttranscripts,andamplificationoftheC-metCyclinE
genes.
Comingtoquestion:
Allthegivenfourgeneticalterationsmaybeassociatedwith
stomachcancer.
HoweveramongthegivenoptionsK-rasisbestanswerasitis
associatedwithgastriccancerinminimumpercentage(amongst
givenoptions):
Source:Textbookofmechanismofcarcinogenesisandcancer

Source:Textbookofmechanismofcarcinogenesisandcancer
prevention
K-rasmutation
-4 <10%
p53mutation
30-60%
C-erbB-2amplifcation --> 20%
C-metamplifciation
--> 20%

1264.A55yearoldwomanpresentedwith
historyofrecurrentepisodesofright
upperabdominalpainforthelastone
year.Shepresentedtoemergencywith
historyofjaundiceandfeverfor2days.
Onexamination,thepatientappeared
toxicandhadabloodpressureof
100/60mmHg.Shewasstartedon
intravenousantibiotics.Ultrasoundof
theabdomenshowedpresenceof
stonesinthecommonbileduct.What
wouldbethebesttreatmentoptionfor
her-

a)ERCPandbileductstoneextraction
b)Laparoscopiccholecystectomy
c)Opensurgeryandbileductstoneextraction
d)Lithotripsy
CorrectAnswer-A
Answer-A.ERCPandbileductstoneextraction
Methodsofbiliarydecompression
a.ERCPwithsphincterotomyandstoneextraction
Itistheprocedureofchoice,
Earlyendoscopyallowsnotonlydiagnosisbycholangiographyand

directvisualizationoftheampullabutalsopermitsbiliary
decompressionbysphincterotomyandstoneextraction.
Ifthestonescannotberemoved,anasobiliarycatheterorstentis
insertedtodecompressthebiliarytract.
b.Percutaneoustranshepaticroute(PTC)
PTCisperformedif
Theendoscopicprocedurehasfailedornotavailable.
Iftheobstructionismoreproximalorperihilar.
Ifthereisastrictureinabiliaryentericanastomosis.
c.Surgicalbiliarydecompression

1265.Rigler'ssignissuggestiveof-
a)Pneumothorax
b)Pneumoperitoneum
c)Peritonitis
d)Hemothorax
CorrectAnswer-B
Answer-B.Pneumoperitoneum
Bothsidesofbowelbecomevisiblebecauseoffreegasonan
abdominalx-ray.ThisisknownasRigler'sSign.

1266.Whatisthemostcommonpositionofappendix?
a)Pelvic
b)Paracolic
c)Retrocaecal
d)Retroperitoneal
CorrectAnswer-C
Retrocecalisthemostcommonpositionoftheappendix.
Positionsofappendix:
Retrocecal:74%
Subcecal:1.5%
Pelvic:21%
Post-ileal:0.5%
Pre-ileal/retro-ileal:1%

1267.A70yearpldmalecomplainingofper
rectalbleedingwasdiagnosedof
havingrectal/anorectalcancer.The
distalmarginofthetumorwas5cm
fromtheanalvergethetreatmentof
choicewouldbe-

a)PalliativeRadiotherapy
b)Abdominoperinealrecection
c)Lowanteriorresection
d)LocalExcision
CorrectAnswer-C
Answer-C.Lowanteriorresection
Surgeries
Pre-operativeneo-adjuvantradiotherapyinresectablerectalcancer
reducestheincidenceoflocalrecurrence
Adjuvantchemotherapyimprovessurvivalinnode-positivecases
Anterior(low)resection:sphinctersavingprocedure,midrectum
Proximal2/3rdofrectum(lesions6cmsabovethedentateline/2or
morecmsaboveanalcanal)
Welldifferentiatedtumoursizetumour
T1/T2,NOtumours
Rectosigmoidtumorsandupperthirdrectaltumors:Highanterior
resection(rectumandmesorectumaretakentoamargin5cmdistal
tothetumourandcolorectalanastomosisisperformed)
Tumoursinthemiddleandlowerthirdsofrectum:completeremoval
ofrectumandmesorectum(TME-totalmesorectalexcision)

Abdominoperinealresection(Mike'sprocedure)-Lowerrectum
Hartmann'sprocedure?forelderlyandunstablepatientswho
cannotwithstandlongprocedureofAPR
Colonoscopyisalwaysperformedeitherbefore(forsynchronous
tumours)orwithinafewmonths(formetachronoustumours)of
surgicalresectionfortumourdetection

1268.Allofthefollowingaretrueabout
Hirschsprungdiseaseexcept-
a)AbsenceofGanglioncellsintheinvolvedsegment
b)Swenson,DuhamelandSoavearesurgicalproceduresforthis
condition
c)Mainlypresentsininfancy
d)Thenonperistalticaffectedsegmentisdialted
CorrectAnswer-D
Answer-D.Thenonperistalticaffectedsegmentisdialted
InHirchprung'sdiseasetheabsenceofganglioncellsgivesrisetoa
contractednonperistalticsegmentwithadilatedhypertrophied
segmentofnormalcolonaboveit.
Almostallcasespresentininfancyandchildhood.Approximately
halfarediagnosedintheneonatalperiodandmostoftheremainder
arediagnosedinearlychildhood.
Inthenewbornperiod,mostcommonsymptomsareabdominal
distentionandfailuretopassmeconium.
Olderpatientsmayexperiencechronicorintermittentconstipation.
Occasionally,infantsmaypresentwithadramaticcomplicationof
Hirschprung'sdiseasecalledenterocolitis.
Surgeryisalwaysrequired.Thebasicprincipleisremovalofmostor
alloftheaganglionicsegmentandanastomosingtheganglionic
boweltotherectum

1269.Commonestcauseofintussusception
is-
a)Submucouslipoma
b)Meckel'sdiverticulum
c)Hypertrophyofsubmucouspeyer'spatches
d)Polyp
CorrectAnswer-C
Ans.is'c'i.e.,Hypertrophyofsubmucosapeyer'spatches

1270.Allofthefollowingareclinicalfeatures
suggestiveoftrachea-esophagalfistula
except-

a)ChokingandCoughing
b)Regurgitation
c)Cyanosis
d)Fever
CorrectAnswer-D
Answer-D.Fever
Repeatedepisodesofcoughing,chokingandcyanosisoccuron
feedingwithTOF.

1271.Mostimportantpresentingfeatureof
periampullarycarcinomais-
a)Jaundice
b)Pain
c)WeightLoss
d)PalpableMass
CorrectAnswer-A
Answer-A
Hallmarkofpresentationofperiampullarycarcinomaisobstructive
Jaundice.
Jaundicesecondarytoobstructionofthedistalbileductisthemost
commonsymptomthatdrawsattentiontoampullaryandpancreatic
headtumors.
Itischaracteristicallypainlessjaundicebutmaybeassociatedwith
nauseaandepigastricdiscomfort.

1272.Bestmarkertoassessprognosisafter
Surgeryforcoloncarcinoma:
March2005

a)CA19-9
b)CA-125
c)Alphafetoprotein
d)CEA
CorrectAnswer-D
Ans.D:CEA
CEAisafamilyofrelatedglycoproteinsinitiallyfoundinembryonic
tissueandcolonmalignancies.
Thehalf-lifeofCEAisapproximately2weeks.PlasmalevelsofCEA
canbedeterminedreadilybyradioimmunoassay.However,its
usefulnessincoloncarcinogenesisscreeningislimitedbecauseof
itshighlevelintheplasmaofapatient'smalignanciesoriginating
fromothersitessuchasbreast,pancreas,stomachandlung.
Furthermore,CEAplasmalevelsmayalsobeincreasedinsmokers
orpatientswithchronicdiseases,suchasinflammatorybowel
disease,bronchitisandalcoholicliverdisease.
TheclinicalusefulnessofCEAforscreeningpurposesisfurther
limitedbythefactthatitsincreaseinleveltypicallyoccursonlywhen
thetumorpenetratesthroughtheserosa.Thus,earlylesionsmay
notbedetectedbyserumCEAlevelchanges.CEAistypically
elevatedwithmetastaticliverdisease,butrarelywithperitoneal
involvement.Thus,whilethespecificityforCEAtoidentifyoccult
CRCsishigh,thesensitivityislowinmoststudiesforscreening.
Accordingly,CEAisnotausefultestforthescreeningofCRC.
Regardlessoftheclinicopathologicstage,apreoperativeelevation

Regardlessoftheclinicopathologicstage,apreoperativeelevation
oftheplasmaCEAlevelpredictseventualtumourrecurrence.
HighlevelofCA19-9isseenintheplasmaofapatient's
malignanciesoriginatingfromcolon,breastandpancreas

1273.Acutepancreatitiscausesallofthe
followingexcept:
March2005,March2013(g)

a)Hypercalcemia
b)Increasedamylaselevel
c)Subcutaneousfatnecrosis
d)Hyperlipidemia
CorrectAnswer-A
Ans.A:Hypercalcemia
Complicationsofacutepancreatitis:
Intra-abdominal:
Pancreatic"collections":
Phlegmon
Pseudocyst
Abscess
Necrosis
Intestinal:
Paralyticileus
Gastrointestinalhaemorrhage-fromstressulceration,gastric
varicesduetosplenicveinobstruction,orruptureof
pseudoaneurysm
Necrotisingobstructionorfistulisationofcolon
*Hepatic-obstructivejaundiceduetooedematousnarrowingof
commonbileduct
*Renal-hydronephrosisandhydroureterofrightkidneydueto
peripancreaticinflammationinperirenalspace
*Spleen-ruptureorhaematoma,fromspreadofperipancreatic

inflammation
Systemiccomplicationsrangefromminorpyrexiatorapidly
fatal,multipleorgan-systemfailure.Theyinclude:
*Circulatoryshock-kininactivation,haemorrhage
*Disseminatedintravascularcoagulation
*Respiratoryinsufficiency:
Milde.g.hypoxaemia,atelectasis,pleuraleffusion
Severee.g.adultrespiratorydistresssyndrome
*Acuterenalfailure
*Metabolic:
Hypocalcaemia
Hyperglycaemia/diabetesmellitus
Hypertriglyceridaemia
*Pancreaticencephalopathy-confusions,delusions,coma
*Retinalarteriolarobstructioncausingsuddenblindness
Purtscher'sretinopathy
*MetastaticfatnecrosisAserumlevelthreetofourtimesabove
normalserumamylaselevel,isindicativeofacutepancreatitis,buta
normalleveldoesnotexcludethedisease.

1274.Heller'smyotomyisdonefor:
September2007,2009,2010
a)Esophagealcarcinoma
b)Pylorichypertrophy
c)Achalasiacardia
d)Inguinalhernia
CorrectAnswer-C
Ans.C:Achalasiacardia
Achalasiaisassociatedwithlossofganglioncellsintheesophageal
myentericplexus.
TheseimportantinhibitoryneuronsinduceLESrelaxationand
coordinateproximal-to-distalperistalticcontractionoftheesophagus
Achalasiaisanesophagealmotordisordercharacterizedby
increasedloweresophagealsphincter(LES)pressure,diminished-
to-absentperistalsisinthedistalportionoftheesophaguscomposed
ofsmoothmuscle,andlackofacoordinatedLESrelaxationin
responsetoswallowing.
Bariumradiologymayshow'bird'sbeak'appearance.
Esophageal(Heller)myotomyisasurgicalprocedurethatis
performedwithminimallyinvasivetechniques.Thelaparoscopic
approachappearstobemostappropriate.

1275.Allofthefollowingaresurgicaloptions
inmanagementofesophageal
carcinomaexcept-

a)IvorLewisApproach
b)Mckeown'sApproach
c)Transhiatalremoval
d)Sistrunkoperation
CorrectAnswer-D
Answer-D.Sistrunkoperation
IvorlewisoperationissubtotalesophagectomydoneforCa
esophagusinlowerthirdofesophagus
Intranshiatalesophagectomy(THE)Esophagusisremovedthrough
thediaphragmatichiatusbymakingincisionsintheabdomenand
theneck.
McKeownsSurgery-3IncisionApproach

1276.Bilateralparotidenlargementoccursin
all,Except:
a)Sjogren'ssyndrome
b)SLE
c)HIV
d)Chronicpancreatitis
CorrectAnswer-B
AnswerisB(SLE):
BilateralparotidenlargementisnotafeatureofSLE.
CausesofBilateralparotidenlargement

Viral
Metabolic

Endocrinal
Miscellaneous
infections
causes
?Mumps
? Diabetes
? Acromegaly
? Sarcoidosis
?Influenza
? mellitus
? Gonadal
? Amyloidosis
?Epstein
? Hyper
hypofunction
? Sjogren
barrvirus
? lipoproteinemia
syndrome
?Coxackie
Chronic
virusA
pancreatitise
?CMV
Cirrhosis
?HIV

1277.Chronichemolyticanaemiais
associatedwithwhichofthefollowing-
a)BrownPigmentstoneofthegallbladder
b)BlackPigmentstoneofthegallbladder
c)UricacidRenalCalculus
d)IntestinalObstruction
CorrectAnswer-B
Answer-B.BlackPigmentstoneofthegallbladder
Hemolysiscausesblackpigmentstoneofgallbladder.
Chronichemolyticstates(eg.hereditaryspherocytosis,sicklecell
disease).

1278.Allofthefollowingarefeaturesof
ZollingerEllisonsyndromeexcept
a)Intractablepepticulcers
b)Severediarrhoea
c)Betacelltumorsofthepancreas
d)Veryhighacidoutput.
CorrectAnswer-C
Ans.is'c'i.e.,Betacelltumoursofpancrease
GastrinomaorZollingerEllisonsyndromeisanon13cell
neuroendocrinetumourofthepancreasItsecretesgastrin
PathophysiologyofGastrinoma
Gastrinoma-->Increasesecretionofgastrin-->markedgastric
acidhypersecretionpepticulcer
PancreaticNeuroendocrineTumors
Tumour
Biologically Tumour
active location
Malignant Mainsymptoms
peptide
percentage andsigns
secreted
Gastrinoma
-
(non13cell
?Pain(79
Duodenum
tumour)
100%)
(70%)
-
Pancreas
Gastrin
60-90 'Diarrhoea(30
(25%)
73%)
othersites
?GERD(30-
(5%)
35%)
?Pepticulcer

Pancreas>
99%
?Symptomsof
(Insulinomas
hypoglycemia
Insulinoma
are
?Symptoms
(f3cell
Insulin
<10
distributed
releiveon
tumour)
equallyon
administrationof
headbody
glucose
andtailof
pancrease)

?Watery
diarrhoea
VIPOMA
(90-100%)
(Verner-
?-
Morrison
Vasoactive
Hypokalemia(80
Pancreas
syndrome,
intestinal
40-70 100%)
90%
pancreatic
peptide
?
cholera,
Hypochlorhydria
WDHA)
?Dehydration
(83%)
?Flushing(20%)
Pancreas
?Dermatitis
100%
(migratory
(usually
necrolytic
occursusuall
erythema)
singlyin
67-90%
pancreatic
?Glucose
tail)
intolerance
(40-90%)
Glucagonoma Glucagon
50-80% ?Weightloss(66
to96%)
?Anemia(33-
85%)
?Diarrhoea(15-
29%)
?
Thromboembolism

1279.Allofthefollowingistrueaboutheart
transplantationexcept-
a)Immmunosupressionisstartedpreoperatively
b)Itisonlyorthotopicandnotheterotopic
c)ABeatingheartcadaver/donorisneeded
d)HighPulmonaryarterialresistanceinacontraindication
CorrectAnswer-B
Answer-B.Itisonlyorthotopicandnotheterotopic
Typesoftransplant:
Heterotopic
Orthotopic
Atypicalhearttransplantationbeginswhenasuitabledonorheartis
identified.Theheartcomesfromarecentlydeceasedorbraindead
donor,alsocalledabeatingheartcadaver.

1280.Lateralaberrantthyroidrefersto-
a)Congenitalthyroidabnormality
b)Metastaticfocusinlymphnodes
c)Strumaovarii
d)Lingualthyroid
CorrectAnswer-B
Ansis"B"i.e.Metastaticfocusinlymphnodes
Lateralaberrantthyroidreferstoametastaticfocusinlymph
nodes
.
Lateralaberrantthyroidisamisnomerandalwaysreferstothe
presenceofmetastaticthyroidcarcinoma(papillary)incervical
lymphnodes.Itdoesnotindicatethepresenceofectopicthyroid
tissueasthenamesuggests.

1281.Achildpresentedwithbluntabdominal
trauma,thefirstinvestigationtobe
doneis-

a)USG
b)CTScan
c)CompleteHemogram
d)AbdominalXray
CorrectAnswer-A
Answer-A.USG
U/Sistheinvestigationofchoiceinbothstableaswellasunstable
patients.

1282.Vacuumassistedclosureis
contraindicatedinwhichofthe
followingconditions-

a)Chronicosteomyelitis
b)Largeamountofnecrotictissuewitheschar
c)Abdominalwound
d)Surgicalwounddehiscence
CorrectAnswer-B
Answer-B.Largeamountofnecrotictissuewitheschar
Negative-pressurewoundtherapy(NPWT),alsoknownasavacuum
dressingorV.A.C.dressing("vacuumassistedclosure"),is
atherapeutictechniqueusingasuctiondressingtoremove
excessexudationandpromotehealinginacuteorchronic
woundsandsecond-andthird-degreeburns.
ContraindicationsforNPWTuse
1. Malignancyinthewound
2. UntreatedOsteomyelitis
3. Nonentericandunexploredfistulas
4. Necrotictissuewithescharpresent
5. Exposedbloodvessels,anastomoticsites,organsandnervesinthe
periwoundarea(mustavoiddirectfoamcontactwiththese
structures)

1283.Trueregardingcystichygromais-
a)Nontransilluminant
b)Linedbycolumnarepitheliumepithelium
c)Developsfromjugularlymphaticsequestration
d)All
CorrectAnswer-C
Ans.isci.e.,Developsfromjugularlymphaticsequestration
Cystichygroma
Cystichygromaisaswellingusuallyoccuringinthelowerthirdofthe
neck
Itismostcommonlyseeninposteriortriangleoftheneck,butmay
alsooccurinaxilla,groin&mediastinum
Itresultsduetosequestrationofaportionofthejugularlymphsac
fromthelymphaticsystem.
Itusuallymanifestsintheneonateorinearlyinfancy(occasionally
presentatbirth)
Theswellingissoftandpartiallycompressibleandinvariably
increasesinsizewhenthechildcoughsorcries.
Thecharacteristicthatdistinguishesitfromallotherneckswellings
isthatitisbrilliantlytranslucent.
Thecystsarefilledwithclearlymphandarelinedbyendothelium.
Mostlythesearemultiplecystsbutoccasionallytheycanbe
unilocular.
Itmayshowspontaneousregression.
Treatment[Ref.:Sabiston18/ep2053;Schwartz9/ep1415]
Therearetwomethodsoftreatment:Surgicalexcision&
Sclerotherapy
Sabistonwrites-"Completesurgicalexcisionisthepreferred

treatment;however,thismaybeimpossiblebecauseofthehygroma
infiltratingwithinandaroundimportantneurovascularstructures.
Becausehygromasarenotneoplastictumors,radicalresectionwith
removalofmajorbloodvesselsandnervesisnotindicated.
Injectionofsclerosingagentssuchasbleomycinorthederivativeof
StreptococcuspvogenesOK-432havealsobeenreportedtobe
effectiveinthemanagementofcystichygromas.Intracysticinjection
ofsclerosantsappearstobemosteffectiveformacrocystic
hygromas,asopposedtothemicrocysticvariety."
"Themodernmanagementofmostcystichygromasincludesthe
combinationofsurgicalexcisionandimage-guidedsclerotherapy."-
Schwartz

1284.MostcommonsiteforCysticHygroma
is-
a)Lowerthirdofneck
b)Overlyingtheparotidgland
c)AlongtheZygomaticProminence
d)Postauricular
CorrectAnswer-A
Answer-A.Lowerthirdoftheneck
Mostcystichygromasinvolvethelymphaticjugularsacsandpresent
intheposteriorneckregion.
Theothercommonsitesaretheaxillary,mediastinum,inguinal,and
retroperitonealregions,andapproximately50%ofthempresentat
birth.
Cystichygromasusuallypresentassoftcysticmassesthatdistort
thesurroundinganatomy,includingtheairway,whichcanresultin
acuteairwayobstruction.

1285.Thesizebeyondwhichtheriskof
ruptureofanabdominalaneurysm
significantlyincreasesisgreaterthan-

a)5.5cm
b)6cm
c)6.5cm
d)7cm
CorrectAnswer-A
Answer-A.5.5cm
HighriskAAA
Endovascularaneurysmalrepair(EVAR)
AAArepair-5.5cm/>5cmfemales
Symptomaticaneurysm

1286.Chvosteksigncouldbeseenafter-
a)Gastrojejunostomy
b)Totalthyroidectomy
c)SubtotalThyroidectomy
d)HellersCardiomyotomy
CorrectAnswer-B
Answer-B.Totalthyroidectomy
Chvostek'ssignreferstothecontractionoftheipsilateralfacial
musclesonpercussionofthefacialnervebelowthezygoma.
Itisduetohypocalcemiaandonecommoncauseofhypocalcemia
isacomplicationduringthyroidectomies.
Theparathyroidglandproducesaparathyroidhormone(PTH),and
resectioncanleadtoadecreaseinPTHlevels.
PTHregulatescalciumlevelsinthebody;hence,areductioninits
serumlevelwillleadtohypocalcemia.
So,amongthemostcommonpostoperativecomplicationsfollowing
thyroidsurgeryishypocalcemia.

1287.ReactionaryHemorrhageoccursdueto
-
a)Dislodgementofclot
b)Infection
c)Damagetoabloodvessel
d)Pressurenecrosis
CorrectAnswer-A
Answer-A.Dislodgementofclot
Dislodgementofclot
Ligatureslip
NoramlisationofBPand
vasodilation

1288.Transplantedkidneyisrelocatedto
whichregionintherecepient'sbody?
a)Retroperitonealrgion
b)Lumbarregion
c)Epigastrium
d)BesidethedysfunctionalKidney
CorrectAnswer-A
Answer-A.Retroperitonealrgion
Themostcommonlocationforplacingakidneytransplantisin
retroperitonealiliacfossai.
InmostcasesKidneyisplacedretroperitoneallyandtheiliacarteries
andveinsareusedforperfusionandureteristransplanteddirectly
intobladder.

1289.Claudicationduetopopliteofemoral
incompetenceisprimaryseenin
a)Thigh
b)Calf
c)Buttocks
d)Feet
CorrectAnswer-B
Answer-B.Calf
AortaandCommonIliac-Buttocks
FemoralArtery-Thigh
Superficialfemoralartery-Calfandpoplitealartery
PosteriortibialArtery-Feet

1290.Whichofthefollowingiscorrect
managementofabdominal
compartmentsyndrome

a)Antihypertensives
b)UrgentOpeningofthesurgicalwoundandapplicationofthe
Bogotabag
c)UrgentFasciotomy
d)Waitandmonitorfor24hours
CorrectAnswer-B
Answer-B.UrgentOpeningofthesurgicalwoundand
applicationoftheBogotabag
Abdominalcompartmentsyndromeisasurgicalemergencyand
treatmentincludesrapiddecompressionoftheelevated
intraabdominalpressurebyopeningtheabdominalwoundand
performingatemporaryclosureoftheabdominalwallwithmeshora
plasticbag.Permanentclosureisdone5to7dayslaterwhenthe
conditionresolves.

1291.Allofthefollowingistrueabout
congenitalhypertrophicpyloric
stenosisexcept

a)RamStedtPyloromyotomyisthetreatmentofchoice
b)NonBiliousvomitingisseen
c)Metabolicacidosisoccurs
d)Morecommoninmales
CorrectAnswer-C
Answer-C.Metabolicacidosisoccurs
Charactersticallythefirstbornmalechildisaffected.
Theconditonismostcommonlyseenat4weeksafterbirthranging
fromthe3rdweektoonrareoccasion
Non-biliousvomiting,becomingincreasinglyprojectile,occursover
severaldaystoweeks
Infantsdevelopametabolicalkalosiswithseveredepletionof
potassiumandchlorideions.
Treatment-Surgery:Fredet-Ramstedtpyloromyotomy.(Initthe
pyloricmassissplitwithoutcuttingthemucosa)

1292.Foodcancommonlygetobstructedin
theesophagusatallofthefollowing
locationsexcept

a)Crossingofleftbronchus
b)Crossingofarchofaorta
c)Diagphragmaticaperture
d)Crossingofthehemiazygousvein
CorrectAnswer-D
Answer-D.Crossingofthehemiazygousvein
Pharyngoesophagaljunction-6inches9

1293.Afullthicknesswoundthatisnot
suturedhealsby
a)PrimaryHealing
b)SecondaryHealing
c)DelayedprimaryHealing
d)Reepithelization
CorrectAnswer-B
Answer-B.SecondaryHealing
SecondaryHealing
Athirdtypeofhealingisknownassecondaryhealingorhealingby
secondaryintention.Inthistypeofhealing,afull-thicknesswoundis
allowedtocloseandheal.Secondaryhealingresultsinan
inflammatoryresponsethatismoreintensethanwithprimarywound
healing.Inaddition,alargerquantityofgranulomatoustissueis
fabricatedbecauseoftheneedforwoundclosure.Secondary
healingresultsinpronouncedcontractionofwounds.

1294.AnIncisionalwoundhealsby
a)PrimaryHealing
b)SecondaryHealing
c)DelayedprimaryHealing
d)Reepithelization
CorrectAnswer-A
Answer-A.PrimaryHealing
PrimaryHealing
Primarywoundhealingorhealingbyfirstintentionoccurswithin
hoursofrepairingafull-thicknesssurgicalincision.Thissurgical
insultresultsinthemortalityofaminimalnumberofcellular
constituents.

1295.Acysticmassatthebaseofumbilical
cordinaneonatecouldbe
a)AllantoicCyst
b)Meckel'sDiverticulum
c)Ventralmesogastrium
d)CysticHygroma
CorrectAnswer-A
Answer-A.AllantoicCyst
Allantoiccystsareatypeoftruecystoftheumbilicalcord.
Theallantoisformsfromthepartofthefetalyolksacthateventually
becomestheprimitivehindgut(thecloaca).Thecloacadividesinto
thehindgutposteriorlyandtheurogenitalsinusanteriorly.

1296.TrueaboutMALTomais
a)Theyaresecondarygastriclymphomas
b)H.Pyloriinfectionisariskfactor
c)Commonlyseeningastriccardia
d)TheyareatypeofTcelllymphoma
CorrectAnswer-B
Answer-B.H.Pyloriinfectionisariskfactor
Stomachisthemostcommonextranodalsiteoflymphoma.
Itisoftwotypes-Primary&Secondary
Mostoftheprimarylymphomas(about60%)ariseinMALT(mucosa
associatedlymphoidtissue).MALTisusuallyassociatedwith
chronicH.pyloriinfection.Solow-gradeMALTlymphomasare
thoughttoarisebecauseofchronicH.pyloriinfection.Theselow
gradelymphomasmaylaterondegeneratetohighgrade
lymphomas.
mucosaassociatedlymphoidtissue.ItisusuallyprimaryGI
lymphoma(4%ofgastriclymphoma);ofnon-Hodgkin'sBcell
type.

1297.FalseaboutGastriclymphomais:
a)Stomachisthemostcommonsite
b)AsosciatewithH.pyloriinfection
c)Totalgastrectomywithadjuvantchemotherapyisthetreatment
ofchoice
d)5yrsurvivalrateaftertreatmentis60%.
CorrectAnswer-C
Totalgastrectomywithadjuvantchemotherapyisthetreatmentof
choice[Ref:Sabiston18/ep1269(17/cp1312);Harrison17/e,p
573;Schwartz9/ep935(8/e,p981)]
Stomachisthemostcommonextranodalsiteoflymphoma.
Itisoftwotypes-Primary&Secondary
Primarygastriclymphoma-islymphomaofthestomachwhich
exhibitsnoevidenceofliver,spleen,mediastinallymphnodesor
bonemarrowinvolvementatthetimeofdiagnosis(regionallymph
nodeinvolvementmaybepresent)
Primarygastriclymphomaismainly(>95%)Non-Hodgkin's
lymphomaofBcellorigin.
Mostoftheprimarylymphomas(about60%)ariseinMALT(mucosa
associatedlymphoidtissue).
MALTisusuallyassociatedwith
chronicH.pyloriinfection.Solow-gradeMALTlymphomasare
thoughttoarisebecauseofchronicH.pyloriinfection.Theselow
gradelymphomasmaylaterondegeneratetohighgrade
lymphomas
ImmunodeficiencyandH.pyloriinfectionareriskfactorsforBcell
lymphoma.
Mostcommonsiteofinvolvement(likethatofadenocarcinoma)is
gastricantrum.

Secondarygastriclymphoma
Stomachisthemostcommonextranodalsiteofsystemiclymphoma.
AlmostallofthemareNon-Hodgkinstypes
Treatment
Theroleofgastricresectioniscontroversial.Recentstudieshave
shownsimilardiseasefree5yearsurvivalratesinpatientstreated
withsurgery+chemotherapy+radiotherapyandpatientstreatedwith
chemotherapy+radiationtherapyalone.(Mostpatientswithhigh
gradegastriclymphomasarenowtreatedwithchemoradiation
alone,withoutsurgicalresection.Fordiseaselimitedtothestomach
andregionalnodes,radicalsubtotalgastrectomymaybeperformed,
especiallyforbulkytumorswithbleedingand/orobstruction.)
Mostcommonchemotherapeuticregimenis?CHOPplusrituximab
[CHOPstandsforcyclophosphamide,doxorubicin,vincristine,&
prednisone]
aboutoptiondi.e.(5yrsurvivalrateaftertreatmentis60%.)
Harrison
17/ewrites-"Subtotalgastrectomy,usuallyfollowedby
combinationchemotherapy,hasledto5-yearsurvivalratesof40-
60%inpatientswithlocalizedhigh-gradelymphomas."


1298.Whichofthefollowinggasismostcommonlyusedinlaproscopy?
a)CO2
b)N2O
c)O2
d)Helium
CorrectAnswer-A
CO2isusedinlaproscopybecauseitiscommontothehumanbody
andcanbeabsorbedbytissueandremovedbytherespiratory
system.
Itisalsonon-flammable,whichisimportantbecauseelectrosurgical
devicesarecommonlyusedinlaparoscopicprocedures.

1299.Strawberrygallbladderisseenin?
a)Gangreneofgallbladder
b)Porcelaingallbladder
c)Adenomatosis
d)Cholesterosis
CorrectAnswer-D
Ans.is'i.e.Cholesterosis
Cholecystosisisthechronicinflammatoryconditionofthe
gallbladderwithcholesteroldeposits.
Types
I.Aggregationsofcholesterolcrystalsinthemucosaorsubmucosa
--cholesterosis(Strawberrygallbladder).
Lipoidcontentsarepresentinlargefoamycellsthathave
phagocytosedcholesterol.Herecysticductisnormal.
Thediseaseoccursonlyingallbladder.Itisapremalignant
condition.
II.Cholesterolladenpolypoidprojectionsinthemucosa
--cholesterolpolyposis(Gallbladderpolyp).
III.Granulomatousthickeningandhyperplasiaofthegallbladder
--cholecystitisglandularisproliferans.
IV.Diverticulaformationinthewallofthegallbladder--diverticulosis
ofgallbladder.
V.Gallbladderwallfistula.

1300.Allofthefollowingarecausesof
UrothelialCarcinomasExcept-
a)Smoking
b)Industrialsolvents
c)Exposuretothorotrast
d)Alcoholconsumption
CorrectAnswer-D
Answer-D.Alcoholconsumption
Industrialdyesorsolvents
Excessiveanalgesicintake,
Balkannephropathy,and
ThoseexposedtoThorotrast,(acontrastagentpreviouslyusedfor
retrogradepyelography

1301.Inapatientwithrenalcellcarcinoma
withathrombusinIVCrenalvein,
whichisthebestinvestigationfor
diagnosis?

a)CTscan
b)Angiography
c)Colourdopplerimaging
d)IVP
CorrectAnswer-A
Ans.CTscan
InvestigationofchoiceinRCC-CTscan.

1302.Whichofthefollowingprecancerous
conditionsiftreatedwouldnotleadto
cancer

a)CervicalintraepithelialNeoplasia
b)Ductalcarcinomainsituofbreast
c)LobularCarcinomainsituofbreast
d)VaginalintraepithelialNeoplasia
CorrectAnswer-A
Answer-A.CervicalintraepithelialNeoplasia
Thecancers,whichhavewellknownprecancerouscondition,canbe
diagnosedandtreatedatpre-cancerousstagetopreventthe
developmentofcancer:-
1. CervixGIN
2. ColonPolyp

1303.Commonestsiteofpepticulceris:
a)1stpartofDuodenum
b)IIndpartofduodenum
c)Distal1/3ofstomach
d)Pylorusofthestomach
CorrectAnswer-A
AnswerisA(1stpartofDuodenum)
Firstpartofduodenumisthemostcommonsiteforpepticulceration.
Sitesinorderofdecreasingfrequency:
1. Duodenum,1stportionQ
2. Stomach,usuallyantrumQ
3. Atthegastro-esophagealjunction,inthesettingof
gastroesophagealrefluxQ
4. WithinthemarginsofagastrojejunostomyQ
5. Intheduodenum,stomochorjejunumofpatientswithZollinger-
EllisonSyndromeQ
6. WithinoradjacenttoaMeckelsdiverticulumthatcontainsectopic
gastricmucosaQ
Pepticulcersareusuallysolitarylesionslessthan4cmindiameter


1304.Whichofthefollowingistheagentofchoiceforcryosurgery?
a)Nitrousoxide
b)CO2snow
c)Liquidnitrogenspray
d)Freons
CorrectAnswer-C
Thishasatemperatureof-196c(spray/probe)&theonlycryogenadvocatedformalignant
skinlesions.
Ref:Textbookofdermatosurgery&cosmetology,satishSsavant,E-2,P-117.

1305.Incaseofpolytraumawithmultiple
injuriestothechest,neckand
abdomen,highestpriorityisgivento

a)Stabilizationofcervicalspine
b)Staringofkoids
c)Vasopressors
d)Assesingdisability
CorrectAnswer-A
Answer-A.Stabilizationofcervicalspine
1.Primarysurvey-itshouldbedonefirstwhenanypolytrauma
patientpresentsinthefollowingorder:
1.1Airwaymaintenancewithcervicalspineprotection
1.2Breathingandventilation
1.3Circulationwithhemorrhagecontrol
1.4Disability/Neurologicassessment
1.5Exposureandenvironmentalcontrol
2.Secondarysurvey-completehistoryandexamination.

1306.Thyroidnoduleina65yearoldmale
whoisclinicallyeuthyroidismostlikely
tobe

a)Follicularadenoma
b)FollicularCarcinoma
c)ThyroidCyst
d)Multinodulargoiter
CorrectAnswer-A
Answer-A.Follicularadenoma
Mostcommonsolitarythyroidnoduleisbenigncolloidnodule.
2"dmostcommoncauseofsolitarythyroidnoduleisfollicular
adenoma.

1307.InSubtotalThyroidectomy,Whatis
true
a)Removalofonelobeandisthmus
b)Removalofbothlobesleavingbehind6-8gramsoftissue
c)Removalofentirethyroidwithcervicallymphnodes
d)Removalof1lobewithisthmusandthesecondlobepartially
CorrectAnswer-B
Answer-B.Removalofbothlobesleavingbehind6-8gramsof
tissue
Subtotalthyroidectomy--Removalofmajorityofbothlobesleaving
behind4-5grams(equivalenttothesizeofanormalthyroidgland)
ofthyroidtissueononeorbothsides--thisusedtobethemost
commonoperationformultinodulargoitre.

1308.Whichofthefollowingstatements
aboutBranchialcystsistrue:
a)50-70%areseeninlungs
b)Mostcommonsiteismediastinum
c)Theyarepremalignantlesions
d)InfectionisuncommoninPulmonarybronchogeniccysts
CorrectAnswer-B
AnswerisB(Mostcommonsiteismediastinum):
Mostcommonsiteofbronchial/bronchogeniccystsismediastinum.
Onlyabout15%ofbronchogeniccystsoccurinthelungs
(pulmonarybronchialcysts).Pulmonarybronchogeniccystsoften
becomeinfected.Bronchogeniccystsarebenignlesionanddonot
havemalignantpotential(notpremalignant).
BronchialCyst/Bronchogeniccysts:Review
Bronchialcystsrepresentislandsofbronchialtissueleftbehind
duringthebranchingoftheairwaysduringearlyfetaldevelopment.
Theyariseduetoabnormalbuddingofthetracheobronchialtreeand
foregutandarelinedbybronchialepithelium.
Themostcommonsiteofbronchialcystsismediastinum.
Theothersiteofbronchialcystiswithinthepulmonaryparenchyma
(lung)(Lesscommonsite-15%)
Mediastinalbronchialcyst
Pulmonaryparenchymalcyst(lungs)

Mostcommonsitefor
Lesscommonsite(-15%)
bronchialcysts
Mostcommonsiteisthelower
Mostcommonsiteismiddle
lobes
mediastinum
Commonlyarisewhenbronchial
Commonlyarisewhen
tissueisseparatedfromairways

bronchialtissueis
lateingestation
separatedfromairwaysearly Communicationwith
ingestation
tracheobronchialtreeismore
Communicationwith
commonthanwithmediastinal
tracheobronchialtreeisrare
cysts.
Thesecystsoftenbecomeinfected.
Mostbronchogeniccystsareasymptomaticanddiscoveredas
incidentalradiographicfindingsinayoungadult.
Whensymptomsdooccurtheyresultmostcommonlyfrominfection.
Pulmonaryparenchymalcystsoftenbecomeinfected?Rudolph
Bronchogeniccystsarenotconsideredpremalignantlesions.
HoweveraccordingtoRudolph'stextbookthereisasmallriskof
malignantchangeandthebestapproachisremovalandhistological
examination.


1309.A65yearoldfemalepresentswitha
swellingintheneckdiagnosedasa
solitarythyroidnodule.Thepatientis
investigatedandascanshows
increaseduptakeofiodine.SerumT3
andT4areelevated.Mostprobabe
diagnosisis

a)BenignColloidNodule
b)ToxicAdenoma
c)Follicularadenoma
d)ToxicMultinodulargoitre
CorrectAnswer-B
Answer-B.ToxicAdenoma
Elavatedthyroidhormonelevelswithahyperfunctioningnoduleis
suggestiveofatoxicadenoma.
Acoldnoduleismorelikelytobemalignantthenahotnodule.

1310.MostcommoncauseofgoiterinIndia
is
a)DiffuseEndemicGoitre
b)PapillaryCarcinoma
c)ToxicMultinodularGoitre
d)Hashimoto'sThyroiditis
CorrectAnswer-A
Answer-A.DiffuseEndemicGoitre
Thelackoftheiodineleadstodecreasedsynthesisofthyroid
hormonesandacompensatoryincreaseinTSH,whichinturnleads
tofollicularcellhypertrophyandhyperplasiaandgoitrous
enlargement-Diffusehyperplasticgoiter.
Mostlypatientsareeuthyroid.

1311.Whichofthefollowingisanindication
forthoracotomyincaseofhemothorax
?

a)Persistantdrainageof250ml/hr
b)Totaloutputof1000m1ofblood
c)Fallingbloodpressure
d)Shiftofmediastimumtotheoppositeside
CorrectAnswer-A
Answer-A.Persistantdrainageof250ml/hr
Itisclassifiedaccordingtotheamountofblood.
Minimalhemothorax350ml
Moderatehemothorax350-1500ml
Massivehemothorax1500mlormore
Totalhemorrhagicoutputexceeds1500mlofblood

1312.A40yearoldmalewithchesttrauma
presentswithbreathlessness,
decreasedrespiratorysoundsonthe
rightside,hyperresonanceon
percussionanddistendedneckveins.
Thepossiblediagnosisis

a)TensionPneumothorax
b)CardiacTamonade
c)FlailChest
d)MyocardialInfarction
CorrectAnswer-A
Answer-A.TensionPneumothorax
Tensionpneumothoraxoccurswhenairbecomestrappedinthe
pleuralspaceunderpressure.
Itdevelopswhena'one-wayvalve'airleakoccurseitherfromthe
lungorthroughthechestwall.
causesare
Penetratingchesttrauma,
Bluntchesttrauma,
Iatrogeniclungpunctures(e.g.duetosubclaviancentral
venepuncture)and
Mechanicalpositive-pressureventilation
Clinical
Clinicalsignsandsymptomsincludedyspnea,tachypnea,
hypotension,diaphoresis,anddistendedneckveins.

1313.A40yearoldmalepresentedwitha
penetratingtraumatochest.Heis
dyspnoeicwithdistendedneckveins
withhypotensionandmediastinumis
shiftedtooppositeside.Thereisa
suckingwoundoverthechest.The
mostappropriatemanagementwould
be

a)Insertionofalargeboreneedleinthe2"ICSinthemid
clavicularline
b)FluidResuscitation
c)StartingInotropicsupport
d)EndotrachealIntubation
CorrectAnswer-A
Answer-A.Insertionofalargeboreneedleinthe2"ICSinthe
midclavicularline
Thisisacaseoftensionpneumothorax.
Treatmentconsistsofimmediatedecompression,initiallybyrapid
insertionofalarge-boreneedleintothesecondintercostalspacein
themidcalvicularlineoftheaffectedhemithorax,andthenfollowed
byinsertionofachesttubethroughthefifthintercostalspaceinthe
anterioraxillaryline.


1314.Whichofthefollowingstagesoflip
carcinomadonothavenodal
involvement?

a)T2N1
b)T3NO
c)T1N1
d)T2N2
CorrectAnswer-B
Answer-B.T3NO

1315.Allofthefollowingmayleadto
pneumatoceleformationexcept
a)Staphylococcalpneumonia
b)Positivepressureventilation
c)Hydrocarboninhalation
d)ARDS
CorrectAnswer-D
Answer-D
Pneumatoceleformationoccursasasequelatoacutepneumonia,
commonlycausedbyStaphylococcusaureus.However,
pneumatoceleformationalsooccurswithotheragents,including
Streptococcuspneumoniae,Haemophilusinfluenzae,Escherichia
coli,groupAstreptococci,Serratiamarcescens,Klebsiella
pneumonia,adenovirus,andtuberculosis.
Noninfectiousetiologiesincludehydrocarboningestion,trauma,and
positivepressureventilation.
Inprematureinfantswithrespiratorydistresssyndrome,
pneumatocelesresultmostlyfromventilator-inducedlunginjury.

1316.A54yearoldwomanisdiagnosedas
havingcarcinomaoftherenalpelvisof
sizelessthan4cmwithoutany
metastasis.Thebesttreatmentoption
is

a)Partialnephrectomy
b)RadicalNephrectomy
c)Chemotherapyandimmunaotherapy
d)PalliativeRadiotherapy
CorrectAnswer-A
Answer-A.Partialnephrectomy
Partialnephrectomyisnowbeingusedasprimarysurgicaltherapy
forpatientswithtumourlessthan4cminsize,earlierRadical
nephrectomywasthet/tofchoicefortumorofanysize.

1317.Ainhumisseenin?
a)Baseofgreattoe
b)Baseoffingerstips
c)Baseoftoe
d)Ankle
CorrectAnswer-C
Ans.is'c'i.e.,BaseofToe
Ainhum:isaconditionusuallyaffectingNegromales(butsome
females)whohaverunbarefootinchildhood.
ClinicalFeatures:Afissureappearsatinterphalangealjointoftoe-
usuallythefifth.
Thisfissurebecomesafibrousband,encirclesthedigitandcauses
necrosis.

1318.Theinvestigationofchoicefor
dysphagiais
a)Endoscopy
b)ManometricStudy
c)CtScan
d)BariumSwallow
CorrectAnswer-A
Answer-A.Endoscopy
Theinvestigationofchoiceinisendoscopy.
Bariumswallowisthefirstinvestigation.

1319.MostcommonprecipitantofRaynaud's
phenomenonis
a)Exposuretocold
b)Exposuretoheat
c)Psychosocialtriggers
d)Exertion
CorrectAnswer-A
Answer-A.Exposuretocold
Isdsofyoungwomen(F:Mratiois5:1)
Commonlytheupperlimbsareaffectedspeciallythefingers(the
thumbisgenerallyescaped)
Exposuretocoldismaintrigger

1320.Allofthefollowingareprinciplesof
negativepressurewoundtherapy
except

a)Stabilizationofwoundenvironment
b)Clearanceofinfection
c)Macrodeformationofthewound
d)Decreasededema
CorrectAnswer-B
Answer-B.Clearanceofinfection
FourprimaryeffectsofNPWTonwoundhealing:
Macro-deformation-drawingthewoundedgestogetherleadingto
contraction.
Stabilisationofthewoundenvironment-ensuringitisprotectedfrom
outsidemicroorganismsinawarmandmoistenvironment.
Reducedoedema-withremovalofsofttissueexudates.
Micro-deformation-leadingtocellularproliferationonthewound
surface.

1321.Whichisthemaincontraindicationfora
liverbiopsy?
a)Thrombocytopenia
b)Hemangioma
c)Ascites
d)All
CorrectAnswer-D
Answer-D
Contraindicationsforliverbiopsyincludethefollowing:
Increasedprothrombintime(PT),internationalnormalizedratio
(INR)greaterthan1.6
Thrombocytopenia,plateletcountlowerthan60,000/L
Ascites(transjugularroutepreferred)[10]
Difficultbodyhabitus(transjugularroutepreferred)
Suspectedhemangioma
Suspectedechinococcalinfection
Uncooperativepatient

1322.Aneurysmaldilationofthesmallbowel
isseenin
a)SmallbowelLymphoma
b)GallStoneIleus
c)DuodenalAtresia
d)SjogrensSyndrome
CorrectAnswer-A
Answer-A.SmallbowelLymphoma
Aneurysmaldilatation:30%,itoccursduetoreplacementof
muscularisbytumourorinfiltrationofmyentericnerveplexus
Despitetheextensiveinvolvement,smallbowelobstructionis
uncommonbecauseoflackofdesmoplasticreaction,and
perforationisrare

1323.Spontaneousesophagealruptureis
mostcommonin
a)Belowthediagphragmaticaperture
b)Pharyngoesophagaljunction
c)Abovethediagphragmaticaperture
d)Atthecrossingofthearchofaorta
CorrectAnswer-C
AnswerC.Abovethediagphragmaticaperture
Instrumentalperforationiscommoninthepharynxordistal
esophagus.
Spontaneousrupturemayoccurjustabovethediaphragminthe
posterolateralwalloftheesophagus.

1324.Whatisthetreatmentofchoicein
desmoidtumors?
a)Irradiation
b)Wideexcision
c)Localexcision
d)Localexcisionfollowingradiation
CorrectAnswer-B
Ans.is'b'i.e.,Wideexcision

1325.TrueaboutDentigerouscyst:
a)Arisesinrelationtouneruptedteeth
b)Itmostcommonlyencroachesmaxillaryantrum
c)Mandibularthirdmolariscommonsite
d)Commoninmandible
e)All
CorrectAnswer-A:C:D
Answer-A,ArisesinrelationtouneruptedteethC,Mandibular
thirdmolariscommonsiteD,Commoninmandible
Commoninlowerjaw(mandible)inwomen30-40years.
Itoccursinrelationtounerupted,permanent,molartooth,most
commonlytheupperorlowerthirdmolar.

1326.Apatientpresentswithdifficultywith
swallowingliquidsbutnotsolids.The
bestinvestigationtomakeadiagnosis
is

a)Endoscopy
b)Endoscopicultrasound
c)Manometry
d)PETCT
CorrectAnswer-C
Answer-C.Manometry
H/Oofdysphagiawithmoretoliquidsthansolidssuggestsachalasia
cardia.Inallotherobstructivecausesofesophagus,dysphagiafor
solidsisseenfirst
Manometryisusedfordiagnosis.

1327.Whichofthefollowingistrueabout
menetrier'sdisease
a)Itispremalignantcondition
b)Thereisincreasedgastricacidsecretion
c)Atrophiedmucosalfoldsareseen
d)Affectsthestomachandsmallintestines
CorrectAnswer-A
Answer-A.Itispremalignantcondition
M?n?trierdiseaseisarare,acquired,premalignantdiseaseofthe
stomachcharacterizedbymassivegastricfolds,excessivemucous
productionwithresultantproteinloss,andlittleornoacidproduction.
Thedisorderisassociatedwithexcessivesecretionoftransforming
growthfactoralpha(TGF-).

1328.Allareresectedinwhipplesoperation
except?
a)Duodenum
b)Headofpancreas
c)Neckofpancreas
d)Commonbileduct
CorrectAnswer-C
Ansis'c'
Whipplesoperation(Pancreaticoduodenectomy)
isthemostcommonlyperformedoperationforcarcinomaofheadof
pancreas.
Itincludesresectionof:
-distalstomach-duodenum
-gallbladder-proximaljejunum
-CBD-regionallymphatics
-headofpancreas
Restorationofgastrointestinalcontinuityrequires-
pancreaticojejunostomycholedochojejunostomy&
-gastrojejunostomy

1329.CABreastmaylocallyspreadtoallof
thefollowingmusclesexcept
a)PectoralisMajor
b)PectoralisMinor
c)LatissimusDorsi
d)SerratusAnterior
CorrectAnswer-C
Answer-C.LatissimusDorsi
latissimusDorsiisusedforbreastreconstructionandmayrarelybe
involvedinCABreastlocalSpread'
Musclesinvolvedinbreastcancer
1. Pectoralmuscles:Pectoralismajor,Pectoralisminor.
2. Musclesofchestwall:Intercostalmuscles,serratusanterior

1330.Whichisthemostcommontypeofmale
breastcancer?
a)InfiltratingDuctalCarcinoma
b)LobularCarcinoma
c)MucinousCarcinoma
d)ColloidCarcinoma
CorrectAnswer-A
Answer-A.InfiltratingDuctalCarcinoma
Ittendstopresentasalumpandismostcommonlyaninfiltrating
ductalcarcinoma.
Morethan90%ofcasesareinfiltratingductcarcinoma,about10%
areductalcarcinomainsitu(DCIS).
Theknownpredisposingcausesincludegynecomastiaandexcess
endogenousorexogenousestrogen.

1331.InPatey'smastectomythestepnot
doneis
a)Nippleandareolaremoved
b)Surroundingnormaltissueoftumorisremoved.
c)Pectoralismajorremoved
d)Pectoralisminorremoved
CorrectAnswer-C
Ans.is'c'i.e.(Pectoralismajorremoved)
Letsseethenomenclatureofvarioussurgeriesonbreast
SimpleorTotalmastectomy
*itremovesallbreasttissue,thenipple-areolacomplex,andskin.
Extendedsimplemastectomy
*Simplemastectomy+removaloflevelIaxillarylymphnodes.
Modifiedradicalmastectomy
itremovesallbreasttissue,thenipple-areolacomplex,skinand
thelevelIandlevelIIaxillarylymphnodes.
Halstead'sRadicalmastectomy
removesallbreasttissueandskin,thenippleareolacomplex,
thepectoralismajorandminormusclesandthelevelI,IIand
IIIaxillarylymphnodes.
ModifiedRadicalMastectomy
Twoformsofmodifiedradicalmastectomyareinuse
Auchincloss(pronouncedas'aushincloss')procedure
*Hereboththepectoralismajorandminormusclesarepreserved
withremovaloflevelIandIIaxillarylymphnodes
Patey'sProcedure
herethepectoralisminormuscleisremovedtoallowcomplete
dissectionoflevelIIIaxillarylymphnodes


Scanlon'smodificationofPatey'sprocedure
*herethepectoralisminormuscleisdividedinsteadofremoving.
Divisionofpectoralisminormuscleallowscompleteremovalof
levelIIIlymphnodes
HalsteadRadicalMastectomy
Inthisoperationfollowingstructuresareremoved.
i)thewholebreast
ii)theportionofskinoverlyingthetumor,whichincludesthe
nipple-areolacomplex.
iii)thesubcutaneousfatandthedeepfasciaverticallyfromthe
lowerborderoftheclavicleuptotheupperquarterofthesheathof
therectusabdominisandhorizontallyfromthestrenumtothe
anteriorborderoflattissimusdorsi
iv)pectoralismajormuscle
v)pectoralisminormuscleandclavipectoralfascia
vi)upperpartoftheaponeurosisoftheexternalobliqueand
anteriorpartsofafewdigitationsoftheserratusanteriormuscle
vii)allfattyandlooseareolartissuealongwithlevelI,II&III
axillarylymphnodes
Structuressavedare:
i)theaxillaryveinandthecephalicvein
ii)thelongthoracicnerveofBell(Nervetoserratusanterior).
Thenervetolatissimusdorsimaybesacrifiedifrequired.
Alsoknow
ExtendedRadicalMastectomy-Radicalmastectomy+removalof
internalmammarylymphnodes
SuperRadicalMastectomy-Radicalmastectomy+removalof
internalmammary,mediastinalandsupraclavicularlymphnodes.


1332.45yearoldfemaleunderwentmodified
radicalmastectomywithaxillary
clearanceforCAbreast.Aftersurgery
shecouldnotliftherarmabovehead.
Whichnerveislikelytobeinjured?

a)Intercostobrachialnerve
b)LongthorasicnerveofBell
c)NervetolatissimusDorsi
d)LateralPectoralnerve
CorrectAnswer-B
Answer-B.LongthorasicnerveofBell
Overheadabductioniscausedby:-

1. Serratusanterior:suppliedbylongthoracicnerve.
2. Trapezius:suppliedbyspinalaccessorynerve.

1333.A22yearoldwomancomeswithanon
progressivemassinaleftbreastsince
6months.Therearenoassociated
symptoms.Examinationshowsa
mobilemassnotattachedtothe
overlyingskinorunderlyingtissue.The
possiblediagnosisis

a)Fibroadenoma
b)CystasarcomaPhylloides
c)ScirrhousCarcinoma
d)Fibroadenosis
CorrectAnswer-A
Answer-A.Fibroadenoma
Clinically,theypresentasfirmmassesthatareeasilymovable.
(MOUSEINTHEBREAST)Theyslideeasilyundertheexamining
fingersandmaybelobulatedorsmooth.
Mammographyisoflittlehelpindiscriminatingbetweencystsand
fibroadenomas;however,ultrasonographycanreadilydistinguish
betweenthembecauseeachhasspecificcharacteristics.
FNAbiopsycanalsobeusedtoconfirmtheimagingfindings

1334.Mostcommonlocationofbreastcancer
is
a)Lowerinnerquadrant
b)Nipple
c)Upperinnerquadrant
d)Upperouterquadrant
CorrectAnswer-D
Answer-D.Upperouterquadrant
Breastcancerisfoundmostfrequentlyintheupperouterquadrant.
Leastfrequentlyinlowerinnerquadrant.


1335.A55yearsoldmalewithaknoen
historyofgallstonespresentswith
chiefcomplintsofsevereabdominal
painandelevatedlevelsofserumlipase
withperiumbilicalecchymosis.Allof
thefollowingareprognosticcriteriato
predictseverityofthecondition
except

a)Age
b)SerumLDH
c)Basedeficit
d)SerumGGT
CorrectAnswer-D
Answer-D.SerumGGT

1336.Allaretrueaboutcarcinomapalate,
except-
a)Slowgrowing
b)Bilaterallymphaticspread
c)Adenocarcinoma
d)Presentswithpain
CorrectAnswer-D
Ans.is'd'i.e.,Presentswithpain

1337.A45yearoldlawyerpresentswithpain
intheabdomenmoresointhe
epigastricregionthatworsenswith
eatingspicyfoodandisrelievedby
bendingforward.Complicationsofthe
abovementionedconditioncouldbeall
except

a)Perforation
b)Bleeding
c)GastricOutletObstruction
d)SplenicVeinThrombosis
CorrectAnswer-D
Ans.-D.SplenicVeinThrombosis
Bleeding,perforationandgastricoutletobstructionarethe
complicationsofpepticulcer.
Perforation:Thisallowsstomachcontentstoescapeintothe
peritoneum,causingperitonitis.Itismorecommoninduodenalthan
ingastriculcers.
Gastricoutletobstruction:Themostcommoncauseisanulcernear
thepylorus,butoccasionalcasesareduetoantralcanceroradult
hypertrophicpyloricstenosis.
Bleeding

1338.Whichofthefollowingisatumour
markerforbladdercancer?
a)AFP
b)CEA
c)Bladdersurfaceprotein
d)NuclearMatrixprotein22
CorrectAnswer-D
Answer-D.NuclearMatrixprotein22
TumourmarkersinCABladder
Nuclearmatrixprotein22
BTA
TRAK

1339.RPLNDandChemotherapymaybeused
inmanagementof
a)NonseminomatousGermcelltumoursoftestis
b)Nongermcelltumours
c)SeminomatousGermcelltumours
d)Lymphomatestis
CorrectAnswer-A
Answer-A.NonseminomatousGermcelltumoursoftestis
RPLNDretroperitoneallymphnodedissection
ExtragonadalGermcelltumorsInfrequentlyGCTsarisefroman
extragonadalsite.Theyhavepoorprognosis.Theyaretreatedby
chemotherapy.

1340.Allofthefollowingtesticulartumours
aregermcelltumoursEXCEPT:
March2013(b)

a)Seminoma
b)Teratoma
c)Choriocarcinoma
d)Sertolicelltumour
CorrectAnswer-D
Ans.Di.e.Sertolicelltumour
Testicularcarcinoma
Bilateralin10%ofcases,
Abdominalcryptorchidtestesareathigherriskascomparedto
inguinalcryptorchidtestes
Testicularfeminizationsyndromeincreasestheriskoftesticular
germcelltumour
MCCaoftestesinyoung:Seminoma
MCCainelderly:Lymphoma
MCCaininfants:Yolksactumour
Seminoma
Radiosensitive,
Correspondstodysgerminomaofovary&
Treatmentofchoiceis:Surgery
GermCellTumors
Precursorlesion
Intratubulargermcellneoplasm,unclassified
Intratubulargermcellneoplasm,specifictype
Tumorsof1histologictype

Seminoma
Variant:Seminomawithsyncytiotrophoblasticcells
Partiallyregressedtumorshowingseminomawithscar
SpermatocyticseminomaEmbryonalcarcinoma
Variant:Spermatocyticseminomawithasarcomatouscomponent
Yolksactumor
ChoriocarcinomaPlacentalsitetrophoblastictumor
Variant:"Monophasic"type
Trophoblastictumor,unclassified
Teratoma
Withasecondarysomatictypemalignantcomponent
Monodermalvariants
Carcinoid
Primitiveneuroectodermaltumor
Others

1341.TheGrayhackshuntisestablished
between:
a)Corporacavernosaanddorsalvein
b)Corporacavernosaandsaphenousvein
c)Corporacavernosaandglans
d)Corporacavernosaandcorporaspongiosa
CorrectAnswer-B
Ansis'b'i.e.Corporacavernosaandsaphenousvein
TheGrayhackshuntisasurgicalshuntbetweencorporacavernosa
andthesaphenousveindoneforthetreatmentofischemicpriapism.
Priapismisanuncommonconditionofprolongederection.Itis
usuallypainfulforthepatient,andnosexualexcitementordesireis
present.
Priapismmaybeclassifiedintohigh-andlow-flowtypes
(Nonischemicandischemic).
Nonischemic(Highflow)priapism:
Nonischemicpriapism,alsotermedarterialorhigh-flowpriapism,
featureselevatedvascularflowthroughthecorporacavernosa.
Itusuallyoccurssecondarytoperinealtrauma,whichinjuresthe
centralpenilearteriesandresultsinlossofpenileblood-flow
regulation.
Aspirationofpenilebloodforblood-gasdeterminationdemonstrates
highoxygenandnormalcarbondioxidelevels.
Arteriographyisusefultodemonstrateaneurysmsthatwillrespond
toembolization.
Erectilefunctionisusuallypreserved.
Ischemic(low-flow)priapism:
Ischemicpriapism,alsotermedveno-occlusiveorlow-flowpriapism,

featureslittleorabsentintracorporalbloodflow.Itrepresentsatrue
compartmentsyndromeinvolvingthepenis,needingemergency
management.
Itistypicallypainful.
Thecorporacavernosaistensewithcongestedbloodandtenderto
palpation.Theglanspenisandcorpusspongiosumaresoftand
uninvolvedintheprocess.
Thecurrenttheoriesregardingthemechanismofpriapismremainin
debate,butmostauthoritiesbelievethemajorabnormalitytobe
physiologicobstructionofthevenousdrainage.Thisobstruction
causesbuildupofhighlyviscous,poorlyoxygenatedblood(low02,
highCO2)withinthecorporacavernosa.
Iftheprocesscontinuesforseveraldays,interstitialedemaand
fibrosisofthecorporacavernosawilldevelop,causingimpotence.
Treatment
Ischernicpriapismisaurologicemergency.
-First-linetreatmentconsistsofaspirationofbloodandirrigationof
thecorporacavernosa(viaaneedleputinthecorporacavernosa)
alongwithintracavernousinjectionofana-adrenergic
sympathomimeticagent(phenylephrine).(Sympathomimeticagents
canbeexpectedtoexertcontractileeffectsonthecavernoustissue
andthusfacilitatedetumescence.)
-Surgicalshuntingisneededwhentheintracavernoustreatment
fails.
Asurgicalshunthastheobjectiveoffacilitatingblooddrainagefrom
thecorporacavernosa,bypassingthevenoocclusivemechanismof
thesestructures.Avarietyofshuntproceduresmaybeperformed.A
distalcavernoglanular(corporoglanular)shuntisthefirstchoice.

1342.Whichofthefollowingisfalseabout
undescendedtestis?
a)Morecommonontherightside
b)Hormonaltherapyiseffective
c)Increasedriskofmalignancy
d)Secondarysexualcharacteristicsarenormal
CorrectAnswer-B
Answer-B.Hormonaltherapyiseffective
Inundescendedtestis-thetestesisarrestedinsomepartofitspath
tothescrotum.
InEctopictestis-thetestisisabnormallyplacedoutsideitspath.
Retractiletestis-ininfancy80%ofinapparenttestisareretractile
testisandrequirenot/t.
Approx70-77%ofcryptorchidtesteswillspontaneouslydescend,
usuallyby3monthsofage.
Morecommoninpreterm,smallforgestationalage,LBW&twin
neonates.
MorecommononRt.Side
Secondarysexualcharactersticsarenormal

1343.A45yearoldmaleisdiagnosedwith
carcinomapenis.Thesurgeonmust
lookoutforwhichlymphnodes

a)Paraaortic
b)Externaliliac
c)InternalIliac
d)Inguinal
CorrectAnswer-D
Answer-D.Inguinal
Morethan50%ofpatientspresentwithenlargedinguinallymph
nodes(buthalfofthesearereactiveenlargementd/tsepsis).
Thepresenceandtheextentofmetastasistotheinguinalregionis
themostimportantprognosticfactorforsurvivalinpatientswithCa
Penis.

1344.A45yearoldmalepresentingwith
penilecancerextendinguptotheglans
penisistreatedwith

a)PartialPenectomywith2cmmargin
b)Circumcision
c)PartialpenectomywithInguinalnodesexploration
d)Partialpenectomywith4cmmargin
CorrectAnswer-A
Answer-A.PartialPenectomywith2cmmargin
Thegoaloft/tininvasivepenilecarcinomasiscomplete
excisionwithadequatemargins:
a)Forlesioninvolvingtheprepuce
Simplecircumcisioniseffective.
b)Forlesionofglansordistalshaft
Partialpenectomywitha2cmmargin(lessaggressivesurgical
resectionssuchasMohsmicrographicsurgeryandlocalexcisions
directedatpenilepreservationcanbedone).
c)Forlesioninvolvingtheproximalshaftorwhenpartialpenectomy
resultsinapenilestumpofinsufficientlengthforsexualfunctionor
directingtheurinarystream
Totalpenectomywithperinealurethrostomy

1345.Ayoungmalepresentswithatesticular
massontherightside.TheAFPis
elevatedwhiletheHCGisnormal.The
mostappropriatenextstepis

a)Biopsy
b)USG
c)Orchidectomy
d)WaitandWatch
CorrectAnswer-B
Answer-B
Apainlesstesticularmassispathognomicforatesticular
malignancy.USGofthetestisisindicatedwheneveratesticular
malignancyisconsideredandforpersistentorpainfultesticular
swelling.

1346.A65yearoldmalepresentingwith
acurepancreatitisisnowhaving
refractoryhypoxia.TheXRAYofchest
wouldshow

a)Bilateralinfiltrates
b)Pneumatocoeles
c)Groundglassappearances
d)Hilarlymphadenopathy
CorrectAnswer-A
Answer-A.Bilateralinfiltrates
Systemiccomplicationsofpancreatitis
Hypovolemicshock
DIC
ARDS
Diabetes

1347.Allofthefollowingaretrueabout
NissenFundoplicationexcept
a)ItisdoneforGERD
b)Reinforcmentisdoneonlyintheanteriorhalf
c)Upperpartofstomachisplicatedaroundtheloweresophagus
d)Itisdoneforparaesophagalhiatushernia
CorrectAnswer-B
Answer-B.Reinforcmentisdoneonlyintheanteriorhalf
Inafundoplication,thegastricfundus(upperpart)ofthestomachis
wrapped,orplicated,aroundthelowerendoftheesophagusand
stitchedinplace,reinforcingtheclosingfunctionofthelower
esophagealsphincter.Theesophagealhiatusisalsonarroweddown
bysuturestopreventortreatconcurrenthiatalhernia,inwhichthe
fundusslidesupthroughtheenlargedesophagealhiatusofthe
diaphragm.
InaNissenfundoplication,alsocalledacompletefundoplication,the
fundusiswrappedalltheway360degreesaroundtheesophagus.

1348.Congenitalhydroceleisbestt/tby-
a)Eversionofsac
b)Excisionofsac
c)Lordsprocedure
d)herniotomy
CorrectAnswer-D
Ans.is'd'ie.Herniotomy
Baileywrites?
"Congenitalhydroceleareaspecialformofindirectinguinalhernia
andaretreatedbyherniotomy."
Congenitalhydrocele
Inthisconditontheprocessusvaginalisremainpatentsothereis
directcommunicationofthetunicavaginaliswiththeperitoneal
cavity.Thecommunicatingorificeatthedeepinguinalringistoo
smallforthedevelopmentofhernia.
Itispresentsincebirth*.
Incontradicationtoassumption.congenitalhydroceleisnoteasily
reducible*duetonarrownessofthedeepinguinalringbutwhenthe
childliessupine,itdisappears.

1349.Falseregardinghypernephromais-
a)Radiosensitive
b)Arisefromcortexusuallyfrompreexistingadenoma
c)Maypresentwithrapidlydevelopingvaricocele
d)Usuallyadenocarcinoma
CorrectAnswer-A
Ans.is'a'i.e.,Radiosensitive
RCCorhypernephromaisoneofthemostradioresistantand
chemoresistanttumors.
Inmenrapidlydevelopingvaricoceleisrarebutimpressivesignfor
RCCoccuringmostoftenontheleftside.Itoccursbecauseleft
gonadalveinisobstructedwhereitjoinstheleftrenalvein.
AsalreadystatedRCCoriginatesinthecortexandtendstogrowout
intoperinephrictissuecausingcharacteristicbulgeormasseffect.
Aboutadenomas
Adenomasarethemostcommonbenignrenalparenchymallesions.
-Despitetheclassificationofadenomaasabenigntumor,no
clinical,histologicorimmunohistochemicalcriteriacandifferentiate
renaladenomafromrenalcarcinoma.
Smith'sUrologywritesaboutadenomas?
-"Previously,allrenaltumorslessthan3cmwereconsidered
adenomas.However,evensuchsmalltumorscanmetastasizeand
arenowclassifiedasrenalcellcarcinoma.Adenomasofanysize
shouldbetreatedasafortuitousfindingrepresentativeofanearly
renalcancer,andthepatientshouldbeevaluatedandtreated
appropriately."


1350.TheBipolarcauteryispreferredover
monopolarcauteryinthefollowing
surgeriesexcept

a)HandSurgery
b)SurgeryaroundPenis
c)SurgeryoftheHip
d)Surgeryaroundtheface
CorrectAnswer-C
Answer-C.SurgeryoftheHip
Bipolarcauterypreferredin:
Handsurgery
Surgeryaroundappendageslikepenis
Surgeryaroundtheface

1351.Theposteriorurethraisbestvisualized
by?
a)Staticcystogram
b)Retrogradeurethrogram
c)Voidingcystogram
d)CTcystogram
CorrectAnswer-C
Ans.is'c'i.e.,VoidingCystogram
Voidingcystourethrographyisthebestmethodtovisualizeposterior
urethra.
Remember,
Urethracanbeimagedradiographicallyintwoways.
Anterogradetechniques-->Bestforvisualizationofposterior
urethra.
(Thisisdonealongwithvoidingcystourethrographyorwith
voidingfollowing
excretoryurography)
Retrogradetechnique-->Bestforexaminingthe
anterior(penile)urethra
(Contrastisinjectedthroughtipofurethra).

1352.Earlystageofnonsmallcelllung
cancercanbetreatedby
a)Surgicalresection
b)Surgicalresectionwithadjuvantchemotherapy
c)Radiotherapy
d)Immunotherapy
CorrectAnswer-C
Answer-C.Radiotherapy
Cystoscopyisendoscopyoftheurinarybladderviatheurethra.Itis
carriedoutwithacystoscope.

1353.Allofthefollowingabout
Gastrointestinalcarcinoidtumorsare
true,Except:

a)Smallintestineandappendixaccountforalmost60%ofall
gastrointestinalcarcinoid
b)5yearsurvivalforcarcinoidtumorsis>60%
c)Rectumisspared
d)Appendicialcarcinoidsaremorecommoninfemalesthan
males
CorrectAnswer-C
Ansis'c'i.e.Rectumisspared
Rectumisnotspared,butisinfactacommonsiteforcarinoidtumor.
Aboutotheroptions:
5yearsurvivalforcarcinoidtumorsis>60%
Sabistonwrites-"Carcinoidtumorshavethebestprognosisofall
smallboweltumors,whetherthediseaseislocalizedormetastatic.
Resectionofacarcinoidtumorlocalizedtoitsprimarysite
approachesa100%survivalrate.Five-yearsurvivalratesareabout
65%amongpatientswithregionaldiseaseand25%to35%among
thosewithdistantmetastasis."
Appendicialcarcinoidsaremorecommoninfemalesthanmales
"Appendicialcarcinoidsaremorecommoninfemales.Twotothree
casesofappendecialcarcinoidsarefemale."-Sleisengerand
Fordtran'sGastrointestinalandLiverDisease8/ep609

Smallintestineandappendixaccountforalmost60%ofall
gastrointestinalcarcinoid
Datafromvariousbooksvarybutasfarasthequestiongoesthis

canbetakentobetrueasoption'c'isdefinitelywrong.InGITsmall
intestineandtheappendixarethemostcommonsites.Infactsmall
intestineisthe2ndmostcommonsiteinbodyafterbronchus.
[NotethatseveraltextbooksincludingSchwartzandSabiston,
mentionAppendixasthemostcommonsiteofGIcarcinoids,which
isnottrueaccordingSEERdata(giveninHarrison)]


1354.Commonestinfantiletesticulartumour
is?
a)Seminoma
b)Teratoma
c)Yolksactumor
d)Dysgerminoma
CorrectAnswer-C
Ans.is'c'i.e.,Yolksactumor
oMostcommontumoroftestisSeminoma.
oMostcommontumoroftestisinchildhood--->yolksactumor
(endodermalsinustumororinfantileembryonalcarcinoma).

1355.Allofthefollowingaretrueaboutthe
bareareaoftheliverexcept
a)Infectioncanspreadfromtheabdominaltothoraciccavityat
thisarea
b)Itisnotasiteofportocavalanastomosis
c)Formedbythereflectionsofcoronaryligaments
d)Itistriangularinshape
CorrectAnswer-B
Answer-B.Itisnotasiteofportocavalanastomosis
Thecoronaryligamentsrepresentreflectionsofthevisceral
peritoneumcoveringtheliverontothediaphragm.Assuch,between
thetwolayersofthecoronaryligamentliesthebareareaoftheliver,
andisattachedtothediaphragmbyareolartissue.
ThebareareaoftheliverisstillcoveredbyGlisson'scapsule,the
fibrouscapsulethatsheathestheentireliver.
Bareareaisasiteofportocavalanastomoses

1356.Whichistheinvestigationofchoicefor
stagingofalowerlimbsarcoma?
a)MRI
b)CTScan
c)PETScan
d)PETCT
CorrectAnswer-A
Answer-A.MRI
MRI:Investigationofchoiceforsofttissuesarcomasinextremities.
CTSCAN:Investigationofchoiceforretroperitonealsarcomas.

1357.Whichofthefollowingisanabsorbable
suture
a)Polyglactin
b)Silk
c)Polyester
d)Ethilon
CorrectAnswer-A
Answer-A.Polyglactin
Absorbablesutures-

1. Catgut
2. Polyglactin
3. Polyglyconate
4. Polyglycolicacid
5. Polydioxanone
6. Poliglecaprone

1358.Polydiaxononesutureisnormally
absorbedin
a)2weeks
b)4weeks
c)6weeks
d)6months
CorrectAnswer-D
Answer-D.6months
Retains70%ofitsoriginaltensilestrengthat2weeks,50%at4
weeks,and25%at6weeks.
Prolongeddermalsupportforatleast6monthshasbeenassociated
withdecreasedscarspreading

1359.Mosttissuereactionisseenwith
a)PlainCatgut
b)Polydiaxonone
c)Silk
d)Chromiccatgut
CorrectAnswer-C
Answer-C.Silk
Reactiontocatgutdependsonthestageofabsorptionandismainly
histocyticintype.Nonabsorbablesuturesareencapsulatedbyarim
ofconnectivetissue,whilenearthesuturehistocytes,giantcellsand
lymphocytesarefound.Thisismostmarkedwithsilkandcotton,
lesssowithDacron,andleastwithnylonandwire.

1360.TraumaandInjurySeverityScore
(TRISS)includes:
a)GCS+BP+RR
b)RTS+ISS+Age
c)RTS+ISS+GCS
d)RTS+GCS+Age
CorrectAnswer-B
Ansis`b'i.e.RTS+ISS+Age
TRISS(TraumaandInjurySeverityScore):
InjurySeverityScore
RevisedTraumaScore
Age
MechanismofInjury(blunt/penetrating)

1361.Whichofthefollowingismost
malignanttumor?
a)GlioblastomaMultiforme
b)Meningioma
c)Osteochondroma
d)Giantcelltumor
CorrectAnswer-A
Answer-A.GlioblastomaMultiforme
PrimaryBrainTumours
1.Gliomas(43%).
a.Astrocytomasarethecommonesttype.Theyareusually
malignant.
GradeI?Cystic
GradeII?Diffuse
GradeIII?Anaplastic
GradeIV?Glioblastomamultiforme(MC)
2.Meningiomas(18%)
3.Schwannoma(8%)
4.Pituitarytumors(12%)
5.Craniopharyngiomas(5%)
6.Bloodvesseltumors(2%)
Othertumors
Theyarepinealregiontumors,pituitaryadenomas,
craniopharyngiomas,choroidplexustumors,etc.

1362.Allofthefollowingaremedicalusesof
erythropoietinexcept-
a)Treatmentofanaemiaassociatedwithrenaldisease
b)Chemotherapyinducedanemia
c)AnaemiaAssociatedwithCrohn'sDisease
d)MegaloblasticAnaemis
CorrectAnswer-D
Answer-D.MegaloblasticAnaemis
Usesoferythropoetin
Anaemiaofrenalfailure.
Anaemiaassociatedwithchemotherapyfordiseaseslike
mylodysplasia.
Anaemiaassociatedwithinflammatoryboweldisease.

1363.Epulisarisesfrom-
a)Enamel
b)Rootofteeth
c)Gingiva
d)Pulp
CorrectAnswer-C
Answeris'c'i.e.Gingiva
Epulisliterallymeans'uponthegum'.Thusitisaswellingsituated
onthegum.
Itcanoriginatefromthemucousmembrane,theperiosteumorthe
bonegivingrisetodifferentvarietiesofEpulis.

1364.

Traumaticopticneuropathyduetoclosed
headtraumacommonlyaffectswhichpartof
opticnerve-

a)Opticcanal
b)Intraocularpart
c)Intracranialpart
d)Optictract
CorrectAnswer-A
Answer-A.Opticcanal
Indirectopticnerveinjury
Studieshaveshownthatforcesappliedtothefrontalboneand
malareminencesaretransferredandconcentratedintheareanear
theopticcanal.Thetightadherenceoftheopticnerve'sduralsheath
totheperiosteumwithintheopticcanalisalsothoughttocontribute
tothissegmentofthenervebeingextremelysusceptibletothe
deformativestressesoftheskullbones.Suchinjuryleadsto
ischemicinjurytotheretinalganglioncellswithintheopticcanal.

1365.IQinmildmentalretardationis
a)50-70
b)35-49
c)20-34
d)<20
CorrectAnswer-A
Ans.is'a'i.e.,50-70

1366.WhatistheIQofaborderline
deficiency?
a)70-80
b)50-69
c)20-49
d)0-20
CorrectAnswer-A
Answer-A.70-80
IQRange
IQClassification
70andBelow ExtremelyLow
71-79
Borderline
80-89
LowAverage
90-110
Average
111-120
Bright
121-130
VeryBright
131andOver ExtremelyBright

1367.AccordingtoWechslerintelligence
scalescoring,averageIQofanormal
childis:

a)50
b)75
c)90
d)111
CorrectAnswer-C
Ans.c.90

1368.Whichisfalseaboutdevelopment
milestonesat6monthsofage?
a)Watchingselfinmirror
b)Sittingintripodposition
c)Pincergrasp
d)Monosyllablesounds
CorrectAnswer-A
Answer-A.Watchingselfinmirror
Milestonesat6monthsofage
Inpronepositionliftshisheadandgreaterpartofhischestwhile
supportingweightonextendedarms.
Producesmonosyllablesoundslikeda,ma.
Enjoyswatchinghisownimageinthemirror.
Binocularvisiondevelops(between3-6months).
Purposefulmovementsinspace(6-8months).
Sitsintripodposition.

1369.Atwhatagechildbeginstousepast
andpresenttense
a)1Years
b)2Years
c)18Months
d)30Months
CorrectAnswer-D
Answer-D.30Months
Begintoidentifyobjectsfromagroupbytheirfunctionandparts(ie.
"whichonehaswheels","whichonecanweeat")
Begintouseverbswith"ing"endings(i.e."eating");
Earlyconceptssuchas"big,little"areidentified;
Childwilluse"no,not"andanswer"where"questions

1370.Milestonesat1yearofageareall
except
a)Playingasimpleballgame
b)Using2wordsthataremeaningful
c)Spontaneousscribbling
d)Walkingupstairs1stepatatime
CorrectAnswer-D
Answer-D.Walkingupstairs1stepatatime
Triestoremovehiscoatandattemptstowearhissocksorshoes
withoutsuccess.
Doesmimicry.
Playsasimpleballgame
Canuse2wordswithmeaning
TriestobuildaTowerof2cubes
Triestoscribblespontaneously(between12-24months).

1371.Strangeranxietydevelopsat
a)3months
b)4months
c)7months
d)11months
CorrectAnswer-C
Answer-C.7months
Milestonesatmonth
Holdstheobjectswithcrudegraspfrompalm(palmargrasp)
Pivots
Showsstrangersanxiety
Resistsifatoyispulledfromhishand.
Babbles

1372.Whatistheaverageweightgainofthe
neonateperday
a)5-10g
b)25-30g
c)50-60g
d)100-150g
CorrectAnswer-B
Answer-B.25-30g
Theygainweightatarateofapproximately25to30gmperdayfor
thefirst3monthsoflife.Thereaftertheygainabout400gmof
weighteverymonthfortheremainingpartoffirstyear.
[RefGhai7th/ep.6]

1373.Newbornloseshowmuchweightinfirst
week?
a)5-10%
b)1-2%
c)10-20%
d)None
CorrectAnswer-A
Answer-A.5-10%
Theaveragebirthweightofneonatesisabout3Kg.
Duringfirstfewdaysafterbirth,thenewbornlosesextracellularfluid
equivalenttoabout10%ofthebodyweight.

1374.Anewbornbabyhasahead
circumferenceof35ems.atbirth,His
optimalheadcircumferencewillbe43
emsat-

a)4monthsofage
b)6monthsofage
c)8monthsofage
d)12monthsofage
CorrectAnswer-B
Ans.is'b'i.e.,6monthsofage
oAt6monthofageheadcircumferenceisbetween40.0-43.5cm.

1375."Potter'ssyndrome"isassociatedwith-
a)Renalanomalies
b)Severeoligohydramnio's
c)Flattenednose
d)Alltheabove
CorrectAnswer-D
Ans.is'd'i.e.,Alloftheabove
Pottersyndrome
Pottersyndromeischaracterizedby-
i)Bilateralrenalagenesis
ii)Pulmonaryhypoplasia
iii)Potterfacies-->
widelyseparatedeyeswithepicanthicfolds,
lowsetears,broadcompressedflatnose,recedingchin.
*Thisconditionisincompatiblewithlife,deathoccursshortlyafter
birthfrompulmonaryhypoplasia.
*MaternalUSGdemonstrates-
i)Oligohydramniosii)Nonvisulizationof
bladderiii)Absentkidney

1376.Whatisthecardiothoracicratioin
childrenis-
a)30-35%
b)40-45%
c)50-55%
d)60-65%
CorrectAnswer-C
Answer-C.50-55%
"Thecardiacsilhouetteoccupies50-55%ofthechestwidth.
Cardiomegalyispresentwhenthecardiothoracic(CT)ratioismore
than55%."

1377.Thefetalcirculationchangestonormal
circulationatbirthwith
a)Closureofpatentductusarteriosus
b)Closureofductusvenosus
c)IncreasedactivityofrightVentricle
d)OpeningoffossaOvalis
CorrectAnswer-A
Answer-A.Closureofpatentductusarteriosus
Theincreaseintheconcentrationofoxygeninthebloodleadstoa
decreaseinprostaglandins,causingclosureoftheductusarteriosus.
Theseclosurespreventbloodfrombypassingpulmonarycirculation,
andthereforeallowtheneonate'sbloodtobecomeoxygenatedin
thenewlyoperationallungs.

1378.Mostcommoncauseofpneumoniain
earlyonsetsepsisaneonateis
a)Pnemococcus
b)S.Pyogens
c)EColi
d)S.Aureus
CorrectAnswer-C
Answer-C.EColi
Earlyonsetsepsis
Itiscausedbyorganismsprevalentinthegenitaltractorinthelabor
roomandmaternityoperationtheatre.
InthewestitismostlycausedbygroupBstreptococcusandE.coli.
Inourcountryitismostlyduetogramnegativeorganisms-E.coli,
ldebsiellaandenterobactorsp.

1379.Neonatalresuscitation-whichofthe
followingdrugsisused?
a)Dopamine
b)SodiumBicarbonate
c)Noradrenaline
d)Dobutamine
CorrectAnswer-B
Answer-B.SodiumBicarbonate
Importantdrugsusedforneonatalresuscitationare
Epinephrine(adrenaline),
Normalsalineorringerlactate,
Naloxoneand
Sodabicarbonate.

1380.Whendoescryingstopincyanotic
spells?
a)ForcedExpiration
b)Forcedinspiration
c)Midinspiration
d)Cryingiscontinuous
CorrectAnswer-A
Answer-A.ForcedExpiration
Cyanoticform(cyanoticspells)
Thisismorecommonandisprovokedinresponsetofrustrationand
angerprecipitatedbyupsettingorscoldinginfant/child.
Cyanoticspellsareduetocentralsympatheticoveractivity.
Clinicalfeaturesincludegeneralizedcyanosis,apnea,forced
expiration(cryingstops),opisthotonus,shrillcryandbradycardia.
Seizuresmayoccurduetocerebralhypoxia,butantiepilepticsare
notrequired.
Theonlytreatmentissupportandreassurancetoparents

1381.Treatmentofchoiceforsymptomatic
neonatalhypoglycemiais
a)Dextrosenormalsaline
b)5%dextrose
c)10%dextrose
d)25%dextrose
CorrectAnswer-C
Answer-C.10%dextrose
Symptomaticorasymptomaticwithbloodglucose<20mg/dl
Bolus10%dextrose2ml/kgisgivenIV.Followedbycontinuous
infusionof6mg/kg/minute.Ifnormoglycemiaisnotachievedwithin
24hours,glucocorticoids(prednisoneorhydrocortisone)shouldbe
administered.Forintractablehypoglycemia,glucagon,epinephrine
ordiazoxidecanbegiven.
Inhypoglycemicseizures,doseof10%dextroseis4ml/kg

1382.ALLofthefollowingarecausesof
neonatalbradycardiaexcept
a)Hypoxia
b)Hypothermia
c)Headinjury
d)BCGVaccine
CorrectAnswer-D
Answer-D.BCGVaccine
Neonatalbradycardiaisdefinedasadecreaseinheartby30bpm
frombaseline.Regardingneonatalresuscitation,bradycardiais
concerningwhentheheartrateislessthan100bpm.

1383.Whichofthefollowingisnotacauseof
neonatalanaemia?
a)SubgalealHemorrhage
b)Abruptioplacentae
c)DiamondBlackfansyndeome
d)Wilson'sDisease
CorrectAnswer-D
Answer-D.Wilson'sDisease
Internalhemorrhagesuchasintracranialhemorrhage,subgaleal
hemorrhage,cephalohematoma,adrenalhemorrhage,subcapsular
hematomaofliverorrupturedviscus
Obstetricalcauses:placentalabruption,placentaprevia,traumato
placentaorumbilicalcordduringdeliveryandruptureofanomalous
placentalvessels
Twin-twintransfusion
RBCdestruction
RBCproduction

1384.3montholdchildwithindrawingchest
withrespitratoryrate52/minclassified
as

a)SIRS
b)Respiratorydistress
c)Tachypnoea
d)ARDS
CorrectAnswer-B
Answer-B.Respiratorydistress
Tachypnea(fastbreathing):Fastbreathingisdefinedas:-

1. Lessthan2monthsofage->60breathsperminute
2. Childaged2monthsupto12months-50breathsperminute
3. Childaged12monthsupto5years->40breathsperminute

1385.45dayoldinfantpresentswith
seizures.Examinationrevealsheis
icteric,havingbulgingfontanellesand
opisthotonicposture.Treatmentisall
except

a)Phototherapy
b)ExchangeTransfusion
c)Phenobarbitone
d)Chlorpromazine
CorrectAnswer-D
Answer-D.Chlorpromazine
Chlorpromazineisnotusedinhyperbilirubinemia.
Treatmentofhyperbilirubinemiaincludes
1. Pharmacologicaltharapy:Barbiturates(phenobarbitone),
metallloporphyrins(Tin/SnandZinc/Zn)
2. Phototherapy
3. Exchangetronsfusion

1386.Chroniclungdiseaseinainfancyis
definedas
a)Needforsupplementaloxygenat36weeksafterconception
b)Tachypnoea>50breaths/minwithin1weekofbirth
c)PresenceofbilateralinfiltratesonchestXrayfor2weeks
d)ReticulogranularpatternonchestXrayfor6weeks
CorrectAnswer-A
Answer-A.Needforsupplementaloxygenat36weeksafter
conception
Chroniclungdiseaseofinfancywasformerlycalled
bronchopulmonarydysplasia.
Bronchopulmonarydysplasiaisusuallydefinedasaneedfor
supplementaloxygenat36weeksafterconception.
Bronchopulmonarydysplasiaisusuallydefinedasaneedfor
supplementaloxygenat36weeksafterconception.BPDisusually
definedasaneedforsupplementaloxygenat36wkafter
conception.
BPDisaresultoflunginjuryininfantsrequiringmechanical
ventilationandsupplementaloxygen.

1387.Mostcommonantigeninvolvedin
erythroblastosisfetalisis
a)CantigeninRhgroup
b)DantigeninRhgroup
c)EantigeninRhgroup
d)Duffyantigen
CorrectAnswer-B
Answer-B.DantigeninRhgroup
RBCantigensarecapableofelicitinganantibodyresponse,
significantdiseaseisassociatedprimarilyDantigenofRhgroupand
withABOincompatibility

1388.Eryhtematousblotchyrashisseenon
theabdomen,trunkandfaceofa3day
oldchildalongwithyellowishpapules.
Howeverthechildfeelswell.Whatis
themanagement?

a)Steroidandantibioticlotion
b)Notreatment
c)Steroidcream
d)Urgentintravenousantibiotics
CorrectAnswer-B
Answer-B.Notreatment
Erythematoxicumneonatorumisabenignself-limitederuption
occurringprimarilyinhealthynewbornsintheearlyneonatalperiod.
ItischaracterizedbyErythematouspapulesontrunk&face.They
appearon2nd&3rddayanddisappearspontaneously.

1389.Furtherinvestigationisessentialina
newbornwithwhichcondition?
a)Erythematoxicum
b)Vaginalbleed
c)Subconjunctivalhemorrhage
d)Lensopacity
CorrectAnswer-D
Answer-D.Lensopacity
Theproblemsare

1. Milia
2. Erythematoxicum
3. Storkbites
4. Peelingofskin
5. Subconjuctivalhemorrhages
6. Breastengorgment
7. Epsteinpearl
8. Pre-deciduous(natalteeth)
9. Vaginalbleeding
10. Vaginalmucoiddischarge
11. Hymenaltags
12. Physiologicalphymosis
13. Mongolianspots

1390.Anewbornpresentswith
subconjunctivalhemorrhage.The
treatmentis

a)Notreatment
b)Antibioticeyedrops
c)Aspitation
d)Antibioticandsteroiddrops
CorrectAnswer-A
Answer-A.Notreatment
Subconjunctivalhemorrhageinnewbornisanormalphenomenom
whichdisappearsspontaneously.

1391.Whatistheshapeofcaecuminthe
newborn?
a)Ovoid
b)Trapezoid
c)Globular
d)Conical
CorrectAnswer-D
Answer-D.Conical
Theshapeofthecaecuminaninfantisconicalwiththeappendix
borneatthebaseofthecone.

1392.MostcommoncomplicationofMeckel's
Diverticulumlitchildren
a)Abdominalpain
b)Pepticulcers
c)Intestinalobstruction
d)PainlessRectalbleeding
CorrectAnswer-D
Answer-D.PainlessRectalbleeding
MostcomnmonpresentationofMeckel'sDiverticuluminchildrenis
painlessrectalBleeding.
[RefAlemayehuH,HallM,DesaiAA,StPeterSD,SnyderCL.
Demographicdisparitiesofchildrenpresentingwithsymptomatic
Meckel'sdiverticuluminchildren'shospitals.PediatricSurgery
International.2014Jun30.6:649-653]

1393.WhichofthefollowingisaXlinked
metabolicdisorder?
a)Fabry'sdisease
b)Sandoffsdisease
c)Pompedisease
d)Gaucherdisease
CorrectAnswer-A
Answer-A.Fabry'sdisease
Alllysosomaldisordersare`autosomalrecessive'exceptfor
Hunter'ssyndromeandFabry'sdisease,whichareX-linked
recessive.ThusHunter'ssyndromeandFabry'sdiseaseaffectonly
male.

1394.Testesarenotpalpablein
a)SRYdeletion
b)DAX1deletion
c)WNT-4genemutation
d)RSPO-1genemutation
CorrectAnswer-A
Answer-A.SRYdeletion
SRYgeneisinvolvedindevelopmentofmalegonds(testes)from
primitive(bipotentialgonads).
DAX-1,WNT-4andRSPO1genesareinvolvedindevelopmentof
femalegonads(ovary).

1395.Childwith10episodesofdiarrheain
last24hourswithsunkendryeyes,
veryslowskinpinch,andabsenttears.
Managementis

a)ORSsolution
b)breastfeeding
c)Start10%dextrose
d)StartRinger'slactate
CorrectAnswer-D
Answer-D.StartRinger'slactate
Thisisacaseofseveredehydration.
SevereDehydration
StartIVfluidsimmediately
BestIVfluidsolutionisRingerlactate
Normalsalinecanbeused
Dextroseisnoteffective
100ml/kgistobegivenasshownbelow:
AGE
FIRST
THEN
<12months
30nil/kginIhours 70ml/kg5hours
12monthsto5yrs. 30minutes
21/2hrs

1396.1yearoldchildwithmultipleepisodes
ofdiarrheapresentswithsunkendry
eyes,depressedfontanelles,veryslow
skinpinch.Theamountoffluidtobe
giveninthefirst6hoursis

a)600ml
b)900ml
c)1200ml
d)1500ml
CorrectAnswer-B
Answer-B.900ml
Theapproximateweightofa1yearoldchildis9kg(iethricethe
birthweight)
Thedescriptionofdehydrationgivenaboveisconsistentwithsevere
dehydration.T

1397.Whatisthefirstlinetreatmentofa4
yearoldchildpresentingwith
intussusception?

a)Conservativemanagementwithwaitandwatchpolicy
b)Immediateattempttoreductionusingbariumedema
c)Surgicalcorrection
d)Exploratorylaparotomywithresectionoftheaffectedsegment
CorrectAnswer-B
Answer-B.Immediateattempttoreductionusingbariumedema
Correctionofintussusceptionbybariumenemaistheinitial
managementofchoice.Ifitfails,surgicalcorrectionisdone.

1398.Hutchison'sTriadisseenin
a)CongenitalSyphilis
b)Tertiarysyphilis
c)SecondarySyphilis
d)Primarysyphilis
CorrectAnswer-A
Answer-A.CongenitalSyphilis
Hutchinson'striadisacommonpatternofpresentationofCongenital
syphilis.
Itconsistsofthreephenomena:
1. Interstitialkeratitis,
2. Hutchinsonincisor,
3. Eighthnervedeafness

1399.Hydrocephalusisbestdetected
antenatelyby:
a)X-rayabdomen
b)Amniocentesis
c)Clinicalexamination
d)Ultrasonography
CorrectAnswer-D
Ans.isdi.e.Ultrasonography
Hydrocephalusisaconditioninwhichthereisanabnormalincrease
incerebrospinalfluidwithintheventricularandsubarachnoidspaces
ofbrain.
"Theprenataldiagnosisofhydrocephalusisusuallymadeby
demonstrationofadilatedventricularsysteminanultrasound
examination."
Earliestandmostaccuratesonographicsignofhydrocephalus?
Enlargedlateralventricles.?
Thelateralventricleismeasuredatthelevelofatrium.
Normaltransversediameterofatriumis7mm?1mm.(Itremains
constantduringthesecondandthirdtrimester)
Whendiameterofatriumis>10mmitiscalledasVentriculomegaly/
Hydrocephalus.
OthersignsofhydrocephalusonUSG:
Danglingchoroidplexuses.
Thinningoutofcerebralcortex.
ExtraEdge:
Friends,thetermsventriculomegalyandhydrocephalusareoften
usedinterchangeablybuthaveslightlydifferentmeanings.
*Ventriculomegaly:Theconditioninwhichlateralventriclesofthe

brainarefilledwithexcessivefluidandenlarge.
*Hydrocephalus:Thereisventriculomegalyalongwithanincrease
intheheadcircumference.
Normalfetalheadcircumferenceattermrangesbetween32and38
cms.
Withhydrocephalus,thecircumferenceexceeds50cms.
Ventriculomegalycanalsobecausedby?:
*Spinabifida?
*Chromosomalabnormalities?
*Congenitalinfectionslikecytomegalovirus.toxoplasmosis,syphilis
andinfluenza.?
DandywalkersyndromeincludesHydrocephalus+Posteriorfossa
cyst?+Defectincerebellarvermis.

1400.Triadofnormalpressure
hydrocephalusincludesallexcept-
a)Dementia
b)Gaitdisturbance
c)Urinaryincontinence
d)Browache
CorrectAnswer-D
Answer-D.Browache
Triadofnormalpressurehydrocephalus/Adam'striad/Hakim's
triad
Dementia
Gaitdisturbance
Urinaryincontinece(wet,wackyandwobbly)

1401.Medulloblatomaarisesexclusivelyfrom
thecellsof
a)Immatureembryonalcells
b)Ependymalcells
c)Neurons
d)Spindleshapedcells
CorrectAnswer-A
Answer-A.Immatureembryonalcells
MedulloblastomaisthemostcommonPNET(primitive
neuroectodermaltumor)locatedinposteriorcranialfossa.
[RefHinz,Chris;Hesser,Deneen.FocusingOnBrainTumors:
Medulloblastoma.AmericanBrainTumorAssociation]

1402.HypoxicIschemicencephalopathytrue
is?
a)Lowerlimbsaffectedmorethanupperlimbs
b)Prox.Muscles>distalmuscles
c)Seizure
d)Trunkinvolved
CorrectAnswer-C
Ans.is'c'i.e.,Seizure
Clinicalfeaturesofhvpoxicischemicencephalopath'
oEncephalopathyprogressovertime?
1)Birthto12hours-->
Decreasedlevelofconciousness,poor
tone,decreasedspontaneousmovement,periodicbreathingor
apnea,seizures.
2)12-24hours
-4Moreseizuers,Apneicspells,jitteriness,
weakness.
3)After24hours-->Hypotonia,conciousness,poorfeeding,
brainstemsigns(oculomotor)andpupillarydisturbances.

1403.Whichofthefollowingisnottrueabout
encephalocoele?
a)Itisaneuraltubedefect
b)Commoninthefrontalregion
c)Canbeassociatedwithhydrocephalus
d)Itisprotrusionofneuraltissuethroughadefect
CorrectAnswer-A
Answer-A.Itisaneuraltubedefect
Perinatalasphyxia,moreappropriatelyknownashypoxic-ischemic
encephalopathy(HIE),ischaracterizedbyclinicalandlaboratory
evidenceofacuteorsubacutebraininjuryduetoasphyxia.The
primarycausesofthisconditionaresystemichypoxemiaand/or
reducedcerebralbloodflow(CBF)

1404.Trueaboutcephalhematomais-
a)Itishemorrhagebetweentheskullandperiosteum
b)Itishemorrhagewithinthesubcutaneoustissuearoundthe
skull
c)Itistypeofsubduralhemorrhage
d)Itisextraperiostealbleedingintheskull
CorrectAnswer-A
Amswer-A.Itishemorrhagebetweentheskullandperiosteum
Cephalhematomaissubperiostealbleeding,i.e.betweenskullbone
andperiosteum.

1405.A8yearoldchildpresentswithamass
inthelumbarregionwithabdominal
painwithexcruciatingbonepain.
Possiblediagnosisis-

a)Neuroblastoma
b)Wilm'sTumour
c)Lymphoma
d)Angiomyolipoma
CorrectAnswer-A
Amswer-A.Neuroblastoma
Thisisacaseofneuroblastomathathasmetastatized
Metastasisispresentin60-70%atthetimeofdiagnosis.
Commonestsiteofmetastasisisskeletalsystemandneuroblastoma
isthemostcommonchildhoodmalignancymetastasizestobone.

1406.A6yearoldchildpresentswithan
abdominalmass,fever,bonepainand
IVCthrombosis,thediagnosiscouldbe
-

a)Wilm'stumour
b)Neuroblastoma
c)LangerhanscellHistiocytosis
d)Gastriclymphoma
CorrectAnswer-B
Amswer-B.Neuroblastoma
Renalveininvasionismorecharacteristicofneuroblastoma(itis
rareinwilmstumor).

1407.Kwashiorkor-Triadincludesallexcept-
a)Psychomotorchanges
b)Hypoglycemia
c)Edema
d)Growthretardation
CorrectAnswer-B
Answer-B.Hypoglycemia
Classicaltriadofkwashiorkorismarkedlyretardedgrowth,
psychomotor(mental)changesandedema.

1408.Whatisthemaintainancefluid
requirementina6kgchild?
a)240ml/day
b)600ml/day
c)300ml/day
d)1200ml/day
CorrectAnswer-B
Answer-B.600ml/day
FluidRequirementsin:InfantsandChildren

HOLLIDAY-SEGARMETHOD
100ml/kg
First10kgl
Na+3
(4ml/kg/hr)
1000ml+50ml/kgforeachKg>10kg
10-20kg
K+2
(40ml/hr+2ml/kg/hr(wt-10kg)
1500ml+20ml/kgforeachKg>20kg
>20kg
Cl-2
(60ml/hr+1ml/kg/hr(wt-20kg)

1409.Mostcommonsiteforopeningof
TAPVCis-
a)Supracardiac
b)Cardiac
c)Infracardiac
d)Multiple
CorrectAnswer-A
Answer-A.Supracardiac
TypeI(SupraCardiac)TAPVCMostcommon45%
TypeII(Cardiaclevel)TAPVC-25%
TypeIII(InfraCardiac)TAPVC-25%
TypeIV(Multiplelevel)TAPVC45%

1410.Whichoneofthefollowingstatementsisfalseabout
Xanthogranulomatouspyelonephritisinchildren?
a)Oftenaffectsthoseyoungerthan8yearsofage
b)Itaffectsthekidneyfocallymorefrequentlythandiffusely
c)Clinicalpresentationinchildrenissameasinadults
d)Boysareaffectedmorefrequently
CorrectAnswer-D
Itismostfrequentlyaffectedinfemalescomparedtomales.Itisan
unusualformofchronicpyelonephritischaracterizedby
granulomatousabscessformation,severekidneydestruction,anda
clinicalpicturethatmayresemblerenalcellcarcinomaandother
inflammatoryrenalparenchymaldiseases.
Xanthogranulomatouspyelonephritisisaformofchronic
pyelonephritischaracterisedbydestructionofrenalparenchymaand
thepresenceofgranulomas,abscessesandcollectionoflipidladen
foamymacrophages(foamcells).
Xanthogranulomatouspvelonephritisinchildren

*Ageofpresentationrangesfrominfancyto16years.
*FocalfirmbeingmorecommoninchildrenAppearhealthy.
*Thosewhoaffecteddiffusely,presentwithnon-specific
symptoms
ofchronicinfection.
i)Weightlossii)Feveriii)Lethargyiv)
Failuretothrive


*Proteusisthemostcommoncausativeorganism.
Ref:ByJamesPattison,DavidGoldsmith,BarrieHartley,Fernando
C.FervenzaandJosephP.Grande(2004),Chapter6,"Renal
InfectionsandStructuralAbnormalities",Inthebook,"AColour
HandbookofRenalMedicine",UK,Page100.

1411.AutosomalrecessivePolycystic
kidneys-allaretrueexcept-
a)Seeninadults
b)DefectivegeneisPKHD1
c)Bothkidneysshowinnumerablecysts
d)USGshowssaltandpepperappearance
CorrectAnswer-A
Amswer-A.Seeninadults
Childhoodpolycystickidneydiseasehasautosomalrecessive
inheritance,thereforeitisalsoknownasautosomalrecessive
polycystickidneydisease.
DefectivegeneisthePKHD1(PolycysticKidneyandHepatic
Disease1)whichcodesforaproteinfibrocystin
Bothkidneysaremarkedlyenlargedandshowinnumerablecysts
radiatingfrommedullathecortex.
MRIofkidneyshowsradiallyarrangedfusiformdilatedcollecting
ducts.
PrenatalUSGshowsasaltandpepperappearanceofkidney.

1412.Investigationofchoicefor
CONFIRMINGHenochSchonlein
purpurais-

a)SerumIgAlevels
b)CRPlevels
c)RenalBiopsy
d)DTPA
CorrectAnswer-C
Amswer=C.RenalBiopsy
Biopsyofthekidneymaybeperformedbothtoestablishthe
diagnosisortoassesstheseverityofalreadysuspectedkidney
disease.

1413.AbdominalpaininHenochSchonlein
purpuraisdueto-
a)MucosalerosionsandswellingoftheGImucosa
b)Gastrointestinalhemorrhage
c)Volvulus
d)Associatedpancreaticinflammation
CorrectAnswer-A
Answer-A.MucosalerosionsandswellingoftheGImucosa
ThesecondmostfrequentsymptomofHenoch-SchOnleinpurpura
isabdominalpain,whichoccursinupto65percentofcases.The
mostcommoncomplaintiscolickyabdominalpain,whichmaybe
severeandassociatedwithvomiting.
Endoscopicevaluationoftenshowsmucosalerosionsandswelling.

1414.WhichofthefollowingisnotafeatureofJuvenileIdiopathicArthritis?
a)Rheumatoidnodules
b)Spikesofhighfever
c)Uveitis
d)Raynaud'sphenomenon
CorrectAnswer-D
Raynaud'sPhenomenonisnotmentionedinassociationwithJuvenileIdiopathicArthritis
(JIA).
Ref:CurrentDiagnosisandTreatmentinRheumatology,2ndEdition,Pages196-197;
Nelson'sTextbookofPediatrics,18thEdition,Page1003;PrimeronTheRheumatic
DiseasesByJohnH.Klippel,Page145

1415.Whichofthefollowingiscorrectabout
shockinchild?
a)Tachycardiaisaverysensitiveindicatorofdepletionof
intravascularvolume
b)Mottlingofextremitiesisseeninearlyshock
c)Confusion,stuporandcomaareearlysigns
d)Respiratoryrateismoresensitivethanheartrateasan
indicatorofearlyshock
CorrectAnswer-A
Answer-A.Tachycardiaisaverysensitiveindicatorof
depletionofintravascularvolume
Hypovolemicshockinchildrenmayhavefollowingstages:-
i)Earlycompensated
Immediatelyafterhypovolemia,bodytriestomaintaintheBPto
maintainadequateperfusiontovitalorgansthroughacompensatory
mechanisms.
Anincreaseinheartrate(Tachycardia)istheearliestandmost
sensitiveindicatorforintravascularvolumereduction
ii)Lateuncompensated
Ifshockstatecontinuesorthecompensatorymechanismsarenot
enoughtomaintainthemetabolicneedsofthetissue,theshock,
goesintouncompensatedphase.

1416.Inotropicsupportforseverely
dehydratedchildwithdopamineisdone
atwhatrate-

a)0.1-0-5microgram/kg/min
b)1-5microgram/kg/min
c)1-5mg/kg/min
d)10-15mg/kg/min
CorrectAnswer-B
Answer-B.1-5microgram/kg/min
Inhighdoses,itactsonalpha-adrenergicreceptorstoincrease
systemicvascularresistanceandraisebloodpressure.
1-5mcg/kg/minIV,increasedto5-20mcg/kg/min;nottoexceed50
mcg/kg/min.

1417.Testdonetodiagnosesyphilisin
newbornifmotherissyphilitic-
a)SyphilisCapitaMtest
b)DetectionofIgG
c)ZNstaining
d)Fluoroescentantigentest
CorrectAnswer-A
Answer-A.SyphilisCapitaMtest
TheteststodetectIgMare-

1. FTA-ABS(19SIgMFTA-ABS).
2. SyphilisCapitaMtest.

1418.Eyelidpapulesandhoarsecryinachild
issuggestiveof-
a)Congenitalsyphilis
b)Croup
c)Lipoidproteinosis
d)Acrodermatitisenterohepatica
CorrectAnswer-C
Answer-C.Lipoidproteinosis
Lipoidproteinosisalsoknownashyalinosiscutisetmucosaor
Urbach-Weithediseaseisarareautosomalrecessivedisorder.
characterizedclinicallybyamyriadsignsandsymptomsthatinclude
hoarsenessofthevoice,beadedeyelidpapules(Moniliform
blepharosis),yellowish-whitemucocutaneousinfiltrates,andatrophic
pock-likescars.

1419.Inacutediarrheafollowingisusedto
decreasedurationandseverity-
a)Zn
b)Mg
c)Fe
d)Ca
CorrectAnswer-A
Answer-A.Zn
Recentstudiessuggestthatadministrationofzincalongwithnew
lowosmolarityoralrehydrationsolutions/salts(ORS),canreduce
thedurationandseverityofdiarrhealepisodesforuptothree
months.
[RefSachdevHP,MittalNK,YadavHS.Oralzincsupplementation
inpersistentdiarrheaininfants.AnnTropPaediatr.1990;10:63-9.]

1420.Japaneseencephalitisvaccinein
routinescheduleisgiveninhowmany
doses-

a)Twodoses1monthapartwithaboosterafter1-2yearsif
needed
b)Singledosevaccine
c)Threedoses1monthapartfollowedbyaboosterifneeded
d)Threedoseswiththeseconddose1monthand3'dose6
monthsafterthefirstdose
CorrectAnswer-A
Answer-A.Twodoses1monthapartwithaboosterafter1-2
yearsifneeded
AVerocell-derived,inactivatedandalum-adjuvantedJEvaccine
basedontheSA14-14-2strainisused.Theprimarytwodosesare
administered4weeksapart.Aboosterdoseisrecommended1-2
yearsaftertheprimaryimmunization

1421.Consanguinousmarriagesincreaserisk
ofdiseases-
a)Autosomaldominantdisease
b)Autosomalrecessivedisease
c)Xlinkeddominantdiseases
d)Environmentaldiseases
CorrectAnswer-B
Answer-B.Autosomalrecessivedisease
Increasesriskof-autosomalrecessivedisease
Nochangeinriskof-autosomaldominant,Xlinkedrecessive(if
neitherparentaffected)
NotProven-complexlateonsetdiseaseslikediabetes,
schizophrenia,cardiovasculardiseases

1422.Floorofnasalcavityinchildrenismade
of-
a)Palatineboneandvomer
b)Sphenoidandethmoid
c)Nasalboneandmaxilla
d)Palatineboneandmaxilla
CorrectAnswer-D
Answer-D.Palatineboneandmaxilla
"Thefloorofthenasalcavities,whichalsoformtheroofofthe
mouth,ismadeupbythebonesofthehardpalate:thehorizontal
plateofthepalatineboneposteriorlyandthepalatineprocessofthe
maxillaanteriorly."
[RefMoore,KeithL;Dailey,ArthurF.(1999).ClinicallyOriented
Anatomy.LippincottWilliamserWilkins.]

1423.Mostcommonsiteforbonemarrow
aspirationinneonatesis-
a)Anteriorsuperioriliaccrest
b)Posteriorsuperioriliaccrest
c)Sternum
d)Anteromedialtibia
CorrectAnswer-D
Answer-D.Anteromedialtibia
Preferredsiteforbonemarrowaspirationinchildren-Posterior
superioriliaccrest.
Inchildren<18monthofageAnteromedialtibiaispreferred.

1424.ReducedosmolarityORSdoesnot
containwhichofthefollowingion-
a)Sodium
b)Potassium
c)Lactate
d)Citrate
CorrectAnswer-C
Answer-C.Lactate
Table1:CompositionofstandardandreducedosmolarityORS
solutions
ORS
Standard ReducedOsmolarity
Contents mEq/L
mEq/L
Glucose
111
75
Sodium
90
75
Choride
80
65
Potassium 20
20
Citrate
10
10
Osmoarity 311
245
30mmol/lofbicarbonateinsteadof10mmol/lofcitrate

1425.WhichisfoundinDiGeorge'ssyndrome
-
a)Tetany
b)Eczema
c)TotalabsenceofTcells
d)AbsentBandTcells
CorrectAnswer-A
Ans.is'a'i.e.,Tetany
Digeorgesyndrome
DigeorgesyndromeisanexampleofaTcelldeficiencythatresults
fromfailureofdevelopmentofthethirdandfourthpharyngeal
pouches.
*Clinicalfeatures?
*Enhancedsusceptibilitytoviral,fungal(mucocutaneous
candidiasis)
andbacterialinfections.
*Facialabnormalities:Hypertelorism,abnormalears,shortphiltrum
andmicrognathia.
Hypocalcemictetanyduetofailureofparathyroiddevelopment.
Congenitalheartdiseasesegfallot'stetralogy.
*Absenceofanormalthymus.
*Serumimmunoglobulinconcentrationarefrequentlynormal,but
antibodyresponses,particularlyofIgGandIgAisotypes,areusually
impaired.
*Tcelllevelsarereduced(nottotalabsence).
*Bcelllevelsarenormal.

1426.Therepeuticphlebotomyisnotdonein
whichofthefollowingconditions?
a)CML
b)Polycythemiavera
c)Hemochromatosis
d)Porphyriacutaneatarda
CorrectAnswer-A
Answer-A.CML
Indicationsfortherepeuticphlebotomy-
Polycythemiavera
Hemochromatosis
Secondarypolycythemia
Porphyriacutaneatarda
[RefCurrentapplicationsoftherapeuticphlebotomyTarekBouAssi
andElizabethBaz.BloodTransfus.2014Jan;12(Suppl1):s75-s83]

1427.Whichofthefollowingispresentina
XYchildbutnotinaXXchild?
a)Epoophoron
b)Paroophoron
c)Cowper'sglands
d)Gartner'sduct
CorrectAnswer-C
Answer-C.Cowper'sglands
Bulbourethralgland(Cowper'sgland)arefoundinmales(XY)and
arehomologoustoBartholin'sglandinfemales(XX).
[Ref:"Differentiationoftheurogenitalsinusinmales".Embryology]

1428.TreatmentofchoiceforKawasaki
Diseaseis:
a)IVImmunoglobulins
b)Steroids
c)Dapsone
d)Methotrexate
CorrectAnswer-A
AnswerisA(IVImmunoglobulins):
ThetreatmentofchoiceinKawasakidiseaseisintravenous
immunglobulins


1429.Puffofsmokeappearanceoncerebral
angiographyisseenin:
a)ACAaneurysm
b)Cavernoussinusthrombosis
c)Moyamoyadisease
d)VeinofGalenmalformation
CorrectAnswer-C
Ans.c.Moyamoyadisease
Moyamoyadiseaseisanidiopathic,noninflammatory,non
atheroscleroticprogressivevasculo-occlusivediseaseinvolvingthe
circleofWillis,typicallythesupraclinoidinternalcarotidarteries.
Smallabnormalnet-likevesselsproliferategivingthe
characteristic"puffofsmoke"appearanceondirect
angiography.
CTAandMRAisnotalwaysabletodemonstratethis
appearanceonaccountoflowerflowandspatialresolution.

1430.Osteoclastshaveallofthefollowing
except-
a)Boneresorption
b)Receptorforparathormone
c)Ruffledborder
d)RANKligand
CorrectAnswer-B
Answer-B.Receptorforparathormone
Osteoclastsarefoundinsitesinwhichboneisbeingremodeled
Thesecellsaretheprincipalmediatorofboneresorption
Thecharacteristicfeatureistheareaofinfoldedplasmamembrane
knownasruffledborderwhichissurroundedbyanorganellefree
clearzonethroughwhichosteoclastattachestobone&whichisthe
siteofboneresorption.
RANKbindstoRANKLigandwhichstimulatesboneresorption.
Parathormonearepresentonosteoblasts(notonosteoclasts).

1431.PTHactsdirectlyonwhichcells?
a)Osteoclasts
b)Osteocytes
c)Osteoblasts
d)Macrophages
CorrectAnswer-C
Answer-C.Osteoblasts
Parathormoneactivatesosteoblastswhichthensecretemediatorsof
osteoclastogenesisthatstimulateosteoclastsforboneresorption.

1432.Mostvascularzoneoftheboneis-
a)Metaphysis
b)Diaphysis
c)Epiphysis
d)MedullaryCavity
CorrectAnswer-A
Answer-A.Metaphysis
Metaphysisisthemostvascularzoneoftheboneespeciallyin
childrenasithaslonghairpinlooparrangedarteriolesandvenules
runningthroughit.

1433.Snappingkneesyndromeisdueto
involvementof-
a)PesAnserinus
b)QuadriepsTendon
c)Gastrocnemiusorigin
d)lateralcollateralligament
CorrectAnswer-A
Answer-A.PesAnserinus
Snappingkneesyndromeischaracterizedbypainfulclicks/catching
sensationsexperiencedduringeverymovementofflexionand
extension.Itisexperiencedusuallyattheposteromedialcornerof
thekneeandususallyisduetoinvolvementofsemitendinosusand
gracilistendons.

1434.Tenniselbowischaracterizedby?
a)Tendernessoverthemedialepicondyle
b)Tendinitsofcommonextensororigin
c)Tendinitisofcommonflexororigin
d)Painfulflexionandextension
CorrectAnswer-B
Ans.is'b'i.e.,Tendinitsofcommonextensororigin
Tenniselbowisextraarticularaffectioncharacterizedbypainand
acutetendernessattheoriginoftheextensormusclesofthe
forearmfromthelateralepicondyle.
Itisbelievedtobecausedbystrainoftheforearmextensor
muscles,particularlytheextensorcarpiradialisbrevis,atthepointof
theiroriginfromlateralepicondyle.

1435.FlexorDigitorumProfundustendon
avulsioninjuryleadsto-
a)JerseyFinger
b)MalletFinger
c)GamekeepersThumb
d)BoutonierreDeformity
CorrectAnswer-A
Answer-A.JerseyFinger
JerseyFinger-FlexordigitorumprofundusInjury
MalletFinger-Avulsioninjuryofextensordigitorumtendon
GamekeepersThumb-chronicinjurytoulnarcollateralligamentof
thumb
BennetsFracture-Intraarticularfractureatthebaseof1st
metacarpal

1436.Knailcanbeusedforallofthe
followingfracturesexcePt-
a)Isthmicfemurshaftfractures
b)Intertrochantericfractures
c)Lowsubtrochantericfractures
d)Distalfemurshaftfractures
CorrectAnswer-B
Answer-B.Intertrochantericfractures
Knailisacloverleafshapednailthatreliesontheprincipleofthree
pointfixation
Knailcanneverbeusedforintertrochantericfracturesasitcannot
providestabilityinthisfracture.

1437.ExtensorCarpiRadialisLongusis-
a)Extensorandulnardeviatorofthewrist
b)ExtensorandradialDeviatorofthewrist
c)InjuredinPosteriorinterosseusnerveinjury
d)Weakextensorofthewrist
CorrectAnswer-B
Answer-B.ExtensorandradialDeviatorofthewrist
ExtensorcarpiRadialisLongusisaprimaryextensorandradial
deviatorofthewrist.
Lossoffunctioncauseswristdrop.

1438.Thomastestisusedfortesting?
a)Hipflexion
b)Kneeflexion
c)Hipabduction
d)Hiprotation
CorrectAnswer-A
A
HipflexionREF:LangeInstantAccess:OrthopedicsandSports
Medicine,byAnilPatel,page106
ThomasTest:Withthepatientlyingsupine,maximallyflexbothhips.
Allowthefemurontheipsilateralsidetofallintoasmuchextension
aspossible,whileholdingtheotherhipup.Theanglebetweenthe
femurandexaminingtableistheresidualflexionandrepresentsthe
flexioncontracture.

1439.Achildpresentedtoanorthopaedic
clinicwithalimp.Thesurgeon
suspectedhimtohaveafixedfledon
deformityofthehip.Whichtestshould
thesurgeonperformtoconfirmhis
finding?

a)Thomastest
b)Trendelenburgstest
c)Nelatonstest
d)Telescopingtest
CorrectAnswer-A
Answer-A.Thomastest
Thomastestisdoneforflexiondeformityofhip.

1440.TriggerFingerInvolvesWhichjoint?
a)ProximalInterphalngealjoint
b)DistalInterphalngealfoint
c)Metacarpophalangealjoint
d)Carpometacarpaljoint
CorrectAnswer-C
Answer-C.Metacarpophalangealjoint
Themouthofthefibrousdigitalsheathisatthelevelofmetacarpo-
phalangealjoint.

1441.Effectofhypoparathyroidisrnonbones
include-
a)Browntumours
b)SubperiostealResorptionofBone
c)MultipleCystsinBone
d)NoneoftheAbove
CorrectAnswer-D
Answer-D.NoneoftheAbove
FeaturesofHypoparathyroidism
Prematureclosureofepiphyses
Generalizedincreaseinbonedensity(Osteosclerosis)
Calvarialthickening
Sacroiliacsclerosis
Bandlikedensityinmetaphysic
Thickenedlaminadura+wideneddiploe
Deformedhip(thickening&Sclerosisoffemoralhead&acetabulum)
Intracranialcalcification
Calcificationofspinal&Otherligaments
Subcutaneouscalcification
Ectopicboneformation
Ossificationofmuscleinsertions

1442.Bonetransportcanbeusedinthe
managementof-
a)Gapnonunion
b)DeformityCorrection
c)Communitedshaftfemurfracture
d)AvscularNecrosisofFemoralHead
CorrectAnswer-A
Answer-A.Gapnonunion
Bonetransporttechniqueisprimarilyusedinthemanagementof
gapnonunions.
Anosteotomyismadeinthenormalboneandasegamntofboneis
transportedtothenonunionsite

1443.Aeroplanesplintisusedin?
a)Radialnerveinjury
b)Ulnarnerveinjury
c)Brachialplexusinjury
d)Scoliosis
CorrectAnswer-C
Ans.is'c'i.e.,Brachialplexusinjury
Name
Use
Cramer-wiresplint
Emergencyimmobilisation
Thomassplint
Fracturefemur-anywhere
Bohler-Braunsplint
Fracturefemur-anywhere
Aluminiumsplint
Immobilizationoffingers
DennisBrownsplint
CIEV
Cock-upsplint
Radialnervepalsy
Knuckle-bendersplint Ulnarnervepalsy
Toe-raisingsplint
Footdrop
Volkmann'ssplint
Volkmann'sischaemiccontracture(VIC)
Four-postcollar
Neckimmbilisation
Aeroplanesplint
Brachialplexusinjury
SOMIbrace
Cervicalspineinjury
ASHE(Anteriorspinal Dorso-lumbarspinalinjury
hyperextension)
brace
Taylor'sbrace
Dorso-lumbarimmobilisation
Milwaukeebrace
Scoliosis
Bostonbracet
Scoliosis
Lumbarcorset
Backache

1444.AIIofthefollowingaretrueregarding
applicationofPOPCastexcept-
a)PuttingthePlasterrollinwarmwaterhastenssettingtime
b)ItisanhydrousCalciumphosphate
c)Itcanappliedinpresenceofextremeswelling
d)Gangreneisknowncomplicationofatightplastercast
CorrectAnswer-B
AnswerB.ItisanhydrousCalciumphosphate
POPischemicallyhemihydrouscalciumsulphate.
Warmwaterhastenswhilecoldwaterslowsthesettingtime.
Plastercastshouldbeavoidedwhenthereisextremeswelling.
Commoncomplicationsofplastercastsincludecompartment
syndrome,gangreneandplastersores.

1445.Inuncementedarthroplastyofthehip,
thestemremainsattachedtothebone
by-

a)BoneIngrowth/ongrowthoverthesurfaceofthestem
b)Mechanicalbondingbetweenthestemandbone
c)Pressfittingofthesteminthetightcanal
d)Adhesionbetweenthestemandboneduetoashesive
propertiesofthestem
CorrectAnswer-A
Answer-A.BoneIngrowth/ongrowthoverthesurfaceofthe
stem
Boneingrowth-
Overporoussurface.
Optimalporesizeshouldbe100to400microns.
Fibermeshorbeadsarepresentoverthestemsurface.
Stemcreatedbysinteringordiffusionbondingprocesses.

1446.Duringperfomingatotalhip
replacement,thesurgeonfound
destructionofthearticularcartilageand
rnultiplewedgeshapedsubchondral
depressions.Whatisthiscalled?

a)Osteolysis
b)Osteomyelitis
c)Osteonecrosis
d)Osteogenesis
CorrectAnswer-C
Answer-C.Osteonecrosis
Presenceofdensewedgeshapedopacitiesintheanterolateral
quadrantofthefemoralheadthisiscalledsectoralinvolvement.
Presenceofmultiplecysticandscleroticareas
Acetabularinvolvementleadstodevelopmentofseverearthritis

1447.Suddendorsiflexionoffootmayleadto
whichofthefollowinginjuries-
a)Anteriortalofibularligamentinjury
b)TendoAchillesavulsioninjury
c)Ruptureofdeltoidligament
d)Tarsaltunnelsyndrome
CorrectAnswer-B
Answer-B.TendoAchillesavulsioninjury
AruptureoftheAchillestendonoccurswhenthetendonisstretched
outsidebeyonditscapacity.
Themostcommonsiteofruptureisthe"watershed"lessvascular
areaoftendon,whichis4cmproximaltoitsinsertiononcalcaneum.

1448.SalterHarrisclassificationisusedfor-
a)Supracondylarhumerusfracturesinchildren
b)Estimationofgrowthofthephyses
c)Physealinjuries
d)Severityofdeglovinginjuriestothelimb
CorrectAnswer-C
Answer-C.Physealinjuries

1449.MilwaukeeBraceisusedin-
a)CongenitalKyphosis
b)ScheurmannsDisease
c)AdolescentldiopathicScoliosis
d)Spondylolisthesis
CorrectAnswer-A
Answer-A.CongenitalKyphosis
TheMilwaukeebraceisaplasticbodyjacketusedinthetreatment
ofadolescentswithidiopathicscoliosisandScheuermann'sdisease.
TheMilwaukeebrace,alsorefferredtoasaCervico-Thoraco-
Lumbo-Sacral-Orthosisbrace,issimilartobracesforthelowerback,
butalsoincludesaneckringheldinplacebyverticalbarsattached
tothebodyofthebrace.

1450.VertebraPlanaisseeninallexcept-
a)HistioctosisX
b)Leukemia
c)Excessiveuseofsystemicsteroids
d)ScheurmannsDisease
CorrectAnswer-D
Answer-D.ScheurmannsDisease
CausesofVertebraPlana

1. Histocytosis-X(Eosinophilicgranuloma)
2. Leukemia
3. TB
4. Metastasis,Multiplemyeloma,Ewing'ssarcoma,lymphoma
5. Osteochondritisofvertebralbody(Calve'sdisease)
6. Hemangioma
7. Trauma
8. Steroids

1451.Whatisluxatioerecta?
a)AnteriorDislocationoftheshoulderjoint
b)InferiorDislocationoftheshoulderjoint
c)AnteriorDislocationoftheHIPjoint
d)PosteriorDislocationofHipjoint
CorrectAnswer-B
Answer-B.InferiorDislocationoftheshoulderjoint
Theheadofthehumerusisbelowtheglenoidcavityandthe
humeralshaftispointingoverhead.
Itisduetohyperabductioninjury.
Itisrareandalsocalledluxatioerectabecausethehumeralheadis
subluxated(dislocatedinferiorly)andhumerusshaftpointsupwards
(erected).

1452.Whichofthefollowingcasts/splintsis
usedforfractureshafthumerus?
a)Hangingcasts
b)Knucklebendersplint
c)AeroplaneSplint
d)Aboveelbowcast
CorrectAnswer-A
Answer-A.Hangingcasts
Hangingcast-Fractureofthehumerus
Turn-bucklecast-Scoliosis

1453.Liftofftestisdonetoassessthe
functionof:
a)Supraspinatus
b)Infraspinatus
c)TeresMinor
d)Subscapularis
CorrectAnswer-D
Di.e.Subscapularis
-Failuretoperformmaximuminternalrotation(astestedinbelly
pressandliftofftest)orinabilitytomaintainpositionofmaximal
internalrotation(internalrotationlagsign)indicatetearof
subscapularistendon.
Liftofftestisdonetoassessthestrengthofsubscapularismuscle
anddetectanisolatedruptureofsubscapularistendoninarotator
cufftear.

1454.Allaretrueaboutmenisciofkneejoint
except
a)Lateralmeniscuscoversmorearticularsurfaceoftibia-
b)Lateralmeniscusismoremobile
c)Lateralmeniscusismorepronetoinjury
d)Lateralmeniscusissemicircular
CorrectAnswer-C
Ans.is'c'i.e.,Lateralmeniscusismorepronetoinjury

1455.Threepointbonyrelationshipofthe
elbowisdisturbedin-
a)SupracondylarFractureofthehumerus
b)Fracturelateralcondyleofthehumerus
c)MonteggiaFracturedislocation
d)FractureofProximalRadius
CorrectAnswer-B
Answer-B.Fracturelateralcondyleofthehumerus
Threeprominentbonypointsaroundelbowaremedialepicondyle,
lateralepicondyleandtipoftheolecranon.
Inelbowinjuries,followingisseeninthreebonyrelationship:
1. Maintained
2. Disturbed

1456.HamiltonRulertestsignispositivein
whichoftheabovementioned
conditions?

a)Anteriordislocationofshoulder
b)Acrornioclavivularjointdislocation
c)Posteriordislocationofshoulder
d)luxatioerecta
CorrectAnswer-A
Answer-A.Anteriordislocationofshoulder
Hamiltonrulertest:Becauseofflatteningofshoulder,itispossible
toplacearuleronthelateralsideofarmandittouchesacromian&
lateralcondyleofhumerussimultaneously(innormalitwouldnot
duetoshouldercontour).

1457.CubitusValgusDeformityiscommonly
seeninwhichofthefollowing
conditions-

a)MalunitedLateralCondylarfractureofHumerus
b)MalunitedSupracondylarFractureofHumerus
c)Posteriordislocationofelbow
d)Fracturemedialcondyleofhumerus
CorrectAnswer-A
Answer-A.MalunitedLateralCondylarfractureofHumerus
Fracturescommonlyshowingcubitusvalgusdeformitydueto
malunion:
Fracturelateralcondylehumerus
MonteggiaFractureDislocation

1458.TrueregardingMonteggiafractureis:
March2007,March2013(a,b,d,f)
a)Upperulnarfracture&dislocatedradialhead
b)Upperradialfracture&dislocatedulna
c)Lowerradialfracture&dislocatedulna
d)Lowerulnarfracture&dislocatedradius
CorrectAnswer-A
Ans.A:Upperulnarfracture&dislocatedradialhead
Monteggiafracture-dislocationsareclassifiedbytheBado
system
BadotypeIinjuriesarecharacterizedbyaproximalulnarfracture
withanteriordislocationoftheradialhead.Thisisduetoaforceful
pronationinjuryoftheforearmandisthemostcommontype.
BadotypeIIinjuriesare"reversed"Monteggiafracture-dislocation
injuries.
Here,thereisposteriorangulationoftheulnarfracturesiteand
posteriordislocationoftheradialhead.BadotypeIIIandIVarerare
injuries.

1459.Dinnerforkdeformityisseenin?
a)Colle'sfracture
b)Marchfracture
c)Lateralcondylefracture
d)Supracondylarfracture
CorrectAnswer-A
Ans.is'a'i.e.,Colle'sfracture
Complicationsofcolle'sfracture
Complicationsincolle'sfracturearehigh(50-60%).Following
complicationsmayoccur
1)Stiffnessoffingersandjoints
Stiffnessoffinger,wristandshoulderisthemostcommonavoidable
complicationofcolle'sfracture.
Thisoccursduetolackofexercise.Therefore,patientshouldbe
encouragedforactiveexerciseoffingerandshoulder.
2)Malunion
Itisthesecondmostcommoncomplication
Itresultsindinnerforkdeformity
3)Sudek'sosteodystrophy
(reflexsympatheticdystrophy)
4)Carpaltunnelsyndrome:
Mediannervemaygetcompressedincarpaltunnel
5)Carpalinstability
6)Ruptureoftheextensorpollicislongustendon
7)Frozenshouldersyndrome
Thisisatroublesomecomplicationwhichdevelopsdueto
unnecessoryvoluntaryshoulderimmobilizationbythepatientonthe
affectedsideforfearoffracturedisplacement.
8)TFCCinjury

9)Non-unionisveryrare

1460.AVNfollowingtranscervicalneckfemur
fracturesoccursduetodamageto
whichofthefollowingbloodvessels?

a)Lateralretinacularbranchoflateralcircumflexfemoralartery
b)Lateralretinacularbranchofmedialcircumflexfemoralartery
c)Medialretinacularbranchoflateralcircumflexfemoralartery
d)Obduratorartery
CorrectAnswer-A
Answer-A.Lateralretinacularbranchoflateralcircumflex
femoralartery
LateralcircumflexfemoralarteryItsuppliesthroughanterior
retinacularartery.

1461.Brodie-Trendelenburgtestispositive
in:
a)Deepveinthrombosis
b)Sapheno-femoralincompetence
c)Thromboangiitisobliterans
d)Below-kneeperforatorsincompetence
CorrectAnswer-B
Ans.B:Sapheno-femoralIncompetence
Atesttoassessthecompetenceofthesaphenofemoraljunction.
TheBrodie-Trendelenburgtestisusedtodetectvenous
incompetenceandtodifferentiatebetweenperforatorandGSV
incompetence.
TheBrodie-Trendelenburgtestishighlysensitiveforthe
identificationofsuperficialandperforatorreflux.
SFJ(saphenofemoraljunction)incompetenceisdiagnosedifthe
distalveinsfillrapidlyuponreleaseofthetourniquet.
SometextbooksrefertotheTrendelenburgtestandthetourniquet
testinterchangeably.

1462.Lockingofthekneeinvolves-
a)Externalrotationoffemurwiththefootofftheground
b)Internalrotationofthetibiawiththefootontheground
c)Contractionofpopliteus
d)Internalrotationoffemurwithfootontheground
CorrectAnswer-D
Answer-D.Internalrotationoffemurwithfootontheground
Physiologicallockingoccursinextensionwhenthefemuris
internally(medially)rotatedonafixedtibia.Lockingisamechanism
thatallowsthekneetoremaininthepositionoffullextensionasin
standingwithoutmuchmusculareffortsandiscausedbyquadriceps
femoris.

1463.Footdropoccursduetothe
involvementof:
September2006,March2013(b,fg,h)

a)Sciaticnerve
b)Directinjurytothedorsiflexors
c)Commonperonealnervepalsy
d)Alloftheabove
CorrectAnswer-D
Ans.D:Alloftheabove
Footdropmayfollowdirectinjurytothedorsiflexors.
Afewcasesofruptureofthetibialisanteriortendonleadingtofoot
dropandsuspicionofperonealnervepalsyhaveoccurred.This
subcutaneoustendonruptureusuallyoccursafteraminortrauma
withthefootinplantarflexion.
Compartmentsyndromesalsomayleadtofootdrop.
Marchgangrene,aformofanteriorcompartmentsyndrome,is
thoughttobeduetoedemaandsmallhemorrhagesinthemuscles
oftheanteriorcompartmentoccurringafterstrenuousactivityin
individualsnotaccustomedtoit.Deepposteriorcompartment
syndromealsomayresultinfootdropasalatesequeladueto
resultantcontractureformation.
Neurologiccausesoffootdropincludemononeuropathiesofthe
deepperoneal,commonperoneal,orsciaticnerves.Lumbosacral
plexopathy,lumbarradiculopathy,motorneurondisease,or
parasagittalcorticalorsubcorticalcerebrallesionsalsocanmanifest
asfootdrop.Theselesionscanbedifferentiatedthroughclinicaland
electrodiagnosticexaminations.

Acommonbehavioralcauseoffootdropishabitualcrossingofthe
legs.Thesecasestypicallyresolvewithdiscontinuationofthehabit.
Footdropalsomaybeseenasacombinationofneurologic,
muscular,andanatomicdysfunction.Charcotfootisoneexample.

1464.Fractureneckoffemurin80yearold
malesustained1weekbackThe
treatmentofchoiceis-

a)Hemiarthroplasty
b)ExcisionarthroPlastY
c)Closedreductionandfixationwiththreecancellousscrews
d)Longitudinalskintractionfor6weeks
CorrectAnswer-A
Answer-A.Hemiarthroplasty
Closedreduction&screwfixation
If2attemptsofclosedreductionfail,hemiarthroplastyisdone

1465.Whichofthefollowingfracturesofthe
neckoffemurareassociatedwith
maximalcompromiseinbloodsupply?

a)Intertrochantericfractures
b)Basicervicalfracture
c)Transcervicalfracture
d)SubCapitalfractures
CorrectAnswer-D
Answer-D.SubCapitalfractures
Thenon-unionandAVNwillbemostcommoninsubcapitalfracture
andleastinbasicervicalfracture.

1466.AVNisseeninwhichtypeof#offemur:
a)Intertrochantric#
b)Subcapital#
c)Transcervical#
d)BANDC
CorrectAnswer-D
B.i.e.Subcapital>C.i.e.Transcervical
Fractureoftheneckoffemur
FractureoftheScaphoid(Proximalpole>Waist)
Fractureneckoftalus
Posteriordislocationofthehip

1467.Tinelssignisseenin-
a)AvascularnecrosisofscaPhoid
b)KienbocksDisease
c)lstcarpometacarpaljointarthritis
d)Carpaltunnelsyndrome
CorrectAnswer-D
Answer-D.Carpaltunnelsyndrome
Mediannervepercussiontest(Tinel'ssign):-Themediannerveis
gentlytappedatthewrist.Thetestispositiveifthereistingling
sensation

1468.Whichofthefollowingisusedasa
substituteforwristextensorsinradial
nervePalsY?

a)PronatorTeres
b)PalmarisLongus
c)FlexorDigitorumSuPerficialis
d)FlexorDigitorumProfundus
CorrectAnswer-A
Answer-A.PronatorTeres
Pronatorteresisacommonmuscleusedasasubstituteforwrist
extensorsincaseofwristdropoccurringasaresultofradialnerve
palsy.Pronatorteresisapronatoroftheforearmsuppliedby
Mediannerve.

1469.Mostcommoncauseofca4raltunnel
syndromeis?
a)Pregnancy
b)Idiopathic
c)Alcoholism
d)Occupational-Excessiveuseofvibratoryinstruments
CorrectAnswer-B
Answer-B.Idiopathic
Mostcommontypeofcarpaltunnelsyndromeisidiopathicno
knowncause.

1470.A45yearoldcarpenterwithablunt
traumatohisarmsustainedafracture
followingwhichhedevelopedwristdrop,
lossofextensionatfingersandlossof
sensationsonthelateralaspectofthe
wristjoint.Whichofthefollowingis
true?

a)Patienthasaninjurytothemediannerve
b)Heshouldhavealsolostextentionoftheforearm
c)Patienthasinjuredtheradialnerveinthespirdgroove
d)Thereiscombinedinvolvementoftheradialnerveandmedian
nerve
CorrectAnswer-C
Answer-C.Patienthasinjuredtheradialnerveinthespird
groove
Radialnerveinjurymaybehighorlow.
1)Highradialnervepalsy
Injuryisbeforethespiralgroove
Allmusclessuppliedbyradialnerveareparalysed
2)Iflesionishigh
Wristdrop,thumbdropandfingerdrop.
Inabilitytoextendelbow,wrist,thumb&fingers(MPjoint)
Patientcanextendinterphalangealjointsduetoactionoflumbricals
andinterossei.
Sensorylossoverposteriorsurfaceofarm&forearmandlower
lateralhalfofforearm.


1471.Wristdropisduetoinjuryto?
a)Radialnerve
b)Ulnarnerve
c)Mediannerve
d)Posteriorinterosseousnerve
CorrectAnswer-A
Ans.(A)Radialnerve
Clinicalfeaturesofradialnervepalsy
Clinicalfeaturesdependuponthesiteoflesion.
1)Iflesionishigh
Wristdrop,thumbdropandfingerdrop.
Inabilitytoextendelbow,wrist,thumb&fingers(MPjoint)
Patientcanextendinterphalangealjointsduetoactionoflumbricals
andinterossei.
Sensorylossoverposteriorsurfaceofarm&forearmandlower
lateralhalfofforearm.
2)Iflesionislow
a)TypeI
Wristdrop,thumbdropandfingerdrop.
Elbowextensionispreserved.
Sensorylossoverthedorsumoffirstwebspace.
b)TypeII
Thumbdropandfingerdrop
Elbowandwristextensionispreserved
Sensorylossoverthedorsumoffirstwebspace.

1472.Inwhichofthefollowingdeformitiesis
thedistalinterphalangealjointextended
?

a)Boutonnieredeformity
b)Swanneckdeformity
c)Zdeforrnity
d)ClawHand
CorrectAnswer-A
Answer-A.Boutonnieredeformity
Boutonnieredeformity:FlexioncontractureofPIPjointand
extensionofDIPjoint.

1473.Clubfootclinicallypresentaswhat
deformity?
a)Calcaneovalgus
b)Equinovarus
c)EquinoCavovarus
d)Calcaneovarus
CorrectAnswer-C
Answer-C.EquinoCavovarus
CTEVisthecommonestandmostimportantcongenitaldeformityof
thefoot.
Thedeformityconsistsoffollowingelements:
1. Equinus,i.e.Plantarflexionatanklejoint(tibiotalarjoint)
2. Inversionoffootatsubtalarjoint(talocalcanealjoint)
3. Forefootadduction,atmid-tarsaljoints,especiallyattalo-navicular
joint.
4. Sometimesforefootcavus,i.e.excessivearchingofthefootatmid-
tarsaljoints

1474.4yearoldchildpresentedtotheclinic
withahistoryoffallonoutstretched
hand.Radiographsrevealedabroken
anteriorcortexwithanintactposterior
cortexOftheradiuswithan
exaggeratedbowingoftheradius.The
fracturesustainedisknownas-

a)TorusFracture
b)Greenstickfracture
c)GalleaziFracture
d)MonteggiaFrtactureDislocation
CorrectAnswer-B
Answer-B.Greenstickfracture
GreenstickFracture:Incompletefractureofthebonewithplastic
deformationontheconcavesideofthebone.Thefractureneedsto
becompletedtoobtainreduction.

1475.Greenstick/Nightstickfracturesare
seenin-
a)Children
b)Elderly
c)Youngadults
d)Commoninallagegroups
CorrectAnswer-A
Answer-A.Children

1476.Waddlinggaitdueto:
March2009
a)Bilateralcongenitaldysplasiaofhip
b)Coxavalga
c)CTEV
d)Bilateralcoxavalgum
CorrectAnswer-A
Ans.A:Bilateralcongenitaldysplasiaofhip
Awaddlinggaitisthestyleofwalkingthatisseeninapatient
withproximalmyopathy.Itischaracterisedby:
Abroad-basedgaitwithaduck-likewaddletotheswingphase
Thepelvisdropstothesideofthelegbeingraised
Forwardcurvatureofthelumbarspine
Markedbodyswing
Thisgaitmaybeseeninpatientswithbilateralcongenitalhip
dislocationandpregnancy.

1477.Blount'sdiseaseisassociatedwithallofthefollowing,EXCEPT:
a)Genuvarum
b)GenuRecurvatum
c)InternalTibialTorsion
d)ExternalTibialTorsion
CorrectAnswer-D
Blount'sdiseaseischaracterizedbyvarusandinternaltibialtorsionandgenurecurvatum,
withvarusoftheproximaltibiabeingtheprimarydeformity.Itisnotassociatedwithexternal
tibialtorsion.
Ref:Operativetechniquesinpediatricorthopedics,byJohnM.Flynn,SamW.Wiesel,Page
205,206.

1478.WhodevisedcorrectionofCTEVby
serialcastinga-
a)IgnasioPonseti
b)GerhardtKuntscher
c)GavrilIlizarov
d)HughOwenThomas
CorrectAnswer-A
Answer-A.IgnasioPonseti
IgnasioPonsetipropoundedthetechniqueofserialweeklycasts.

1479.Osteosclerosisisafeatureofwhichof
thefollowinga-
a)Rickets
b)Hyperparathyroidism
c)PagetsDisease
d)OsteogenesisImperfecta
CorrectAnswer-C
Answer-C.PagetsDisease
Paget'sdiseaseischaracterizedbyincreasedboneturnoverand
enlargementandthickeningofthebone,buttheinternalarchitecture
isabnormalandtheboneisusuallybrittle.
Primarydefectisinosteoclastswithincreasedosteoclasticactivity.
Thisresultssecondarilyincreaseinosteoblasticactivity(normal
osteoclastsandosteoblastsactinaco-ordinatedmanner).

1480.Allofthefollowingaretrueregarding
PagetsDiseaseexcept-
a)Pelvisisthemostcommonsite
b)Cranialnerveinvolvementmaybeseen
c)Highoutputcardiacfailureisoneofthecomplications
d)Itmayprogresstoasecondarychondrosarcoma
CorrectAnswer-D
Answer-D.Itmayprogresstoasecondarychondrosarcoma
Pagetsdiseasemayprogresstoasecondaryosteosarcomanot
chondrosarcoma

1481.Whichofthefollowingis/aretrueaboutperthesdisease?

1.Avascularnecrosisoffemoralhead
2.Onsetbefore10yearsofage
3.Osteotomyisusedfortreatment
4.Limbshortening

a)1,2true&3,4false
b)2,3True&1,4false
c)1,2,3true&4false
d)Allaretrue
CorrectAnswer-D
ItisalsoknownastheparsplanaandPseudoCoxalgia.Itisanosteochondritisofthe
femoralhead.Thefemoralheadbecomespartlyorwhollyavascularanddeformed.
Etiologyisnotdefinetlyknown.Itissupposedtobeduetoreccurentepisoedesof
ischemiaofthefemoralheadinthesuceptibleagegroup,probablyperecipitaedby
episodesofsynovitis.
Pathology:thediseaseprogressesinthreeilldefinedstages:
Stageofsynovitis
Stageoftrabecularnecrosis
Stageofhealing
Clinicalfeatures:
Commonlyseenintheagegroupof5to10years.Childpresentwithpaininthehipwhich
oftenradiatestothekneeormayalsopresentwithlimporhipstiffness.Onexamination,
thefindingareminimal.sometimestheonlyfindingbeinglimitationofabductionand
internalrotationandshortening.
X-rayshowscollapseandsclerosisoftheepiphysisofthefemoralhead.hipjointspaceis
increased.Bonescanmayshowadecreaseduptakebytheheadofthefemur.
Treatment:Theprimaryaimistopreventtheheadfromillshapingwhiletheboneisinthe
softeningphase.Theheadisrequiredtobekeptinsidetheacetabulumwhilethe
revascularizationtakesplace(i.e.containment).Thismaybeachievedbycorrective
methods(plaster,splintetc.)orbyoperation(containmentosteotomy).

Ref:Maheshwari3/e,Page269-70.

1482.Microorganismwhichcausespyogenic
osteomyelitisis?
a)Streptococcus
b)Staphaureus
c)Corynebacterium
d)Neisseriagonorrhoeae
CorrectAnswer-B
Staphylococcusaureuscauses80?90%ofcasesofacute
pyogenicosteomyelitis.
Saureushasareceptorforcollagen,which
contributestoitspathogenicity.Otherorganismsthatcause
osteomyelitisinclude:
EscherichiacoliandPseudomonasinintravenousdrugusersand
patientswithurinarytractinfections.
HaemophilusinfluenzaeandGroupBStreptococcusinneonates.
Salmonellainpatientswithsicklecelldisease.
Ref:WyattC.,KempW.L.,MoosP.J.,BurnsD.K.,BrownT.G.
(2008).Chapter19.PathologyoftheBonesandJoints.InC.Wyatt,
W.L.Kemp,P.J.Moos,D.K.Burns,T.G.Brown(Eds),Pathology:
TheBigPicture.


1483.Commonestsiteofoccurrenceof
chondrosarcomais-
a)Pelvis
b)Ribs
c)Femur
d)Proximaltibia
CorrectAnswer-A
Answer-A.Pelvis
Achondrosarcomaisamalignanttumorderivedfromcartilagecells
andittendstomaintainitscartilaginouscharacterthroughoutits
evolution
Mostcommonlyinvolvedbonesarepelvis(mostcommon),femur
(2ndmostcommon),humerus,ribsandshouldergirdles.
Itmaybeatmetaphysisordiaphysis.

1484.OnionpeelappearanaceonXrayis
seeninwhichofthefollowing
conditions?

a)EwingsSarcoma
b)Osteosarcoma
c)Giantcelltumour
d)Eosinophilicgranuloma
CorrectAnswer-A
Answer-A.EwingsSarcoma
OnionPeelAppearance EwingsSarcoma
CodmansTriangle
Osteosarcoma
SunBurstAppearance
Osteosarcoma
SoapBubbleappearance GCT

1485.

Whichpartofthespineismostcommonly
affectedinRheumatoidarthritis:

a)Cervical
b)Lumbar
c)Thoracic
d)Sacral
CorrectAnswer-A
AnswerisA(Cervical):
Rheumatoidarthritiscommonlyinvolvesthejointsofhands,wrist,
elbow,knees,ankle,andfeetinasymmetricalmanner.
AxialskeletoninvolvementisusuallylimitedtoUpperCervical
Spine.


1486.Hammertoedeformityisseenin-
a)Rheumatoidarthritis
b)Fracturedistalphalanxofgreattoe
c)Bunion
d)Osteochondritis
CorrectAnswer-A
Answer-A.Rheumatoidarthritis
FootHalluxvalgus,Hammertoe,etc.

1487.Nervedamagedduetolunate
dislocation(incarpaltunnel):
a)Median&ulnar
b)Median
c)Ulnar
d)Radial
CorrectAnswer-B
B.i.e.Mediannerve
Commoncausesofmediannervepalsyincarpaltunnelarecarpal
tunnelsyndrome(mostcommon)andInnatedislocation.

1488.Malodorousvaginaldischargeisdue
to
a)Bacterialvaginosis
b)Chlamydiatrachomatis
c)Trichomonasvaginalis
d)Neisseriagonorrhea
CorrectAnswer-A
Answer-A.Bacterialvaginosis
Bacterialvaginosisisanalterationofthenormalvaginalflorawith
consequentovergrowthofpathogenicbacteria.
Thereisfallintheabsolutenumberofhydrogenperoxide-
producinglactobacilli,leadingtoariseinpHandincreaseinthe
absolutenumberofG.vaginalis,anaerobicgram-negativerods,
Mobiluncusspp.andMycoplasmahominis.

1489.Quickeninginmultiparaeisfeltatwhat
weeksofgestation?
a)14weeks
b)16weeks
c)18weeks
d)20weeks
CorrectAnswer-B
Answer-B.16weeks
Quickeningisfeelingoflife.Itdenotesperceptionofactivefetal
movementsbythewomen.Itisusuallyfeltaboutthe18thweek,
about2weeksearlierinmultiparae.Itsappearanceisanuseful
guidetocalculatetheexpecteddateofdeliverywithreasonable
accuracy.

1490.Trueaboutplacentalsitetrophoblastic
diseaseis
a)HighlyMalignantbehavior
b)Hysterectomyfollowedbychemoradiationisthetreatmentof
choice
c)Secreteshumanplacentallactogen
d)Containssyncytiotrophoblastsmainly
CorrectAnswer-C
Answer-C.Secreteshumanplacentallactogen
PlacentalSiteTrophoblasticTumour
Arisesfromtheplacentalbedtrophoblastsandinvadesthe
myometrium.
Followsafull-termnormaldelivery.
Tumorcontainsmainlycytotrophoblastswithfeworno
syncytiotrophoblasts.
Mostofthesetumorsrunabenigncourse,malignancyisrare.
[RefShaw'sGynaele/ep.313;Dutta'sObs8thiep.231]

1491.Crowningis
a)Biparietaldiameterattheinletofpelvis
b)Biparietaldiameterattheischialspine
c)Biparietaldiameteratthevulvaloutlet
d)BiparietaldiameterjustoutsidethevulvaloutletAnswer-
CorrectAnswer-C
Answer-C.Biparietaldiameteratthevulvaloutlet
Crowning:Afterinternalrotationofthehead,furtherdescentoccurs
untilthesubocciputliesunderneaththepubicarch.Atthisstage,the
maximumdiameterofthehead(biparietaldiameter)stretchesthe
vulvaloutletwithoutanyrecessionoftheheadevenafterthe
contractionisover-called"crowningofthehead".

1492.Prolongedsecondstageoflabourmay
occurdueto
a)Uterineinertia
b)Epiduralanalgesia
c)Cephalopelvicdisproportion
d)Alloftheabove
CorrectAnswer-D
Answer-D.Alloftheabove
Prolongationofsecondstageoflabourcanoccurduetoanyof
thefollowingreasons:
1)Faultinthepower
Uterineinertia
Inabilitytobeardown
Epiduralanalgesia
Constrictionring
2)Faultinthepassage
Cephalopelvicdisproportion,androidpelvis,contractedpelvis
Undueresistanceofthepelvicfloororperineumduetospasmorold
scarring
Softtissuepelvictumor.
3)Faultinthepassenger
Malposition(occipito-posterior)
Malpresentation
Bigbaby
Congenitalmalformationofthebaby

1493.Progesteroneofchoiceinemergencycontraceptionis?
a)Norethisterone
b)Medroxyprogesterone
c)Oxytocin
d)Levonorgestrel
CorrectAnswer-D
Unprotectedintercoursewithoutregardtothetimeofthemonthcarriesan8%incidenceof
pregnancy,anincidencethatcanbereducedto2%bytheuseofemergency
contraceptiveswithin72hoursofunprotectedintercourse.
0.75mglevonorgestrelarenowapprovedforpostcoitalcontraceptionandareavailable
overthecounterforwomenaged>17years.Levonorgestrelismoreeffectiveandis
associatedwithfewersideeffectsthanthecombinationestrogen-progestinregimens.
Basicallythemethodsusedinterferewiththephysiologicaleventsbeforeimplantation,for
e.g.inhibitionordelayingofovulationorinterferencewithpostovulatoryeventsnecessary
forimplantation&longevityoftheblastocyst.
Ref:HallJ.E.(2012).Chapter347.TheFemaleReproductiveSystem,Infertility,and
Contraception.InD.L.Longo,A.S.Fauci,D.L.Kasper,S.L.Hauser,J.L.Jameson,J.
Loscalzo(Eds),Harrison'sPrinciplesofInternalMedicine,18e.

1494.Lovsetmanoeuvreisusedindeliveryof
:
a)Head
b)Breech
c)Foot
d)Arms
CorrectAnswer-D
Arms
Lovset'smaneuver
Principle:Becauseofthecurvedbirthcanal,whentheanterior
shoulderremainsabovethesymphysispubis,theposteriorshoulder
willbebelowthesacralpromontory.Ifthefetaltrunkisrotated
keepingthebackanteriorandmaintainingadownwardtraction,the
posteriorshoulderwillappearbelowthesymphysispubis.

1495.Commonmisdiagnosisofpartialmole
is
a)Threatenedabortion
b)Choriocarcinoma
c)Completemole
d)Ectopicpregnancy
CorrectAnswer-A
Answer-A.Threatenedabortion
Theclinicalpictureofpartialmoleisconfusedwiththreatened
abortionormissedabortionduetopainandbleedingalongwitha
fetus(usuallydead)inutero.

1496.Whenfetusisatstation+2&fetalskull
reachespelvicfloor,whichofthe
followingistrue

a)Forcepscanbeapplied
b)Besttimetogiveepisiotomy
c)Calledascrowning
d)Mayleadtodeeptransversearrest
CorrectAnswer-A
Answer-A.Forcepscanbeapplied
Whenthefetalskullhasreachedthelevelofpelvicfloorandstation
ofheadisat+2ormore,outletforcepscanbeapplied.
However,crowningisdefinedasstretchingofthevulva'outletbythe
maximumdiameterofthefetalhead(biparietaldiameter)without
anyrecession.Thusthestationis+5.
Besttimetogiveepisiotomyisatthetimeofcrowningofhead.
[RefDutta'sObstetrics81h/ep.651]

1497.Mostcommonbreechpositionis
a)Completebreech
b)Frankbreech
c)Footlingbreech
d)Kneepresentation
CorrectAnswer-B
Answer-B.Frankbreech
Frankbreech(Breechwithextendedlegs)isthemostcommon
breechpresentation.
Breechwithextendedlegs(Frankbreech):Thighsareflexedathip
andlegsareextendedatknee.Itiscommonlyseeninprimigravidae
(70%).
Itisthemostcommontypeofbreech.

1498.Whichisnotariskfactorforgestational
hypertension
a)Obesity
b)Smoking
c)Primigravida
d)FactorVLeidenmutation
CorrectAnswer-B
Answer-B.Smoking
Primigravida:Youngorelderly(firsttimeexposuretovilli)
Familyhistory(hypertension,pre-eclampsia)
Placentalischemia
Obesity
Thrombophilia(antiphospholipidsyndrome),proteinCandS
deficiency,factorVLeidenmutation)
Molarpregnancy(earlyonsetpre-eclampsia)

1499.Methodnotusedforshoulderdystocia
a)McRobert'smaneuver
b)Hegar'smaneuver
c)Zanavellimaneuver
d)Wood'smaneuver
CorrectAnswer-B
Answer-B.Hegar'smaneuver
McRobert's,ZanavelliandWood'smaneuvers'aremaneuversinthe
managementofshoulderdystocia.

1500.Occipitoanteriorposition
a)Anteriorfontanalleisposterior
b)SagittalsutureisalongtheTransverseplaneofthematernal
pelvis
c)Coronalsutureisalongtheantero-posteriorplaneofthe
maternalpelvis
d)Alloftheabove
CorrectAnswer-A
Answer-A.Anteriorfontanalleisposterior
Indirectoccipitoanteriorposition,thesagittalsutureofthefetuswill
bealongtheantero-posteriorplaneofthematernalpelvisandthe
coronalsuturealongthetransversediameter.

1501.InMedicalterminationofpregnancy,
accordingtoFDA,Misoprostolisgiven
afterhowmanyhoursofMifepristone?

a)24hours
b)48hours
c)72hours
d)96hours
CorrectAnswer-B
Ans.is'b'i.e.,48hours
FDAapprovedprotocol-(Originalprotocol)
600mgofmifepristone(i.e.,3tablets)givenorallyonday1followed
2days(48hours)laterbyoralmisoprostol400tig(2tablets)onday
3.
Thetreatmentshouldbestartednomorethan48daysfromthestart
ofthelastmenstrualperiod.
Butaccordingtotherecentprotocol
200mgofmifepristone(itisaseffectiveas600mgofmifepristone)
isgivenorallyonDay1followed2days(48hours)laterbyvaginal
misoprostol800pg.
Thisregimeprovideshighestefficacywithin63daysofamenorrhea.

1502.Thebestmethodforinducingmid
trimesterabortionis:
a)InjectionofHypertonicSaline
b)Ethacrydine
c)Prostaglandins
d)DandC
CorrectAnswer-C
Prostaglandins
Midtrimesterterminationofpregnancy
A)Medicalmethods
i)Prostaglandins:misoprostol(PGE1)withorwithoutMife
pristone,gemeprost(PGE1),dinoprostone(PGE2),carboprost
(PGE2)
ii)Oxytocin

1503.Twinpregnancyofthesameageand
sexrulesout?
a)Superfetation
b)Maternaltwins
c)Superfecundation
d)Noneoftheabove
CorrectAnswer-A
Ans.is'a'i.e.,Superfetation
Superfetation
Itisthesimultaneousoccurrenceofmorethanonestageof
developingoffspringinthesameanimal.
Inmammals,itmanifestsastheformationofanembryofroma
differentestrouscyclewhileanotherembryoorfetusisalready
presentintheuterus.

1504.Propulsivestageinlabourinmultipara
a)10minutes
b)20minutes
c)40minutes
d)1hour
CorrectAnswer-B
Answer-B.20minutes
Secondstageoflabourhastwophases:

1. Propulsivephase:fromfulldilatationofcervixuntilheadtouches
pelvicfloor.
2. Expulsivephase:Sincethetimethereisirresistiblematernaldesire
tobeardownuntilthebodyisdelivered.
Meandurationofsecondstageis50minutesfornulliparaand20
minutesinmultipara.

1505.Duringpregnancyincreasedsizeof
pituitaryiscausedbyincreasedsizeof
cellswhichsecrete?

a)Growthhormone
b)Prolactin
c)ACTH
d)TSH
CorrectAnswer-B
Answer-B.Prolactin
Prolactinlevelsrisegraduallythroughoutpregnancy,preparingthe
breastforlactation.
Thislactotrophhyperplasiahasimportantimplicationsforthepatient
withaprolactinomawhodesirespregnancy.

1506.Therapeuticuseoffolicacidis
a)Previouspregnancywithchildhavingneuraltubedefect
b)Megaloblasticanaemia
c)Hemoglobinopathies
d)Alloftheabove
CorrectAnswer-B
Answer-B.Megaloblasticanaemia
Folicacidisgiveninallthegivenconditions.Butanswerhearis
optionb.
Thisquestionsistricky:-
Examinerisaskingabouttherapeuticusesoffolicacid(not
preventive)
Inmegaloblasticanemia,folicisgivenfortreatment-->therapeutic
uses
Inpregnancyitisgivenforpreventionofneuraltubedefect-->
prophylacticuse(preventiveuse)

1507.Whatwouldbethetypeofpresentationwhentheengagingdiameteris
mentovertical?
a)Face
b)Brow
c)Vertex
d)Breech
CorrectAnswer-B
Inbrowpresentation,theengagingdiameterismentovertical(14cm).Browisthe
rarestvarietyofcephalicpresentationwherethepresentingpartisthebrowandtheattitude
oftheoftheheadisshortofthatdegreeofextensionnecessarytoproduceface
presentation.
Infacepresentation,thepresentingpartisface,attitudeofthefetusshowscomplete
flexionofthelimbswithextensionofthespine.Thecommonestpositionisleftmento
anterior.Theengagingdiameteroftheheadissubmentobregmatic9.5cminfully
extendedheadorsubmentovertical11.5cminpartiallyextendedhead.

Ref:TextbookofObstetricsByD.CDutta,6thedn,page392-3

1508.Cardiacoutputinpregnancyincreases
fromwhichweekofgestation
a)5weeks
b)15weeks
c)25weeks
d)35weeks
CorrectAnswer-A
Answer-A.5weeks
Cardiacoutputstartstoincreasefromthe5thweekofgestation,
reachesitspeak40-50%at30-34weeks.

1509.Strokevolumeincreasesinpregnancy
by
a)20%
b)25%
c)40%
d)45%
CorrectAnswer-B
Answer-B.25%
Hemodynamicchangesduringpregnancy
Pregnancynear
Non-pregnant
Change
term
Cardiacoutput
4.5
6.26
+40%
(lit/m1)
Strokevolume(ml)
65
75
+27%
Heartrate(per
70
85
+17%
minute)
Unaffectedormidpregnancydropofdiastolic
Bloodpressure
pressureby5-10mmHg
8-10cm
Venouspressure
20cm
+100%
(femoral)
Colloidoncotic
20
18
-14%
pressure(mmHg)
Systemicvascular
-21%
resistance
Pulmonaryvascular
-34%
rests

1510.Oxygenconsumptionincreasesin
pregnancyby
a)10%
b)20%
c)30%
d)40%
CorrectAnswer-B
Answer-B.20%
Oxygenconsumptionincreasesapproximately20%during
pregnancy,anditisapproximately10%higherinmultifetal
gestation.
Duringlabour,oxygenconsumptionincreases40-60%.

1511.Whichofthefollowingaboutabdominal
pregnancyistrue?
a)Primaryabdominalpregnancyismorecommon
b)Ifplacentaisdenselyadhered,itshouldbeseparatedalongwith
theorganitoverlies
c)Around50%ofthefetusreachfulltermandsurvive
d)Ifplacentaisleftbehind,infectioncanoccur
CorrectAnswer-D
Answer-D.Ifplacentaisleftbehind,infectioncanoccur
AbdominalPregnancy
A)PrimaryAbdominalPregnancy
CriteriatodiagnoseprimaryabdominalpregnancybyStudiford:
1. Boththetubesandovariesarenormalwithoutevidenceofrecent
injury
2. Absenceofuteroplacentalfistula
3. Presenceofapregnancyrelatedexclusivelytotheperitoneal
surfaceandyoungenoughtoeliminatethepossibilityofsecondary
implantationfollowingprimarynidationinthetube
B)Secondary
Almostalwayssecondary,theprimarysitesbeingtube,ovaryor
eventheuterus-theconceptusescapesoutthroughtherentinthe
uterinescar.

1512.45yrsfemalewithG5P4A0L4withLMP
25.8.15,gestationalagewillbehow
manyweeksondate11.5.15?

a)32weeks
b)35weeks
c)36weeks
d)40weeks
CorrectAnswer-C
Answer-C.36weeks
ifthepatient'sLMPis25.8.15thenEDDwouldbe3.6.16.
On11.5.15,patientwouldbe23daysleftwouldforEDD,thatis3
weeksand2days.Subtractingthisfrom40weeks,thepatients
gestationalagewouldbe36weeksand5days.

1513.

Incidenceofscarruptureinasubsequent
pregnancyincaseofLowerSegment
CesareanSection(LSCS)is:

a)2%
b)4%
c)5%
d)8%
CorrectAnswer-A
2%
Theriskoflowersegmentscar-ruptureislow(0.2-1.5%)andevenif
itdoesoccur,maternaldeathismuchlessandtheperinatal
mortalityisabout1in8.

1514.Whichofthefollowingisnotaprobable
signofpregnancy
a)Jacquemier'ssign
b)Dalrymplesign
c)Hegar'ssign
d)Palmer'ssign
CorrectAnswer-B
Answer-B.Dalrymplesign
Probablesignsofpregnancy:

Jacquemier'ssign/Chadwick'ssign
Osiander'ssign
Goodell'ssign
Piskacek'ssign
Hegar'ssign
Palmer'ssign
Braxtor-hickscontraction
Externalballotment
Abdominalenlargement
Outliningthefetus

1515.Whichofthefollowingovariantumoris
mostpronetoundergotorsionduring
pregnancy:

a)Serouscystadenoma
b)Mucinouscystadenoma
c)Dermoidcyst
d)Thecaluteincyst
CorrectAnswer-C
Ans.isci.e.Dermoidcyst
"Abenigncysticteratomaisthemostcommonneoplasmtoundergo
torsion,andittotheM/Cbenigntumordiagnosedduring
pregnancy."
Abenigncysticteratomaissynonymoustodermoidcyst.
Remember:
Mostcommonovariantumourinpregnancyisserouscystadenoma
butmostlyremainsundiagnosed.
Incidenceofdermoidcystisincreasedtwofoldduringpregnancy
anditisthemostcommonneoplasmdiagnosedduringpregnancy.
Hingoranisignseen:-Inovariantumorduringpregnancy.
Trendelenburg'spositioncanelicitthegroovebetweentwoseparate
swelling.Usedtodistinguishbetweenovariantumourandfibroid.

1516.Whatisthepreferredtreatmentof
completeprolapseinafemalewith
completedfamily?

a)Slingsurgery
b)Vaginalhysterectomy
c)LeForte'srepair
d)Pessary
CorrectAnswer-B
Answer-B.Vaginalhysterectomy
Vaginalhysterectomyiscommonlyperformedformajordegree
uterineprolapseinafemalewithcompletedfamily.

1517.InMRKHsyndrome,whichamongthe
followingisabsent?
a)Vagina
b)Breastdevelopment
c)Pubichairdevelopment
d)Testes
CorrectAnswer-D
Answer-D.Testes
MRKHsyndromeisalsoknownasMURCSsyndrome(Mullerian
agenesis,Renalaplasiaandcervicothoracicsomitedysplasia).
MRKH(MayerRokitanskyKusterHauser)syndromehasa
karyotypeof46,XX.Sothegonadspresentareovariesincontrast
toAndrogenInsensitivitySyndromewheretestesarepresent.
TheMRKHsyndromeischaracterizedbycongenitalaplasiaofthe
uterusandtheupperpart(2/3)ofthevaginainwomenshowing
normaldevelopmentofsecondarysexualcharacteristicsanda
normal46,XXkaryotype.

1518.Ifthesymphysiofundalheightis40cm
andthestationoftheheadisat-1,
weightofthefetusisapproximately

a)3kg
b)3.3kg
c)4kg
d)4.3kg
CorrectAnswer-D
Answer-D.4.3kg
Johnson'sFormulaforestimationoffetalweight:
Heightoftheuterusabovethesymphysispubisincentimeters
minus12,ifthevertexisatorabovethelevelofischialspinesor
minus11,ifthevertexisbelowthelevelofischialspines-multiplied
by155givestheweightingrams.
Solution:(40-12)x155=4340gms

1519.Whichofthefollowingistrueregarding
precociouspuberty:
a)Sexualmaturityisattainedearly
b)Mentalfunctionisincreased
c)Noreproductivefunction
d)Bodyproportionsareenlarged
CorrectAnswer-A
Ans.isai.e.Sexualmaturityisattainedearly
Precociouspubertyistheappearanceofappropriatesecondary
sexualcharactersbeforetheageof8yearsandoccurrenceof
menstruationbefore10yearsofchronologicalage.
Alsoknow:Delayedpuberty:isconsidereddelayedwhenthe
secondarysexualcharactersdonotappearbytheageof14,and
menarcheisnotestablishedby16yearsofage.

1520.DuringwhichgestationalageispeakserumHCGlevelsattained?
a)7-9weeks
b)11-13weeks
c)20weeks
d)25weeks
CorrectAnswer-A
HCGrisesprogressivelyfromconception.Levelsdoubleontheaverage,every30.9hours
untilvaluesreach6500mIU/ml(6,500IU/L)atapproximatelytheeighthweekafterthelast
menstrualperiod(LMP).
Afterthattherateofrisebecomesindividualized,peakingbetweenthe60thand70th
day(9to10weeks)LMP
.
HCGdecreasesslightlybetweenthe12thand16thweekpostLMP,andthenremains
constantuntilbirth.

1521.Cephalicindexis
a)BPD/OFD
b)BPD/HC
c)OFD/BPD
d)HC/FL
CorrectAnswer-A
Answer-A.BPD/OFD
Cephalicindex=BPD/OFD(Biparietaldiameterdividedbythe
occipito-frontaldiameter)

1522.Theroleofhumanplacentallactogenis
:
a)Stimulatemilkproduction
b)Fetalbreastdevelopment
c)Growthoffetus
d)Endocrineregulation
CorrectAnswer-C
Growthoffetus
Functions:
a)Providefetalnutritionbyantagonizingtheactionofinsulinin
maternalcirculation,breakdownoffatsandproteinsandtransportof
fattyacidsandaminoacidsfrommaternaltofetacirculation.
b)Potentangiogenic,helpsdevelopfetalvasculature.
c)Promotesgrowthofbreastforlactation.

1523.Hotflushesareexperiencedasaresult
of
a)Increasednoradrenaline
b)Decreasedestrogen
c)Increasednoradrenalineanddecreasedestrogen
d)Increasednoradrenalineandestrogen
CorrectAnswer-C
Answer-C.Increasednoradrenalineanddecreasedestrogen
Hotflushesarecausedbynoradrenaline,whichdisturbsthe
thermoregulatorysystem.Oestrogendeficiencyreduces
hypothalamicendorphins,whichreleasemorenorepinephrineand
serotonin.Thisleadstoinappropriateheatlossmechanism.
Othercausesthatcanbeassociatedwiththesymptomofhot
flushesinclude:thyroiddisease,epilepsy,pheochromocytoma,
carcinoidsyndromes,autoimmunedisorders,mastcelldisorders,
insulinoma,pancreatictumoursandevenleukemias.

1524.Kallmanssyndromeisassociatedwith
allofthefollowingexcept
a)Amenorrhea
b)ExcessstimulationoftheHPOaxis
c)Geneticmutation
d)Anosmia
CorrectAnswer-B
Answer-B.ExcessstimulationoftheHPOaxis
WhencongenitalGnRHdeficiencyisassociatedwithanosmiaor
hyposmia(anabsentorgrosslyimpairedsenseofsmell),the
disorderisknownasKallmann'ssyndrome.
TwogeneticmutationsassociatedwithKallman'sSyndrome:
1. KALgene-X-linkedinheritance(Xp22.3)encodinganosmin-1.
2. GeneencodingFGFR1(Fibroblastgrowthfactor-1receptor)-
autosomaldominantform.
Anosmin-1isaneuraladhesionmoleculethatpromotesmigration
ofGnRHneuronsandolfactoryneurons,fromtheolfactoryplacode
intothehypothalamusduringembryonicdevelopment.

1525.PatientwithNTD,doseoffolicacidin
nextpregnancy
a)0.5mg
b)1mg
c)2mg
d)4mg
CorrectAnswer-D
Answer-D.4mg
Folicacidsupplementation4mgdaily1monthbeforeconceptionto
about12weeksofpregnancy.

1526.AbsolutecontraindicationforIUD(Intra
UterineContraceptiveDevice)areall
except?

a)Pregnancy
b)Undiagnosedvaginalbleeding
c)Pelvicinflammatorydisease
d)Uterinemalformation
CorrectAnswer-D
Ans.is'di.e.,Uterinemalformation
Contraindications
ABSOLUTE:
a)Suspectedpregnancy
b)Pelvicinflammatorydisease
c)Vaginalbleedingofundiagnosedetiology
d)Cancerofthecervix,uterusoradnexaandotherpelvic
tumours
e)Previousectopicpregnancy
RELATIVE:
a)Anaemia
b)Menorrhagia
c)HistoryofPID(PelvicInflammatoryDisease)sincelast
pregnancy
d)Purulentcervicaldischarge
e)Distortionsoftheuterinecavityduetocongenital
malformations,fibroid
f)Unmotivatedperson

1527.G2P1L1femalewith1:4antiDtitresat
28weeksgestation,managementis
a)MCADoppler
b)Caesareansection
c)Inductionoflabour
d)Amniocentesis
CorrectAnswer-A
Answer-A.MCADoppler
IfIndirectCoomb'stestispositiveinanantenatalpatientwithRh
negativebloodgroup,andantibodytitres>1:16orAblevel>10
IU/ml
1. SerialMCADoppler,every1-2weeksfrom20weeks
2. Serialultrasonographyevery2-3weeksfrom20weeks


1528.AllareofvalueinmodifiedBishop
scoreexcept
a)Dilatation
b)Effacement
c)Cervicallength
d)Consistency
CorrectAnswer-B
Answer-B.Effacement

1529.Antimullerianhormoneissecretedby?
a)Granulosacells
b)Lyedigcells
c)Sertolicells
d)None
CorrectAnswer-C
Ans.is'c'i.e.,Sertolicells
AntimullerianhormoneorMullerianinhibitingsubstanceissecreted
bysertolicells.

1530.Numberofstemvilliatterminhuman
placentais
a)60
b)120
c)240
d)480
CorrectAnswer-A
Answer-A.60
Functionalsubunitiscalledalobulewhichisderivedfromatertiary
stemvilli.About60stemvillipersistinhumanplacenta.Thuseach
cotyledon(total15-29)contains3-4majorstemvilli.Thevilliare
thefunctionalunitoftheplacenta.Thetotalvillisurface,for
exchange,approximatelyvariesbetween10to14squaremetres.
Thefetalcapillarysystemwithinthevilliisalmost50kmlong.

1531.Uterusisreceptiveforimplantationfor
howmanydaysafterfertilization-
a)6days
b)12days
c)6weeks
d)12weeks
CorrectAnswer-D
Answer-D.12weeks
Superfetationisthefertilizationoftwoovareleasedintwodifferent
menstrualcycles.Thenidationanddevelopmentofonefetusover
anotherfetusistheoreticallypossibleuntilthedecidualspaceis
obliteratedby12weeksofpregnancy.

1532.Humanplacentais?
a)Discoid
b)Hemochorial
c)Deciduate
d)Alltheabove
CorrectAnswer-D
Ans.is'd'i.e.,Alltheabove
Thehumanplacentais:
Discoid,becauseofitsshape.
Hemochorial,becauseofdirectcontactofthechorionwiththe
maternalbloodandDeciduate,becausesomematernaltissueis
shedatparturition.

1533.Allaretrueaboututeroplacental
circulationexcept
a)Bloodintheintervillousspaceiscompletelyreplaced3-4
timesperminute
b)Thevillidependonthematernalbloodfortheirnutrition
c)Amatureplacentahas150mlofbloodinthevillisystemand
350mlofbloodintheintervillousspace
d)Intervillousbloodflowattermis500-600mlperminute
CorrectAnswer-C
Answer-C.Amatureplacentahas150mlofbloodinthevilli
systemand350mlofbloodintheintervillousspace
Amatureplacentahasavolumeofabout500mlofblood;350ml
beingoccupiedinthevillisystemand150mllyingintheintervillous
space.
Intervillousbloodflowattermisaround500-600mlperminute.
Thebloodintheintervilllousspaceiscompletelyreplacedabout3-
4timesperminute.
Thevillidependonmaternalcirculationfornutrition,thusitis
possibleforthechorionicvillitosurviveforavaryingperiodeven
afterthefetusisdead.

1534.Whichisincreasedinpremature
ovarianfailure:
a)Sr.Inhibin
b)Sr.FSH
c)Sr.Estradiol
d)BothAandB
CorrectAnswer-B
Answer-B.Sr.FSH
Inprematureovarianfailure:-

1. FSHlevelincreased(40mIU/mlormore).
2. Estrogenisdecreased(E2Level20pg/mlorless)
3. InhibinBisdecreased.
[RefShaw'sGynaecology10h/ep.74;SperoffsClinical
GynaecologicEndocrinologyandInfertility5th/ep.463]

1535.Allofthefollowingaremarkersof
ovarianreserveexcept-
a)InhibinA
b)Estradiolconcentration
c)InhibinB
d)Ovarianvolume
CorrectAnswer-A
Answer-A.InhibinA
BasalFSHandEstradiolconcentration
ClomipheneCitrateChallengeTest
InhibinB
Antimullerianhormone
Antralfolliclecount
Ovarianvolume

1536.Mostcommonsiteinvolvedingenital
TB-
a)Fallopiantubes
b)Endometrium
c)Ovaries
d)Vulvo-vaginalpart
CorrectAnswer-A
Answer-A.Fallopiantubes
FallopianTubes90-100

1537.WhichisnotasideeffectofPOP
[Progestinonlypill]
a)Ovariancysts
b)Venousthromboembolism
c)Increasedriskofdiabetesmellitus
d)Ectopicpregnancy
CorrectAnswer-B
Answer-B.Venousthromboembolism
Adverseeffectsofprogestinonlypill(minipill)
Mensturalirregularities
Headache,nausea,dizziners
Bloatingorweightgain
IncreasedriskofINDDM
Ovariancysts
Breasttenderness
Acne
Ectopicpregnancy

1538.Maternalageisnotassociatedwith-
a)Pretermlabour
b)Postmaturity
c)Aneuploidy
d)Hydatidiformmole
CorrectAnswer-A
Answer-A.Pretermlabour
Recentevidencehoweversuggestslowestincidenceofpreterm
laborbetween18and35yearsofageandhigherriskinlowerand
higheragegroups.

1539.Whatistobedoneif2OCPismissed
onday17-18ofthecycle-
a)Take2pillsonthenext2days
b)Usebackupcontraceptive
c)Bothaandb
d)Continuetakingsinglepillperday
CorrectAnswer-B
Answer-B.Usebackupcontraceptive
MissingpillswhileonOCP-Management:
Missingonepill(lateupto24hours)-takethemissedpillatonceand
continuethesameregimen.
Missingtwopillsinthefirstweek(days1-7)-taketwopillsoneach
ofthenexttwodaysandthencontinuetheschedule.Extra-
precaution(backup)-alternativecontraception
Missingtwopillsinthethirdweek(days15-21)orifmorethantwo
pillsaremissedatanytime-anotherformofcontraception.Startthe
nextpackwithoutabreak.
Missinganyofthe7inactivepills-throwawaythemissedpills.Start
thenewpackasusual.

1540.Mostcommoncomplicationofdermoid
cystis-
a)CystRupture
b)Torsion
c)Malignantdegeneration
d)Noneoftheabove
CorrectAnswer-B
Answer-B.Torsion
TorsionisthemostcommoncomplicationoccurringinDermoidcyst.
Becauseofthefatcontentofthecyst,itislighterthanotherovarian
tumorsandthereforeeasilytorsioned.Almost15%ofdermoidcysts
undergotorsion.

1541.Causeofunilateraldysmenorrhea:
a)Onehornofmalformeduterus
b)Endometriosiswithunilateraldistribution
c)Smallfibroidattheuterotubaljunction
d)Alloftheabove
CorrectAnswer-D
Alloftheabove
Causesofunilateraldysmenorrhea:
Onehornofmalformeduterus
Endometriosiswithunilateraldistribution
Smallfibroidattheutero-tubaljunction.
Ovariandysmenorrhea
Rightovarianveinsyndrome
Colonicorcaecalspasm.

1542.Managementoftubalectopicpregnancy
of2.5x3cmis-
a)Medicalmanagement
b)Salpingectomy
c)Accordingtopresenceoffetalcardiacactivity
d)Observation
CorrectAnswer-C
Answer-C.Accordingtopresenceoffetalcardiacactivity
Conservativemanagement
Medicalmanagement:

1. Hemodynamicallystable
2. SerumhCGlevel<3000IU/L
3. Tubaldiameter<4cmwithoutanyfetalcardiacactivity
4. Nointraabdominalhaemorrhage
Surgicalmanagement:
1. Hemodynamicinstability
2. SerumhCGlevel>3000IU/L
3. Tubaldiameter>4cm
4. Presenceoffetalcardiacactivity

1543.Doseofdexamethasoneforfetallung
maturityis-
a)6mg
b)12mg
c)18mg
d)24mg
CorrectAnswer-A
Answer-A.6mg
Betamethasone-12mgi.m.24hoursapartfor2doses
Dexamethasone-6mgi.m.12hoursapartfor4doses.
Betamethasone-Steroidofchoice.

1544.MostcommoncauseofsecondaryPPH
is:
a)Uterineinertia
b)Retainedplacenta
c)Episiotomy
d)Cervicaltear
CorrectAnswer-B
Retainedplacenta
SecondaryPPH
Bleedingusuallyoccursbetween8thto14thdayofdelivery.
CausesoflatePPH:
Retainedbitsofcotyledonormembranes(M.C.)
Infectionandseparationofsloughoveradeepcervico-vaginal
laceration.
Endometritisandsubinvolutionoftheplacentalsite.

1545.Managementofapatientwithcomplete
placentapreviaat38weeksgestation
withoutanyvaginalbleedingis-

a)Expectantmanagement
b)MacafeeandJohnsonregimen
c)Electivecaesareansection
d)Emergencycaesareansection
CorrectAnswer-C
Answer-C.Electivecaesareansection
Asthispatientismorethan37weeksgestation,activeinterference
ismandatory.
Butasthispatientisnotactivelybleedingthereforeelective
caesareansectioncanbeplanned.

1546.Trueaboutnabothiancystisallexcept-

a)Squamousepitheliumoccludesthemouthoftheglands
b)Itisseeninchronicirritationandinflammation
c)Itisapathologyofthecervix
d)Itispre-malignant
CorrectAnswer-D
Answer-D.Itispre-malignant
Seeninchronicinflammationofcervix.
Itistheresultofblockageofmouthoftheglandsofthecervix.
Duringtheprocessofhealing,thesquamousepitheliumreplacesthe
columnarepithelium.
Theblockedglandsbecomedistendedwithsecretionandformsmall
cystswhichcanbeseenwiththenakedeye,theso-callednabothian
follicles.
Theconditionisneithermalignantnorpre-malignant.

1547.Mostcommoncauseofpelvic
inflammatorydiseaseis-
a)Sexuallytransmitteddisease
b)IUCD
c)Pelvicperitonitis
d)Puerperalsepsis
CorrectAnswer-A
Answer-A.Sexuallytransmitteddisease
Sexuallytransmitteddiseaseisthemostcommoncause.
Gonococcalandchlamydialinfectionsarethemostcommoncause.
Postabortalorpuerperalsepsis.
IUCD
Tuberculosis
Pelvicperitonitis,duetoappendicitisanddiverticulitis.

1548.Thereisoverlappingofskullsutures
whichcanbereducedwithgental
pressure.Whatisthegradeofmoulding
?

a)Grade1
b)Grade2
c)Grade3
d)Grade4
CorrectAnswer-B
Answer-B.Grade2
Therearethreegradingsofmoulding:
Grade1-thebonestouchingbutnotoverlapping
Grade2-overlappingbuteasilyseparated
Grade3-fixedoverlapping.

1549.Riskfactorforcervicalcarcinomais-
a)Smoking
b)Humanpapillomavirus
c)Lowsocioeconomicstatus
d)Alloftheabove
CorrectAnswer-D
Answer-D.Alloftheabove
HPVinfection.
Coitusbefore18years.
Multiplesexualpartners.
Deliveryofthefirstbabybeforetheageof20years.
Multiparitywithpoorbirthspacingbetweenpregnancies.
Poorpersonalhygiene.
Poorsocioeconomicstatus.
Smoking.
Immunosupressivedisease.

1550.Whatisthestageofcarcinomacervix
involvingbodyofuterus-
a)StageI
b)StageII
c)StageIII
d)StageIV
CorrectAnswer-A
Answer-A.StageI
CervicalCancer-FIGOStaging
Stage Carcinomainsitu
0
StageI Intraepithelialcarcinomaconfinedtothecervix
Stage Diagnosedonlybymicroscopy
IA
Stage Microinvasivacarcinomawithstromalinvasion<3mmin
IA1
depth&<7mminwide.
Stage Microinvasivacarcinomanotexceeding5mmindepth/7
IA2
mminwidth.
Stage Clinicallyvisibleormicroscopiclesion>IA2.
IB
Stage Clinicallesionnotexceeding4cmindiameter.
IB1
Stage Clinicallesionmorethen4cmindiameter.
IB2
Stage Extensionbeyondthecervixbutnottothepelvicwall.
II
Stage Involvementofvaginabutnotthelowerthird.
IIA

Involvementofvaginabutnotthelowerthird.
IIA
Stage Clinicallyvisiblelesionmorethen4cm.
IIA1
Stage Clinicallyvisiblelesionmorethen4cm.
IIA2
Stage Parametrialinvolvementnotreachingthepelcvicsidewall.
IIB

1551.Mostcommoncauseofdeathin
cervicalcanceris-
a)Renalfailure
b)Infection
c)Haemorrhage
d)Metastasistovitalorgans
CorrectAnswer-A
Answer-A.Renalfailure
Renalfailureisthemostcommoncauseofdeathincervicalcancer.
Renalfailureincervicalcanceroccursduetoinvolvementofureters
inStage1IIB.Itcanalsooccurduetodirectinvolvementofkidney
asinstageIVB.

1552.TreatmentoptionsforCINIIIincludeall
ofthefollowingexcept-
a)LLETZ
b)Conization
c)Hysterectomy
d)Wertheim'shysterectomy
CorrectAnswer-D
Answer-D.Wertheim'shysterectomy
Treatmentoptionsof(INIII

1. Conservativeablation:coagulation,cryoscrgery,laserablation
2. Localexcision:conization,laserconization,lleT2,LEEP,NETZ
3. Radicalexcision:Trachelectomy,hystrectomy(withorwithout
removalofvaginalcuff).
Wertheim'shysterectomyisgenerallynotdoneincasesofCINIIIif
vaginaorlymphnodesarenotinvolved.

1553.A40yearoldwomanpresentswith
abnormalcervicalcytologyonPAP
smearsuggestiveofCINIII(HSIL).The
nextbeststepinmanagementis:

a)Hysterectomy
b)ColposcopyandLEEP
c)ColposcopyandCryotherapy
d)Conization
CorrectAnswer-B
AccordingtoFIGOclassification,cervicalintraepithelialneoplasia3
(CIN3)belongtostage0.
Loopelectrocauteryexcisionproceduredoneundercolposcopic
visualizationisthemodeoftreatmentforCINIIandCINIIIlesions.
Ref:Novak's,14thEdition,Page582,583;William'sGynoecology,
1stEdition,Page635;COGDT,10thEdition,Pages841,837;
Dewhurst's,6thEdition,Pages575,574.

1554.Whatsizeofthehegar'sdilatorif
passedthroughtheinternaloscanbe
labeledascervicalincompetence?

a)4
b)6
c)8
d)10
CorrectAnswer-C
Answer-C.8
PassagewithoutresistanceandpainofNo.8Hegar'sdilatorisa
screeningtestforcervicalincompetence.

1555.Funnelingincervicogramisseenin-
a)Inlabour
b)Cervicalincompetence
c)Cervicalectopic
d)DuringTVS
CorrectAnswer-B
Answer-B.Cervicalincompetence
Cervicogramisdonefordiagnosisofcervicalincompetence.Inother
conditions,cervicogramisnotdone.

1556.Incervicalincompetencediameterof
internalosofcervixis-
a)1cm
b)1.5cm
c)2cm
d)2.5cm
CorrectAnswer-A
Answer-A.1cm
Cervicalsonography:
FunellingofthecervixwithchangesintheformofY,V,U.
Cervicallength<2.5cm.
Funnellingoftheinternalos>1cm
Speculumexamination:Detectionofdilatationofinternaloswith
herniationofthemembranes.
Cervicalindex=(Funnellength+1)/(endocervicallength).
The"+1"allowsanindextobecalculatedwhenfunellingisabsent.
Predictorsofpretermbirth:
Cervicalindex>0.52.
Cervicallength<18mm
Funnellength>9mm.
Funnelwidth>6mm.

1557.Contractedpelvisisdefinedas
shorteningofoneormoreplanesby-
a)0.5cm
b)1cm
c)1.25cm
d)1.5cm
CorrectAnswer-A
Answer-A.0.5cm
Anatomically,contractedpelvisisdefinedasshorteningofoneor
moreplanesby0.5cm.

1558.Vasapreviaisassociatedwith-
a)Marginalplacenta
b)Velamentousplacenta
c)Battledore
d)Placentaprevia
CorrectAnswer-B
Answer-B.Velamentousplacenta
Ifaleashofbloodvesselshappentotraversethroughthe
membranesoverlyingtheinternalis,infrontofthepresentingpart,
theconditioniscalledasvasapraevia.
Itisassociatedwithvelamentousplacenta.
Theunsupportedumbilicalvesselsinvelamentousplacenta,lie
belowthepresentingpartandrunacrossthecervicalos.
Ruptureofmembranesinvolvingtheoverlyingvesselsleadsto
vaginalbleeding.
Asitisentirelyfetalblood,thismayresultinfetalexsanguination
andevendeath.
[RefDutta'sObstetrics8th/ep.301]

1559.Firstsymptominvulvalcanceris-
a)Pain
b)Pruritis
c)Ulcer
d)Blooddischarge
CorrectAnswer-B
Ans.B.Pruritis
WomenwithWNandyulvarcancercommonlypresentwithpruritus
andavisiblelesion.
However,pain,bleeding,andulcerationmayalsobeinitial
complaints'

1560.Mostcommontypeofconjointtwinis-
a)Thoracopagus
b)Omphalopagus
c)Craniopagus
d)Rachipagus
CorrectAnswer-A
Answer-A.Thoracopagus
Fourtypesoffusionmayoccur:
Thorapagus-Mostcommon
Pyopagus(posteriorfusion)
Craniopagus(cephalic)
Ischiopagus(caudal)

1561.Uterineheightisgreaterthan
gestationalageofthepatientinacase
ofallexcept-

a)Fibroiduterus
b)IUGR
c)Wrongdates
d)Polyhydramnios
CorrectAnswer-B
Answer-B.IUGR
Uterineheightgreaterthangestationalageincaseof:
Wrongdates
Polyhydramnios
Pregnancywithfibroiduterus
Multiplepregnancy

1562.Pregnancyiscontraindicatedinwhich
cardiacdisease-
a)Mitralstenosis
b)Primarypulmonaryhypertension
c)VSD
d)Mitralregurgitation
CorrectAnswer-B
Answer-B.Primarypulmonaryhypertension
Placeoftherapeuticterminationincaseoffollowingheartdiseases.
Consideringhighmaternaldeaths,absoluteindicationsare:
Primarypulmonaryhypertension
Eisenmenger'ssyndrome
Pulmonaryveno-occlusivedisease.

1563.Followingdelivery,tearinvolves
perineum,externalanalspincterwith
intactmucosa,gradeoftearis-

a)Firstdegree
b)Seconddegree
c)Thirddegree
d)Fourthdegree
CorrectAnswer-C
Answer-C.Thirddegree
Thirddegree:Injurytoperineum,involvingtheanalsphincter
complex(boththeexternalandinternal).
3a:ifhalfthicknessexternalanalsphincterinvolved.
3b:iffullthicknessexternalanalsphincterinvolved.
3c:ifinternalanalsphincterinvoloved.

1564.Notacauseofoligohydramnios-
a)IUGR
b)Renalagenesis
c)Amnionnodosum
d)Chorioangioma
CorrectAnswer-D
Answer-D.Chorioangioma
Fetal,
Chromosomalorstructuralanamolies
Renalagenesis
Obstructiveuropathy
Spontaneousruptureofmembrane
Intrauterineinfection
Drugs:PGinhibitors,ACEinhibitors
Postmaturity
IUGR
Amnionnodosum
Maternal
Hypertensivedisorders
Uteorplacentalinsufficiency
Dehydration
Idiopathic

1565.Falseaboutchorionicvinoussampling
-
a)Isusedforprentalgeneticdiagnosis
b)Isperformedonlyinsecondtrimesterofpregnancy
c)Villicollectedfromchorionfrondosum
d)Cancauselimbdeformities
CorrectAnswer-B
Answer-B.Isperformedonlyinsecondtrimesterofpregnancy
Isusedforprenatalgeneticdiagnosis.
Performedtranscervicallythrough10-12weeks(firsttrimester)and
transabdominallyfrom10weekstoterm.(Advantageover
amniocentesiswhichisperformedinsecondtrimesterofpregnancy)
Villiarecollectedfromchorionfrondosum.
Cancauseoromandibularlimbdeformitiesorlimbreductiondefects
ifperformedbefore10weeksofgestation.

1566.A28yearoldprimigravidawith32
weeksofgestationcomeswith
complainofthin,frothy,profuse
dischargethroughthevaginasince
yesterday.ShewasadvisedUSGwhich
showedSingleliveintrauterine
gestationalsacwithFLandAC
correspondingtotheweeksof
gestationandAFIasadequate.Whatis
thediagnosis?

a)PPROM
b)Trichomoniasis
c)Normalfinding
d)Candidiasis
CorrectAnswer-B
Answer-B.Trichomoniasis
PatientsinfectedwithTrichomonasvaginaliscomplainofatypical
discharge,whichisthin,profuse,frothy,irritating,creamyorslight
greenincolour.
Slightgreencolourofthedischargeisoftennotcomplainedbythe
patientbutspottedbythephysician.

1567.Riskfactorsformolargestationareall
ofthefollowingexcept-
a)Orientalcountries
b)Disturbedmaternalimmunemechanism
c)Higherratioofmaternal/paternalchromosomes
d)Faultynutrition
CorrectAnswer-C
Answer-C.Higherratioofmaternal/paternalchromosomes
RiskfactorsforH.mole:

1. Orientalcountries,highestincidenceinPhilippines.
2. Teenagepregnanciesor>35years.
3. Faultynutrition.
4. Disturbedmaternalimmunemechanism.
5. Higherratioofpaternal/maternalchromosomes(not
maternal/paternal)
Highertheratio,greaterthemolarchange.Completemolesshow
2:0paternal/maternalratiowhereaspartialmoleshows2:1ratio.

1568.Poorprognosticfactorforhydatidiform
moleis-
a)Priormolarpregnancy
b)Metastasistolung
c)Nopriorchemotherapy
d)WHOscore>8
CorrectAnswer-D
Answer-D.WHOscore>8
HighRisk(PoorPrognosis):
Longdurationofdisease(>4months)
InitialserumhCGlevel>40,000mIU/ml
Brainorlivermetastasis
Failureofpriorchemotherapy
Followingtermpregnancy
WHOscore>8

1569.Drugsusedinendometriosisis
a)Combinedoralcontraceptives
b)Letrozole
c)Mifepristone
d)Alloftheabove
CorrectAnswer-D
Answer-D.Alloftheabove
Combinedoralcontraceptives-Administeredintermittentlyor
continuously,oralcontraceptivesmayalleviatethedisease.
Oralprogestogens-Exertanti-estrogeniceffectandtheir
continuousadministrationcausesdecidualizationandendometrial
atrophy.
Danazol-inhibitspituitarygonadotropins
GnRHanalogues-Downregulateandsuppresspituitary
gonadotropins.
Aromataseinhibitors(letrozole):Anti-estrogenicaction
Anti-progestin(mifepristone)

1570.Embryogetsimplantedatwhatstageof
development?
a)Twocellstage
b)Fourcellstage
c)Morula
d)Blastocyst
CorrectAnswer-D
Answer-D.Blastocyst
Afterthezygoteformation,typicalmitoticdivisionofthenucleus
occursproducingtwoblastomeres.
Twocellstageisreachedapproximately30hoursafterfertilization.
Theblastomerescontinuetodividebybinarydivisionthrough4,8,
16cellstageuntilaclusterofcellsisformedandiscalledmorula,
resemblingamulberry.
Morulaafterspendingabout3daysintheuterinetubeentersthe
uterinecavitythroughthenarrowuterineostium(1mm)onthe4th
dayinthe16-64cellstage.
Implantationoccursonthe6thdaywhichcorrespondstothe20th
dayofregularmenstrualcycle.
Implantationoccursthrough4stages:apposition,adhesion,
penetrationandinvasion.

1571.Leydigcellsoffetustestissecretes
a)hCG
b)LH
c)Testosterone
d)Mullerianinhibitingsubstance
CorrectAnswer-C
Answer-C.Testosterone
Leydigcellsinfetaltestisarethecellularsiteoftestosterone
synthesis.
Fetaltestisalsosecretesmullerianinhibitingsubstanceproducedby
sertolicellswhichactslocallyasaparacrinefactortocause
mullerianductregression.Mullerianductregressioncompletesby9
to10weeks'gestation,whichismuchbeforetestosteronesecretion
hascommenced.

1572.LeFortrepairisdonefor
a)Uterovaginaldescent
b)Vaultprolapse
c)VVF
d)RVF
CorrectAnswer-A
Answer-A.Uterovaginaldescent
LeForte'srepair
Veryelderlymenopausalwomen
Advancedprolapse
Unfitforanymajorsurgicalprocedure
[RefShaw'sGynaecology16th/ep.360]

1573.Bacteriaresponsibleforectopic
pregnancyis
a)Staphylococcus
b)Chlamydia
c)Peptostreptococcus
d)Trichomonasvaginalis
CorrectAnswer-B
Answer-B.Chlamydia
SalpingitisandPID(PelvicInflammatoryDisease)isthemost
importantriskfactorforectopicpregnancy.Chlamydiatrachomatis
infectionisthemostcommonriskfactorforPIDandsalpingitis.
[RefDutta'sObstetrics8th/ep..207]

1574.MostcommonsiteofImplantationof
tubalpregnancyis:
a)Interstitialportionoffallopiantube
b)Isthmus
c)Ampulla
d)Infundibulum
CorrectAnswer-C
Ampulla
Maximumpropensitytorupture-->Isthmicectopicpregnancy.
Minimumpropensitytorupture-3Ampullaryectopicpregnancy.


1575.

Inwhichpartoffallopiantubeectopicpregnancywillhavelongestsurvival?
a)Isthmus
b)Ampulla
c)Cornua
d)Interstitium
CorrectAnswer-D
Ans.D.Interstitium
Iftheimplantationoccursintheantimesentericborderinthe
ampulla,thepregnancymaycontinuealittlelongertime.
Earliestinterruptionoccursintheisthmialimplantationand
pregnancymaycontinueupto3-4monthsininterstitial
implantation.
Alsoknow:
Isthmicruptureusuallyoccursat6-8weeks,theampullaryoneat8-
12weeksandtheinterstitialoneatabout4months.
Ref:TextbookofObstetricsbyDCDutta,6thedition,Page181.

1576.Whichofthefollowingistrueabout
ovarianectopicpregnancy?
a)Studdifordcriteriaisusedfordiagnosis
b)Thereshouldbenorentontheovaryortube
c)Absenceofevidenceofpregnancyatanyothersitethan
ovarian
d)Conservativesurgerycanbedoneifdiagnosedearly
CorrectAnswer-D
Answer-D.Conservativesurgerycanbedoneifdiagnosed
early
Ovarianectopicisarareentitythatisdifficulttodiagnoseclinically.It
usuallydoesnotproceedpastthefirstfourweeksofpregnancy.
Spiegelberg'scriteriaisusedfordiagnosisofovarian
pregnancy.Itincludes:

1. Tubeontheaffectedsidemustbeintact.
2. Thegestationsacmustbeinthepositionoftheovary.
3. Thegestationsacisconnectedtotheuterusbytheovarian
ligament.
4. Theovariantissuemustbefoundonitswallonhistological
examination.

1577.Planeofcleavageduringplacental
separationrunsthrough
a)Betweencompactandspongylayerofdeciduabasalis
b)Betweendeciduabasalisandchorionfrondosum
c)Throughdeepspongylayerofdeciduabasalis
d)Betweenlayersofchorionfrondosum
CorrectAnswer-C
Answer-C.Throughdeepspongylayerofdeciduabasalis
Markedretractionreduceseffectivelythesurfaceareaatthe
placentalsitetoaboutitshalf.Butastheplacentaisinelastic,it
cannotkeeppacewithsuchanextentofdiminutionresultinginits
buckling.Ashearingforceisinstitutedbetweentheplacentaandthe
placentalsitewhichbringsaboutitsultimateseparation.

1578.Amselcriteriaisfor
a)Bacterialvaginosis
b)Antiphospholipidantibodysyndrome
c)Ovarianectopicpregnancy
d)HELLPSyndrome
CorrectAnswer-A
Answer-A.Bacterialvaginosis
Amselcriteria:Threeoutoffourofthefollowingshouldbe
present
Awhite/grayhomogenousdischarge
AvaginaldischargepHof>4.5
Apositiveaminetest(dropof10%potassiumhydroxideaddedto
dropofdischargeonaslideproducesfishyodour),alsoknownas
Whifftest.
Microscopydemonstratescluecells.

1579.TrueaboutKeratinocyteis?
a)Ectodermderivedcell
b)Presentonlyinbasallayer
c)Matureinbasallayer
d)Differentiateinbasallayer
CorrectAnswer-A
Ans.A.Ectodermderivedcell
Theprincipalcellsofepidermisarekeratinocytes.
Keratinfilamentisthehallmarkofkeratinocytes.Keratinocyteshas
followingfeatures:
1. Proliferate(divide)inbasallayer
2. DifferentiationoccursinstratumsPinosumandstratumgranulosum.
3. Completelymatureanddieinstratumcorneum.
So,keratinocytesarepresentinallfourlayersofepidermis,
Keratinocytesarederivedfromectoderm.

1580. Odlandbodiesareseeninwhichlayer
ofepidermis?
a)Basalcelllayer
b)Pricklecelllayer
c)Stratumgranulosum
d)Stratumcorneum
CorrectAnswer-C
Ans.C.Stratumgranulosum
Odlandbodiesaremembranecoatedgranulesinstratum
granulosum,whichcontainlipidswhichisresponsibleforbarrier
functionofthislayer.
Thislayeralsocontainsdiamondshapedkeratohyalinegranules,
whicharethecharacteristicfeaturesofthislayer.
Thesekeratohyalinegranulescontainfilaggrinproteinwhichis
responsibleforaggregationofkeratinfilaments.

1581.Anagenphaseofthehairindicates:
a)Thephaseofactivityandgrowth
b)Thephaseoftransition
c)Thephaseofresting
d)Thephaseofdegeneration
CorrectAnswer-A
Ai.e.Thephaseofactivity&growth

1582.Agirlabouttomarryhascomedonal
acne.Drugtotreatsuchacaseis:
March2013

a)Topicalantibiotic
b)Benzoylperoxide
c)Retinoids
d)Estrogen
CorrectAnswer-C
Ans.Ci.e.Retinoids
Retinoidsaremainlycomedolytic
Acne
Comedonesarecharacteristicof:Acnevulgaris
Comedonesare:
*Smallcysts,
*formedinhairfollicles,
*duetoblockageofthefollicularorifice
*bytheretentionofsebumandkeratinousmaterial
Acenvulgarisiscausedby:Obstructionofpilosebaceousduct
Nodulocysticacne(NA)
TreatmentofNA:Isotretinonin(syntheticretinoicacid)

1583.Inwhichofthefollowingphototherapy
isusefulintreatment?
a)Psoriasis
b)Tineacorporis
c)Pemphigus
d)PMLE
CorrectAnswer-A
Ans.A.Psoriasis
lndicationsforPUVAandUVB

1. Establishedmajorindications:-Psoriasis,atopicdermatitis,vitiligo,
mycosisfungoides,Polymorphiclighteruption,pompholyx.
2. Lessfrequentlytreated(lessevidenceofeffectiveness):-Pityriasis
rosea,morphea,chronicurticaria,Pityriasislichenoidchronica,
Alopeciatotalis&Universalis,Lichenplanus,Pityriasisrubrapilaris,
Granulomaannulare,Generalizedpruritis,Nacrobiosislipoidics.

1584.Acanthosisnigricansischaracterized
byallofthefollowingexcept?
a)Commoninobesepeople
b)Associatedwiththickskinwithhyperpigmentaion
c)Histologicallythereishypermelanosis
d)Maybeasignofinternalmalignancy
CorrectAnswer-C
Ans.C.Histologicallythereishypermelanosis
Acanthosisnigricansisbrowntoblackdiscolourationwhichusually
affectsbodyfoldslikeaxilla,groin,umblicus,forehead.
Acanthosisnigricansoccursinindividualsyoungerthan40yearsof
age.
Itisassociatedwithobesity(mostcommon);endocrinopathylike
insulinresistanceDM,hypothyroidism,Bloomsynd.,PCOD,and
internalmalignancye.g.gastricadenocarcinoma.
Histopathologically,Papillomatosisisthecharacteristicfeature
whereasthereisnohypermelanosis.n

1585.AfterhepatitisBvaccinationchildwith
allergicfamilyhistoryandpruritis
involvingface&convexitiesdeveloped
numerousumblicatedvesicles;which
becamepustular&haemorhagic&
crusted.After2dayschilddeveloped
highfeverandlymphadenopathy.The
diagnosisis

a)Secondaryinfectedatopicdermatitis
b)Molluscumcontagiosum
c)Eczemaherpaticum
d)Eczemavaccinatum
CorrectAnswer-C
C.i.e.Eczemaherpaticurrtx
*EczemaherpeticumorKaposi'svaricelliformeruptionresultsfrom
widespread(usually)primaryHSV-1(herpessimplexl)infectionin
skindamagedbyatopicdermatitis(eczema)Q.
*
Patientswithatopiceczemamaydevelopsevereorofacialherpes
simplexvirus(HSV)infectionQ(eczemaherpaticum),
whichmay
rapidlyinvolveextensiveareasofskin&occasionallydisseminateto
visceralorgans.Systemicacyclovirorvalacicloviristreatmentof
choice.
*Inatopicdermatitispatients,smallpoxvaccinationoreven
exposuretovaccinatedindividual,
maycauseseverewidespread
erruption(k/aeczemavaccinatum)thatresemblestoeczema

herpaticum.
*Kaposisvaricelliformeruptionsmanifesteitheras-eczema
herpeticumoreczemavaccinatum.
Causative
Disease
Virus
HSV-1(Herpes
Eczema
simplex
honimis

herpeticum virus)Q
Eczema
Vacciniavirus
vaccinatum dueto
inadvertent
vaccinationof
smallpox
withlivevirus
vaccine
Milker's
Paravaccinia/
node
Pseudocowpox

1586.Epidermalnevusfollows?
a)Blaschko'slines
b)Langer'slines
c)Vasculature
d)Lymphatics
CorrectAnswer-A
Ans.A.Blaschko'slines
Blaschko'slinescorrespondtothepathwaysfollowedby
keratinocytesmigratingfromtheneuralcrestduringembryogenesis.
LinesofBlaschkorepresentnon-randomlinesofdevelopmentof
skin.
EpidermalnevusfollowtheBlaschko'sline
TheskinlesionsthatfollowtheBlaschko'slinesare-
1. Pigmenteddisorders:-Nevusachromicus(includingHypomelanosis
ofito),Epidermalnevus(Nevussebaceous,Inflammatorylinear
verrucousnevus).
2. X-linkedgeneticskinconditions:-Incontinentiapigmenti,CHILD
syndrome.
3. Acquiredinflammatoryskinrashes:-Lichenstriatus,Lichenplanus,
lupuserythematosus.
4. Chimerism

1587.Whichofthefollowingorganismhasa
roletoplayinSeborrhicdermatitis?
a)Pityrosporumovale
b)Canidaalbicans
c)Propionibacterium
d)Noneoftheabove
CorrectAnswer-A
Ans.A.Pityrosporumovale
MalaseziafurfuroritsyeastformPityrosporumovaleplaysan
etilogicalroleinSeborrhicdermatitis."

1588.Pruritusisafeatureofwhichofthe
following?
a)Pemphigusfoliaceous
b)Pemphigusvulgaris
c)BullousPemphigoid
d)Alloftheabove
CorrectAnswer-C
Ans.C.BullousPemphigoid
Diseasescausingmildornoitching:Psonasis,Pityriasisrosea,SLE,
parapsoriasis,Secondarysyphilis,Pemphigus.
Diseasescausingmoderateitching:Contactdermatitis,Dryskin,
Bullouspemphigoid,Photosensitivity(sunburn).
Diseasescausingsevereitching:Lichenplanls,Lichensimplex
chronicus,Herpesgestationis,Mastocytosis,Dermatitis
herpetiformis,scabies,Prurigonodularis.

1589.Goekarmanregimenusedinfor
treatmentofpsoriasisis?
a)UVBpluscoaltar
b)UVBplusanthralin
c)Coaltarplusanthralin
d)UVBplusmethotrexate
CorrectAnswer-A
Ans.A.UVBpluscoaltar
Goekermanregimenwasinventedforthetreatmentofmildto
moderatepsoriasis.
Inthistherapyapplicationofcrudecoaltarapplicationfor2-10hours
isfollowedbyexposuretoUVBlight.
Treatmentisnowobsolete.
SimilarlyUVBplusanthralincombinationisknownasIngram
regimen.

1590.Amongvarioustypesofpsoriatic
arthrirtis,whichvarietyismost
common?

a)Classic
b)Oligoarticular
c)Rheumatoid
d)Spondylitis
CorrectAnswer-B
Ans.B.Oligoarticular
Psoriaticarthritis(Affects5-1Ochofpsoriaticpatients)
Classicorpolyarticulartype(16%):-affectsDIPjointswith"sausage
shapeapearance"offingers
andtoes.Nailinvolvementiscommon.
Monoarticularoroligoarticulartype(70%)mostcommontype,
affectslargejointslikeknee.
Rheumatoidtype(15%):-RAlikepresentation.Symmetrical.
AffectsPIPjointsleadingto"swanneckdeformity."
Axialtypeorspondylitis:-associatedwithHLA-B27.

1591.Drugofchoiceforbubosinapregnant
femaleis?
a)Tetracycline
b)Doxycycline
c)Ceftriaxone
d)Erythromycin
CorrectAnswer-D
Ans.D.Erythromycin
BuboesaremostcommonlycausedbyLGV.
Doxycycline/tetracyclineisthedrugofchoiceforLGVinanon-
pregnantfemale.
Inapregnantfemaleandchildrenbelow8years,itshouldbetreated
witherythromycin.

1592.Which'P'isnotafeatureoflichen
planus?
a)Polygonal
b)Polyhedral
c)Pruritus
d)Plane
CorrectAnswer-B
Ans.B.Polyhedral
5characteristic'P'oflichenplanus

1. Pruritic(itching)
2. Polygonal
3. Purple(violaceous)
4. Plane(flattopped)
5. Papuleorplaque
These5Psarethecharacteristicfeaturesoflichenplanuslesions
presentation.

1593.Inlichenplanusallthefollowingsites
areaffectedexcept?
a)Flexoraspectofupperextrimities
b)Oralmucosa
c)Nails
d)Extensoraspectofupperextrimities
CorrectAnswer-D
Ans.D.Extensoraspectofupperextrimities
Sitesofinvolvementinlichenplanus
Flexorsaspectofupperextremitiesandlegsarethemostcommon
siteinvolved.
Oralmucosainvolvementmayleadtolacypatternoflesions
Nailinvolvementmaycausepterygium(mostcharacteristic),
onychorrhexia,nailatrophy,anychia.
Hairinvolvementcausescaringalopecia
Inverselichenplanusaffectsaxillae,groin,infraaxillaryareas.

1594.Notahemorrhagiclesion?
a)Petechiae
b)Echymosis
c)Plaque
d)Noneoftheabove
CorrectAnswer-C
Ans.C.Plaque
Petechiaearesmallpinpointpurpuricmacularlesionsthatoccur
duetoextravasationofredbloodcellsfromcutaneousvesselsinto
theskin.
Echryosisarelargerbruiselikehemorrhagiclesions.Causeis
generallynoninflammatory.
Plaqueisasolidplateaulikeelevationthatoccupiesalargesurface
areaincomparisontoitsheightabovethenormalskinandhasa
diametermorethan0.5cms.Thereisnohemorrhageinvolvedas
such

1595.Maculaeceruleaisseenin?
a)Pediculosishominiscorporis
b)Pediculosiscapitis
c)Scabies
d)Lupuserythematosus
CorrectAnswer-A
Ans.A.Pediculosishominiscorporis
Maculaecerulea:
Thisisalatinnameforblue-greymacula.
Pathognomicofliceinfestation(Pediculosiscorporisand
Pediculosispthiris)
Theyarehemosiderin-stainedpurpuricspotsatthesiteofinsect
bite.
Enzymesininsectsalivebreaksdownhumanbilirubintobiliverdin,
causingthecolorchangeintheskin.

1596.Incontinentiapigmentiinvolvesall
exept?
a)Skin
b)Bones
c)Teeth
d)Heart
CorrectAnswer-D
Ans.D.Heart
Incontinentiapigmenti(alsok/aBloch-Sulzbergersyndrome)is
aX-linkeddominantgeneticdisorder.Itinvolves
Skin(100%ofaffectedpeople)
Teeth(80-90%ofaffectedpeople)
Bones(30-40%ofaffectedpeople)
CNSi.e.brain&spinalcord(30-40%ofaffectedpeople)
Eyes(25-35%ofaffectedpeople)

1597.Mostcommonsiteforatopicdermatitis
-
a)Scalp
b)Trunk
c)Poplitealfossa
d)Knees
CorrectAnswer-C
Ans.C.Poplitealfossa
Sitesofitchingpatchinatopicdermatitis
Infant;Face(especiallycheek),extensorsofforearm&legs.
Childhood&adult>Flexures(antecubitalrbitalfoxa,Poplitealfossa).

1598.Bull'seyelesionsarefoundin?
a)Erythemanodosum
b)Erythemagangrenosum
c)Erythemamultiforme
d)Erythroderma
CorrectAnswer-C
Ans.C.ErythemaMultiforme
Clinicalfeaturesoferythemamultiforme
TypicallesionofEMisaTargetlesion(lrislesionorBulle'seye
lesion)whichconsistsofthreeconcentriccomponents:-
1. Centralduskyerythma,surroundedbyvesiclebulla
2. Paleedematousring
3. Erythematoushalo

1599.

Allofthefollowingsarepartofthetreatment
ofscabiesexcept?

a)TopicalPermathrin
b)Oralivermactin
c)Oralantihistamines
d)Longtermoralsteroids
CorrectAnswer-D
Ans.D.Longtermoralsteroids
Drugsusedinscabies

1. Topical:Permethrin(drugofchoice),GBH/BHC,benzyl-benzoate,
crotamine,malathion.
2. Oral:Ivermectin.
3. Forpruritis:Antihistaminics

1600.Apregnantladycomeswithitchyhand
lesionsasshown.Herhusbandalso
getsit.Thebesttreatmentthatcanbe
givenis?

a)Permathrin
b)Ivermectin
c)GBH
d)Benzylbenzoate
CorrectAnswer-A
Ans.is'a'i.e.,Permathrin
Permethrin5%creamisconsideredthedrugofchoicefortreating
scabiesinpatients,includingpregnantwomen.
Otherscabicidesconsideredsafeforuseduringpregnancyare
sulfur5-10%inpetrolatumandcrotamiton10%.
Sulfuriseffectiveandithasagoodsafetyprofile;however,sulfur
preparationscanstainclothingandtheyareodorous.
Crotamiton10%creamisnotabsorbedpercutaneouslyandis
consideredsafeinpregnancyalthoughitisnoteffectiveasother
therapies

1601.TrueregardingBowen'sdiseaseis?
a)InsituBCC
b)Morecommonindarkskinnedpeople
c)HSVinfectionplaysarole
d)Chronicsundamageplaysarole
CorrectAnswer-D
Ans.D.Chronicsundamageplaysarole
Bowen'sDisease
Bowen'sdiseaseisSCCinsitu.
Morecommoninfairskinnedpeople.
Morecommononsunexposedareassuchashead&neckfollowed
bylimbs.
Whenarisesonglanspenis,itiscalled"erythroplasiaofQueytat."

1602.TrueaboutpemphigusvulgarisA/E:
a)Subepidermal
b)Autoimmunedisease
c)Tzancksmearshowsacanthoylticcells
d)Antibodyareformedagainstdesmogleins
CorrectAnswer-A
Ai.e.Subepidermal

1603.Whichofthefollowingischaracterized
byasolitarypainlessulceron
genitalia?

a)Herpes
b)Softchancre[chancroid]
c)Hardchancre
d)Traumaticulcer
CorrectAnswer-C
Ans.C.HardChancre
Characteristicsofvariousdifferenttypesofgenitalulcers-
Primarysyphilis(hardchancre):-punchedout,painless,non-
bleeding(avascular)ulcerwith6rminduration.
Donovanosis:-oneormore,painless,bleeding(vascular)ulcers
withinduration.
Chancroid(softchancre):-multiple,painful,bleeding
(vascular)ulcerswithnoorsoftinduration.
LGV:-single,painless,non-bleedingulcer.
Herpesgenitalis:-multiplepainful/asymptomaticulcers.

1604.LinesofBlaschkorepresent:
a)Linesalonglymphatics
b)Linesalongbloodvessels
c)Linesalongnerves
d)Linesofdevelopment
CorrectAnswer-D
ThelinesofBlaschkoaredefinedbyapatterndeterminedbynevoid
representingnonrandomlinesonthehumanskinormucosa.
BlaschkolinesorthelinesofBlaschkoarethoughttorepresent
pathwaysofepidermalcellmigrationandproliferationduringthe
developmentofthefetus.
LinesofBlaschkorepresentnon-randomdevelopmentallinesofthe
skinfundamentallydifferingfromthesystemof
dermatomes.Theyfollowa'V'shapeovertheback,'S'shaped
whorlsoverthechest,stomach,andsides,andwaryshapesonthe
head.Thelinesarebelivedtotracethemigrationofembryonic
epidermalcells.Thestripesareatypeofgeneticmosaicism.
Theselinescharacteristicallydonotfollowtheunderlyingnervous,
Vascular,muscularorlymplaticstructuresinthe
skin.LinesofBlaschkoarenotordinarilyvisibk,butarerecognized
inseveralcutaneousdisordersthatfollowthesearallelstreaks.
Ref:Syndromes:Rapidrecognitionandperioperativeimplications,
byBrunoBisonnette,Page400;Neurocutaneousdisorders:
PhakomatosisandHamartoneoplasticSyndromes,byMartino
Ruggieri,Page364.

1605.Allaretrueaboutactiniclichenplanus
except?
a)Associatedwithseverepruritus
b)Autoimmuneetiology
c)Voilaceousbrownpapules
d)Usuallyaffectsexposedareasofbody
CorrectAnswer-A
Ans.A.Associatedwithseverepruritus
ActinicLichenPlanus(lichenplanussubtropicus/tropicus/lichen
planusactinicus)
Lichenplanusisanautoimmunedisease.
Commoninspringandsummersintropicalcountries.
Usuallyaffectschildrenandyoungadults
Sunlightisconsideredtobethepredisposingfactorthereforelesions
aremorecommononsunexposedareas(face,dorsumofhand,
forearmsandarms,napeoftheneck).
Papulesarehyperpigmentedwithvoilaceous-brownwithwell-
definedhypopigmentedborder.
Pruritrusandscalingareminimal.

1606.Muehrckelinesinnailsareseenin
a)Nephroticsyndrome
b)Barrtersyndrome
c)Nailpatellasyndrome
d)Acutetubularnecrosis
CorrectAnswer-A
Ans.is'a'i.e.,Nephroticsyndrome
Muehrcke'slinesarecharacteristicofhypoalbuminemia.Nephrotic
syndromecauseshypoalbunemia.

1607.Whichofthefollowingdrugcanleadto
pemphigus?
a)Penicillamine
b)Isoniazid
c)Carbamazepine
d)Furosemide
CorrectAnswer-A
Ans.A.Penicillamine
Drugscausingpemphigus-
Penicillamine
PenicillinsandCefalosporins
Captopril,
Iodine,
Lithium
Phenophthelin,
Propanolol
Rifampicine
Phenytoin,
Phenylbutazone,
Salicylates,
Piroxicam,
Sulphonamides

1608.Periungualdesquamation,whichisa
characteristicfeatureofKawasaki
syndrome,occursat?

a)1st-2ndweek
b)2nd-3rdweek
c)3rd-4thweek
d)4th_5thweek
CorrectAnswer-B
Ans.B.2nd-3rdweek
Periungualdesequamationinkawasakidiseasestartsbetween10-18
days.

1609.Armtonguetimeis?
a)13secs
b)15secs
c)20secs
d)40secs
CorrectAnswer-A
Ans.A.13secs
Armtotonguetimeismethodforknowingthecirculationtimei.e.
timetakenbyaparticleinthebloodtoflowfromonepointin
circulationtoother.
Itmeasuresthelinearvelocityofblood.
Tocalculate-thearmtotonguetime5mlof2%ofdecholineis
injectedintocubitalvein.
Assoonasdrugreachesthetongue,patientfeelsabittertaste.
Thetotaltimetakenfromarmtotongueis13seconds.
Similarlyarmtolungtimeiscalculatedwiththehelpofether.Itisd
seconds.

1610.Respiratoryfailureinapostoperative
patientis?
a)Type1
b)Type2
c)Type3
d)Type4
CorrectAnswer-C
Ans.C.Type3
Type3orperioperativerespiratoryfailure
Increasedatelectasisduetolowfunctionalresidualcapacity(FRC)
inthesettingofabnormalabdominalwallmechanics.
OftenresultsintypeIortypeIIrespiratoryfailure.
Canbeamelioratedbyanestheticoroperativetechnique,posture
posture,incentivespirometry,post-operativeanalgesia,attemptsto
rowerintra-abdominal-pressure.

1611.Laryngealmaskairway[LMA]is
contraindicatedin?
a)Difficultairways
b)Ocularsurgeries
c)Pregnantfemale
d)InCPR
CorrectAnswer-C
Ans.C.Pregnantfemale
ContraindicationsofLMA

1. Conditionswithhighriskofaspiration.i.e.,fullstomachpatients,
hiatushernia,pregnancy.
2. Oropharyngealabscessormass(tumor).
3. Massivethoracicinjury
4. Massivemaxillofacialtrauma

1612.Allofthefollowingareadvantagesof
LMAexcept?
a)Morereliablethanfacemask
b)Preventaspiration
c)AlternativetoEndotrachealintubation
d)Doesnotrequirelaryngoscope&visualization
CorrectAnswer-B
Ans.B.Preventaspiration
LMAisintermediatebetweenthefacemaskandEndotracheal
intubationintermsofreliability,
invasivenessandfacilitationofgasexchange(facemaskhas
minimumandendotrachealintubationhasmaximum).
LMAdoesnotpreventasPiration+shouldnotbeusedinfull
stomachpatients.
LMAcanbeusedasanalternativetoendotrachealintubationfor
minorsurgeries,whereanaesthetistwantstoavoidintubation.
LMAisintroducedblindly(withoutlaryngoscopy)

1613.WhichanestheticgaswasusedbyWTG
Mortoninhisexperiment?
a)Nitrousoxide
b)Ammonia
c)Diethylether
d)Trichloroethylene
CorrectAnswer-C
Ans.C.Diethylether
WilliamThomasGreenMorton,adentistandmedicalstudentat
Boston,afterexperimentingponanimals,gaveademonstrationof
generalanesthesia,In1846.

1614.Mouthtomouthrespirationprovideswhatpercentage
ofoxygen?
a)10%
b)16%
c)21%
d)100%
CorrectAnswer-B
Ans.B.16%
Mouthtomouthbreathingprovides0.8to1.2litersofexhaledairper
breathand16%ofoxygenwhichisenoughtosustainlife.
TheuseofAmbubagandroomairprovides21%O2.
TheAmericanHeartAssociationrecommendstidalvolumesof700
to1000mLduringmouth-to-mouthventilation,butsmallertidal
volumesof500mLmaybeofadvantagetodecreasethelikelihood
ofstomachinflation,asmouth-to-mouthventilationgascontainsonly
17%oxygen,but4%carbondioxide.

1615.Themostappropriatecircuitfor
ventilatingaspontaneouslybreathing
infantduringanaesthesiais?

a)jacksonReesmodificationofAyre'sTpiece
b)MaplesonAorMagill'scircuit
c)MaplesonCorWaterstoandfrocanister
d)Bainscircuit
CorrectAnswer-A
Ans.is'a'i.e.,JacksonReesmodification

1616.Bestuterinerelaxationisseenwith?
a)Chloroform
b)Nitrousoxide
c)Ether
d)Halothane
CorrectAnswer-D
Ans.'d'i.e.,Halothane
Halogenatedinhalationalanaestheticagentslikehalothaneare
powerfultocolyticagents.Halothaneisanaestheticofchoicefor
internalversionandmanualremovalofplacenta.

1617.Hepatotoxicagentis
a)Ketamine
b)Ether
c)N20
d)Halothane
CorrectAnswer-D
Di.e.Halothane

1618.Whichofthefollowingdrugsproduces
dissociativeanesthesia
a)Ketamine
b)Propofol
c)Thiopentone
d)Enflurane
CorrectAnswer-A
Ai.e.Ketamine

1619.Whichoneofthefollowingagents
sensitizesthemyocardiumto
catecholamines-

a)Isoflurane
b)Ether
c)Halothane
d)Propofol
CorrectAnswer-C
Ans.is'c'i.e.,Halothane
oHalothanetendstosensitizethehearttoarrhythmogenicactionof
adrenaline-
contraindicatedinpheochromocytoma.

1620.Anestheticagent(s)safetouseinTICP
a)Halothane
b)Thiopentone
c)Ketamine
d)Ether
CorrectAnswer-B
Bi.e.Thiopentone
Anestheticagentssafetouseinraisedintracranialpressure(ICP)
arethiopentone,propofol&etomidateQ

1621.Ketaminecanbeusedinallofthe
situationsexcept
a)Statusasthamaticus
b)Foranalgesia&sedation
c)Obstetrichemorrhage
d)Ischemicheartdisease
CorrectAnswer-D
Di.e.Ischemicheartdisease
KetaminecausesprofoundanalgesiaQ,disociative
anesthesiaQ,
andemergencepsychomimetichallucinationsand
deliriumQ.
Ketamineincreasescerebralmetabolism,02consumption,blood
flow&intracranialpressure.Q
Ketamine
*ItcausesprofoundanalgesiaQanddissociativeanesthesiaQ
(i.e.
patientappearsconsciouse.g.eyeopening,swallowingbut
unabletoprocessorrespondtosensoryinput).Ketamine
causesDissociativeAnesthesiaQbyactingoncortexand
subcorticalareas(notonRAS)l/tfeelingofdissociationfromones
ownbodyandsurroundings.PostOperativeDeliriumand
HallucinationQ
ispartofthisDissociativephenomenon.Drugof
choiceforpostop.delirium&hallucinationisLorazepam.Q
*Itisclosesttobeingacompleteanestheticsinceitinduces
analgesia,amnesia&unconsciousness.
*Itisassociatedwithemergencepsychotomimeticsideeffects
(delirium,illusions,hallucination)Q.
Itislesscommoninchildrenand
pretreatmentwithlorazepam(drugofchoice)Q.

1622.Whichofthefollowinganestheticagent
isapotentbronchodilator-
a)Propofol
b)Ketamine
c)Thiopentone
d)Methoxytone
CorrectAnswer-B
Ans.B.Ketamine
Ketamineisapotentbronchodilator,thereforeitisthei.v.
anaestheticagentofchoiceinbronchialasthmapatients.

1623.Blood:Gaspartitioncoefficientisa
measureof?
a)Potencyofanaestheticagent
b)Speedofinductionandrecovery
c)Lipidsolubilityofagent
d)None
CorrectAnswer-B
Ans.is'b'i.e.,Speedofinductionandrecovery
Minimumalveolarconcentration(MAC)-->Measureofpotency.
Blood:GaspartitioncoefficientBloodsolubilityofanaestheticagent
anddeterminesthespeedofinduction&recovery.
Oil:Gaspartitioncoefficient-4Lipidsolubilityofanaestheticagent
andisrelatedtopotencyofanaestheticagent.

1624.Attheendofanaesthesiaafter
discontinuationofnitrousoxideand
removalofendotrachealtube,1000/0
oxygenisadministeredtothepatientto
prevent:

a)DiffusionHypoxia
b)Secondgaseffect
c)Hyperoxia
d)Bronchospasm
CorrectAnswer-A
Ai.e.DiffusionHypoxia
(Ref:Willer8/ep656,3401)
OndiscontinuationofN2Oadministration,nitrousoxidegascan
diffusefrombloodtothealveoli,dilutingO2inthelung.
Produceaneffectcalled"Diffusionalhypoxia".
Toavoidhypoxia,100%O2,ratherthanairshouldbeadministered
whenN2Odiscontinued.

1625.Landmarkforpudendalnerveblockis?
a)Ischialtuberosity
b)Iliacspine
c)Sacroiliacjoint
d)Noneoftheabove
CorrectAnswer-A
Ans.A.Ischialtuberosity
Pudendalblock:
Whenperformingatransvaginalpudendalnerveblock,theischial
spineisthepalpatedthroughthewallofthevaginaandtheneedleis
thenpassedthroughthevaginalmucousmembranetowardthe
ischialspine.
Inaperinealpudendalnerveblock,theischialtuberosityispalpated
throughthebuttockandtheneedleisinsertedintothepudendal
canalaboutoneinchdeepmedialtotheischialtuberosity.

1626.Postduralpunctureheadacheusually
presentswithin?
a)0-6Hrs
b)6-12Hrs
c)12-72Hrs
d)72-96Hrs
CorrectAnswer-C
Ans.C.12-72Hrs
PostduralpunctureheadacheisduetoCSFleak.
Typicallocationisbifrontaloroccipital.
Headachegetsworsenonsittingoruprightpostureandisrelieved
bylyingdownpositionandabdominalpressure).
Thehallmarkofpostduralpunctureheadachei.e.,associationwith
bodyposition.
Theonsetofheadacheisusually12-72hrsfollowingtheprocedure,
however,itmaybeseenalmostimmediately.
Inmostcasesitlastsfor7_10days.

1627.Notincludedinneuraxialblock?
a)Spinalblock
b)Epiduralblock
c)Bier'sblock
d)Caudalblock
CorrectAnswer-C
Ans,C.Bier'sblock
Centralneuraxialblock,asthenamesuggests,isthepertainsto
localanaestheticsplacedforwaroundthenervesofthecentral
nervoussystem.
Examplesarespinalanaesthesia,Epiduralanaesthesiaand
caudallaesthesia.
BIER'sblockorintravenousregionalanesthesia(IVRA)isaformof
regionalanesthesiausedmostoftenforsurgeryoftheforearmand
hand.

1628.Contraindicationtoneruaxialblockis?
a)Hypertension
b)Renaldisease
c)Clottingdisorders
d)Diabetes
CorrectAnswer-C
Ans.C.Clottingdisorders
Contraindicationsofcentralneuroaxialblock:
Absolute:-Infectionatthesiteofinjection,patientrefusal,
coagulopathyorotherbleedingdisorder,severehypovolemia,
increasedICT,severeaorticormitralstenosis.
Relative:-Sepsis,unco-operativepatient,pre-existingneurological
deficit,demyelinatinglesions,severespinaldeformity,stenotic
valvularheartdisease

1629.Longactinglocalanaesthetic?
a)Procaine
b)Lignocaine
c)Prilocaine
d)Dibucaine
CorrectAnswer-D
Ans.is'd'i.e.,Dibucaine

1630.Shortestactinglocalanestheticagentis
a)Procaine
b)Lidocaine
c)Tetracaine
d)Bupivacaine
CorrectAnswer-A
Ai.e.Procaine

1631.Mostcommoncauseofmaternal
mortalityinspinalanesthesiais?
a)Allergytolocalanesthesia
b)Nerveinjury
c)Highblock
d)Hypotension
CorrectAnswer-C
Ans.C.Highblock
Mostcommoncauseofmaternaldeathorbraindamageinneuraxial
anesthesiaclaimswashighblock"

1632.Caudaequinasyndromeisassociated
with?
a)Lidocaine
b)Halothane
c)N20
d)Ether
CorrectAnswer-A
Ans.A.Lidocaine
Caudaequinasyndromemayoccurasararedevastating
complicationofspinalanesthesiaduetomaldistributionofinjected
localanesthesiaaroundcaudaequinanerverootsresultinginloss
offunctionofthelumbarplexus.
Associatedwithalllocalanestheticlikelidocaine,bupivacaine,
chlorprocaineetc.

1633.Localanaestheticinjecteddirectlyinto
thetissue?
a)Infiltrationanaesthesia
b)Nerveblock
c)Fieldblock
d)Bier'sblock
CorrectAnswer-A
Ans.is'a'i.e.,Infiltrationanaesthesia
oInfiltrationanaesthesiaistheinjectionoflocalanaestheticdirectly
intotissuewithouttakingintoconsiderationthecourseofcutaneous
nerve.

1634.Compositionofsodalimeis:
NaOH-Ca(OH)2-KOH-Moisture
a)4%-90%-1%-5%
b)4%-80%-1%-15%
c)80%-10%-5%-5%
d)15%-80%-1%-4%
CorrectAnswer-B
Sodalimeisthemorecommonabsorbentandiscapableofabsorbingupto23LofCO2
per100gofabsorbent.
Itconsistsprimarilyofcalciumhydroxide(80%),alongwithsodiumhydroxide(4%),water,
andasmallamountofpotassiumhydroxide(1%).
Commercialsodalimehasawatercontentof14%to19%.

CO2absorbents(eg,sodalimeorcalciumhydroxidelime)containhydroxidesaltsthatare
capableofneutralizingcarbonicacid.Reactionendproductsincludeheat(theheatof
neutralization),water,andcalciumcarbonate.
Ref:ButterworthIVJ.F.,ButterworthIVJ.F.,MackeyD.C.,WasnickJ.D.,MackeyD.C.,
WasnickJ.D.(2013).Chapter3.BreathingSystems.InJ.F.ButterworthIV,J.F.Butterworth
IV,D.C.Mackey,J.D.Wasnick,D.C.Mackey,J.D.Wasnick(Eds),Morgan&Mikhail's
ClinicalAnesthesiology
,5e.

1635.
Suxamethoniumactsthroughwhich
channels?
a)Sodiumchannels
b)Potassiumchannels
c)Calciumchannels
d)Chloridechannels
CorrectAnswer-A
Ans.A.Sodiumchannels
Suxamethonium(succinylcholine)isadepolarizingneuromuscular
blockingagentwhichactbyopeningtheNa+channelsatmuscle
endPlate.

1636.Onlyavailabledepolarizingmuscle
relaxantis?
a)Decamethonium
b)Suxamethonium
c)Mivacurium
d)None
CorrectAnswer-B
Ans.B.Suxamethonium
Suxamethonium(succinylcholine)anddecamethoniumcomeunder
depolarizingmusclerelaxants.
Outoftheseonlysuxamethoniumisavalaibleforclinicaluse.

1637.Allofthefollowingstatementsaboutneuromuscularblockageproduced
bysuccinylcholinearetrue,except:
a)NofadeonTrainoffourstimulation
b)Fadeontetanicstimulation
c)Noposttetanicfacilitation
d)Trainoffourratio>0.4
CorrectAnswer-B
Succinylcholineisadepolarizingneuromuscularblocker.
Withsuccinylcholinenofadingisobservedaftertrainoffourortetanicstimulation.
AllfourstimulatoryresponsesafterTOFstimulationaresuppressedtothesameextent.
Ref:NeuromuscularMonitoringinClinicalPracticeandResearchByThomasFuchs-Buder,
Page16

1638.AllofthefollowingaboutMRIare
correctexcept:
a)MRIiscontraindicatedinpatientswithpacemakers
b)MRIisusefulforevaluatingbonemarrow
c)MRIisbetterforcalcifiedlesions
d)MRIisusefulforlocalizingsmalllesioneinthebrain
CorrectAnswer-C
Ci.e.MRIisbetterforcalcifiedlesions
*MRIisverypoorindetectionofcalcification.ItisinferiortoCT
scan,mammographyandx-ray
indetectingcalcification.Thatiswhy
itlagsbehindmammographyinearlydetectionofnoninvasiveductal
carcinomainsitu(DCIS)Q,
whichmostcommonly
hasmicrocalcificationasitsonlypresentingfeature.Andsimilarlyit
hasaverylimitedroleindetectionofrenalstonesandgall
stonesQ.
However,itisimportanttonotethatonlyupto60%ofgall
stoneshaveenoughcalciumdensity(morethanthatofbile)toget
visualizedonCT.Becausofitssuperiorcalcificationdetection
abilities,MDCTisusedinAgatstonscoring(Coronarycalcium
scoring)ofcalcifiedplaquesofcoronaryarteryusingcoronary
calciumasasurrogatemarkertodetectthepresenceandmeasure
theamountofcoronaryatherosclerosis.Becausewithexceptionof
patientswithrenalfailurecalcificationofarteriesoccursexclusively
incontextofatherosclerosis.
SimilarlynonenhancedhelicalCTissuperiortoallotherimaging
modalitiesindiagnosisofurinarytractcalculiQ
butatthecostof
higherradiationexposure.
NowthereisnoneedtosaythatMRIisbetterthatCTforevaluation
ofbonemarrow,smallbrainlesions,meniscus/ligamentinjuries,
softtissuetumorsandmeningealpathology.ButMRIisverypoorin

softtissuetumorsandmeningealpathology.ButMRIisverypoorin
detectionofcalcification.

1639.Whichofthefollowingisnota
contraindicationofMRI
a)Cardiacpacemaker
b)Cochlearimplant
c)Ryle'stube
d)Metallicsplinterineye
CorrectAnswer-C
Answer-C.Ryle'stube
ContraindicationsforMRI
A)Absolute
Electronically,magnetically,andmechanicallyactivatedimplants.
Ferromagneticorelectronicallyoperatedactivedeviceslike
automaticcardioverterdefibrillators.
Cardiacpacemaker
Metallicsplintersintheeye.
FerromagnetichaemostaticclipsintheCNS.
B)Relativecontraindications
Cochlearimplants
Prostheticheartvalves
Otherpacemakers,e.g.,forCarotidsinus
Haemostaticclips
Insulinpumpsandnervestimulators
Non-ferromagenticstapedialimplants
Leadwiresorsimilarwires
Womenwithafirst-trimesterpregnancy

1640.Allaredonetominimizeradiation,
exposuretothepatientunder
fluroscopy,except

a)Decreaseinfieldofview
b)IncreasingtheKvofradiation
c)Decreasingfluroscopictime
d)Usinglowdoseofradiation
CorrectAnswer-A
Answer-A.Decreaseinfieldofview
Decreasingthefieldofviewduringfluroscopyincreasethe
radiationdoserate:-

1. Fieldofviewdiameter25cmhasdoserate0.3mGy/s
2. Fieldofviewdiameter17cmhasdose0.6mGy/s

1641.Amountofradiationexposurein1CT-
scanofchestis
a)1mSv
b)3mSv
c)5mSv
d)7mSv
CorrectAnswer-D
Answer-D.7mSv
CTchestcausesradiationexposureof7mSv.
CTabdomen-pelviscausesradiationexposureof10mSv.
CTheadcausesradiationexposureof2mSv.

1642.Whichofthefollowingisalatesevere
adverseeffectofradiationtherapy
a)Nausea
b)Erythema
c)Anemia
d)Osteoradionecrosis
CorrectAnswer-D
Answer-D.Osteoradionecrosis
Osteoradionecrosisisalatecomplicationofradiationwhilenausea,
erythemaandanemiaareearlycomplications.

1643.SubstanceusedforPETscanis
a)18FPCT
b)Gadolinium
c)Gastrogarfin
d)Iodine
CorrectAnswer-A
Answer-A.18FPCT
Fluorine-18-labeled2Ali-carbomethoxy-3AY-(4-chloropheny1)-8-(-3-
fluoropropyl)nortropane(FPCT)hasbeensynthesizedasanew
dopaminetransporterimagingagent.
OthersubstancesusedforPETscanare:
1. FDG
2. 64Cu-ATSM(4Cudiacetyl-bis(N4-methylthiosemicarbazone)
3. 18F-fluoride
4. FLT(3'-deoxy-3'-[18F]fluorothymidine)
5. FMISO(18F-fluoromisonidazole)
6. Gallium
7. Technetium-99m
8. Thallium

1644.WhichcommontracerinPETisusually
administeredintheformofaglucose
sugar

a)Oxygen15
b)Fluorine18
c)Saccharide-12
d)Aluminum-12
CorrectAnswer-B
Answer-B.Fluorine18
Fluorine-18isusedintheformofFDGinPETscan.
Themostcommontracerhasacomplicatednamebutismostly
knownasFDG(whichstandsfor2-[18F]fluoro-2-deoxy-D-glucose).
ThebiologicallyactivemoleculemostcommonlyusedforPETis2-
deoxy-2-18F-fluoro--D-glucose(18F-FDG),ananalogueof
glucose,forearlydetectionoftumors.

1645.Intraoperativeradiotherapyisusedin
a)Gastriccancer
b)Coloncarcinoma
c)Pancreaticcarcinoma
d)Alloftheabove
CorrectAnswer-D
Answer-D.Alloftheabove
IntraoperativeradiotherapycanbeusedIn-

1. Pancreaticcarcinoma
2. Retroperitonealsarcomas
3. Gastriccarcinoma
4. Genitourinarycancers
5. Colorectalcarcinoma
6. Braintumor
7. Head&Neckcancers
8. Somegynecologicalmalignancies

1646.Preciselydirectedhighdoseradiation
isusedin
a)IMRT
b)EBRT
c)Stereotecticradiosurgery
d)Noneoftheabove
CorrectAnswer-A
Answer-A.IMRT
Intensitymodulatedradiationtherapy(IMRT)isanadvancedmode
ofhighprecisionradiotherapythatutilizescomputercontrolledX-ray
acceleratorstodeliverpreciseradiationdosestoamalignanttumour
orspecificareaswithinthetumour.

1647.Stereotacticradiosurgeryisdonefor-
a)Glioblastomamultiforme
b)Medulloblastomaspinalcord
c)Ependymoma
d)AVmalformationofbrain
CorrectAnswer-D
Ans.is'D'i.e.,AVmalformationofbrain
Itisalsousedfor-
1. Solitarycerebralmetastasis
2. Arteriovenousmalformation
3. Smallmeningiomas
4. Schwannomas
5. Pituitaryadenomas

1648.Notusedforinternalradiotherapy
a)Iodine-125
b)Iodine-131
c)Cobalt-60
d)Iridium-192
CorrectAnswer-B
Answer-B.Iodine-131
lsotpesusedforinternalradiotherapy(brachytherapy)
A)Interstitialbrachytherapy
Permanentimplantsofinterstitialbrachytherapy:-Cesium-131,
Yttrium,Gold-198(Au-198),125I,Radon-222(Rn-222),
Pallidum-103(Pd-103).
TemporaryimplantsofInterstitialbrachytherapy:-Iridium-192(Ir-
192),Cesium-137(Cs-137),Cobalt-60(Co-60),Californium,
Radium-226(Ra-226),Tantolum.

1649.Radioiodinegenerateswhichtypeof
radiation
a)X-rays
b)Alphaandbetarays
c)Betaandgammarays
d)Alphaandbetarays
CorrectAnswer-C
Answer-C.Betaandgammarays
Radioiodinegeneratesbothbetaandgammaraysbutpredominantly
betarays.

1650.Radioactiveiodineisadministered
throughwhichroute
a)Intravenous
b)Subcutaneous
c)Oral
d)Alloftheabove
CorrectAnswer-D
Answer-D.Alloftheabove
Radioactiveiodineisadministeredorallyaswellasparentrallyi.e.
IV,SC&IM.

1651."Treeinbudappearance"onCTisseen
in
a)Pulmonarytuberculosis
b)Silicosis
c)Pulmonaryhydatidcyst
d)Smallcellcarcinoma
CorrectAnswer-A
Answer-A.Pulmonarytuberculosis
ItisusuallyvisibleonstandardCT,however,itisbestseenonHRCTchest.
Typicallythecentrilobularnodulesare2-4mmindiameterandperipheral,within5mmof
thepleuralsurface.
Theconnectiontoopacifiedorthickenedbranchingstructuresextendsproximally
(representingthedilatedandopacifiedbronchiolesorinflamedarterioles)
AssociatedCTfindingsofbronchiolitisareseeninabout70%ofpatientswith
bronchiectasis.Smallcentrilobularnodularandlinearbranchingopacities(tree-in-budsign)
expressinflammatoryandinfectiousbronchiolitis
Pathogenesis-
Thetree-in-budsignoccursasaresultofanumberofprocesses,althoughoftentheyco-
existinthesamecondition:
a.bronchiolesfilledwithpusorinflammatoryexudate
e.g.pulmonarytuberculosis,aspirationbronchopneumonia
b.bronchiolitis:thickeningofbronchiolarwallsandbronchovascularbundle
e.g.cytomegaloviruspneumonitis,obliterativebronchiolitis
c.bronchiectasiswithmucusplugging
e.g.cysticfibrosis
d.tumorembolitocentrilobulararteries(orcarcinomatousendarteritis)
e.g.breastcancer,stomachcancer

e.bronchovascularinterstitialinfiltration
e.g.sarcoidosis,lymphoma,leukemia

1652.Investigationofchoiceforlungabscess
is
a)ChestX-ray
b)CECTscan
c)MRI
d)Ultrasound
CorrectAnswer-B
Answer-B.CECTscan
ContrastenhancedCTinusuallyconsideredtobeinvestigationof
choiceforlungabscess,showingacavitywiththickwallsand
centralmobilefluid.Ithelpstodifferentiateabscessfromempyma,
necrotizingpneumonia,sequestration,pneumatoceleorunderlying
congenitalabnormalitiessuchasbronchogeniccyst.

1653.OnCTchest'halosign'isnotedin
a)Pulmonaryhydatidcyst
b)Invasivepulmonaryaspergilossis
c)Roundpneumonia
d)Bronchiectasis
CorrectAnswer-B
Answer-B.Invasivepulmonaryaspergilossis
Thehalosign(HS)inchestimagingisafeatureseenonlung
windowsettings(typicallyHRCT),groundglassopacitysurrounding
apulmonarynoduleormassandrepresentshemorrhage.
Itistypicallyseeninangioinvasiveaspergillosis.

1654.

A50yearoldmalepresentswithfeverand
malaisefor4months&paininthekneesand
ankles.Bloodtestsarenormalapartfroma
raisedESR.Chestx-rayshowsbilateralhilar
adenopathyandpulmonaryinfiltratesmost
severeintheupperandmidzones.Mantoux
testisnegative.Whatisthemostlikely
diagnosis

a)Tuberculosis
b)Sarcoidosis
c)Asbestosis
d)Berylliosis
CorrectAnswer-B
Answer-B.Sarcoidosis
Sarcoidosisisthemostlikelydiagnosisgiventhepresentationwith
malaise,arthralgiaandachestx-rayshowingbilateralhilar
adenopathy.
"Thecharacteristicradiologicalfindinginpatientswithpulmonary
sarcoidosisisbilateralhilarlymphadenopathy"

1655.22-year-oldwomenpresentstothe
emergencydepartmentwithachief
complaintofsevereleftupperquadrant
[LUQ]painafterbeingpunchedbyher
husband.Herbloodpressureis110/76,
herpulseis80bpm,andherrespiration
rateis24breathsperminute.Thebest
meanstoestablishadiagnosisiswhich
ofthefollowing?

a)Four-quadranttapoftheabdomen
b)CToftheabdomen
c)Peritoneallavage
d)Uppergastrointestinal[GI]series
CorrectAnswer-B
Answer-B.CToftheabdomen
Clinicalpictureofthepatientinabovequestionindicatesthatpatient
ishemodynamicallystable.Thereforebestmodeofevaluation
shouldbeCTscanofabdomentoseetheextentofinjury.
Ifthepatientsishemodynamicallystableandcanbeshifted-CT
scan

1656.A35year-oldfemalepresentedtothe
emergencydepartmentwiththesudden
onsetofsevereepigastricpain.Shehad
ahistoryofheartburnanddyspeptic
symptomsforpast10years.On
physicalexam,shehadatemperature
of101.4?F,apulseof118andablood
pressureof128/72.Abdomenwas
tender&rigid.ExpectedfindingonX-
raywillbe

a)Bloodunderdiaphragm
b)Airunderdiaphragm
c)Hazylungfields
d)Prominentmarkings
CorrectAnswer-B
Answer-B.Airunderdiaphragm
Aboveclinicalpictureissuggestiveofperforatedpepticulcer,Which
isthemostcommoncauseofpneumoperitoneum(airunder
diaphragm).
Perforationresultsinpneumoperitoneumandthebestviewtosee
pneumoperitoneumischestx-rayinerectpositionwhichdetectsair
underthedomesofdiaphragm.

1657.A50year-oldchronicalcoholicmale
patient,afteralargebingeofalcohol,
presentedtotheemergencydepartment
insubconsciousstate.Hevomited
severaltimes,fewofthemmixedwith
blood.Hehadahistoryofheartburn
anddyspepticsymptomsforpastfew
tears.Onphysicalexam,hehada
temperatureof102?F,apulseof110,
respiratoryrateof20perminuteanda
bloodpressureof90/60.Onphysical
examinationtherewasabdominal
guardingandtenderness.Aplaneerect
chestX-rayrevealsairunder
diaphragm.Probablediagnosisis

a)Perforatedpepticulcer
b)AcuteMI
c)Dissectedabdominalaorta
d)Noneoftheabove
CorrectAnswer-A
Answer-A.Perforatedpepticulcer
Historyofheartburnanddyspepsia,acuteonsetofbloodyvomiting
afterbingealcohol,generalandphysicalexaminationfindingsand
airunderdiaphragmonchestX-ray,alltheseindicatetowardsthe

airunderdiaphragmonchestX-ray,alltheseindicatetowardsthe
perforationofpepticulcer.

1658.RingenhancinglesiononCTisa
featureof
a)Toxoplasmsis
b)Intracranialhemorrhage
c)Cysts
d)Hamartoma
CorrectAnswer-A
Answer-A.Toxoplasmsis
Ringenhancingbrainlesions

1. Neoplasms:-Highgradeglioma,meningioma,lymphoma,acoustic
schwannoma,craniopharyngioma,metastasis.
2. Abscess:-Pyogenic,tuberculoma,toxoplasmosis,cysticercosis,
empyema.
3. Hemorrhagic-ischemiclesion:-Resolvinginfarction,Aging
hematoma,operativebedfollowingresection.
4. Demyelinatingdisorder

1659.Aboutlipoma,radiologicallytrueis-
a)LowattenuationonCT
b)AnechoiconUS
c)Hypo-intenseonT1-MRI
d)Hypo-intenseonT2-MRI
CorrectAnswer-A
Answer-A.LowattenuationonCT
Lipomasarebenignfattumorswhichshow
VariableechosonUS
Hyper-intenseshadowsonbothT1&T2MRI
LowattenuationonCT.

1660.FollowingX-rayfindingisassociated
withChilaiditisyndrome-
a)Pseudopneumoperitoneum
b)Pseudopneumothorax
c)Pneumothorax
d)Hydropneumothorax
CorrectAnswer-A
Answer-A.Pseudopneumoperitoneum
Chilaiditisyndromeistheanteriorinterpositionofthecolon(usually
tranversecolon)totheliverreachingtheunder-surfaceoftheright
hemidiaphragmwithassociatedupperabdominalpain.
Itisoneofthecausesofpseudopneumoperitoneum

1661.WhichofthefollowingagentsisusedtomeasureGlomerularFiltration
Rate(GFR)?
a)Iodohippurate
b)Tc99m-DTPA
c)Tc99m-MAG3
d)Tc99m-DMSA
CorrectAnswer-B
Tc99m-DiethyleneTriaminePentothenicAcid(DTPA)isanagentofchoicewidelyusedin
measuringGFR.
Ref:RenalDisease:TechniquesandProtocolsByMichaelSGoligorsky,Page87;
PaediatricUroradiologyByRichardFotter,ALbertLBaert,Page47

1662.Ultrasonographicfindingofautosomal
recessivepolycystickidneydisease
[ARPKD]areallexcept

a)Cystsmorethan2cm
b)Corticomedullarydifferentiationiseventuallylost
c)Enlargedkidney
d)Oligohydramnios
CorrectAnswer-A
Answer-A.Cystsmorethan2cm
Onantenatalultrasoundassociatedoligohydramniosmaybe
identified
Cysts
Initiallytoosmalltoresolvebutwithtimemaybecomediscernible
UnlikeADPKDthecystsrarelyexceed1-2cmindiameter
Thekidneysappearenlargedandechogenicbutusuallyretaina
reniformshape
Medullarypyramids

1663.Cobraheadappearanceonexcretory
urographyissuggestiveof:
March2010

a)Horseshoekidney
b)Duplicationofrenalpelvis
c)Simplecystofkidney
d)Ureterocele
CorrectAnswer-D
Ans.D:Ureterocele
Thetermureteroceledenotesacysticballooningofthedistalendof
theureter.Thistypeofureteroceleisalsotermedorthotopic,sinceit
arisesfromaureterwithanormalinsertionintothetrigone.
Anintravesicalureteroceleresultsfromtheprolapseofthemucosa
oftheterminalsegmentoftheureterthroughtheureterovesical
orificeintothebladder.
Thisprolapsedureteralmucosacarrieswithitaportionofthe
continuoussheetofthebladdermucosaaroundtheorifice.The
prolapsedsegmentthushasawallthatconsistsofathinlayerof
muscleandcollageninterposedbetweenthebladderuroepithelium
andtheureteruroepithelium.
Sincetheterminalureteralorificeisusuallynarrowedandpartially
obstructed,andsincethereisnomusclesupportforthedouble
mucosalwallsoftheprolapsedsegment,itdilates.Thisdilated
segmentfillswithurineandprotrudesintothebladder.
Onexcretoryurography,cobraheadsignisclassicallyseenwithan
intravesicalureterocele.

1664.CTofgastricVolvulusshows-
a)Shortenedtwistedstomach
b)Enlargedtwistedstomach
c)Normaltwistedstomach
d)Noneoftheabove
CorrectAnswer-B
Answer-B.Enlargedtwistedstomach
CTscanofgastricVolvulusshowsenlargedtwistedstomachin
thoraxwithoneormoresitesoftorsion.Itisusefulinshowingthe
sitesofischemia.

1665.Investigationofchoicetoevaluate
intracranialhemorrhageoflessthan48
hoursis-

a)CTscan
b)MRI
c)PET
d)SPECT
CorrectAnswer-A
Answer-A.CTscan
Primaryprocedureofchoiceforevaluatingintracranialcomplications
ofacuteheadinjury4CTscan.
Bestmodalityforassessingfracturesoftheskullbase,calvarium
andfacialbone4CTscan.
Investigationofchoicefordemyelinatingdisorders4MRI.

1666.Investigationofchoiceforacute
intracerebralhemorrhageis-
a)NCCT
b)MRI
c)PETscan
d)Noneoftheabove
CorrectAnswer-A
Answer-A.NCCT
"NCCTremainsthemainstayofemergencyimagingofstrokein
ordertoexcludeintracranialhemorrhage".
"Ininvestigationofstrokewithdelayedpresentationgradient-echo
MRIistheinvestigationofchoiceforexclusionofoldhemorrhage".

1667.EpiduralhematomaonCTscanshows-
a)Crescentshapedhyperdenselesion
b)Biconvexhyperdenselesion
c)Biconcavehyperdenselesion
d)Crescentshapedhypodenselesion
CorrectAnswer-B
Answer-B.Biconvexhyperdenselesion
Acuteextradural(epidural)hematomaisbiconvexhyperdenseor
mixeddensitylesion.
Extradural(epidural)hematomaappearanceradiolgically:-
Biconvex(lensshapedorlenticular)
InAcutecases4Hyperdense(2/3)ormixeddensity(1/3).
Inchroniccases4Hypodense

1668.Investigationofchoiceforacute
subarachnoidhemorrhageis-
a)MRI
b)CTscan
c)EnhanceMRI
d)Angiography
CorrectAnswer-B
Answer-B.CTscan
InvestigationofchoiceforacuteSAH-CTscan
InvestigationofchoiceforchronicSAH-MRI

1669.Sausagefingerappearanceisseenin-
a)Psoriaticarthritis
b)Rickets
c)Hyperthyroidism
d)Addison'sdisease
CorrectAnswer-A
Answer-A.Psoriaticarthritis
Thecommoncausesofsausagedigitare:
Psoriaticarthropathy.
Ankylosingspondylitis
Tuberculosis
Osteomyelitis.
Sicklecelldisease
Leprosy

1670.SkylineviewX-rayisusefulin
diagnosing-
a)Patellofemoralproblem
b)Radioulnarsproblem
c)Tibiofibularproblem
d)Skullfracture
CorrectAnswer-A
Answer-A.Patellofemoralproblem
A'Skyline'or'Sunrise'or'sunset'or'axial'or'tangential'or'mountain
view'givesmostinformationaboutpatellofemoraljoint.

1671.Investigationofchoiceforopticneuritis
is?
a)MRIBrainandorbit
b)CtscanBrainandorbit
c)Vitreousbiopsy
d)Electooculogram
CorrectAnswer-A
Ans.is'a'i.e.,MRIBrainandorbit[RefKanski8th/eChap.19,p.
784]
MagneticResonanceImaging(MRI)isfarmoresuperiorforthe
studyofsofttissueandthus,formostneuro-ophthalmicconditions,
MRIistheinvestigationofchoice.

1672.Investigationofchoiceforsofttissue
sarcomais-
a)CT
b)MRI
c)Ultrasound
d)X-ray
CorrectAnswer-B
Answer-B.MRI
"AnypatientwithasuspectedSTSshouldbereferredtoadiagnostic
centrefortripleassessmentwithclinicalhistory,imagingandbiopsy.
WhilstthepreferredmethodofimagingisMRI,otheroptions
includingcomputerizedtomography(CT)orultrasoundmaybe
appropriatedependingonlocalexpertise."

1673.TigriodpatternonMRIisseenin-
a)Wilson'sdisease
b)Metachromaticleukodystrophy
c)Parkinsonism
d)GBsyndrome
CorrectAnswer-B
Answer-B.Metachromaticleukodystrophy
Itischaracteristicallyseenin-
Metachromaticleukodystrophy
Pelizaeus-Merzbacherdisease
Autosomalrecessivespasticataxiaofcharlevoix.

1674.Theoryofhumanmotivationwasgiven
by?
a)Pavlov
b)AbrahamMaslow
c)AliosAlzheimer
d)AaronBeck
CorrectAnswer-B
Ans.B.AbrahamMaslow
Maslordshierarchyofneedsisatheoryinpsyehslagproposedby
AbrahamMaslowinhis1943paper.
"ATheoryOfHumanMotivation"inPsychologicalReview.
Maslowsubsequentlyextendedtoincludehisobservationsof
humansinnatecuriosity'.
Histheoriesparallelmanyothertheoriesofhumandevelopmental
psychology,someofwhichfocusondescribingthestagesofgrowth
inhumans.
Maslowusedtheterms"physiological","safety',"beloftingness"and
love,esteem,self-actualization",and"self-transcendence"to
describethepatternthathumanmotivationsgenerallyhavethrough.

1675.Illusionis:
September2007
a)Afalseunshakenbeliefnotkeepingone'ssociaocultural
background
b)Perceptionwithoutstimuli
c)Abnormalperceptionbyasensorymisinterpretationofactual
stimulus
d)Fearofclosedspaces
CorrectAnswer-C
Ans.C:Abnormalperceptionbyasensorymisinterpretationof
actualstimulus
Illusionisanabnormalperceptioncausedbyasensory
misinterpretationofactualstimulus,sometimesprecipitatedby
strongemotion,e.g.fearprovokingapersontoimaginetheyhave
seenanintruderintheshadows.

1676.Medicaltreatmentforparaphillia
includes?
a)SSRIs
b)Benzodiazepines
c)Opioids
d)Barbiturates
CorrectAnswer-A
Ans.A.SSRIs
Medicaltreatmentforparaphillia-
Antidepressants,suchaslithiumandvariousselectiveserotonin
reuptakeinhibitors(SSRIs)
Long-actinggonadotropin-releasinghormones(ie,medical
castration),suchasleuprolideacetateandtriptorelin
Antiandrogens(tolowersexdrive)'suchas
medroxyprogesteroneacetate(10mgqt2hr,withthedosagedoubled
every3daystoamaximumof200mg/day,thenmaintainedforI
monthandadjustedasnecessary).
Phenothiazines,suchasfluphenazine
Moodstabilizers

1677.Husbandhavingsuspicionthathiswife
ishavingaffairwithanotherman,
diagnosisis?

a)Illusion
b)Delusion
c)Hallucination
d)Delirium
CorrectAnswer-B
Ans.B.Delusion
Adelusionisfalse,firm(unshakeable)beliefthatisnotacceptedby
othermembersofpatient'scultureandsociety.
Aboveisanexampleofdelusionofinfedility(morbidjealousy,
othellosyndrome)i.e-.Falsebeliefthatone'sloverhasbeen
unfaithful.

1678.A25yearoldmalebelievesthathis
penisisdecreasinginsizeeveryday
andonedayitwilldisappearoneday
andhewilldie.Diagnosisis?

a)Obsession
b)Somatization
c)Hypochondriasis
d)Delusiondisorder
CorrectAnswer-D
Ans.D.Delusiondisorder
DescriptiongiveninthequestionisofKorosyndrome,adelusional
disorder.
Korosyndrome
Koroisaculture-specificsyndromedelusionaldisorderinwhichan
individualhasanoverpoweringbeliefthatone'sgenitaliaare
retractingandwilldisappear,despitethelackofanytrue
longstandingchangestothegenitals.
Koroisalsoknownasshrinkingpenis.Thesyndromeiscommonly
knownasgenitalretractionsyndrome.

1679.Mostcommonsubstanceofabusein
India?
a)Cannabis
b)Tobacco
c)Alcohol
d)Opium
CorrectAnswer-B
Ans.B.Tobacco
NationalHouseholdSurveyofDrugandAlcoholAbuseinIndia
?Estimatedusers:(2001)

1. Tobacco-162million.
2. Alcohol-62million
3. Cannabis-9million.
4. Opiates-2.5lakh

1680.InNeurolepticmalignantsyndrome,
causeofdeathis?
a)Respiratoryfailure
b)Liverfailure
c)Drugtoxicity
d)Noneoftheabove
CorrectAnswer-A
Ans.A.Respiratoryfailure
CausesofdeathinNMS
Respiratoryfailure
Renalfailure
Pneumonia
Thromboembolism
Cardiacfailure

1681.ThesiteoflesioninKorsakoff's
psychosisis
a)Frontallobe
b)Corpusstriatum
c)MammilaryBody
d)Cingulategyrus
CorrectAnswer-C
Ci.e.Mammilarybody

1682.Sexreassignmentsurgeryisdonein?
a)Genderidentitydisorder
b)Prematureejaculation
c)Erectiledysfunction
d)Orgasmicdysfunction
CorrectAnswer-A
Ans.A.Genderidentitydisorder
Manypersonswithgenderidentitydisorderhavesoughtsex-
reassignmentsurgery,thatis,physicalchangethatisconstantwith
theircrosssexualidentity.

1683.Allarereversiblecausesofdementia
except
a)Hypothyroidism
b)Hydrocephalus
c)Meningoencephalitis
d)Alzheimer'sdisease
CorrectAnswer-D
Ans.'D'i.e.,Alzheimer'sdisease
Reversiblecausesofdementia
1)Surgicallytreatable:-Normalpressurehydrocephalus,brain
tumors(frontallobetumor),meningioma,subduralhematomadueto
headinjury,hydrocephalus.
2)Medicallytreatable:-Depression,hypothyroidism,alcohol
abuse,vitaminB12/folate/Niacindeficiency,anymetabolicor
endocrinedisturbance,neurosyphilis,Hashimoto'sencephalopathy,
Wilson'sdisease,celiacdisease,Whipple'sdisease,chronic
meningoencephalitis,drugsandtoxin(toxicdementia).
Irreversiblecausesofdementia:-
Alzheimer'sdisease,vascular(multi-infarct)dementia,
Parkinsonism,Huntington'schorea,Lewybodydementia,Pick's
disease.

1684.Whichofthefollowingisnotassociated
withdementia:
March2011

a)Forgetfulness
b)Alterationofconsciousness
c)Reducedpersonalcare
d)Lossofneuronsinbrain
CorrectAnswer-B
Ans.B:Alterationofconsciousness
Thereisimpairmentofjudgementandimpulsecontrol,and
alsoimpairmentofabstractthinkingThereishoweverusually
noimpairmentofconsciousnessindementia(unlikein
delirium)Dementia:
Chroniconset
Nodisturbanceofconsciousness
Cognitiveimpairment
Personalityalteration
Impairedmemory
Impairedjudgment
Absentdiurnalvariation

1685.Agirlwithbadbehaviorlikesmashing
andthrowingobjectswasadmittedin
thehospital.Therealsoshewas
behavingverybadlywithstaff&
abusingnurses.Butshebehavesvery
wellwithaoneverygoodlooking
residentdoctor.Diagnosisis?

a)Bipolardisorder
b)Schizoaffectivedisorder
c)Borderlinepersonalitydisorder
d)Antisocialpersonality
CorrectAnswer-D
Ans.D.Antisocialpersonality
Antisocial(Dissocial)personalitydisorder
Theessentialfeaturesofantisocialpersonalitydisorderarea
disregardforandviolationoftherightsoftheotherandtheruIesof
thesociety.
Itischaracterizedbyrepeatedviolationofthelawandrulesofthe
society(drugabuse);physicalaggressiveness;Recklessdisregard
forsafetyofselforothers;consistentirresponsibilityinworkand
familyenvironmentandlackofremorse.
Thisdisorderissynonymouswithpreviouslyusedtermslike
psychopathorsociopath.

1686.Trichophagiaischaracterizedby?
a)Compulsivepullingofhair
b)Compulsiveeatingofhair
c)Compulsiveshopping
d)Compulsivestealing
CorrectAnswer-B
Ans.B.Compulsiveeatingofhair
Trichophagiaiscompulsiveeatingofhairandisusuallyassociated
withtrichotillomaniai.e.compulsivepullingofhair.

1687.Allofthefollowingareessential
featuresofattentiondeficithyperactive
disease(ADHD)except-

a)Lackofconcentration
b)Impulsivity
c)Mentalretardation
d)Hyperactivity
CorrectAnswer-C
Ans.is'c'i.e.,MentalRetardation
Inattentive

Thismustincludeatleast6ofthefollowingsymptomsofinattention
thatmusthavepersistedforatleast6monthstoadegreethatis
maladaptiveandinconsistentwithdevelopmentallevel:
Oftenfailstogivecloseattentiontodetailsormakescareless
mistakesinschoolwork,work,orotheractivities
Oftenhasdifficultysustainingattentionintasksorplayactivities
Oftendoesnotseemtolistentowhatisbeingsaid
Oftendoesnotfollowthroughoninstructionsandfailstofinish
schoolwork,chores,ordutiesintheworkplace(notdueto
oppositionalbehaviororfailuretounderstandinstructions)
Oftenhasdifficultiesorganizingtasksandactivities
Oftenavoidsorstronglydislikestasks(suchasschoolworkor
homework)thatrequiresustainedmentaleffort
Oftenlosesthingsnecessaryfortasksoractivities(school
assignments,pencils,books,tools,ortoys)
Ofteniseasilydistractedbyextraneousstimuli

Oftenforgetfulindailyactivities
Hyperactivity/impulsivity
Thismustincludeatleast6ofthefollowingsymptomsof
hyperactivity-impulsivitythatmusthavepersistedforatleast6
monthstoadegreethatismaladaptiveandinconsistentwith
developmentallevel:
Fidgetingwithortappinghandsorfeet,squirminginseat
Leavingseatinclassroomorinothersituationsinwhichremaining
seatedisexpected
Runningaboutorclimbingexcessivelyinsituationswherethis
behaviorisinappropriate(inadolescentsoradults,thismaybe
limitedtosubjectivefeelingsofrestlessness)
Difficultyplayingorengaginginleisureactivitiesquietly
Unabletobeoruncomfortablebeingstillforextendedperiodsof
time(maybeexperiencedbyothersas"onthego"ordifficultto
keepupwith)
Excessivetalking
Blurtingoutanswerstoquestionsbeforethequestionshavebeen
completed
Difficultywaitinginlinesorawaitingturningamesorgroup
situations
Interruptingorintrudingonothers(foradolescentsandadults,may
intrudeintoortakeoverwhatothersaredoing)
Other
Onsetisnolaterthanage12years
Symptomsmustbepresentin2ormoresituations,suchasschool,
work,orhome
Thedisturbancecausesclinicallysignificantdistressorimpairment
insocial,academic,oroccupationalfunctioning
Disorderdoesnotoccurexclusivelyduringthecourseof
schizophreniaorotherpsychoticdisorderandisnotbetter
accountedforbymood,anxiety,dissociative,personalitydisorderor
substanceintoxicationorwithdrawal

1688.MCcauseofdelirium?
a)Infection
b)Liverfailure
c)Belladonnapoisoning
d)Noneoftheabove
CorrectAnswer-A
Ans.A.Infection
Infectionisoneofthemostcommoncausesofdelirium.

1689.Capgrassyndromeis?
a)Sharingofdelusion
b)Delusionofdouble
c)Erotomania
d)Hypochondriacaldelusions
CorrectAnswer-B
Ans.is'B'i.e.,Delusionofdouble
Capgrassyndromeisadelusionofdouble.
Thedelusionalmisidentificationsyndrome(DSM)
DSMischaracterizedbymisidentificationdelusionsofothersorself.
Fourmainsyndromesaredifferentiated:?
1. Capgrassyndrome(Delusionofdouble):-Patientfalselyseesa
familiarpersonasacompletestrangerwhoisimposingonhimasa
familiarperson.
2. Fregolisyndrome(illusiondefregola):-Thepatientfalselyidentifies
strangerasafamiliarperson.
3. Syndromeofsubjectivedouble:-Thepatient'sownselfisperceived
asbeingreplacedbyadouble.
4. Syndromeofintermetamorphosis:-Afalsebeliefthatapersoncan
transformintoanotherperson.
Thesesyndromesmostcommonlyappearinschizophrenia.Other
causesareAlzheimersyndrome,headinjuries,anddelusional
disorders.

1690.Jamaisvuis-
a)Illusionthatwhatoneishearing,onehasheardpreviously
b)Aunfamiliarthoughtregardedasrepetitionofaprevious
thought
c)Unfamiliarsituationsoreventsfeelstrangelyfamiliar
d)Feelingofstrangenesstofamiliarsituation
CorrectAnswer-D
Ans.D.Feelingofstrangenesstofamiliarsituation
famaisvu:-Afeelingofstrangenesstofamiliarsituationsorevents.

1691.Featureslikeincreasedpsychomotor
acitivity,waxyflexinbilityattimeare
seenclassicallyin:

a)Simpleschizophrenia
b)Hebephrenicschizophrenia
c)Catatonicschizophrenia
d)Noneoftheabove
CorrectAnswer-C
C.i.e.Catatonicschizophrenia

1692.Characteristichallucinationof
schizophreniais
a)Auditoryhallucinationscommandingthepatient
b)Auditoryhallucinationsgivingrunningcommentary
c)Auditoryhallucinationscriticizingthepatient
d)Auditoryhallucinationstalkingtopatient
CorrectAnswer-B
Ans.B.Auditoryhallucinationsgivingrunningcommentary
Third-personhallucinations,e.g.voicesheardarguing,commenting
ordiscussingthepatientorgivingarunningcommentaryonone's
action;arecharacteristicofschizophrenia.
Inschizophreniaauditoryhallucinationsarethemostcommon
typeofhallucinations.

1. First-personhallucination:-Audibleself-thoughts
2. Secondpersonhallucination:-Voicesaddressthepersondirectlyor
commandingone'sactionand
3. Third-personhallucinations:-voicesheardarguing,commentingor
discussingthepatientorgivingarunningcommentaryonhisaction
orthought.Onlythe"thirdpersonhallucinations'arecharacteristicof
schizophrenia.

1693.Goodprognosticfactorfor
schizophreniais?
a)Bluntedaffect
b)Earlyonset
c)Presenceofdepression
d)Malesex
CorrectAnswer-C
Ans.is'c'i.e.,Presenceofdepression
Goodprognosticfactors:-Acuteonset;lateonset(onsetafter35
yearsofage);Presenceofprecipitatingstressor;Goodpremorbid
adjustment;catatonic(bestprognosis)&Paranoid(2ndbest);short
duration(<6months);Married;Positivesymptoms;Presenceof
depression;
familyhistoryofmooddisorder;firstepisode;pyknic
(fat)physique;femalesex;goodtreatmentcompliance&good
responsetotreatment;goodsocialsupport;presenceofconfusion
orperplexity;normalbrainCTScan;outpatienttreatment.

1694.Selfmutilationisafeatureof?
a)Von-Goghsyndrome
b)Catatonicschizophrenia
c)Paranoidschizophrenia
d)Noneoftheabove
CorrectAnswer-A
Ans.A.Von-Goghsyndrome
Dramaticself-mutilationoccurringinschizophreniahasalsobeen
calledVon-Goghsyndrome'

1695.Apatientinventingnewwords,isa
featureof?
a)Neurosis
b)Schizophrenia
c)OCD
d)Von-Goghsyndrome
CorrectAnswer-B
AnsB.Schizophrenia
Neologism:-Newwordsorcondensationsofseveralwordsthatare
notreadilyunderstoodbyothers.
Neologismisseeninschizophreniaandorganicbrainsyndrome.

1696.Increaseddopaminelevelsare
associated?
a)Depression
b)Mania
c)Delirium
d)Schizophrenia
CorrectAnswer-D
Ans.D.Schizophrenia
Dopaminehypothesisisthemostacceptedhypothesisfor
schizophrenia.
Thereishyperactivityofdopaminergicsystem.
Otherneurotransmittersinvolvedare:-Increasedserotonin,
DecreasedGABA,variablechange(Increasedordecreased)
glutamate,andincreasednorepinephrine.

1697.Looseningofassociationisanexample
of
a)Formalthoughtdisorder
b)Schneider'sfirstsymptoms
c)Perseveration
d)Concretethinking
CorrectAnswer-A
Ans.is'a'i.e.,Formalthoughtdisorder
Thoughtdisorders
Formalthought
Disordersofthought
disorders(Disordersof process
thoughtprocess)
Racingthoughts:-
Anxiety,Schizophrenia
Retardedthoughts:-
Depression
Delusion:-Psychosis
Circumstantiality:-
(Schizophreniamania,
Mania,Schizophrenia
depression&others
Thought
Obsession
blocking:-
Compulsion
Schizophrenia,Severe
Preoccupations
anxiety
Phobias
Perseveration:-Organic Depersonalization&
braindisease,
Derealization
Schizophrenia
(occasionally
Looseningof
association:-

association:-
Schizophrenia
Flightofideas:-Mania
Tangentiality
Clunging&punning:-
Mania&schizophrenia
Neologism,wordsalad,
Echolalia:-
Schizophrenia
Inschizophreniaandmooddisorders(depression,mania)allpartsof
thethought(thoughtprocessaswellascontent)areinvolved.
However,schizophreniaisconventionallyreferredasformalthought
disorder.

1698.Visualhallucinationsisseenin:
a)Alcoholism
b)Mania
c)Depression
d)Phobia
CorrectAnswer-A
A.i.e.Alcololism

1699.
Treatmentofchoiceforacutepanic
attacksis?
a)Barbiturates
b)Benzodiazepines
c)TCAs
d)MAOinhibitors
CorrectAnswer-B
Ans.B.Benzodiazepines
DOCforPanicdisorders-SSRI's
DOCforacutepanicattack-Benzodiazepines.

1700.Whichofthefollowingisnottrueabout
sleep?
a)REMsleepcomesearlierthanNREMsleep
b)REMsleepisalsocalledparadoxicalsleep
c)SleepwalkingcomesinNREMsleep
d)DreamscomeinREMsleep
CorrectAnswer-A
Ans.A.REMsleepcomesearlierthanNREMsleep
TheNREM(withitsfourstages)andREMsleeprepeatseveral
timesanightincyclicmanner,4-6timesdependingonthelengthof
sleep.
AtypicalcyclestartswithstageIoftheNREMsleepwhichis
followedbysecond,thirdandfourthstages.
REMsleepoccupies20-30%oftotalsleepandNREMsleep
occupies60-70%(StageIoccupies5-10%;Stage2occupies40-
50%;Stage3occupies15-20%

1701.GammawavesofREMsleepinsleep
cycleareassociatedwith?
a)Intenseattention
b)Subconsciousthinking
c)Deepsubconsciousthinking
d)Deepsleep
CorrectAnswer-A
Ans.A.Intenseattention
Betaandgammawaves(20-80Hz)occurspontaneouslyduring
REMsleepandwakingandareevokedbyintenseattention,
conditionedresponses,tasksrequiringfinemovementsandsensory
stimulus

1702.Behavioraltherapyisdonein?
a)Schizophrenia
b)Agoraphobia
c)Delirium
d)Neuroticdepression
CorrectAnswer-B
Ans.B.Agoraphobia
Mostimportantuseofbehavioraltherapy-Phobia&OCD.
Otherusersare-Otheranxietydisorders(includingpanic),Eating
disorders,Autism,ADHD,somepersonalitydisorder,sexual
dysfunctions,depression.

1703.Notseeninanorexianervosa:
September2009
a)Osteoporosis
b)Refusaltofeeds
c)Weightloss
d)Menorrhagia
CorrectAnswer-D
Ans.D:Menorrhagia
PhysicalSignsofanorexianervosa
*Excessiveweightloss
*Scantyorabsentmenstrualperiods
*Thinninghair
-Dryskin
*Coldorswollenhandsandfeet
Bloatedorupsetstomach
*DownyhaircoveringthebodyLowbloodpressure
*Fatigue
*Abnormalheartrhythms
*Osteoporosis
PsychologicalandBehavioralSignsinanorexianervosa
*Distortedperceptionofself(insistingtheyareoverweightwhen
theyarethin)
*BeingpreoccupiedwithfoodRefusingtoeat
*Inabilitytorememberthings
*Refusingtoacknowledgetheseriousnessoftheillness
Obsessive-compulsivebehavior
*Depression

1704.Maindifferencebetweenanorexia
nervosaandbulimianervosalies
in:
March2013

a)Symptomatology
b)Weight
c)Gender
d)Age
CorrectAnswer-B
Ans.Bi.e.Weight
Anorexianervosaandbulimia
Bothanorexianervosaandbulimiaarecharacterizedbyan
overvalueddriveforthinnessandadisturbanceineatingbehavior.
Themaindifferencebetweendiagnosesisthatanorexianervosais
asyndromeofself-starvationinvolvingsignificantweightlossof15
percentormoreofidealbodyweight,whereaspatientswithbulimia
nervosaare,bydefinition,atnormalweightorabove.
Bulimiaischaracterizedbyacycleofdieting,binge-eatingand
compensatorypurgingbehaviortopreventweightgain.
Purgingbehaviorincludesvomiting,diureticorlaxativeabuse.
Excessiveexerciseaimedatweightlossoratpreventingweightgain
iscommoninbothanorexianervosaandinbulimia.

1705.Eatingdisorderwithnormalweightis?
a)Anorexianervosa
b)Bulimianervosa
c)Bingeeatingdisorder
d)Noneoftheabove
CorrectAnswer-B
Ans.B.Bulimianervosa
Anorexianervosa-Underweight
Bulimianervosa-NormalWeight
Bingeeatingdisorder-Overweight.

1706.Bingeeatingdisorderischaracterized
by?
a)Normalweight
b)Weightloss
c)Obesity
d)Selfinducedvomiting
CorrectAnswer-C
Ans.C.Obesity
Bingeeatingdisorder(BED)ischaracterizedbyinsatiablecravings
thatcanoccuranytimedayornight,usuallysecretive,andfilledwith
shame.
TherearenocomPensatorymechanismsassociatedwiththebinge
togetridofcalories,soindividulaswithBEDaremorelikelytobe
overweightorobese,whilepatientswithbulimianervosamaybe
underweight,normalweight.

1707.Tricyclicantidepressantshaveallofthe
followingactionsexcept?
a)Anticholinergicaction
b)MAOinhibition
c)Block5-HTorNEreuptake
d)Causessedation
CorrectAnswer-B
Ans-B.MAOinhibition
Tricyclicantidepressants(TCAS)areeitherNA+5HTreuptake
inhibitors(e.g.-Imipramine,
Amitriptyline)orpredominantlyNAreuptakeinhibitors(e.g.
Desipramine,nortriptyline).
TCAsarenotMAOinhibitors.
AdverseeffectsofTCFsare:anticholinergicsideeffects,sedatior,
mentalconfusion,weakness,increaseappetite&weightgain,
sweating&finetremor,posturalhypotensionandcardiac
arrhythmia.

1708.Whichofthefollowingisafeatureof
opioidwithdrawal?
a)Tremors
b)Gooseflesh
c)Drynoseandmouth
d)Constipation
CorrectAnswer-B
Ans.B.Gooseflesh
Manifestationsofmorphinewithdrawal-
Lacrimation
Sweating
Yawning
Gooseflash(Piloerection)
Mydriasis
Anxiety&fear
Restlessness
Insomnia
Abdominalcolic
Diarrhea
Dehydration
Hypertension
Palpitation
Rapidweightloss

1709.Phototherapyisusedinthetreatment
of?
a)Anorexianervosa
b)Seasonalaffectivedisorder
c)Schizophrenia
d)Obsessivecompulsivedisorder
CorrectAnswer-B
Ans.B.Seasonalaffectivedisorder
Phototherapyisprimarilyindicatedinthetreatmentofseasonal
depressions.
Inadditiontoseasonaldepression,theothermajorindicationfor
phototherapymaybeinsleepdisorders.

1710.Featureassociatedwithmaniais?
a)Neologism
b)Perseveration
c)Echolalia
d)Flightsofideas
CorrectAnswer-D
Ans.D.Flightsofideas
Diagnosticcriteriaformania
Threeormoreofthefollowingforatleast1week:-
1. Inflatedselfvesteemorgrandiosity
2. Decreasedneedforsleep
3. Overtalkativeness
4. Flightofvideas
5. Distractibility
6. PsychomotoragitationorIncreasedgoaldirectedactivities
7. Excessiveinvolvementinpleasurableactivities

1711.DrugnotusedinprophylaxisofMDP?
a)Haloperidol
b)Lithium
c)Carbamazepine
d)Valproate
CorrectAnswer-A
Ans.A.Haloperidol
Prophylactictreatmentforbipolardisorder:

1. Lithium(drugofchoice)
2. Carbamazepine
3. Valproate
4. Otherdrugswhichcanbeusedaretopiramate,Lamotrigine,atypical
antipsychotics(artpiprazale,olanzapine,quetiapine,risperidone,
Clozapine)andGabapentin.

1712.Characterizedbychronic,multipletics
?
a)Parkinson'sdisease
b)Wilson'sdisease
c)Shy-Dragersyndrome
d)Tourette'ssyndrome
CorrectAnswer-D
Ans.D.Tourette'ssyndrome
Tourettesyndrome(alsocalledTourette'ssyndrome,Tourette's
disorder,GillesdelaTourettesyndrome,GTSor,morecommonly,
simplyTourette'sorTS)isaninheritedneuropsychiatricdisorder
withonsetinchildhood,characterizedbymultiplephysical(motor)
ticsandatleastonevocal(phonic)tic.

1713.Tourettesyndromeisatypeof?
a)Ticdisorder
b)Mentalretardationdisorder
c)Seizuredisorder
d)Noneoftheabove
CorrectAnswer-A
Ans.A.Ticdisorder
Tourettesyndrome(alsocalledTourette'ssyndrome,Tourette's
disorder,GillesdelaTourettesyndrome,GTSor,morecommonly,
simplyTourette'sorTS)isaninheritedneuropsychiatricdisorder
withonsetinchildhood,characterizedbymultiplephysical(motor)
ticsandatleastonevocal(phonic)tic.
Averageonsetbetweentheagesof3and9years.
Malesareaffectedaboutthreetofourtimesmoreoftenthan
females.
CommonassociationsareADHDandOCD.

1714.Trueaboutflumazenilis?
a)Canbeusedinbarbituratepoisoning
b)Specificantidoteforopiateoverdose
c)Canbeusedinbenzodiazepineoverdose
d)Alloftheabove
CorrectAnswer-C
Ans.C.Canbeusedinbenzodiazepineoverdose
Fumazenilisanimidazobenzodiazepinederivativeandapotent
benzodiazepinerecePtorantagonistthatcompetitivelyinhibitsthe
activityatthebenzodiazepinerecognitionsiteontheGABA
benzodiazepinereceptorcomplextherebyreversingtheeffectsof
benzodiazepineonthecentralnervoussystem.
Flumazenildoesnotantagonizethecentralnervoussystemeffects
ofdrugsaffectingGABA-ergicneuronsbymeansotherthanthe
benzodiazepinereceptor(includingethanol,barbiturates,orgeneral
anesthetics)anddoesnotreversetheeffectsofopioids.

1715.Whichofthefollowingtypical
antipsychoticdrugisnotavailablein
depotform?

a)Haloperidol
b)Risperidone
c)Olanzapine
d)Chlorpromazine
CorrectAnswer-D
Ans.D.Chlorpromazine
Antipsychoticdrugswithdepotpreparationsarerisperidone,
poliperidone,halopetidol,fluphenazine,flupenthixol,zuclopenthixol,
olannpine,clozapine,imipramineandquetiapine.

1716.Conventionaldrugusedinthe
treatmentofdeliriumis?
a)Haloperidol
b)Lithium
c)SSRIs
d)Morphine
CorrectAnswer-A
Ans.A.Haloperidol
Drugsusedfordelirium:
Typicalantipsychotics:-Haloperidol(Doc),Thioridazine,
chlorpromazine.
Atypicalantipsychotics:-Risperidone,quetiapine,olanzapine.
Benzndiazepines(fordeliriumtremens):-Chlordiazepoxide,
Diazepam,Lorazepam,Clonazepam.

1717.Generalparalysisofinsaneis
associatedwith?
a)Neurosyphillis
b)Alzheimer'sdisease
c)Parkinson'sdisease
d)Noneoftheabove
CorrectAnswer-A
Ans,A.Neurosyphillis
Generalparesis,alsoknownasgeneralparalysisoftheinsaneor
paralyticdementia,isasevereneuropsychiatricdisorder,classified
asanorganicmentaldisorderandcausedbythechronic
meningoencephalitisthatleadstocerebralatrophyinlate-
stagesyphilis.
Degenerativechangesareassociatedprimarilywiththefrontaland
temporallobarcortex-
GeneralparalysisoftheInsane(GPI)isprogressivedeteriorationof
thewholementalandphysicalpersonality.
Symptomsincludedexaggeratedkneejerk,Iackofreactionofthe
pupilstolight,aninabilitytopayattention,a'clouding'of
consciousness,poorshort-termmemory,tremulousvoice,reflex
disturbances,andretinalanomalies'anddiminishedskinsensation.
Patientswereoftenrecognizablebytheirstriking'delusionsof
grandeur"

1718.Signofoppositionalisminayoung
childisdueto?
a)Mentalretardation
b)Organicmentaldisorder
c)Mentaldistress
d)Alloftheabove
CorrectAnswer-C
Ans.C.Mentaldistress
"Oppositionalism,tempertantrumsandbreathholdingspellsarenot
unusualduring1styearsoflifeandareagetypicalexpressionof
frustrationandanger"

1719.Allofthefollowingareassociated
increasedREMlatency,except?
a)Firstnighteffect
b)SSRIs
c)Narcolepsy
d)Restlesslegsyndrome
CorrectAnswer-C
Ans.C.Narcolepsy
IncreasedREMlatency-
PTSD
Restlesslegsyndrome.
Firstnighteffect
SSRI's
TCA's
Ethanol
Lithium

1720.Modafinilisusedforthetreatmentof?
a)Narcolepsy
b)Sexualdysfunction
c)Depression
d)Anxiety
CorrectAnswer-A
Ans.A.Narcolepsy
Modafinilisawakefulness-promotingagent(oreugeroic)usedfor
treatmentofdisorderssuchasnarcolepsy,shiftworksleepdisorder,
andexcessivedaytimesleepinessassociatedwithobstructivesleep
apnea'

1721.Drugwithnomoodstabilizingproperty
is-
a)Lithium
b)Lamotrigine
c)Imipramine
d)Carbamazepine
CorrectAnswer-C
Ans.C.Imipramine
The5individualdrugsthatcanbeusedasmoodstabilizers
are:

1. Lithium
2. CarbamazePine
3. Lamotrigine
4. Valproate
5. Asenapine

1722.Haloperidolinducedextarpyramidal
sideeffectsaretreatedby?
a)Benzodiazepines
b)Barbiturates
c)Anticholinergicdrugs
d)SSRIs
CorrectAnswer-C
Ans.C.Anticholinergicdrugs
Commonlyusedmedicationsforextra-pyramidalsymptomsare
anticholinergicagentssuchasBenztropine,Diphenhydramineand
trihexyphenidyl.
Anothercommoncourseoftreatmentincludesdopamineagonist
agentssuchaspramipexole.

1723.Mostcommoncomplicationofmodified
ECT
a)IntracerebellarBleed
b)Fracturespine
c)Bodyache
d)Amnesia
CorrectAnswer-D
Di.e.Amnesia
*Electroconvulsivetherapy(seizure)increasesproductionofbrain
derivedneurotrophicfactor(BDNF)Q.
Madsenproposedgeneration
ofnewneuronsinthehippocampus
maybeanimportant
neurobiologicalelementunderlyingtheclinicaleffectsofECT.
*ECTismostcommonlyindicated(-85%ofallECT)andmost
effectiveintreatmentofmajordepression(withpsychosis/
delusions/orsuicidaltendency)Q.ECTisindicatedinpsychotic
(delusional)depressionQ
becausenihilisticdelusionsmayinduce
suicidaltendency,whereasECTisnotindicatedinneurotic
depression(dysthynia),cyclothymiaQ.
ECTmaybeusefulinMDP
(bipolar)indecreasingdurationofdepressiveepisode.
ECTisnot(orless)usefulinchronicconditionslikechronic
schizophrenia(withnegativsymptomsesp)Q.
UseofECTisnotthe
firstline(choice)oftreatmentinmania&schizophrenia(lithium&
antipsychoticsaremainstayoftreatment).Panicdisorderisalso
treatedwithdrugsnotECT.
IndicationsofECT
*ECTismostusefulinacuteandpositivesymptoms(egsuicidal
tendency,catatonia),whereasitisleasteffectiveinchronic
conditionsandnegativesymptoms

*ClinicalindicationsofECTinclude
Primaryuse
Secondaryuse
1.Rapiddefinitive
1.Failureto
response
respondor
requiredonmedical intoleranceto
or
pharmacotherapyin
psychiatricgrounds current
2.Riskof
episodei.e.drugs
alternative
are
treatmentsout
ineffective,
weightbenefits
contraindicatedor
3.Pasthistoryof
haveseriousside
poorresponseto
effects
psychotropicsor 2.Rapiddefinitive
goodresponseto response
necessitatedby
ECT
deteriorationof
4.Patient
thepatients
preferance
condition.
*ECTshouldbeconsideredwhentheonsetofdisorderisacute,
whenchangesinmood,thought,andmotoractivitiesare
pronounced,
whenthecauseofdisorderisbelievedtobe
biochemicalorphysiological,whentheconditioninterfereswiththe
dailylifeorwhenothertreatmentshavefailed.Diagnosisforwhich
ECTmaybeindicatedinclude(especiallywhena/wdelusions).
Major(Severe)ManiaSchizophreniaOther
Depressionwith(Bipolar(Severeconditions
1.Psychosis(i.e.disorder)Psychosis)1.Deliriumd/t
delusionalor1.Psychosis1.Catatoniageneral
medical
psychotic2.ManicparticularlyQconditionsor
depression)Q,whodelirium2.Schizoaffectivesubstance
areguiltriddenor3.Rapidcyclingdisorder(i.e.intoxication

feelworthless,whostatesdepressive2.Catatoniad/t
believeothers4.Acutefeatures)QandGMC,neurolept
controltheirlives&uncontrolledschizophreniform
malignantContraindications
I.Absolute(relatively)RaisedIntraCranialTensionQd/tfearofbrain
herniation.
1. Relative
*CerebrovascularAccident(CVA)=intracerebralhemorrhage
*RecentMyocardialinfarction(MI)
*Severehypertension
*Pheochromocytoma-SeverePulmonarydisease
*Retinaldetachment-Spaceoccupyingintracerebrallesions
(exceptforsmall,slowgrowingtumorswithoutedemaorothermass
effect)
*UnstablevascularaneurysmsormalformationsComplications
Direct(Modified)ECT(whenECTisgivenwithoutmusclerelaxant&
anesthesia)
MostcommonsideeffectisfractureT4T8spineQ
Causesdecreasedintraoculartension(JOT)
.
ModifiedECT(withMR&anesthesia)Bothretrograde&antegrade
amnesiaisfoundbutthemostcommoncomplicationisretrograde
amnesiaQ
Antegradeamnesiausuallyresolveswithin5hours
whereasretrogradememorydeficitsmaytake6-9monthsQ

1724.Causesofneuroregressioninachild
canbeallexcept?
a)Wilson'sdisease
b)VitaminB12deficiency
c)ADHD
d)Ataxiatelengiectasia
CorrectAnswer-C
Ans.C.ADHD
Causesofneuroregression
Inherited
A.GreYmatterinvolvement-
Withvisceromegaly:-Niemannpickdisease,Sandholfdisease
(GM2),sialidosis,Goucherdisease,Mucopolyschharidoses(MPS)
Withoutvisceromegaly:-TaySachDisease,Rettsyndrome,Menke's
kinkyhairdisease
B.Whitematterinvolvement:-
Leukodystrophies:-Metachromaticleukodystrophy,Krebbe's
disease,Adrenoleukodytrophy,Alexanderdisease,Cannavan
disease.
Acquired/Demyelinating:-
Multiplesclerosis,schilder'sdisease,Devicdisease
C.Basalgangliainvolvement-Wilson'sdisease,Dystoniamuscular
deformans,Huntington'sdisease
D.Spinocerebeller:-Friedrich'sataxia,Ataxiatelengiectasis
E.Acquired
Infections:-SSPE,progressiverubellasyndrome,HIV
Metabolic:Chronicleadpoisoning,Hypothyoidism,VitB12&Edeficienry,Drugs(anticonvulsants).

1725.Mechanismofactionofduloxetineis?
a)SelectiveInhibitionofserotoninreuptake
b)Selectiveinhibitionofnor-epinephrinereuptake
c)Selectiveinhibitionofbothserotoninandnor-epinephrine
reuptake
d)Noneoftheabove
CorrectAnswer-C
Ans.C.Selectiveinhibitionofbothserotoninandnor-
epinephrinereuptake
Duloxetine,isadrugclassifiedunderserotonin-norepinephrine
reuptakeinhibitors(sNRJs).
Atlowerdosesitismoreprominentserotoninreuptakeinhibitor.
Whereasathigherdosesitismoreprominentinhibitorofnor-
epinephrinereuptake.
Ithasminimaldopamineagonistaction.


1726.Whichofthefollowingdescribesthe
pointsmarkedinthediagram?
a)Obliqueconjugate
b)Diagonalconjugate
c)Obstetricconjugate
d)Trueconjugate
CorrectAnswer-B
Answer-B.Diagonalconjugate
Diagonalconjugate:Distancebetweenthelowerborderofpubic
symphysistothemidpointonthesacralpromontory.Itmeasures12
cm.
invalidquestionid

This post was last modified on 30 July 2021